You are on page 1of 586

Solutions Manual

This booklet is provided in Glencoe Geometry Answer Key Maker (0-07-860264-5).


Also provided are solutions for problems in the Prerequisite Skills, Extra Practice,
and Mixed Problem Solving sections.

Copyright © by The McGraw-Hill Companies, Inc. All rights reserved.


Printed in the United States of America. Permission is granted to reproduce the
material contained herein on the condition that such material be reproduced only
for classroom use; be provided to students, teacher, and families without charge;
and be used solely in conjunction with Glencoe Geometry. Any other reproduction,
for use or sale, is prohibited without prior written permission of the publisher.

Send all inquiries to:


Glencoe/McGraw-Hill
8787 Orion Place
Columbus, OH 43240-4027

ISBN: 0-07-860204-1 Geometry


Solutions Manual

1 2 3 4 5 6 7 8 9 10 009 12 11 10 09 08 07 06 05 04 03
CONTENTS
Chapter 1 Points, Lines, Planes, and Angles. . . . . . . . . . . . . . . . . . . . . . . . . . . . . . 1
Chapter 2 Reasoning and Proof . . . . . . . . . . . . . . . . . . . . . . . . . . . . . . . . . . . . . . . .. 24
Chapter 3 Parallel and Perpendicular Lines. . . . . . . . . . . . . . . . . . . . . . . . . . . . . . 54
Chapter 4 Congruent Triangles . . . . . . . . . . . . . . . . . . . . . . . . . . . . . . . . . . . . . . . . . 85
Chapter 5 Relationships in Triangles . . . . . . . . . . . . . . . . . . . . . . . . . . . . . . . . . . .120
Chapter 6 Proportions and Similarity . . . . . . . . . . . . . . . . . . . . . . . . . . . . . . . . . . .151
Chapter 7 Right Triangles and Trigonometry. . . . . . . . . . . . . . . . . . . . . . . . . . .. 189
Chapter 8 Quadrilaterals. . . . . . . . . . . . . . . . . . . . . . . . . . . . . . . . . . . . . . . . . . . . . . . 238
Chapter 9 Transformations. . . . . . . . . . . . . . . . . . . . . . . . . . . . . . . . . . . . . . . . . . . . . 289
Chapter 10 Circles . . . . . . . . . . . . . . . . . . . . . . . . . . . . . . . . . . . . . . . . . . . . . . . . . . . . . 345
Chapter 11 Areas of Polygons and Circles . . . . . . . . . . . . . . . . . . . . . . . . . . . . . . 383
Chapter 12 Surface Area . . . . . . . . . . . . . . . . . . . . . . . . . . . . . . . . . . . . . . . . . . . . . . .. 411
Chapter 13 Volume………………………………………………………...447
Prerequisite Skills …………………………………………………………477
Extra Practice………………………………………………………………502
Mixed Problem Solving and Proof…………………………………………567
Chapter 1 Points, Lines, Planes, and Angles
Page 5 Getting Started 2. See students’ work; sample answer: Two lines
1–4. intersect at a point.
y
3. Micha; the points must be noncollinear to
D( 1, 2) determine a plane.
4. Sample answers: line p; plane R
B(4, 0) 5. Sample answer: y
O x

A(3, 2)
C( 4, 4) W
x
O
5. 3
 3 6 3 X
4  8  8  8 Z
 9 1
8  18

5
Y
1  37 4
1
16  5 8 ¬ 16  8
6. 2   
37 82 6.
¬16  16
   r
P
119 7
¬16 ¬7 16 s
 9 
7. 7  9
14 Q
8  16 ¬ 16  16
5 7. There are six planes: plane ABC, plane AGE,
¬16
plane CDE, plane BCD, plane FAB, and plane
8. 11 1 7  23 15
2  9 16 ¬ 2  16
   1
DEF.
184 15 1
¬16  16
 8. A, K, B or B, J, C
33 9. No; A, C, and J lie in plane ABC, but D does not.
¬ 1
16  2 16

10. line 11. point 12. plane
9. 2  17  15
10. 23  (14)¬ 23  14
 37 Pages 9–11 Practice and Apply
11. [7  (2)]2¬ (7  2)2 13. n 14. F
 (5)2  25 15. R 16. W
12. 92  132¬ 81  169 
17. Sample answer: PR
 250
13. P¬ 4s 18. Yes, it intersects both m and n when all three
 4(5)  20 lines are extended.
The perimeter is 20 in. 19. (D, 9) 20. Charlotte
14. P¬ 2  2w 21.
B
 
¬ 2(6)  2 21

2
¬ 12  5  17 W
The perimeter is 17 ft. Q
15. P¬ 2  2w A
 2(4.8)  2(7.5)
 9.6  15  24.6 22.
The perimeter is 24.6 m. R
W
T
23. Sample answer:
1-1 Points, Lines, and Planes
y

Page 8 Geometry Activity R S


1. no 2. no Z

3. On CD; see students’ work. O x
4. See students’ work.
Q

Page 9 Check for Understanding


1. point, line, plane

1 Chapter 1
24. Sample answer: 36. Sample answer: points E, A, and B are coplanar,
y but points E, A, B, and C are not.
37. 
AC 38. point
C R 39. lines 40. plane
41. plane
P
D 42. two planes intersecting in a line
43. point 44. intersecting lines
S O x 45. point 46. line
47.

25.
a
b
c

48.
26. a
F b
V
c
27. s
49. See students’ work.
C r 50. Sample answer: the image is rotated so that the
D front or back plane is not angled.
M 51. Sample answer:

28. s
A

B C

52. See picture. 53. vertical


54. Sample answer: the paths flown by airplanes
flying in formation
29. points that seem collinear; sample answer:
55. Sample answer: Chairs wobble because all four
(0, 2), (1, 3), (2, 4), (3, 5)
legs do not touch the floor at the same time.
y Answers should include the following.
• The ends of the legs represent points. If all points
lie in the same plane, the chair will not wobble.
O x
• Because it only takes three points to determine a
plane, a chair with three legs will never wobble.
56. C; Three lines intersect in a maximum of three
points. The fourth line can cross the other three
lines only one time each, adding three points to
the figure. For example,
30. There are five planes: plane ABC, plane BCE,
plane ABE, plane ADE, and plane CDE.
31. 1; There is exactly one plane through any three
noncollinear points.
32. E, F, C
33. Because A and B determine a line, add point G
anywhere on AB.
34. E, F 35. A, B, C, D or E, F, C, B

Chapter 1 2
57. B; 2  x  2  x
2x22x2 1-2 Linear Measure and Precision
x  x
x  x  x  x
2x  0 Page 16 Check for Understanding
Thus, x must be 0. 1. Align the 0 point on the ruler with the leftmost
58. y endpoint of the segment. Align the edge of the
ruler along the segment. Note where the
rightmost endpoint falls on the scale and read
the closest eighth of an inch measurement.
2. Sample answers: rectangle, square, equilateral
triangle
3. Each inch on the ruler is divided into eighths.
O x Point Q is closer to the 16 
8 -inch mark. Thus, PQ is
about 16
 or 13 inches long.
8 4
a line 4. The long marks on the ruler are centimeters, and
59. y the shorter marks are millimeters. There are
10 millimeters for each centimeter. Thus, the bee
is 13 millimeters or 1.3 centimeters long.
O x 5. The measurement is precise to within 0.5 meter.
So, a measurement of 14 meters could be 13.5 to
14.5 meters.
1
6. The measuring tool is divided into 4-inch
increments. Thus, the measurement is precise
to within 1
 1   1
2 4 or 8 inch. Therefore, the
part of the coordinate plane above the line measurement could be between 31 1
4  8  38
 1
y  2x  1 inches and 31 1 3
4  8  3 8 inches.

60. 1
 4 1 3
2  8 , so 2 in.  8 in. 7. EG  EF  FG
 4
61. 1 4 1 EG  2.4  1.3
4  16 , so 16 in.  4 in. EG  3.7
 8
62. 4 4 6
5  10 , so 5 in.  10 in. G
So, E is 3.7 centimeters long.
63. 10 mm  1 cm 8. XY  YZ¬ XZ
64. 2.5 cm  25 mm, so 2.5 cm  28 mm XY  15 
8 ¬ 3
65. 0.025 cm  0.25 mm, so 0.025 cm  25 mm XY  15 5 5
8  1 8 ¬ 3  1 8

3
XY¬ 18
3
Y
So, X is 18 inches long.
Page 12 Reading Mathematics
9.
1. Points P, Q, and R lie on . Point T is not collinear 5x 3x
with P, Q, and R. 15
2. Planes F, G, and H intersect at line j. N L M
3. The intersection of planes W, X, Y, and Z is NL  5x¬ 15
point P. 5x 15
  
4. 5 ¬ 5
A x¬ 3
B LM  3x
C LM  3(3)
LM  9
10. 30

6x  5 2x  3

N L M
NM¬ NL  LM
30¬ 6x  5  2x  3
30¬ 8x  2
30  2¬ 8x  2  2
32¬ 8x
3
 2 8
x
8 ¬ 8

3 Chapter 1
4¬ x XZ  5

4
LM  2x  3
So, X is 5
Z  1
4 or 1 4 inches long.
LM  2(4)  3  8  3
LM  11 24. PR¬ PQ  QR
C
11. BC D because they both have length
 5
21
4 ¬ 16  QR
10 inches. 9  5
16 ¬ 
5
16  QR 
5
4 16
E
BE D because they both have length 8 inches. 36 5
A
BD A because they both have length 16  16 ¬ QR


14.4 inches. 31
 
16 ¬ QR
31 15
So, QR is    
16 or 1 16 inches long.
Pages 17–19 Practice and Apply 25. RT¬ RS  ST
12. Each inch on the ruler is divided into sixteenths. 4.0¬ 1.2  ST
Point B is closer to the 1 5
B
-inch mark. Thus, A 4.0  1.2¬ 1.2  ST  1.2
16 2.8¬ ST
5
is about 1  inches long.
16 T
So, S is 2.8 centimeters long.
13. The long marks on the ruler are centimeters, and
26. WY  WX  XY
the shorter marks are millimeters. Point D is
4.8  WX  WX X
W  is congruent to X
Y
.
closer to the 45-millimeter mark. Thus, C D
 is
4.8  2WX
about 45 millimeters or 4.5 centimeters long.
4.8 2WX
2  2
14. The long marks on the ruler are centimeters, and   
the shorter marks are millimeters. The right end 2.4  WX
of the key is closer to the 33-millimeter mark. X
So, W  is 2.4 centimeters long.
Thus, the key is about 33 millimeters or 27. AD¬ AB  BC  CD
3.3 centimeters long.
33
4 ¬ BC  BC  BC A
 B is congruent to B
C

15. Each inch on the ruler is divided into sixteenths.
D
and C is congruent to BC.
The right tip of the paperclip is closer to the 33

4 4 4 ¬ 3BC
1 -inch mark. Thus, the paperclip is about 1
16
1
or 14 inches long.
16
 
1 
1
5 1
3 4  3 (3BC)
16. The measurement is precise to within 1 
2 inch. So,
15

12 ¬ BC
a measurement of 80 inches could be 79 1  1
2 to 80 2 C is 1 3 1
So, B 12 or 14 inches long.
inches.
28. 7a 12a
17. The measurement is precise to within 0.5
millimeter. So, a measurement of 22 millimeters 28
could be 21.5 to 22.5 millimeters. R S T
18. The measuring tool is divided into 1  RS  7a¬ 28
2 -inch
7a 28
increments. Thus, the measurement is precise to ¬ 
7 7
within 1  
 1 1
2 2 or 4 inch. Therefore, the a¬ 4
measurement could be between 161 1 1
2  4  16 4
 ST  12a
1 1 3
inches and 162  4  164 inches. ST  12(4)
19. The measurement is precise to within 0.5 ST  48
centimeter. So, a measurement of 308 centimeters 29.
could be between 307.5 and 308.5 centimeters. 34

20. The measurement is precise to within 0.005 meter 12 2x


or 5 millimeters. So, a measurement of 3.75
meters  3750 millimeters could be between 3745
and 3755 millimeters. R S T
RT  RS  ST
21. The measuring tool is divided into 1

4 -foot 34  12  2x
increments. Thus, the measurement is precise to 34  12  12  2x  12
within 1  
 1 1
2 4 or 8 foot. Therefore, the 22  2x
measurement could be between 22 2
x
2  2
 
31
 1 1 1 1 3
4  8  3 8 feet and 3 4  8  3 8 feet. 11  x
22. AC  AB  BC ST  2x
AC  16.7  12.8  29.5 ST  2(11)
C
So, A is 29.5 millimeters long. ST  22
23. XZ  XY  YZ
XZ  1
 3 2 3
2  4  4  4

Chapter 1 4
30. 25 34. yes; AB  CD  3 cm
35. no; EF  6 ft and FG  8 ft
2x 3x 36. no; NP  1.75 in. and LM  0.75 in.
37. yes; WX  XY  6 m
R S T 38. not from the information given
RT¬ RS  ST 39. yes; TR¬ 3(a  b)  3c  3a  3b  3c
25¬ 2x  3x SU¬ 3a  3(b  c)  3a  3b  3c
25¬ 5x 40. The width of a music CD is 12 centimeters.
5
2  5x
5 ¬ 5 41. F
C D G, AB
H I, C
E
E D
E FE G
5¬ x 42. 144 cm3; 343 mL could be actually as much as
ST¬ 3x 343.5 mL and 200 mL as little as 199.5 mL;
ST¬ 3(5) 343.5  199.5  144.
ST¬ 15
43. The lengths of the bars are given in tenths of
31. 5x  10 millions, and 0.1 million  100,000. So the graph
is precise to within 50,000 visitors.
16 2x
44. 50,000  0.05 million, so a measurement of 98.5
million could be 98.45 million to 98.55 million
R S T visitors.
RT¬ RS  ST 45. No; the number of visitors to Washington state
5x  10¬ 16  2x parks could be as low as 46.35 million or as high
5x  10  10¬ 16  2x  10 as 46.45 million. The visitors to Illinois state
5x¬ 6  2x parks could be as low as 44.45 million or as high
5x  2x¬ 6  2x  2x as 44.55 million visitors. The difference in visitors
3x¬ 6 could be as high as 2.0 million.
3x 6
3  3
 46. 12.5 cm; Each measurement is accurate within
x¬ 2 0.5 cm, so the least perimeter is 2.5 cm  4.5 cm
ST¬ 2x  5.5 cm.
ST¬ 2(2) 47. 15.5 cm; Each measurement is accurate within
ST¬ 4 0.5 cm, so the greatest perimeter is 3.5 cm 
32. 5.5 cm  6.5 cm.
21
48.
E
3y  1 2y F
EF 4(CD)

R S T
49. 2(CD)
RT¬ RS  ST
21¬ 3y  1  2y E
F
21¬ 5y  1
21  1¬ 5y  1  1 3(AB)
20¬ 5y 50a. 2 50b. 5 50c. 7
0
2  5y
5 ¬ 5 51. Sample answer: Units of measure are used to
4¬ y differentiate between size and distance, as well as
ST¬ 2y for accuracy. Answers should include the
ST¬ 2(4) following.
ST¬ 8 • When a measurement is stated, you do not
33. 5y
know the precision of the instrument used to
make the measure. Therefore, the actual
4y  1 2y  1 measure could be greater or less than that
stated.
• You can assume equal measures when
R S T segments are shown to be congruent.
RT¬ RS  ST
allowab
le error 0.5
ft
5y¬ 4y  1  2y  1 52.  measure 
27 ft  0.019 or 1.9%
5y¬ 6y  2 allowab
le error 0.25in.
5y  2¬ 6y  2  2 53.  measure 
14.5 in.  0.017 or 1.7%
5y  2¬ 6y allowab
le error 0.05cm
5y  2  5y¬ 6y  5y 54.  measure 
42.3 cm  0.001 or 0.1%
2¬ y allowab
le error 0.05km
55.  63.7 km  0.0008 or 0.08%
ST¬ 2y  1 measure
ST¬ 2(2)  1 56. B; 5(12 in.)  60 in. or 5 ft
ST¬ 3

5 Chapter 1
57. D; forty percent are jazz tapes so sixty percent are
blues tapes; 0.60(80)  48. 1-3 Distance and Midpoints

Page 19 Maintain Your Skills Page 22 Geometry Activity: Midpoint of a


58. B, G, E Segment
59. Sample answer: planes ABC and BCD 1. y
B C A
60. C
61. There are five planes shown: plane ABC, plane
BCDE, plane DEF, plane ACDF, and plane ABEF.
62. 2a  2b¬ 2(3)  2(8) O x
¬ 6  16  22
63. ac  bc¬ (3)(2)  (8)(2)
 6  16  22
a c 3 2 1
64.  2  
2  2
(2, 5)
65. (c
)  a¬
2 (2
)  32
 (1)
2 2. d ¬(x
 2 x
1)  (
2 y2 y1)2
 1
1 AC ¬(2  5)2  (5
 5)2
AC ¬(3)
 2 02
Page 19 Practice Quiz 1 AC ¬9

1. PR 2. T 
3. PR
AC ¬3
4. 6 4.5 C
A ¬C
B, so both are 3 units long.
3. y Y
S R T Z
TS  RS  TR
TS  6  4.5
TS  10.5 X
5. 11.75
O x
3.4

S R T (1, 5)
TS¬ RS  TR 4. d ¬(x 2 x 
1)  (y
2
2  y
1)
2

11.75¬ RS  3.4 XZ ¬ [1  (4)]


2  (5 3)2
11.75  3.4¬ RS  3.4  3.4
XZ ¬3 2  2
2
8.35¬ RS
XZ ¬ 13
Z
X  ¬Z
Y, so both are  13 units or about 3.6 units
Page 20 Geometry Activity: Probability and long.
Segment Measure 5. Sample answer: The x-coordinate of the midpoint
1. WZ  WX  XY  YZ is one half the sum of the x-coordinates of the
WZ  2  1  3  6 endpoints. The y-coordinate of the midpoint is one
XY half the sum of the y-coordinates of the endpoints.
P(J lies in XY)  
WZ
 1

6
2. WZ  WX  XY  YZ Page 25 Check for Understanding
WZ  2  1  3  6 1. Sample answers: (1) Use one of the Midpoint
YZ Formulas if you know the coordinates of the
P(R lies in YZ)  WZ endpoints. (2) Draw a segment and fold the paper
 3
  1
6 2 so that the endpoints match to locate the middle of
3. WY  WX  XY the segment. (3) Use a compass and straightedge
WY  2  1 to construct the bisector of the segment.
WY  3
XY
P(S lies in XY)  
WY
 1
3

Y
4. 1; X  contains all points that lie on both WY
and X Z
.
5. 0; if point U lies on WX
, it cannot lie on Y
Z
.

Chapter 1 6
2. Sample answer: 4  2x 6  2x
12. B; M(7, 8)  M ,   2 2 
P 4  2x 6  2x
7   8  
2 2
14  4  2x 16  6  2x
7 mm 7 mm 10  2x 10  2x
A M B 5x 5x

Q
Pages 25–27 Practice and Apply
3. AB  |2  10| 13. DE ¬|2  4| 14. CF ¬|0  7|
 |8| or 8 ¬|2| or 2 ¬|7| or 7
4. CD  |3  4| 15. AB ¬|4  (1)| 16. AC ¬|4  0|
 |7| or 7 ¬|3| or 3 ¬|4| or 4
5. 17. AF ¬|4  7| 18. BE ¬|1  4|
y
¬|11| or 11 ¬|5| or 5
12 X
19. y
8
8 B

Y 4
W
x
x
8 4 A 4 X
8 4 4 8
4

(XY)2  (XW)2 + (YW)2 8


(XY)2  (6)2 + (8)2
(AB)2  (AX)2  (BX)2
(XY)2  36 + 64 (AB)2  (8)2  (6)2
(XY)2  100 (AB)2  100
XY  10 AB  10
20.
6. d ¬
(x2 x
1)  (
2 y2  y
1)
2 y

DE ¬(8  2
)2  (6
 0)2 ¬
62  62
DE ¬36  36 D
DE ¬72
DE ¬8.49
10 (2)
7. M   2 C
12 X

2
 6 x 0
3  6 (CD)2  (CX)2 (DX)2
8. M   2

(CD)2  (3)2  (4)2
 3

2 or 1.5 (CD)2  25
x1  x2 y1  y2 4  (1) 3  5 CD  5
9.  ,     ,  
2 2 2  2  21. y
5 8

 2, 2 or (2.5, 4)  12
F
x1  x2 y1  y2 2  (2) 8  2
10.   ,    ,  
2 2 2  
2  8

 (0, 5) 4
x1  (3) y1  6
11. B(0, 5.5)  B 
2 
,  
2  8 8
x
x1  (3) y1  6 E X
0   5.5    4
2 2
0  x1  (3) 11  y1  6
3  x1 5  y1 (EF)2  (EX)2  (FX)2
The coordinates of A are (3, 5) (EF)2  (5)2  (12)2
(EF)2  169
EF  13

7 Chapter 1
22. y H 30. d ¬
(x2 x 2 
1)  (y 2  y
1)
2
8
AB ¬ [5 (4)]2 [1 2
 (3)]
4 AB ¬(1)
 2  4  
2 1  16
x AB ¬ 17
8 4 0 4 8 Because the figure is a square, the four sides
are congruent. So the perimeter of the square is
417  16.5 units.
G8 X
6  0 2  8
31. M ¬ 2
 32. M ¬ 2
6 10
(GH)2  (GX)2  (HX)2 ¬2 ¬2
(GH)2  (8)2  (15)2 ¬3 ¬5
(GH)2  289 0  5 3  2
GH  17 33. M ¬ 2 34. M ¬2

d ¬
(x2 x ¬5 ¬ 1
2 or 0.5
1)  (
y2  y 2 or 2.5
23. 2 2
1)
JK ¬
(12 
0)2 
(9  0
)2  
122 
92 6  8 3  5
35. M ¬ 2
 36. M ¬ 2

JK ¬225
  15 2
¬2 ¬2

2
24. d ¬
(x2  
x1)2 (y2 
y1)2 ¬1 ¬1
LM ¬
(7  3)
2  (9 5)2  
42  42 x1  x2 y1  y2 8  12 4  2
LM ¬   32
16  16 
37.  ,  ¬ 2, 2
2 2   
LM ¬5.7
2
¬  
0 6
2 , 2 
25. d ¬
(x2 x
1)  (
2 y2  y
1)
2
¬(10, 3)
ST ¬[6  (
3)]2 
 (5 
2)2 x1  x2 y1  y2 9  17 5  4
ST ¬9 2  3  
2 81  9
38.  ,  ¬ 2, 2
2 2   
ST ¬90 ¬2 , 2
26 9

ST ¬9.5 ¬(13, 4.5)



¬ (x2 x1)2  (y2  y1)2 x1  x2 y1  y2 11  (9) 4  (2)
26. d
UV ¬
(5  2)
2  (7 3)2  
32  42
39.  ,  ¬ 2, 2
2 2   
20 6
UV ¬9  16  25
 ¬  
2, 2 
UV ¬5 ¬(10, 3)
27. d ¬
(x2 x
1)  (
2 y2  y
1)
2

NP ¬ 
[3  (2)] 2
 [4 
(2)]2 x1  x2 y1  y2 4  8 2  (6)
NP ¬5 2  6  
2 
25  36
40.  ,  ¬ 2, 2
2 2   

NP ¬61 ¬2 , 2 
12
  4
NP ¬7.8 ¬(6, 2)
28. d ¬
(x2 x 2 
1)  (y 2  y
1)
2
x1  x2 y1  y2 3.4  7.8 2.1  3.6

QR ¬[1  5)]2
(  (5
  3)2
41.  ,  ¬ 2, 2
2 2   
QR ¬ 6  2  
2 2 36  4
11
¬  
.2 
2 , 2
5.7
 
QR ¬ 40 ¬(5.6, 2.85)
QR ¬6.3 x x y y 1.4  2.6
3.2  (5.4)
29. d ¬(x2 x1)  (
2 y2  y1)
2
42. 
1

2
,
1

2
2
¬ 2, 
2  
2 
2
XY ¬[2  (2)] 
2  [5 (1)]2 ¬2 , 2 
1.2
  .2

XY ¬4 2  6  
2 16  36 ¬(0.6, 1.1)
XY ¬52 x1  (4) y1  3
YZ ¬(4  2)2  (3 5)2
43. S(1, 5)   2
,  
2 
x1  (4) y1  3
YZ ¬2  (
2 2)  
2 44 1 ¬ 5 ¬ 
2 2
YZ ¬8 2 ¬x1  (4) 10 ¬y1  3
XZ ¬[4  (
2)]2   [3 (1)]2 2 ¬x1 7 ¬y1
XZ ¬6   
2  42 36  16 The coordinates of R are (2, 7).
x 2 y 8
XZ ¬52 44. S(2, 2)  
1

, 
1

2 2 
XY  YZ  XZ   52   8  52 x1  2 y 8
 17.3 units 2 ¬  2 ¬ 1

2 2
4 ¬x1  2 4 ¬y1  8
6 ¬x1 4 ¬y1
The coordinates of R are (6, 4).

Chapter 1 8
2  x2 5  y 52. The new coordinates are A(3, 9), B(18, 30), and

45. S 5
  3
3, 3  
2 , 2
2  C(33, 54).
2  x2 d ¬ (x2 x
1)  (
2 y2  y
1)
2
5  y
5 ¬ 
3
3 2 3 ¬2 AB ¬ (18 3) 
2 (30  9)2
2
10
  ¬2  x2 6 ¬5  y2 AB ¬ 15 
2 21  
2 225  441
3 3
AB ¬ 666
8 ¬x2 11 ¬y2
3 BC ¬ (33 18)2  (54 30)2
The coordinates of T are 8 
3 , 11 .  BC ¬ 15 
2 24  
2 225  576
x1  31.8 y1  106.4
46. M(31.1, 99.3)  , 
2 2  BC ¬ 801
x1  31.8 y1  106.4 AC ¬ (33 3)2 (54  9)2
31.1 ¬ 99.3 ¬ 
2 2 AC ¬ 302 452   900  2025
62.2 ¬x1  31.8 198.6 ¬y1  106.4 AC ¬ 2925
30.4 ¬x1 92.2 ¬y1 The perimeter of ABC is  666  
801 
The other endpoint is at (30.4°, 92.2°). 2925, which is approximately 108.2 units.
53. Sample answer: The perimeter increases by the
47. LaFayette, LA is near (30.4°, 92.2°). same factor.
48. Sample answer:  SQRT((A2  C2)  2 
x1  x2 y1  y2 26 66
(B2  D2)  2) 
54a. F  
,   F 2, 2
2 2  
49a. (54 113)2  (12 2  111.8
0  215)
 F2, 2 or F(4, 6)
8 1 2
49b. (68 175) 
2  (153  336)2  212.0
49c. (421 502) 2  (45 2  353.4
4  798) x1  x2 y1  y2 66 62
49d. (837  612)2  (980  62
5)2  420.3

E
2 2 
,   E 2, 2  
49e. (1967 1998)  2  2  37.4
(3  24)  E2, 2 or E(6, 4)
12 8

49f. 
(4173.5  2080.6)  (3
2 2  2092.9
4.9  22.4) 54b. G(4, 4); it has the same x-coordinate as F and
d ¬(x
 the same y-coordinate as E.
2 x 
1)  (y 2  y
50. 2 2
1)
AB ¬ (6 1) 2  (10  3)2 G
54c. D G B ; use the Distance Formula to show
DG  GB.
AB ¬5 2  7  
2 25  49 
d ¬(x 2 x 
1)  (y
2
2  y
1)
2
AB ¬ 74
DG ¬(4  2)2  (4 2)2
BC ¬ (11  6)2  (18 10)2
DG ¬2  2  
2 2 44
BC ¬5  2  8  
2 25  64
DG ¬8
BC ¬ 89
GB ¬(6  4
)2  (6  4)2
AC ¬ (11 1) 2 (18  3)2
GB ¬2  2  
2 2 44
AC ¬10 2 15  
2 100  225
GB ¬8
AC ¬ 325
Thus, DG  GB.
The perimeter of ABC is  74  89
  325
,
which is approximately 36.1 units. 55. 1 1
4 (x2  x1)  4 [5  (3)]

51. The new coordinates are A(2, 6), B(12, 20),  1
4 (8)  2

C(22, 36).
1(y2  y1)  1[12  (8)]
d ¬(x 2 x1)  (y
2 2  y1)
2 4 4
AB ¬(12 2)2 (20 6)2  1
4 (20)  5


AB ¬102 142   100  196 (3  2, 8  5)  (1, 3)


Verify that the coordinates of X are (1, 3):
AB ¬296
WX   [1  (3)]2 [ 2
3  (8)]
BC ¬(22 12)2 (36 20)2
WX  2  2  5
2
BC ¬10 
2 16  
2 100  256
WX   4  25
BC ¬356
WX   29
AC ¬(22 2)2 (36 6)2
WZ   [5  (
3)]2  [12 2
 (8)]
AC ¬20 
2 30  
2 400  900
WZ  8  2  2
0 2
AC ¬1300
The perimeter of ABC is  296  356
 WZ   64  400
1300, which is approximately 72.1 units. WZ   464
29  5.385
464  21.540
5.385  1 1
4 (21.540), so WX  4 WZ.


9 Chapter 1
56. Sample answer: You can copy the segment 66. 12m  7 ¬3m  52
onto a coordinate plane and then use either the 12m ¬3m  45
Pythagorean Theorem or the Distance Formula 9m ¬45
to find its length. Answers should include the m ¬5
following. 67. 8x  7 ¬5x  20
• To use the Pythagorean Theorem, draw a 8x ¬5x  13
vertical segment from one endpoint and a 3x ¬13
horizontal segment from the other endpoint 1
x ¬ 3
to form a triangle. Use the measures of these 3
68. 13n  18 ¬5n  32
segments as a and b in the formula
13n ¬5n  50
a2  b2  c2. Then solve for c. To use the
8n ¬50
Distance Formula, assign the coordinates of the 50
n ¬ 8 or 6.25
endpoints of the segment as (x1, y1) and (x2, y2).
Then use them in d  (x 2 x
1)  (
2 y2  y 2
1) to
find the length of the segment. Page 28 Geometry Activity: Modeling the
•  61  7.8 units Pythagorean Theorem
57. B; d ¬(x
 2 x 
1)  (y
2
2  y
1)
2
1. 25, 144, 169
d ¬(2 6)2 
(4 
11)2 2. 25  144  169
d ¬(8)
 2 
 (15)2 3. a2, b2, c2
d ¬64  2
25 4. The formula for the Pythagorean Theorem can
d ¬289 be expressed as a2  b2  c2.
d ¬17 5. All of these fit the a2  b2  c2 pattern.
58. A 6. The number of grid squares is 52  52, which is
50 grid squares.

Page 27 Maintain Your Skills


59. WY  WX  XY 1-4 Angle Measure
1
WY  13
4  22
 
WY  41 4

Page 32 Geometry Activity: Bisect an Angle

W Y is 41
4 in. long. 1. They are congruent.
60. AC  AB  BC 2. See students’ work.
8.5  3  BC
3. A segment bisector separates a segment into two
5.5  BC
congruent segments; an angle bisector separates

BC is 5.5 cm long.
an angle into two congruent angles.
61. Sample answer:
A
B
C D
Page 33 Check for Understanding
1. Yes; they all have the same measure.
62. m 2. Sample answer:

A T
n S R

m QPR  60
P Q m QPT  90
63. 2k ¬5k  30 mA  mZ
3. m QPS  120
3k ¬30
4. C
k ¬10
5. 
BA, 
BC
64. 14x  31 ¬12x  8
14x ¬12x  39 6. CDB, 1
2x ¬39 7. 135°; 135  90 and 135  180 so WXY is obtuse.
39
x ¬ 
2 or 19.5 8. 45°; 45  90 so WXZ is acute.
65. 180  8t ¬90  2t
90  8t ¬2t
90 ¬10t
9 ¬t

Chapter 1 10
9. 
QT bisects RQS, so RQT  SQT. 35. 
YT bisects XYW, so 1  2.
mRQT  mSQT m1 ¬m2
6x  5 ¬7x  2 5x  10 ¬8x  23
6x  7 ¬7x 5x  33 ¬8x
7 ¬x 33 ¬3x
mRQT  6x  5 11 ¬x
 6(7)  5 m2 ¬8x  23
 42  5 or 47 ¬8(11)  23

10. QT bisects RQS, so RQT  TQS and ¬88  23
mRQS  2 mRQT. ¬65

36. YT bisects XYW, so mXYW  2 m1.
22a  11 ¬2(12a 8)
6y  24 ¬2y
22a  11 ¬24a  16
24 ¬4y
22a  5 ¬24a
6 ¬y
5 ¬2a
37. 
YU bisects WYZ, so mWYZ  2 mZYU.
5 ¬a
2 82 ¬2(4r  25)
mTQS  m RQT 82 ¬8r  50
 12a  8 32 ¬8r
 12 5  
2 8
 4 ¬r
 bisects ZYW, so ZYU  UYW, and
38. YU
 30  8 or 22
mZYU  mUYW.
11. 1, right; 2, acute; 3, obtuse
 and YZ
YX  are opposite rays, so
mWYX  mUYW  mZYU ¬180.
Pages 34–35 Practice and Apply
2(12b  7)  9b  1  9b  1 ¬180
12. E 13. B 14. A
24b  14  18b  2 ¬180
15. A A
16. D 
, DB 17. A, 
B AD 42b  12 ¬180
18.  
ED, EG  
19. AD, AE 20. ABC, CBA 42b ¬168
21. FEA, 4 b ¬4
22. 2, DBA, EBA, ABE, FBA, ABF mUYW  mZYU
 9b  1
23. AED, DEA, AEB, BEA, AEC, CEA
 9(4)  1
24. D, H 25. 2  36  1 or 35
26. Sample answer: 4, 3 39. 
YU bisects ZYW, so mZYU  1 2 mZYW.

27.  bisects EAB so 5  6.
AD
1 mZYU ¬1
2 mZYW

m5 ¬mEAB
2 13a  7 ¬1 
2 (90)
1
¬(60) 13a  7 ¬45
2
¬30 13a ¬52
m6 ¬m5 a ¬4
¬30 40. The angle at which the dogs must turn to get the
28. BFD is marked with a right angle symbol, so scent of the article they wish to find is an acute angle.
mBFD  90; BFD is a right angle. 41. Sample answer: Acute can mean something that
29. 60°; 60  90, so AFB is acute. is sharp or having a very fine tip like a pen, a
30. 30°; 30  90, so DFE is acute. knife, or a needle. Obtuse means not pointed or
blunt, so something that is obtuse would be wide.
31. 90°; EFC is a right angle.
36
0
32. 150°; 150  90 and 150  180, so AFD is obtuse. 42. m1  6  60
36
0
33. 120°; 120  90 and 120  180, so EFB is obtuse. m2  
12  30
34. 
YU bisects ZYW, so ZYU  UYW. 36
0
m3  4  90
mZYU ¬mUYW 36
0
m4  6  60
8p  10 ¬10p  20
8p  10 ¬10p 36
0
m5  3  120
10 ¬2p 36
0
m6  6  60
5 ¬p
mZYU ¬8p  10
 8(5) ¬10
 40 ¬10 or 30

11 Chapter 1
43. m(angle of reflection) ¬1
2 mIBR
 d ¬(x
 2 x 
1)  (y 2  y
54. 2 2
1)
1
¬2(62) EF ¬
[5  
(3)]2
 [8 2
 (2)]
¬31
 EF ¬ 2  
82  10 
64  100
BN is at a right angle to the barrier.
So mIBA  90  31 or 59. EF ¬164

44. You can only compare the measures of the angles. EF ¬12.8
x x y y
The arcs indicate both measures are the same
regardless of the length of the rays.
M ¬ 
1
 2
2 1
, 2
2

3  5 2  8
45. 1, 3, 6, 10, 15 ¬2, 2
46. 3 rays: (3
2) 2  3 angles;
¬2, 2 or (1, 3)
2 6
4 rays: (4
3) 2  6 angles;
5 rays: (5
4) 2  10 angles; 55. WX  WR  RX
6 rays: (6
5) 2  15 angles 5 1
WX  3 
12  6 4
47. 7 rays: (7
6) 2 ¬21 angles
8 2
10 rays: (10
9) 2 ¬45 angles WX  9 12 or 9 3
n(n  1) 
48. a   2, for a  number of angles and WX is 92

3 ft long.
n  number of rays
1 56. XZ ¬XY  YZ
49. Sample answer: A degree is  360 of a circle. 15.1 ¬3.7  YZ
Answers should include the following. 11.4 ¬YZ
• Place one side of the angle to coincide with 0 on Z
Y is 11.4 mm long.
the protractor and the vertex of the angle at 57. 6x  5 2x  7
the center point of the protractor. Observe the
point at which the other side of the angle
intersects the scale of the protractor. P Q R
• See students’ work. PQ ¬QR
50. D 6x  5 ¬2x  7
6x ¬2x  12
51. C; 5n  4 ¬7(n  1)  2n
4x ¬12
5n  4 ¬7n  7  2n
x ¬3
5n  4 ¬5n  7
PQ ¬6x  5
4 ¬7
PQ ¬6(3)  5
PQ ¬18  5 or 13
Page 36 Maintain Your Skills 58. Five planes are shown: plane FJK, plane HJK,
plane GHK, plane FGK, and plane FGH.
52. d ¬(x
 
2 x1)2  (y
2  y
1)
2
59. F, L, J 60. G or L
AB ¬(5  2)2  (7
 3)2
61. 14x  (6x  10) ¬90
AB ¬3  4
2 2
20x  10 ¬90
AB ¬9  16   25 20x ¬100
AB ¬5 x ¬5
x x y y
 2
M ¬ 
1
2 1
, 2
2
 62. 2k  30 ¬180
2k ¬150
25 37
¬2, 2 k ¬75
63. 180  5y ¬90  7y
¬,  or (3.5, 5)
7 10
2 2
90  5y ¬7y
90 ¬2y
53. d ¬(x
 2 x 
1)  (y
2
2  y
1)
2
45 ¬y
CD ¬ 
[6  (2)]2
 (4 
0)2 64.
1
90  4t ¬4 (180  t)
CD ¬    
82 
64  16
42 4(90  4t) ¬180  t
CD ¬80
 360  16t ¬180  t
CD ¬8.9 180  16t ¬t
x x y y 180 ¬15t
 2
M ¬ 
1
2 1
, 2
2
 12 ¬t
2  6 0  4 (6m  8)  (3m  10) ¬90
¬, 
65.
2 2 9m  18 ¬90
¬,  or (2, 2)
4 4 9m ¬72
2 2 m ¬8
66. (7n  9)  (5n  45) ¬180
12n  36 ¬180
12n ¬144
n ¬12

Chapter 1 12
Page 36 Practice Quiz 2
x x y y 1-5 Angle Relationships
 2
1. M ¬ 
1
2
,
1
2
2 
3  (4) 1  3
¬, 
2 2 Page 38 Geometry Activity: Angle Relationships
¬,  or 1, 1
 1 2 1. BCE  DCA 2. DCB  ACE
2 2 2 3. See students’ work.
d ¬
(x2 x
1)  (
2 y2  y
1)
2
4. ACD and ECB, DCB and ACE; measures
AB ¬
(4 
3)2  2
[3  (1)] for each pair of vertical angles should be the
same.
AB ¬(7)
 
2 42   
49  16
5. ACD and DCB, DCB and BCE, BCE and
AB ¬65
 ECA, ECA and ACD; measures for each
linear pair should add to 180.
AB ¬8.1
6. Sample answers: The measures of vertical angles
x x y y
 2
2. M ¬ 
1
2
,
1
2
2 
are equal or vertical angles are congruent.
The sum of the measures of a linear pair is 180 or
6  2 4  (8)
¬,  angles that form a linear pair are supplementary.
2 2
8 4
¬,  or (4, 2)
2 2
d ¬(x

Page 41 Check for Understanding
2 x 
1)  (y 2  y
2 2
1)
1.
CD ¬
(2  6 
)2  (8  4)2
CD ¬
(4)2  2  
 (12) 
16  144
70 110
CD ¬160

2. Sample answer: When two angles form a linear
CD ¬12.6 pair, then their noncommon sides form a straight
x x y y
 2
3. M ¬ 
1
2
,
1

2 
2 angle, which measures 180. When the sum of the
measures of two angles is 180, then the angles are
10  (10) 20  (20)
¬,  supplementary.
2 2
3. Sample answer: The noncommon sides of a linear
¬(0, 0) pair of angles form a straight line.
d ¬
(x2 x
1)  (
2 y2  y
1)
2 4. Sample answer: ABF and CBD are vertical
angles. They each have measures less than 90°, so
EF ¬(10
  10)2
 (
20  2
0)2 they are acute.
EF ¬(20)
2  (
40)2 5. Sample answer: ABC and CBE are adjacent
angles. They each have measures greater than
EF ¬
400 
1600
90°, so they are obtuse.
EF ¬2000
 6. Explore: The problem involves three angles: an
angle, its supplement, and its complement.
EF ¬44.7
Plan: Let A be the given angle, B its
4. mRXT ¬mSXT  mRXS supplement and C the complement. Then
111 ¬3a  4  2a  5 mA  mB  180 or mB  180  mA, and
111 ¬5a  1 mA  mC  90 or mC  90  mA. The
110 ¬5a problem states that mB  3 mC  60, so
22 ¬a substitute for mB and mC and solve for mA.
mRXS  2a  5
Solve: mB ¬3 mC  60
 2(22)  5
180  mA ¬3(90  mA)  60
 44  5 or 49
180  mA ¬270  3 mA  60
5. mQXS ¬mQXR  mRXS 180  2 mA ¬210
4a  1 ¬a  10  91 2 mA ¬30
4a  1 ¬a  101 mA ¬15
4a ¬a  102
Examine: Check to see if the answer satisfies the
3a ¬102
problem.
a ¬34
mQXS  4a  1 The measure of the complement of an angle with
 4(34)  1 a measure of 15° is 75°. The measure of the
 136  1 or 135 supplement of the original angle is 165°.
3 75  60 ¬165
165 ¬165
The answer checks.

13 Chapter 1
7. Lines p and q are perpendicular if angles 1 and 2 mSQR  9  4a
are both right angles. Then m1  m2  90. So  9  4(7)
3x  18  90 8y  70  90  9  28 or 37
3x  72 8y  160 Examine: Add the angle measures to verify their
x  24 y  20 sum is 90.
8. No; while SRT appears to be a right angle, no 53  37  90
information verifies this. 18. Explore: The problem relates the measures of two
9. Yes; they share a common side and vertex, so they complementary angles. You know that the sum of
R
are adjacent. Since P  falls between P
Q
 and P
S, the measures of complementary angles is 90.
mQPR  90, so the two angles cannot be Plan: The angles are complementary, so the sum
complementary or supplementary. of the measures of the angles is 90.
10. m4  60 and 2 and 4 are vertical angles, so Solve: 16z  9  4z  3 ¬90
m2  60. 20z  6 ¬90
1 and 4 are supplementary angles. 20z ¬96
m1  m4 ¬180 z ¬4.8
m1  60 ¬180 16z  9 ¬16(4.8)  9
m1 ¬120 ¬76.8  9 or 67.8
1 and 3 are vertical angles so m3  120. 4z  3 ¬4(4.8)  3
¬19.2  3 or 22.2
Examine: Add the angle measures to verify that
Pages 42–43 Practice and Apply the angles are complementary.
11. WUT and VUX are vertical angles. They each 67.8  22.2  90
have measures less than 90°, so they are acute. 19. Explore: The problem involves an angle and its
12. WUV and XUT are vertical angles. They each supplement. You know that the sum of the
have measures greater than 90°, so they are measures of two supplementary angles is 180.
obtuse. Plan: Let mT  x. Its supplement has measure
13. ZWU is a right angle and ZWU and YWU 180  x. The problem also states that mT is 20
are supplementary so YWU is a right angle. more than four times its supplement.
Then UWT and TWY are adjacent angles that Solve: x ¬4(180  x)  20
are complementary because x ¬720  4x  20
mUWT  mTWY  mYWU. 5x ¬740
14. VXU and WYT are nonadjacent angles that x ¬148
are complementary because So mT  148.
mVXU  mWYT  60  30 or 90. Examine: Check to see if the answer satisfies the
15. WTY and WTU is a linear pair with vertex T. problem.
If mT  148, its supplement has a measure of 32.
16. UVX
4
32  20  148
17. Explore: The problem relates the measures The answer checks.
formed by perpendicular rays.
   20. Explore: The problem involves an angle and its
Plan: QP QR, so PQS and SQR are
supplement.
complementary.
Plan: Let the measure of one angle be x. Its
mPQS  mSQR  90
supplement has measure 180  x. The problem
R states that the measure of the angle’s supplement
is 44 less than x.
Solve: 180  x ¬x  44
S
224  x ¬x
9 4a
224 ¬2x
112 ¬x
180  x ¬180  112 or 68
4 7a The measures of the angle and its supplement are
112 and 68.
Q P Examine: 112  44  68
Solve: 4  7a  9  4a ¬90 21. Explore: The problem involves an angle and its
13  11a ¬90 supplement.
11a ¬77 Plan: Let the measure of one angle be x. Its
a ¬7 supplement has measure 180  x. The problem
mPQS  4  7a states that one angle measures 12° more than
 4  7(7) the other.
 4  49 or 53

Chapter 1 14
Solve: x  12 ¬180  x 29. BFA and DFE are vertical angles so they have
x ¬168  x the same measure.
2x ¬168 3r  12 ¬8r  210
x ¬84 3r ¬8r  198
180  x  180  84 or 96 11r ¬198
The measures of the angle and its supplement are r ¬18
84 and 96. mBFA  3r  12
Examine: 96  84  12  3(18)  12
 54  12 or 66
22. Explore: The problem states that m1 is five
BFA and AFE are adjacent supplementary
less than 4 m2 and 1 and 2 form a linear
angles, so mBFA  mAFE  180.
pair. So m1  m2  180.
66  mAFE ¬180
Plan: Let m1  x.
mAFE ¬114
Solve: m1  m2 ¬180
x  m2 ¬180 30. ¬L and M are complementary.
mL  mM ¬90
4 m2  5  m2 ¬180
y  2  2x  3 ¬90
5 m2 ¬185
y  2x  1 ¬90
m2 ¬37
y  2x ¬89
m1  4 m2  5 y ¬89  2x
 4(37)  5 N and P are complementary.
 148  5 or 143
mN  mP ¬90
Examine: 4(37)  5 ¬143 2x  y  x  1 ¬90
143 ¬143 3x  y  1 ¬90
1 and 2 are supplementary because 3x  y ¬91
143  37  180. y ¬91  3x
23. Always; the sum of two angles that each measure y ¬3x  91
less than 90° can never equal 180°, so if one angle Equate the two expressions for y and solve for x.
is acute the other must be obtuse. 89  2x ¬3x  91
24. Always; complementary angles are angles whose 180  2x ¬3x
measures have a sum of 90°, so each angle must 180 ¬5x
measure less than 90°. 36 ¬x
25. Sometimes; for example, consider the following: Now substitute the value of x into either
mA  90 expression for y and solve for y.
mB  90 y ¬3x  91
mC  90 y ¬3(36)  91
Then mA  mB  180 and mB  mC  180 y ¬108  91 or 17
and mA  mC  180. mL ¬y  2
However, now consider mL ¬17  2 or 15
mA  100 mM ¬2x  3
mB  80 mM ¬2(36)  3
mC  100 mM ¬72  3 or 75
mA  mB  180 and mB  mC  180 but mN ¬2x  y
mA  mC  200, so A is not supplementary mN ¬2(36)  17
to C. mN ¬72  17 or 55
26. Never; PN and PQ
 have point P in common and mP ¬x  1
form a right angle. NPQ is formed by P N
 and mP ¬36  1 or 35
Q
P . Since P is the vertex, NPQ is a right angle. 31. Yes; the symbol denotes that DAB is a right
27. CF  
 FD if CFD is a right angle. So find a so angle.
that mCFD  12a  45 is equal to 90. 32. Yes; they are vertical angles.
12a  45 ¬90 33. Yes; the sum of their measures is mADC, which
12a ¬45 is 90.
a ¬3.75
34. No; there is no indication of the measures of these
28. mAFC ¬mAFB  mBFC angles.
90 ¬8x  6  14x  8
35. No; we do not know mABC.
90 ¬22x  2
88 ¬22x 36. Sample answer: Complementary means serving to
4 ¬x fill out or complete, while complimentary means
given as a courtesy or favor. Complementary has
the mathematical meaning of an angle completing
the measure to make 90°.

15 Chapter 1
37. Sample answer: 52. d ¬(x
 2 x 
1)  (y
2
2  y
1)
2

1 EF ¬
[4 
(2)]2
 [1
0  (
10)]2

2
EF ¬ 
(2)2  202
EF ¬   404
4  400 
38. mAKB  mBKC  mCKD  20  25  45 or EF ¬20.1
90, so AKD is a right angle and A
KK D.
53. d ¬(x
 2 x 
1)  (y
2
2  y
1)
2
mDKE  mEKF  60  30 or 90, so DKF is
a right angle and K K
D F. GH ¬
(6 
7)2 
(0  2)2
mEKF  mFKG  30  60 or 90, so EKG is GH ¬(13)
2  (
2)2
a right angle and K K
E G.
GH ¬169 
4  173

39. Because WUT and TUV are supplementary, GH ¬13.2
let mWUT  x and mTUV  180  x. A
bisector creates measures that are half of the 54. d ¬(x
 2 x 
1)  (y
2
2  y
1)
2

original angle, so mYUT  1 x


2 mWUT or 2 and JK ¬ 
[4  (8)]2
 (7 
9)2
x
mTUZ  1

2 mTUV or
180 

2 . Then mYUZ  JK ¬12
 2 
(2)2
mYUT  mTUZ or
x JK ¬144 
4  148

x  
180  18
. This sum simplifies to  0
2 2 2 or JK ¬12.2
90. Because mYUZ  90, Y UU Z.
55. d ¬(x
 2 x 
1)  (y
2
2  y
1)
2
40. Sample answer: The types of angles formed
depends on how the streets intersect. There may LM ¬
(3  1
)2  (
1  3)2
be as few as two angles or many more if there are LM ¬2
 2  
2  (4) 4  16
more than two lines intersecting. Answers should LM ¬20

include the following. LM ¬4.5
• linear pairs, vertical angles, adjacent angles
56.  3x
• See students’ work.
41. A; my ¬89 and mx  my  180. 1x 4x  17
mx ¬180  my
mx ¬180  89 or 91
42. Let x be the number. P Q R
4 5 6 ¬2(10  x) PR ¬PQ  QR
120 ¬20  2x 3x ¬1  x  4x  17
100 ¬2x 3x ¬3x  18
50 ¬x 6x ¬18
43. Lines , m, and n are in plane E, so   AB , x ¬3
m  AB , and n  AB. QR ¬4x  17
QR ¬4(3)  17
QR ¬12  17 or 5
Page 43 Maintain Your Skills 57.
7n  8
44. mKFG  90, so KFG is acute.
45. mHFG  90, so HFG is obtuse. 4n  3 6n  2
46. mHFK  90, so HFK is a right angle.
47. JFE is marked with a right angle symbol, P Q R
so JFE is a right angle. PR ¬PQ  QR
48. mHFJ  90, so HFJ is acute. 7n  8 ¬4n  3  6n  2
49. mEFK  90, so EFK is obtuse. 7n  8 ¬10n  1
7n  9 ¬10n
50. d ¬(x
 2 x 
1)  (y
2
2  y
1)
2
9 ¬3n
AB ¬
(0  3
)2  (1
 5)2 3 ¬n
AB ¬(3)
 2
 (4
)2 QR ¬6n  2
QR ¬6(3)  2
AB ¬9  16  25

QR ¬18  2 or 20
AB ¬5
58.  ¬3, w  8
51. d ¬(x2 x
1)  (
2 y2  y
1)
2
2  2w ¬2(3)  2(8)
CD ¬
(5  5)
2  (9
 1)2 ¬6  16 or 22
CD ¬02  82  64
 59.  ¬3, w  8
CD ¬8 w ¬3 8
¬24

Chapter 1 16
60. s ¬2 PS ¬[03)]
 ( 2 (4 4)2
4s ¬4(2) ¬ 3  0
2 2
¬8 ¬ 93
61.   3, w ¬8, s  2 Perimeter  PQ  QR  RS  PS
w  ws ¬3 8  8 2  5  3  5  3 or 16 units
¬24  16 or 40 10. AB  BC  CD  AD ¬95
62.   3, w ¬8, s  2 3a  2  2(a  1)  6a  4  5a  5 ¬95
s(  w) ¬2(3  8) 3a  2  2a  2  6a  4  5a  5 ¬95
¬2(11) or 22 16a  1 ¬95
16a ¬96
a ¬6
Page 44 Geometry Activity: Constructing AB  3a  2 BC ¬2(a  1)
Perpendiculars  3(6)  2 ¬2(6  1)
1. See students’ work.  18  2  20 ¬2(5)  10
2. The first step of the construction locates two CD  6a  4 AD ¬5a  5
points on the line. Then the process is very similar  6(6)  4 ¬5(6)  5
to the construction through a point on a line.  36  4  40 ¬30  5  25
11. P  5s
 5(921)
 4605
1-6 Polygons The perimeter of the outside of the Pentagon is
4605 feet.

Page 48 Check for Understanding


1. A regular decagon has 10 congruent sides, Pages 49–50 Practice and Apply
so divide the perimeter by 10. 12. There are 4 sides, so the polygon is a quadrila-
2. Saul; Tiki’s figure is not a polygon. teral. No line containing any of the sides will pass
3. P  3s through the interior of the quadrilateral, so it is
4. Sample answer: Some of the lines containing the convex. The sides are congruent but the angles
sides pass through the interior of the pentagon. are not so it is irregular.
13. There are 8 sides, so the polygon is an octagon. No
line containing any of the sides will pass through
the interior of the octagon, so it is convex. The
sides are congruent, and the angles are
congruent, so it is regular.
14. There are 10 sides, so the polygon is a decagon.
5. There are 5 sides, so the polygon is a pentagon. A line containing any of the sides will pass
A line containing side N M
 will pass through the through the interior of the decagon, so it is
interior of the pentagon, so it is concave. The sides concave. The sides are congruent, but the decagon
are not congruent, so it is irregular. is concave so it cannot be regular. It is irregular.
6. There are 6 sides, so the polygon is a hexagon. 15. There are 5 sides, so the sign is a pentagon.
No line containing any of the sides will pass 16. There are 4 sides, so the sign is a quadrilateral.
through the interior of the hexagon, so it is
17. There are 3 sides, so the sign is a triangle.
convex. The sides are congruent, and the angles
are congruent, so it is regular. 18. There are 12 sides, so the sign is a dodecagon.
7. Perimeter  8  8  6  5  6 19. P  2  2w
 33 ft  2(28)  2(13)
 56  26  82 ft
8. The new side lengths would be 16 ft, 16 ft, 12 ft,
10 ft, and 12 ft so the new perimeter would be 20. P  6  12  8 15  15
16  16  12  10  12  66 ft. The perimeter  56 m
doubles. 21. P  6  2  2  6  2  2  6  2  2  6  2  2
9. d ¬ (x2 x  40 units
1)  (y2  y
2 2
1)
PQ ¬[[03)]
 ( 2  (8 4)2 22. The new length would be 4(28) or 112 feet.
¬ 3  4
2 2 The new width would be 4(13) or 52 feet.
P  2  2w
¬ 25  5
 2(112)  2(52)  328
QR ¬(3 )  0 2  (8 8)2
The perimeter is multiplied by 4.
¬ 3  0
2 2
23. The new side lengths are 18 m, 36 m, 24 m, 45 m,
¬ 9 or 3
and 45 m. The new perimeter is 18  36  24 
RS ¬(0 )  3 2  (4 8)2 45  45 or 168 m. The perimeter is tripled.
¬(3)
 2 ) (42

¬ 25  5

17 Chapter 1
24. The new side lengths are 3 and 1 units. The new VZ ¬ (2  3)2 
(12  0)2
perimeter is 3  1  1  3  1  1  3  1  1  ¬(5)
 2  (12) 2
3  1  1 or 20 units. The perimeter is divided by 2. ¬ 169  13
25. The length of each side is multiplied by 10, so 10 The perimeter is VW  WX  XY  YZ  VZ or
can be factored out of the sum of the sides. Thus 13  17   85  6 13  58.2 units.
the new perimeter is multiplied by 10 so the 29. There are 6 sides and all sides are congruent.
perimeter is 12.5(10)  125 m. All sides are 90
6 or 15 cm.
26. d ¬(x  2 x1)  (y
2  2  y
1)
2
30. There are 4 sides and all sides are congruent.
AB ¬[31)]
 ( 2  (4  1)2 All sides are 14
4 or 3.5 mi.
¬4  3
2 2
31. P ¬x  1  x  7  3x  5
¬25  5 31 ¬5x  1
BC ¬(6)  3 2  (0  4)2 30 ¬5x
¬3 2  (4) 2
6 ¬x
¬25  5 x  1 ¬6  1  5
CD ¬ (2  6)2  ( 3  0)2 x  7 ¬6  7  13
¬(4)
 2  (3) 2 3x  5 ¬3(6)  5  13
¬25  5 The sides are 13 units, 13 units, and 5 units.
AD ¬[2  (1)]2   (3  1)2 32. P ¬6x  3  8x  3  6x  4
¬3 2  (4) 2 84 ¬20x  4
80 ¬20x
¬25  5
4 ¬x
The perimeter is AB  BC  CD  AD  5  5 
6x  3  6(4)  3  21
5  5 or 20 units.
8x  3  8(4)  3  35
27. d ¬(x  2 x1)  (y
2  2  y
1)
2
6x  4  6(4)  4  28
PQ ¬ [3  (2)]  2  (3  3)2 The sides are 21 m, 35 m, and 28 m.
¬5 2  02
33. P ¬2  2w
¬25  5 42 ¬2(3n  2)  2(n  1)
QR ¬ (7  3) 2  (0  3)2 42 ¬6n  4  2n  2
¬4 2  (3) 2
42 ¬8n  2
¬ 25  5 40 ¬8n
RS ¬ (3  7 )2  ( 3  0)2 5 ¬n
¬(4)
 2  (3) 2 3n  2 ¬3(5)  2  17
¬25  5 n  1 ¬5  1  4
The sides are 4 in., 4 in., 17 in., and 17 in.
ST ¬ (2  3)2  [3  (3)]2
¬(5)
 2 0 2 34. P ¬2x  1  2x  x  2x
41 ¬7x  1
¬25  5
42 ¬7x
TU ¬ [6  (2)]2  [0 (3 )]2
6 ¬x
¬(4)
 2 3 2
2x  1 ¬2(6)  1  11
¬25  5 2x ¬2(6)  12
PU ¬ [6  (2)]2  (0 3)2 The sides are 6 yd, 11 yd, 12 yd, and 12 yd.
¬(4)
 2  (3) 2
35. 52 units; count the units in the figure.
¬25  5 36.
The perimeter is PQ  QR  RS  ST  TU  PU 5 squares
or 5  5  5  5  5  5  30 units.
28. d ¬ (x2 x 1)  (
2 y2  y1)
2 8 squares
VW ¬ (2  3) 
2 (12  0)2
¬ (5) 
2  12 2

¬ 169  13 9 squares


WX ¬ [10   (2)]  2  (
3   12)2
¬ (8) 
2  (1 5) 2

¬ 289  17
36a. It is a square with side length of 3 units.
XY ¬ [8  (10)]  2  [12  (3)]2
36b. In Part a, the rectangle with the greatest
¬ 
2  (9)
2 2
number of squares was a square with side of 3
¬85
units. So if a rectangle has perimeter of 36 units,
YZ ¬ [2  (8)]2  [ 12  (12)]2 a square would have the largest area. The side of
¬6 2  02 36
this square would be  4  9 units.
¬36  6

Chapter 1 18
37. Sample answer: Some toys use pieces to form Page 52 Geometry Software Investigation:
polygons. Others have polygon-shaped pieces that Measuring Polygons
connect together. Answers should include the 1. The sum of the side measures equals the
following. perimeter.
• triangles, quadrilaterals, pentagons
2. 35.53  90.38  54.09  180

3. See students’ work.
4. Sample answer: When the lengths of the sides are
doubled, the perimeter is doubled.
5. See student’s work.
6. Sample answer: The sum of the measures of the
angles of a triangle is 180.
38. A square has four congruent sides. If one side 7. Sample answer: The sum of the measures of the
remains to be fenced, three sides have been angles of a quadrilateral is 360; pentagon  540;
fenced. So each side has length 1

3 (3x) or x meters.
hexagon  720.
39. D; 5n  5 ¬10 8. Sample answer: The sum of the measures of the
5n ¬5 angles of polygons increases by 180 for each
n ¬1 additional side.
11  n ¬11  1  10 9. yes; sample answer: triangle: 3 sides, angle
measure sum: 180; quadrilateral: 4 sides, angle
measure sum: 180  180  360; pentagon: 5 sides,
Page 50 Maintain Your Skills angle measure sum: 360  180  540; hexagon:
40. Always; true by the definitions of linear pairs and 6 sides, angle measure sum: 540  180  720
supplementary angles. 10. Yes; sample answer: If the sides of a polygon are
41. Sometimes; angles with measures 100 and 80 are a, b, c, and d, then its perimeter is a  b  c  d.
supplementary but two right angles are also If each of the sides are increased by a factor of n
supplementary. then the sides measure na, nb, nc, and nd, and
42. 
AM bisects RAL, so MAR  MAL. the perimeter is na  nb  nc  nd. By factoring,
mMAR ¬mMAL the perimeter is n(a  b  c  d), which is the
2x  13 ¬4x  3 original perimeter increased by the same factor as
2x  16 ¬4x the sides.
16 ¬2x
8 ¬x
mRAL ¬mMAR  mMAL Chapter 1 Study Guide and Review
¬2x  13  4x  3
¬2(8)  13  4(8)  3 or 58 Pages 53–56
43. 
AM bisects RAL, so MAR  LAM and 1. B
d; ABC
, so C
B is the midpoint of A .
mRAL ¬2 mMAR. 2. h 3. f 4. e
x  32 ¬2(x  31) 5. b 6. g 7. p or m
x  32 ¬2x  62 8. K or L 9. F 10. S
x  94 ¬2x 11. 12.
94 ¬x S T U
C
mLAM  mMAR
m
¬x  31 V
¬94  31  63 13.
 bisects MAR, so RAS  SAM. 25
44. AS

AM bisects LAR, so mLAR  2 mMAR. 7 3x
mRAS ¬mSAM
25  2x ¬3x  5
20  2x ¬3x A P B
20 ¬5x AB ¬AP  PB
4 ¬x 25 ¬7  3x
mLAR ¬2 mMAR 18 ¬3x
¬2(mSAM  mRAS) 6 ¬x
¬2(3x  5  25  2x) PB ¬3x
¬2(x  30) ¬3(6)
¬2x  60 ¬18
¬2(4)  60  68

19 Chapter 1
14. 9 23. d ¬(x
 2 x 
1)  (y
2
2  y
1)
2

MP ¬[6 (4)]2
 (1
9  16)2
4c 2c
¬(2)
 2  3  
2 13
¬3.6
A P B x x y y
AB ¬AP  PB
 2
24. M ¬ 
1
2
,
1
2
2 
0  22 0  (18)
9 ¬4c  2c ¬2, 2
9 ¬6c
22 18
3 ¬c or c  1.5 ¬2,  2 or (11, 9)
2
PB ¬2c x x y y
25. M ¬, 
1 2 1 2

 
¬2 3

2
2 2
6  12 3  (7)
¬3 ¬2, 2
15. 6 10
8s  7 ¬2, 2 or (3, 5)
x x y y
s2 4s 26. M ¬, 
1 2 1 2
2 2
2  (1) 5  (1)
¬2, 2
A P B
¬2, 2 or (0.5, 2)
1 4
AB ¬AP  PB
8s  7 ¬s  2  4s x x y y
27. M ¬, 
1 2 1 2
8s  7 ¬5s  2 2 2
8s ¬5s  9 3.4  (2.2) 7.3  (5.4)
¬
2
 , 2
3s ¬9
1.2 12.7
s ¬3 ¬2, 2 or (0.6, 6.35)
PB ¬4s
¬4(3) 28. D , FG
29. FE  30. DEH
¬12 31. 70°; 70  90, so SQT is acute.
16. 11 32. 110°; 110  90 and 110  180, so PQT is obtuse.
k6 33. 50°; 50  90, so T is acute.
 2k 34. 70°; 70  90, so PRT is acute.
35. 
XV bisects YXW, so YXV  VXW.
A P B mYXV ¬mVXW
AB ¬AP  PB 3x ¬2x  6
11 ¬2k  k  6 x ¬6
11 ¬k  6 mYXW ¬mYXV  mVXW
5 ¬k ¬3x  2x  6
5 ¬k ¬3(6)  2(6)  6
PB ¬k  6 ¬36
¬5  6 
36. XW bisects YXZ, so YXW  WXZ.
¬1 mYXW ¬mWXZ
17. yes, because HI  KJ  9 m 12x  10 ¬8(x  1)
18. no, because AB  13.6 cm and AC  19.3 cm 12x  10 ¬8x  8
19. not enough information to determine if 12x ¬8x  18
5x  1  4x  3 4x ¬18
20. d ¬(x
 2 x 
1)  (y
2
2  y
1)
2 x ¬9

2
AB ¬(3  1)2 
(2  0
)2 mYXZ ¬mYXW  mWXZ
¬12x  10  8(x  1)
¬(4)
 2 
 2  20
2 
¬4.5
¬12 9 
 9
2  10  8 2  1 
¬54  10  44
21. d ¬(x
 2 xy
1)  (
2
2  y
1)
2
¬88
GL ¬[3
  (
7)]2 
 (3  4)2
¬10
 2 
(1) ¬
2 101
¬10.0
22. d ¬(x
 2 x 
1)  (y
2
2  y
1)
2

JK ¬(4  0)2  (


1  0)2
¬4 2  (
1) ¬
2 17
¬4.1

Chapter 1 20
37. 
XW bisects YXZ, so YXW  WXZ and d ¬(x
 2 x 
1)  (y 2  y
46. 2 2
1)
mWXZ ¬1 2 mYXZ.

WX¬[73)]
 ( 2 (1 5)2
mWXZ ¬1
2 mYXZ
 ¬
10 
2 (4)  116
2 
7x  9 ¬1
2 (9x  17)
 XY ¬(5  7
)2  ( 4  1)2
7x  9 ¬9 17 ¬
(2)2 
 (5 )2   29
2 x 2
  
9 35 YZ ¬(5  5) 
2 2
[0  (4)]
7x ¬ 
2 x  2
5
¬
(10)2  4  
2 116
35
x ¬ 
2 2 WZ ¬[5  (3)]2  (0 5)2
x ¬7 ¬
(2)2 
 (5 )2   29
mYXW ¬mWXZ Perimeter ¬WX  XY  YZ  WZ
¬7x  9
¬ 116   29  116  29

¬7(7)  9
¬40 ¬32.3 units
38. TWY, WYX 39. TWY, XWY
40. W
T W Z if mTWZ  90.
mTWZ ¬2c  36 Chapter 1 Practice Test
90 ¬2c  36
54 ¬2c Page 57
27 ¬c 1. true 2. true
41. ZWX ¬ZWY  YWX 3. False; the sum of two supplementary angles is
90 ¬4k  2  5k  11 180.
90 ¬9k  9 4. true 5. m
81 ¬9k
6. D 7. C
9 ¬k
8. 29
42. There are 4 sides, so the polygon is a quadrilateral.
No line containing any of the sides will pass 2
through the interior of the quadrilateral, so it is 3x
convex. The sides are congruent, and the angles
are congruent, so it is regular. UV W
43. The figure is not a polygon because there are UW ¬UV  VW
sides that intersect more than two other sides. 29 ¬2  3x
44. There are 8 sides, so the polygon is an octagon. 27 ¬3x
There is a side such that a line containing that 9 ¬x
side will pass through the interior of the octagon, VW ¬3x
so it is concave. The sides are not congruent, so ¬3(9)  27
the octagon is irregular. 9.
42
45. d ¬(x  2 x 
1)  (y
2
2  y
1)
2

AB ¬ (5  1) 2  (1 2)2 r 6r


¬4 2  
2  (1) 17
BC ¬ (9  5) 2  (2 1)2 U V W
¬4 2  1  
2 17 UW ¬UV  VW
CD ¬ (9  9) 2  (5 2)2 42 ¬r  6r
¬0 2  323 42 ¬7r
6 ¬r
DE ¬ (5  9) 2  (6 5)2
VW ¬6r
¬ (4)
 2  1  
2 17 ¬6(6)  36
EF ¬ (1  5) 2  (5 6)2 10. 15
¬ (4)
 2 ) (12  17

AF ¬ (1  1) 2  (5 2)2
4p  3 5p
¬0 2  323
Perimeter ¬AB  BC  CD  DE  EF  AF U V W
¬ 17   17  3  17 
17  3 UW  UV  VW
¬6  4 17 15 ¬4p  3  5p
¬22.5 units 15 ¬9p  3
18 ¬9p
2 ¬p
VW ¬5p
¬5(2)  10

21 Chapter 1
11.  4c ¬0
 2  4
2  (4)

DE ¬(4  0)2 (3  0)2


3c  29  2c  4
¬(4)
 2
 (3)2  25  5
AE ¬[4  (6)]
2  (3  2)2
U V W
¬2
2   
 (5) 2 29
UW ¬UV  VW Perimeter ¬AB  BC  CD  DE  AE
4c ¬3c  29  (2c  4) ¬29
  5  4  5  29 
4c ¬c  25 ¬24.8 units
5c ¬25
23.
c ¬5
VW ¬2c  4
¬2(5)  4  6 B
12. d ¬(x2 x 
1)  (y
2
2  y
1)
2

GH ¬(3  0)2 
(4  0)2
¬(3) 
2  4  
2 25 C
¬5
13. d ¬(x2 x
1)  (
2 y2  y1)
2

NK ¬(2  5)2 
(8  2)2 S X Y
¬(7) 
2  6  
2 85 Use the information known about the cities to
¬9.2 place them on a coordinate plane. Then use the
14. d ¬(x2 x
1)  (
2 y2  y
1)
2 gridlines to form a triangle using the points S for
Springfield, B for Brighton, and Y. Use the
AW ¬ [2  (4)]2 [2 2
 (4)]
Pythagorean Theorem.
¬2  6  
2 2 40 (SB)2 ¬(SY)2  (BY)2
¬6.3 (SB)2 ¬62  72
15. C 16. 
EC, 
ED (SB)2 ¬85
17. ABD or ABE 18. 9 SB ¬9.2
19. 4r  7  r  2¬ 180 Highway 1 is approximately 9.2 miles long.
5r  5¬ 180 24.
5r¬ 175
r¬ 35 B
4r  7 ¬4(35)  7 or 147
r  2 ¬35  2 or 33
20. Let x be the measure of one angle. Then the other
angle has measure x  26 and x  x  26  90. C
x  x  26¬ 90
2x  26¬ 90
2x¬ 64
x¬ 32 S X Y
x  26  32  26  58 Use the information known about the cities to
21. d ¬(x
 2 x 
1)  (y
2
2  y
1)
2 place them on a coordinate plane. Then use the
PQ ¬[1  (
6)]2  [1 2
 (3)] gridlines to form a triangle using the points S for
Springfield, C for Capital City, and X. Use the
¬72  2  
2 53
Pythagorean Theorem.
QR ¬(1  1)2  [ 2
5  (1)] (SC)2 ¬(SX)2  (CX)2
¬0 2  4
2  (4)
(SC)2 ¬52  32
PR ¬[1  (
6)]2  [5 2
 (3)] (SC)2 ¬34
¬72   
 (2) 2 53 SC ¬5.8
Perimeter ¬PQ  QR  PR Highway 4 is approximately 5.8 miles long.
¬ 53  4  53 25. C; the figure has sides with a common endpoint
¬18.6 units that are collinear.
22. d ¬(x
 2 x 
1)  (y
2
2  y
1)
2

AB ¬
[4  (6)]2 (7 2)2
¬2
2  5  
2 29
BC ¬
[0  (
4)]2  (4  7)2
¬4
 2
2  (3)

¬
25  5
CD ¬
(0  0)2  (0 4)2

Chapter 1 22
Chapter 1 Standardized Test Practice y ¬2x  3
y ¬2(2)  3 or 7
The solution is (2, 7).
Pages 58–59 12. y
1. C; there are 15 hours between 7 A.M. and 10 P.M., B C
so there are 15 60  900 minutes during that
time. Then Juanita blinks 11 900 or 9900 times
during the time she is awake.
2. A x
O
2x2  12x  16 2(x2  6x  8)
3. B;  
2x  4 ¬ 
2(x  2) A D
2(x  4)(x  2)
¬ 
2(x  2)
¬x  4 ABCD is a rectangle, so D must have coordinates
4. A; three noncollinear points determine a plane, so D(3, 1).
x x y y
if the two planes are distinct, their intersection is
not a plane. Two planes intersect in only one line.
13. M  
1
2
 2
,
1
2
2 
5. B; 1 fathom  6 feet so 55 fathoms  55(6) or  2, 2
2  (4) 1  3

330 feet, which is 110 yards.

6. C; use the Pythagorean Theorem, where x is the  2, 2  (1, 1)
2 2

height up the side of the house that the ladder 14. (AB)2 ¬602  1102
reaches. (AB)2 ¬3600  12,100
(18)2 ¬(6)2  x2 (AB)2 ¬15,700
324 ¬36  x2 AB ¬125 m
288 ¬x2 15. P  2  2w
17.0 ¬x  2(16)  2(24)
7. C; mABD ¬mCBD  80
2x  14 ¬5x  10 The perimeter of the basement is 80 feet. If the
2x  14  10 ¬5x  10  10 80
pieces of plasterboard are 4 feet wide, it takes 4
2x  24 ¬5x or 20 pieces of plasterboard to cover the walls.
2x  24  2x ¬5x  2x 16a. y
24 ¬3x
8 ¬x
mABD  2x  14
 2(8)  14
 30 O x
8. D; let x be the measure of FEG.
mDEG  mFEG ¬180
612x  x ¬180
15
2 x ¬180

x x y y
x ¬24 16b. M ¬ 
1
2
,
2
1

2
2 
mDEG ¬61

2 (24)
¬156
4 
 0 
2 , 2
0 4

¬(2, 2)

9. D; treasure 4
2
 0 0  (4)
, 2 ¬(2, 2) 


4
 0 0  (4)
2, 2 ¬(2, 2) 
4 
 
0 04
2 , 2 ¬(2, 2) 
17a. You are given that a  25. The measure of the
other angle marked with a single arc is also
115 25 because the arcs tell us that the angles are
Kaitlin Henry congruent. Both of the angles marked with
double arcs have a measure of a  10, which
is 25  10 or 35. In the remaining angle,
If Kaitlin turns 115°, Henry turns 180°  115°
b  180  (25  35  25  35) or 60 because
or 65°.
the angles can be combined to form linear pairs,
10. 2x  6  4x2  x  x2  5  5x2  x  1 which are supplementary.
11. 2y ¬3x  8 17b. All are acute.
y ¬2x  3
2(2x  3) ¬3x  8
4x  6 ¬3x  8
x ¬2

23 Chapter 1
Chapter 2 Reasoning and Proof
Page 61 Getting Started
1. 3n  2  3(4)  2 2-1 Inductive Reasoning and
 12  2  10 Conjecture
2. (n  1)  n  (6  1)  6
76 Pages 63–64 Check for Understanding
 13 1. Sample answer: After the news is over, it’s time
3. n2  3n  (3)2  3(3) for dinner.
 9  3(3) 2. Sometimes; the conjecture is true when E is
990 between D and F; otherwise it is false.
4. 180(n  2)  180(5  2) 3. Sample answer: When it is cloudy, it rains.
 180(3)  540 Counterexample: It is often cloudy and it does not
5. nn
2  ¬10 2 
 10
  rain.
¬10(5)  50 4. There is one of each shape in the first figure.
There are two of each shape in the second figure
n(n  3) 8(8  3)
6. 2 ¬ 2
 and three of each shape in the third figure. So the
8(5)
¬  next figure will have four of each shape.
2
4 0
¬2  20
5. 8 5 2 1 4
7. 6x  42 ¬4x   
3 3 3 3
42 ¬2x
The numbers in the sequence increase by 3. The
21 ¬x
next number will increase by 3. So, it will be 4  3
8. 8  3n ¬2  2n or 7.
10  3n ¬2n
6. PQ  TU
10 ¬5n
2 ¬n P Q
9. 3(y  2) ¬12  y R S
3y  6 ¬12  y
3y ¬18  y T U
2y ¬18
y ¬9 7. 
AB and 
CD intersect at a single point P, so the
10. 12  7x ¬x  18 lines are distinct. Thus the points A, B, C, and D
7x ¬x  30 are not all on the same line. So, A, B, C, and D are
6x ¬30 noncollinear.
x ¬5
A D
11. 3x  4 ¬1
2x  5

P
3x ¬1
2x  9

C B
5x ¬9
2
18 8. False; if x  2, then x  (2) or 2.
x ¬ 
5 9. True; opposite sides of a rectangle are congruent,
2
12. 2  2x ¬3x  2 and the sides of the rectangle can be determined
4  2x ¬2  from the order of the letters in its name.
3x
10. Sample answer: Snow will not stick on a roof with
4 ¬8

3x a steep angle.
3
 ¬x
2
13. AGB and EGD are vertical angles, so
mAGB  mEGD. Pages 64–66 Practice and Apply
4x  7 ¬71 11. Each figure is formed by adding another row of
4x ¬64 dots to the top and another column of dots on the
x ¬16 side. The number of dots in each figure is 2, 6, 12,
20.
14. mBGC  mCGD  mDGE  180
2 6 12 20
45  8x  4  15x  7 ¬180   
4 6 8
23x  42 ¬180
23x ¬138
x ¬6

Chapter 2 24
The numbers increase by 4, 6, and 8. The next 20. Each arrangement of blocks is formed by adding a
number will increase by 10. So, it will be 30. level of blocks to the bottom. The number of
blocks on the bottom level of the figures is 1, 3,
and 6.
1 3 6
 
2 3

The number of blocks on the bottom increase by 2


12. Each figure adds a triangle and changes the and 3. The next increase will be 4. So there will be
orientation of the triangles. The next figure will 10 blocks on the bottom of the fourth figure. The
have five triangles with the same orientation as upper levels of the figure have a total of 10 blocks.
the three triangles in the second figure. The total number of blocks in this figure is 20.

13. 1 2 4 8 16
  
1 2 4 8

The numbers increase by 1, 2, 4, and 8, which are


the first four numbers in the sequence. The next
number will increase by the fifth number in the 21. Perpendicular lines form four right angles, so
sequence, or 16. So, the next number will be lines  and m form four right angles.
16  16 or 32.
14. 4 6 9 13 18
  
2 3 4 5

The numbers increase by 2, 3, 4, and 5. The next


number will increase by 6. So, it will be 18  6 or m
24. 22. Graph A, B, and C. Connect the points to see that
15. 1 1 5 7 3 they lie on the same line. Thus A, B, and C are
3 3 3
2
2
2
2
collinear.
3 3 3 3
y C (5, 10)
The numbers increase by 2

3 . The next number will 8
be 3  2 1
 or 
3 3
1
.
16. 1 1 1 1 1 4
2 4 8 16
   
1 1
2 1
2 1
2 2 B(2, 1)
The numbers are multiplied by 1 –8 –4 O 4 8x
2 . The next
1 1 1
number will be 
16  
2 or 
32 . –4

17. 2 6 18 54 –8
  
(3) (3) (3)
A(–2, –11)
The numbers are multiplied by 3. The next
number will be 54  (3) or 162. 23. Linear pairs of angles are supplementary, so 3
18. 5 25 125 625 and 4 are supplementary.
  
(5) (5) (5)

The numbers are multiplied by 5. The next


3 4
number will be 625  (5) or 3125.
19. Each arrangement of blocks is formed by adding a 24. 
BD bisects ABC so the two angles formed are
level of blocks to the bottom. The numbers of congruent: ABD  DBC.
blocks in the sequence are 1, 5, and 14.
A
1 5 14
  D
4 9

The numbers increase by 4 and 9, which are C


squares of 2 and 3, respectively. The next number B
will increase by the square of 4, or 16. So it will 25. Graph P, Q, and R. The points form a triangle.
be 14  16 or 30. Find the distance between each pair of points to
determine the type of triangle.
d  (x
x2 1) 
2 (y2 
y1)2
PQ  [61)]
 ( 2  (2
 7)2
 79)
2  ( 2

 130

QR  (6)  62  [5)]
 (22
 02  7
2

 49
 or 7

25 Chapter 2
PR  [6
1)]
 ( 2 (5 
7)2 
AB is horizontal, and B
C is vertical. So the
 72)
2  ( 2 segments are perpendicular to each other. So B
 53
 is a right angle. Therefore, ABC is a right
The lengths PQ, QR, and PR are all different, so triangle.
PQR is a scalene triangle. 33. True; the square of any real number is a
y nonnegative number.
P
R(6, 5)
34. False; D, E, and F do not have to be collinear.
(–1, 7)
35. False; JKLM may not have a right angle.
36. True; any three noncollinear points form a
triangle.
37. trial and error, a process of inductive reasoning
O x 38. The first three alkanes have 1, 2, and 3 carbon
atoms. The fourth alkane will have 4 carbon
Q (6, –2) atoms. The first three alkanes have 4, 6, and 8
hydrogen atoms. The fourth alkane, butane, will
26. A square has 4 congruent sides. From the name of
have 8  2 or 10 hydrogen atoms.
I, IJ
the square the sides are H , J
K
, and K
H
.
Thus, HI  IJ  JK  KH. H H H H
H I
H C C C C H

H H H H
K J 39. The 1st alkane has formula CH4 (or C1H4).
27. A rectangle has 4 sides where opposite sides are The 2nd alkane has formula C2H6.
congruent. From the name of the rectangle the The 3rd alkane has formula C3H8.
Q
sides are P, QR
, S
R
, and P
S. The pairs of The subscripts for C increase by 1 and are the
Q
opposite sides are P , S
R
 and QR
, P
S. Thus, same number as the number of the alkane in the
PQ  SR and QR  PS. series. The subscripts for H increase by 2. So the
P Q 7th alkane in the series has formula
C7H(8  2  2  2  2) or C7H16.
40. The nth alkane has n carbon atoms so the
subscript for C is n. The number of hydrogen
S R atoms in the nth alkane is 2 more than twice the
28. A triangle that has a right angle is a right numbers of the alkane in the series, or 2n  2. So
triangle. The Pythagorean Theorem is true for the subscript for H is 2n  2. The formula is
every right triangle. Since B is the right angle of CnH2n  2.
C
the triangle, the hypotenuse is A  and the legs 41. False; if n  41, then n2  n  41  (41)2  41 
B
are A  and B
C . So (AB)2  (BC)2  (AC)2. 41 or 412, which is not prime.
A
42. Sample answer: By past experience, when dark
clouds appear, there is a chance of rain. Answers
C should include the following.
B
• When there is precipitation in the summer, it is
29. False;
usually rain because the temperature is above
freezing. When the temperature is below
2 freezing, as in the winter, ice or snow forms.
1
• See students’ work.
30. False; if y  7 and m  5, then 7  5  10 and 43. C; 1, 1, 2, 3, 5, 8
5  4, but 7  6. 112
31. False; 123
W X Y Z 235
358
32. True; Each number in the sequence is found by adding
y the two numbers before it. 5  8  13, so the next
A B
number in the sequence is 13.
44. D; Let x be the sum of the six numbers and y be
the sum of the three numbers whose average is 15.
x
6  18
C
x  18(6) or 108

x
O

Chapter 2 26
y
3  15 59. PN  3x and PN  24.
y  15(3) or 45 3x ¬24
The sum of the remaining three numbers is x ¬8
108  45 or 63. MP ¬7x
¬7(8) or 56
60. PN  9c and PN  63.
Page 66 Maintain Your Skills 9c ¬63
c ¬7
45. There are 6 sides, so the polygon is a hexagon. No
MP ¬2c
line containing any of the sides will pass through
¬2(7) or 14
the interior of the hexagon, so it is convex. Not all
sides are congruent, so the hexagon is irregular. 61. MN  MP  PN
36 ¬4x  5x
46. There are 5 sides, so the polygon is a pentagon.
36 ¬9x
No line containing any of the sides will pass
4 ¬x
through the interior of the pentagon, so it is
MP  4x
convex. The sides and angles are congruent, so the
¬4(4) or 16
pentagon is regular.
62. MN  MP  PN
47. There are 7 sides, so the polygon is a heptagon. 60 ¬6q  6q
There is a side such that a line containing the 60 ¬12q
side will pass through the interior of the 5 ¬q
heptagon, so it is concave. Not all sides are MP  6q
congruent, so the heptagon is irregular. ¬6(5) or 30
48. Yes; the symbol denotes that KJN is a right 63. MN  MP  PN
angle. 63 ¬4y  3  2y
49. No; we do not know anything about the angle 63 ¬6y  3
measures. 60 ¬6y
50. No; we do not know whether MNP is a right 10 ¬y
angle. MP  4y  3
51. Yes; they form a linear pair. ¬4(10)  3
¬43
52. Yes; since the other three angles in rectangle KLPJ
are right angles, KLP must also be a right angle. 64. MN  MP  PN
x x y y 43 ¬2b  7  8b
53. M  
1

2
,
1

2
2
2  43 ¬10b  7
1 
 2 , 3  (5) 50 ¬10b
2
  5 
5 ¬b

 4
2 , 2  or (2, 1)
  2
MP ¬2b  7
x1  x2 y1  y2 ¬2(5)  7
54. M   2
, 
2  ¬3

4  (3) 
 2, 2 7 
1  65. x  2  5
2  2  4. 4 5. So 2  2
5.
 1
2 , 2  or (0.5, 4)
 8 3  2  5. 5  5. So 3  2
5.
x1  x2 y1  y2 4  2  6. 6  5. So 4  2  5.
55. M   2
, 
2  5  2  7. 7  5. So 5  2  5.

 
4  (2) 9  (15) The values in the replacement set that make the
 2, 2
inequality true are 4 and 5.
24
 2
2 , 2  or (1, 12)
   66. 12  x 0
x1  x2 y1  y2 12  11  1. 1 ¬0. So 12  11 0.
56. M   2
, 
2  12  12  0. 0 ¬0. So 12  12 0.
12  13  1. 1 ¬0. So 12  13 0.
 2
5  , 22
7   
4
12  14  2. 2 ¬0. So 12  14 0.

 2 
 2
2 , 2 or (1, 1)
The values in the replacement set that make the
inequality true are 13 and 14.
x1  x2 y1  y2
57. M   2
, 
2  67. 5x  1  25
5(4)  1  21. 21 25. So 5(4)  1
25.
 2, 2
8  3 1.8  6.2
 5(5)  1  26. 26  25. So 5(5)  1  25.
5(6)  1  31. 31  25. So 5(6)  1  25.
 11  4.4

2 , 2 or (5.5, 2.2) 5(7)  1  36. 36  25. So 5(7)  1  25.
x1  x2 y1  y2
58. 
M  , 
2 2  The values in the replacement set that make the
inequality true are 5, 6, and 7.
1.5  6 

 2,  (4)
2
3

5 
  .5  3

2 , 2 or (2.75, 1.5)

27 Chapter 2
14. p q r p  q (p  q)  r
2-2 Logic T T T T T
T T F T T
Pages 71–72 Check for Understanding T F T T T
1. The conjunction (p and q) is represented by the T F F T T
intersection of the two circles. F T T T T
2a. Sample answer: October has 31 days or F T F T T
5  3  8. F F T F T
2b. Sample answer: A square has five right angles F F F F F
and the Postal Service does not deliver mail on
15. The states that produce more than 100 million
Sundays.
bushels of corn are represented by the set labeled
2c. Sample answer: July 5th is not a national Corn. There are 14 states that produce more than
holiday. 100 million bushels of corn.
3. A conjunction is a compound statement using the 16. The states that produce more than 100 million
word and, while a disjunction is a compound bushels of wheat are represented by the set
statement using the word or. labeled Wheat. There are 7 states that produce
4. 9  5  14 and February has 30 days.; false, more than 100 million bushels of wheat.
because p is true and q is false 17. The states that produce more than 100 million
5. 9  5  14 and a square has four sides.; true, bushels of corn and wheat are represented by the
because p is true and r is true intersection of the sets. There are 3 states that
6. February has 30 days and a square has four produce more than 100 million bushels of corn
sides.; false, because q is false and r is true and wheat.
7. 9  5  14 or February does not have 30 days.;
true, because p is true and q is true
8. February has 30 days or a square has four sides.; Pages 72–74 Practice and Apply
true, because r is true 18. 64
  8 and an equilateral triangle has three
9. 9  5 14 or a square does not have four sides.; congruent sides; false, because p is false and q is
false, because p is false and r is false true.
10. 19. 64
  8 or an equilateral triangle has three
p q q p  q congruent sides; true, because q is true.
T T F F 20. 64
  8 and 0 0; false, because p is false and
T F T T r is false.
F T F F 21. 0 0 and an obtuse angle measures greater than
90° and less than 180°; false, because r is false
F F T F and s is true.
11. 22. An equilateral triangle has three congruent sides
p q pq or 0 0; true, because q is true.
T T T 23. An equilateral triangle has three congruent sides
T F F and an obtuse angle measures greater than 90°
and less than 180°; true, because q is true and
F T F
s is true.
F F F 24. 64
  8 and an obtuse angle measures greater
12. than 90° and less than 180°; false, because
q r qr p is false and s is true.
T T T 25. An equilateral triangle has three congruent sides
T F T and 0 0; false, because q is true and r is false.
F T T 26. 0 0 or 64
  8; false, because r is false and
p is false.
F F F
27. An obtuse angle measures greater than 90° and
13. less than 180° or an equilateral triangle has three
p r p p  r
congruent sides; true, because s is true and
T T F F q is true.
T F F F 28. 64
  8 and an equilateral triangle has three
F T T T congruent side, or an obtuse angle measures
greater than 90° and less than 180°; true, because
F F T F s is true.

Chapter 2 28
29. An obtuse angle measures greater than 90° and 37. q r r q  r
less than 180°, or an equilateral triangle has
three congruent sides and 0 0; true, because T T F F
s is true. T F T T
30. p q p p  q F T F F
T T F T F F T F
T F F F 38. p q p q p  q
F T T T
F F T T T T F F F
T F F T F
31. p q p q p  q F T T F F
T T F F F F F T T T
T F F T F 39. p q r p r q  r p  (q  r)
F T T F F
T T T F F F F
F F T T T T T F F T T T
32. T F T F F F F
p q r pq (p  q)  r
T F F F T F F
T T T T T
F T T T F F T
T T F T F
F T F T T T T
T F T T T
T F F T F F F T T F F T

F T T T T F F F T T F T
F T F T F 40. p q r q r q  r p  (q  r)
F F T F F
T T T F F F F
F F F F F
T T F F T T T
33. q r q and r T F T T F T T
T T T T F F T T T T
T F F F T T F F F F
F T F F T F F T T F
F F F F F T T F T F
F F F T T T F
34. p q p or q
41. The teens that said they listened to none of these
T T T types of music are represented by the region
T F T outside the Pop, Country, and Rap sets. There are
F T T 42 teens that said they listened to none of these
types of music.
F F F
42. The teens that said they listened to all three
35. types of music are represented by the intersection
p r p or r
of three sets. There are 7 teens that said they
T T T listened to all three types of music.
T F T 43. The teens that said they listened to only pop and
rap music are represented by the intersection of
F T T
the Pop and Rap sets excluding the teens that
F F F also listen to country music (hence listen to all
three types of music). There are 25 teens that said
36. p q p and q they listened to only pop and rap music.
T T T 44. The teens that said they listened to pop, rap, or
country music are represented by the union of the
T F F
Pop, Rap, and Country sets. There are
F T F 175  25  7  34  45  10  62 or 358 teens
F F F that said they listened to pop, rap, or country
music.

29 Chapter 2
45. Level of Participation 57. 1 3 9 27
Among 310 Students   
3 3 3

The numbers in the sequence are multiplied by 3.


The next number will be multiplied by 3. So, it
Sports Academic will be 27  3 or 81.
95 20 Clubs
60 58. 6 3 3 3
2 4
1
1
1
2 2 2

The numbers in the sequence are multiplied by 1 


2.
46. The students that participate in either clubs or 1
sports are represented by the union of the sets. The next number will be multiplied by 2. So, it
There are 60  20  95 or 175 students that will be 3
 1 3
4  2 or 8 .
participate in clubs or sports. 59. 17 13 9 5
47. The students that do not participate in either   
4 4 4
clubs or sports are the students outside the union The numbers in the sequence decrease by 4. The
of the sets. There are 310 – 175 or 135 students next number will decrease by 4. So, it will be
that do not participate in either clubs or sports. 5  4 or 1.
48. false 60. 64 16 4 1
  
49. True; Rochester is located on Lake Ontario but 1
 1
 1

4 4 4
Syracuse is not. The statement is a disjunction, so The numbers in the sequence are multiplied by 1 
it is true. 4.
The next number will be multiplied by 1
. So, it
4
50. False; Buffalo is located on Lake Erie, so the
negation of the statement is false. will be 1  1
 or 1.
4 4
51. 61. 5 15 45 135
  
3 3 3
B
The numbers in the sequence are multiplied by 3.
C The next number will be multiplied by 3. So, it
A
will be 135  3 or 405.
62. d  (x x 2 1) 
2 (y2  y1)2
52. b; the relationship between Teams A and C is not AB  [1 6)]
 ( 2  (3  7)2
known, so statements a and c might not be true.   7  (
2 4)  65
2 
It is known that every member of Team A is also BC  (2   1)2  (7  3)2
a member of Team B so b is true.   (3)2   (1 0)2  109 
53. Sample answer: Logic can be used to eliminate AC  [2   (6)]7 2  (  7)2
false choices on a multiple choice test. Answers  4 14)
2  (  212 2 
should include the following.
Perimeter  AB  BC  AC
• Math is my favorite subject and drama club is
 65   109   212 
my favorite activity.
 33.1
• See students’ work.
63. d  (x x 2 1) 
2 (y2  y1)2
54. A; the marks on A B
 and B C indicate that
B
A B C
, so AB  BC is true. Statement A is the DE  [5  )] (102 2  [(9)]
 2
only true statement about ABC.  5  2  72 
55. C; let x be the first of the integers. Then x  2 is  74 
the second integer. Their sum is 78. P  4s
x  x  2 ¬78  4 DE
2x  2 ¬78  474   34.4
2x ¬76 64. d  (x x 2 1)  (y2 
2 y1)2
x ¬38, x  2  40
HI   (8  5) 
2 [9  (10)]2
The two integers are 38 and 40, the greater of
which is 40.  (13)
 2  1  170
2 
IJ   [5  (8)]2  [ 5  ( 9)]2
  3  4
2 2

Page 74 Maintain Your Skills  255


56. 3 5 7 9 JK   [2  (5)]2  [ 4  ( 5)]2
 
2 2 2
 3  2  12   10 
The numbers in the sequence increase by 2. The HK   (2  5) 
2 [4  (10)]2
next number will increase by 2. So, it will be 9  2
 (7)
 2  6  85
2 
or 11.
Perimeter  HI  IJ  JK  HK
 170  5  10   85 
 30.4

Chapter 2 30
65. d  (xx 2 1) 
2 (y2 y1)2 4. Hypothesis: it rains on Monday;
LM  (4 )  2 2  (5  1) 2 Conclusion: I will stay home
 2  2  42 5. Hypothesis: x  3  7;
Conclusion: x  10
 20  25 
6. Hypothesis: a polygon has six sides;
MN  (6 )  4 2  (4  5)2
Conclusion: it is a hexagon
 21)
2  (  5 2 
7. Sample answer: If a pitcher is a 32-ounce pitcher,
NP  (7 )  642  (  4)2 then it holds a quart of liquid.
 1 8)
2  ( 2  65 8. Sample answer: If two angles are supplementary,
PQ  (5 )  782  [4)]  (2 then the sum of the measures of the angles is 180.
 (2)
 2 ) (4 2
9. Sample answer: If an angle is formed by
 20  25  perpendicular lines, then it is a right angle.
QR  (3 )  572  [8)]  (2 10. The hypothesis is true because you drove 70 miles
 (2)
 2  1 2 per hour, and the conclusion is true because you
 5  received a speeding ticket. Since the promised
result is true, the conditional statement is true.
LR  (3 )  272  (  1)2
11. The hypothesis is false, and the statement does
 1 8)
2  ( 2
not say what happens if you drive 65 miles per
 65 hour or less. You could still get a speeding ticket if
Perimeter you are driving in a zone where the posted speed
 LM  MN  NP  PQ  QR  LR limit is less than 65 miles per hour. In this case,
 25   5    65  25   5
  65
 we cannot say that the statement is false so the
 65   2 65 statement is true.
 29.5 12. The hypothesis is true, but the conclusion is false.
66. 145°; 90 145 180 so ABC is obtuse. Because the result is not what was promised, the
67. 55°; 55 90 so DBC is acute. conditional statement is false.
68. 90°; right 13. Converse: If plants grow, then they have water;
69. The front and back could be as much as 35.5 feet true.
each and the sides could be as much as 75.5 feet Inverse: If plants do not have water, then they
each. will not grow; true.
P  2  2w Contrapositive: If plants do not grow, then they do
 2(35.5)  2(75.5) not have water. False; they may have been killed
 71  151  222 by overwatering.
Michelle should buy 222 feet of fencing. 14. Conditional in if-then form: If you are flying in an
70. 5a  2b  5(4)  2(3) airplane, then you are safer than riding in a car.
 20  6 or 14 Converse: If you are safer than riding in a car,
then you are flying in an airplane. False; there
71. 4cd  2d  4(5)(2)  2(2)
are other places that are safer than riding in a car.
 40  4 or 44
Inverse: If you are not flying in an airplane, then
72. 4e  3f  4(1)  3(2) you are not safer than riding in a car. False; there
 4  (6) or 10 are other places that are safer than riding in a car.
73. 3g  h  3(8)2  (8)
2
Contrapositive: If you are not safer than riding in
 3(64)  (8) a car, then you are not flying in an airplane; true.
 192  (8) or 184 15. Sample answer: If you are in Colorado, then
aspen trees cover high areas of the mountains.
If you are in Florida, then cypress trees rise from
the swamps.
2-3 Conditional Statements If you are in Vermont, then maple trees are
prevalent.
Page 78 Check for Understanding
1. Writing a conditional in if-then form is helpful so
that the hypothesis and conclusion are easily Pages 78–80 Practice and Apply
recognizable. 16. Hypothesis: 2x  6  10;
2. Sample answer: If you eat your peas, then you Conclusion: x  2
will have dessert. 17. Hypothesis: you are a teenager;
3. In the inverse, you negate both the hypothesis Conclusion: you are at least 13 years old
and the conclusion of the conditional. In the 18. Hypothesis: you have a driver’s license;
contrapositive, you negate the hypothesis and the Conclusion: you are at least 16 years old
conclusion of the converse. 19. Hypothesis: three points lie on a line;
Conclusion: the points are collinear

31 Chapter 2
20. Hypothesis: a man hasn’t discovered something 40. Converse: If you live in Texas, then you live in
he will die for; Dallas. False; you could live in Austin.
Conclusion: he isn’t fit to live Inverse: If you don’t live in Dallas, then you don’t
21. Hypothesis: the measure of an angle is between live in Texas. False; you could live in Austin.
0 and 90; Contrapositive: If you don’t live in Texas, then you
Conclusion: the angle is acute don’t live in Dallas; true.
22. Sample answer: If you buy a 1-year fitness plan, 41. Converse: If you are in good shape, then you
then you get a free visit. exercise regularly; true.
23. Sample answer: If you are a math teacher, then Inverse: If you do not exercise regularly, then you
you love to solve problems. are not in good shape; true.
Contrapositive: If you are not in good shape, then
24. Sample answer: If I think, then I am.
you do not exercise regularly. False; an ill person
25. Sample answer: If two angles are adjacent, then may exercise a lot, but still not be in good shape.
they have a common side.
42. Conditional: If two angles are complementary,
26. Sample answer: If two angles are vertical, then then their sum is 90.
they are congruent. Converse: If the sum of two angles is 90, then
27. Sample answer: If two triangles are equiangular, they are complementary; true.
then they are equilateral. Inverse: If two angles are not complementary,
28. The hypothesis is true because you are 19 years then their sum is not 90; true.
old, and the conclusion is true because you vote. Contrapositive: If the sum of two angles is not 90,
Since the predicted result is true, the conditional then they are not complementary; true.
statement is true. 43. Conditional: If a figure is a rectangle, then it is a
29. The hypothesis is false, and the conclusion is true. quadrilateral.
The statement doesn’t say what happens if you Converse: If a figure is a quadrilateral, then it is
are younger than 18 years old. It is possible that a rectangle. False; it could be a rhombus.
you vote in a school election. In this case, we Inverse: If a figure is not a rectangle, then it is
cannot say that the statement is false. Thus, the not a quadrilateral. False; it could be a rhombus.
statement is true. Contrapositive: If a figure is not a quadrilateral,
30. The hypothesis is true, and the conclusion is false. then it is not a rectangle; true.
Because the result is not what was predicted, the 44. Conditional: If an angle is a right angle, then its
conditional statement is false. measure is 90.
31. The hypothesis is false, and the conclusion is Converse: If an angle has a measure of 90, then it
false. The statement doesn’t say what happens if is a right angle; true.
you are younger than 18 years old. In this case, Inverse: If an angle is not a right angle, then its
we cannot say that the statement is false. Thus, measure is not 90; true.
the statement is true. Contrapositive: If an angle does not have a
measure of 90, then it is not a right angle; true.
32. The hypothesis is true because your sister is 21
years old, and the conslusion is true because she 45. Conditional: If an angle is acute, then its measure
votes. Since the predicted result is true, the is less than 90.
conditional statement is true. Converse: If an angle has measure less than 90,
then it is acute; true.
33. The hypothesis is true, and the conclusion is false.
Inverse: If an angle is not acute, then its measure
Because the result is not what was predicted, the
is not less than 90; true.
conditional statement is false.
Contrapositive: If an angle’s measure is not less
34. True; P, Q, and R are collinear, and P is in plane than 90, then it is not acute; true.
M and Q is in plane N. The line containing P and
46. Sample answer: In Alaska, if it is summer, then
Q is the intersection of M and N, so the line that
there are more hours of daylight than darkness.
is the intersection of these planes is the line
In Alaska, if it is winter, then there are more
through P and Q, and thus R. So P, Q, and R are
hours of darkness than daylight.
in M.
47. Sample answer: In Alaska, if there are more
35. True; points Q and B lie in plane N, so the line
hours of daylight than darkness, then it is
that connects them also lies in plane N.
summer. In Alaska, if there are more hours of
36. True; Q is on the line through P that is the darkness than daylight, then it is winter.
intersection of planes M and N. The line is in M,
48. Sample answer: If I am exercising, then I am
so Q is in M.
asleep. If I am exercising, then I am not asleep.
37. False; P and A lie in plane M and Q and B lie in
49. Conditional statements can be used to describe
plane N. M and N are distinct planes, so P, Q, A,
how to get a discount, rebate, or refund. Sample
and B are not coplanar.
answers should include the following.
38. false
• If you are not 100% satisfied, then return the
39. True; line RQ is the same as line PQ since P, Q, product for a full refund.
and R are collinear. P is in M and Q is in N so M
• Wearing a seatbelt reduces the risk of injuries.
and N intersect at line PQ and hence line RQ.

Chapter 2 32
50. C; The contrapositive of a statement always has 61. PQR is a right angle.
the same truth value as the statement. P
51. B; let x be the number of girls in class. Then there
are 32  x boys in class.
x Q R
 ¬5
32  x 3
3x ¬5(32  x) 62. d  (xx 2 1) 
2 (y2  y1)2
3x ¬160  5x CD  [02)] ( 2  [3 (1)]2
8x ¬160  2  2  4  20
2 
x ¬20  4.5
32  x ¬12 63. d  (xx 2 1) 
2 (y2  y1)2
Thus, there are 20 girls and 12 boys in class, so JK  [13)]
 ( 2 (0  5)2
there are 20  12 or 8 more girls than boys.  4 5)
2  (  41
2 
 6.4
64. d  (xx 2 1) 
2 (y2  y1)2
Page 80 Maintain Your Skills PQ   
[2  (3)] 2  [3  (
1)]2
52. George Washington was the first president of the
 52)
2  (  29
2 
United States and a hexagon has 5 sides.
 5.4
p  q is false because p is true and q is false.
65. d  (xx 2 1) 
2 (y2  y1)2
53. A hexagon has five sides or 60  3  18.
q r is false because q is false and r is false. RS  (4
 1)2 7)][3  (2
54. George Washington was the first president of the  (5)
 2  10  125
2 
United States or a hexagon has five sides.  11.2
p q is true because p is true. 66. Subtract 4 from each side.
55. A hexagon doesn’t have five sides or 60  3  18. 67. Multiply each side by 2.
q r is true because q is true. 68. Divide each side by 8.
56. George Washington was the first president of the
United States and a hexagon doesn’t have five
sides. Page 80 Practice Quiz 1
p  q is true because p is true and q is true. 1. False
57. George Washington was not the first president of W X
the United States and 60  3 18.
p  r is false because p is false and r is
true.
58. AB  CD; AD  BC
D C
Y
2. True; m1  m2 ¬90
A B m2 ¬90  m1
59. The sum of the measures of the angles in a m2  m3 ¬90
triangle is 180. (90  m1)  m3 ¬90
G m1  m3 ¬0
67 m3 ¬m1
3.
F 45 p q p p  q
68 T T F F
H
T F F F
60. JKL has two sides congruent.
F T T T
y K
F F T F

J L

O x

33 Chapter 2
4. p q r qr p  (q  r)
T T T T T
2-4 Deductive Reasoning
T T F F T
T F T F T Page 84 Check for Understanding
T F F F T 1. Sample answer: a: If it is rainy, the game will be
F T T T T cancelled.
F T F F F b: It is rainy.
c: The game will be cancelled.
F F T F F
F F F F F 2. Transitive Property of Equality: a  b and b  c
implies a  c. Law of Syllogism. a implies b and b
5. Converse: If two angles have a common vertex, implies c implies a implies c. Each statement
then the angles are adjacent. False; ABD is not establishes a relationship between a and c
adjacent to ABC. through their relationships to b.
A C 3. Lakeisha; if you are dizzy, that does not
necessarily mean that you are seasick and thus
D have an upset stomach.
B
4. Valid; the conditional is true and the hypothesis is
Inverse: If two angles are not adjacent, then they
true, so the conclusion is true.
do not have a common vertex; False, ABC and
DBE have a common vertex and are not 5. Invalid; congruent angles do not have to be
adjacent. vertical.
A C 6. no conclusion
B 7. Let p, q, and r represent the parts of the
statement.
D E p: the midpoint divides a segment
Contrapositive: If two angles do not have a q: two segments are congruent
common vertex, then they are not adjacent; true. r: two segments have equal measures
The given statements are true,
Statement (1): p → q
Statement (2): q → r
Page 81 Reading Mathematics
So by the Law of Syllogism p → r. Thus, the
1. Conditional: If a calculator runs, then it has
midpoint of a segment divides it into two
batteries.
segments with equal measures.
Converse: If a calculator has batteries, then it will
run. 8. p: Molly arrives at school at 7:30 A.M.
False; a calculator may be solar powered. q: she will get help in math
r: she will pass her math test
2. Conditional: If two lines intersect, then they are
Statement (3) is a valid conclusion by the Law of
not vertical.
Syllogism.
Converse: If two lines are not vertical, then they
intersect. 9. Invalid; not all angles that are congruent are
False; two parallel horizontal lines will not right angles.
intersect. 10. A 35-year old female pays $14.35 per month for
3. Conditional: If two angles are congruent, then $30,000 of insurance, and Ann is a 35-year old
they have the same measure. female, so by the Law of Detachment, Ann pays
Converse: If two angles have the same measure, $14.35 per month.
then they are congruent. 11. No; Terry could be a man or a woman. She could
true be 45 and have purchased $30,000 of life
4. Conditional: If 3x  4  20, then x  7. insurance.
Converse: If x  7, then 3x  4  20.
False; 3x  4  17 when x  7.
5. Conditional: If a line is a segment bisector, then it Pages 85–87 Practice and Apply
intersects the segment at its midpoint. 12. invalid; 10  12  22
Converse: If a line intersects a segment at its 13. Valid; since 5 and 7 are odd, the Law of
midpoint, then it is a segment bisector. Detachment indicates that their sum is even.
true 14. Valid; since 11 and 23 are odd, the Law of
Detachment indicates that their sum is even.
15. Invalid; the sum is even.
16. Valid; A, B, and C are noncollinear, and by
definition three noncollinear points determine a
plane.

Chapter 2 34
17. Invalid; E, F, and G are not necessarily • Doctors use what is known to be true about
noncollinear. diseases and when symptoms appear, then
18. Invalid; the hypothesis is false as there are only deduce that the patient has a particular illness.
two points. 34. C; if A were true, then Yasahiro would be a
19. Valid; the vertices of a triangle are noncollinear, professional athlete by I, contradicting II. If B
and therefore determine a plane. were true, then either Yasahiro is a professional
20. no conclusion athlete (contradicting II), or Yasahiro gets paid,
which, together with III and I, contradicts II. D
21. If the measure of an angle is less than 90, then it
contradicts III. If C was not true, then II would
is not obtuse.
be contradicted. Therefore C must be true.
22. If X is the midpoint of YZ
, then YXX Z
.
35. B; 15% off the $16 meal means the diner’s meal
23. no conclusion cost (0.85)($16) or $13.60. 20% of $13.60 means
24. p: you are an in-line skater the diner left a tip of (0.20)($13.60) or $2.72.
q: you live dangerously Thus, the diner paid a total of $13.60  $2.72 or
r: you like to dance $16.32.
Yes, statement (3) follows from (1) and (2) by the
Law of Syllogism.
25. p: the measure of an angle is greater than 90 Page 87 Maintain Your Skills
q: the angle is obtuse 36. If you try Casa Fiesta, then you’re looking for a
Yes, statement (3) follows from (1) and (2) by the fast, easy way to add some fun to your family’s
Law of Detachment. menu.
26. Invalid; statement (1) is true, but statement (3) 37. They are a fast, easy way to add fun to your
does not follow from (2). Not all congruent angles family’s menu.
are vertical angles.
38. No; the conclusion is implied.
27. p: an angle is obtuse
39. q r qr
q: the angle cannot be acute
Yes, statement (3) follows from (1) and (2) by the T T T
Law of Detachment. T F F
28. p: you drive safely F T F
q: you can avoid accidents
F F F
Yes, statement (3) follows from (1) and (2) by the
Law of Detachment. 40. p r p p  r
29. p: you are a customer
T T F T
q: you are always right
r: you are a teenager T F F F
r → p does not follow from (p → q)  (r → q); F T T T
invalid F F T T
30. p: John Steinbeck lived in Monterey
41. p q r qr p  (q  r)
q: during the 1940s, one could hear the grating
noise of the fish canneries. T T T T T
If John Steinbeck lived in Monterey in 1941, then T T F T T
he could hear the grating noise of the fish
T F T T T
canneries.
T F F F F
31. p: Catriona Le May Doan skated her second 500
meters in 37.45 seconds F T T T F
q: She beat the time of monique Garbrecht- F T F T F
Enfeldt F F T T F
r: She would win the race
F F F F F
By the Law of Syllogism, if Catriona Le May Doan
skated her second 500 meters in 37.45 seconds, 42. p q r q q  r p  (q  r)
then she would win the race.
T T T F F T
32. Sample answer: Stacey assumed that the
conditional statement was true. T T F F F T
33. Sample answer: Doctors and nurses use charts to T F T T T T
assist in determining medications and their doses T F F T F T
for patients. Answers should include the F T T F F F
following. F T F F F F
• Doctors need to note a patient’s symptoms to
F F T T T T
determine which medication to prescribe, then
determine how much to prescribe based on F F F T F F
weight, age, severity of the illness, and so on.

35 Chapter 2
43. HDC is a right angle and HDC and HDF are 52.
a linear pair so HDF is a right angle. Thus, M
HDG is complementary to FDG because H
F G
mHDG  mFDG  90
44. Sample answer: KHJ and DHG
45. Sample answer: JHK and DHK
53.
46. Congruent, adjacent, supplementary, linear pair W r
47. Yes, slashes on the segments indicate that they
are congruent. s
48. 54. Q
B y
P
R
55.
C A A
n
B
x
O
(AB)2  (AC)2  (BC)2
(AB)2 ¬32  42  25
AB ¬5 M
56. Sample answer: AC M, CNB N, AM  CM,
49. y CN  BN, M is midpoint of A
C
, N is midpoint
C
of B.
D
57. Sample answer: 1 and 2 are complementary,
m1  m2  90.
58. Sample answer: 4 and 5 are supplementary,
m4  m5  180, 5 and 6 are supplementary,
x
m5  m6  180, 4  6, m4  m6.
O

C E
(CD)2  (CE)2  (DE)2 Page 88 Geometry Activity: Matrix Logic
(CD)2  62  82  100 1.
Job Nate John Nick
CD  10
50. y Veterinarian’s office ✓ ✗ ✗

Computer store ✗ ✗ ✓

Restaurant ✗ ✓ ✗
F
Nate works at the veterinarian’s office, John
works at the restaurant, and Nick works at the
x computer store.
O G H 2. Apartment Anita Kelli Scott Eric Ava Roberto
(FG)2  (GH)2  (FH)2 A ✗ ✗ ✗ ✗ ✗ ✓
(FG)2  62  42  52 B ✗ ✗ ✗ ✓ ✗ ✗
FG  52  7.2
C ✓ ✗ ✗ ✗ ✗ ✗
51. y
D ✗ ✗ ✗ ✗ ✓ ✗
N E ✗ ✗ ✓ ✗ ✗ ✗
F ✗ ✓ ✗ ✗ ✗ ✗
Roberto lives in A, Eric lives in B, Anita lives in C,
Ava lives in D, Scott lives in E, and Kelli lives in F.

x
M O P
(MN)2  (MP)2  (NP)2
(MN)2  92  72  130
MN  130
  11.4

Chapter 2 36
10. Given: P is the midpoint of QR and ST, and
Postulates and Paragraph QR  ST.
Proofs Prove: PQ  PT
Q T
P
Page 91 Check for Understanding
1. Deductive reasoning is used to support claims
that are made in a proof. S R
2. Proof: Since P is the midpoint of QR and ST
,
PQ  PR  1 2
 QR and PS = PT = 1 ST by the
2
definition of midpoint. We are given QR
S T
 so
QR  ST by the definition of congruent segments.
By the Multiplication Property, 1
 1
2 QR = 2 ST. So,
by substitution, PQ = PT.
11. Explore: There are six students, and each
student is connected to five other students with
ribbons.
3. postulates, theorems, algebraic properties,
Plan: Draw a diagram to illustrate the solution.
definitions
4. Explore: There are four points, and each pair is A B
to be connected by a segment.
Plan: Draw a diagram to illustrate the solution.
A B F C

E D
Solve: Let noncollinear points A, B, C, D, E, and
D C F represent the six students. Connect each point
Solve: Connect each point with every other point. with every other point. Then, count the number of
Then, count the number of segments. Between segments. Between every two points there is
every two points there is exactly one segment. For exactly one segment. For the 6 points, 15
the four points, six segments can be drawn. segments can be drawn.
Examine: The six segments that can be drawn Examine: In the figure, AB
, AC
, A
D
, A
E, A
F
, B
C
,
B
are A , A
C
, A
D
, BC, B
D
, and C
D
. D
B , B
E
, BF
, C
D
, C
E
, C
F
, D
E
, DF
, and E
F each
5. Explore: There are six points, and each pair is to represent a ribbon between two students. There
be connected by a segment. are 15 segments, so 15 ribbons are needed.
Plan: Draw a diagram to illustrate the solution 12. Explore: There are four points, and each pair is
B to be connected by a segment.
A Plan: Draw a diagram.
C
A
F
D
E
Solve: Connect each point with every other point.
Then, count the number of segments. Between
every two points there is exactly one segment. For
B C D
the 6 points, 15 segments can be drawn.
Solve: Connect each point with every other point.
Examine: The 15 segments that can be drawn
Then, count the number of segments. Between
are AB
, A
C
, A
D, A
E, A
F
, B
C
, B
D
, BE
, B
F, C
D, C
E
,
every two points there is exactly one segment. For
F
C , D
E
, D
F, and EF
.
the four points, six segments can be drawn.
6. Sometimes; if the planes have a common Examine: The six segments that can be drawn
intersection, then their intersection is one line. B
are A , A
C
, A
D
, BC, B
D
, and C
D
.
7. Definition of collinear
8. Through any three points not on the same line,
there is exactly one plane.
9. Through any two points, there is exactly one line.

37 Chapter 2
13. Explore: There are five points, and each pair is 21. Sometimes;  and m could be skew, so they would
to be connected by a segment. not lie in the same plane.
Plan: Draw a diagram. 22. Postulate 2.1: Through any two points, there is
A B exactly one line.
23. Postulate 2.5: If two points lie in a plane, then the
entire line containing those points lies in that plane.
24. Postulate 2.2: Through any three points not on
the same line, there is exactly one plane.
C D E 25. Postulate 2.5: If two points lie in a plane, then the
Solve: Connect each point with every other point. entire line containing those points lies in the plane.
Then, count the number of segments. Between 26. Postulate 2.1: Through any two points, there is
every two points there is exactly one segment. For exactly one line.
the five points, ten segments can be drawn. 27. Postulate 2.2: Through any three points not on
Examine: The ten segments that can be drawn the same line, there is exactly one plane.
are 
AB, 
A
C, 
AD, 
AE, 
B
C, 
B
D, 
B
E, 
C
D, 
C
E, and 
D
E.
28. Given: C is the midpoint of A B.
14. Explore: There are six points, and each pair is to D
B is the midpoint of C .
be connected by a segment. Prove: A CB D
Plan: Draw a diagram.
B A C B D
Proof: We are given that C is the midpoint of A B,
and B is the midpoint of C D. By the definition of
midpoint A C
C B
 and C B
B D. Using the
A C definition of congruent segments, AC  CB, and
F CB  BD. AC  BD by the Transitive Property of
Equality. Thus, A C
B D
 by the definition of
congruent segments.
E D
Solve: Connect each point with every other point. 29. There are 4 points, call them A, B, C, and D. Then
Then, count the number of segments. Between there is exactly one line between each pair of
every two points there is exactly one segment. For points, so there are 6 lines: 
AB, 
AC, 
AD, 
BC, 
BD,
the 6 points, 15 segments can be drawn. and CD. The points are noncollinear and
Examine: The 15 segments that can be drawn noncoplanar, and through any three points not on
are AB
, A
C
, A
D, A
E
, AF
, B
C
, B
D
, B
E
, B
F
, C
D
, C
E
, the same line there is exactly one plane. So there
F
C , D
E
, D
F, and E
F. are 4 different planes: plane ABC, plane ACD,
15. Explore: There are seven points, and each pair is plane BCD, and plane ABD.
to be connected by a segment. 30. Sample answer: Lawyers make final arguments,
Plan: Draw a diagram which is a speech that uses deductive reasoning,
A B in court cases.
31. It’s possible that all five points lie in one plane.
The points are noncollinear, and through any
three points not on the same line there is exactly
C E one plane. If the five points are points A, B, C, D,
D
and E, then there are as many as 10 planes
defined by these points: plane ABC, plane ABD,
F G plane ABE, plane ACD, plane ACE, plane ADE,
Solve: Connect each point with every other point. plane BCD, plane BCE, plane BDE, and plane CDE.
Then, count the number of segments. Between 32. Sample answer: The forms and structures of
every two points there is exactly one segment. For different types of writing are accepted as true,
the 7 points, 21 segments can be drawn. such as the structure of a poem. Answers should
Examine: The 21 segments that can be drawn include the following.
are AB
, A
C, A
D
, A
E, A
F
, A
G, B
C, B
D
, B
E, B
F
, B
G, • The Declaration of Independence, “We hold
D
C , C
E, C
F
, C
G, D
E
, D
F, D
G, E
F, E
G
, and FG
. these truths to be self-evident, ...”
16. Sometimes; the three points cannot be on the • Through any two points, there is exactly one line.
same line. 33. C; A is true because Postulate 2.2 states that
17. Always; if two points lie in a plane, then the entire through any 3 points not on the same line, there
line containing those points lies in that plane. is exactly one plane.
18. Never; the intersection of a line and a plane can be B is true because Postulate 2.6 states that if 2
a point, but the intersection of two planes is a line. lines intersect, then their intersection is one point.
19. Sometimes; the three points cannot be on the D is true by the Midpoint Theorem.
same line. C is not true because it contradicts Postulate 2.1
which states that through any two points, there is
20. Always; one plane contains at least three points,
exactly one line.
so it must contain two.

Chapter 2 38
34. A; 44. 3y  57
(8x4  2x2  3x  5)  (2x4  x3  3x  5) y  19
 8x4  2x2  3x  5  2x4  x3  3x  5 y
45.   12 ¬14
 6x4  x3  2x2  10 6
y
 ¬2
6
y ¬12
Page 93 Maintain Your Skills 46. t  3 ¬27
35. p: one has a part-time job t ¬24
q: one must work 20 hours per week t ¬24
Statement (3) is a valid conclusion by the Law of 47. 8n  39 ¬41
Detachment. 8n ¬80
36. Converse: If you have a computer, then you have n ¬10
access to the Internet at your house. False; you 48. 6x  33 ¬0
can have a computer and not have access to the 6x ¬33
Internet. 11
Inverse: If you do not have access to the Internet x ¬
2
at your house, then you do not have a computer.
False; it is possible to not have access to the
Internet and still have a computer.
Contrapositive: If you do not have a computer, 2-6 Algebraic Proof
then you do not have access to the Internet at
your house. False; you could have Internet access
through your television or wireless phone.
Page 97 Check for Understanding
1. Sample answer: If x  2 and x  y  6, then
37. Converse: If ABC has an angle measure greater
2  y  6.
than 90, then ABC is a right triangle. False; the
triangle would be obtuse. 2. given and prove statements and two columns, one
Inverse: If ABC is not a right triangle, none of of statements and one of reasons.
its angle measures are greater than 90. False; it 3. hypothesis; conclusion
could be an obtuse triangle. 4. Division Property
Contrapositive: If ABC does not have an angle 5. Multiplication Property
measure greater than 90, ABC is not a right 6. Substitution
triangle. False; mABC could still be 90 and
7. Addition Property
ABC be a right triangle.
38. 8. Given: 2x  4x  7  11
Animal Prove: x  4
Arthropod Proof:
Butterfly Statements Reasons
1. 2x  4x  7  11 1. Given

d  (x
x 2 1)  (y2  2. 22x  4x  7  2(11)
39. 2 y1)2 2. Mult. Prop.
DF  (4
)  312  (  3)2
3. x  8x  14  22 3. Distributive Prop.
 1  (
2 4)  17
2 
 4.1 4. 9x  14  22 4. Substitution
40. d  (xx 2 1) 
2 (y2 y1)2 5. 9x  36 5. Add. Prop.
MN  (5
  0) )
2 (5  22
6. x  4 6. Div. Prop.
 (5)
 2  3  34
2 
 5.8 9. Given: 5  2
3x  1

41. d  (xx 2 1) 
2 (y2 y1)2 Prove: x  6
PQ  [1
8)]
 ( 2  (3 2)2 Proof:
 95)
2  (  106
2 
Statements Reasons
 10.3
42. d  (xx a. ? 5  2
3x  1
 a. Given
2 1)  (y2 
2 y1)2
RS  [2
5)]
 ( 2  (1 12)2
b. 35  2
3 x  3(1)
 b. ? Mult. Prop.
 711)
2  (  170 2 
 13.0 c. 15  2x  3 c. ? Dist. Prop.
43. m  17  8
d. ? 2x  12 d. Subt. Prop.
m  25
e. x  6 e. ? Div. Prop.

39 Chapter 2
3x  5
10. Given: 25  7( y  3)  5y 24. Given:  2  7
Prove: 2  y Prove: x  3
Proof: Proof:
Statements Reasons
Statements Reasons
1. 25  7( y  3)  5y 1. Given 3x  5
a.  2  7 a. ? Given
2. 25  7y  21  5y 2. Dist. Prop.
3x 
3. 25  2y  21 3. Substitution b. ? 
2 
2
5

 2(7) b. Mult. Prop.

4. 4  2y 4. Subt. Prop. c. 3x  5  14 c. ? Substitution


5. 2  y 5. Div. Prop. d. 3x  9 d. ? Subt. Prop.

11. Given: Rectangle ABCD, AD  3, AB  10 e. ? x3 e. Div. Prop.


Prove: AC  BD
10 25. Given: 2x  7  1
3x  2

A B
3 3 Prove: x  3
Proof:
D C
10
Statements Reasons
Proof:
a. ? 2x  7  1
3x  2
 a. Given
Statements Reasons
1. Rectangle ABCD, AD  3, 1. Given b. ? 
3(2x  7)  3 1
3x  2
  b. Mult. Prop.
AB  10
2. Draw segments AC and 2. Two points c. 6x  21  x  6 c. ? Dist. Prop.
DB. determine a line. d. ? 5x  21  6 d. Subt. Prop.
3. ABC and BCD are
3. Def. of rt.  e. 5x  15 e. ? Add. Prop.
right triangles.
4. AC  3
0
2  1,
2 f. ? x3 f. Div. Prop.
4. Pythag. Thm.
DB  3
0
2  12
26. Given: 4  1 7
2a  2  a

5. AC  BD 5. Substitution Prove: a  1
Proof:
12. Given: c2  a2  b2
Prove: a  c 2  b2
 Statements Reasons
Proof:
1. 4  1 7
2a  2  a
 1. Given
Statements Reasons
1. c2  a2  b2 1. Given 
2. 2 4  1  
7
2 a 2 2  a
  2. Mult. Prop.

2. c2  b2  a2 2. Subt. Prop. 3. 8  a  7  2a 3. Dist. Prop.


3. a2  c2  b2 3. Symmetric Prop.
4. 1  a  2a 4. Subt. Prop.
4. a
2  c
2  b2
 4. Square Root Prop.
5. 1  1a 5. Add. Prop.
5. a  c
2  b2
 5. Square Root Prop.
6. 1  a 6. Div. Prop.
13. C; 8  x  12
x ¬4 7. a  1 7. Symmetric Prop.
4  x ¬4  4 or 0
27. Given: 2y  3
2 8

13
Prove: y  4
Pages 97–99 Practice and Apply
Proof:
14. Transitive Property
15. Subtraction Property Statements Reasons
16. Substitution 1. 2y  3
2 8
 1. Given
17. Substitution
18. Division or Multiplication Property 
2. 2 2y  3 
2  2(8) 2. Mult. Prop.

19. Reflexive Property
3. 4y  3  16 3. Dist. Prop.
20. Distributive Property
21. Substitution 4. 4y  13 4. Subt. Prop.
22. Division or Multiplication Property 13
5. y    5. Div. Prop.
4
23. Transitive Property

Chapter 2 40
28. Given: 1
2m  9
 3. mXCA  mACB  3. Substitution
Prove: m  18 mYBA  mABC
Proof: 4. mXCA  mACB  4. Substitution
Statements Reasons mYBA  mACB
5. mXCA  mYBA 5. Subt. Prop.
1. 1
2m  9
 1. Given
32. Given: Ek  hf  W

2. 2 1
 
2 m  2(9)
2. Mult. Prop. Ek  W
Prove: f  h
3. m  18 3. Substitution Proof:
2z  1
29. Given: 5  
3 Statements Reasons
Prove: z  6 1. Ek  hf  W 1. Given
Proof:
2. Ek  W  hf 2. Subt. Prop.
Statements Reasons
Ek  W
3. h  f 3. Div. Prop.
1. 5  2
3z  1
 1. Given
Ek  W

2. 3 5  2
 
3 z  3(1) 2. Mult. Prop. 4. f  h 4. Symmetric Prop.

3. 15  2x  3 3. Dist. Prop. 33. Given: mACB  mDCE


Prove: mACB  mACG
4. 15  2x  15  3  15 4. Subt. Prop. mDCE  mECF
Proof:
5. 2x  12 5. Substitution
2
x 1
2 Statements Reasons
2  2
6.   6. Div. Prop.
1. mACB  mDCE 1. Given
7. x  6 7. Substitution
2. mACB  mECF 2. Def. of vert. 
30. Given: XZ  ZY, X 4x 1 Z mDCE  mACG
XZ  4x  1, and
3. mACB  mACG 3. Transitive Prop.
ZY  6x  13
Prove: x  7
6x 13 mDCE  mECF
Thus, all of the angle measures would be equal.
Proof: Y
34. Sample answer: Michael has a symmetric
Statements Reasons relationship of first cousin with Chris, Kevin,
1. XZ  ZY, XZ  4x  1, 1. Given Diane, Dierdre, and Steven. Diane, Dierdre, and
and ZY  6x  13 Steven have a symmetric and transitive
relationship of sibling. Any direct line from
2. 4x  1  6x  13 2. Substitution bottom to top has a transitive descendent
3. 4x  1  4x 3. Subt. Prop. relationship.
 6x  13  4x 35. See students’ work.
4. 1  2x  13 4. Substitution 36. Sample answer: Lawyers use evidence and
testimony as reasons for justifying statements
5. 1  13  2x  13  13 5. Add. Prop. and actions. All of the evidence and testimony is
linked together to prove a lawyer’s case similar to
6. 14  2x 6. Substitution
a proof in mathematics.
14 2
x
7. 2  2
  7. Div. Prop. Answers should include the following.
8. 7  x 8. Substitution • Evidence is used to verify facts from witnesses
or materials.
9. x  7 9. Symmetric • Postulates, theorems, definitions, and properties
can be used to justify statements made in
31. Given: mACB  mABC
mathematics.
Prove: mXCA  mYBA
Proof: A

X C B Y
Statements Reasons
1. mACB  mABC 1. Given
2. mXCA  mACB  180 2. Def. of supp. 
mYBA  mABC  180

41 Chapter 2
37. B; mP  mQ  mR ¬180 3. If two lines intersect, then their intersection is
mQ  mQ  2(mQ) ¬180 exactly one point.
4(mQ) ¬180 4. If two points lie in a plane, then the entire line
mQ ¬45 containing those points lies in that plane.
mP ¬mQ 5. Given: 2(n  3)  5  3(n  1)
¬45 Prove: n  2
38. B; 4  x  y  5 Proof:
x ¬y  9
Statements Reasons
1. 2(n  3)  5  3(n  1) 1. Given
Page 100 Maintain Your Skills
39. Let the four buildings be named A, B, C, and D. In 2. 2n  6  5  3n  3 2. Dist. Prop.
order to have exactly one sidewalk between each 3. 2n  1  3n  3 3. Substitution
building, there should be 6 sidewalks. If A B is the
sidewalk between buildings A and B, then the 6 4. 2n  1  2n  3n  3  2n 4. Subt. Prop.
sidewalks are AB, A
C, A
D
, BC
, B
D, and C D
.
5. 1  n  3 5. Substitution
40. Valid; since 24 is divisible by 6, the Law of
Detachment says it is divisible by 3. 6. 1  3  n  3  3 6. Add. Prop.
41. Invalid; 27  6  4.5, which is not an integer.
7. 2  n 7. Substitution
42. Valid; since 85 is not divisible by 3, the
contrapositive of the statement and the Law of 8. n  2 8. Symmetric
Detachment say that 85 is not divisible by 6. Prop.
43. Sample answer: If people are happy, then they
rarely correct their faults.
44. Sample answer: If you don’t know where you are 2-7 Proving Segment Relationships
going, then you will probably end up somewhere
else.
45. Sample answer: If a person is a champion, then Page 101 Geometry Software Investigation:
the person is afraid of losing. Adding Segment Measures
46. Sample answer: If we would have new knowledge, 1. See students’ work. The sum AB  BC should
then we must get a whole new world of questions. always equal AC.
47. The measurement is precise to within 1 
2 foot. So, 2. See students’ work. The sum AB  BC should
1
a measurement of 13 feet could be 122 feet to always equal AC.
131

2 feet. 3. See students’ work. The sum AB  BC should
48. The measurement is precise to within 0.05 meter. always equal AC.
So, a measurement of 5.9 meters could be 4. AB  BC  AC
5.85 meters to 5.95 meters. 5. no
49. The measurement is precise to within 0.5 inch.
So, a measurement of 74 inches could be
73.5 inches to 74.5 inches. Pages 103–104 Check for Understanding
50. The measurement is precise to within 1. Sample answer: The distance from Cleveland to
0.05 kilometer. So, a measurement of Chicago is the same as the distance from
3.1 kilometers could be 3.05 kilometers to Cleveland to Chicago.
3.15 kilometers.
B
2. Sample answer: If AX Y and X
YP Q
, then
51. JL ¬JK  KL B
AP Q
.
25 ¬14  KL P
11 ¬KL B
52. PS ¬PQ  QS A
51 ¬23  QS Y
28 ¬QS
53. WZ ¬WY  YZ X
Q
WZ ¬38  9
WZ ¬47 3. If A, B, and C are collinear and AB  BC  AC,
then B is between A and C.
4. Reflexive
Page 100 Practice Quiz 2 5. Symmetric
1. Invalid; not all real numbers are integers. 6. Subtraction
2. Through any three points not on the same line,
there is exactly one plane.

Chapter 2 42
Q
7. Given: P R
S, Q
S
S
T
 10. Given: 
ABC
D
 B
R S T
S
Prove: P R
T
 
Prove: C DA
B A
Q
C
D
P
Proof:
Proof:
Statements Reasons
Statements Reasons
B
1. AC
D
 1. Given
a. ? , ? Q
P R
S
, a. Given
S
QS T
 2. AB  CD 2. Def. of  segs.

b. PQ  RS, QS  ST b. ? Def. of  segments 3. CD  AB 3. Symmetric Prop.

c. PS  PQ  QS, c. ? Segment Addition D


4. CA
B
 4. Def. of  segs.
RT  RS  ST Post.
d. ? PQ  QS d. Addition Property 11. Since Aberdeen is in South Dakota while Helena,
 RS  ST Miles City, and Missoula are in Montana,
Aberdeen is at one end of the line segment along
e. ? PS  RT e. Substitution which the four cities lie. Miles City is closest to
S
f. P R
T
 f. ? Def. of  segments Aberdeen (473 miles), Helena is next closest to
Aberdeen (860 miles), and Missoula is farthest
8. Given: 
APC
P
 from Aberdeen (972 miles). Thus, Helena is
A P D
P
B D
P between Missoula and Miles City.
B
Prove: A C
D C B
Proof:

Statements Reasons
Pages 104–106 Practice and Apply
12. Symmetric
P
1. AC
P
 and B
P
D
P
 1. Given
13. Substitution
2. AP  CP and BP  DP 2. Def. of  segs. 14. Segment Addition
3. AP  PB  AB 3. Seg. Add. Post. 15. Transitive
4. CP  DP  AB 4. Substitution 16. Addition
5. CP  PD  CD 5. Seg. Add. Post. 17. Subtraction
18. Given: AD
C E , D
B
E
B
 B
6. AB  CD 6. Transitive Prop.
Prove: AB
  C
B

B
7. AC
D
 7. Def. of  segs.
D E
9. Given: 
HI  T
U
 H
and HJTV

Prove: IJ
UV I A C
Proof:

T U V Statements: Reasons:
J
Proof: a. ? D
AC
E
, D
B
E
B
 a. Given

Statements Reasons b. AD  CE, DB  EB b. ? Def. of  segs.


I  T
1. H U
 and H
J
T
V
 1. Given
c. AD  DB  CE  EB c. ? Add. Prop.
2. HI  TU and HJ  TV 2. Def. of  segs.
d. ? AB  AD  DB, d. Segment Addition
3. HI  IJ  HJ 3. Seg. Add. Post.
CB  CE  EB Postulate
4. TU  IJ  TV 4. Substitution
e. AB  CB e. ? Substitution
5. TU  UV  TV 5. Seg. Add. Post.
6. TU  IJ  TU  UV 6. Substitution B
f. A  C
B
 f. ? Def. of  segs.
7. TU  TU 7. Reflexive Prop.
8. IJ  UV 8. Subt. Prop.
9. IJ
U
V
 9. Def. of  segs.

43 Chapter 2
19. Given: 
X
YWZ and 
W
Z
A
B W d. WY  WA  AY, d. ? Segment
B

Prove: X YA
B
 ZX  ZA  AX Addition Post.
A
X e. WA  AY  ZA  AX e. ? Substitution
Y Z
Proof: f. WA  WA  ZA  ZA f. ? Substitution

Statements Reasons g. 2WA  2ZA g. ? Substitution

Y
1. XW
Z
 and W
Z
A
B
 1. Given h. ? WA  ZA h. Division Property

2. XY  WZ and WZ  AB 2. Def. of  segs. A


i. WZ
A
 i. ? Def. of  segs.

3. XY  AB 3. Transitive Prop. 22. Given: 


LMP N
 L P
M
and XX N
 M
X
Y
4. XA
B
 4. Def. of  segs. X
Prove: L PX N

Proof:
20. Given: 
A
B  A

C and 
P
C  Q

B C B
P
Prove: A A Q
 Statements Reasons
P Q
M
1. LP
N and X
M
X
N
 1. Given
A 2. LM  PN and XM  XN 2. Def. of  segs.
Proof:
3. LM  LX  XM, 3. Seg. Add. Post.
Statements Reasons
PN  PX  XN
B
1. A  A
C
 and P
C
Q
B
 1. Given 4. LX  XM  PX  XN 4. Substitution

2. AB  AC, PC  QB 2. Def. of  segs. 5. LX  XN  PX  XN 5. Substitution

3. AB  AQ  QB, 3. Seg. Add. Post. 6. XN  XN 6. Reflexive Prop.


AC  AP  PC
7. LX  PX 7. Subt. Prop.
4. AQ  QB  AP  PC 4. Substitution M
8. LP
X
 8. Def. of  segs.

5. AQ  QB  AP  QB 5. Substitution 23. Given: AB  BC


Prove: AC  2BC A B C
6. QB  QB 6. Reflexive Prop. Proof:

7. AP  AQ 7. Subt. Prop. Statements Reasons

P
8. AA
Q
 8. Def. of  segs. 1. AB  BC 1. Given

21. Given: 
WYZ X
 W 2. AC  AB  BC 2. Seg. Add. Post.
Y
A is the midpoint of W .
A is the midpoint of 
ZX. 3. AC  BC  BC 3. Substitution
A
Prove: W Z A
 Z X
A 4. AC  2BC 4. Substitution

24. Given: 
AB
Proof: Y 
Prove: A BA
B
 A B
Proof:
Statements Reasons
Statements Reasons
Y
a. W Z X
 a. ? Given B
1. A 1. Given
Y
A is the midpoint of W .
A is the midpoint of 
ZX. 2. AB  AB 2. Reflexive Prop.
B
3. AA
B
 3. Def. of  segs.
b. WY  ZX b. ? Def. of  segs.

c. ? WA  AY, ZA  AX c. Definition of
midpoint

Chapter 2 44
25. Given: 
ABD E AQ ¬1

2 AD
 
C is the midpoint
¬1
 1
2 39 4
D
of B.
C
Prove: A C E ¬195 
8
A B C D E 1
BP ¬2BC
Proof:

Statements Reasons
¬1 
2 12 4
3 
¬63 
B
1. AD E, C is the 1. Given 8
D
midpoint of B AP ¬AB  BP
2. BC  CD 2. Def. of midpoint ¬141
 3
4  68
¬205

3. AB  DE 3. Def. of  segs. 8
AP ¬AQ  QP
4. AB  BC  CD  DE 4. Add. Prop.
205 5
8 ¬19 8  QP

5. AB  BC  AC 5. Seg. Add. Post. 1 ¬QP
CD  DE  CE
30. Let x be the price of a box of popcorn. Then a tub
6. AC  CE 6. Substitution of popcorn costs 2x.
60(2x) ¬150
C
7. AC
E
 7. Def. of  segs.
120x ¬150
26. Given: 
AB  E
F and x ¬1.25
C
B   D
E
 Thus, a box of popcorn costs $1.25 and a tub costs
C
Prove: A D F 2(1.25) or $2.50.
C D Total popcorn sales were $275 and $150 of that
A F
B E was for tubs, so boxes account for $275  $150 or
125
$125. So the number of boxes sold was  1.2
5
or 100.
Proof:

Statements Reasons
Page 106 Maintain Your Skills
B
1. A  E
F and 1. Given 31. Substitution
C
B   D
E
 32. Distributive Property
2. AB  EF and BC  DE 2. Def. of  segs. 33. Addition Property
34. Transitive Property
3. AB  BC  DE  EF 3. Add. Prop.
35. Never; the midpoint of a segment divides it into
4. AC  AB  BC, 4. Seg. Add. Post. two congruent segments.
1DF  DE  EF 36. Sometimes; if the lines have a common
intersection point, then it is a single point.
5. AC  DF 5. Substitution
37. Always; if two planes intersect, they intersect in
C
6. AD
F
 6. Def. of  segs. a line.
38. Sometimes; if the points are noncollinear, then
27. Sample answers: L N
Q O and
they lie on three distinct lines.
M
L M NR SS TQ PP O
39. P ¬2  2w
28. Sample answer: You can use segment addition to
44 ¬2(2x  7)  2(x  6)
find the total distance between two destinations
44 ¬4x  14  2x  12
by adding the distances of various points in
44 ¬6x  26
between. Answers should include the following.
18 ¬6x
• A passenger can add the distance from San 3 ¬x
Diego to Phoenix and the distance from Phoenix 2x  7 ¬2(3)  7 or 13
to Dallas to find the distance from San Diego to x  6 ¬3  6 or 9
Dallas. The dimensions of the rectangle are 9 cm by
• The Segment Addition Postulate can be useful if 13 cm.
you are traveling in a straight line. 40. 2x  x ¬90
29. B; AD ¬AB  BC  CD 3x ¬90
¬141 3 1 x ¬30
4  12 4  12 4

41. 2x  4x ¬90
¬391

4 6x ¬90
x ¬15

45 Chapter 2
42. 3x  2  x ¬90 6. Given: 1 and 2 are supplementary, 3 and 4
4x  2 ¬90 are supplementary, 1  4
4x ¬88 Prove: 2  3
x ¬22
43. x  3x ¬180
4x ¬180 1 2 3 4
x ¬45 Proof:
44. 26x  10x ¬180
36x ¬180 Statements Reasons
x ¬5 a. 1 and 2 are a. ? Given
45. 4x  10  3x  5 ¬180 supplementary.
7x  5 ¬180 3 and 4 are
7x ¬175 supplementary.
x ¬25 1  4
b. m1  m2  180 b. ? Def. of supp.
m3  m4  180 
2-8 Proving Angle Relationships c. m1  m2  c. ? Substitution
m3  m4
Page 110 Geometry Activity: Right Angles d. m1  m4 d. ? Def. of  
1. The lines are perpendicular.
e. m2  m3 e. ? Subt. Prob.
2. They are congruent and they form linear pairs.
3. 90 f. 2  3 f. ? Def. of  
4. They form right angles.
7. Given: 
VX bisects WVY, W
5. They all measure 90° and are congruent. 
VY bisects XVZ.
Prove: WVX  YVZ X
V
Pages 111–112 Check for Understanding Y
1. Tomas; Jacob’s answer left out the part of ABC
represented by EBF. Z
2. Sample Answer: If 1  2 and 2  3, Proof:
then 1  3.
1 2 3 Statements Reasons

3. 1  2 because they are vertical angles.  bisects WVY.


1. VX 1. Given
m1 ¬m2  bisects XVZ.
VY
65 ¬m2
4. m6  m8 ¬90 2. WVX  XVY 2. Def. of bisector
m6  47 ¬90
m6 ¬43 3. XVY  YVZ 3. Def. of bisector
m6  m7 ¬m8 ¬180 4. WVX  YVZ 4. Trans. Prop.
43  m7  47 ¬180
m7  90 ¬180 8. sometimes
m7 ¬90 9. sometimes
5. m11  m12 ¬180 10. Given: Two angles form a linear pair.
x  4  2x  5 ¬180 Prove: The angles are supplementary.
3x  9 ¬180
3x ¬189
x ¬63 1 2
m11 ¬x  4 Paragraph Proof: When two angles form a
¬63  4 or 59 linear pair, the resulting angle is a straight angle
m12 ¬2x  5 whose measure is 180. By definition, two angles
¬2(63)  5 or 121 are supplementary if the sum of their measures is
180. By the Angle Addition Postulate,
m1  m2  180. Thus, if two angles form a
linear pair, then the angles are supplementary.

Chapter 2 46
11. Given: ABC is a right angle. C m11  4x
B
Prove: 1 and 2 are 2  4(31) or 124
1
complementary angles. m12  2x  6
A  2(31)  6 or 56
21. 13  14
Proof: m13 ¬m14
2x  94 ¬7x  49
Statements Reasons 2x  45 ¬7x
1. ABC is a right angle. 1. Given 45 ¬5x
9 ¬x
2. mABC  90 2. Def. of rt.  m13  2x  94
3. mABC  m1  m2 3. Angle Add. Post.  2(9)  94 or 112
4. m1  m2  90 4. Substitution m14  7x  49
 7(9)  49 or 112
5. 1 and 2 are 5. Def. of comp. 
complementary angles. 22. 15  16
m15 ¬m16
12. 1 and 2 are complementary to X, so x ¬6x  290
1  2. 5x ¬290
m1 ¬m2 x ¬58
2n  2 ¬n  32 m15  m16  58
2n ¬n  30 23. 17  18
n ¬30 m17 ¬m18
13. m1 ¬2n  2 2x  7 ¬x  30
¬2(30)  2 2x ¬x  23
¬62 x ¬23
14. m2  n  32 m17  2x  7
 30  32  2(23)  7
 62  53
15. mX  m1 ¬90 m18  x  30
mX  62 ¬90  23  30 or 53
mX ¬28 24. m19  m20 ¬180
100  20x  20x ¬180
100  40x ¬180
Pages 112–114 Practice and Apply 40x ¬80
16. m1  m2 ¬180 x ¬2
m1  67 ¬180 m19  100  20x
m1 ¬113  100  20(2) or 140
17. m3  m4 ¬90 m20  20x
38  m4 ¬90  20(2) or 40
m4 ¬52 25. Given: A
18. 7 and 8 are complementary, so Prove: A  A
m7  m8  90. Also,
A
m5  m6  m7  m8  180, so by Proof:
substitution m5  m6  90. m6  29, so
m5  61. 5  8 so m8  m5  61. Finally, Statements Reasons
m7  m8  90 so m7  90  61 or 29.
1. A is an angle. 1. Given
19. m9  m10 ¬180
2x  4  2x  4 ¬180 2. mA  mA 2. Reflexive Prop.
4x ¬180 3. A  A 3. Def. of  angles
x ¬45
m9 ¬2x  4
¬2(45)  4 or 86
m10 ¬2x  4
¬2(45)  4 or 94
20. m11  m12 ¬180
4x  2x  6 ¬180
6x  6 ¬180
6x ¬186
x ¬31

47 Chapter 2
26. Given: 1  2, 2  3 35. Given:   m
Prove: 1  3 Prove: 1  2
1 2 m
3 4
1 2 3
Proof:
Proof:
Statements Reasons
Statements Reasons
1. 1  2, 2  3 1. Given
1.   m 1. Given
2. m1  m2, 2. Def. of  angles
m2  m3 2. 1 and 2 are rt.  2.  lines intersect
to form 4 rt. .
3. m1  m3 3. Trans. Prop.
3. 1  2 3. All rt.  are .
4. 1  3 4. Def. of  angles
27. sometimes
36. Given: 1  2, 1 and 2
28. always are supplementary.
29. always Prove: 1 and 2 are rt. . 1 2
30. sometimes Proof:
31. sometimes
Statements Reasons
32. always
33. Given:   m 1. 1  2, 1 and 2 1. Given
Prove: 2, 3, 4 are rt.  are supplementary.
1 2 m 2. m1  m2  180 2. Def. of supp. 
3 4
3. m1  m2 3. Def. of  angle
Proof: 4. m1  m1  180 4. Substitution
Statements Reasons
5. 2(m1)  180 5. Add. Prop.
11.   m 11. Given
12. 1 is a right angle. 12. Def. of  6. m1  90 6. Div. Prop.

13. m1  90 13. Def. of rt.  7. m2  90 7. Substitution


(steps 3, 6)
14. 1  4 14. Vert.  are 
15. m1  m4 15. Def. of   8. 1 and 2 are rt. . 8. Def. of rt. 
16. m4  90 16. Substitution
37. Given: ABD  CBD, ABD and CBD
17. 1 and 2 form a linear 17. Def. of linear form a linear pair
pair. pair Prove: ABD and CBD are rt. .
3 and 4 form a linear
pair. D
18. m1  m2  180, 18. Linear pairs are
m4  m3  180 supplementary. A B C
19. 90  m2  180, 19. Substitution Proof:
, \90  m3  180
10. m2  90, m3  90 10. Subt. Prop. Statements Reasons
11. 2, 3, 4, are rt. . 11. Def. of rt.  1. ABD  CBD, ABD 1. Given
(steps 6, 10) and CBD form a
linear pair.
34. Given: 1 and 2 are rt. .
Prove: 1  2 2. ABD and CBD are 2. Linear pairs are
1 2 supplementary. supplementary.
Proof: 3. ABD and CBD are 3. If  are  and supp.,
rt. . they are rt. .
Statements Reasons

1. 1 and 2 are rt. . 1. Given


2. m1  90, m2  90 2. Def. of rt. 
3. m1  m  2 3. Substitution
4. 1  2 4. Def. of  angles

Chapter 2 48
38. Given: ABD  YXZ 42. m1  m4 ¬90;
Prove: CBD  WXZ m1  m2  m3  m4 ¬180
W X Y m1  m1  m4  m4 ¬180
2(m1)  2(m4) ¬180
D 2(m1  m4) ¬180
Z
m1  m4 ¬90
43. Two angles that are supplementary to the same
A B C
angle are congruent. Answers should include the
Proof: following.
Statements Reasons • 1 and 2 are supplementary; 2 and 3 are
supplementary.
1. ABD  YXZ, ABD and 1. Given; from • 1 and 3 are vertical angles, and are
CBD form a linear pair. the figure therefore congruent.
YXZ and WXZ form a • If two angles are complementary to the same
linear pair. angle, then the angles are congruent.
2. mABD  mCBD  180, 2. Linear pairs 44. B; Let x be the measure of one angle. Then the
mYXZ  mWXZ  180 are supple- measure of the other angle is 90  x.
mentary. x 4

90  x ¬1
3. mABD  mCBD 3. Substitution x ¬4(90  x)
 mYXZ  mWXZ x ¬360  4x
5x ¬360
4. mABD  mYXZ 4. Def. of   x ¬72
The other angle has mesure 90  72 or 18.
5. mYXZ  mCBD 5. Substitution
 mYXZ  mWXZ 45. B; The members of set T are 1, 4, 9, 16, 25, 36,
and 49. The median of these numbers is 16.
6. mYXZ  mYXZ 6. Reflexive
Prop.
7. mCBD  mWXZ 7. Subt. Prop. Page 114 Maintain Your Skills
8. CBD  WXZ 8. Def. of   46. Given: G is between F and H.
H is between G and J.
39. Given: mRSW  mTSU Prove: FG  GJ  FH  HJ
Prove: mRST  mWSU
J
G H
T F
R
W Proof:

U Statements Reasons
S
1. G is between F and H; 1. Given
Proof: H is between G and J.
Statements Reasons 2. FG  GJ  FJ, 2. Segment Addition
FH  HJ  FJ Postulate
1. mRSW  mTSU 1. Given
3. FJ  FH  HJ 3. Symmetric Prop.
2. mRSW  mRST  2. Angle Addition
4. FG  GJ  FH  HJ 4. Transitive Prop.
mTSW, mTSU Postulate
 mTSW  mWSU Y
47. Given: X is the midpoint of W.
3. mRST  mTSW 3. Substitution Prove: WX  YZ  XZ
 mTSW  mWSU
4. mTSW  mTSW 4. Reflexive Prop. W X Y
5. mRST  mWSU 5. Subt. Prop. Z
Proof:
40. m1  m2 ¬180
28  m2 ¬180
m2 ¬152 Statements Reasons
41. Because the lines are perpendicular, the angles Y
1. X is the midpoint of W. 1. Given
formed are right angles. All right angles are
2. WX  XY 2. Def. of midpoint
congruent. Therefore, 1 is congruent to 2.
3. XY  YZ  XZ 3. Segment Addition
Postulate
4. WX  YZ  XZ 4. Substitution

49 Chapter 2
48. Given: AC  BD 14. The sum of the measures of two supplementary
A B C D angles is 180 and 1  0.; false, because r is true
Prove: AB  CD
Proof: and p is false.
15. 1  0, and in a right triangle with right angle C,
Statements Reasons a2  b2  c2, or the sum of the measures of two
1. AC  BD 1. Given supplementary angles is 180.; false, because q is
true and r is true so q r is true but p is false.
2. AB  BC  AC, 2. Segment Addition
16. In a right triangle with right angle C,
BC  CD  BD Postulate
a2  b2  c2, or 1  0 or the sum of the
3. BC  BC 3. Reflexive Prop. measures of two supplementary angles is 180.;
4. AB  BC  BC  CD 4. Substitution (2 and 3) true, because p is false and r is true so p r is
true but q is true.
5. AB  CD 5. Subt. Prop.
17. In a right triangle with right angle C,
49. ONM, MNR a2  b2  c2 and the sum of the measures of two
supplementary angles is 180, and 1  0.; false,
50. PMQ  QMN
because q is true and r is true so q  r is true but
51. N or R p is false.
52. POQ, QON, NOM, MOP 18. Converse: If an angle is obtuse, then it measures
53. obtuse 120. False; the measure could be any value
54. Sample answer: 
NR and 
NP between 90 and 180.
55. NML, NMP, NMO, RNM, ONM Inverse: If an angle measure does not equal 120,
then it is not obtuse. False; the measure could be
any value other than 120 between 90 and 180.
Contrapositive: If an angle is not obtuse, then its
Chapter 2 Study Guide and Review measure does not equal 120; true.
19. Converse: If a month has 31 days, then it is
March. False; July has 31 days.
Page 115 Vocabulary and Concept Check Inverse: If a month is not March, then it does not
1. conjecture have 31 days. False; July has 31 days.
2. truth value Contrapositive: If a month does not have 31 days,
3. compound then it is not March; true.
4. and 20. Converse: If a point lies on the y-axis, then its
ordered pair has 0 for its x-coordinate; true.
5. hypothesis
Inverse: If an ordered pair does not have 0 for its
6. converse x-coordinate, then the point does not lie on the
7. Postulates y-axis; true.
8. informal proof Contrapositive: If a point does not lie on the
y-axis, then its ordered pair does not have 0 for its
x-coordinate; true.
Pages 115–120 Lesson-by-Lesson Review 21. true, because the hypothesis is satisfied and the
9. mA  mB  180 conclusion follows
22. true, because the hypothesis is not satisfied and
we cannot say the statement is false
45 135 23. false, because the hypothesis is satisfied yet the
A B conclusion does not follow
24. true, because the hypothesis is not satisfied and
Z
10. Y is the midpoint of X. we cannot say the statement is false
X Y Z 25. Valid; by definition, adjacent angles have a
common vertex.
11. LMNO is a square.
26. Invalid; vertical angles also have a common
M N vertex.
27. yes; Law of Detachment
p: a student attends North High School
L O q: a student has an ID number
28. Invalid; Statements (1) and (2) are true, but (3)
12. 1  0 and in a right triangle with right angle C, does not follow from (1) and (2).
a2  b2  c2.; false, because p is false and q is true.
29. yes; Law of Syllogism
13. In a right triangle with right angle C, p: you like pizza with everything
a2  b2  c2 or the sum of the measures of two q: you like Cardo’s Pizza
supplementary angles is 180.; true, because q is r: you are a pizza connoisseur
true and r is true.

Chapter 2 50
30. Never; the intersection of two lines is a point. 45. Given: AC  AB, AC  4x  1, AB  6x  13
31. Always; if P is the midpoint of XY
, then X
PPY. Prove: x  7
By definition of congruent segments, XP  PY. A 6x 13 B
32. sometimes; if M, X, and Y are collinear
33. sometimes; if the points are collinear 4x 1 C
34. Always; there is exactly one line through Q and R. Proof:
The line lies in at least one plane.
35. sometimes; if the right angles form a linear pair Statements Reasons
36. Always; the Reflexive Property states that 1  1. 1. AC  AB, AC  4x  1,
1. Given
37. Never; adjacent angles must share a common AB  6x  13
side, and vertical angles do not. 2. 4x  1  6x  13 2. Substitution
38. Given: M is the midpoint of A B
 and Q is the
3. 4x  1  1  6x  13  1 3. Subt. Prop.
midpoint of A M
.
Prove: AQ  1 
4 AB 4. 4x  6x  14 4. Substitution
A Q M B 5. 4x  6x  6x  14  6x 5. Subt. Prop.
B
Proof: If M is the midpoint of A, then
6. 2x  14 6. Substitution
AM  1
 M
2 (AB). Since Q is the midpoint of A,
2
x 1
4
2  2
7.   7. Div. Prop.
AQ  1 1 1 1
2 AM or 2 2 (AB)  4 AB.
 
8. x  7 8. Substitution
39. Distributive Property
40. Division Property
41. Subtraction Property 46. Given: MN  PQ, PQ  RS
42. Transitive Property Prove: MN  RS
43. Given: 5  2  1

2x
M N R
Prove: x  6
Proof: P Q
S
Statements Reasons Proof:

1. 5  2  1

2x 1. Given Statements Reasons
1. MN  PQ, PQ  RS 1. Given
2. 5  2  2  1
2x  2
 2. Subt. Prop.
2. MN  RS 2. Transitive Prop.
3. 3  1

2x 3. Substitution

4. 2(3)  2 1

2x  4. Mult. Prop. 47. Reflexive Property
48. Symmetric Property
5. 6  x 5. Substitution
49. Addition Property
6. x  6 6. Symmetric Prop. 50. Transitive Property
x  10 51. Division or Multiplication Property
44. Given: x  1   2
52. Addition Property
Prove: x  4
Proof: 53. Given: BC  EC, CA  CD B E
Prove: BA  DE
Statements Reasons
C
x 
10
1. x  1   2 1. Given
D
x 
2. 2(x  1)  2  10
2   2. Mult. Prop.
Proof:
A

3. 2x  2  x  10 3. Dist. Prop. Reasons


Statements
4. 2x  2  2  x  10  2 4. Subt. Prop. 1. BC  EC, CA  CD 1. Given
5. 2x  x  12 5. Substitution 2. BC  CA  EC  CA 2. Add. Prop.

6. 2x  x  x  12  x 6. Subt. Prop. 3. BC  CA  EC  CD 3. Substitution


4. BC  CA  BA 4. Seg. Add. Post.
7. 3x  12 7. Substitution EC  CD  DE
3
x 1
2 5. BA  DE 5. Substitution
3  3
8.   8. Div. Prop.

9. x  4 9. Substitution

51 Chapter 2
54. Given: AB  CD 9. 3  2, or 3x  12 when x  4 and an equilateral
Prove: AC  BD triangle is also equiangular.; true, because
q is true and r is true so q  r is true and
A B C D p is false
Proof: 10. Hypothesis: you eat an apple a day; Conclusion:
the doctor will stay away; If you eat an apple a
Statements Reasons
day, then the doctor will stay away.
1. AB  CD 1. Given Converse: If the doctor stays away, then you eat
2. BC  BC 2. Reflexive Prop. an apple a day.
Inverse: If you do not eat an apple a day, then the
3. AB  BC  CD  BC 3. Add. Prop.
doctor will not stay away.
4. AB  BC  AC 4. Seg. Add. Post. Contrapositive: If the doctor does not stay away,
CD  BC  BD then you do not eat an apple a day.
5. AC  BD 5. Substitution 11. Hypothesis: a stone is rolling; Conclusion: it
gathers no moss; If a stone is rolling, then it
55. m6  180  35 or 145 gathers no moss.
56. m7  180  157 or 23 Converse: If a stone gathers no moss, then it is
57. m8  180  90 or 90 rolling.
58. Given: 1 and 2 form a linear Inverse: If a stone is not rolling, then it gathers
pair. m2  2(m1) moss.
1 2
Prove: m1  60 Contrapositive: If a stone gathers moss, then it is
Proof: not rolling.
12. valid; Law of Detachment
Statements Reasons p: two lines are perpendicular
a. 1 and 2 form a linear q: the lines intersect
a. ? Given
pair. 13. m1  73  95
b. 1 and 2 are b. ? Supplement m1  22
supplementary. Theorem 14. m2  180  (m1  73)
c. ? m1  m2 c. Definition of  180  (22  73)
 180 supplementary  85
angles 15. m3  m2
 85
d. m2  2(m1) d. ? Given
16. Given: y  4x  9; x  2
e. ? m1  2(m1) e. Substitution Prove: y  17
 180 Proof:
f. ? 3(m1)  180 f. Substitution Statements Reasons
3(m1) 18
0 1. y  4x  9; x  2 1. Given
g. 3   3 g. ? Division
Property 2. y  4(2)  9 2. Substitution
h. ? m1  60 h. Substitution 3. y  8  9 3. Substitution

4. y  17 4. Substitution
Chapter 2 Practice Test
17. Given: AM  CN, MB  ND A M B
Prove: AB  CD
Page 121
1. Sample answer: Formal is the two-column proof,
informal can be paragraph proofs. D N C
2. Sample answer: You can use a counterexample. Proof:
3. Sample answer: statements and reasons to justify We are given that AM  CN, MB  ND. By the
statements Addition Property, AM  MB  CN  MB. Then
by Substitution, AM  MB  CN  ND. Using the
4. true; Symmetric Prop.
Segment Addition Postulate, AB  AM  MB, and
5. false; y  2 CD  CN  ND. Then, by Substitution AB  CD.
6. false; a  4 18. Hypothesis: you are a hard-working person;
7. 3  2 and 3x  12 when x  4.; false, because Conclusion: you deserve a great vacation; If you
p is false and q is true are a hard-working person, then you deserve a
8. 3  2 or 3x  12 when x  4.; true, because great vacation.
p is false and q is true 19. A

Chapter 2 52
Chapter 2 Standardized Test Practice 14a. Possible lengths and Perimeter of
widths where area is rectangle with given
100 sq ft length and width
Pages 122–123 1 ft by 100 ft 202 ft
1. D; 49 0.143 2.646 49
2. C; 2(7)  3  11, so (7, 11) is on the line. 2 ft by 50 ft 104 ft
2(4)  3  5, so (4, 5) is on the line. 4 ft by 25 ft 58 ft
2(2)  3  7, so (2, 10) is not on the line.
2(5)  3  13, so (5, 13) is on the line. 5 ft by 20 ft 50 ft
3. A; a protractor is used to measure angles, not 10 ft by 10 ft 40 ft
lengths. A calculator is not a measuring tool. A
centimeter ruler is more accurate than a The dimensions that require the least amount of
yardstick because its unit of measurement fencing are 10 ft by 10 ft.
(centimeters) is smaller. 14b. Sample answer: Make a list of all possible whole-
E
4. B; D E F so DE  EF. number lengths and widths that will form a
8x  3 ¬3x  7 100-square-foot area. Then find the perimeter of
8x ¬3x  10 each rectangle. Choose the length and width
5x ¬10 combination that has the smallest perimeter.
x ¬2 14c. As the length and width get closer to having the
5. A; ACF  DCF  ACD by the Angle Addition same measure as one another, the amount of
Postulate. So mACF  mDCF  90 since fencing required decreases.
ACD is a right angle. Then ACF and DCF 15. Given: 1 and 3 are vertical angles.
are complementary angles. m1  3x  5, m3  2x  8
6. C; inductive reasoning uses specific examples to Prove: m1  14
make a conjecture.
2 3
7. A 1 4
8. A; divide both sides by 3.
9. The shortest distance is the length of the Proof:
hypotenuse of the right triangle whose legs have
Statements Reasons
lengths 120 yd and 531 
3 yd. Use the Pythagorean a. 1 and 3 are vertical
Theorem to find the length of the hypotenuse.
angles. m1  3x  5, a. Given
Call this length d.
2 m3  2x  8

d2  (120)2  531

3  b. 1  3 b. Vert.  are 
25,600
d2  14,400   
9 c. m1  m3 c. Def. of  
d ¬17,24
4.4  d. 3x  5  2x  8 d. Substitution
d ¬131 yd
10. inverse e. x  5  8 e. Subt. Prop.
11. (p → q)  (q → r) → (p → r) f. x  3 f. Subt. Prop.
Martina drank 300 mg of calcium.
g. m1  3(3)  5 g. Substitution
12. Segment Addition Postulate
13. Sample answer: Marti can measure a third h. m1  14 h. Substitution
distance c, the distance between the ends of the
two sides, and make sure it satisfies the equation
a2  b2  c2.

53 Chapter 2
Chapter 3 Parallel and Perpendicular Lines
Page 125 Getting Started 2. Juanita; Eric has listed interior angles, but they
1. 
PQ are not alternate interior angles.
2. 
PR or 
RS 3. Sample answer: looking down railroad tracks
3. 
ST 4. ABC, JKL, ABK, CDM
4.  
TR or TP B
5. A , J
K, L
M
5. The arcs in the figure indicate that 2 is K
6. B , C
L, J
K
, L
M
, BL
, K
M
congruent to 4, 6, and 8. 7. q and r, q and t, r and t
6. The arcs in the figure indicate that 5 is 8. p and q, p and t, q and t
congruent to 1, 3, and 7. 9. p and r, p and t, r and t
7. The arcs in the figure indicate that 3 is 10. p and q, p and r, q and r
congruent to 1, 5, and 7. 11. alternate interior
8. The arcs in the figure indicate that 8 is 12. corresponding
congruent to 2, 4, and 6.
13. consecutive interior
9. y  7x  12
14. alternate exterior
 7(3)  12
 21  12  9 15. p; consecutive interior
2 16. p; alternate exterior
10. y ¬3x  4
17. q; alternate interior
2
¬3(8)  4 18. Sample answer: The pillars form parallel lines.
1612
¬3   4
3  3
 19. Sample answer: The roof and the floor are parallel
planes.
11. 2x  4y ¬18
2(6)  4y ¬18 20. Sample answer: One of the west pillars and the
12  4y ¬18 base on the east side form skew lines.
4y ¬6 21. Sample answer: The top of the memorial “cuts”
y ¬6
4 or  2
 3 the pillars.

Pages 129–131 Practice and Apply


3-1 Parallel Lines and Transversals 22. 
DE, P
Q, ST
23. ABC, ABQ, PQR, CDS, APU, DET
Page 126 Geometry Activity: Draw a 24. C
B , E
 F
 , Q
R

Rectangular Prism 25. P
A , B
Q, CR, F
U
, PU
, Q
R
, RS, T
U

1. Planes ABC and EFG are parallel because those 26. ABC, AFU, BCR, CDS, EFU, PQR
are the planes given as parallel. Planes BCG and 27. C
B , C
D, DE, E
F, Q
R
, R
S, S
T
, TU

ADH are parallel, as well as planes ABF and 28. b and c, b and r, r and c
DCG because opposite sides of a rectangular
29. a and c, a and r, r and c
prism are parallel the same way opposite sides of
a rectangle are parallel. 30. a and b, a and r, b and r
2. Plane ABF intersects plane ABC at  ; plane
AB 31. a and b, a and c, b and c
DCG intersects plane ABC at DC ; plane ADH 32. corresponding
intersects plane ABC at AD; plane BCG intersects 33. alternate exterior
plane ABC at 
BC. 34. alternate interior
E
3. A , C
G
, and DH
 are parallel to BF
 because 35. corresponding
planes BCG and ADH are parallel and planes 36. alternate exterior
ABF and DCG are parallel.
37. alternate interior
38. consecutive interior
Pages 128–129 Check for Understanding 39. consecutive interior
1. Sample answer: The bottom and top of a cylinder 40. p; corresponding
are contained in parallel planes. 41. p; alternate interior
42. m; alternate exterior
43. ; alternate exterior
44. m; corresponding
45. q; alternate interior
46. ; corresponding
47. m; consecutive interior

Chapter 3 54
48. Skew lines; the planes are flying in different 61. mEFG is less than 90; Law of Detachment
directions and at different altitudes. p: an angle is acute
G
49. C , DH, E
I q: its measure is less than 90
E
50. D , FG
, H
I, G
H
, B
F
, D
H
, E
I 62. d ¬
(x2  
x1)2 
(y2 
y1)2
51. No; plane ADE will intersect all the planes if they AB ¬
[3  (
1)]2 
 [4 
(8)]2
are extended.
¬ 2
42  12
52. Sample answers: parallel bars in gymnastics,
¬160

parallel port on a computer, parallel events, parallel
voices in a choir, latitude parallels on a map ¬12.65
53. Infinite number; consider any line through P in 63. d ¬(x2  
x1)2 
(y2 
y1)2
any plane that does not contain . CD ¬
(2 
0)2 
(9  
1)2
54. 1 ¬(2)2 
 82 ¬68
55. Sample answer: Parallel lines and planes are used ¬8.25
in architecture to make structures that will be 64. d ¬(x2  
x1)2 
(y2 
y1)2
stable. Answers should include the following.
EF ¬[5
  ( 
3)]2  [4  2
(12)]
• Opposite walls should form parallel planes; the
floor may be parallel to the ceiling. ¬
82  1
62 ¬320

• The plane that forms a stairway will not be ¬17.89
parallel to some of the walls. 65. d ¬
(x2  
x1)2 
(y2 
y1)2
56. A GH ¬(9
  4
)2  [ 
25  (10)]2
57. The elements of M are 15, 18, 21, 24, 27, and 30. ¬
52  (
15)2 ¬250

The elements of P are 16, 20, 24, and 28. The
numbers that are in P but not in M are 16, 20, ¬15.81
and 28. Select any one of these three numbers. 66. d ¬(x
 2
x1)2 
(y2 
y1)2


2
JK ¬ (3  1)2  7
4  4
 1
Page 131 Maintain Your Skills ¬
(4)2 
 (2)2 ¬20

58. Given: mABC ¬mDFE
¬4.47
m1 ¬m4
Prove: m2 ¬m3 A D 67. d ¬(x2  
x1)2 
(y2 
y1)2


1 4 2
2 3 LM ¬ [5  (5)]2  2
5  5 
 8
Proof: B C E F
¬102 
(2)2 ¬104

Statements Reasons ¬10.20
68.
1. mABC  mDFE 1. Given M
m1  m4
2. mABC  m1  m2 2. Angle Addition A P B
mDFE  m3  m4 Post. N
3. m1  m2  m3  3. Substitution Prop.
m4 69. T
4. m4  m2  m3  4. Substitution Prop. R
m4
S
5. m2  m3 5. Subt. Prop.
70. 50, 180  50 or 130
The measures of the angles are 50 and 130.
59. P Q R
71. 90, 180  90 or 90
The measures of the angles are 90 and 90.
72. x  2x ¬180
X Y Z 3x ¬180
x ¬60
Q
Since P Z Y and QR
X Y, PQ  ZY and
60, 2(60) or 120
QR  XY by the definition of congruent segments.
By the Addition Property, PQ  QR  ZY  XY. The measures of the angles are 60 and 120.
Using the Segment Addition Postulate, 73. 2y  3y ¬180
PR  PQ  QR and XZ  XY  YZ. By 5y ¬180
substitution, PR  XZ. Because the measures are y ¬36
R
equal, PX Z by the definition of congruent 2y ¬2(36) or 72
segments. 3y ¬3(36) or 108
60. no conclusion The measures of the angles are 72 and 108.

55 Chapter 3
74. x  2x  3x ¬180 2.
6x ¬180 35
x ¬30
35
There are two linear pairs in the figure. In one
linear pair the angles measure x and
3x  2x  5x, or 30 and 150. In the other pair the 3. 1; all other angles can be determined using the
angles measure 3x and x  2x  3x, or 90 and 90. Corresponding Angles Postulate, the Alternate
75. 3x  1  2x  6 ¬180 Interior Angles Theorem, the Consecutive Interior
5x  5 ¬180 Angles Theorem, and the Alternate Exterior
5x ¬175 Angles Theorem.
x ¬35 4. Alternate Interior Angles Theorem
3x  1 ¬3(35)  1
5. 1 ¬3
¬104
m1 ¬m3
2x  6 ¬2(35)  6
m1 ¬110
¬76
6. 6 ¬3
The measures of the angles are 76 and 104.
m6 ¬m3
m6 ¬110
Page 132 Geometry Software Investigation: 7. 2 and 3 are supplementary.
Angles and Parallel Lines m2  m3 ¬180
1. The pairs of corresponding angles are AEG and m2  110 ¬180
CFE, AEF and CFH, BEG and DFE, m2 ¬70
BEF and DFH. The pairs of alternate interior 8. 10 ¬12
angles are AEF and DFE, BEF and CFE. m10 ¬m12
The pairs of alternate exterior angles are AEG m10 ¬55
and DFH, BEG and CFH. The pairs of 9. 13 ¬12
consecutive interior angles are AEF and CFE, m13 ¬m12
BEF and DFE. m13 ¬55
2. The pairs of corresponding angles in Exercise 1 10. 15 ¬12
that have the same measure are AEG and m15 ¬m12
CFE, AEF and CFH, BEG and DFE, m15 ¬55
BEF and DFH. The pairs of alternate interior 11. 8y  2  25y  20 ¬180
angles that have the same measure are AEF 33y  18 ¬180
and DFE, BEF and CFE. The pairs of 33y ¬198
alternate exterior angles that have the same y ¬6
measure are AEG and DFH, BEG and CFH. 10x  8y  2 ¬180
3. They are supplementary. 10x  8(6)  2 ¬180
4a. If two parallel lines are cut by a transversal, 10x  50 ¬180
then corresponding angles are congruent. 10x ¬130
4b. If two parallel lines are cut by a transversal, x ¬13
then alternate interior angles are congruent. 12. 4x  5 ¬3x  11
4c. If two parallel lines are cut by a transversal, 4x ¬3x  16
then alternate exterior angles are congruent. x ¬16
4d. If two parallel lines are cut by a transversal, then 4x  5  3y  1 ¬180
consecutive interior angles are supplementary. 4(16)  5  3y 1 ¬180
60  3y ¬180
5. Yes; the angle pairs show the same relationships.
3y ¬120
6. See students’ work. y ¬40
7a. Sample answer: All of the angles measure 90°. 13. 36
7b. Sample answer: If two parallel lines are cut by a 2
transversal so that it is perpendicular to one of 1
the lines, then the transversal is perpendicular 3
to the other line. 31

Draw a third line through the vertex of 1


parallel to the two given lines.
3-2 Angles and Parallel Lines 2 is congruent to the angle whose measure is
labeled 36° by the Alternate Interior Angles
Theorem. So m2  36.
Page 136 Check for Understanding 3 is congruent to the angle whose measure is
1. Sometimes; if the transversal is perpendicular to labeled 31° by the Alternate Interior Angles
the parallel lines, then 1 and 2 are right
angles and are congruent.

Chapter 3 56
Theorem. So m3  31. 27. 5 ¬3
m1 ¬m2  m3 m5 ¬m3
¬36  31 or 67 m5 ¬60
28. 2 and 6 are supplementary.
m2  m6 ¬180
Pages 136–138 Practice and Apply m4  m5  m6 ¬180
14. 3 ¬9 m6 ¬180  (m4  m5)
m3 ¬m9 m2  180  (m4  m5) ¬180
m3 ¬75 m2 ¬m4  m5
15. 5 ¬9 5 ¬3
m5 ¬m9 m5 ¬m3
m5 ¬75 From Exercise 26,
16. 6 and 9 are supplementary. m4 ¬m1  50
m6  m9 ¬180 m2 ¬m1  m3
m6  75 ¬180 m2 ¬50  60
m6 ¬105 m2 ¬110
17. 7 ¬9 29. m6  m4  m5 ¬180
m7 ¬75 m6  50  60 ¬180
7 and 8 are supplementary. m6 ¬70
m7  m8 ¬180 30. 7 ¬2
75  m8 ¬180 m7 ¬m2
m8 ¬105 m2 ¬110 (from Exercise 28)
18. 11 ¬9 So m7 ¬110.
m11 ¬m9 31. 8 is congruent to an angle that forms a linear
m11 ¬75 pair with 3.
19. 12 and 9 are supplementary. m8  m3 ¬180
m12  m9 ¬180 m8  60 ¬180
m12  75 ¬180 m8 ¬120
m12 ¬105 32. 4x  56 ¬180
20. 2 and 3 are supplementary. 4x ¬124
m2  m3 ¬180 x ¬31
m2  43 ¬180 3y  11  56 ¬180
m2 ¬137 3y  45 ¬180
21. 7 ¬3 3y ¬135
m7 ¬m3 y ¬45
m7 ¬43 33. 2x ¬68
22. 10 ¬2 x ¬34
m10 ¬m2 68  3x  15  y2 ¬180
m10 ¬137 68  3(34)  15  y2 ¬180
155  y2 ¬180
23. 11 ¬3
y2 ¬25
m11 ¬m3
y ¬5
m11 ¬43
34.
24. 13 ¬3
m13 ¬m3
m13 ¬43 110
4
25. 16 ¬2
m16 ¬m2 2
1
m16 ¬137 3
26. 37
7
2
6 p Draw a third line through the vertex of 1
9 4 5 8
parallel to the two given lines.
1
3
q 2  4 by the Alternate Interior Angles
j Theorem. m4  110  180, so m4  70 and
k m n hence m2  70.
4 ¬9
3 is congruent to the angle whose measure is
9 ¬1
labeled 37° by the Alternate Interior Angles
4 ¬1
Theorem. So m3 ¬37.
m4 ¬m1
m1 ¬m2  m3
m4 ¬50
¬70  37 or 107

57 Chapter 3
35. 39. Given:  m p
Prove: 1 ¬8
2 ¬7 1 2
3 4
1 5 6
5 7 8
m
4 90 3 157
2 Proof:
Statements Reasons
1.  m 1. Given
Extend the ray that forms the 157° angle in the 2. 1 ¬5, 2 ¬6 2. Corresponding
opposite direction so that the line crosses the left Angles Postulate
line of the pair of parallel lines.
3. 5 ¬8, 6 ¬7 3. Vertical Angles
Then m4  90.
Theorem
m5  m4  m2 ¬180
m5 ¬180  m4  m2 4. 1 ¬8, 2 ¬7 4. Transitive Property
m5 ¬180  90  m2
40. Given: m n,  is
m5 ¬90  m2
a transversal. m
2 ¬3 1 3
Prove: 1 and 2 are
m2 ¬m3 2 4
supplementary; n
m3  157 ¬180
3 and 4 are
m3 ¬23
m2 ¬23 supplementary.
m1  m5 ¬180 Proof:
m1  90  m2 ¬180 Statements Reasons
m1  90  23 ¬180
m1  67 ¬180 1. m n,  is a transversal 1. Given
m1 ¬113
2. 1 and 3 form a 2. Def. of linear
36. x ¬90 linear pair; 2 and 4 pair
3y  11 ¬y  19 form a linear pair.
3y ¬y  30
2y ¬30 3. 1 and 3 are 3. If two angles form
y ¬15 supplementary; a linear pair, then
3y  11  4z  2  x ¬180 2 and 4 are they are
3(15)  11  4z  2  90 ¬180 supplementary. supplementary.
4z  126 ¬180 4. 1 ¬4, 2 ¬3 4. Alt. int.  ¬
4z ¬54
z ¬13.5 5. 1 and 2 are 5. Substitution
37. 7y  4  7x  9 ¬180 supplementary;
7y  7x  5 ¬180 3 and 4 are
7y  7x ¬175 supplementary.
7y ¬175  7x
y ¬1 41. Given:   m, m n
7 (175  7x)

Prove:   n
y ¬25  x
2y  5  11x  1 ¬180
2(25  x)  5  11x  1 ¬180 1 2
m
50  2x  5  11x  1 ¬180 3 4
54  9x ¬180 n
9x ¬126
x ¬14
Proof: Since   m, we know that 1  2,
y ¬25  x
because perpendicular lines form congruent right
¬25  14 or 11
angles. Then by the Corresponding Angles
z  7x  9 ¬180
Postulate, 1  3 and 2  4. By the definition
z  7(14)  9 ¬180
of congruent angles, m1  m2, m1  m3 and
z  107 ¬180
m2  m4. By substitution, m3  m4.
z ¬73
Because 3 and 4 form a congruent linear pair,
38. The angle with measure 40° is congruent to an
they are right angles. By definition,   n.
angle that forms a linear pair with the angle
whose measure is x°. So 40  x  180. 42. The angle formed by the pipe on the other side of
Then x  140. the road is supplementary to the angle that
measures 65°. So the angle is 180  65 or 115.

Chapter 3 58
43. 2 and 6 are consecutive interior angles for 5. 4 ¬2
the same transversal, which makes them m4 ¬m2
supplementary because W X Y
Z
. 4 and 6 are m2  m1 ¬180
not necessarily supplementary because  WZ may m2  105 ¬180
not be parallel to XY. m2 ¬75
44. Sample answer: Angles and lines are used in art m4 ¬75
to show depth, and to create realistic objects.
Answers should include the following.
• Rectangular shapes are made by drawing
parallel lines and perpendiculars.
3-3 Slopes of Lines
• M. C. Escher and Pablo Picasso use lines and
angles in their art. Page 142 Check for Understanding
45. C; Let y° be the measure of the third angle of the 1. horizontal; vertical
right triangle in the figure. 2. Curtis; Lori added the coordinates instead of
160 ¬y  120 finding the difference.
40 ¬y
3. horizontal line, vertical line
x  y  90 ¬180 (y  y )
x  40  90 ¬180 4. m ¬
2
1
(x2  x1)
x ¬50 1  3
¬ 
2  (4)
46. C; ax ¬bx  c
ax  bx ¬c 4
¬2 or 2
(a  b)x ¬c
c 5. Line  goes through P(0, 4) and Q(4, 2).
x ¬
a b (y  y )
m ¬
2
1
(x2  x1)
2 4
Page 138 Maintain Your Skills ¬
40
  2 1
47. FG ¬4 or 2
48. B
A , D
E
, FG
, IJ
, A
E
, F
J

6. Line m goes through C(0, 3) and D(3, 1).
49. CDH (y  y )
50. G
B , C
H, F
G
, HI m ¬
2
1
(x2  x1)
51. m1  124 ¬180 1  (3)
m1 ¬56 ¬ 
30
52. m2 ¬53 2
¬3
53. Hypothesis: it rains this evening
7. Line  has slope 1 
2 (from Exercise 5). Any line
Conclusion: I will mow the lawn tomorrow
perpendicular to  has a slope that is the opposite
54. Hypothesis: you eat a balanced diet reciprocal of 1

Conclusion: it will keep you healthy 2 , or 2.
 ¬0  13
7 9 2 2 8. slope of GH 
8  5  3 or  3
55.   11  14
1313
3  6 9 ¬
2
5 or 
25
56.  8 ¬
2  6
 ¬5  7
¬3

2
slope of RS 4 
(3)
14 11 12
3 ¬

23  15  8
57.  1 or 12
15 23 8 The slopes are not the same, so 
GH and 
RS are
8
14  11  3 or  3
58.   156
not parallel. The product of the slopes is 25 , so

GH and RS are not perpendicular. Therefore, 

9   5  ¬ 45
36 GH
59. 2
 1
 8 
4 and 
RS are neither parallel nor perpendicular.
¬5
9  (9)
9. slope of 
GH ¬ 
9  15
0
¬
6 or 0
Page 138 Practice Quiz 1 1 
 ¬
slope of RS (1)
1. p; alternate exterior 3  (4)
2. ; consecutive interior ¬0

7 or 0
3. q; alternate interior  and RS
The slopes are the same so GH  are
4. 6  1 parallel.
m6  m1
m6  105

59 Chapter 3
10. Start at (1, 2). Move up 2 units and then move right Pages 142–144 Practice and Apply
1 unit. Draw the line through this point and (1, 2). (y  y )
15. m ¬
2
1
y (x2  x1)
32 1
¬
7 0 or 7
(y2  y1)
P (1, 2) 16. m ¬ 
(x2  x1)
5  (3)
O x ¬6  (2)
2 1
¬4 or 2
(y2  y1)
17. m ¬ 
(x2  x1)
. 3 2
11. Find the slope of MN ¬43
(y  y ) 5
m ¬
2
1
¬1 or 5
(x2  x1)
20 (y2  y1)
¬1  18. m ¬ 
5 (x2  x1)
2
¬4 or 1  37
2 ¬
41
Since 22  1, the slope of the line
1
 through A(6, 4) is 2. ¬4

3
perpendicular to MN
1  (2)
Graph the line. Start at (6, 4). Move up 2 units 19. slope of 
PQ ¬ 
9  (3)
and then move right 1 unit. Draw the line 3 1
¬
12 or 4
through this point and (6, 4).
2 
 ¬ 6
y slope of UV 53
8
12 ¬2 or 4
The product of the slopes is 4(4) or 1. So, 
1
PQ
8 
is perpendicular to UV.
 ¬ 3 
0
4 A(6, 4) 20. slope of PQ 0  (4)
3
¬4
O 8 12 x 6  (
3)
slope of 
UV ¬
8  (4)
9 3
12. The hill has an 8% grade, so the road will rise or ¬
1
2 or 4
fall 8 units vertically with every 100 horizontal  is parallel to UV
The slopes are the same, so PQ .
8 2
units traveled. So the slope is either 
100  25  ¬
21. slope of PQ 1  7
8 2 2  (10)
or  
100   
25
. 6 1
2 ¬
12 or 

13. Let (x1, y1)  (0, 0) and m   . Then y2  120 2
25 10
 ¬
because the biker is 120 meters below her slope of UV 
64
starting position. 1
¬2
(y2  y1)
m ¬  The slopes are not the same, so PQ  and UV
 are
(x2  x1)
not parallel. The product of the slopes is 21
2
1

 ¬ x120
2   0
25 20 1  and UV
or 4, so PQ  are not perpendicular.

25 ¬ x1
2  20
Therefore, PQ and UV
 are neither parallel nor
2
x2 ¬1500 perpendicular.
12
If m   2
then x2  1500. So the current 22. slope of 
PQ ¬ 
0  (9)
25
1
position of the biker is represented ¬9
by (1500, 120) or (1500, 120). 1  8
 ¬
14. The distance is the same no matter which
slope of UV 2 
(1)
9
coordinates are used for the biker’s current ¬
1 or 9
position.
The product of the slopes is 9(9) or 1. So, PQ

1
d ¬ (x2  x1)2 (y2 y1)2 .
is perpendicular to UV
¬ (1500  0) 
2  (120  0)2 81
 ¬
23. slope of PQ 
¬1500
 2 
 (120) 2 91
7
¬ 2,264 ,400 ¬8
¬1505  ¬ 8 
1
slope of UV 2  (6)
The biker has traveled 1505 meters down the hill. 7
¬8
 is parallel to UV
The slopes are the same, so PQ .

Chapter 3 60
0  (4)
24. slope of 
PQ ¬ 
10  5 The line to be graphed is parallel to 
CD so the
line has slope 1.
¬4

5 Start at (1, 3). Move down 1 unit and then
 ¬
slope of UV 13 (8)
move right 1 unit. Draw the line through this
59
¬5
4
 point and (1, 3).
The slopes are not the same, so 
PQ and 
UV are y
not parallel. The product of the slopes is 4
5  4 
 5
or 1, so 
PQ and 
UV are not perpendicular.
 and UV
Therefore, PQ  are neither parallel nor
O x
perpendicular.
(y2  y1)
25. m ¬ 
(x2  x1)
 1 2
A( 1, 3)
¬
0  (1)
3
¬1 or 3

26. m ¬
2(y  y )
1 35. Find the slope of 
GH.
(x2  x1) (y  y )
m ¬
2
1
4  (
5) (x2  x1)
¬1  (4) 03
¬ 3 
 0
¬9
5
 3
¬3 or 1
(y2  y1)
27. m ¬  Since 1(1)  1, the slope of the line
(x2  x1)
 through M(4, 1), is 1.
perpendicular to GH
1  5
¬3  (2) Start at (4, 1). Move down 1 unit and then move
6 right 1 unit. Draw the line through this point and
¬ 1 or 6 (4, 1).
(y2  y1)
28. m ¬ y
(x2  x1)
 4 (4)
¬ 4  ( 2)
¬0
6
 or 0
 is 6 (from Exercise 27). A line
29. The slope of LM M(4, 1)
parallel to 
LM has the same slope as 
LM, thus
has slope 6. O x
30. The slope of 
9
PQ is 5 (from Exercise 26). A line
 has slope that is the opposite
perpendicular to PQ
9 5 36. Start at (7, 1). Move up 2 units and then move
reciprocal of 5, or 9. right 5 units. Draw the line through this point
 is 0 (from Exercise 28). EF
31. The slope of EF  is and (7, 1).
horizontal, so a line perpendicular to 
EF is y
vertical and has undefined slope.
32. The slope of 
AB is 3 (from Exercise 25). A line
parallel to 
AB has the same slope as 
AB, thus has
slope 3.
33. Start at (2, 1). Move down 4 units and then O x
move right 1 unit. Draw the line through this J ( 7, 1)
point and (2, 1).
y
P ( 2, 1)
O x

34. Find the slope of 


CD.
(y  y )
m¬¬
2
1
(x2  x1)
1 7
¬
5  ( 1)
6
¬6 or 1

61 Chapter 3
1
37. Find the slope of 
KL. 41. Let (x1, y1)  (2000, 35.3) and m  3.
(y  y ) (y  y )
m ¬
2
1
m ¬
2
1
(x2  x1) (x2  x1)
40.6  35.3
 7
¬ 2122
1
 ¬
3 x2  2000
 19
¬0 which is undefined x2  2000 ¬3(5.3)

KL is a vertical line, so a line parallel to 
KL x2 ¬2015.9
through Q(2, 4) is also vertical. The median age will be 40.6 in 2016.
Draw a vertical line through (2, 4). (y  y )
42. m ¬
2
1
y (x2  x1)
3 1  2
 7 ¬ x 
   6
3 3
7 ¬
x 6
O x 7 ¬x 6
13 ¬x
y
12
Q( 2, 4)
8

38. Find the slope of 


DE. 4
(y  y ) (6, 2)
m ¬
2
1
(x2  x1) x
0  2 O 4 8
¬50 (13, 1)
2 4
¬ 5

Since  5 2  1, the slope of the line
2 5
43. m1 ¬
2
1 (y  y )
perpendicular to DE through W(6, 4) is 5  (x2  x1)
2.  18
Start at (6, 4). Move up 5 units and then move ¬ 2 4
right 2 units. Draw the line through this point 9
¬2
and (6, 4).
y 9229  1, so the line containing (x, 2) and
4
W (6, 4) (4, 5) perpendicular to the line containing (4, 8)
2
and (2, 1) has slope 9.
4 O 8 x
(y2  y1)
m2 ¬ 
4 (x2  x1)
2 52
9 ¬
4 
x
8
2 3
9 ¬
4 
x
12
2(4  x) ¬27
39. Sample answer: The median age in 1970 is 8  2x ¬27
approximately 28. The median age in 2000 is 35.3. 2x ¬19
19
Find the slope of the line through (1970, 28) and x ¬2
(2000, 35.3). y
(y  y )
m ¬
2
1
(4, 8)
(x2  x1) 8
35.3  28
¬
2000  1970
( 4, 5) 4 (1–9–, 2)
7.3 2
¬
30
¬0.24 4 O 4 8 x
The annual rate of change is approximately (2, 1)
4
0.24 year per year.
40. Sample answer: Let (x1, y1)  (2000, 35.3) and
m  0.24. (y  y )
44. m ¬
2
1
(y2  y1) (x2  x1)
m ¬  77  51
(x2  x1) ¬
2000  1998
y2  35.3 26
0.24 ¬  ¬ or 13
2010  2000 2
y2  35.3 The percent changes at a rate of 13% per year.
0.24 ¬  (y  y )
10
45. m ¬ 2
 1
(x  x )
2.4 ¬y2  35.3 2 1
77  64
37.7 ¬y2 ¬ 
2000  1999
The median age will be 37.7 in 2010. ¬13

Chapter 3 62
90  77
13 ¬ 
x  2000 Page 144 Maintain Your Skills
13 (x  2000) ¬13 51. 6 ¬1
x  2000 ¬1 m6 ¬m1
x ¬2001 m6 ¬131
90% of classrooms will have Internet access in 52. 7 is supplementary to 6.
2001. m7  m 6 ¬180
46. No; the graph can only rise until it reaches 100%. m7  131 ¬180
47. The y-intercept can be found by setting the m7 ¬49
equation for x equal to zero, solving for t, then 53. 4 ¬7
using this value in the equation for y. m4 ¬m 7
x ¬5  2t m4 ¬49
0 ¬5  2t 54. m2  m1 ¬180
5 ¬2t m2  131 ¬180
5
2 ¬t m2 ¬49
y ¬3  t 55. m5  m1 ¬180
m5  131 ¬180
y ¬3  5
2
 m5 ¬49
11
y ¬2 56. 8 ¬6
Find the x-intercept in a similar manner. 6 ¬1
y ¬3  t 8 ¬1
0 ¬3  t m8 ¬m1
3 ¬t m8 ¬131
x ¬5  2t 57. ; alternate exterior
x ¬5  2(3) 58. ; corresponding
x ¬11 59. p; alternate interior
11
So two points on the line are (0, 2) and (11, 0).
(y  y ) 60. q; consecutive interior
m ¬
2
1
61. m; alternate interior
(x2  x1)
0  2
11 62. q; corresponding
¬ 
11  0 63. H, I, and J are noncollinear.
11 H

2 1
¬  or 2 I
11
J
The slope-intercept form of the equation of the
1 11 64. XZ  ZY  XY.
line is y  2x  
2 .
48. Sample answer: Slope is used when driving X Z Y
through hills to determine how fast to go. 65. R, S, and T are collinear.
Answers should include the following. y
• Drivers should be notified of the grade so that
they can adjust their speed accordingly. A
positive slope indicates that the driver must O x
speed up, while a negative slope indicates that
the driver should slow down.
S T R
• An escalator must be at a steep enough slope to
be efficient, but also must be gradual enough to
ensure comfort.
(y  y )
49. C; m ¬
2
1
66. acute
(x2  x1)
2  1 67. obtuse
¬ 3  (5)
68. right
 3
¬2 69. obtuse
2x  y ¬7
23 23  1, so the slope of the line 70.
y ¬2x  7
perpendicular to the line containing (5, 1) and
71. 2x  4y ¬5
(3, 2) is 2

3. 4y ¬2x  5
24
50. A; the winning sailboat completed the race in  9
 1
y ¬2x  5

2 4
hours, or 23 hours. The second-place boat 72. 5x  2y  4 ¬0
24
completed the race in  8 hours, or 3 hours. The 5x  4 ¬2y
difference in times is 1

3 hour, or 20 minutes.
5
2x  2 ¬y
5
y ¬2 x  2

63 Chapter 3
Alternate plan: y ¬0.95x  4.95
3-4 Equations of Lines ¬0.95(60)  4.95
¬61.95
He should keep his current plan, based on his
Pages 147–148 Check for Understanding average usage.
1. Sample answer: Use the point-slope form where
(x1, y1)  (2, 8) and m  2

5.
2. Sample answer: y  2x  3, y  x  6 Pages 148–149 Practice and Apply
3. Sample answer: y  x 15. y ¬mx  b
y y ¬1
6x  4

y x
16. y ¬mx  b
y ¬2
3x  8

17. y ¬mx  b
O x y ¬5
8x  6

18. y ¬mx  b
y ¬2
9x  3
 1
19. y ¬mx  b
4. y ¬mx  b y ¬x  3
1 20. y¬¬mx  b
y ¬2x  4 1
y¬¬ x1
5. y ¬mx  b 12
3 21. y  y1 ¬m(x  x1)
y ¬5x  2
y  1 ¬2(x  3)
6. y ¬mx  b
22. y  y1 ¬m(x  x1)
y ¬3x  4
y  7 ¬5(x  4)
7. y  y1 ¬m(x  x1)
23. y  y1 ¬m(x  x1)
y  (1) ¬3
2 (x  4)

y  (5) ¬4
5 [x  (12)]

y  1 ¬3
2 (x  4)

y  5 ¬4
5 (x  12)

8. y  y1 ¬m(x  x1)
24. y  y1 ¬m(x  x1)
y  5 ¬3(x  7) 1
y  11 ¬
16 (x  3)
9. y  y1 ¬m(x  x1)
25. y  y1 ¬m(x  x1)
y  137.5 ¬1.25(x  20) y  17.12 ¬0.48(x  5)
(y  y )
10. m ¬2
1 26. y  y1 ¬m(x  x1)
(x2  x1)
y  87.5 ¬1.3 (x  10)
53
¬ 
0  (1) 27. Find the slope of k using (0, 2) and (1, 1).
(y  y )
¬2 m ¬
2
1
(x2  x1)
y ¬2x  5
(y  y ) 1  (2)
11. m ¬2
1 ¬1 0
(x2  x1)
¬3
32
¬1 
0 y ¬mx  b
¬1 y ¬3x  2
y ¬x  2 28. Find the slope of  using (0, 5) and (1, 4).
12. The slope of  is 2 (from Exercise 10). The line (y  y )
m ¬
2

1
parallel to  that contains (4, 4) also has slope 2. (x2  x1)
y  y1 ¬m(x  x1) 45
¬
1 
0
y  4 ¬2(x  4)
y  4 ¬2x  8 ¬1
y ¬2x  4 y ¬mx  b
13. The total monthly cost of Justin’s current plan is y ¬x  5
y  39.95. For the other provider, the cost 29. Find the slope of m using (2, 0) and (1, 2).
increases $0.95 for each hour of connection so the (y  y )
m ¬
2
1

slope is 0.95. The y-intercept is where 0 hours are (x2  x1)


used, or $4.95. 2  0
¬1 2
y ¬mx  b
¬2
y ¬0.95x  4.95
y  y1 ¬m(x  x1)
14. Current plan: y  39.95 (no matter how many y  0 ¬2(x  2)
hours Justin uses) y ¬2x  4

Chapter 3 64
30. Find the slope of n using (0, 6) and (8, 5). y  y1 ¬m(x  x1)
(y2  y1)
m ¬  y  (3) ¬1
5 [x (5)]

(x2  x1)
1
5 
¬ 6 y  3 ¬5x  1
80
1 y ¬1
5x  4

¬8
(y  y )
y ¬mx  b 40. m ¬
2
1
(x2  x1)
y ¬1
8x  6

¬1 0
05
31. Since the slope of line  is 1 (from Exercise 28),
the slope of a line perpendicular to it is 1. ¬1
5

y  y1 ¬m(x  x1) y ¬mx  b
y  6 ¬1[x  (1)] y ¬1
5x  1

y  6 ¬x  1 (y  y )
y ¬x  5 41. m ¬
2
1
(x2  x1)
32. Since the slope of line k is 3 (from Exercise 27), ¬
4  8
the slope of a line parallel to it is 3. 6  (6)
12
y  y1 ¬m(x  x1) ¬ 
0 which is undefined
y  0 ¬3(x  7) There is no slope-intercept form for this line. An
y ¬3x  21 equation for the line is x  6.
1 (y  y )
33. Since the slope of line n is 8 (from Exercise 30), 42. m ¬
2
1
1 (x2  x1)
the slope of a line parallel to it is 8.
5  (1)
y  y1 ¬m(x  x1) ¬
8  (4)
4
y  0 ¬1 ¬
4 or 1
8 (x  0)

y  y1¬m(x  x1)
y ¬1

8x y  (1) ¬1[x  (4)]
34. Since the slope of line m is 2 (from Exercise 29), y  1 ¬x  4
the slope of a line perpendicular to it is 1
 y ¬x  3
2.
y  y1 ¬m(x  x1) 43. 2x  5y ¬8
5y ¬2x  8
y  (3) ¬12 [x  (3)]

1 3
y ¬2
5x  5
 8
y  3 ¬2 x  2 The slope of the line is 2

5 , so the slope of a line
y ¬1 9
2x  2
 parallel to it is 2
5
.
35. y ¬mx  b y  y1 ¬m(x  x1)
y ¬3x  5 y  (2) ¬2
5 (x  7)

36. y ¬mx  b y  2 ¬2 14
5x  5
  
y ¬0  x  6
y ¬6 y ¬2
 24
5x  5

(y  y ) 44. 2y  2 ¬7
4 (x  7)

37. m ¬
2
1
(x2  x1)
2y  2 ¬7 4
4x  4
  9
3  0
¬ 2y ¬7 41
4x  4
05   
¬3 y ¬7 41
8x  8
5   
y ¬mx  b 7
The slope of the line is 8, so the slope of a line
y ¬3
5x  3
 perpendicular to it is 8

7.
(y2  y1) y  y1 ¬m(x  x1)
38. m ¬
(x2  x1) y  (3) ¬8
7 [x  (2)]

1 
(1) y  3 ¬8 16
7x  7
¬   
2  4
¬
0 y ¬8
7x  7
 5

6 or 0
y  y1 ¬m(x  x1) 45. For each appliance Ann sells she earns $50, so
Ann earns 15($50) or $750 plus commission. If the
y  (1) ¬0(x  4)
total price of the appliances Ann sells is x dollars,
y  1 ¬0
her commission is 0.05x. So, Ann earned y 
y ¬1
(y  y ) 0.05x  750 dollars in a week in which she sold
39. m ¬
2
1
15 appliances.
(x2  x1)
6  (3) 46. 750x
¬10  (5) 47. The number of gallons of paint in stock decreases
3 1
¬15 or 5
at a rate of 750 gallons per day.
y  750x  10,800

65 Chapter 3
48. 16 Page 150 Maintain Your Skills

Gallons of Paint (thousands)


14 (y  y )
56. m ¬
2
1
(x2  x1)
12
0 
10 ¬ 4  6

0
 6 3
8 ¬4 or 2
6 (y2  y1)
57. m ¬ 
(x2  x1)
4
6 1
2 ¬ 88
7
0 2 4 6 8 10 12 14 16
¬ 0 , which is undefined
Days (y2  y1)
58. m ¬ 
49. Find x when y ¬0. (x2  x1)
y ¬750x  10,800 3 3
¬6  6
0 ¬750x  10,800
0
750x ¬10,800 ¬12 or 0
x ¬14 59. 7 ¬1
The store will run out of paint after 14 days, so m7 ¬m1
the manager should order more paint after 14  4 m7 ¬58
or 10 days. 60. 5 ¬2
50. Two points on the line are (120, 60) and m5 ¬m2
(130, 70). m5 ¬47
(y  y )
m ¬
2
1 61. m1  m2  m6 ¬180
(x2  x1)
58  47  m6 ¬180
70 (60) m6 ¬75
¬130  120
62. m4  m2  m3 ¬180
10
¬ 10 or 1 m4  47  26 ¬180
y  y1 ¬m(x  x1) m4 ¬107
y  (60) ¬1(x  120) 63. m8 ¬m2  m3
y  60 ¬x  120 m8 ¬47  26
y ¬x  60 m8 ¬73
51. The slope of the Jeff Davis/Reeves County line
64. From Exercise 59, m7 ¬58.
is 1 (from Exercise 50), so the slope of the
From Exercise 63, m8 ¬73.
Reeves/Pecos County line is 1.
m7  m8  m9 ¬180
y  y1 ¬m(x  x1)
58  73  m9 ¬180
y  (60) ¬1(x  120)
m9 ¬49
y  60 ¬x  120
65. Given: AC ¬DF
y ¬x  180 A B C
AB ¬DE
52. The equation y  mx is the special case of Prove: BC ¬EF
y  m(x  x1)  y1 when x1  y1  0. D E F
53. Sample answer: In the equation of a line, the Proof:
b value indicates the fixed rate, while the mx Statements Reasons
value indicates charges based on usage. Answers
1. AC ¬DF 1. Given
should include the following.
AB ¬DE
• The fee for air time can be considered the slope
of the equation. 2. AC  AB  BC 2. Segment Addition
• We can find where the equations intersect to DF  DE  EF Postulate
see where the plans would be equal. 3. AB  BC  DE  EF 3. Substitution
54. A; 2x  8y ¬16 Property
8y ¬2x  16 4. BC  EF 4. Subtraction
1 Property
y ¬x  16
1 4
4(4) ¬1 66. d ¬(x
 2
x1)2 
(y2 
y1)2
55. B; y2  1, so y  1 and y  1, thus y  1 and AB ¬
(2 10)2  [8 (6)]2
y 1.
¬
(12)2
 ( 2)2
¬
148
BC ¬
[5 (2)]2 [ 2
7  (8)]
¬
(3) 
2 12

¬
10

Chapter 3 66
AC ¬(5  10)2  [7  (6)]2 6. From Exercise 4, q has slope 1

2 and has
¬(15) 2  ( 1) ¬
2 226 y-intercept 2.
y ¬mx  b
AB  BC  AC ¬ 148   10  226

¬30.36 y ¬1
2x  2


67. d ¬(x2  x1)2  (y2y 2 7. From Exercise 5, r has slope 4



5 . A line parallel
1)
AB ¬[2  (3)] 2  (9  2)2 to r also has slope 4
.
5
y  y1 ¬m(x  x1)
¬ 2  
52  (11) 146
BC ¬(0  2 )  [
2 10  ( 9)]2 y  4 ¬4
5 [x  (1)]

¬(2) 
2  (1 )  
2 5 y  4 ¬4 4
5x  5

AC ¬[0  (3)]2   (1 0  2)2
y ¬4 1
5x  5
  6
¬ 2  
32  (12) 153
AB  BC  AC ¬ 146   5  
153 8. From Exercise 3, p has slope 7

2 . A line
¬26.69 perpendicular to p has slope 2 
7.
68. 2 and 5, 3 and 8 y ¬mx  b
69. 1 and 5, 2 and 6, 4 and 8, 3 and 7
y ¬2
7x  0

70. 1 and 7, 4 and 6
71. 2 and 8, 3 and 5 y ¬2

7x
9. The lines are parallel so they have the same
slope: 1
4.
Page 150 Practice Quiz 2 y  y1 ¬m(x  x1)
1  (
1. slope of 
AB ¬  1)
y  (8) ¬1
4 (x  5)
63 
¬2

y  8 ¬1
4 (x  5)
3 
4  (
2)
slope of 
CD ¬ 2  (2) 10. The line y  3 is a horizontal line, so a line
¬6
 3 perpendicular to this line is vertical. An equation
4 or 2 of the vertical line through (4, 4) is 0  x  4.
The slopes are not the same, so 
AB and 
CD are
not parallel. The product of the slopes is 2
3  2 
 3
 
or 1, so AB and CD are not perpendicular.
 and CD
 are neither parallel nor
3-5 Proving Lines Parallel
Therefore, AB
perpendicular.
2. 13  (
 ¬
slope of AB 11) Page 154 Check for Understanding
3  (3) 1. Sample answer: Use a pair of alternate exterior 
24
¬ 6 or 4
 that are congruent and cut by a transversal; show
 8  (6) that a pair of consecutive interior angles are
slope of CD ¬ 80 supplementary; show that alternate interior 
2 1
¬ 8 or 4 are ; show two lines are  to same line; show
The product of the slopes is 41 corresponding  are .
4  or 1. So AB is
 
perpendicular to 
CD. 2. Sample answer:
3. Line p contains (0, 4) and (2, 3). t
(y2  y1)
m ¬ 
(x2  x1)
3  ( 4) 7
¬ 2  0 ¬ 2
 m
4. A line parallel to q has the same slope as q.
Line q contains (0, 2) and (2, 3).
(y  y ) 3. Sample answer: A basketball court has parallel
m ¬
2
1
lines, as does a newspaper. The edges should be
(x2  x1)
3  2 equidistant along the entire line.
¬ 1
2  0 ¬ 2 4.  m; corresponding angles
5. Line r contains (1, 1) and (4, 3). 5.  m; alternate interior angles
(y  y )
m ¬
2
1 6. p q; consecutive interior angles
(x2  x1)
3  (1)
7. p q; alternate exterior angles
4
¬4  1 ¬5

4554  1, so a line perpendicular to r has


slope 5

4.

67 Chapter 3
8. mADE ¬9x  5
¬9(11.375)  5
A
¬97.375
B (5x  9) Verify the angle measure by using the value of x
to find mABC.
C mABC ¬7x  3
D (14x  9) ¬7(11.375)  3
¬82.625
Now mCBD  180  mABC  180  82.625
or 97.375. Since mCBD  mADE,
E
 m CBD  ADE and  m.
10. Given: 1  2
Explore: From the figure, you know that
Prove:  m 1
mABC  5x  90 and mADE  14x  9. You
also know that ABC and ADE are 3
corresponding angles. 2
m
Plan: For line  to be parallel to line m, the Proof:
corresponding angles must be congruent. So,
mABC  mADE. Substitute the given angle Statements Reasons
measures into this equation and solve for x. 1. 1  2 1. Given
Solve: mABC ¬mADE
2. 2  3 2. Vertical angles are
5x  90 ¬14x  9
congruent.
90 ¬9x  9
81 ¬9x 3. 1  3 3. Trans. Prop. of 
9 ¬x 4.  m 4. If corr.  are , then
Examine: Now use the value of x to find lines are .
mABC. 2 
mABC ¬5x  90 11. slope of 
AB ¬ (3) 1
0  (7) or 7
¬5(9)  90  135 7
  1
4 2
Verify the angle measure by using the value of x slope of 
CD ¬ 
6  (4)
to find mADE. That is, 14x  9  14(9)  9 or
5
135. Since mABC  mADE, ABC  ADE 4 1
¬ 
10 or 8
and  m.
9. The slope of 
CD is 1 
8 , and the slope of line AB is
A 1
E . The slopes are not equal, so the lines are not
7
parallel.
(9x  5)
(7x  3) 12. Yes; sample answer: Pairs of alternate
B interior angles are congruent.

D Pages 155–157 Practice and Apply


C
m  13. a b; alternate interior 
14. none
Explore: From the figure, you know that
mABC  7x  3 and mADE  9x  5. You also 15.  m; corresponding 
know that ABC and CBD are supplementary, 16. none
and CBD and ADE are alternate interior 17. 
AE and BF;  corresponding 
angles. 18. 
AE and BF;  corresponding 
Plan: For line  to be parallel to line m, the 19.  and EG
AC ;  alternate interior 
alternate interior angles must be congruent. So, 20.  and EG
AC ; supplementary consecutive
mCBD  mADE, where mCBD  mABC  interior 
180. Thus mCBD  180  mABC, so  and JT;  corresponding 
21. HS
mADE  180  mABC. Substitute the given
22.  and JT
HS ;  alternate interior 
angle measures into this equation and solve for x.
Solve: mADE ¬180  mABC 23.  and PR
KN ; supplementary consecutive
9x  5 ¬180  (7x  3) interior 
9x  5 ¬180  7x  3 24.  and JT
HS ; 2 lines  the same line
16x  5 ¬180  3
16x ¬182
x ¬11.375
Examine: Now use the value of x to find
mADE.

Chapter 3 68
25. Given:   t t Solve: mABC ¬mDEF
mt 8x  4 ¬9x  11
Prove:  m
1  4 ¬x  11
15 ¬x
2 Examine: Verify the angle measures by using the
value of x to find mABC and mDEF.
m
mABC ¬8x  4
Proof: ¬8(15)  4 or 124
mDEF ¬9x  11
Statements Reasons
¬9(15)  11 or 124
1.   t, m  t 1. Given Since mABC  mDEF, ABC  DEF
2. 1 and 2 are right 2. Definition of and  m.
angles. perpendicular 28.  m
3. 1  2 3. All rt.  are . D
4.  m 4. If corr.  are ,
then lines are .
A B
26. A
 (7x  1) E F

(9x  4)
B
m C
D
C
E Explore: From the figure, you know that
140
mABC  7x  1 and DEF is a right angle and
hence has measure 90. You also know that ABC
F and DEF are alternate exterior angles.
Plan: For line  to be parallel to line m, the
Explore: From the figure, you know that mABC alternate exterior angles must be congruent. So,
 9x  4 and mDEF  140. You also know that mABC  mDEF. Substitute the given angle
ABC and DEF are alternate exterior angles. measures into this equation and solve for x.
Plan: For line  to be parallel to line m, the Solve: mABC ¬mDEF
alternate exterior angles must be congruent. So, 7x  1 ¬90
mABC  mDEF. Substitute the given angle 7x ¬91
measures into this equation and solve for x. x ¬13
Solve: mABC ¬mDEF Examine: Verify the angle measure by using
9x  4 ¬140 the value of x to find mABC. That is, 7x  1
9x ¬144  7(13)  1 or 90. Since mABC  mDEF,
x ¬16 ABC  DEF and  m.
Examine: Verify the angle measure by using 29.  m
the value of x to find mABC. That is, 9x  4 A E
D
 9 (16)  4 or 140. Since mABC  mDEF,
ABC  DEF and  m.
27. (4  5x) (7x  100)
A
C

(8x  4) E D
B
C B (9x  11)

F Explore: From the figure, you know that


 m mABC  4  5x and mECD  7x  100. You
Explore: From the figure, you know that mABC also know that ABC and ECD are
 8x  4 and mDEF  9x  11. You also know corresponding angles.
that ABC and DEF are alternate exterior angles. Plan: For line  to be parallel to line m, the
Plan: For line  to be parallel to line m, the corresponding angles must be congruent. So,
alternate exterior angles must be congruent. So, mABC  mECD. Substitute the given angle
mABC  mDEF. Substitute the given angle measures into this equation and solve for x.
measures into this equation and solve for x.

69 Chapter 3
Solve: mABC ¬mECD Plan: For line  to be parallel to line m, the
4  5x ¬7x  100 alternate exterior angles must be congruent. So,
4 ¬12x  100 mABC  mDEF. Substitute the given angle
96 ¬12x measures into this equation and solve for x.
8 ¬x Solve: mABC ¬mDEF
Examine: Verify the angle measures by using the 178  3x ¬7x  38
value of x to find mABC and mECD. 178 ¬10x  38
mABC ¬4  5x 216 ¬10x
¬4  5(8) or 44 21.6 ¬x
mECD ¬7x  100 Examine: Verify the angle measures by using the
¬7(8)  100 or 44 value of x to find mABC and mDEF.
Since mABC  mECD, ABC  ECD mABC ¬178  3x
and  m. ¬178  3(21.6) or 113.2
30. mDEF ¬7x  38
A
¬7(21.6)  38 or 113.2
Since mABC  mDEF, ABC  DEF and  m.
32. Given: 1 and 2
(14x  9)
B are supplementary.
1
D Prove:  m 2
3
m
E C 
(5x  90)
Proof:
Statements Reasons
F m 1. 1 and 2 are 1. Given
supplementary.
Explore: From the figure, you know that mABC
 14x  9 and mDEF  5x  90. You also know 2. 2 and 3 form a 2. Definition of
that ABC and DEF are alternate exterior angles. linear pair. linear pair
Plan: For line  to be parallel to line m, the 3. 2 and 3 are 3. Supplement Th.
alternate exterior angles must be congruent. So, supplementary.
mABC  mDEF. Substitute the given angle
measures into this equation and solve for x. 4. 1  3 4.  suppl. to same
Solve: mABC ¬mDEF  or   are .
14x  9 ¬5x  90 5.  m 5. If corr.  are ,
9x  9 ¬90 then lines are .
9x ¬81
x ¬9 33. Given: 4  6
Examine: Verify the angle measures by using the Prove:  m
4
value of x to find mABC and mDEF.
6 m
mABC ¬14x  9 7
¬14(9)  9 or 135
mDEF ¬5x  90
¬5(9)  90 or 135 Proof: We know that 4  6. Because 6 and
Since mABC  mDEF, ABC  DEF 7 are vertical angles they are congruent. By the
and  m. Transitive Property of Congruence, 4  7.
31. Since 4 and 7 are corresponding angles, and
 they are congruent,  m.
A
(178  3x) C 34. Given: 2  1, 1  3 S V
Prove: ST
 U V
 1
W
2

B m 3
E T U
Proof:
Statements Reasons
(7x  38)
D F 1. 2  1, 1  3 1. Given
2. 2  3 2. Trans. Prop.
Explore: From the figure, you know that mABC 3. If alt. int.  are
T
3. S U
V

 178  3x and mDEF 7x  38. You also know , lines are .
that ABC and DEF are alternate exterior angles.

Chapter 3 70
1.5  
(0.75)
35. Given: 
ADC
D, 1  2 C D 39. slope of 
AB ¬ 2  (1)

Prove: B CC
D
 2 0.75
1 ¬  or 0.25
3
B 1.5  1.8
slope of 
CD ¬ 
0  (1 .5)
A
 0.3
Proof: ¬ 
1.5 or 0.2
No, the lines are not parallel since the slopes are
Statements Reasons
not the same.
D
1. AC
D
, 1  2 1. Given 40. When he measures the angle that each picket
D
2. A B
C
 2. If alt. int.  are makes with the 2 by 4, he is measuring
, lines are . corresponding angles. When all of the
corresponding angles are congruent, the pickets
C
3. BC
D
 3. Perpendicular
must be parallel.
Transversal Th.
41. The 10-yard lines will be parallel because they
M
36. Given: J KN
, 1  2, 3  4 are all perpendicular to the sideline and two or
more lines perpendicular to the same line are
M
Prove: K  L
N

J K L parallel.
42. Consecutive angles are supplementary; opposite
1 2 3 4
angles are congruent; the sum of the measures of
the angles is 360.
43. See students’ work.
M N 44. Sample answer: They should appear to have the
Proof: same slope. Answers should include the following.
Statements Reasons • The corresponding angles must be equal in
order for the lines to be parallel.
M
1. J K N
, 1  2, 1. Given • The parking lot spaces have right angles.
3  4 45. B; 2 and 3 are supplementary to 1 because
2. 1  3 2. If lines are , corr. they each form a linear pair with 1. 4 is not
 are . necessarily supplementary to 1 because 1 and
3. 2  4 4 are vertical angles and hence are congruent.
3. Substitution
1  5 because  m and 1 and 5 are
M
4. K L
N 4. If corr.  are corresponding angles. 7 and 6 are
, lines are . supplementary to 5 because they each form a
linear pair with 5, and so by substitution 7
37. Given: RSP  PQR, R S and 6 are supplementary to 1. 8 is not
QRS and PQR necessarily supplementary to 1 because 8 and
are supplementary 5 are vertical angles, so 8  5 and 8  1.
S
Prove: P Q R
 46. D; let p, n, d, and q represent the numbers of
Q P pennies, nickels, dimes, and quarters Kendra has,
Proof: respectively. Then we are given that p  3n,
Statements Reasons n  d, and d  2q. So we can see that n  2q so
p  3(2q) or 6q, and thus all the numbers of coins
1. RSP  PQR, QRS 1. Given depend on the number of quarters. Kendra has at
and PQR are least one quarter, so if q  1 then d  2(1) or 2,
supplementary n  2, and p  6(1) or 6.
2. mRSP  mPQR 2. Def. of   Then 0.01p  0.05n  0.10d  0.25q
 0.01(6)  0.05(2)  0.10(2)  0.25(1)
3. mQRS  mPQR 3. Definition of  0.61
 180 suppl. 
4. mQRS  mRSP 4. Substitution Page 157 Maintain Your Skills
180
47. y ¬mx  b
5. QRS and RSP are 5. Def. of suppl.  y ¬0.3x  6
supplementary. 48. y  y1 ¬m(x  x1)
 QR
 6. If cons. int.  y  (15) ¬1
3 [x  (3)]
6. PS 
are suppl., lines
y  15 ¬1
3x  1

are .
y ¬1
3 x  14

4 
38. slope of 
AB ¬4  (
2
4) or 4
1
1 
slope of 
CD ¬ 0
4  0 or 4
1
Yes, the lines are parallel since the slopes are the
same.

71 Chapter 3
(y2  y1)
49. m ¬  63. d ¬
(x2  
x1)2 
(y2 
y1)2
(x2  x1)
11
¬ 
 7 ¬(1  8)2 (2  0)2
3  5
¬(9)
 2 2 2
¬8 or 1
4 
2 ¬85
y  y1 ¬ m(x  x1) ¬9.22
y  7 ¬1
2 (x  5)
 64. d ¬(x2  x1)2 (y2 
y1)2
y  7 ¬1
 5
2x  2 ¬[8 (6)]2
 [ 2
2  (4)]
y ¬1 1
9 ¬(2)
 2 22
2x  2
 
¬8
50. The slope of y  1 1
2 x 14 is 2 , so a line

¬2.83
perpendicular to y  2x  4 has slope 2.
y  y1 ¬m(x  x1)
y  1 ¬2(x  4)
Page 158 Graphing Calculator Investigation:
y  1 ¬2x  8
y ¬2x  9
Points of Intersection
3 2  1. Enter the equations in the Y  list and graph in
51. slope of 
5
BD ¬ 40 or 
4 the standard viewing window.
3  (3)
 ¬ KEYSTROKES: Y  2 X, T,
, n  10 ENTER 2
52. slope of CD  or 0
4  (1)
 2  (2) X, T,
, n  2 ENTER () 1 2
53. slope of AB ¬ 0  (
 4) or 1
0 
54. slope of 
EO   2 1
0  4 or 2 X, T,
, N  4 ZOOM 6
2  (
55. slope of 
DE   
44
3)
Use the CALC menu to find the points of
¬5

0 , which is undefined
intersection.
Any line parallel to 
DE also has undefined slope. Find the intersection of a and t.

56. slope of 
5 KEYSTROKES: 2nd [CALC] 5 ENTER 
BD   4 (from Exercise 51)
The slope of any line perpendicular to 
BD is 4
5. ENTER ENTER
57.
p q p and q
T T T
T F F
F T F
F F F
58. p q
q p or
q
T T F T Lines a and t intersect at (5.6, 1.2).
T F T T Find the intersection of b and t.
F T F F KEYSTROKES: 2nd [CALC] 5  ENTER
F F T T ENTER ENTER
59. p q
p
p q
T T F F
T F F F
F T T T
F F T F
60.
p q
p
q
p
q
Lines b and t intersect at (2.4, 2.8).
T T F F F
2. Enter the equations in the Y  list and graph in
T F F T F
the standard viewing window.
F T T F F
F F T T T KEYSTROKES: Y  () X, T,
, n  3 ENTER

61. A picture frame has right angles, so the angles () X, T,


, n  5 ENTER X, T,
, n  6
must be complementary angles. ZOOM 6
62. d ¬(x2  
x1)2 
(y2 
y1)2
Use the CALC menu to find the points of
¬
(7  2
)2  (1
9  7)2 intersection.
¬5
2  1
2 2

¬
169 or 13

Chapter 3 72
Find the intersection of a and t. Find the intersection of a and t.
KEYSTROKES: 2nd [CALC] 5 ENTER  KEYSTROKES: 2nd [CALC] 5 ENTER 
ENTER ENTER ENTER ENTER

Lines a and t intersect at (1.5, 4.5). Lines a and t intersect at (2.1, 7.3)
Find the intersection of b and t. Find the intersection of b and t.
KEYSTROKES: 2nd [CALC] 5  KEYSTROKES: 2nd [CALC] 5  ENTER
ENTER ENTER ENTER ENTER ENTER

Lines b and t intersect at (3.3, 6.9)


Lines b and t intersect at (5.5, 0.5).
5. Enter the equations in the Y  list and graph in
3. Enter the equations for a and b in the Y = list.
the standard viewing window.
Use the DRAW menu to graph t, since it is a
vertical line. KEYSTROKES: Y  4 5 X, T,
, n  2
KEYSTROKES: Y  6 ENTER 0 ENTER 2nd ENTER 4 5 X, T,
, n  7 ENTER ()
[DRAW] 4 () 2 ENTER 5 4 X, T,
, n ZOOM 6
Use the TRACE function to find the points of
Use the CALC menu to find the points of
intersection.
intersection.
Find the intersection of a and t.
Find the intersection of a and t.
KEYSTROKES: TRACE () 2 ENTER
KEYSTROKES: 2nd [CALC] 5 ENTER 
ENTER ENTER

Lines a and t intersect at (2, 6).


Find the intersection of b and t.
KEYSTROKES: TRACE  () 2 ENTER Lines a and t intersect at (1.0, 1.2).
Find the intersection of b and t.
KEYSTROKES: 2nd [CALC] 5  ENTER
ENTER ENTER

Lines b and t intersect at (2, 0).


4. Enter the equations in the Y  list and graph in
the standard viewing window.
KEYSTROKES: Y  () 3 X, T,
, n  1
Lines b and t intersect at (3.4, 4.3).
ENTER
() 3 X, T,
, n  3 ENTER 1
3 X, T,
, n  8 ZOOM 6
Use the CALC menu to find the points of
intersection.

73 Chapter 3
6. Enter the equations in the Y  list and graph in 6. y
the decimal viewing window.
KEYSTROKES: Y  () 1 6 X, T,
, n
O x
 2 3 ENTER () 1 6 X, T,
, n X

 5 12 ENTER 6 X, T,
, n 
Z
2 ZOOM 4 A
Use the CALC menu to find the points of Y
intersection. 1. Graph line  and point A. Place the compass
Find the intersection of a and t. point at point A. Make the setting wide enough
KEYSTROKES: 2nd [CALC] 5 ENTER  so that when an arc is drawn, it intersects  in
ENTER ENTER two places. Label these points X and Y.
2. Put the compass at point X and draw an arc
above line .
. AZ
4. Draw AZ   . The segment constructed
from point A(2, 6) perpendicular to line 
appears to intersect line  at (2, 4). Use the
Distance Formula to find the distance between
point A and .
Lines a and t intersect at (0.2, 0.7).
Find the intersection of b and t. d ¬(x
 2 x1)2 (y2 y1)2
KEYSTROKES: 2nd [CALC] 5  ENTER ¬
(2 
2)2 
[4 
(6)]2
ENTER ENTER ¬ 
(4)2  (2)2  16  4  20  4.47
The distance between A and  is about
4.47 units.
7. First, write an equation of a line p perpendicular
to the given lines. The slope of p is the opposite
reciprocal of 3 4
4 , or  3 . Use the y-intercept of

y  3
4 x  1, (0, 1), as one of the endpoints of the

Lines b and t intersect at (0.3, 0.5). perpendicular segment.
y  y1 ¬m(x  x1)
y  (1) ¬4
3 (x  0)

3-6 Perpendiculars and Distance
y  1 ¬4

3x

Page 162 Check for Understanding y ¬4


3x  1


1. Construct a perpendicular line between them. Next, use a system of equations to determine the
2. Sample answer: You are hiking and need to find point of intersection of y  3 1
4 x  8 and p.

the shortest path to a shelter. 3x  1 ¬4x  1
4 8 3
3. Sample answer: Measure distances at different
parts; compare slopes; measure angles. Finding 3x  4x ¬1  1
4 3 8
slopes is the most readily available method. 25
 x ¬9
12 8
4. L M 27
x ¬ 
50
y ¬3 27

4  50  8
   1
7
K N y ¬ 25

5. C
2
The point of intersection is 7
 7
50 ,  25 . 
Then, use the Distance Formula to determine the
B D
27
distance between (0, 1) and  
7
50 ,  25 .
 
d ¬(x2  
x1) 
2 (y2 y1)2

 2 
2 2
 0    (1)
27 7
A ¬ 
50  5
E
¬.81

¬0.9
The distance between the lines is 0.9 unit.

Chapter 3 74
8. x  3y ¬6 4. Draw 
PQ. 
PQ is perpendicular to y  3 1
4x  4.

3y ¬x  6 Label point R at the intersection of the lines.
y ¬1
3x  2
 The segment constructed from P(2, 5)
perpendicular to y  3 1
4 x  4 appears to

x  3y ¬14
intersect y  3
x  1 at R(1, 1).
4 4
3y ¬x  14 Use the Distance Formula to find the distance
y ¬1 1
3x  3
  4
between P and y  3 4x  4.
 1

First, write an equation of a line p perpendicular d ¬


(x2  
x1)2 
(y2 
y1)2
to the given lines. The slope of p is the opposite
¬ 
[2  (1)]2
 (5 
1)2
reciprocal of 1
3 , or 3. Use the y-intercept of
1
y   3 x  2, (0, 2), as one of the endpoints of the
 ¬3
2  42

perpendicular segment. ¬25 ¬5
y  y1 ¬m(x  x1) The distance between P and y  3 1
4 x  4 is 5

y  2 ¬3(x  0) units.
y  2 ¬3x
10.
y ¬3x  2
Next, use a system of equations to determine the water
point of intersection of y  1 1
4
3 x  3 and p.
  main
1 1 4
3x  3 ¬3x  2 connection
1
 14
3 x  3x ¬2  3

10 20
 3 x ¬ 3

x ¬2
y ¬3(2)  2 Pages 162–164 Practice and Apply
11. A B
y ¬4
The point of intersection is (2, 4).
Then, use the Distance Formula to determine the
distance between (0, 2) and (2, 4).
D C
d ¬
(x2  
x1)2 
(y2 
y1)2 12. J
¬(2 0)2 
(4  2)2
¬40

¬210
The distance between the lines is 2 10
, or L K
approximately 6.32 units. 13. R S
9. 3– 1–
y
y x
4 4 P (2, 5)

Q P
( 1, 1)
O x 14. W X

Z Y
1. Graph y  3  1
4 x  4 and point P. Place the
15. M G
compass at point P. Make the setting wide
enough so that when an arc is drawn, it
intersects y  3  1
4 x  4 in two places. Label
L H
these points of intersection A and B.
2. Put the compass at point A and draw an arc K J
below the line.
16. R
3. Using the same compass setting, put the S
Q
compass at point B and draw an arc to
intersect the one drawn in step 2. Label the T
point of intersection Q. X
U
W
V

75 Chapter 3
17. Graph line  and point P. Construct m 21. First, write an equation of a line p perpendicular
perpendicular to  through P. Line m appears to to the lines y  2x  2 and y  2x  3. The slope
intersect line  at (4, 0). Use the Distance of p is the opposite reciprocal of 2, or 1
2 . Use the
Formula to find the distance between P and . y-intercept of the line y  2x  2, (0, 2), as one of
d ¬(x2  x1)2 (y2  y1)2 the endpoints of the perpendicular segment.
¬(4  4)  (3
2  0) 2 y  y1 ¬m(x  x1)
¬93 y  2 ¬1
2 (x  0)

The distance between P and  is 3 units. y  2 ¬1

2x
y
y ¬1
2x  2

P (4, 3) Next, use a system of equations to determine the
point of intersection of the line y  2x  3 and p.
2x  3 ¬1 2x  2

O x
2x  1
2 x ¬2  3

5
2x ¬5
x ¬2
y ¬2(2)  3
18. Graph line  and point P. Construct m
perpendicular to  through P. Line m appears to y ¬1
intersect line  at (1, 3). Use the Distance The point of intersection is (2, 1).
Formula to find the distance between P and . Then, use the Distance Formula to determine the
d ¬
(x2  
x1)2 
(y2 
y1)2 distance between (0, 2) and (2, 1).
d ¬ (x2  x1)2  (y2 y1)2
¬
(4 
1)2  2
(4  3)
¬ (2  0) 2  (1 2)2
¬ (5)2 
 12  26
 ¬ (2)2  (1)2
The distance between P and  is 26
 units.
¬ 5
P y
The distance between the lines is 5  units.
22. First, write an equation of a line p perpendicular
m to the lines y  4x and y  4x  17. The slope of p
is the opposite reciprocal of 4, or 1 
4 . Use the
O x y-intercept of the line y  4x, (0, 0), as one of the
endpoints of the perpendicular segment.
y  y1 ¬m(x  x1)
y  0 ¬14 (x  0)

y ¬1
4x
19. The lines y  3 and y  1 are horizontal lines. Next, use a system of equations to determine the
The y-axis is perpendicular to these lines. point of intersection of the line y  4x  17 and p.
The y-intercept of y  3 is (0, 3). The y-axis 4x  17 ¬1 
4x
intersects the line y  1 at the point (0, 1). Use 17
17 ¬ 4 x
the Distance Formula to determine the distance
4 ¬x
between (0, 3) and (0, 1).
y ¬4(4)  17
d (x  2x1)2 (y2 y1)2 y ¬1
¬ (0  0
)2  [1 (3 )]2 The point of intersection is (4, 1).
¬0 2  42 Then, use the Distance Formula to determine the
¬ 16 or 4 distance between (0, 0) and (4, 1).
The distance between the lines is 4 units. d ¬
(x2  
x1)2 (y2 y1)2
20. The lines x  4 and x  2 are vertical lines. The ¬
(1  0) 
2 (4  0
)2
x-axis is perpendicular to these lines. The line ¬
(1)2 
 (4)2
x  4 intersects the x-axis at the point (4, 0). The
¬
17
line x  2 intersects the x-axis at the point
(2, 0). Use the Distance Formula to determine The distance between the lines is 17
 units.
the distance between (4, 0) and (2, 0).
d ¬ (x2  
x1)2 (y2 y1)2
¬ (2  4) 
2 (0  0)2
¬(6)
 2
 02   36 or 6
The distance between the lines is 6 units.

Chapter 3 76
23. 2x  y ¬4 3 4
4 x  5 ¬ 3 x  1

y ¬2x  4
3 4
4 x  3 x ¬1  5

y ¬2x  4
25
First, write an equation of a line p perpendicular 
12 x ¬6

to the lines y  2x  3 and y  2x  4. The slope 72
of p is the opposite reciprocal of 2, or 1
 x ¬
2 . Use the 25

4  25   5
y-intercept of the line y  2x  3, (0, 3), as one of y ¬3
 72

the endpoints of the perpendicular segment. 71
y ¬
y  y1 ¬m(x  x1) 25
The point of intersection is 
25 , 25 .
72
  71
y  (3) ¬1
2 (x  0)

Then, use the Distance Formula to determine the
y  3 ¬1 distance between (0, 1) and  25 , 25 .
 72 71
2x   
y ¬1
2x  3
 d ¬
(x2  
x1)2 
(y2 
y1)2


25  25  
Next, use a system of equations to determine the 2 2
¬  0  
72  (1)
71
point of intersection of the line y  2x  4 and p.
¬ 
2 2
2x  4 ¬1   
72 96
2x  3
 25 25
2x  1
2 x ¬3  4


57
¬  6 2
25 or 5
4

5x ¬7 24


2 The distance between the lines is 5 units.
14
x ¬ 5 25. y
y ¬2 5 4
14 ( 2, 5)

y ¬85
 ( 2, 4) y 5
The point of intersection is 
14

8
5 , 5 .
Then, use the Distance Formula to determine the
distance between (0, 3) and  5 ,  5 .
14 8 O x
d ¬
(x2  
x1)2 
(y2 
y1)2

 
0   5 
14 2 8 2
¬ 5   (3)
The distance from a line to a point not on the line
¬154 
 5
2 2 is the length of the segment perpendicular to the
7
 line from the point. From the figure, this
distance is 1.

19
¬  6 49
25  25

26. y

¬

24
5 y 2x 2
25 or 9.8

O x
The distance between the lines is 9.8
 units.
24. 3x  4y ¬20
4y ¬3x  20
( 3, 4)
y ¬34x  5

( 1, 5)
First, write an equation of a line p perpendicular
to the lines y  3 3
4 x  1 and y   4 x  5. The
 The perpendicular segment from the point
slope of p is the opposite reciprocal of 3 4
4 , or 3 . (1, 5) to the line y  2x  2 appears to
3
Use the y-intercept of the line y  4x  1, intersect the line y  2x  2 at (3, 4). Use the
(0, 1), as one of the endpoints of the Distance Formula to find the distance between
perpendicular segment. (1, 5) and y  2x  2.
y  y1 ¬m(x  x1) d ¬ (x2  
x1)2  (y2 y1)2
¬ [1 (3)] 2  [5  (
4)]2
y  (1) ¬4
3 (x  0)

¬2 2  (
1) 2

y  1 ¬4

3x ¬ 5
y ¬4 The distance between the line y  2x  2 and the
3x  1

Next, use a system of equations to determine point (1, 5) is  5 units.
the point of intersection of the line y  3
4x  5

and p.

77 Chapter 3
27. 2x  3y ¬9 32b. 1
3y ¬2x  9 P
y ¬2
3x  3

2
y
32c.
y 2–
3
x 3 Q1 P Q2
(0, 3)
P

O (2, 0) x

32d.
The perpendicular segment from the point (2, 0) P
to the line y  2
3 x  3 appears to intersect the
 Q1
line y  2
3
x  3 at (0, 3). Use the Distance
Formula to find the distance between (2, 0) and
y  2
Q2
3 x  3.
 P
d ¬
(x2  
x1)2 (y2 
y1)2
¬
(2  0
)2  (0 3)2 32e.

¬ 2
22  (3) Q1 P Q2
¬13

The distance between the line y  2
3 x  3 and the

point (2, 0) is 13
 units.
28. Given:  is equidistant to m. 32f.
n is equidistant to m.
Q1 P Q2
Prove:  n

m
n
33. Sample answer: We want new shelves to be
Paragraph proof: We are given that  is parallel so they will line up.
equidistant to m, and n is equidistant to m. By Answers should include the following.
definition of equidistant,  is parallel to m, and n • After marking several points, a slope can be
is parallel to m. By definition of parallel lines, calculated, which should be the same slope as
slope of   slope of m, and slope of n  slope of the original brace.
m. By substitution, slope of   slope of n. Then,
• Building walls requires parallel lines.
by definition of parallel lines,  n.
29. It is everywhere equidistant from the ceiling. 34. A
30. The plumb line will be vertical and will be C
perpendicular to the floor. The shortest distance from
a point to the floor will be along the plumb line.
31. a  3, b  4, c  6, (x1, y1)  (4, 6) X
|ax1  by1  c| |3(4)  4(6)  6|
 ¬ 
a2  b2 32  42
|30|
¬  B D
25 AB  16 and X is the midpoint of A
B, so XB  8.
30
¬  CD  20 and X is the midpoint of C
D, so XD  10.
5 or 6
D
B A B
, so XBD is a right triangle. Use the
32a. 1
Pythagorean Theorem to find BD.
P p (XD)2 ¬(XB)2  (BD)2
102 ¬82  (BD)2
2
36 ¬(BD)2
6 ¬BD

Chapter 3 78
35. D; The coin came up heads 14 times, but since the 44. Given: NL ¬NM, AL ¬BM
first and last flips were both heads and there Prove: NA ¬NB
were 24 total flips, it’s not possible to have all L M
14 times heads came up be consecutive. But it is
possible that the first 13 flips were heads, or the A B
last 13 flips were heads.
N
Proof:
Page 164 Maintain Your Skills
36.  CF
DE ; alternate interior  Statements Reasons
37.  
DA EF; corresponding  1. NL  NM 1. Given
38. 
DA 
EF; 1  4 and consecutive interior  are AL  BM
supplementary 2. NL  NA  AL 2. Segment Addition
39. Find the slope of a using (0, 3) and (2, 4). NM  NB  BM Post.
(y  y )
m ¬
2
1
3. NA  AL  NB  BM 3. Substitution
(x2  x1)
4 3 4. NA  BM  NB  BM 4. Substitution
¬ 1
2  0 or 2
5. NA  NB 5. Subt. Prop.
y ¬mx  b
y ¬1
2x  3
 Pages 165–166 Geometry Activity:
40. Find the slope of b using (0, 5) and (1, 4). Non-Euclidean Geometry
(y  y ) 1. The great circle is finite.
m ¬
2
1
2. A curved path on the great circle passing through
(x2  x1)
4 5
two points is the shortest distance between the
¬
1  0 or 1 two points.
y ¬mx  b 3. There exist no parallel lines.
y ¬x  5 4. Two distinct great circles intersect in exactly two
41. Find the slope of c using (0, 2) and (3, 0). points.
(y  y ) 5. A pair of perpendicular great circles divides the
m ¬
2
1
(x2  x1) sphere into four finite congruent regions.
0  ( 2) 2 6. There exist no parallel lines.
¬ 
3  0 or 3
7. There are two distances between two points.
y ¬mx  b
8. true
y ¬2
3x  2

9. False; in spherical geometry, if three points are
42. Line a has slope 1
2 (from Exercise 39), so a line collinear, any point can be between the other two.
perpendicular to a has slope 2. 10. False; in spherical geometry, there are no parallel
y  y1 ¬m(x  x1) lines.
y  (4) ¬2[x  (1)]
y  4 ¬2x  2
y ¬2x  6 Chapter 3 Study Guide and Review
43. Line c has slope 2
3 (from Exercise 41), so a line
parallel to c has slope 2

3.
Page 167 Vocabulary and Concept Check
y  y1 ¬m(x  x1) 1. alternate
2. perpendicular
y  5 ¬2
3 (x  2)

3. parallel
y  5 ¬2
 4
3x  3 4. transversal
y ¬2 11
3x  3
  5. alternate exterior
6. congruent
7. consecutive

Pages 167-170 Lesson-by-Lesson Review


8. corresponding
9. alternate exterior
10. consecutive interior
11. corresponding
12. alternate interior

79 Chapter 3
5  (
13. consecutive interior 25. slope of 
AB ¬ 
41
3)
14. alternate exterior
¬8

3
15. alternate interior
2  (1)
16. 1 and 2 are supplementary. slope of 
CD ¬ 7  1
m1  m2 ¬180 ¬3
8

53  m2 ¬180 The product of the slopes is 8
3  3  or 1.
 8
m2 ¬127 So, 
AB is perpendicular to 
CD.
17. 3 ¬6  3 
26. slope of AB ¬ 6  2
 0
6 ¬1
3 ¬1 ¬3

4
m3 ¬m1 1 
slope of CD ¬ (4)
3  (1)
m3 ¬53
18. m4  m3 ¬180 ¬3

4
 and CD
The slopes are the same, so AB  are
m4  53 ¬180
m4 ¬127 parallel.
19. m5  m6  180 
27. First, find the slope of AB.
(y2  y1)
6 ¬1 m ¬
(x2  x1)
m6 ¬m1
62
m5  m1 ¬180 ¬1  (1 )
m5  53 ¬180 4
¬2 or 2
m5 ¬127
Parallel lines have the same slope, so the slope of
20. 6 ¬1 the line to be drawn is 2.
m6 ¬m1 Graph the line. Start at (2, 3). Move up 2 units
m6 ¬53 and then move right 1 unit. Draw the line
21. m7  m6 ¬180 through this point and (2, 3).
m7  53 ¬180 y
m7 ¬127
22. Find a. (2, 3)
1 and 2 are supplementary, so
m1  m2  180.
3a  40  2a  25 ¬180 O x
5a  65 ¬180
5a ¬115
a ¬23
Find b.
.
28. First, find the slope of PQ
2  4 and m4  m3  180, so
m2  m3  180. (y  y )
m ¬
2
1
2a  25  5b  26 ¬180 (x2  x1)
2(23)  25  5b  26 ¬180 4 2
¬ 35
5b  45 ¬180 6
5b ¬135 ¬2 or 3
b ¬27 Since 313   1, the slope of the line to be

1
 ¬   1
23. slope of AB 3  (4) drawn is 1 3 . Graph the line. Start at (2, 2).

Move down 1 unit and then move right 3 units.
¬2

7 Draw the line through this point and (2, 2).
 ¬
9  2
slope of CD 02 y
¬7

2
The slopes are not the same, so 
AB and 
CD are
not parallel. The product of the slopes is 2
7  2 
 7
 
or 1, so AB and CD are not perpendicular. O x
 and CD
Therefore, AB  are neither parallel nor
perpendicular. ( 2, 2)
2
2 
24. slope of 
AB ¬ 26
4
¬
4 or 1 29. y  y1 ¬m(x  x1)
 ¬2  (
4)
slope of CD 5  (1) y  (5) ¬2 (x  1)
y  5 ¬2x  2
¬6

6 or 1
 and CD
 are y ¬2x  7
The slopes are the same, so AB
parallel.

Chapter 3 80
(y  y )
30. m ¬
2
1
d ¬
(x2  
x1)2 
(y2 
y1)2
(x2  x1)
1  5 ¬
(2 
0)2 
[3 
(4)]2
¬ 
2  2
6 3 ¬ 
(2)2  12
¬
4 or 2
y  y1 ¬m (x  x1) ¬5 
The distance between the lines is 5
 units.
y  5 ¬3
2 (x  2)

42. First, write an equation of a line p perpendicular
3
y  5 ¬2x  3
to y  1 1
2 x and y  2 x  5. The slope of p is the

y ¬3
2x  2
 opposite reciprocal of 1
2 , or 2. Use the

1
31. y ¬mx  b y-intercept of y  2 x, (0, 0), as one of the

y ¬2
7x  4
 endpoints of the perpendicular segment.
y  y1 ¬m(x  x1)
32. y  y1 ¬m(x  x1) y  0 ¬2 (x  0)
y  (4) ¬3 y ¬2x
2 (x  2)

Next, use a system of equations to determine the
y  4 ¬3
2x  3

point of intersection of the line y  1
2 x  5 and p.

y ¬3
2x  1
 1x  5 ¬2x
2
33. y ¬mx  b 5 ¬2x  1

2x
y ¬5x  3 5 ¬5
x
2
(y  y )
34. m ¬
2
1 2 ¬x
(x2  x1)
y ¬1
2 (2)  5

¬ 64 (
 1)
3 y ¬4
7
¬7 or 1
The point of intersection is (2, 4).
y  y1 ¬m(x  x1) Then, use the Distance Formula to determine the
y  (1) ¬1(x  3) distance between (0, 0) and (2, 4).
y  1 ¬x  3
d ¬
(x2  
x1)2 
(y2 
y1)2
y ¬x  2
35. 
AL and 
BJ, alternate exterior  are  ¬
(2 
0)2 
(4  0)2
36.  and BJ
AL , consecutive interior  are ¬ 
(2)2  42
supplementary
¬20  25 
37.  and GK
CF , 2 lines  to the same line
The distance between the lines is 20
 units.
38.  
AL and BJ, alternate interior  are 
39. 
CF and 
GK, consecutive interior  are
supplementary
40. 
CF and 
GK, corresponding  are 
Chapter 3 Practice Test
41. First, write an equation of a line p perpendicular
Page 171
to y  2x  4 and y  2x  1. The slope of p is the
opposite reciprocal of 2, or 1 1. The slope of a line perpendicular to y  3x  2

7 is
2 . Use the y-intrecept
of y  2x  4, (0, 4), as one of the endpoints of the opposite reciprocal of 3, or 1
.
3
the perpendicular segment. Sample answer: y  1 3x  1


y  y1 ¬m(x  x1) 2. Sample answer: If alternate interior  are ,


y  (4) ¬1 then lines are .
2 (x  0)

3. 2 and 6 each form a linear pair with 1, so 2
y  4 ¬1

2x and 6 are supplementary to 1.
y ¬1
2x  4
 3  1, so 3 is not necessarily supplementary
Next, use a system of equations to determine the to 1. m3  m4  m5  180, so
point of intersection of the line y  2x  1 and p. m1  m4  m5  180. So 4 and 5 are not
2x  1 ¬1 supplementary to 1.
2x  4

2x  1
x ¬4  1 4. m8  m12 ¬180
2
m8  64 ¬180
5x ¬5
2 m8 ¬116
x ¬2
5. 13 ¬12
y ¬2(2)  1
m13 ¬m12
y ¬3
m13 ¬64
The point of intersection is (2, 3).
Then use the Distance Formula to determine the 6. 7 ¬12
distance between (0, 4) and (2, 3). m7 ¬m12
m7 ¬64

81 Chapter 3
1  5
7. m11  m12 ¬180 14. slope of 
FG ¬ 
3  3
m11  64 ¬180 6
m11 ¬116 ¬
6 or 1
8. m3  m4 ¬180 Start at (1, 1). Move 1 unit up and then move
4 ¬12 1 unit right. Draw the line through this point and
m4 ¬m12 (1, 1).
m3  m12 ¬180 y F (3, 5)
m3  64 ¬180
m3 ¬116
9. 4 ¬12
m4 ¬m12
m4 ¬64 O x
10. m9  m10  180 G M(1, 1)
( 3, 1)
10 ¬12
m10 ¬m12
m9  m12 ¬180 15. Start at (3, 2). Move 4 units up and then move 3
m9  64 ¬180 units left. Draw the line through this point
m9 ¬116 and (3, 2).
11. 5 ¬7 y
7 ¬12
5 ¬12
m5 ¬m12
m5 ¬64
O x
12. Start at (2, 1). Move 1 unit down and then move K(3, 2)
1 unit right. Draw the line through this point and
(2, 1).
y
16. ABD  ACE
mACE  mECF ¬180
mABD  mECF ¬180
( 2, 1)
3x  60  2x  15 ¬180
O x
5x  45 ¬180
5x ¬225
x ¬45
17. DBC ¬ECF
30 mDBC ¬mECF
13. slope of 
AB ¬ 
4  (2) y ¬2(45)  15
y ¬105
¬3
 1
6 or 2
18. mFCF ¬2x  15
1
2 (2)  1, so the slope of the line to be graphed

¬2(45)  15
is 2. ¬105
Start at (1, 3). Move 2 units down and then
move 1 unit right. Draw the line through this 19. mABD ¬3x  60
point and (1, 3). ¬3(45)  60
¬75
y
B(4, 3) 20. mBCE  mFCE ¬180
Q ( 1, 3) mBCE  105 ¬180
mBCE ¬75
A( 2, 0)
21. mCBD ¬y
O x
¬105

Chapter 3 82
22. The slope of a line perpendicular to y  2x  1 24. The slope of a line perpendicular to y  x  4
and y  2x  9 is 1 
2 . Use the y-intercept of and y  x is 1. Use the y-intercept of y  x,
y  2x  1, (0, 1), as one of the endpoints of the (0, 0), as one of the endpoints of the perpendicular
perpendicular segment. segment.
y  y1 ¬m(x  x1) y  y1 ¬m(x  x1)
y  (1) ¬1 y  0 ¬1(x  0)
2 (x  0)

y ¬x
y  1 ¬1

2x Use a system of equations to determine the point
y ¬1
2x  1
 of intersection of the line y  x  4 and the
perpendicular segment.
Use a system of equations to determine the point
x  4 ¬x
of intersection of the line y  2x  9 and the
4 ¬2x
perpendicular segment.
2 ¬x
2x  9 ¬1
2x  1
 y ¬(2)  4
2x  1 y ¬2
2 x ¬1  9

The point of intersection is (2, 2).
5x ¬10
2 Use the Distance Formula to determine the
x ¬4
y ¬2(4)  9 distance between (0, 0) and (2, 2).
y ¬1 d ¬ (x2  x1)2 
(y2 
y1)2
The point of intersection is (4, 1). ¬ (2  0) 
2 (2 0)2
Use the Distance Formula to determine the ¬(2)
 2  (2) ¬
2 8
distance between (0, 1) and (4, 1). ¬2.83
d ¬
(x2  
x1)2 
(y2 
y1)2 The distance between the lines y  x and
y  x  4 is about 2.83 units. So, the distance
¬
(4 
0)2  2
[1  (1)] between Lorain Road and Detroit Road is about
2.83 miles.
¬
(4)2 
 22
25. B; 1  3, and 3 and 4 are supplementary so
¬20
 1 and 4 are supplementary.
The distance between the lines is 20  or about m4  180  m1 so m4  107 and hence
4.47 units. m4 73. Furthermore, 1 is not congruent to
23. The slope of a line perpendicular to y  x  4 4. Lines m and  are parallel so consecutive
and y  x  2 is 1. Use the y-intercept of interior angles are supplementary. Hence,
y  x  4, (0, 4), as one of the endpoints of the m2  m3  180. So the only statement that
perpendicular segment. cannot be true is B.
y  y1 ¬m (x  x1)
y  4 ¬1(x  0)
y  4 ¬x Chapter 3 Standardized Test Practice
y ¬x  4
Use a system of equations to determine the point Pages 172–173
of intersection of the line y  x  2 and the 1. B; 2 m  200 cm
perpendicular segment. 2. C; d ¬(x
 2x1)2 
(y2 
y1)2
x  2 ¬x  4
¬ (2  2)2 (3 4)2
2x ¬6
x ¬3 ¬(4
 )2
 (7)2 ¬65

y ¬( 3)  2 3. A; statement A is true by definition of angle
y ¬1 bisector. There is not enough information to
The point of intersection is (3, 1). establish that any of the other statements are
Use the Distance Formula to determine the true.
distance between (0, 4) and (3, 1). 4. D; 180  72  108
d ¬ (x2  
x1)2 (y2 y1)2 5. D; 4x  4 ¬6x  8
4  8 ¬6x  4x
¬(3  0)2 (1  4
)2
12 ¬2x
¬(3)
 2  (3)
2
6 ¬x
¬18 6. C
The distance between the lines is 18
 or about
7. B; 1 and 3 are corresponding angles.
4.24 units.

83 Chapter 3
(y  y )
8. C; 4y  x ¬8 15a. m ¬
2
1
(x2  x1)
4y ¬x  8
48 44
¬
y ¬1
4x  2
 21
¬4
14(4)  1, so the slope of the perpendicular 15b. The slope represents the increase in the average
line is 4. y  4x  15 is the only choice with monthly cable bill each year.
slope 4. 15c. The slope of the line through the points is 4. One
9. C; the number 2 in y  2x  5 is the slope, which point on the line is (1, 44). Find the equation of
determines the steepness of the line. the line.
4x 
10. 3 3  3(10)
6 y  44 ¬4(x  1)
y  44 ¬4x  4
11. mFHC  mFHD  180
y ¬4x  40
FHD  HGB
mFHD  mHGB After 10 years the cable bill will be
mFHC  mHGB ¬180 y  4(10)  40, or $80.
mFHC  70 ¬180
mFHC ¬110
The flag holder rotates 110°.
12. CHG ¬HGB
mCHG ¬mHGB
mCHG ¬70
(y  y )
13. m ¬
2
1
(x2  x1)
6  4
¬93
¬2
6 or 3
1
14. The ball did not reach home plate. The distance
between second base and home plate forms the
hypotenuse of a right triangle, with second base
to third base as one leg, and third base to home
plate as the other leg. The Pythagorean Theorem
is used to find the distance between second base
and home plate.
902  902 ¬c2
8100  8100 ¬c2
16,200 ¬c2
 16,200 ¬c
127.3 ¬c
Since the ball traveled 120 feet and the distance
from second base to home plate is about 127.3
feet, the ball did not make it to home plate.

Chapter 3 84
Chapter 4 Congruent Triangles
Page 177 Getting Started 10. 12 is supplementary to 4 linear pair
1. 2x  18 ¬5 2. 3m 16 ¬12 12 is supplementary to 16 linear pair
2x ¬13 3m ¬28 12 is supplementary to 11 12  3
x ¬13
or 6 1 m ¬28
 1 corresponding ,
2 2 3 or 9 3 3 is supplemen-
3. 4y  12 ¬16 4. 10 ¬8  3z tary to 11
4y ¬4 2 ¬3z linear pair
y ¬1 2
3 ¬z
 12 is supplementary to 14 12 is supple-
mentary to 16
5. 6 ¬2a  1

2 6. 2
3 b  9 ¬15

linear pair,
11
  ¬2a 2b ¬24
2 3 16  14
11 corresponding 
4 ¬a b ¬36
 
12 is supplementary to 5 12 is supple-
2 3
4 ¬a
 mentary to 10
7. 8  2 vertical  consecutive
8  12 corresponding  interior ,
8  15 alternate exterior  10  5
8  6 corresponding  alternate
8  9 alternate exterior  exterior 
8  3 8  15 alternate exterior , 12 is supplementary to 1 12 is supple-
15  3 alternate interior , mentary to 10
transitivity consecutive
8  13 8  15 alternate exterior , interior ,
15  13 corresponding , 10  1
transitivity corresponding 
8. 13  2 13  15 corresponding , 12 is supplementary to 7 12  8
15  2 corresponding , corresponding ,
transitivity 8 is supplemen-
13  12 alternate exterior  tary to 7
13  15 corresponding  linear pair
13  6 alternate exterior  12 is supplementary to 10 consecutive
13  9 corresponding  interior 
13  3 vertical  11. d ¬(x
x2  
1)2 (y 2 y 1)
2

13  8 13  15 corresponding , ¬(4


 
6)2 )(3  82
15  8 alternate exterior , ¬(10)
25)  (2 ¬125 
transitivity ¬11.2
9. 1 is supplementary to 6 linear pair d ¬(x
x2 1)  (y2 
12. 2 y1)2
1 is supplementary to 9 linear pair
¬[6
15)]
 (28  (1)  122
1 is supplementary to 3 consecutive
¬21
 2 
62 ¬477

interior 
¬21.8
1 is supplementary to 13 1 is supplemen-
tary to 3, 13. d ¬(x
x2 1) 
2 (y2  y1)2
3  13 ¬(3
 
11)2  [4  (8)]2
vertical  ¬(14)
2  42 ¬212
1 is supplementary to 2 consecutive ¬14.6
interior 
1 is supplementary to 8 1 is supplemen-
tary to 2,
2  8 4-1 Classifying Triangles
vertical 
1 is supplementary to 12 1  10
corresponding ,
Page 179 Geometry Activity: Equilateral
10 is supplemen- Triangles
tary to 12 1. Yes, all edges of the paper are an equal length.
consecutive 2. See students’ work.
interior  3. See students’ work.
1 is supplementary to 15 1  14
corresponding ,
14 supplementary
to 15 consecutive
interior 

85 Chapter 4
Pages 180–181 Check for Understanding 12. 8 scalene triangles (green), 8 isosceles triangles in
1. Triangles are classified by sides and angles. For the middle (blue), 4 isosceles triangles around the
example, a triangle can have a right angle and middle (yellow), and 4 isosceles triangles at the
have no two sides congruent. corners of the square (purple).
2. Sample answer:

Pages 181–183 Practice and Apply


13. The triangle has a right angle, so it is a right
triangle.
3. Always; equiangular triangles have three acute
14. The triangle has three acute angles, so it is an
angles.
acute triangle.
4. Never; right triangles have one right angle and
15. The triangle has three acute angles, so it is an
acute triangles have all acute angles.
acute triangle.
5. The triangle has one angle with measure greater
16. The triangle has one angle with measure greater
than 90, so it is an obtuse triangle.
than 90, so it is an obtuse triangle.
6. The triangle has three congruent angles, so it is
17. The triangle has one angle with measure greater
an equiangular triangle.
than 90, so it is an obtuse triangle.
7. MJK is obtuse because mMJK  90.
18. The triangle has a right angle, so it is a right
KLM is obtuse because mKLM  mMJK and
triangle.
mMJK  90.
JKN is obtuse because mJNK  180  19. The triangle has three congruent sides and three
mJNM  180  52 or 128. congruent angles, so it is equilateral and
LMN is obtuse because mLNM  180  equiangular.
mJNM  180  52 or 128. 20. See students’ work.
8. GHD is a right triangle because G HD F
 so 21. The triangles have two congruent sides, so they
GHD is a right angle. are isosceles. The triangles have three acute
GHJ is a right triangle because G HD F
 so angles, so they are acute.
GHJ is a right angle. 22. BGD is a right angle, so A DB C. Then AGB,
IJF is a right triangle because IJ
G H, AGC, DGB, and DGC are right angles, so
H
G D F  so IJ
DF and IJF is a right angle. AGB, AGC, DGB, and DGC are right
EIG is a right triangle because G I  E
F
 so triangles.
EIG is a right angle. 23. BAC and CDB are obtuse, so BAC and
M
9. JM N  so JM  MN. CDB are obtuse triangles.
2x  5 ¬3x  9 24. AGB, AGC, DGB, and DGC are scalene
5  9 ¬3x  2x because they each have no congruent sides.
4 ¬x
JM ¬2x  5 25. ABD, ACD, BAC, and CDB are isosceles
¬2(4)  5 ¬3 triangles because they each have two congruent
MN ¬3x  9 sides.
¬3(4)  9 ¬3 G
26. H J G
 so HG  JG.
JN ¬x  2 x  7 ¬3x  5
¬4  2 ¬2 7  5 ¬3x  x
10. Since QRS is equilateral, each side has the 12 ¬2x
same length. So QR  RS. 6 ¬x
4x ¬2x  1 GH ¬x  7
2x ¬1 ¬6  7 ¬13
x ¬1
 GJ ¬3x  5
2 ¬3(6)  5 ¬13
QR ¬4x
HJ ¬x  1
 
¬4 1
2 ¬2
 ¬6  1 ¬5
RS ¬2x  1 27. MPN is equilateral so all three sides are
 
¬2 1
2  1 ¬2
 congruent.
QS ¬6x  1 MN ¬MP
3x  6 ¬x  4
 
¬6 1
2 1 ¬2

3x  x ¬4  6
11. TW ¬(4)  25
2  (  6)2 2x ¬10
¬4 12125
1  x ¬5
WZ ¬(3
 4)2 5)]
[0  (2 MN ¬3x  6
¬49
5  2  74 ¬3(5)  6  9
TZ ¬(3
 2)2 )
(0  62 MP ¬x  4
¬25
6  3  61 ¬5  4  9
TWZ is scalene because no two sides are NP ¬2x  1
congruent. ¬2(5)  1  9

Chapter 4 86
28. QRS is equilateral so all three sides are 32. AB ¬(3 )  51
2  (  4)2
congruent. ¬4  25  29 
QR ¬2x  2 BC ¬(7)  31
2  [1)] (2
RS ¬x  6 ¬16
  0  16  or 4
QS ¬3x  10 AC ¬(7)  51
2  (  4)2
QR ¬RS ¬4  25  29 
2x  2 ¬x  6 B
Since A  and A C have the same length, ABC is
2x  x ¬6  2 isosceles.
x ¬8 33. AB ¬[5 4)]
 ( 2 (6 1)2
QR ¬2x  2 ¬815  2  106 
¬2(8)  2 ¬14 BC ¬(3
  5)2  (7  6)2
RS ¬x  6 ¬6469  1  233 
¬8  6 ¬14 AC ¬[3
  (4)]27 (  1)2
QS ¬3x  10 ¬1   64  65 
¬3(8)  10 ¬14 ABC is scalene because no two sides are
29. JL ¬2x  5 congruent.
JK ¬x  3 34. AB ¬[7  (7)]21 ()  92
KL ¬x  1 ¬0 0  10  100  or 10
J
K  ¬L
J BC ¬[47)]
 ( 2 [11)]
 (2
KJ ¬LJ ¬121
  0  121 or 11
x  3 ¬2x  5 AC ¬[47)]
 ( 2 (1  9)2
3  5 ¬2x  x ¬121
  100  221
8 ¬x ABC is scalene because no two sides are
JL ¬2x  5 congruent.
¬2(8)  5 ¬11
35. AB ¬[2 3)]
 ( 2 [1 (1)]2
JK ¬x  3
¬25  4  29
¬8  3 ¬11
BC ¬(2 )  23
2  (  1)2
KL ¬x  1
¬0   16  16 or 4
¬8  1 ¬7
AC ¬[23)]
 ( 2 [31)]
 (2
30. P is the midpoint of MN, so MP  PN  1 
2 (24) or ¬25  4  29
12. OPM N, so MPO and NPO are right B
Since A  and A C have the same length, ABC is
triangles. Use the Pythagorean Theorem to find isosceles.
MO and NO.
36. AB ¬ (53 
 0)2 
 (2  5)2
(MO)2 ¬(MP)2  (OP)2
¬75  9  84 
(MO)2 ¬122  122
BC ¬ (0  53)2 
 (1  2)2
(MO)2 ¬288
¬75  9  84 
MO ¬288 
AC ¬(0)  01
2  ()  52
(NO)2 ¬(PN)2  (OP)2
¬0   36  36  or 6
(NO)2 ¬122  122
B
Since A  and B C have the same length, ABC is
(NO)2 ¬288
isosceles.
NO ¬288
MPO and NPO are not equilateral because 37. AB ¬ [5  (9)]2 (63  0)2
MO  NO  288 . ¬1608  1  124 
31. Let x be the distance from Lexington to Nashville. BC ¬ [1  (5)]2 (0 63 
 )2
Then the distance from Cairo to Nashville is ¬1608  1  124 
x  40, the distance from Cairo to Lexington is AC ¬[1
  (9)]2  (0 0)2
x  81, and (x  40)  (x  81)  x  593 ¬64  0  64  or 8
3x  41 ¬593 B
Since A  and B C have the same length, ABC is
3x ¬552 isosceles.
x ¬184
x  40 ¬184  40 or 144
x  81 ¬184  81 or 265
The triangle formed is scalene; it is 184 miles
from Lexington to Nashville, 265 miles from Cairo
to Lexington, and 144 miles from Cairo to
Nashville.

87 Chapter 4
38. Given: EUI is equiangular. 42. Use the Distance Formula and Slope Formula.
L
QU I KL ¬(4 ) 2 2  (2
 6)2
Prove: EQL is equiangular. ¬4   16
E ¬20
L
KL M
, so LM  20 .
2 6
L
slope of K ¬42
4
Q L ¬ 2 or 2
KLM is a right angle, so K LL M
 and L
M
 has
slope 1

2.
U I
Let (x2, y2) be the coordinates of M.
Proof: 2 y 2
M
slope of L ¬ 
Statements Reasons x2  4
y2  2
1. EUI is equiangular. 1. Given 1 ¬ 
2 x2  4
L
QU
I x2  4 ¬2(y2  2)
2. E  EUI  EIU 2. Definition of equi- LM ¬(x 4)
2  (y
2  2)
2
2
angular triangle 20
 ¬[2(y  2)]2  (y  2)2
2 2
3. EUI  EQL 3. Corresponding  20
 ¬4(y  2  2) 
2 (y2  2)2
EIU  ELQ are . 20
 ¬5(y  2  2)2

4. E  E 4. Reflexive Property 20 ¬5(y2  2)2


4 ¬(y2  2)2
5. E  EQL  ELQ 5. Transitive 2 ¬y2  2 or 2  y2  2
Property 4 ¬y2 or 0  y2
x2 ¬2(y2  2)  4
6. EQL is equiangular. 6. Definition of equi-
x2 ¬2(4  2)  4 or x2  2(0  2)  4
angular triangles
x2 ¬8 or x2  0
39. Given: mNPM  33 M has coordinates (8, 4) or (0, 0).
Prove: RPM is obtuse. 43. Sample answer: Triangles are used in
M N construction as structural support. Answers
33 should include the following.
P • Triangles can be classified by sides and angles.
If the measure of each angle is less than 90, the
triangle is acute. If the measure of one angle is
R
greater than 90, the triangle is obtuse. If one
Proof: NPM and RPM form a linear pair. angle equals 90°, the triangle is right. If each
NPM and RPM are supplementary because if angle has the same measure, the triangle is
two angles form a linear pair, then they are
equiangular. If no two sides are congruent, the
supplementary. So, mNPM  mRPM  180. It
triangle is scalene. If at least two sides are
is given that mNPM  33. By substitution,
congruent, it is isosceles. If all of the sides are
33  mRPM  180. Subtract to find that
congruent, the triangle is equilateral.
mRPM  147. RPM is obtuse by definition.
• Isosceles triangles seem to be used more often
RPM is obtuse by definition.
in architecture and construction.
40. TS  [7
 (4)]2 (8 14)2
44. C; AB ¬[1 1)]
 ( 2 (3  1)2
 9  36 or 45
¬4  4  8 
SR  [10
 )](722  (  8)2
BC ¬(3)  112  ()  32
 9  36 or 45
¬4  16  20 
S is the midpoint of RT
.
AC ¬[31)]
 ( 2 (1  1)2
UT  [0 4)]
 ( 2 (8 14)2
¬16
  4  20 
 166  3 or 52
VU  (4 ) 0 2  (2
 8)2 Since AC and B C
 have the same length, ABC is
 166  3 or 52 isosceles.
U is the midpoint of TV. y

 0  a2  b)
B
  (0
2
41. AD ¬ 2

A
¬ 2  (b )   a4  b
2 2
a 2 2 x

CD ¬ 2
  (0  b)
2 C
a  a 2

¬ 2  (b)   a4  b


2 2
a 2 2

AD  CD, so A
D
C
D
. ADC is isosceles by All three angles of ABC are acute, so ABC is
definition. acute.

Chapter 4 88
x  y  15  35
45. B;   ¬25 The perpendicular segment from the point (6, 2)
4
to the line y  7 intersects the line y  7 at (6, 7).
x  y  50 ¬100
Use the Distance Formula to find the distance
x  y ¬50
between (6, 2) and y  7.
x  15  35
 ¬27 d ¬(xx2 1)  (y2 
2 y1)2
3
x  50 ¬81 ¬(6)  62
2  (  7)2
x ¬31 ¬0   81
Substituting 31 for x, ¬81 or 9
31  y ¬50 The distance between the line y  7 and the point
y ¬19 (6, 2) is 9 units.
49.
A
C 110˚
p
Page 183 Maintain Your Skills B
46. D E
y q
F
(4x 10)˚
y x 2
Explore: From the figure, you know that
mABC  110 and mDEF  4x  10.
O x
(2, –2)
Plan: For line p to be parallel to line q,
corresponding angles must be congruent, so
ABC  AED. Since AED is supplementary to
The perpendicular segment from the point (2, 2) DEF because they are a linear pair, it must be
to the line y  x  2 appears to intersect the line true that p  q if ABC is supplementary to
y  x  2 at (1, 1). Use the Distance Formula to DEF, or mABC  mDEF  180. Substitute
find the distance between (2, 2) and y  x  2. the given angle measures into this equation and
d ¬(xx2 1) 
2 (y2 y1)2 solve for x.
¬(1
 2) 2)]
2 [1  (2 Solve: mABC  mDEF ¬180
¬9 9 110  4x  10 ¬180
¬18
 4x ¬60
The distance between the line y  x  2 and the x ¬15
point (2, 2) is 18
 units. Examine: Verify the measure of DEF by using
47. y the value of x. That is, 4x  10  4(15)  10 or 70,
(3, 3) and 110  70  180. Since mABC  mDEF
 180, ABC is supplementary to DEF and p  q.
x y 2 50. p q

O x A
(3x 50)˚ E F
C B (2x 5)˚
xy2
y  x  2 D
The perpendicular segment from the point (3, 3)
to the line x  y  2 appears to intersect the line Explore: From the figure, you know that
x  y  2 at (1, 1). Use the Distance Formula to mABC  3x  50 and mDEF  2x  5. You
find the distance between (3, 3) and x  y  2. also know that ABC and DEF are alternate
d ¬(xx2 1) 
2 (y2  y1)2 exterior angles.
¬(3)  1 2  (3 1)2 Plan: For line p to be parallel to line q, the
¬4  4 alternate exterior angles must be congruent. So
¬8  mABC  mDEF. Substitute the given angle
The distance between the line x  y  2 and the measures into this equation and solve for x.
point (3, 3) is 8  units. Solve: mABC ¬mDEF
48. 3x  50 ¬2x  5
y
x ¬45
Examine: Verify the angle measures by using the
y 7 value of x to find mABC and mDEF.
mABC ¬3x  50
¬3(45)  50
¬85
mDEF ¬2x  5
O x ¬2(45)  5
¬85
(6, –2) Since mABC  mDEF, ABC  DEF and p  q.

89 Chapter 4
51. A
57˚ 4-2 Angles of Triangles
p
B C
E q
D Pages 188–189 Check for Understanding
F (3x 9)˚ 1. Sample answer: 2 and 3 are the remote
Explore: From the figure, you know that interior angles of exterior 1.
mABC  57 and mDEF  3x  9. 2
Plan: For line p to be parallel to line q,
corresponding angles must be congruent, so 1 3
ABC  AED. Since AED is supplementary to
DEF, because they are a linear pair, it must be 2. Najee; the sum of the measures of the remote
true that p  q if ABC is supplementary to interior angles is equal to the measure of the
DEF, or mABC  mDEF  180. Substitute corresponding exterior angle.
the given angle measures into this equation and 3. Let P be the unknown angle at Pittsburgh.
solve for x. mP  85  52 ¬180
Solve: mABC  mDEF ¬180 mP  137 ¬180
57  3x  9 ¬180 mP ¬43
3x ¬132 4. Let A be the unknown angle in the figure.
x ¬44 mA  62  19 ¬180
Examine: Verify the measure of DEF by using mA  81 ¬180
the value of x. That is, 3x  9  3(44)  9 or 123, mA ¬99
and 57  123  180. Since mABC  5. m1  23  32
mDEF  180, mABC is supplementary to  55
DEF, and p  q.
6. m2  22 ¬m1
52. 1. Given m2  22 ¬55
2. Subtraction Property m2 ¬33
3. Addition Property
7. m3  22  (180  m1)
4. Division Property
 22  180  55
53. any three: 2 and 11, 3 and 6, 4 and 7,  147
3 and 12, 7 and 10, 8 and 11
8. m1  25 ¬90
54. 1 and 4, 1 and 10, 5 and 2, 5 and 8, m1 ¬65
9 and 6, 9 and 12
9. m2  65  90
55. 6, 9, and 12 by alternate interior  and m2 ¬25
transitivity
10. m1  m2 ¬90
56. 1, 4, and 10 by alternate interior  and m1  70 ¬90
transitivity m1 ¬20
57. 2, 5, and 8 by alternate interior  and
transitivity
Pages 189–191 Practice and Apply
11. Let X be the third angle in the triangle.
Page 184 Geometry Activity: Angles of Triangles mX  40  47 ¬180
1. congruent mX  87 ¬180
2. congruent mX ¬93
3. congruent 12. Let X be one of the two congruent angles of the
4. 180, because DFA  DFE  AFE, and AFE triangle.
and EFC form a linear pair. mX  mX  39 ¬180
2mX  39 ¬180
5. mA  mB  mC  180 by substitution
2mX ¬141
6. The sum of the measures of the angles of any mX ¬70.5
triangle is 180. The missing angles have measure 70.5 and 70.5.
7. mA  mB is the measure of the exterior angle 13. Let X be one of the two congruent angles of the
at C. triangle.
8. See students’ work. mX  mX  50 ¬180
9. yes 2mX  50 ¬180
10. See students’ work. 2mX ¬130
11. See students’ work. mX ¬65
The missing angles have measure 65 and 65.
12. The measure of an exterior angle is equal to the
sum of the measures of the two remote interior
angles.

Chapter 4 90
14. Let X be the unknown acute angle of the 35. m4  m2 ¬90
triangle. m4  50 ¬90
mX  27 ¬90 m4 ¬40
mX ¬63 36. m1  26  101 ¬180
15. m1  47  57 ¬180 m1  127 ¬180
m1  104 ¬180 m1 ¬53
m1 ¬76 37. m2  26  103
16. m2  m1  129
m2  76 38. m3  101  (180  128)
17. m3  m2  55 ¬180  153
m3  76  55 ¬180 39. Given: FGI  IGH
m3  131 ¬180 I  F
G H
m3 ¬49 Prove: F  H
18. m1  69  47 ¬180 G
m1  116 ¬180
m1 ¬64
19. m1  m2  63 ¬180
64  m2  63 ¬180
m2  127 ¬180 F I H
m2 ¬53 Proof:
20. m3  m2  63 GI ⊥ FH
 53  63 Given
 116
21. m4  m5  m3 ¬180
2m4  116 ¬180 GIF and GIH
2m4 ¬64 are right angles. FGI IGH
m4 ¬32 ⊥ lines form rt. . Given
22. m5  m4 23. m6  136 ¬180
m5  32 m6 ¬44
GIF GIH F H
24. m7  47 ¬136
All rt. are . Third Angle Theorem
m7 ¬89
25. m1  33  24 ¬180 40. Given: ABCD is a quadrilateral.
m1  57 ¬180 Prove: mDAB  mB  mBCD  mD  360
m1 ¬123 A B
1 2
26. m2  95 ¬m1
m2  95 ¬123 3
m2 ¬28 4
D C
27. m3  109 ¬m1
Proof:
m3  109 ¬123
m3 ¬14 Statements Reasons
28. m1  126 ¬180 29. m2  73 ¬126 1. ABCD is a quadrilateral. 1. Given
m1 ¬54 m2 ¬53
2. m2  m3  mB  180 2. Angle Sum
30. m3  43 ¬180 m1  m4  mD  180 Theorem
m3 ¬137
3. m2  m3  mB  m1 3. Addition
31. m4  34  43 ¬180
 m4  mD  360 Property
m4  77 ¬180
m4 ¬103 4. mDAB  m1  m2 4. Angle
32. m1  mDGF ¬90 mBCD  m3  m4 addition
m1  53 ¬90 5. mDAB  mB  mBCD  5. Substitution
m1 ¬37 mD  360
33. m2  mAGC ¬90
m2  40 ¬90
m2 ¬50
34. m3  mAGC ¬90
m3  40 ¬90
m3 ¬50

91 Chapter 4
41. Given: ABC Given: PQR
Prove: mCBD  mA  mC P is obtuse.
C Prove: There can be at most one obtuse angle in
a triangle.
D Q
A
B
R
Proof:
P
Statements Reasons
Proof: In PQR, P is obtuse. So mP  90.
1. ABC 1. Given mP  mQ  mR  180. It must be
that mQ  mR  90. So, Q and R must be
2. CBD and ABC form 2. Def. of linear
acute.
a linear pair. pair
44. Given: A  D
3. CBD and ABC are 3. If 2  form a B  E
supplementary. linear pair, they Prove: C  F
are suppl.
F
4. mCBD  mABC  180 4. Def. of suppl. C
D E
5. mA  mABC 5. Angle Sum A B
 mC  180 Theorem Proof:
6. mA  mABC 6. Subsitution Statements Reasons
 mC  mCBD Property 1. A  D 1. Given
 mABC B  E
7. mA  mC 7. Subtraction 2. mA  mD 2. Def. of  
 mCBD Property mB  mE
42. Given: RST
3. mA  mB  mC  180 3. Angle Sum
R is a right angle
mD  mE  mF  180 Theorem
Prove: S and T are complementary
S 4. mA  mB  mC 4. Transitive
 mD  mE  mF Property
5. mD  mE  mC 5. Substitution
R T  mD  mE  mF Property
Proof:
6. mC  mF 6. Subtraction
R is a rt. . Property
Given 7. C  F 7. Def. of  
45. m1  m2  m3 ¬180
m R 90 m R m S m T 180 4x  5x  6x ¬180
Def. of rt. Angle Sum Theorem 15x ¬180
x ¬12
90 m S m T 180 m1  4(12) or 48
Substitution m2  5(12) or 60
m3  6(12) or 72
46. Sample answer: The shape of a kite is symmetric. If
m S m T 90
triangles are used on one side of the kite, congruent
Subtraction Prop.
triangles are used on the opposite side. The wings
of this kite are made from congruent right
S and T are complementary triangles. Answers should include the following.
Def. of complementary • By the Third Angle Theorem, if two angles of
two congruent triangles are congruent, then the
43. Given: MNO N third angles of each triangle are congruent.
M is a right • If one angle measures 90, the other two angles
angle. are both acute.
Prove: There can be at M O
47. A; mZ  mX ¬90
most one right
a
2  2a ¬90
angle in a triangle.
a  4a ¬180
Proof: In MNO, M is a right angle.
5a ¬180
mM  mN  mO  180. mM  90, so
a ¬36
mN  mO  90. If N were a right angle,
then mO  0. But that is impossible, so there mZ ¬a 
2
36
cannot be two right angles in a triangle. ¬2 or 18

Chapter 4 92
48. B; let x be the measure of the first angle. Then the d ¬(xx2 1) 
2 (y2 
y1)2
other angles have measure 3x and x  25. ¬(4
  0 ) )
2 (1  32
x  3x  x  25 ¬180 ¬16 4
5x  25 ¬180 ¬20
5x ¬155 The distance between the lines is 25  units.
x ¬31
54. 4x  y ¬20
3x ¬3(31) or 93
y ¬4x  20
x  25 ¬31  25 or 56
y ¬4x  20
4x  y ¬3
y ¬4x  3
Page 191 Maintain Your Skills y ¬4x  3
49. AED is scalene because no two sides are First, write an equation of a line p perpendicular
congruent. to 4x  y  20 and 4x  y  3. The slope of p is
50. AED is obtuse because mAED  90. the opposite reciprocal of 4, or 1

4 . Use the
mAED  mBEC, so BEC is obtuse. y-intercept of 4x  y  20, (0, 20), as one of the
51. BEC is isosceles because E B
E C. endpoints of the perpendicular segment.
y  y1 ¬m(x  x1)
52. First, write an equation of a line p perpendicular
to y  x  6 and y  x  10. The slope of p is the y  (20) ¬1
4 (x  0)

opposite reciprocal of 1, or 1. Use the y-intercept y  20 ¬1

4x
of y  x  6, (0, 6), as one of the endpoints of the
perpendicular segment. y ¬1
4 x  20

y  y1 ¬m(x  x1) Next, use a system of equations to determine the
y  6 ¬1(x  0) point of intersection of line 4x  y  3 and p.
y  6 ¬x 4x  3 ¬14 x  20

y ¬x  6 17
4 x ¬17

Next, use a system of equations to determine the
point of intersection of line y  x  10 and p. x ¬4
x  10 ¬x  6 y  14 (4)  20

2x ¬16 y  19
x ¬8 The point of intersection is (4,19).
y  8  10 Then, use the Distance Formula to determine the
y  2 distance between (0, 20) and (4,19).
The point of intersection is (8, 2). d ¬(xx2 1) (y2 
2 y1)2
Then, use the Distance Formula to determine the
¬(4
 0) 
2 [190)]
 (22
distance between (0, 6) and (8, 2).
¬16 1
d ¬(x x
2 1) 
2 (y2 y1)2
¬17
¬(8 ) 02
2  ()  62 The distance between the lines is 17  units.
¬64 4 6
55. 2x  3y ¬9
¬128
3y ¬2x  9
The distance between the lines is 82  units.
y ¬2
3x  3

53. First, write an equation of a line p perpendicular
to y  2x  3 and y  2x  7. The slope of p is 2x  3y ¬6
the opposite reciprocal of 2, or 1  3y ¬2x  6
2 . Use the
y-intercept of y  2x  3, (0, 3), as one of the y ¬2
3x  2

endpoints of the perpendicular segment. First, write an equation of a line p perpendicular
y  y1 ¬m(x  x1) to 2x  3y  9 and 2x  3y  6. The slope of p
y  3 ¬1 is the opposite reciprocal of 2 3
3 , or  2 . Use the

2 (x  0)

y-intercept of 2x  3y  9, (0, 3), as one of the
y  3 ¬1

2x endpoints of the perpendicular segment.
y ¬1
2x  3
 y  y1 ¬m(x  x1)
Next, use a system of equations to determine the y  3 ¬3
2 (x  0)

point of intersection of line y  2x  7 and p.
y  3 ¬3

2x
2x  7 ¬12x  3

5 y ¬32x  3

2 x ¬10

x ¬4 Next, use a system of equations to determine the
y  2(4)  7 point of intersection of line 2x  3y  6 and p.
y1 2 x  2 ¬3 x  3
3 2
The point of intersection is (4, 1). 13
Then, use the Distance Formula to determine the 
6 x ¬1
6
distance between (0, 3), and (4, 1). x ¬
13

93 Chapter 4
6. The red triangles are congruent: BME, ANG,
 6
y  3  
2 13  3 DKH, CLF. The blue triangles are congruent:
9 30
y  13  3 or 13
 EMJ, GNJ, HKJ, FLJ. The purple
The point of intersection is  
6 

30

13 , 13 .
triangles are congruent to each other and to the
triangles made up of a blue triangle and a red
Then, use the Distance Formula to determine the
triangle: BLJ, AMJ, JND, JKC, BMJ,
distance between (0, 3) and 1 
6 

30
 .
3 , 13 ANJ, JKD, JLC. Another set of congruent
d ¬(x
x 2
1) 
2 (y2 
y1)2 triangles consists of triangles made up of a red, a
blue, and a purple triangle: BAJ, ADJ, DCJ,

163 
0 
3103 
3
2 2
¬ CBJ. Another set of congruent triangles consists
of the triangles which are each half of the square:

36
¬  81
169  169
  BCD, ADC, CBA, DAB.
7.
¬
11
7 1
17 y
  
169 or 13 Q Q’
1
17
The distance between the lines is 1
3 units.
56. 2y  8  142 ¬180 linear pair
x
2y  150 ¬180
2y ¬30
y ¬15 T
R T’ R’
4x  6 ¬142 corresponding angles
4x ¬136
x ¬34
Use the Distance Formula to find the length of
z ¬4x  6 alternate exterior angles
each side in the triangles.
z ¬4(34)  6
QR ¬[4  (4)]22  ()  32
z ¬142
¬0   25 or 5
57. x  68 ¬180 supplementary consecutive interior QR ¬(4)  422  ()  32
angles ¬0   25 or 5
x ¬112 RT ¬[1  (4)]22  [2)] (2
4y  68 ¬180 linear pair ¬9   0 or 3
4y ¬112 RT ¬(1)  422  [2)]  (2
y ¬28
¬9   0 or 3
5z  2 ¬x alternate interior angles
QT ¬[1  (4)]22  (  3)2
5z  2 ¬112
¬9   25 or 34 
5z ¬110
QT ¬(1)  42
2  (  3)2
z ¬22
¬9   25 or 34 
58. 3x ¬48 alternate interior angles The lengths of the corresponding sides of two
x ¬16 triangles are equal. Therefore, by the definition of
y  42  48 ¬180 Angle Sum Theorem congruence, Q RQ R
, R
T
R T
, and
y  90 ¬180 T
Q Q T
. Use a protractor to measure the
y ¬90 angles of the triangles. You will find that the
z  42 alternate interior angles measures are the same. In conclusion, because
59. reflexive 60. symmetric R
Q   QR, R T
R T
, Q
TQ T, Q  Q,
61. symmetric 62. transitive R  R, and T  T, QRT  QRT.
63. transitive 64. transitive QRT is a flip of QRT.
8. G  K, H  L, J  P, G HK L
,
J
H L P, G
J K P
4-3 Congruent Triangles
Pages 195–198 Practice and Apply
9. CFH  JKL 10. RSV  TSV
Page 195 Check for Understanding 11. WPZ  QVS 12. EFH  GHF
1. The sides and the angles of the triangle are not 13. T  X, U  Y, V  Z, T U
X Y
,
affected by a congruence transformation, so V
U YZ, T
VXZ

congruence is preserved.
14. C  R, D  S, G  W, C D
R S
,
2. Sample answer: G
D SW, CGRW

15. B  D, C  G, F  H, B C
D G
,
F
CG H, BFDH

16. A  H, D  K, G  L, A D
H K,
G
D K L, A
G
H L
3. AFC  DFB 4. HJT  TKH 17. 1  10, 2  9, 3  8, 4  7,
5. W  S, X  T, Z  J, W
X
S
T
, 5  6
Z
XTJ
, W
Z
S J
 18. s 1–4, s 5–12, s 13–20

Chapter 4 94
19. s 1, 5, 6, and 11, s 3, 8, 10, and 12, s 2, 4, 7, Therefore, GHF  GHF.
and 9 GHF is a slide of GHF.
20. UFS, TDV, ACB 25. Use the figure and the Distance Formula to find
21. We need to know that all the corresponding the length of each side in the triangles.
angles are congruent and that the other JK ¬[2
 (4)]23  ()  32
corresponding sides are congruent. ¬4  36 or 40 
22. Use the figure and the Distance Formula to find JK ¬(8
)  21
2  [3)]  (2
the length of each side in the triangles. ¬36
  4 or 40 
PQ ¬2 KL¬[02)]
 ( 2 [2 (3)]2
PQ ¬2 ¬4  25 or 29 
QV ¬4 KL ¬(3
)  82  [11)]
 (2
QV ¬4 ¬25
  4 or 29 
JL ¬[0
4)]
 ( 2 (2 3)2
PV ¬[2  (4)]2  (4  8)2
¬16
  1 or 17 
¬4   16 or 20 
JL ¬(3
)  22  [13)]
 (2
PV ¬(2 )  4 2  (4  8)2
¬1  16 or 17 
¬4   16 or 20 
The lengths of the corresponding sides of the two
The lengths of the corresponding sides of the two
triangles are equal. Therefore, by the definition of
triangles are equal. Therefore, by the definition of
congruence, JK
J K
, K
LK L
, and J
L
J
L
.
congruence, P QP Q
, Q
V
Q V
, and
Use a protractor to confirm that the
V
P P V
.
corresponding angles are congruent.
Use a protractor to confirm that the
Therefore, JKL  JKL.
corresponding angles are congruent.
JKL is a turn of JKL.
Therefore, PQV  PQV.
PQV is a flip of PQV. 26. False; A  X, B  Y, and C  Z but the
corresponding sides are not congruent.
23. Use the figure and the Distance Formula to find
the length of each side in the triangles. Y
MN ¬8 B
MN ¬8 X Z
NP ¬2
NP ¬2 A C
MP ¬[2 6)]
 ( 2 (2 4)2 27. True; D
¬64   4 or 68 
MP ¬[2 6)]
 ( 2 [24)]
 (2 A
¬64   4 or 68  E F
The lengths of the corresponding sides of the two
B C
triangles are equal. Therefore, by the definition of
congruence, M NM N, N
PN P, and 28. Both statements are correct because the spokes
P
M M P
. A
are the same length, E  IA
, and A
E
A I.
Use a protractor to confirm that the 29. 12
H J
corresponding angles are congruent.
6
Therefore, MNP  MNP. 10
12 G
MNP is a flip of MNP. R S
24. Use the figure and the Distance Formula to find 10 6
the length of each side in the triangles. Q
GF ¬(5 )  2 2  (3  2)2 30. 
HJ corresponds to R
S
, so H
J
R
S
.
¬9   1 or 10  2x – 4 ¬12
GF ¬(12  9)2 2)(3  2 2x ¬16
¬9   1 or 10  x ¬8
GH ¬(3 )  2 2  (5  2)2
31.
¬1   9 or 10  K
GH ¬(10  9)2 2)(5  2 64
¬1   9 or 10  E
HF ¬(5 )  3 2  (3  5)2 J
36 80
L 64
¬4   4 or 8 
HF ¬(12  10)2 5) (3  2 36 80
¬4   4 or 8  D F
The lengths of the corresponding sides of the two 32. D  J, so mD  mJ  36.
triangles are equal. Therefore, by the definition of mD  mE  mF ¬180
congruence, G FG F
, G
H
G H, and 36  64  3x  52 ¬180
F
H H F
. 3x  152 ¬180
Use a protractor to confirm that the 3x ¬28
28
corresponding angles are congruent. x ¬
3

95 Chapter 4
33. Given: RST  XYZ Page 198 Maintain Your Skills
Prove: XYZ  RST 40. x  40 ¬115 41. x  42 ¬100
R S X Y x ¬75 x ¬58
42. x  x  30 ¬180
T Z 2x ¬150
Proof: x ¬75
RST XYZ C
43. B C D
, so BC  CD.
Given 2x  4 ¬10
2x ¬6
R X, S Y, T Z, x ¬3
RS XY, ST YZ, RT XZ BC  2x  4
CPCTC
 2(3)  4 or 10
CD  10
BD  x  2
X R, Y S, Z T,  3  2 or 5
XY RS, YZ ST, XZ RT
44. HKT is equilateral, so HKH T hence
Congruence of and HK  HT.
segments is symmetric.
x  7 ¬4x  8
15 ¬3x
XYZ RST 5 ¬x
Def. of HK  KT  HT  4x  8
 4(5)  8 or 12
34. a. Given (y  y )
b. Given 45. m ¬ 2
 1
(x  x )
2 1
c. Congruence of segments is reflexive. 3  3
¬ 3
d. Given 4  0 or  2
e. Def. of  lines y ¬mx  b
f. Given y ¬ 3
2x  3

g. Def. of  lines 46. y ¬mx  b
h. All right  are .
y ¬3
4x  8

i. Given
j. Alt. int.  are . 47. m ¬4
k. Given y  y1 ¬m(x  x1)
l. Alt. int.  are . y  1 ¬4[x  (3)]
m. Def. of  s y  1 ¬4x  12
35. Given: DEF y ¬4x  11
E
Prove: DEF  DEF 48. y  y1 ¬m(x  x1)
y  2 ¬4[x  (3)]
D F y  2 ¬4x  12
Proof: y ¬4x  10
DEF 49. d ¬(x
x2  1) 
2 (y2 y1)2
Given ¬[1
1)]
 ( 2 (6  7)2
¬4  1 or 5
DE DE, EF EF, D D, E E, d ¬(x
x2  1)  (y2 
50. 2 y1)2
DF DF F F
¬(4
)  82
2  ()  22
Congruence of Congruence of
segments is reflexive. is reflexive.
¬16
6 1 or 32
51. d ¬(x
x2  1) 
2 (y2 y1)2
DEF DEF ¬(5
)  32  (2 5) 2

Def. of ¬4  9 or 13



36. SMP  TNP, MPS  NPT
37. Sample answer: Triangles are used in bridge
design for structure and support. Answers should
Page 198 Practice Quiz 1
include the following. 1. The segments F J, G
J
, H
J
, and D
J
 are all
• The shape of the triangle does not matter. congruent. So DFJ, GJF, HJG, and DJH
• Some of the triangles used in the bridge are isosceles triangles because they each have a
supports seem to be congruent. pair of congruent sides.
38. B; by the order of the vertices in the triangle 2. ABC is equilateral, so all sides are congruent.
names, AC
X Z 2x ¬4x  7
2x ¬7
39. D; DF  [35)]
 ( 2 (7  4)2
x ¬3.5
 64
21 1 or 185 

Chapter 4 96
3. AB  2x NP ¬(3 ) 42  (1 6)2
 2(3.5) or 7 ¬1  25 or 26 
BC  4x  7 EF ¬[4  (2)]2 [62)]
 (2
 4(3.5)  7 or 7 ¬4  64 or 68 
AC  x  3.5 MN ¬(4 ) 22 (6  2)2
 3.5  3.5 or 7 ¬4  16 or 20 
4. m1  50  70 ¬180 The corresponding sides are not congruent, so the
m1  120 ¬180 triangles are not congruent.
m1 ¬60 5. Given: DE and BC B
m2  m1  50 bisect each other.
 60  50 or 110 Prove: DGB  EGC
m3  m2  21 ¬180 D E
G
m3  110  21 ¬180
m3 ¬49 C
5. M  J, N  K, P  L; Proof:
N
MJ K, N
P
K L, M
P
J
L

DE and BC bisect each other.
Given
Page 199 Reading Mathematics
1. Sample answer: If side lengths are given, DG GE, BG GC
determine the number of congruent sides and
Def. of bisector
name the triangle. Some isosceles triangles are of segments
equilateral triangles.
2. ABC is obtuse because mC  90. DGB EGC DGB EGC
3. equiangular or equilateral SAS Vertical are .

M
6. Given: KJ
L
, K
M
JL
 K
Prove: JKM  MLJ
4-4 Proving Congruence—SSS, SAS M
J

Pages 203–204 Check for Understanding L


Proof:
1. Sample answer: In QRS, R is the included
R
angle of the sides Q and R
S
. Statements Reasons
R S
M
1. KJ
L
, K
M
J
L
 1. Given
2. KMJ  LJM 2. Alt. int.  are .
Q
2. Jonathan; the measure of DEF is needed to use M
3. JJ
M
 3. Reflexive Property
SAS. 4. JKM  MLJ 4. SAS
3. EG ¬[2 (4)]23 [3)]
 (2
¬4 0 or 2 7. The triangles have two pairs of sides and the
MP ¬(2 ) 43
2  [3)] (2 included angles congruent, so the triangles are
¬4 0 or 2 congruent by the SAS postulate.
FG ¬[2
 (2)]23 ()  12 8. Each pair of corresponding sides are congruent, so
¬0 16 or 4 the triangles are congruent by the SSS postulate.
NP ¬(2) 232  ()  12 Q
9. Given: T is the midpoint of S. R
¬0 16 or 4 R
SQ R
EF ¬[2
  (4)]2  [1)]
 (32 Prove: SRT  QRT
¬4 16 or 20  S Q
MN ¬(2 ) 4 2  [13)]
 (2 T
¬4 16 or 20  Proof:
EG  MP, FG  NP, and EF  MN. The
Statements Reasons
corresponding sides have the same measure and
are congruent. EFG  MNP by SSS. Q
1. T is the midpoint of S. 1. Given
4. EG ¬[3  (2)]2  [12)]
 (2
T
2. ST
Q
 2. Midpoint Theorem
¬1 9 or 10 
MP ¬(3) 2 2  (1 2)2 R
3. SQ
R
 3. Given
¬1 1 or 2  T
4. RRT
 4. Reflexive Property
FG ¬[3
  (4)]2  (1
 6)2 5. SRT  QRT 5. SSS
¬1 25 or 26 

97 Chapter 4
Pages 204–206 Practice and Apply E
14. Given: AFC
, A
B
BC
, B
E
B
F

10. JK ¬[7
 (3)]2  (4 2)2 Prove: AFB  CEB
¬16
  4 or 20  B
FG ¬(4)  22  (7  3)2
¬4  16 or 20 
KL ¬[1
 (7)]2  (9  4)2 C
¬36
5  2 or 61  A
F
E
GH ¬(9 ) 4 2  (1  7)2
¬25
6  3 or 61  Proof:
JL ¬[1
 (3)]2  (9 2)2
¬4  49 or 53  AE FC
FH ¬(9)  2 2  (1  3)2 Given
¬49
  4 or 53 
Each pair of corresponding sides has the same AE FC
measure so they are congruent. JKL  FGH Def. of seg.
by SSS.
AE EF EF FC EF EF
11. JK ¬[2  (1)]22 ()  12
Addition Prop. Reflexive Prop.
¬1  9 or 10 
FG ¬(3 ) 222  [1)]  (2
AE EC AE EF AF
¬1  1 or 2 
Substitution EF FC EC
KL ¬[5
  (2)]21 [2)] (2
Seg. Addition Post.
¬9  1 or 10 
AF EC
GH ¬(2 ) 3 2  [52)]
 (2
Def. of seg.
¬1  49 or 50 
AB BC
JL ¬[5
  (1)]21 ()  12
AFB CEB BE BF
¬16  4 or 20 
SSS Given
FH ¬(2 ) 2 2  [51)]
 (2
¬0  36 or 6 15. Given: T
RQ Q
 Y
Q
WQ
 R
The corresponding sides are not congruent, so RQY  WQT
JKL is not congruent to FGH. Prove: QWT  QYR
Y
12. JK ¬[0 1)]
 ( 2 [6 (1)]2 Q
¬1  49 or 50 
FG ¬(5 ) 3 2  (3  1)2
¬4  4 or 8  W T
KL ¬(2) 0 2  (3  6)2 Proof:
¬4  9 or 13 
RQ TQ YQ WQ RQY WQT
GH ¬(8 ) 5 2 (1  3)2
Given Given
¬9  4 or 13 
JL ¬[21)]
 ( 2 [3  (1)]2
QWT QYR
¬9  16 or 5
SAS
FH ¬(8 ) 3 2  (1  1)2
¬25  0 or 5 16. Given: CDE is an isosceles triangle.
The corresponding sides are not congruent, so E
G is the midpoint of C.
JKL is not congruent to FGH. Prove: CDG  EDG
D
13. JK ¬(4 ) 3 2  (6  9)2
¬1  9 or 10 
FG ¬(2 ) 1 2  (4  7)2
¬1  9 or 10 
KL ¬(1 ) 4 2  (5  6)2 C G E
¬9  1 or 10  Proof:
GH ¬(1  2)2 4)
(3  2
¬9  1 or 10  Statements Reasons
JL ¬(1) 3 2  (5  9)2 1. CDE is an isosceles 1. Given
¬4  16 or 20  triangle, G is the
FH ¬(1  1)2 7)
(3  2 midpoint of CE.
¬4  16 or 20 
The corresponding sides have the same measure D
2. CD
E
 2. Def. of isos. 
so they are congruent. JKL  FGH by SSS. G
3. CG
E
 3. Midpoint Th.
G
4. DDG
 4. Reflexive Property
5. CDG  EDG 5. SSS

Chapter 4 98
17. Given: MRN  QRP, MNP  QPN 3. R  P 3. Third Angle Theorem
Prove: MNP  QPN M Q
4. RST  PNM 4. SAS
R
21. Given: H
EF F
Proof: N P H
G is the midpoint of E.
Prove: EFG  HFG
Statements Reasons
H
1. MRN  QRP, 1. Given
MNP  QPN F G

N
2. MQ
P
 2. CPCTC E
P
3. NN
P
 3. Reflexive Property Proof:
4. MNP  QPN 4. SAS Statements Reasons

18. Given: 
ACG C A 1. H
EF F
; G is the 1. Given
C
E  bisects A
G
. H
midpoint of E.
Prove: GEC  AEC E C G
2. EG
H
 2. Midpoint Theorem
G
3. FF
G
 3. Reflexive Property
G 4. EFG  HFG 4. SSS
Proof:
Statements Reasons 22. Each pair of corresponding sides is congruent. The
triangles are congruent by the SSS Postulate.
1. 
ACG C, E
C
 1. Given 23. The triangles have two pairs of corresponding
G
bisects A. sides congruent and one pair of angles congruent
E
2. AE
G
 2. Def. of segment bisector but what is needed is the pair of included angles
to be congruent. It is not possible to prove the
C
3. EE
C
 3. Reflexive Property
triangles are congruent.
4. GEC  AEC 4. SSS
24. The triangles have one pair of angles congruent
and one pair of sides (the shared side) congruent. It
19. Given: GHJ  LKJ H K is not possible to prove the triangles are congruent.
Prove: GHL  LKG
J 25. The triangles have three pairs of corresponding
sides congruent and one pair of corresponding
G L
angles congruent. The triangles are congruent by
Proof: the SSS or SAS Postulates.
Statements Reasons 26. Given: T S
S FF H
H T  S
1. GHJ  LKJ TSF, SFH, FHT,
1. Given
and HTS are right P
2. 
HJK
J
, G
J
L
J, 2. CPCTC angles. T F
H
G L
K S
Prove: HT
F

3. HJ  KJ, GJ  LJ 3. Def. of  segments H
4. HJ  LJ  KJ  JG 4. Addition Property
5. KJ  GJ  KG; 5. Segment Addition Proof:
HJ  LJ  HL Statements Reasons
6. KG  HL 6. Substitution
1. 
TSSF F
H
H T
 1. Given
G
7. KH
L
 7. Def. of  segments 2. TSF, SFH, FHT, 2. Given
L
8. GGL
 8. Reflexive Property and HTS are right
9. GHL  LKG 9. SSS angles.
3. STH  THF 3. All right  are .
20. Given: P
RS N
 N
4. STH  THF 4. SAS
T
RM P
 S
S  N S
5. HT
F
 5. CPCTC
M
T  M T
Prove: RST  PNM P
Proof: R
Statements Reasons
S
1. RP
N
, R
T
M
P
 1. Given
2. S  N, and 2. Given
T  M

99 Chapter 4
27. Given: 
TSS
FF H
H T
 S 30. C; using vertical angles and exterior angles,
TSF, SFH, FHT, and a  b  90. Given this fact, it is impossible for the
HTS are right angles. P
other statements to be true.
Prove: SHT  SHF T F 31. B; 3x  6x  7x ¬180
16x ¬180
H x ¬11.25
3x  3(11.25) or 33.75
Proof: 6x  6(11.25) or 67.5
Statements Reasons 7x  7(11.25) or 78.75
Because the angles are all less than 90°, the
1. 
TSS FF
H
H T 1. Given triangle is acute.
2. TSF, SFH, FHT, 2. Given
and HTS are right
angles.
Page 206 Maintain Your Skills
3. STH  SFH 3. All rt.  are . 32. ACB  DCE 33. WXZ  YXZ
4. STH  SFH 4. SAS 34. LMP  NPM 35. m2  78
5. SHT  SHF 5. CPCTC 36. m3  m2 ¬180
m3  78 ¬180
E
28. Given: DFB
, A
E
FC
, A
E
D
B
, C
F
D
B
 m3 ¬102
Prove: ABD  CDB 37. m4  m5 ¬90
A B m5 ¬90  m4
F m4  m3  56 ¬180
m4  102  56 ¬180
m4 ¬22
E
m5 ¬90  m4
D C ¬90  22 or 68
Plan: First use SAS to show that ADE  CBF. 38. m4  m3  56 ¬180
Next use CPCTC and Reflexive Property for m4  102  56 ¬180
segments to show ABD  CDB. m4 ¬22
Proof:
39. m1  m2  43 ¬180
Statements Reasons m1  78  43 ¬180
m1 ¬59
E
1. DF
B
, A
E
F
C
 1. Given
40. m6  m5  78 ¬180
E
2. AD
B
, C
F
D
B
 2. Given m6  68  78 ¬180
3. AED is a right angle. 3.  lines form m6 ¬34
0.3 
1.3
CFB is a right angle. right . 41. rate of change   21
4. AED  CFB 4. All right angles  1
 0.3
1.1 
are . 42. rate of change   32
5. ADE  CBF 5. SAS  1.4
D
6. AB
C
 6. CPCTC 43. There is a steeper rate of decline from the second
quarter to the third.
B
7. DD
B
 7. Reflexive Property
 bisects B
E
44. B , since AE C
for Segments
45. CBD, since BD  bisects ABC
8. CBD  ADB 8. CPCTC
46. BDA, since BDC is a right angle and forms a
9. ABD  CDB 9. SAS linear pair with BDA
29. Sample answer: The properties of congruent D
47. C , since 
BD bisects AC
triangles help land surveyors double check
measurements. Answers should include the
following.
• If each pair of corresponding angles and sides
4-5 Proving Congruence—ASA, AAS
are congruent, the triangles are congruent by
definition. If two pairs of corresponding sides Page 207 Construction
and the included angle are congruent, the
5. JKL  ABC
triangles are congruent by SAS. If each pair of
corresponding sides are congruent, the triangles
are congruent by SSS.
• Sample answer: Architects also use congruent Page 208 Geometry Activity
triangles when designing buildings. 1. They are congruent.
2. The triangles are congruent.

Chapter 4 100
Page 210 Check for Understanding 7. Given: E  K, DGH  DHG, E
G
K
H

1. Two triangles can have corresponding congruent Prove: EGD  KHD
angles without corresponding congruent sides. D
A  D, B  E, and C  F. However,
B
A
D E
, so ABC
 DEF.
B
E G H K
E
A C Proof: Since EGD and DGH are linear pairs,
the angles are supplementary. Likewise, KHD
D F and DHG are supplementary. We are given that
2. Sample answer: In ABC, A
B
 is the included side DGH  DHG. Angles supplementary to
of A and B. congruent angles are congruent so EGD 
C KHD. Since we are given that E  K and
G
E K H, EGD  KHD by ASA.
A B 8. This cannot be determined. The information given
cannot be used with any of the triangle congruence
3. AAS can be proven using the Third Angle Theorem.
postulates, theorems or the definition of congruent
Postulates are accepted as true without proof.
triangles. By construction, two different triangles
H
4. Given: G K J, G
K
HJ G H can be shown with the given information. Therefore,
Prove: GJK  JGH it cannot be determined if SRT  MKL.

K J 5.5
5.5
Proof:
49
GH || KJ GK || HJ 7
Given Given
Pages 211–213 Practice and Apply
HGJ KJG KGJ HJG 9. Given: G
EF H
, F
Alt. int. are . Alt. int. are . F
EGH E
K
Prove: E KH

GJK JGH
K
ASA H
G
GJ GJ Proof:
Reflexive Property
EF || GH
W
5. Given: XY
Z
, X  Z X Y Given EF GH
Prove: WXY  YZW Given
E H
Alt. int. are . EKF HKG
W Z AAS
GKH EKF
Proof:
Vert. are . EK KH
XW || YZ X Z
CPCTC
Given Given
10. Given: 
DEJK J
XWY ZYW WY WY K
D  bisects J
E
.
Alt. int. are . Reflexive Prove: EGD  JGK D
Property
G K
WXY YZW
AAS
E
Proof:
6. Given: 
QS bisects RST; R DK bisects JE.
R  T.
Given
Prove: QRS  QTS DE || JK
Q S
Given EG GJ
Def. of seg. bisector
T E J
Proof: We are given that R  T and Q S Alt. int. are . EGD JGK
bisects RST, so by definition of angle bisector, ASA
RSQ  TSQ. By the Reflexive Property, DGE KGJ
S
Q Q S. QRS  QTS by AAS. Vert. are .

101 Chapter 4
11. Given: V  S, T
V
Q
S
 Proof:
R
Prove: VS R
 MN PQ
T S Given

1 2 MN PQ
R
Def. of seg.
V Q
Proof: MN NP NP PQ NP NP
Addition Prop. Reflexive Prop.
V S
TV QS 1 2
MP NQ MN NP MP
Given Vert. are .
Substitution NP PQ NQ
Seg. Addition Post.
TRV QRS
AAS MP NQ
Def. of seg.
VR SR
CPCTC
MLP QLN M Q
12. Given: F
EJ K
, J
G
K
H
 J K ASA 2 3
F
EGH
 Given
Prove: EJG  FKH T
14. Given: Z is the midpoint of C.
Y
C T E

Z
Prove: YE Z

E F G H
E C
Proof:
EF GH
Given Z

EF GH T Y
Def. of seg. Proof:
CY || TE
EF FG FG GH FG FG Z is the
Given
Addition Prop. Reflexive Prop. midpoint of CT.
Given ETC YCT
EG FH EF FG EG
TEY CYE
Substitution FG GH FH TZ CZ
Seg. Addition Post. Alt. int. are .
Midpt. Th.
EG FH EZT YZC
EJ || FK, JG || KH
Def. of seg. AAS
Given

EJG FKH JEG KFH YZ EZ


ASA JGE KHF CPCTC
Corr. are . 15. Given: NOM  POR, N P
M
N M
R,
13. Given: 
MN  P
Q
, L R
PMR,
M  Q M
N P
R
 M O R
2  3 O
Prove: M O
R
Prove: MLP  QLN
M
Proof: Since N M R  and P
R
MR, M and
1 2 3 4
R are right angles. M  R because all right
M N P Q
angles are congruent. We know that NOM 
POR and N M
P R. By AAS, NMO  PRO.
O
M O R
 by CPCTC.
16. Given: DL
 bisects B
N
. B D
XLN  XDB
Prove: LN
D B
 X
L N
L
Proof: Since D  bisects B
N
, B
X
X
N
. XLN 
XDB. LXN  DXB because vertical angles
are congruent. LXN  DXB by AAS. LN
DB
by CPCTC.

Chapter 4 102
17. Given: F  J, F 4. P  P 4. Reflexive Property
E  H
H 5. IPK  TPB 5. AAS
G
EC H
 G
F
Prove: E HJ
 E C
21. Explore: We are given the measurement of one
J side of each triangle. We need to determine
Proof: We are given that F  J, E  H, whether two triangles are congruent.
and ECG H
. By the Reflexive Property, Plan: CDG H, because the segments have the
G
C C G
. Segment addition results in EG  same measure. CFD  HFG because vertical
EC  CG and CH  CG  GH. By the definition angles are congruent. Since F is the midpoint of
of congruence, EC  GH and CG  CG. Substitute G
D , D
FF G.
to find EG  CH. By AAS, EFG  HJC. By Solve: We are given information about side-side-
CPCTC, E F
H J
. angle (SSA). This is not a method to prove two
18. Given:  S
TX Y
 T X triangles congruent.
TXY  TSY Examine: Use a compass, protractor, and ruler to
Prove: TSY  YXT draw a triangle with the given measurements. For
simplicity of measurement, use 1 inch instead of
S Y 4 feet and 2 inches instead of 8 feet, so the
Proof: Since TX
S Y
, YTX  TYS by measurements of the construction and those of
Y
Alternate Interior Angles Theorem. TTY
 by the garden will be proportional.
the Reflexive Property. Given TXY  TSY,
TSY  YXT by AAS.
19. Given: MYT  NYT M
MTY  NTY
Prove: RYM  RYN 1 in.
R T
Y

N 29
Proof:
2 in.
Statements Reasons
• Draw a segment 2 inches long.
1. MYT  NYT, 1. Given • At one end, draw an angle of 29°. Extend the
MTY  NTY line longer than 2 inches.
T
2. YY
T
, R
Y
R
Y
 2. Reflexive • At the other end of the segment, draw an arc
Property with a radius of 1 inch such that it intersects
the line.
3. MYT  NYT 3. ASA
Notice that there are two possible segments that
4. 
MYN Y
 4. CPCTC could determine the triangle. It cannot be
5. RYM and MYT are a 5. Def. of determined whether CFD  HFG. The
linear pair; RYN and linear pair information given does not lead to a unique
NYT are a linear pair triangle.
22. Explore: We need to determine whether two
6. RYM and MYT are 6. Supplement
triangles are congruent.
supplementary and RYN Theorem
Plan: Since F is the midpoint of DG
, DFF G
. F
and NYT are
H
is also the midpoint of C, so C
FF H. Since
supplementary.
G
D C H, D
F
C F and FG
F H. CFD 
7. RYM  RYN 7.  suppl. to  HFG because vertical angles are congruent.
 are . Solve: CFD  HFG by SAS.
8. RYM  RYN 8. SAS Examine: The corresponding sides and angles
used to determine the triangles are congruent by
20. Given: BMI  KMT B I SAS are D FF G
, CFD  HFG, and C FF H.
IP
P
T P 23. Explore: We need to determine whether two
Prove: IPK  TPB M triangles are congruent.
T Plan: Since N is the midpoint of JL
, J
NN L
.
JNK  LNK because perpendicular lines form
K right angles and right angles are congruent. By
Proof: N
the Reflexive Property, K K N.
Solve: JKN  LKN by SAS.
Statements Reasons
Examine: The corresponding sides and angles
1. BMI  KMT 1. Given used to determine the triangles are congruent by
2. B  K 2. CPCTC SAS are  J
N NL, JNK  LNK, and  K
N KN.

3. IP
P
T 3. Given

103 Chapter 4
x
24. Explore: We are given the measurements of one 32. A; x percent of 10,000 ¬100 (10,000)
side and one angle of each triangle. We need to 1 percent of x percent of 10,000
determine whether the two triangles are congruent.
M
Plan: It is given that J L M
 and NJM 
¬ 1 x
100  
100 (10,000)
x
NLM. By the Reflexive Property, N MN M. ¬
10,000 (10,000)
Solve: We are given information about side-side- ¬x
angle (SSA). This is not a method to prove two
triangles congruent.
Examine: Use a compass, protractor, and ruler Page 213 Maintain Your Skills
to draw a triangle with the given measurements. 33. Given: D
BA E
,
For simplicity of measurement, we will use B D
A
D B
E
 C
centimeters instead of feet, so the measurements Prove: BEA  DAE
of the construction and those of the kite will be A E
proportional. Proof:
• Draw a segment 2.7 centimeters long.
• At one end, draw an angle of 68°. Extend the DA BE
line to exactly 2 centimeters. Given
• At the other end of the segment, draw a seg-ment
that intersects the 2 centimeter segment at any BA DE BEA DAE
place other than either of its endpoints. Given SSS
Because no information is known about the length
of the segment that determines the triangle, an AE AE
infinite number of triangles are possible. It cannot Reflexive Prop.
be determined whetherJNM  LNM. The
34. Given: 
XZW Y, W
information given does not lead to a unique triangle.
Z
X  bisects W
Y
. Z
Prove: WZX  YZX
2 cm Y
X
Proof:
68˚
J M XZ WY
2.7 cm
Given
25. VNR, AAS or ASA 26. VMN, ASA or AAS
XZ bisects WY.
27. MIN, SAS 28. RMI, AAS or ASA WZX and
Given
29. Since Aiko is perpendicular to the ground, two YZX are rt .
right angles are formed and right angles are Def. of ⊥ lines. WZ ZY
congruent. The angles of sight are the same and Def. of seg. bisector
her height is the same for each triangle. The WZX YZX
triangles are congruent by ASA. By CPCTC, the All rt. are . WZX YZX
distances are the same. The method is valid. SAS
XZ XZ
30. Sample answer: The triangular trusses support the
structure. Answers should include the following. Reflexive Prop.
• To determine whether two triangles are
35. RS ¬[1  (2)]2 (1
 2)2
congruent, information is needed about
¬1   1 or 2 
consecutive side-angle-side, side-side-side,
RS ¬(1) 212  [2)]
 (2
angle-side-angle, angle-angle-side, or about each
¬1   1 or 2 
angle and each side.
ST ¬[2
  (1)]2 (1
 1)2
• Triangles that are congruent will support weight
¬1   0 or 1
better because the pressure will be evenly divided.
ST ¬(2 )
 11
2  [1)]
 (2
31. D; mB  mBAC  mBCA ¬180 ¬1   0 or 1
76  mBAC  mBCA ¬180 RT ¬[2  (2)]2 (1
 2)2
mBAC  mBCA ¬104 ¬0   1 or 1
 bisects BAC, so mDAC ¬1
D
A 
2 mBAC.
RT ¬(2) 212  [2)]
 (2
1 ¬0   1 or 1
C
D  bisects BCA, so mDCA ¬2mBCA. RT ¬(2) 212  [2)]
 (2
mDAC  mDCA ¬1 1 ¬0   1 or 1
2 mBAC  2 mBCA

Each pair of corresponding sides has the same
¬1
2 (mBAC  mBCA)

measure, so they are congruent. Use a protractor
¬1

2 (104) or 52
to confirm that the corresponding angles are
mADC  mDAC  mDCA ¬180 congruent. Therefore, RST  RST. RST
mADC  52 ¬180 is a turn of RST.
mADC ¬128

Chapter 4 104
36. MP ¬[1
  (3)]2
 (1 1)2 8. Given: ABC and XYZ are right triangles.
¬4  0 or 2 A and X are right angles.
MP ¬(0)  2 2  (1
 1)2 C
BY Z

¬4  0 or 2 B  Y
MN ¬[3  (3)]2
 (4 1)2 Prove: ABC  XYZ
¬0  9 or 3 C Z
MN ¬(2 )  2 2  (4
 1)2
¬0  9 or 3 A B X Y
NP ¬[1
  (3)]2
 (1 4)2 Proof: We are given that ABC and XZY are
¬4  9 or 13 right triangles with right angles A and X,
NP ¬(2)  0 2  (4
 1)2 C
B Y Z
, and B  Y. Since all right angles
¬4  9 or 13 are congruent, A  X. Therefore,
Each pair of corresponding sides has the same ABC  XYZ by AAS.
measure, so they are congruent. Use a protractor
9. Case 1:
to confirm that the corresponding angles are
Given: ABC and DEF are right triangles.
congruent. Therefore, MPN  MPN.
C
AD F
MPN is a flip of MPN.
C  F
37. If people are happy, then they rarely correct their Prove: ABC  DEF
faults.
C F
38. If a person is a champion, then he or she is afraid
of losing.
A B D E
39. Since two sides are marked congruent, the
triangle is isosceles. Proof: It is given that ABC are DEF are right
C
triangles, A D F , C  F. By the definition
40. Since three sides are marked congruent, the
of right triangles, A and D are right angles.
triangle is equilateral.
Thus, A  D since all right angles are
41. Since two sides are marked congruent, the congruent. ABC  DEF by ASA.
triangle is isosceles. Case 2:
Given: ABC and DEF are right triangles.
C
A D F
Pages 214–215 Geometry Activity: Congruence B  E
in Right Triangles Prove: ABC  DEF
1. yes; a. SAS, b. ASA, c. AAS C F
2a. LL
2b. LA A B D E
2c. HA Proof: If is given that ABC and DEF are right
3. None; two pairs of legs congruent is sufficient for C
triangles, AD F
, and B  E. By the
proving right triangles congruent. definition of right triangle, A and D are right
angles. Thus, A  D since all right angles are
4. yes 5. yes
congruent. ABC  DEF by AAS.
6. SSA is a valid test of congruence for right
10. Given: M LM K, J
KK M, J  L
triangles.
Prove: J M
K L

7. Given: DEF and RST are right triangles.
M
E and S are right angles. L
F
E S T
D
E S R J
Prove: DEF  RST K

F T Proof:

Statements Reasons
D E R S
F
Proof: We are given that ES T, E
DS R, and 1. 
MLMK, J
K
K
M
, 1. Given
E and S are right angles. Since all right J  L
angles are congruent, E  S. Therefore, by 2. LMK and JKM are 2.  lines form
SAS, DEF  RST. rt.   rt. .
3. LMK and JKM are 3. Def. of rt. 
rt. s
K
4. MM
K
 4. Reflexive
Property
5. LMK  JKM 5. LA
M
6. JK
L
 6. CPCTC

105 Chapter 4
11. Given: K
JK M, M 7. Given: CTE is isosceles T
M
JK L
 L with vertex C. 60
L
M J
K
 mT  60 C
L
Prove: M JK
 J Prove: CTE is equilateral.
K
Proof: E
Proof:
Statements Reasons
Statements Reasons
1. K
JK M, J
M
K L
, 1. Given 1. CTE is isosceles 1. Given
L
M J
K
 with vertex C.
2. JKM is a rt.  2.  lines form rt.  T
2. CC
E
 2. Def. of isosceles
triangle
M
3. KM
L
 3. Perpendicular 3. E  T 3. Isosceles Triangle
Transversal Th. Theorem
4. LMK is a rt.  4.  lines form rt.  4. mE  mT 4. Def. of  
K
5. M MK
 5. Reflexive Property 5. mT  60 5. Given
6. JMK  LMK 6. HL 6. mE  60 6. Substitution
L
7. MJ
K
 7. CPCTC 7. mC + mE + mT 7. Angle Sum Theorem
 180
8. mC  60  60  180 8. Substitution
4-6 Isosceles Triangles 9. mC  60 9. Subtraction
10. CTE is equiangular. 10. Def. of equiangular 
11. CTE is equilateral. 11. Equiangular s are
Page 216 Geometry Activity: Isosceles Triangles
equilateral.
1. A  B
2. They are congruent. 8. A;
3. They are congruent. Read the Test Item
PQS is isosceles with base P S
. Likewise, QRS
S
is isosceles with base Q .
Page 219 Check for Understanding Solve the Test Item
1. The measure of only one angle must be given in Step 1 The base angles of QRS are congruent.
an isosceles triangle to determine the measures of Let x  mRSQ  mRQS.
the other two angles. mPRS  mRSQ  mRQS ¬180
X
2. W Z X
; W  Z 72  x  x ¬180
72  2x ¬180
3. Sample answer: Draw a line segment. Set your 2x ¬108
compass to the length of the line segment and x ¬54
draw an arc from each endpoint. Draw segments So, mRSQ  mRQS  54.
from the intersection of the arcs to each endpoint. Step 2 RQS and PQS form a linear pair. Solve
4. ADH is opposite A H and AHD is opposite A D, for mPQS.
so ADH  AHD. mRQS  mPQS ¬180
H
5. B  is opposite BDH and B D is opposite BHD, 54  mPQS ¬180
H
so BB D. mPQS ¬126
6. Each angle of an equilateral triangle measures 60°. Step 3 The base angles of PQS are congruent.
mF ¬3x  4 Let y represent mQPS and mPSQ.
60 ¬3x  4 mPQS  mQPS  mPSQ ¬180
56 ¬3x 126  y  y ¬180
56 126  2y ¬180
3 ¬x
 
2y ¬54
mG ¬6y y ¬27
60 ¬6y The measure of QPS is 27. Choice A is
10 ¬y correct.
mH ¬19z  3
60 ¬19z  3
57 ¬19z Pages 219–221 Practice and Apply
3 ¬z 9. LRT is opposite L
T and LTR is opposite L
R, so
LRT  LTR.
10. LXW is opposite L
W and LWX is opposite LX,
so LXW  LWX.
11. LSQ is opposite Q
L and LQS is opposite S
L,
so LSQ  LQS.

Chapter 4 106
12. 
LX is opposite LYX and L Y
 is opposite LXY, so 24. MLP is isosceles with base M
P, JMP is
X
L L Y
. P
isosceles with base J.
S
13. L  is opposite LRS and L R
 is opposite LSR, so Step 1 The base angles of MLP are congruent.
S
L L R. Let x represent mPML and mMPL.
Y
14. L  is opposite LWY and L W
 is opposite LYW, mPML  mMPL  mPLJ ¬180
Y
so L L W. x  x  58 ¬180
15. The base angles of an isosceles triangle are 2x  58 ¬180
congruent. So LNM  MLN. From the figure, 2x ¬122
mMLN  20. So mLNM  20. x ¬61
16. mLNM  mMLN  mM ¬180 So, mPML  mMPL  61.
20  20  mM ¬180 Step 2 PML and JMP form a linear pair.
mM ¬140 Solve for mJMP.
17. The base angles of an isosceles triangle are mPML  mJMP ¬180
congruent, so mLKN ¬mLNK. 61  mJMP ¬180
mLKN  mLNK  mKLN ¬180 mJMP ¬119
mLKN  mLKN  18 ¬180 Step 3 The base angles of JMP are congruent.
2mLKN ¬162 Let y represent mPJL and mJPM.
mLKN ¬81 mJPM  mPJL  mJMP ¬180
18. mJKN ¬mJKL  mLKN y  y  119 ¬180
130 ¬mJKL  81 2y  119 ¬180
49 ¬mJKL 2y ¬61
mJKL  mJLK  mJ ¬180 y ¬30.5
49  25  mJ ¬180 So, mJPM  mPJL  30.5.
mJ ¬106 25. GKH is isosceles with base HK
. JKH is
19. The base angles of DFG are congruent, so J
isosceles with base H .
DFG D. Step 1 The base angles of GKH are congruent.
mDFG  mD Let x represent mGHK and mGKH.
 28 mHGK  mGHK  mGKH ¬180
20. mDGF  mD  mDFG ¬180 28  x  x ¬180
28  28  mDGF ¬180 2x ¬152
mDGF ¬124 x ¬76
21. FGH and DGF are a linear pair. So, mGHK  mGKH  76.
mDGF  mFGH ¬180 Step 2 GKH and HKJ form a linear pair.
124  mFGH ¬180 Solve for mHKJ.
mFGH ¬56 mGKH  mHKJ ¬180
22. The base angles of FGH are congruent, so 76  mHKJ ¬180
FGH  H. From Exercise 21, mFGH ¬56. mHKJ ¬104
mFGH  mH  mGFH ¬180 Step 3 The base angles of JKH are congruent.
56  56  mGFH ¬180 Let x represent mHJK and mJHK.
mGFH ¬68 mHJK  mJHK  mHKJ ¬180
23. MLP is isosceles with base M P. JMP is x  x  104 ¬180
isosceles with base JP. 2x  104 ¬180
Step 1 The base angles of MLP are congruent. 2x ¬76
Let x represent mPML and mMPL. x ¬38
mPML  mMPL  mPLJ ¬180 So, mHJK  mJHK  38.
x  x  34 ¬180 26. GKH is isosceles with base HK
.
2x  34 ¬180 Step 1 The base angles of GKH are congruent.
2x ¬146 Let x represent mGHK and mGKH.
x ¬73 mHGK  mGHK  mGKH ¬180
So, mPML  mMPL  73. 42  x  x ¬180
Step 2 PML and JMP form a linear pair. 42  2x ¬180
Solve for mJMP. 2x ¬138
mPML  mJMP ¬180 x ¬69
73  mJMP ¬180 So, mGHK  mGKH  69.
mJMP ¬107 Step 2 GKH and HKJ form a linear pair.
Step 3 The base angles of JMP are congruent. Solve for mHKJ.
Let y represent mJ and mJPM. mGKH  mHKJ ¬180
mJPM  mJ  mJMP ¬180 69  mHKJ ¬180
y  y  107 ¬180 mHKJ ¬111
2y 107 ¬180
2y ¬73
y ¬36.5
So, mJPM  mJ  36.5.

107 Chapter 4
27. LMN is equilateral, so LMM NL N. M
P
 10. mLNM  10. Sum of measures of a
N
bisects L, so MP
 bisects LMN. LMN is mLNP  180 linear pair of angles is
equilateral, hence equiangular. So mLMN  180
mMLP  60 and mPML  30. 11. 2mLNM  180 11. Substitution
LM ¬MN
3x  1 ¬4x  2 12. mLNM  90 12. Division
3 ¬x 13. LNM is a right 13. Definition of right
mPLM  mPML  mMPL ¬180 angle. angle
60  30  5y ¬180
N
14. L M
P
 14. Definition of
90  5y ¬180 perpendicular
5y ¬180
y ¬18 31. Case I:
28. LM  3x  1 Given: ABC is an equilateral triangle.
 3(3)  1 or 10 Prove: ABC is an equiangular triangle.
LM  MN  LN, so all sides have measure 10. B
29. Given: XKF is equilateral. X
J
X  bisects X.
Prove: J is the midpoint A C
F
of K .
1 2 Proof:
K J F
Statements Reasons
Proof:
1. ABC is an 1. Given
Statements Reasons equilateral triangle.
1. XKF is equilateral. 1. Given B
2. AA
C
B
C
 2. Def. of equilateral 
X
2. KF
X
 2. Definition of 3. A  B, B  C, 3. Isosceles Triangle
equilateral  A  C Theorem
3. 1  2 3. Isosceles Triangle 4. A  B  C 4. Substitution
Theorem 5. ABC is an 5. Def. of equiangular 
J
4. X bisects X 4. Given equiangular triangle.
5. KXJ  FXJ 5. Def. of  bisector
Case II:
6. KXJ  FXJ 6. ASA Given: ABC is an equiangular triangle.
J
7. KJ
F
 7. CPCTC Prove: ABC is an equilateral triangle.
B
F
8. J is the midpoint of K. 8. Def. of midpoint

30. Given: MLP is isosceles. L


P
N is the midpoint of M. A C
N
Prove: LM P Proof:

Statements Reasons
M N P
Proof: 1. ABC is an 1. Given
equiangular triangle.
Statements Reasons
2. A  B  C 2. Def. of equiangular 
1. MLP is isosceles. 1. Given
3. 
ABA
C
, A
B
B
C
, 3. Conv. of Isos.  Th.
L
2. ML
P 2. Definition of isosceles  C
A B
C
3. M  P 3. Isosceles Triangle B
4. AA
C
B
C
 4. Substitution
Theorem 5. ABC is an 5. Def. of equilateral 
4. N is the midpoint 4. Given equilateral triangle.
P
of M .
N
5. MNP
 5. Midpoint Theorem
6. MNL  PNL 6. SAS
7. LNM  LNP 7. CPCTC
8. mLNM  8. Congruent angles have
mLNP equal measures.
9. LNM and LNP 9. Definition of linear pair
are a linear pair

Chapter 4 108
32. Given: MNO is an O 36. The triangle is isosceles with base angles having
equilateral triangle. measure 3x  8.
Prove: mM  mN  mO  60 (3x  8)  (3x  8)  (2x  20) ¬180
8x  36 ¬180
M N
8x ¬144
Proof: x ¬18
Statements Reasons 37. The triangle on the bottom half of the figure is
isosceles. The base angles are congruent.
1. MNO is an 1. Given 2x  25 ¬x  5
equilateral triangle. x  25 ¬5
N
2. MM
O
N
O
 2. Def. of equilateral  x ¬30
38. There are two sets of 12 isosceles triangles. One
3. M  N  O 3. Isosceles  Thm.
black set forms a circle with their bases on the
4. mM  mN  mO 4. Def. of   outside of the circle. Another black set encircles a
5. mM  mN  5. Angle Sum Theorem circle in the middle.
mO  180
6. 3mM  180 6. Substitution
7. mM  60 7. Division Property
39. The triangles in each set appear to be acute.
8. mM  mN  8. Substitution
40. DCE is equilateral, hence equiangular so
mO  60 mCDE  mDEC  mDCE  60. Then we
know, mACB  mDCE so mACB  60 and
33. Given: ABC B mFCG  mDCE so mFCG  60. This means
A  C
that in isosceles ABC, BAC and ABC are the
B
Prove: AC B

congruent base angles, so mABC  mBAC  42.
A D C Also, in isosceles FCG, CFG and FGC are the
congruent base angles, so mCFG  mFGC  77.
Proof: mACB  mABC  mBAC ¬180
mACB  42  42 ¬180
Statements Reasons mACB ¬96
mFCG  mCFG  mFGC ¬180
1. Let 
BD bisect ABC. 1. Protractor Postulate mFCG  77  77 ¬180
2. ABD  CBD 2. Def. of bisector mFCG ¬26
So m3  26
3. A  C 3. Given
mCFD  mCFG ¬180
D
4. BB
D
 4. Reflexive Property mCFD  77 ¬180
5. ABD  CBD 5. AAS mCFD ¬103
mCGF  mCGE ¬180
B
6. AC
B
 6. CPCTC 77  mCGE ¬180
mCGE ¬103
34. The minimum requirement is that two angles m2  mCDF  mCFD ¬180
measure 60°. m2  60  103 ¬180
35. The front face of the figure has two congruent m2 ¬17
angles and one angle of measure 60. Let y represent m4  mCEG  mCGE ¬180
the measure of each of the congruent angles. m4  60  103 ¬180
y  y  60 ¬180 m4 ¬17
2y  60 ¬180 CDA  CEB by AAS Postulate
2y ¬120 So 1  5
y ¬60 2m1  2(17)  26 ¬mACB
Therefore all angles have measure 60, so the 2m1  60 ¬96
triangle is equiangular and equilateral. All sides 2m1 ¬36
have length 2x  5, in particular the edge m1 ¬18
between the front face and the side face showing. So, m1  18, m2  17, m3  26, m4  17,
Because the side face has two congruent base and m5  18.
angles it is isosceles and the sides opposite the
41. Sample answer: Artists use angles, lines, and
congruent angles are congruent.
shapes to create visual images. Answers should
2x  5 ¬3x  13
include the following.
2x  18 ¬3x
• Rectangle, squares, rhombi, and other polygons
18 ¬x
are used in many works of art.
• There are two rows of isosceles triangles in the
painting. One row has three congruent isosceles
triangles. The other row has six congruent
isosceles triangles.

109 Chapter 4
42. A; mZXY  mPYZ ¬90 47. QR ¬(5) 1 2  [1)]
 (52
mZXY  26 ¬90 ¬166  3 or 52
mZXY ¬64 EG ¬[1
  (4)]2 [2)]
 (32
PYZ is isosceles since YP  YZ, so mYPZ  ¬9   25 or 34 
mYZP. RS ¬(4) 5 2  (0 1)2
mYPZ  mYZP  mPYZ ¬180 ¬1   1 or 2 
2(mYPZ)  26 ¬180 GH ¬[21)]
 ( 2 (1 2)2
2(mYPZ) ¬154 ¬9   1 or 10
mYPZ ¬77 QS ¬(4) 1 2  [0)]
 (52
mXPZ  mYPZ ¬180 ¬9   25 or 34 
mXPZ  77 ¬180 EH ¬[24)]
 ( 2 [1 (3)]2
mXPZ ¬103 ¬366  1 or 52
mXZP  mXPZ  mPXZ ¬180 The corresponding sides are not congruent so
mXZP  103  64 ¬180 QRS is not congruent to EGH.
mXZP ¬13 48. a b a and b
x1  x2 y1  y2

43. D;  2 2  3  9 5  13
,   2, 2  T T T
  2 , 2  or (6, 9)
12
 1
8 T F F
F T F
Page 221 Maintain Your Skills F F F
44. Given: N  D, G N
G  I, S 49. p q p q p or q

A
NSD A
Prove: ANG  SDI D I T T F F F
T F F T T
Proof: We are given N  D and G  I. By
the Third Angle Theorem, A  S. We are also F T T F T
N
given AS D. ANG  SDI by ASA. F F T T T
45. Given: 
VRR
S, V R
T
U S
U,
S
R U
S
 S 50. k m m k and m
Prove: VRS  TUS T T F F
U T
T F T T
Proof: We are given that VRR S
, U
T
S U
 and F T F F
S
RU S . Perpendicular lines form four right F F T F
angles so R and U are right angles. R  U
because all right angles are congruent. RSV 
51. y y or z
UST since vertical angles are congruent. y z
Therefore, VRS  TUS by ASA. T T F T
46. QR ¬[13)]
 ( 2 (2 1)2 T F F F
¬16
  1 or 17
EG ¬(2) 632  [2)]
 (2 F T T T
¬16
  1 or 17 F F T T
RS ¬(1
 1)2 (2 2)2
¬4  16 or 20 x1  x2 y1  y2
GH ¬(4) 2 2  [1)]
 (32 52.  2 2  
2  7 15  9
,   2, 2 
¬4  16 or 20  (4.5, 12)
QS ¬[1
 (3)]22
 (  1)2 x1  x2 y1  y2
¬4  9 or 13 53.  2 2  
4  2 6  (12)
,   2, 2 
EH ¬(4) 6 2  [1)]
 (22  (1, 3)
¬4  9 or 13 x1  x2 y1  y2
Each pair of corresponding sides has the same 54.  ,   2, 2
2 2  
3  7.5 2.5  4

measure so they are congruent. QRS  EGH  (5.25, 3.25)
by SSS.

Chapter 4 110
Page 221 Practice Quiz 2 2. Sample answer:
1. JM ¬[2
 
(4)]2
 (6
 5)2 y
¬4  1 or 5  C(0, b)
BD ¬[4
 (3)]22
 [4)]
 (2
¬1  4 or 5 
ML ¬[1
 (2)]2 (1
 6)2
A(0, 0) B(a, 0) x
¬1  25 or 26 
DG ¬[14)]
 ( 2 [1)]
 (22 3. • Use the origin as vertex F of the triangle.
¬25
  1 or 26  • Place the base of the triangle along the positive
JL ¬[1
 (4)]2 (1
 5)2 x-axis.
¬9  16 or 5 • Position the triangle in the first quadrant.
BG ¬[13)]
 ( 2 [1)]
 (42 • Since H is on the x-axis, its y-coordinate is 0. Its
¬16
  9 or 5 x-coordinate is 2b because the base of the
Each pair of corresponding sides has the same triangle is 2b units long.
measure so they are congruent. JML  BDG • Since FGH is isosceles, the x-coordinate of G
by SSS. is halfway between 0 and 2b or b. We cannot
2. Given: A  H, AEJ  HJE determine the y-coordinate in terms of b, so call
Prove: AJ
E H it c.
y G(b, c)
A E

J H
Proof: F(0, 0) H(2b, 0) x
Statements Reasons 4. • Use the origin as vertex C of the triangle.
• Place side CD
 of the triangle along the positive
1. A  H, AEJ  HJE 1. Given x-axis.
J
2. EE
J
 2. Reflexive • Position the triangle in the first quadrant.
Property • Since D is on the x-axis, its y-coordinate is 0. Its
3. AEJ  HJE 3. AAS x-coordinate is a because each side of the
triangle is a units long.
J
4. AE
H
 4. CPCTC • Since CDE is equilateral, the x-coordinate of E
is halfway between 0 and a, or a 2. We cannot
X
3. From the figure, W X Y
 so the angles opposite
determine the y-coordinate in terms of a, so call
these sides are congruent. That is, mXWY  it b.
mXYW. a
y
mXYW  mXYZ ¬180 E 2 ,b
mXYW  128 ¬180
mXYW ¬52
So, mXWY  52.
4. From Question 3, mXWY  52.
C(0, 0) D (a, 0) x
mWXY  mXWY  mXYW ¬180
mWXY  52  52 ¬180 5. Vertex P is on the y-axis, so its x-coordinate is 0.
mWXY ¬76 R is a right angle and R Q
 has length a, but
5. XYZ is isosceles with YZX  YXZ. there is no other information given to determine
mYZX  mYXZ  mXYZ ¬180 the y-coordinate of P, so let the y-coordinate be b.
2(mYZX)  128 ¬180 So, the coordinates of P are (0, b).
2(mYZX) ¬52 6. Vertex Q is on the x-axis, so its y-coordinate is 0.
mYZX ¬26 PQN is equilateral and the y-axis bisects side
N
Q . The x-coordinate of N is 2a, so the
x-coordinate of Q is 2a. So, the coordinates of Q
are (2a, 0).
4-7 Triangles and Coordinate Proof 7. Vertex N is on the y-axis and no information is
given to determine its y-coordinate, so the
coordinates of N are (0, b) for some b. Vertex Q is
Page 224 Check for Understanding on the x-axis, so its y-coordinate is 0. NRQ is
1. Place one vertex at the origin, place one side of Q
isosceles and the y-axis bisects its base R . The
the triangle on the positive x-axis. Label the x-coordinate of R is a, so the x-coordinate of Q is
coordinates with expressions that will simplify a. So, the coordinates of Q are (a, 0).
the computations.

111 Chapter 4
8. Given: ABC is a right y • Since MNP is equilateral, the x-coordinate of
triangle with B(0, 2b) P is halfway between 0 and 2a, or a. We cannot
hypotenuse  BC. determine the y-coordinate in terms of a, so call
M
M is the midpoint it b.
of B C. y
P (a, b)
Prove: M is equidistant A(0, 0) C(2c, 0) x
from the vertices.
C
Proof: The coordinates of M, the midpoint of B ,

will be 22
c 
, 22b 
  (c, b).
M (0, 0) N (2a, 0) x
The distance from M to each of the vertices can be
found using the Distance Formula. 12. • Use the origin as vertex L of the triangle.
MB  (c ) 02  (b
 2b)2  c2  b2 M
• Place leg L of the triangle along the positive
MC  (c c)
 2b2  ( 0)2  c
2  b2  x-axis.
MA  (c ) 02  (b
 0)2  c2  b2 • Position the triangle in the first quadrant.
Thus, MB  MC  MA, and M is equidistant from • Since M is on the x-axis, its y-coordinate is 0. Its
the vertices. x-coordinate is c because each leg is c units
long.
9. Given: ABC y
B(2, 8) • Since J is on the y-axis, its x-coordinate is 0. Its
Prove: ABC is isosceles.
y-coordinate is c because each leg is c units long.
y
J(0, c)

L(0, 0) M (c, 0) x
A(0, 0) C (4, 0) x
13. • Use the origin as vertex W of the triangle.
Proof: Use the Distance Formula to find AB • Place side W Z of the triangle along the positive
and BC. x-axis.
AB  (2
)  02  (8
 0)2  4 64 or 68 • Position the triangle in the first quadrant.
BC  (4) 22  (0
 8)2  4 64 or 68 • Since Z is on the x-axis, its y-coordinate is 0. Its
Since AB  BC, AB
B C
. Since the legs are
x-coordinate is b2 because each side of the
congruent, ABC is isosceles.
triangle is 1
b units long.
2
• Since WXZ is equilateral, the x-coordinate of X
Pages 224–226 Practice and Apply is halfway between 0 and b b
2, or 4. We cannot
10. • Use the origin as vertex Q of the triangle. determine the y-coordinate in terms of b, so call
• Place the base of the triangle along the positive it c.
y
x-axis. X(1–4b,c)
• Position the triangle in the first quadrant.
• Since R is on the x-axis, its y-coordinate is 0. Its
x-coordinate is b because the base Q R is b units
long.
• Since QRT is isosceles, the x-coordinate of T is W(0, 0) Z (1–2b, 0) x
halfway between 0 and b or b 2. We cannot 14. • Use the origin as vertex P of the triangle.
determine the y-coordinate in terms of b, so call • Place side PW
 of the triangle along the positive
it c. x-axis.
y T (b–2, c) • Position the triangle in the first quadrant.
• Since W is on the x-axis, its y-coordinate is 0. Its
x-coordinate is a  b because the base is a  b
units long.
• Since PWY is isosceles, the x-coordinate of Y is
a b
Q (0, 0) R(b, 0) x halfway between 0 and a  b or  2 . We
cannot determine the y-coordinate in terms of a
11. • Use the origin as vertex M of the triangle. and b, so call it c.
• Place side MN
 of the triangle along the positive y
x-axis. Y (a 2 b , c)
• Position the triangle in the first quadrant.
• Since N is on the x-axis, its y-coordinate is 0. Its
x-coordinate is 2a because each side of the
triangle is 2a units long.
P(0, 0) W(a b, 0) x

Chapter 4 112
15. • Use the origin as vertex Y of the triangle. 24. Since NPQ is right isosceles, N Q P Q
 and
Z
• Place leg Y  of the triangle along the positive Q
N P Q
. Then Q and P have the same
x-axis. x-coordinate, so the x-coordinate of Q is a. Q is on
• Position the triangle in the first quadrant. the x-axis, so the coordinates of Q are (a, 0).
• Since X is on the y-axis, its x-coordinate is 0. Its 25. Given: isosceles ABC y
y-coordinate is b because X Y
 is b units long. C(2a, 2b)
with ACB C

• Since Z is on the x-axis, its y-coordinate is 0. Its R and S are
x-coordinate is 2b because XY  b and ZY  R S
midpoints of
2(XY) or 2b. legs AC
 and BC.
y Prove: ASB R A(0, 0) B(4a, 0) x
X(0, b) Proof: The coordinates of R are

2a  0 2b  0

2, 2 or (a, b).

Y (0, 0) Z(2b, 0) x
The coordinates of S are  2a 
2 
, 2b 2
4a   0

or (3a, b).
16. Since PQR is equilateral, the x-coordinate of R BR ¬(4a  a)2 )
(0  b2
is halfway between 0 and 2a, or a. So, the ¬(3a)
 2 (b)2 or 9a  2 b2
coordinates of R are (a, b). AS ¬(3a  0)2 )
(b  02
17. Since LPQ is right isosceles, L P
P Q and ¬(3a)
 2 (b)2 or 9a
 2  b2
P
L P Q
. Then Q and P have the same Since BR  AS, A SB R.
x-coordinate, so the x-coordinate of P is a. P is on 26. Given: isosceles ABC y C(a, b)
the x-axis, so its y-coordinate is 0. The distance with BCA C

from L to P is a units. The distance from P to Q R, S, and T are
must be the same. So, the coordinates of Q are R S
midpoints of their
(a, a) and the coordinates of P are (a, 0). respective sides.
18. JKN is isosceles, so J KJ N. The distance from Prove: RST is isosceles. A(0, 0) T B(2a, 0) x
J to K is 2a units. The distance from J to N must
be the same. N is on the y-axis, so the coordinates Proof:
of N are (0, 2a). Midpoint R is  
a 
0 b0 a b
2 , 2 or 2, 2 .  
19. CDF is equilateral, so the x-coordinate of F is
halfway between 0 and the x-coordinate of D. So Midpoint S is 2, 2 or 
a  2a b  0
2 , 2 .
3a
 b
the x-coordinate of D is 2b. D is on the x-axis, so Midpoint T is  2a 
2, 2 or (a, 0).
0 00
the coordinates of D are (2b, 0).
RT ¬ a2  a  2   0 
2 2
20. BCE is isosceles and the y-axis intersects side b
C
B  at its midpoint. The distance from the origin
¬ a2  2
2 b 2
to C is the same as the distance from the origin to
B. B is on the x-axis to the left of 0, so the
¬ a2  b2
2 2
coordinates of B are (a, 0). E is on the y-axis,
and we cannot determine the y-coordinate in
ST ¬ 32a  a  b2 
0
2 2
terms of a, so call it b. The coordinates of E
are (0, b).
¬ a2  b2
2 2
21. MNP is isosceles and the y-axis intersects side
N
M  at its midpoint. The distance from the origin
to M is the same as the distance from the origin RT  ST and RTS T and RST is isosceles.
to N. N is on the x-axis, so the coordinates of N 27. Given: ABC y
C(b, c)
are (2b, 0). P is on the y-axis, and we cannot S is the midpoint of 
A
C.
determine the y-coordinate in terms of b, so call it T is the midpoint of 
BC.
c. The coordinates of P are (0, c). Prove: ST
AB
 S T
22. JHG is isosceles and the y-axis intersects side
JH at its midpoint. The distance from the origin A(0, 0) B(a, 0) x
to J is the same as the distance from the origin to
Proof:
H. H is on the x-axis, so the coordinates of H are
(b, 0). G is on the y-axis, and we cannot determine
the y-coordinate in terms of b, so call it c. The

b 
Midpoint S is  0 c0
  
b c
2 , 2 or 2, 2 .
coordinates of G are (0, c). Midpoint T is  2 , 2  or 
a 
 0 
b  c
2 , 2 .
a 
b c

23. JKL is isosceles, so the x-coordinate of J is c  c


halfway between 0 and 2c, or c. We cannot T
Slope of S 

2 2
 
0 or 0.
a
a b b 

2  2 2
determine the y-coordinate in terms of c, so call it
00 0
b. The coordinates of J are (c, b). B
Slope of A 0  a or 0.
a 
T
S and A
B
 have the same slope so S
TA
B
.

113 Chapter 4
28. Given: ABC y 32. Given: JCT
C(b, c)

S is the midpoint of AC. Prove: JCT is a right triangle.
T is the midpoint of 
BC. y T(300, 500)
S T
Prove: ST  1
AB
2
J
A(0, 0) B(a, 0) x (–500, 300)


b 
Midpoint S is  c 
0 
2 , 2
0
or b  
 c
2, 2 .
x
C(0, 0)
Midpoint T is 2, 2 or 
a  b 0  c
2 , 2 .
a b c
300  0 3
C
500 0 or  5 .
Proof: ST ¬ 2 2 2
Proof: The slope of J
   c  c
2 2
a b
 b 2 0
500 
C
The slope of T 5
300  0 or 3 .
 a2 
2
¬ 0 2 The slope of TC is the opposite reciprocal of the
C
slope of J .  T
JC C, so TCJ is a right angle.
¬ 2 or 2
2
a a JCT is a right triangle.
AB ¬(a )  02  (0
 0)2 33. Use the Distance Formula to find the distance
¬a 2  02
 or a between J(500, 300) and T(300, 500).
JT ¬[300
 0)]
(50 2 
(500 )3002
ST ¬1

2 AB ¬680,0
00 
29. Given: ABD, FBD y
¬824.6
AF  6, BD  3 B(3, 4) The distance between Tami and Juan is 680,0 00 
Prove: ABD  FBD
or approximately 824.6 feet.
A(0, 1) 34. X is at the origin, so place Y(a, b) in the first
F(6, 1) quadrant. Draw a perpendicular segment from Y
D(3, 1)
O x to the x-axis, label the intersection point Z. Z has
the same x-coordinate as Y, or a. Z is on the
Proof:   ¬B
BD D by the Reflexive Property. x-axis, so the coordinates of Z are (a, 0).
AD ¬(3 ) 0 2  (1
 1)2  9   0 or 3 y
DF ¬(6 ) 3 2  (1
 1)2  9   0 or 3
Since AD  DF, A DD F .
AB  (3)  02  (4  1)2  9   9 or 32  Y (a, b)
BF  (6)  32  (1  4)2  9   9 or 32 
Since AB  BF, A BB F .
ABD  FBD by SSS.
30. Given: BPR y B(800, 800)
PR  800, O X (0, 0) Z (a, 0) x
BR  800
35. X is at the origin, so place Y(a, b) in the first
Prove: BPR is
A(1600, 0) quadrant. XYZ is isosceles, so place Z on the
an isosceles
x-axis so that the x-coordinate of Y is halfway
right triangle. P(0, 0) R(800, 0) x
between 0 and the x-coordinate of Z. So, the
coordinates of Z are (2a, 0).
Proof: Since PR and BR have the same measure,
y
R
PB R
.
0 0
The slope of PR  
800  0 or 0.
800  0
The slope of BR  
800 800 , which is undefined.

PR BR, so PRB is a right angle. BPR is an Y (a, b)
isosceles right triangle.
31. Given: BPR, y B(800, 800)
BAR x
PR  800, O X (0, 0) Z (2a, 0)
BR  800,
RA  800 A(1600, 0)
Prove: PB BA P(0, 0) R(800, 0) x

Proof:
PB  (800
 0)2 
 (800 0)2 or 1,280
,000
BA  (800
 1600)00
2  (8) 02 or 1,280
,000

PB  BA, so P
BB A.

Chapter 4 114
36. Sample answer: X is at the origin, so place Y(a, b) 42. Given: isosceles triangle K M
in the first quadrant. XYZ is scalene, so place Z JKN with vertex 2
4
on the x-axis so that the x-coordinate c is such that N, JK
L M
 N
XZ YZ XY. So, the coordinates of Z are (c, 0). Prove: NML is isosceles 3
1
y J L
Proof:

Y (a, b) Statements Reasons


1. isosceles triangle 1. Given
JKN with vertex
N
x J
2. N N K 2. Def. of isosceles triangle
O X (0, 0) Z (c, 0) 3. 2  1 3. Isosceles Triangle
37. AB  4a Theorem
AC ¬(0
2a))
 (2a  (2  0)2 K
4. JLM
 4. Given
¬4a
 2 
4a2 or 8a
2
5. 1  3, 4  2 5. Corr.  are .
CB ¬(0
a)
 2(2a
2 
0)2
¬4a
 2 
4a or 8a
2  2 6. 2  3, 4  1 6. Congruence of  is
2a  0 transitive. (Statements
C
Slope of A  
0  (2a) or 1; 3 and 5)
2a  0
B
slope of C0  2a or 1.
 7. 4  3 7. Congruence of  is

ACC B and ACC B
, so ABC is a right transitive. (Statements
isosceles triangle. 3 and 6)
38. Sample answer: Placing the figures on the N
8. LM
N
 8. If 2  of a  are ,
coordinate plane is useful in proofs. We can use then the sides opp.
coordinate geometry to prove theorems and verify those  are .
properties. Answers should include the following. 9. NML is an 9. Def. of isosceles triangle
• flow proof, two-column proofs, paragraph proofs, isosceles triangle.
informal proofs, and coordinate proofs
• Sample answer: The Isosceles Triangle Theorem D
43. Given: ACE
, A
D
CE

can be proved using coordinate proof. Prove: ABD  EBC
A C
39. C; d ¬(3
 1)2 2)]
[1  (2
¬16
  9 or 5 B
x1  x2 y1  y2

40. B;   
5  (2) 4  (1)
,   2, 2
2 2  D
 (3.5, 1.5) E
Proof:
Statements Reasons
Page 226 Maintain Your Skills D
1. AC
E
 1. Given
41. Given: 3  4 Q 2. A  E, D  C 2. Alt. int.  are .
R
Prove: QQ S

D
3. AC
E
 3. Given
3 4
1 2 4. ABD  EBC 4. ASA
R S
Proof: 44. Given: 
WXXY
, W
V  Z V
Statements Reasons V
Prove: W YZ

X
1. 3  4 1. Given
Z
2. 2 and 4 form a 2. Def. of linear pair Y
linear pair. 1 and Proof:
3 form a linear pair. Statements Reasons
3. 2 and 4 are 3. If 2  form a linear
X
1. WX
Y
, V  Z 1. Given
supplementary. 1 pair, then they are
and 3 are suppl. 2. WXV  YXZ 2. Vert.  are .
supplementary. 3. WXV  YXZ 3. AAS
4. 2  1 4. Angles that are V
4. WY
Z
 4. CPCTC
suppl. to   are .
5. Conv. of Isos.  Th. 45. BCA  DAC, so B C
A
D
; if alternate interior
R
5. QQ
S

 are , lines are .

115 Chapter 4
46. h  j; Sample answer: The angle in the figure whose 19. MN ¬(7)  3 2 (4  2)2
measure is 111 is congruent to the angle on the ¬16  4 or 20 
other side of line h such that the angles are vertical QR ¬[4
  (2)]2 (7  3)2
angles. Then 111  69  180, so h  j because ¬4   16 or 20 
consecutive interior angles are supplementary. NP ¬(6)  7 2  (6 4)2
47. l  m; 2 lines  to the same line are parallel ¬1   4 or 5 
RS ¬[6
  (4)]2 (6  7)2
¬4   1 or 5 
MP ¬(6)  3 2  (6 2)2
Chapter 4 Study Guide and Review ¬9   16 or 5
QS ¬[6
  (2)]2 (6  3)2
Page 227 Vocabulary and Concept Check ¬16  9 or 5
1. h 2. g 3. d Each pair of corresponding sides has the same
4. j 5. a 6. c measure. Therefore, MNP  QRS by SSS.
7. b 8. f 20. Given: 
DF bisects D
CDE, C ED F.
Prove: DGC  DGE

Pages 228–230 Lesson-by-Lesson Review C G E


9. ABC has one angle with measure greater than
F
90, so it is an obtuse triangle. ABC has two
Proof:
congruent sides, so it is isosceles.
10. BDP has a right angle, so it is a right triangle. Statements Reasons
The measure of one of the acute angles is 60 so
1. 
DF bisects CDE, 1. Given
the measure of the other acute angle is
E
C D F
.
90  60 or 30. BDP cannot be isosceles or
equilateral so it must be scalene. G
2. DDG 2. Reflexive Property
11. BPQ has at least two congruent sides, and one 3. CDF  EDF 3. Def. of  bisector
of the base angles is 60° so the other base angle 4. DGC is a rt. ; 4. Def. of  segments
must also be 60°. Then the third angle is DGE is a rt. 
180  (60  60) or 60 so BPQ is equiangular 5. DGC  DGE 5. All rt.  are .
and hence equilateral.
6. DGC  DGE 6. ASA
12. m1  45  40
 85 21. Given: DGC  DGE, D
13. m2  m1  70 ¬180 GCF  GEF
m2  85  70 ¬180 Prove: DFC  DFE
m2 ¬25
14. m3  m1 ¬180 C G E
m3  85 ¬180
m3 ¬95 F
Proof:
15.E  D, F  C, G  B, E F
D C
,
G
F C B, GE
B D Statements Reasons
16. FGC  DLC, GCF  LCD, GFC  1. DGC  DGE, 1. Given
LDC, G CL C, C
FC D, FG
D L GCF  GEF
17. KNC  RKE, NCK  KER, CKN 
ERK, N CK E, CK
E R
, KN
R K
 2. CDG  EDG, C
D
E
D
, 2. CPCTC
and CFD  EFD
18. MN ¬(4   0)2 )
(3  32
¬16  0 or 4 3. DFC  DFE 3. AAS
QR ¬(2 )  5 2  (6 6)2
¬9   0 or 3 22. PQU is isosceles with base P
U. The base angles
NP ¬[4  (4)]2 (6 3)2 of PQU are congruent, so P  PUQ. We are
¬0   9 or 3 given mP  32, so mPUQ  32.
RS ¬(2 )  2 2  (2 6)2
¬0   16 or 4
MP ¬(4   0)2 )
(6  32
¬16  9 or 5
QS ¬(2 )  5 2  (2 6)2
¬9   16 or 5
Each pair of corresponding sides does not have the
same measure. Therefore, MNP is not congruent
to QRS. MNP is congruent to SRQ.

Chapter 4 116
23. PQU is isosceles with base P U. PRT is 27. • Use the origin as vertex B of the triangle.
isosceles with base PT
. D
• Place side B along the positive x-axis.
Step 1 The base angles of PQU are congruent. • Position the triangle in the first quadrant.
Let x represent mP and mQUP. • Since D is on the x-axis, its y-coordinate is 0. Its
mP  mQUP  mPQU ¬180 x-coordinate is 6m because each side length is
x  x  40 ¬180 6m units long.
2x ¬140 • Since BCD is equilateral, the x-coordinate of C
x ¬70 is halfway between 0 and 6m, or 3m. We cannot
So, mP  mQUP  70. determine the y-coordinate in terms of m, so call
Step 2 The base angles of PRT are congruent. it n.
By Step 1 we know mP  70, so mT  y
C(3m, n)
70. Let y represent mR.
mP  mT  mR ¬180
70  70  y ¬180
y ¬40
So, mR  40. B(0, 0) D(6m, 0) x
24. RQS is isosceles with base Q S. The base angles
28. • Use the origin as vertex K of the triangle, the
of RQS are congruent, so RQS  RSQ. We
right angle.
are given mRQS  75, so mRSQ  75. Let y
L
• Place leg K  along the positive x-axis.
represent mR.
K
• Place leg J  along the positive y-axis.
mRQS  mRSQ  mR ¬180
• Since L is on the x-axis, its y-coordinate is 0. Its
75  75  y ¬180
x-coordinate is a because one of the leg lengths
y ¬30
is a units.
So, mR 30.
• Since J is on the y-axis, its x-coordinate is 0. Its
25. RQS is isosceles with base Q S. RPT is y-coordinate is b because the other leg length is
isosceles with base PT
. b units.
Step 1 The base angles of RQS are congruent, y
and mRQS  80 so mRSQ  80. Let x J(0, b)
represent mR.
mRQS  mRSQ  mR ¬180
80  80  x ¬180
x ¬20 K(0, 0) x
L(a, 0)
So, mR  20.
Step 2 The base angles of RPT are congruent.
Let y represent mP and mT.
mP  mT  mR ¬180 Chapter 4 Practice Test
y  y  20 ¬180
2y ¬160
y ¬80
Page 231
So, mP  mT  80. 1. b 2. a 3. c
26. • Use the origin as vertex T of the triangle. 4. PCD is obtuse because PCD has measure
• Place the base of the triangle along the positive greater than 90.
x-axis. 5. PAC is isosceles because P
APC
.
• Position the triangle in the first quadrant. 6. PBA, PBC, and PBD are right triangles
• Since I is on the x-axis, its y-coordinate is 0. Its because PBA D
, so PBA and PBD are right
x-coordinate is 4a because the base of the angles.
triangle is 4a units long. 7. m1  100 ¬180
• Since TRI is isosceles, the x-coordinate of R is m1 ¬80
halfway between 0 and 4a, or 2a. We cannot 8. m2  75 ¬180
determine the y-coordinate in terms of a, so call m2 ¬105
it b.
9. m3  m1 ¬m2
y R(2a, b) m3  80 ¬105
m3 ¬25
10. D  P, E  Q, F  R, D EP Q
,
F
E Q R, D
FP R
T(0, 0) I(4a, 0) x 11. F  H, M  N, G  J, F M
H N
,
G
M N J, F
G
H J 
12. X  Z, Y  Y, Z  X, X YZ 
Y,
Z
Y  YX, and X
Z
Z X


117 Chapter 4
13. JK ¬[2 1)]
 ( 2 [3)]
 (22 17. Given: ABE, BCE A B C
¬9  1 or 10
 AB  22, AC  44,
MN ¬[2   (6)]2 [1)]
 (72 AE  36, CD  36
¬164 6 or 80 A and C are
KL ¬(3 ) 22  [1)]
 (32 right angles.
¬1  16 or 17 Prove: ABE  CBD
NP ¬[5 2)]
 ( 2 (3 
1)2 E D
¬49  4 or 53
JL ¬[3 1)]
 ( 2 [1 
(2)]2 Proof: We are given that AB  22 and AC  44.
¬16  9 or 5 By the Segment Addition Postulate,
MP ¬[5 6)]
 ( 2 [3 
(7)]2 AB  BC ¬AC
¬121
  100 or 221
 22  BC ¬44 Substitution
Corresponding sides are not congruent, so JKL BC ¬22 Subtract 22 from each side.
is not congruent to MNP. Since AB  BC, then by the definition of
14. Given: JKM  JNM congruent segments, A BB C.
K
Prove: JKL  JNL We are given that AE  36 and CD  36. Then
also by the definition of congruent segments,
E
A C D
.
J L We are additionally given that both A and C
M
are right angles. Since all right angles are
congruent, A  C.
N B
Since A B C, A  C, and A E
C D, then by
Proof: SAS, ABE  CBD.
JKM JNM 18. C; JGH is isosceles with base J H
.
Given Step 1 FGH is a right triangle, so the sum of
the measures of the two acute angles
JK JN is 90.
KJL NJL mF  mH ¬90
CPCTC 28  mH ¬90
mH ¬62
JKL JNL JL JL Step 2 The base angles of JGH are congruent,
SAS Reflexive Prop. so H  HJG. Then mHJG  mH 
62. Let x represent mJGH.
15. FJH is isosceles with base J H
. The base angles mH  mHJG  mJGH ¬180
are congruent, so mJ  mFHJ. Let x represent 62  62  x ¬180
mJ and mFHJ. x ¬56
mJ  mFHJ  mJFH ¬180 So, mJGH  56.
x  x  34 ¬180
2x ¬146
x ¬73
So, mJ  mFHJ  73. Chapter 4 Standardized Test Practice
16. JFH is isosceles with base J H
. FGH is
H
isosceles with base F . Pages 232–233
Step 1 The base angles of FGH are congruent. 1. B; find when the populations will be equal. Let x
Let x represent mGFH and mGHF. represent the number of years after 2002.
mGFH  mGHF  mG ¬180 2010  150x ¬1040  340x
x  x  32 ¬180 2010 ¬1040  190x
2x ¬148 970 ¬190x
x ¬74 5.1 ¬x
So, mGFH  mGHF  74. The populations will be equal about 5 years after
Step 2 GHF  FHJ  GHJ by the Angle 2002, or 2007.
Addition Postulate. Since the population of Shelbyville is growing at a
mGHF  mFHJ ¬mGHJ faster rate than the population of Capitol City, the
74  mFHJ ¬152 following year, 2008, the population of Shelbyville
mFHJ¬78 will be greater than the population of Capitol
Step 3 The base angles of JFH are congruent, City.
so J  FHJ. Then mJ  mFHJ  2. C; grams measure mass, feet and meters measure
78. Let y represent mJFH. length, and liters measure volumes of liquid.
mJ  mFHJ  mJFH ¬180
78  78  y ¬180
y ¬24
So, mJFH  24.

Chapter 4 118
3. B; let x represent the length of the shadow. Use 12. The angle adjacent to the 105° angle has measure
the Pythagorean Theorem to solve for x to the 180  105 or 75. The tower is an isosceles
nearest foot. triangle, so its base angles are congruent. Let x
122 ¬92  x2 represent the measure of the angle at the top of
144 ¬81  x2 the tower.
63 ¬x2 x  75  75  180
8 ¬x x  30
The shadow is about 8 feet long. The measure of the angle at the top of the tower
4. D is 30.
5. D; the slope of the line in Kris’s graph can be 13. BCA  EFD since all right angles are
found using points (0, 3) and (4, 11). congruent. ACD F
 since the planes are
(y2  y1) 11  3 equidistant from the ground. CAB  FDE
  
4  0 or 2
(x2  x1) since the planes descend at the same angle. So,
The slope of the line in Mitzi’s graph is the same ABC  DEF by ASA.
as the slope of the line in Kris’s graph, or 2. So 14. Let x represent mA. A B
B C so A  C and
the line in Mitzi’s graph has equation y  2x  b, mC mA  x. Also, mB  3(mA) or 3x.
or 2x  y  b. The only answer of this form is x  x  3x ¬180
2x  y  1, so b  1 or b  1. 5x ¬180
6. B; mEFG ¬mFDE  mFED x ¬36
9x  7 ¬5x  5x mC  x
9x  7 ¬10x  36
7 ¬x 15a. The railroad ties that run across train tracks are
mEFG  9(7)  7 parallel. So, x  90 because the angle whose
 70 measure is x is supplementary to an angle that
7. C; ABD  CBD, so CBD is a flip of ABD corresponds to the angle whose measure is
over the x-axis. Corresponding points have the known to be 90.
same x-coordinate and opposite y-coordinates. 15b. Perpendicular lines, because the ties are
8. A; if we know BCC E
 then ACB  DCE by parallel and the tracks are parallel so all angles
SAS. are 90° angles.
9. 3s2(2s3  7)  6s5  21s2 15c. They are congruent. Sample answers: Both are
10. Brian’s second statement was the converse of his right angles; they are supplementary angles;
original statement. they are corresponding angles.
11. Explore: Creston (C) and Dixville (D) are 16a. Sample answer:
endpoints of the base of an isosceles triangle A
formed by Creston, Dixville, and Milford. We are 5x
looking for the coordinates (x, 1) to satisfy these
conditions.
y
(4x 1) (3x 13)
M(1, 3) B C
16b. From the Angle Sum Theorem, we know that
x mA  mB  mC  180. Substituting the
given measures, 5x  4x  1  3x  13  180.
Solve for x to find that x  14. Substitute 14 for x
C(1, 1) D(x, 1) to find the measures: 5x  5(14) or 70, 4x  1 
4(14)  1 or 55, and 3x  13  3(14)  13 or 55.
16c. If two angles of a triangle are congruent, then
Solve: The x-coordinate of the vertex angle is the sides opposite those angles are congruent
halfway between the x-coordinates of C and D. (Converse of the Isosceles Triangle Theorem).
(1  x) Since two sides of this triangle are congruent, it
So, 1 ¬ 2 is an isosceles triangle (Definition of Isosceles
2 ¬1  x
Triangle).
3 ¬x
Examine: C and D are the base vertices of the
isosceles triangle formed by C, D, and M.

119 Chapter 4
Chapter 5 Relationships in Triangles
Page 235 Getting Started 3.
B
x1  x2 y1  y2
1.   2 2  
12  4 5  15
,  ¬ 2, 2  O M
¬(4, 5)
x y y
2. 
1
x
2
,
1
2
2
2 2 
22  10 25  10
¬ 2,    A N C
¬(6, 7.5)
x1  x2 y1  y2
3.  ,  ¬ 2, 2
2 2  
19  (20) 7  (3)
 4. They intersect at the same point.
5.
¬(0.5, 5)
B F
4. m1  104 ¬180 E
m1 ¬76 H
5. m2  36 ¬104
m2 ¬68 A C
D
6. m3  104 ¬180
m3 ¬76 6. They intersect at the same point.
7. m4  40 7.
B
8. m5  m4 ¬104
m5  40 ¬104 N L
m5 ¬64
M
9. m5  m6 ¬90 C
64  m6 ¬90 A
m6 ¬26
8. They intersect at the same point.
10. m7  40 ¬180
9. See students’ work.
m7 ¬140
10. Acute: all intersect inside the triangle; obtuse:
11. m8  m6 ¬m4
perpendicular bisectors and altitudes intersect
m8  26 ¬40
outside the triangle; medians and angle bisectors
m8 ¬14
intersect inside the triangle; right: perpendicular
12. Let p and q represent the parts of the statement. bisectors intersect on the hypotenuse, medians
p: the three sides of one triangle are congruent to intersect inside the triangle, altitudes intersect on
the three sides of a second triangle the vertex of the right angle, and angle bisectors
q: the triangles are congruent intersect inside the triangle.
Statement (1): p → q
11. For an isosceles triangle, the perpendicular
Statement (2): q
bisector and median of the side opposite the
No conclusion can be reached because the truth of
vertex are the same as the altitude from the
p is unknown.
vertex angle and the angle bisector of the vertex
13. Let p and q represent the parts of the statement. angle. In an equilateral triangle, the
p: a polygon is a triangle perpendicular bisector and median of each side is
q: the sum of the measures of the angles is 180 the same as the altitude to each side and the
Statement (1): p → q angle bisector of the angle opposite each side.
Statement (2): p
Since the given statements are true, use the Law
of Detachment to conclude that the sum of the
measures of the angles of polygon JKL is 180. 5-1 Bisectors, Medians, and
Altitudes
Pages 236–237 Geometry Activity: Bisectors,
Medians, and Altitudes Page 242 Check for Understanding
1. B 1. Sample answer: Both pass through the midpoint
R T of a side. A perpendicular bisector is
P D perpendicular to the side of a triangle, and does
N not necessarily pass through the vertex opposite
S U the side, while a median does pass through the
A M C vertex and is not necessarily perpendicular to the
Q side.

2. They intersect at the same point.

Chapter 5 120
2. Sample answer: 5. Given: 
XYX
Z
M
Y and Z
N
 are medians.
M
Prove: Y ZN

X

M N

Z Y
3. Sample answer: An altitude and angle bisector of Proof:
a triangle are the same segment in an equilateral Statements Reasons
triangle. 1. 
XYXZ
, Y
M
 and Z
N
 1. Given
4. Find an equation of the perpendicular bisector are medians.
B
of A.
Z
2. M is the midpoint of X . 2. Def. of median
y Y
N is the midpoint of X.
A (3, 3)
B (3, 2) 3. XY  XZ 3. Def. of 
M
4. XM
Z
, X
N
N
Y
 4. Midpoint
Theorem
x 5. XM  MZ, XN  NY 5. Def. of 
6. XM  MZ  XZ, 6. Segment
XN  NY  XY Addition
C (1, 4) Postulate
1
 is 6, so the slope of the
B
The slope of A 7. XM  MZ  XN  NY 7. Substitution
B
perpendicular bisector is 6. The midpoint of A 8. MZ  MZ  NY  NY 8. Substitution

is 0, 5

2 . 9. 2MZ  2NY 9. Addition
y  y1 ¬m(x  x1) Property
10. MZ  NY 10. Division
y  5
2 ¬6(x  0)

Property
y  5
2 ¬6x

Z
11. MN
Y
 11. Def. of 
y ¬6x  5

2 12. XZY  XYZ 12. Isosceles
Find an equation of the perpendicular bisector of Triangle
C
B . The slope of BC
 is 3, so the slope of the Theorem
perpendicular bisector of B C is 1

3 . The midpoint Z
13. YY
Z
 13. Reflexive
C
of B  is (2, 1). Property
y  y1 ¬m(x  x1) 14. MYZ  NZY 14. SAS
y  (1) ¬1
3 (x  2)
 M
15. YZ
N 15. CPCTC
y  1 ¬1
3x  3
 2 6. Find x.
1 1 TQ ¬TR
y ¬3x  3
2x ¬8
Solve a system of equations to find the point of x ¬4
intersection of the perpendicular bisectors. Find y.
Find x.
PT ¬TR
y ¬6x  5
2

3y  1 ¬8
1 1 5
3x  3 ¬6x  2 3y ¬9
2x  2 ¬36x  15 y ¬3
Find z.
17 ¬38x Line  bisects P
R
, so z  4  7, or z  3.
17
 ¬x
38
17 Pages 243–245 Practice and Apply
Replace x with  
38 in one of the equations to 7. Find an equation of the median from D(4, 0) to
find the y-coordinate. F
the midpoint of E . The midpoint of E
F
 is (1, 5).
y ¬6
38   2
17
 5 The slope of the median is 1.
7 y  y1 ¬m(x  x1)
y ¬38 y  0 ¬1(x  4)
The coordinates of the circumcenter of ABC y ¬x  4
are 38 ,  38 .
17 7

121 Chapter 5
y Find x.
E (2, 4) F (0, 6) y ¬x  4
y ¬3
2x  6

3 x  6 ¬x  4
2
6 ¬52x  4

5
x 2 ¬2 x
D (4, 0) 4
5 ¬x
Replace x with 4 
5 in one of the equations to find
the y-coordinate.
Find an equation of the median from F(0, 6) to the
midpoint of D E
. The midpoint of DE
 is (1, 2). The y ¬4
5  4

slope of the median is 4. y ¬4 4

5
y  y1 ¬m(x  x1)
The coordinates of the orthocenter are 4
5 , 4 5 .
 4
y  6 ¬4(x  0)
y  6 ¬4x 9. y
y ¬4x  6 E (2, 4) F (0, 6)
Solve a system of equations to find the point of
intersection of the medians.
Find x.
y ¬x  4
y ¬4x  6 x
4x  6 ¬x  4 D (4, 0)
6 ¬3x  4
2 ¬3x
2 ¬x
3 Find an equation of the perpendicular bisector
Replace x with 2 
3 in one of the equations to find E
of D . The slope of D is 2
E 
3 , so the slope of the
the y-coordinate.
3
y  2 perpendicular bisector is 2. The midpoint of D E
3 4

is (1, 2).
y  3 1

3 y  y1 ¬m(x  x1)
The coordinates of the centroid are 2
3 , 3 3 .
 1 y  2 ¬3
2 (x  1)

8. y
y  2 ¬3 3
2x  2

E (2, 4) F (0, 6) 3
y ¬2 x  1

2
Find an equation of the perpendicular bisector of
F
E . The slope of E
F
 is 1, so the slope of the
perpendicular bisector is 1. The midpoint of EF
x is (1, 5).
D (4, 0) y  y1 ¬m(x  x1)
y  5 ¬1[x  (1)]
y  5 ¬x  1
y ¬x  4
Find an equation of the altitude from D(4, 0) to
Solve a system of equations to find the point of
F
E . The slope of E
F
 is 1, so the slope of the
intersection of the perpendicular bisectors.
altitude is 1.
Find x.
y  y1 ¬m(x  x1)
y  0 ¬1(x  4) y ¬x  4
y ¬x  4 y ¬3 1
2x  2

Find an equation of the altitude from F(0, 6) to
x  4 ¬3 1
2x  2

E
D . The slope of D
E is 2
3 , so the slope of the
altitude is 3 7 ¬5x
2. 2 2
y  y1 ¬m(x  x1) 7 ¬5x
y  6 ¬3
2 (x  0)
 7 ¬x
5
y  6 ¬3

2x Replace x with 7

5 in one of the equations to find
the y-coordinate.
y ¬3
2x  6

Solve a system of equations to find the point of y ¬7
5 4

intersection of the altitudes. 13
y ¬5
The coordinates of the circumcenter are
75, 153  or 1 25, 2 35.
Chapter 5 122
10. Given:  is the perpendicular bisector of A
CD B
. 12. Given: 
GL is a median of EGH.
D
E is a point on C. M
J is a median of IJK.
Prove: EB  EA EGH  IJK
C L
Prove: G J M

E B E I

D L M
A
Proof: H K
D
C  is the perpendicular bisector of A
B. By G J
definition of perpendicular bisector, D is the Proof:
midpoint of A B. Thus, A
D
B D
 by the Midpoint Statements Reasons
Theorem. CDA and CDB are right angles by 1. 
GL is a median of EGH, 1. Given
definition of perpendicular. Since all right angles M
J is a median of IJK,
are congruent, CDA  CDB. Since E is a point and EGH  IJK.
D
on C , EDA and EDB are right angles and
are congruent. By the Reflexive Property, 2. 
GHJ
K, GHL  2. CPCTC
D
E E D
. Thus, EDA  EDB by SAS. JKM, E
H
  IK

B
E E A
 because CPCTC, and by definition of 3. EH  IK 3. Def. of 
congruence, EB  EA.
11. Given: UVW is isosceles with vertex angle UVW. L
4. EL
H, IM
M
K
 4. Def. of median
V
Y is the bisector of UVW. 5. EL  LH, IM  MK 5. Def. of 
Prove: Y V is a median. 6. EL  LH  EH, IM  6. Segment
U MK  IK Addition
Y V Postulate
W 7. EL  LH  IM  MK 7. Substitution
Proof: 8. LH  LH  MK  MK 8. Substitution
Statements Reasons 9. 2LH  2MK 9. Addition
1. UVW is an isosceles 1. Given Property
triangle with vertex angle
10. LH  MK 10. Division
UVW, YV is the bisector
Property
of UVW.
H
11. LM
K
 11. Def of 
V
2. U W V 2. Def. of isosceles
 12. GHL  JKM 12. SAS
3. UVY  WVY 3. Def. of angle L
13. GJ
M
 13. CPCTC
bisector
4. Y
V
Y V
 4. Reflexive 13. 
MS is an altitude of MNQ, so MSQ is a right
Property angle.
5. UVY  WVY 5. SAS m1  m2 ¬90
3x  11  7x  9 ¬90
Y
6. UW
Y
 6. CPCTC
10x  20 ¬90
W
7. Y is the midpoint of U. 7. Def. of midpoint 10x ¬70
x ¬7
V
8. Y is a median. 8. Def. of median
m2 ¬7(7)  9
¬58
S
14. M  is a median of MNQ, so QSSN
.
3a  14 ¬2a  1
a  14 ¬1
a ¬15
mMSQ ¬7a  1
¬7(15)  1
¬106
S
M  is not an altitude of MNQ because
mMSQ  106.

123 Chapter 5
P
15. W  is an angle bisector, so mHWP  23. 
PX is an altitude of PQR, so PXR is a
1mHWA. right angle.
2
mPXR ¬90
x  12 ¬1
2 (4x  16)

2a  10 ¬90
x  12 ¬2x  8 2a ¬80
12 ¬x  8 a ¬40
20 ¬x Z
24. R  bisects PRQ, so mPRZ  mZRQ.
P
W  is a median, so AP
P H. 4b  17 ¬3b  4
3y  11 ¬7y  5 b  17 ¬4
11 ¬4y  5 b ¬13
16 ¬4y mPRZ ¬4(13)  17
4 ¬y ¬35
mPAW ¬3x  2 Y
25. Q  is a median, so PY  YR.
¬3(20)  2 2c  1 ¬4c  11
¬58 1 ¬2c  11
mPWA  mHWP because W P is an angle 10 ¬2c
bisector. 5 ¬c
So, mPWA ¬x  12 PR ¬PY  YR
¬20  12 or 32 ¬2(5)  1  4(5)  11
mWPA  mPAW  mPWA ¬180 ¬10  1  20  11
mWPA  58  32 ¬180 ¬18
mWPA ¬90
26. 
QY is perpendicular to PR, so QYR is a right
WPA is a right angle, so W P  is also an altitude.
angle.
16. P
W  bisects AH
, so A
PP H. mQYR ¬90
6r  4 ¬22  3r 7b  6 ¬90
3r  4 ¬22 7b ¬84
3r ¬18 b ¬12
r ¬6 27. X is the midpoint of ST
.
P
W  is perpendicular to AH, so mHPW  90. y
mWHA  mHWP  mHPW ¬180
8q  17  10  q  90 ¬180
T (1, 8)
9q  117 ¬180 S (1, 6)
9q ¬63
q ¬7
mHWP ¬10  q
¬10  7 R (3, 3)
¬17
17. Always; each median is always completely x
contained in the interior of the triangle, so the
x x y y 1 
 , 6
2   2 2
1  8
intersection point must also be inside the 1
 2 1
,  2
¬  
2
triangle. ¬(0, 7)
18. sometimes; true for a right triangle but false for
an equilateral triangle 28. d  (x
 2
x1)2 
(y2 
y1)2
19. Never; an angle bisector lies between two sides of y
the triangle and is contained in the triangle up to T (1, 8)
the point where it intersects the opposite side.
so, the intersection point of the three angle S (1, 6)
bisectors must also be inside the triangle.
20. sometimes; true for an obtuse triangle but false
for an acute triangle
R (3, 3)
21. S
P  is a median of PQR, so Q SS R
.
10x  7 ¬5x  3
x
5x  7 ¬3
5x ¬10 RX ¬
(0  3
)2  (7
 3)2
x ¬2
¬
9  16 or 5 units
22. D
A  is an altitude of ABC, so ADC is a right
angle.
mADC ¬90
4x  6 ¬90
4x ¬96
x ¬24

Chapter 5 124
(y  y )
29. m  
2
1
(x2  x1) 13. mCEA  mCEA  13. Substitution
7 3 4 180

0  3 or  3
y 14. 2(mCEA)  180 14. Substitution
15. mCEA  90 15. Division
T (1, 8)
Property
S (1, 6) 16. CEA and CEB are 16. Def. of rt. 
rt. .
D
17. CA
B
 17. Def. of 
R (3, 3) 18. 
CD is the perpendicular 18. Def. of 
B
bisector of A. bisector
x 19. C and D are on the 19. Def. of points
perpendicular bisector on a line
30. No, RX is not an altitude of RST. The slope B
of A.
T
of S is 1. The product of the slopes of S
T
 and R
X

32. Given: BAC, P is in the interior of BAC,
is 4, not 1. Thus, the segments are not
3 PD  PE
perpendicular.
Prove: 
AP is the angle bisector of BAC
y
D
A B
T (1, 8) P
S (1, 6) E
C
Proof:
R (3, 3) Statements Reasons
1. BAC, P is in the interior 1. Given
x of BAC, PD  PE
D
2. P PE
 2. Def. of 
31. Given: 
CAC B, A
DBD

D
3. PA
B
, P
E
 
AC 3. Distance from a
Prove: C and D are on the perpendicular point to a line is
B
bisector of A. measured along
C  segment from
the point to the
line.
A
E B 4. ADP and AEP are 4. Def. of 
rt. 
D 5. ADP and AEP are 5. Def. of rt. 
Proof: rt. s
Statements Reasons P
6. AA
P
 6. Reflexive
Property
A
1. CC
B
, A
D
B
D
 1. Given
7. ADP  AEP 7. HL
D
2. CC
D
 2. Reflexive
Property 8. DAP  EAP 8. CPCTC
3. ACD  BCD 3. SSS  is the angle bisector
9. AP 9. Def. of  bisector
4. ACD  BCD 4. CPCTC of BAC

E
5. CC
E
 5. Reflexive
Property
6. CEA  CEB 6. SAS
E
7. AB
E
 7. CPCTC
B
8. E is the midpoint of A. 8. Def. of
midpoint
9. CEA  CEB 9. CPCTC
10. CEA and CEB form 10. Def. of linear
a linear pair. pair
11. CEA and CEB are 11. Supplement
supplementary. Theorem
12. mCEA  mCEB  12. Def. of suppl. 
180

125 Chapter 5
, 
33. Given: ABC with angle bisectors AD BE, and Page 245 Maintain Your Skills
, K
CF P
AB, K
Q
B C
, K
RA C 43. Sample answer:
Prove: KP  KQ  KR • Use the origin as vertex A of the triangle.
A • Place side AB along the positive x-axis.
R
P E • Position the triangle in the first quadrant.
F K • Since B is on the x-axis, its y-coordinate is 0.
D Its x-coordinate is n because AB is n units long.
B C • Since ABC is equilateral, the x-coordinate of C
Q
is halfway between 0 and n, or n 
2 . We cannot
Proof: determine the y-coordinate in terms of n,
Statements Reasons so call it m.
1. ABC with angle bisectors 1. Given y
C (n–2, m)
, 
AD , K
BE, and CF P
A B,
Q
KB C, K
R
A C

2. KP  KQ, KQ  KR, 2. Any point on
KP  KR the  bisector is A(0, 0) B(n, 0) x
equidistant
from the sides 44. Sample answer:
of the angle. • Use the origin as vertex D of the triangle.
3. KP  KQ  KR 3. Transitive • Place the base of the triangle along the positive
Property x-axis.
• Position the triangle in the first quadrant.
34. The flag is located at the intersection of the angle • Since F is on the x-axis, its y-coordinate is 0.
bisector between Amesbury and Stearns Roads Its x-coordinate is a because the base of the
and the perpendicular bisector of the segment triangle is a units long.
joining Grand Tower and the park entrance. • Since DEF is isosceles, the x-coordinate of E is
16  2 
 (6) halfway between 0 and a, or a 
2 . We cannot
35.  3 4
determine the y-coordinate in terms of a, so call
8  4
 12 it b.
36.  3 8
y
37. C (–6, 12) y E(a–2, b)
12
E(5, 10)
A(16, 8)
8
D(–2, 8)
F (9, 6)
4
B(2, 4) D(0, 0) F(a, 0) x

–4 O 4 8 12 16 x 45. Sample answer:


• Use the origin as vertex H of the triangle.
38. The centroid has the same coordinates as the • Place leg HI along the positive y-axis.
means of the vertices’ coordinates. •GI is the hypotenuse so H is a right angle and
39. The altitude will be the same for both triangles, leg GH is on the x-axis. Position G
H on the
and the bases will be congruent, so the areas will positive x-axis.
be equal. • Since G is on the x-axis, its y-coordinate is 0.
40. Sample answer: You can balance a triangle on a Its x-coordinate is x because G H is x units long.
pencil point by locating the center of gravity of • Since I is on the y-axis, its x-coordinate is 0.
the triangle. Answers should include the Its y-coordinate is 3x because HI  3GH or 3x.
following. y
•centroid I (0, 3x)

H(0, 0) G(x, 0) x
41. C; GJ  JH, so J is the midpoint of GH and 46. 
MTM R
 by the converse of the Isosceles
J
F is a median of FGH. Triangle Theorem.
42. D; 3x ¬0.3y 47. 5  11 by the Isosceles Triangle Theorem.
10.0x ¬y
y 48. 7  10 by the Isosceles Triangle Theorem.
10.0 ¬x
L
49. M M N by the converse of the Isosceles
Triangle Theorem.
50. It is everywhere equidistant.

Chapter 5 126
 5
51. 3 3
8  16 because 8  1
6
6. 2. A
5 5
52. 2.7  3 because 3  1.6
.
19 19 C B
53. 4.25  4 because  4  4.75.
 
18 19 18 Sample answer: mCAB, mACB, mABC; B C
,
54. 
25   27 because  25  0.72 and
  
B
A , A
C

19

27  0.7
 03
. 3. Grace; she placed the shorter side with the
smaller angle and the longer side with the larger
angle.
Page 246 Reading Mathematics: Math Words 4. Explore: Compare the measure of 2 to the
and Everyday Words measures of 1 and 4
Plan: Use properties and theorems of real
1. Sample answer: A median of a triangle is a
numbers to compare the angle measures.
segment that has one endpoint at a vertex and
Solve: Compare m1 to m2.
the other at the midpoint of the opposite side; the
By the Exterior Angle Theorem, m2  m1 
everyday meaning says it is a paved or planted
m4. Since angle measures are positive numbers
strip in the middle of a highway.
and from the definition of inequality,
2. Sample answer: the intersection of two or more m2  m1.
lines or curves, the top of the head, the highest Compare m4 to m2.
point Again, by the Exterior Angle Theorem,
3. Sample answer: in a trapezoid, the segment m2  m1  m4. The definition of inequality
joining the midpoints of the legs; the middle value states that if m2  m1  m4 then
of a set of data that has been arranged into an m2  m4.
ordered sequence Examine: m2 is greater than m1 and m4.
4. Sample answer: a separate piece of something; a Therefore, 2 has the greatest measure.
portion cut off from a geometric figure by one or 5. Explore: Compare the measure of 3 to the
more points, lines, or planes. measures of 2 and 5.
Plan: Use properties and theorems of real
numbers to compare the angle measures.
Solve: Compare m2 to m3.
5-2 Inequalities and Triangles By the Exterior Angle Theorem, m3  m2 
m5. Since angle measures are positive numbers
and from the definition of inequality,
Page 249 Geometry Activity: Inequalities for m3  m2.
Sides and Angles of Triangles Compare m5 to m3.
1. Sample answer: It is the greatest measure. Again, by the Exterior Angle Theorem, m3 
2. Sample answer: It is the least measure. m2  m5. The definition of inequality
3. See students’ work. states that if m3  m2  m5 then
4. Sample answer: The measures of the angles m3  m5.
opposite the sides are in the same order as the Examine: m3 is greater than m2 and m5.
lengths of the respective sides. Therefore, 3 has the greatest measure.
6. Explore: Compare the measure of 3 to the
Page 251 Check for Understanding measures of 1, 2, 4, and 5.
1. never; Plan: Use properties and theorems of real
numbers to compare the angle measures.
L Solve: From Exercise 4, m2  m1 and
m2  m4.
From Exercise 5, m3  m2 and m3  m5.
Then by transitivity, m3  m1 and
K J m3  m4.
Examine: m3 is greater than m1, m2, m4,
J is a right angle. and m5. Therefore, 3 has the greatest
Since mJ = 2  mK, 90 ¬2  mK measure.
45 ¬mK 7. By the Exterior Angle Inequality Theorem,
mL  180  90  45  45 m1  m4 and m1  m5  m6
mL  mK, so LKJ is isosceles, and KJ  LJ. so m1  m5 and m6. Thus, the measures of
Let KJ  LJ  1. If the statement in the problem 4, 5, and 6 are all less than m1.
is true, then LK  2. Since LKJ is a right
8. By the Exterior Angle Inequality Theorem,
triangle, the Pythagorean Theorem applies.
m1  m5  m6 and m7  m5  m6
12  12 ¬22
so m1  m6 and m7  m6. Thus, the
2 ¬4
measures of 1 and 7 are greater than m6.
This is a false statement, so the statement in the
problem is never true.

127 Chapter 5
9. By the Exterior Angle Inequality Theorem, 19. Explore: Compare the measure of 7 to the
m7  m2  m3 and m7  m5  m6, measures of 3 and 5.
so m7  m2, m7  m3, m7  m5, Plan: Use properties and theorems of real
and m7  m6. Thus, the measures of 2, 3, numbers to compare angle measures.
5, and 6 are less than m7. Solve: Compare m7 to m3.
10. The side opposite WXY is longer than the side By the Exterior Angle Inequality Theorem,
opposite XYW, so mWXY  mXYW. m7  m3.
11. The side opposite XZY is shorter than the side Compare m7 to m5.
opposite XYZ, so mXZY  mXYZ. By the Exterior Angle Inequality Theorem,
m7  m5.
12. The side opposite WYX is shorter than the side
Examine: m7 is greater than m3 and m5.
opposite XWY, so mWYX  mXWY.
Therefore, 7 has the greatest measure.
E
13. A  is opposite a 30° angle, and EB
 is opposite a
20. Explore: Compare the measure of 1 to the
110° angle. If one angle of a triangle has a greater
measures of 2 and 6.
measure than another angle, then the side
Plan: Use properties and theorems of real
opposite the greater angle is longer than the side
numbers to compare angle measures.
opposite the lesser angle, so AE  EB.
Solve: Compare m1 to m2.
E
14. C  is opposite CDE, and mCDE  By the Exterior Angle Inequality Theorem,
180  (50  55) or 75. CD
 is opposite a 55° angle. m1  m2.
If one angle of a triangle has a greater measure Compare m1 to m6.
than another angle, then the side opposite the By the Exterior Angle Inequality Theorem,
greater angle is longer than the side opposite the m1  m6.
lesser angle, so CE  CD.
Examine: m1 is greater than m2 and m6.
C
15. B  is opposite BEC, and mBEC  Therefore, 1 has the greatest measure.
180  (40  100) or 40. EC is opposite a 40°
21. Explore: Compare the measure of 7 to the
angle. Thus, BC  EC.
measures of 5 and 8.
16. Second base; the angle opposite the side from Plan: Use properties and theorems of real
third base to second base is smaller than the numbers to compare angle measures.
angle opposite the side from third to first. Solve: Compare m7 to m5.
Therefore, the distance from third to second is By the Exterior Angle Inequality Theorem,
shorter than the distance from third to first. m7  m5.
Compare m7 to m8.
Pages 252–253 Practice and Apply By the Exterior Angle Inequality Theorem,
17. Explore: Compare the measure of 1 to the m7  m8.
measures of 2 and 4. Examine: m7 is greater than m5 and m8.
Plan: Use properties and theorems of real Therefore, 7 has the greatest measure.
numbers to compare angle measures. 22. Explore: Compare the measure of 2 to the
Solve: Compare m1 to m2. measures of 6 and 8.
By the Exterior Angle Inequality Theorem, Plan: Use properties and theorems of real
m1  m2. numbers to compare angle measures.
Compare m1 to m4. Solve: Compare m2 to m6.
By the Exterior Angle Inequality Theorem, By the Exterior Angle Inequality Theorem,
m1  m4. m2  m6.
Examine: m1 is greater than m2 and m4. Compare m2 to m8.
Therefore, 1 has the greatest measure. Let x be the measure of the third angle of the
18. Explore: Compare the measure of 2 to the triangle whose other angles are 3 and 4. Then,
measures of 4 and 6. by the Exterior Angle Inequality Theorem,
Plan: Use properties and theorems of real m2  (x  m8). Since angle measures are
numbers to compare angle measures. positive numbers and from the definition of
Solve: Compare m2 to m4. inequality, m2  m8.
By the Exterior Angle Inequality Theorem, Examine: m2 is greater than m6 and m8.
m2  m4. Therefore, 2 has the greatest measure.
Compare m2 to m6. 23. By the Exterior Angle Inequality Theorem,
By the Exterior Angle Inequality Theorem, m5  m7, m5  m10, and
m2  m6. m5  m2  m8 so m5  m2 and
Examine: m2 is greater than m4 and m6. m5  m8. Thus, the measures of 2, 7, 8,
Therefore, 2 has the greatest measure. and 10 are all less than m5.
24. By the Exterior Angle Inequality Theorem,
m4  m6, m1  m6  m9 so m1  m6
and m11  m6  m9 so m11  m6. Thus,
the measures of 1, 4, and 11 are all greater
than m6.

Chapter 5 128
25. By the Exterior Angle Inequality Theorem, 36. Given: 
PRP
Q; QR  QP
m3  m10 and m5  m10. Thus, the Prove: mP  mQ
measures of 3 and 5 are greater than m10. R
26. By the Exterior Angle Inequality Theorem,
m1  m3, m1  m6, and m1  m9.
Thus, the measures of 3, 6, and 9 are all less
than m1.
Q P
27. By the Exterior Angle Inequality Theorem,
Proof:
m8  m9, m7  m9, m3  m9, and
m1  m9. Thus, the measures of 8, 7, 3, Statements Reasons
and 1 are all greater than m9. 1. QR  QP 1. Given
28. By the Exterior Angle Inequality Theorem, 2. mP  mR 2. If one side of a  is
m8  m2, m8  m4, m8  m5, and longer than another,
m8  m9. Thus, the measures of 2, 4, 5, then the  opp. the
and 9 are all less than m8. longer side is greater
29. The side opposite KAJ is shorter than the side than the  opposite
opposite AJK, so mKAJ  mAJK. the shorter side.
30. The side opposite MJY is longer than the side R
3. P P
Q 3. Given
opposite JYM, so mMJY  mJYM.
4. Q  R 4. Isosceles  Theorem
31. The side opposite SMJ is longer than the side
opposite MJS, so mSMJ  mMJS. 5. mQ  mR 5. Def. of 
32. The side opposite AKJ is longer than the side 6. mP  mQ 6. Substitution
opposite JAK, so mAKJ  mJAK.
33. The side opposite MYJ is shorter than the side 37. 
ZY is opposite a 45° angle. Y
R is opposite YZR,
opposite JMY, so mMYJ  mJMY. and mYZR  180  (95  45) or 40. If one angle
34. The side opposite JSY is longer than the side of a triangle has a greater measure than another
opposite JYS, so mJSY  mJYS. angle, then the side opposite the greater angle is
longer than the side opposite the lesser angle, so
35. Given:  J
JM L

ZY  YR.
L
JK L
Prove: m1  m2 R
38. S  is opposite a 43° angle. Z
S is opposite ZRS,
L and mZRS  180  (43  97) or 40. If one angle
of a triangle has a greater measure than another
2 angle, then the side opposite the greater angle is
longer than the side opposite the lesser angle, so
1 SR  ZS.
K M
J Z
39. R  is opposite a 97° angle. S
R is opposite a 43°
Proof: angle. If one angle of a triangle has a greater
measure than another angle, then the side
Statements Reasons
opposite the greater angle is longer than the side
M
1. JJL
, J
L
KL
 1. Given opposite the lesser angle, so RZ  SR.
2. LKJ  LJK 2. Isosceles  Y
40. Z  is opposite a 45° angle. R
Z is opposite a 95°
Theorem angle. If one angle of a triangle has a greater
3. mLKJ  mLJK 3. Def. of  measure than another angle, then the side
4. m1  mLKJ 4. Ext.  Inequality opposite the greater angle is longer than the side
Theorem opposite the lesser angle, so ZY  RZ.
Y
41. T  is opposite TZY. mTZY  mZYT  91, so
5. m1  mLJK 5. Substitution
mTZY  91  66 or 25. Z Y
 is opposite an 89°
6. mLJK  m2 6. Ext.  Inequality angle. If one angle of a triangle has a greater
Theorem measure than another angle, then the side
7. m1  m2 7. Trans. Prop. of opposite the greater angle is longer than the side
Inequality opposite the lesser angle, so TY  ZY.
Y
42. T  is opposite TZY. mTZY  mZYT  91, so
mTZY  91  66 or 25. Z T
 is opposite a 66°
angle. If one angle of a triangle has a greater
measure than another angle, then the side
opposite the greater angle is longer than the side
opposite the lesser angle, so TY  ZT.

129 Chapter 5
43. KL ¬ (1  3)2 (5  2)2 48. mP  mQ  mR ¬180
¬ 16  9 or 5 9n  4  4n  16  68  2n ¬180
11n  48 ¬180
LM ¬ [3 (1)]
 2 (7  5)2
11n ¬132
¬ 4  14
4
n ¬12
¬ 148
mP ¬9(12)  4 or 104
¬12.2 mQ ¬4(12)  16 or 32
KM ¬ (3  3)2 (7  2)2 mR ¬68  2(12) or 44
¬ 36  8
1 mQ  mR  mP, so the sides of PQR in
¬ 117 R
order from shortest to longest are P , P
Q
, QR.
¬10.8 49. mP  mQ  mR ¬180
The side lengths in order from least to greatest 3n  20  2n  37  4n  15 ¬180
are KL, KM, LM, so the angles in order from the 9n  72 ¬180
least to the greatest measure are M, L, K. 9n ¬108
n ¬12
44. Sample answer: Draw a triangle that satisfies the
given information. Then draw the medians and mP ¬3(12)  20 or 56
measure them to find their lengths. mQ ¬2(12)  37 or 61
mR ¬4(12)  15 or 63
C mP  mQ  mR, so the sides of PQR in
N R
order from shortest to longest are Q , P
R, 
PQ.
M
50. mP  mQ  mR ¬180
4n  61  67  3n  n  74 ¬180
A O B 2n  202 ¬180
2n ¬22
In ABC, AB  AC  BC. Measure A M, B
N, and
n ¬11

CO . In this triangle CO  BN  AM.
mP ¬4(11)  61 or 17
45. 8x  4  11x  37  5x  21 ¬180
mQ ¬67  3(11) or 100
24x  12 ¬180
mR ¬11  74 or 63
24x ¬192
mP  mR  mQ, so the sides of PQR in
x ¬8
R
order from shortest to longest are Q , P
Q, P
R.
8x  4 ¬8(8)  4 or 68
11x  37 ¬11(8)  37 or 51 51. The angle opposite the side with length 3x inches
5x  21 ¬5(8)  21 or 61 has measure 180  163 or 17. The angle opposite
the side with length 2(y  1) inches has measure
68  61  51, so the lengths of the legs of the trip x
180  (17  75) or 88. Thus, 2(y  1)  3.
in order from greatest to least are Phoenix to x
2(y  1) ¬3
Atlanta, Des Moines to Phoenix, Atlanta to
x
Des Moines. y  1 ¬6
46. mP  mQ  mR ¬180 x
y ¬6  1
9n  29  93  5n  10n  2 ¬180
x 6
14n  124 ¬180 y ¬¬ 6
14n ¬56 52. A
n ¬4
mP ¬9(4)  29 or 65
mQ ¬93  5(4) or 73 C
B
mR ¬10(4)  2 or 42 T M
mR  mP  mQ, so the sides of PQR in Given: ABC is scalene; A M is the median from
order from shortest to longest are PQ
, Q
R, P
R
. C
A to B ; A
T
 is the altitude from A to B
C.
47. mP  mQ  mR ¬180 Prove: AM  AT
12n  9  62  3n  16n  2 ¬180 ATB and ATM are right angles by the
25n  55 ¬180 definition of altitude and mATB  mATM
25n ¬125 because all right angles are congruent. By the
n ¬5 Exterior Angle Inequality Theorem,
mP ¬12(5)  9 or 51 mATB  mAMT. So, mATM  mAMT by
mQ ¬62  3(5) or 47 Substitution. If one angle of a triangle has a
mR ¬16(5)  2 or 82 greater measure than another angle, then the
mQ  mP  mR, so the sides of PQR in side opposite the greater angle is longer than the
order from shortest to longest are PR
, Q
R
, PQ
. side opposite the lesser angle. Thus, AM  AT.

Chapter 5 130
53. mA  mB  mC ¬180
 2   2 
2 2
2y  12  y  18  4y  12 ¬180 BF ¬  9  71  12
101
7y  6 ¬180
7y ¬174
y ¬25

¬ 9
4  4
  1
8

mA ¬2(25)  12 or 62 ¬


90

4

mB ¬25  18 or 7 90


mC ¬4(25)  12 or 112 2

 2   2 
2 2
mC  mA so 3x  15 ¬4x  7 DF ¬  12  71  3
101
15 ¬x  7

CB  0, so 4x  7 ¬0
8 ¬x

¬ 9 8
4  4
  1

4x ¬7 ¬
90

4

7
x ¬4 90


7 2
Thus, 4  x  8.
BF  DF, so B
FD F and F is the midpoint of
54. Sample answer: The largest corner is opposite the
D
B. Therefore, E
F
 is a perpendicular bisector of
longest side. Answers should include the
D
B.
following. ab
0  
• the Exterior Angle Inequality Theorem 60. x   3 ¬ a 
3
b

• the angle opposite the side that is 51 feet long 0  0c


y 3 ¬3c
55. A; n  p  180  m so m  n  180  p a 
x  D has coordinates D 3 , 3.
b c
2 x  3 ¬2 5 
1
56. D; 1
 
B
61. Label the midpoints of A, B
C, and C
A
 as E, F,
5x  30 ¬4(x  1) and G respectively. Then the coordinates of
5x  30 ¬4x  4 ab
E, F, and G are a
2, 0, 2, 2, and 2, 2
c b c
x  30 ¬4
x ¬26 F
 c
respectively. The slope of A a  b , and the
slope
Page 254 Maintain Your Skills c
D
of A
a  F
b , so D is on A. The slope of
B
57. D is on C with CDD B
 and the slope of C
D

B
. c c
equals the slope of D G
B
b  D
2a and the slope of B
b  2a ,
DB ¬ (12 9)2 (3  1
2)2
2c
¬9  81 G
so D is on B. The slope of C
E
   and the
2b  a
¬90 2
c
D
slope of C
2b  E
a , so D is on C. Since D is
3 12
B
slope of D ¬
12  9 F
on A , B
G
, and C
E
, it is the intersection point of
9
¬3 or 3 the three segments.
D is 9 units down and 3 units to the right from B. 62. T  X, U  Y, V  Z, T UX Y
,
The point C that is 9 units down and 3 units to V
U Y Z
, TV
X Z 
the right from D has coordinates (12  3, 3  9) or 63. C  R, D  S, G  W, C DR S
,
(15, 6). Check that CD  DB. G
D S W
, CGR W 
CD ¬ (15  12)2 
(6  3)2 64. B  D, C  G, F  H, B C
D G,
¬9  81 F
C G H, BF
D H
¬90 65. slope of line containing (4, 8) and (2, 1):
C has coordinates (15, 6). 1  8 9 9
2 4  2  2
C
58. slope of B   3 slope of line containing (x, 2) and (4, 5):
3 
8 5
slope of 
AD 12  3 or  9 52 3
 x  4  x

4 
No, AD
 is not an altitude of ABC because
5 Solve for x. Since the lines are perpendicular,
 9 (3)  1.
 3 2
D
59. slope of B  3 90

   x ¬9
4 
2
71 3(9) ¬2(4  x)
2 6

 ¬ 
F
slope of E 27 ¬8  2x
1012 6
 19 ¬2x
9.5 ¬x
11
¬ 2 66. true; 2ab ¬2(2)(5)
41

2 ¬20
¬1
 67. false; c(b  a) ¬6(5  2)
3
¬6(3)

BD because 1
EF 3 (3)  1.

¬18

131 Chapter 5
68. true; a  c ¬2  6 or 8 3. Sample answer: ABC is scalene.
a  b ¬2  5 or 7 Given: ABC; AB  BC; BC  AC; AB  AC
a  c ¬a  b since 8  7 Prove: ABC is scalene.
A
Page 254 Practice Quiz 1
1. BD ¬DC C
B
4x  9 ¬7x  6
Proof:
9 ¬3x  6
Step 1 Assume ABC is not scalene.
15 ¬3x
Case 1: ABC is isoceles.
5 ¬x
If ABC is isosceles, then AB  BC,
2. D
A B C, so mADC  90. BC  AC, or AB  AC.
2y  6 ¬90 This contradicts the given information, so
2y ¬96 ABC is not isosceles.
y ¬48 Case 2: ABC is equilateral.
3. Never; a median is a segment from one vertex to In order for a triangle to be equilateral, it
the side opposite the vertex and never intersects must also be isosceles, and Case 1 proved
any other vertex. that ABC is not isosceles. Thus, ABC
4. Always; an angle bisector lies between two sides is not equilateral.
of the triangle and is contained in the triangle up Therefore, ABC is scalene.
to the point where it intersects the opposite side. 4. x 5
So, the intersection point of the three angle 5. The lines are not parallel.
bisectors must also be inside the triangle.
6. The lines are not parallel.
5. sometimes; true for an obtuse triangle but false
7. Given: a  0
for an acute triangle 1
Prove: a 0
6. sometimes; true for right triangles but false for
other triangles Proof:
1
7. No triangle; by Exercise 4, the angle bisectors Step 1 Assume a
0.
1 1
always intersect at a point in the interior of the Step 2 a
0; a  a
0  a, 1
0
triangle. Step 3 The conclusion that 1
0 is false, so the
1
8. mT  mS  mU, so the sides of STU in assumption that a
0 must be false.
1
order from longest to shortest are SU, T
U
, ST. Therefore, a  0.
9. mQ  mR  mS ¬180 8. Given: n is odd.
3x  20  2x  37  4x  15 ¬180 Prove: n2 is odd.
9x  72 ¬180 Proof:
9x ¬108 Step 1 Assume n2 is even.
x ¬12 Step 2 n is odd, so n can be expressed as 2a  1.
mQ ¬3(12)  20 or 56 n2 ¬(2a  1)2 Substitution
mR ¬2(12)  37 or 61 ¬(2a  1)(2a  1) Multiply.
mS ¬4(12)  15 or 63 ¬4a2  4a  1 Simplify.
¬2(2a2  2a)  1 Distributive Property
10. mQ  mR  mS, so the sides of QRS in
Step 3 2(2a2  2a)  1 is an odd number. This
order from shortest to longest are RS, Q
S, Q
R
.
contradicts the assumption, so the
assumption must be false. Thus n2 is odd.
9. Given: ABC
5-3 Indirect Proof Prove: There can be no more than
one obtuse angle in ABC.
B
Pages 257–258 Check for Understanding
1. If a statement is shown to be false, then its A C
opposite must be true. Proof:
2. Sample answer: Indirect proofs are proved using Step 1 Assume that there can be more than one
the contrapositive, showing Q → P. In a direct obtuse angle in ABC.
proof, it would be shown that P → Q. For example, Step 2 An obtuse angle has a measure greater
indirect reasoning can be used to prove that a than 90. Suppose A and B are obtuse
person is not guilty of a crime by assuming the angles. Then mA  mB  180 and
person is guilty, then contradicting evidence to show mA  mB  mC  180.
that the person could not have committed the crime. Step 3 The conclusion contradicts the fact that
the sum of the measures of the angles of a
triangle equals 180. Thus, there can be at
most one obtuse angle in ABC.

Chapter 5 132
10. Given: m
n 16. A median of an isosceles triangle is not an
Prove: Lines m and n intersect at exactly altitude.
one point. 17. Points P, Q, and R are noncollinear.
m 18. The angle bisector of the vertex angle of an
n isosceles triangle is not an altitude of the triangle.
Proof: 1
19. Given: a 0
Case 1: m and n intersect at more than one point.
Prove: a is negative.
Step 1 Assume that m and n intersect at more
Proof:
than one point.
Step 1 Assume a  0. a  0 since that would
Step 2 Lines m and n intersect at points P and Q. 1
make a undefined.
Both lines m and n contain P and Q.
Step 2 1
a¬ 0
Step 3 By postulate, there is exactly one line
through any two points. Thus the 1
 
a a ¬ (0)a
assumption is false, and lines m and n
1¬ 0
intersect in no more than one point.
Step 3 1  0, so the assumption must be false.
Case 2: m and n do not intersect. Thus, a must be negative.
Step 1 Assume that m and n do not intersect.
20. Given: n2 is even.
Step 2 If lines m and n do not intersect, then
Prove: n2 is divisible by 4.
they are parallel.
Proof:
Step 3 This conclusion contradicts the given Step 1 Assume n2 is not divisible by 4. In other
information. Therefore the assumption is words, 4 is not a factor of n2.
false, and lines m and n intersect in at Step 2 If the square of a numbers is even, then
least one point. Combining the two cases, the number is also even. So, if n2 is even,
lines m and n intersect in no more than then n must be even.
one point and no less than one point. So Let n ¬2a.
lines m and n intersect in exactly one n ¬2a
point. n2 ¬(2a)2 or 4a2
11. Given: ABC is a right triangle; C is a right Step 3 4 is a factor of n2, which contradicts the
angle. assumption.
Prove: AB  BC and AB  AC 21. Given: 
PQP R
A 1  2
Prove: PZ is not a median of PQR.
P
C B 12
Proof:
Step 1 Assume that the hypotenuse of a right Q Z R
triangle is not the longest side. That is,
Proof:
AB  BC or AB  AC.
Step 1 Assume P Z is a median of PQR.
Step 2 If AB  BC, then mC  mA. Since Z
Step 2 If P  is a median of PQR, then Z is the
mC  90, mA  90. So, mC  midpoint of Q R, and Q
ZR Z
. P
Z
P Z by
mA  180. By the same reasoning, if the Reflexive Property. PZQ  PZR by
AB  AC, then mC  mB  180. SSS. 1  2 by CPCTC.
Step 3 Both relationships contradict the fact that Step 3 This conclusion contradicts the given fact
the sum of the measures of the angles of a 1  2. Thus, P Z
 is not a median of
triangle equals 180. Therefore, the PQR.
hypotenuse must be the longest side of a 22. Given: m2  m1
right triangle. Prove: 
m
12. Given: x  y  270
t
Prove: x  135 or y  135 1
Proof: 3
Step 1 Assume x
135 and y
135. 2
m
Step 2 x  y
270
Step 3 This contradicts the fact that x  y  270.
Therefore, at least one of the stages was Proof:
longer than 135 miles. Step 1 Assume that 
m.
Step 2 If 
m, then 1  2 because they are
Pages 258–260 Practice and Apply corresponding angles. Thus, m1  m2.
13. 
PQ ST
 Step 3 This contradicts the given fact that
14. x
4 m1  m2. Thus the assumption

m is false. Therefore, 
m.
15. 6 cannot be expressed as a

b.

133 Chapter 5
23. Given: a  0, b  0, and a  b Case 2: If BC  AC, then there must be a point D
a
Prove:   1 between A and C so that D C
B C. Draw
b
Proof: the auxiliary segment B D
. Since DC 
Step 1 Assume that ab
1.
 BC, by the Isosceles Triangle Theorem
Step 2 BDC  DBC. Now BDC is an
Case 1 Case 2 exterior angle of BAD, and by the
a¬¬1 a¬ 1 Exterior Angles Inequality Theorem (the
b b
a¬¬b a¬ b measure of an exterior angle of a triangle
Step 3 The conclusion of both cases contradicts is greater than the measure of either
the given fact a  b. corresponding remote interior angle)
a mBDC  mA. By the Angle Addition
Thus,   1.
b Postulate, mABC  mABD 
24. Given: 
AB C
A
mDBC. Then by the definition of
Prove: 1
 2 inequality, mABC  mDBC. By
A Substitution and the Transitive Property
of Inequality, mABC  mA. But this
contradicts the given statement that
mA  mABC. In both cases, a
C 1 2
B contradiction was found, and hence our
Proof: assumption must have been false.
Step 1 Assume that 1  2. Therefore, BC  AC.
Step 2 If 1  2, then the sides opposite the
27. Use r  dt, t  3, and d  175.

angles are congruent.
Thus A B
A C
. Proof:
Step 3 The conclusion contradicts the given Step 1 Assume that Ramon’s average speed was
information. Thus 1  2 is false. greater than or equal to 60 miles per hour,
Therefore, 1
 2. r 60.
25. Given: ABC and ABD are equilateral. Step 2
ACD is not equilateral. Case 1 Case 2
Prove: BCD is not equilateral. r ¬60 r ¬60
17
5 17
5 ?
C 60 ¬ 
3 ¬60
3
A 60 ¬58.3 58.3 ¬60
Step 3 The conclusions are false, so the
B assumption must be false. Therefore,
D Ramon’s average speed was less than 60
Proof: miles per hour.
Step 1 Assume that BCD is an equilateral
28. A majority is greater than half or 50%.
triangle.
Proof:
Step 2 If BCD is an equilateral triangle, then
Step 1 Assume that the percent of college-bound
C
B C DD B. Since ABC and ABD
seniors receiving information from
C
are equilateral triangles, AA B
B C
guidance counselors is less than 50%.
and AD
A B
D B. By the Transitive
Step 2 By examining the graph, you can see that
C
Property, AA DC D
. Therefore,
56% of college-bound seniors received
ACD is an equilateral triangle.
information from guidance counselors.
Step 3 This conclusion contradicts the given
Step 3 Since 56%  50%, the assumption is false.
information. Thus, the assumption is false.
Therefore, a majority of college-bound
Therefore, BCD is not an equilateral
seniors received information from
triangle.
guidance counselors.
26. Given: mA  mABC
29. 1500  15% ¬225
Prove: BC  AC
1500  0.15 ¬225
D A 225 ¬225
30. teachers and friends; 15%  18%  33%, 33% 
C B 31%
Proof: Assume BC  AC. By the Comparison 31. Yes; if you assume the client was at the scene of
Property, BC  AC or BC  AC. the crime, it is contradicted by his presence in
Case 1: If BC  AC, then ABC  A by the Chicago at that time. Thus, the assumption that
Isosceles Triangle Theorem (If two sides he was present at the crime is false.
of a triangle are congruent, then the 32. See students’ work.
angles opposite those sides are
congruent.) But, ABC  A contradicts
the given statement that
mA  mABC. So, BC  AC.

Chapter 5 134
33. Proof: 40. Given: 
QT is a median. QRS is isosceles with
Step 1 Assume that 2  is a rational number. base RS
.
Step 2 If  2 is a rational number, it can be T
Prove: Q  bisects SQR
written as a 
b , where a and b are integers Q
with no common factors, and b  0. If
2
 2  a
 a
b , then 2  b2 , and 2b  a . Thus a
2 2 2

is an even number, as is a.Because a is


even it can be written as 2n. S R
T
2b2 ¬a2 Proof:
2b2 ¬(2n)2 Statements Reasons
2b2 ¬4n2
b2 ¬2n2 T
1. Q  is a median. QRS is 1. Given
b2 is an even number. So, b is also an even S
isosceles with base R.
number. T
2. RS
T
 2. Def. of median
Step 3 Because b and a are both even numbers,
they have a common factor of 2. This R
3. QQ
S
 3. Def. of isosceles

contradicts the definition of rational
numbers. Therefore,  2 is not rational. T
4. QQ
T
 4. Reflexive Prop.
34. Sample answer: Indirect proof is sometimes used 5. QRT  QST 5. SSS
in mystery novels. Answers should include the 6. SQT  RQT 6. CPCTC
following.
• Sherlock Holmes would disprove all possibilities T
7. Q bisects SQR 7. Def. of  bisector
except the actual solution to a mystery.
• medical diagnosis, trials, scientific research 41. Given: ABC  DEF; B G
 is an angle bisector of
35. D; x  80 ¬140 ABC. E
H
 is an angle bisector of DEF.
x ¬60 G
Prove: BEH
A, B, and C are true. D is not true.
8 7 6 1 A
36. A; 
16  15  14  10 G
C B
D
Page 260 Maintain Your Skills H
37. The angle in MOP with the greatest measure is F E
opposite the side with the greatest measure. Side Proof:
M
O  has measure 9, which is greater than all Statements Reasons
other sides of the triangle. So, P has the
greatest measure. 1. ABC  DEF 1. Given
38. The angle in LMN with the least measure is 2. A  D, A
B
D
E
, 2. CPCTC
opposite the side with the least measure. Side L M
 ABC  DEF
has measure 6, which is less than all other sides
of the triangle. So, N has the least measure. 3. 
BG is an angle bisector of 3. Given
ABC. E H is an angle
39. Given: C D is an angle bisector. C
D
 is an altitude.
bisector of DEF.
Prove: ABC is isosceles.
C 4. ABG  GBC, DEH  4. Def. of 
HEF bisector
5. mABC  mDEF 5. Def. of  
B A 6. mABG  mGBC, 6. Def. of  
D
mDEH  mHEF
Proof:
7. mABC  mABG  7. Angle
Statements Reasons
mGBC, mDEF  Addition
D
1. C is an angle bisector. 1. Given mDEH  mHEF Property
D
C  is an altitude. 8. mABC  mABG  8. Substitution
2. ACD  BCD 2. Def. of  bisector mABG, mDEF 
D
3. CA
B
 3. Def. of altitude mDEH  mDEH
4. CDA and CDB are 4.  lines form 4 rt. 9. mABG  mABG  9. Substitution
rt.  . mDEH  mDEH
5. CDA  CDB 5. All rt.  are . 10. 2mABG  2mDEH 10. Substitution

D
6. CC
D
 6. Reflexive Prop. 11. mABG  mDEH 11. Division

7. ACD  BCD 7. ASA 12. ABG  mDEH 12. Def. of  

C
8. AB
C
 8. CPCTC 13. ABG  DEH 13. ASA
9. ABC is isosceles. 9. Def. of isosceles  G
14. BE
H
 14. CPCTC

135 Chapter 5
42. mR  mS  mA ¬180 8. Let the measure of the third side be n.
41  109  mA ¬180 7  12¬ n 7  n¬ 12 12  n¬ 7
150  mA ¬180 19¬ n or n  19 n¬ 5 n¬ 5
mA ¬30 Graph the inequalities on the same number line.
43. y  y1 ¬m(x  x1)
y  3 ¬2(x  4) 5 3 1 0 1 3 5 7 9 11 13 15 17 19
44. y  y1 ¬m(x  x1) n¬ 19
y  (2) ¬3(x  2)
y  2 ¬3(x  2) 5 3 1 0 1 3 5 7 9 11 13 15 17 19
45. y  y1 ¬m(x  x1) n¬ 5
y  (9) ¬11[x  (4)]
y  9 ¬11(x  4) 5 3 1 0 1 3 5 7 9 11 13 15 17 19
?
46. true; 19  10¬ 11 n¬ 5
9¬ 11
? 5 3 1 0 1 3 5 7 9 11 13 15 17 19
47. false; 31  17¬ 12
Find the intersection.
14¬ 12
? The range of values that fit all three inequalities
48. true; 38  76¬ 109 is 5  n  19.
114¬ 109
9. Let the measure of the third side be n.
14  23¬ n 14  n¬ 23 23  n¬ 14
37¬ n or n  37 n¬ 9 n¬ 9
5-4 The Triangle Inequality Graph the inequalities on the same number line.

9 6 3 0 3 6 9 12 15 18 21 24 27 30 33 36 3739
Pages 263–264 Check for Understanding n¬ 37
1. Sample answer: If the lines are not horizontal,
then the segment connecting their y-intercepts is 9 6 3 0 3 6 9 12 15 18 21 24 27 30 33 36 39
not perpendicular to either line. Since distance is n¬ 9
measured along a perpendicular segment, this
segment cannot be used. 9 6 3 0 3 6 9 12 15 18 21 24 27 30 33 36 39
2. Jameson; 5  10  13 but 5  8 13. n¬ 9
3. Sample answer: 2, 3, 4 and 1, 2, 3
9 6 3 0 3 6 9 12 15 18 21 24 27 30 33 36 3739
3
2 Find the intersection.
The range of values that fit all three inequalities
4 2
1 is 9  n  37.
3 10. Let the measure of the third side be n.
4. Check each inequality. 22  34¬ n 22  n¬ 34 34  n¬ 22
?
5  4¬ 3
?
4  3¬ 5
?
5  3¬ 4 56¬ n or n  56 n¬ 12 n¬ 12
9¬ 3 ✓ 7¬ 5 ✓ 8¬ 4 ✓ Graph the inequalities on the same number line.
All of the inequalities are true, so 5, 4, and 3 can
12 6 0 6 12 18 24 30 36 42 48 54 56
be the lengths of the sides of a triangle.
? n¬ 56
5. 5  10¬ 15
15¬ 15 12 6 0 6 12 18 24 30 36 42 48 54
Because the sum of two measures equals the n¬ 12
third measure, the sides cannot form a triangle.
?
6. 30.1  0.8¬ 31 12 6 0 6 12 18 24 30 36 42 48 54

30.9¬ 31 n¬ 12


Because the sum of two measures is less than the
12 6 0 6 12 18 24 30 36 42 48 54 56
third measure, the sides cannot form a triangle.
7. Check each inequality. Find the intersection.
? ? The range of values that fit all three inequalities
5.6  10.1¬ 5.2 5.6  5.2¬ 10.1 is 12  n  56.
15.7¬ 5.2 ✓ 10.8¬ 10.1 ✓
? 11. Let the measure of the third side be n.
5.2  10.1¬ 5.6 15  18¬ n 15  n¬ 18 18  n¬ 15
15.3¬ 5.6 ✓ 33¬ n or n  33 n¬ 3 n¬ 3
All of the inequalities are true, so 5.6, 10.1, and
5.2 can be the lengths of the sides of a triangle.

Chapter 5 136
Graph the inequalities on the same number line. 18. Check each inequality.
? ? ?
18  32¬ 21 18  21¬ 32 32  21¬ 18
3 0 3 6 9 12 15 18 21 24 27 30 33
50¬ 21 ✓ 39¬ 32 ✓ 53¬ 18 ✓
n  33 All of the inequalities are true, so 18, 32, and 21
can be the lengths of the sides of a triangle.
3 0 3 6 9 12 15 18 21 24 27 30 33
19. Check each inequality.
n3 ? ? ?
9  21¬ 20 9  20¬ 21 20  21¬ 9
3 0 3 6 9 12 15 18 21 24 27 30 33 30¬ 20 ✓ 29¬ 21 ✓ 41¬ 9 ✓
n  3 All of the inequalities are true, so 9, 21, and 20
can be the lengths of the sides of a triangle.
3 0 3 6 9 12 15 18 21 24 27 30 33 ?
20. 5  9¬ 17
Find the intersection. 14¬ 17
The range of values that fit all three inequalities Because the sum of two measures is less than the
is 3  n  33. third measure, the sides cannot form a triangle.
12. Given: PQ  plane M 21. Check each inequality.
Prove: PQ is the shortest segment from P to ? ?
17  30¬ 30 30  30¬ 17
plane M.
47¬ 30 ✓ 60¬ 17 ✓
P All of the inequalities are true, so 17, 30, and 30
can be the lengths of the sides of a triangle.
M
Q 22. Check each inequality.
? ? ?
8.4  7.2¬ 3.5 8.4  3.5¬ 7.2 3.5  7.2¬ 8.4
Proof: By definition, PQ is perpendicular to 15.6¬ 3.5 ✓ 11.9¬ 7.2 ✓ 10.7¬ 8.4 ✓
plane M if it is perpendicular to every line in M All of the inequalities are true, so 8.4, 7.2, and 3.5
that intersects it. But since the perpendicular can be the lengths of the sides of a triangle.
segment from a point to a line is the shortest 23. Check each inequality.
segment from the point to the line, that ? ? ?
4  0.9¬ 4.1 4  4.1¬ 0.9 4.1  0.9¬ 4
perpendicular segment is the shortest segment 4.9¬ 4.1 ✓ 8.1¬ 0.9 ✓ 5¬ 4 ✓
from the point to each of these lines. Therefore, All of the inequalities are true, so 4, 0.9, and 4.1
Q
P  is the shortest segment from P to M. can be the lengths of the sides of a triangle.
13. B; Let x be the length of each of the congruent ?
sides of the triangle. 24. 2.2  12¬ 14.3
x  x¬ 10 x  10¬ x 14.2¬ 14.3
2x¬ 10 10¬ 0 true for all x Because the sum of two measures is less than the
x¬ 5 third measure, the sides cannot form a triangle.
The side length x is a whole number greater ?
25. 0.18  0.21¬ 0.52
than 5. The smallest number x for which this is 0.39¬ 0.52
true is 6. Thus, the answer is choice B. Because the sum of two measures is less than the
third measure, the sides cannot form a triangle.
26. Let the measure of the third side be n.
Pages 264–265 Practice and Apply 5  11¬ n 5  n¬ 11 11  n¬ 5
?
14. 1  2¬ 3 16¬ n or n  16 n¬ 6 n¬ 6
3¬ 3 Graph the inequalities on the same number line.
Because the sum of two measures equals the
6 4 2 0 2 4 6 8 10 12 14 16
third measure, the sides cannot form a triangle.
? n  16
15. 2  6¬ 11
8¬ 11 6 4 2 0 2 4 6 8 10 12 14 16
Because the sum of two measures is less than the n6
third measure, the sides cannot form a triangle.
16. Check each inequality. 6 4 2 0 2 4 6 8 10 12 14 16
? ?
8  8¬ 15 8  15¬ 8 n  6
16¬ 15 ✓ 23¬ 8 ✓
All of the inequalities are true, so 8, 8, and 15 can 6 4 2 0 2 4 6 8 10 12 14 16

be the lengths of the sides of a triangle. Find the intersection.


? The range of values that fit all three inequalities
17. 13  16¬ 29 is 6  n  16.
29¬ 29
Because the sum of two measures equals the
third measure, the sides cannot form a triangle.

137 Chapter 5
27. Let the measure of the third side be n.
7  9¬ n 7  n¬ 9 9  n¬ 7 30 20 10 0 10 20 26 30 40 50 60 70 80

16¬ n or n  16 n¬ 2 n¬ 2 n¬ 26


Graph the inequalities on the same number line.
30 26 20 10 0 10 20 30 40 50 60 70 80
4 2 0 2 4 6 8 10 12 14 16 18 n¬ 26
n¬ 16
30 20 10 0 10 20 26 30 40 50 60 68 70 80
4 2 0 2 4 6 8 10 12 14 16 18 Find the intersection.
n¬ 2 The range of values that fit all three inequalities
is 26  n  68.
4 2 0 2 4 6 8 10 12 14 16 18 31. Let the measure of the third side be n.
n¬ 2 32  61¬ n 32  n¬ 61 61  n¬ 32
93¬ n or n  93 n¬ 29 n¬ 29
4 2 0 2 4 6 8 10 12 14 16 18
Graph the inequalities on the same number line.
Find the intersection.
The range of values that fit all three inequalities 30 20 10 0 10 20 30 40 50 60 70 80 90 93
is 2  n  16. n¬ 93
28. Let the measure of the third side be n.
10  15¬ n 10  n¬ 15 15  n¬ 10 30 20 10 0 10 20 29 30 40 50 60 70 80 90

25¬ n or n  25 n¬ 5 n¬ 5 n¬ 29


Graph the inequalities on the same number line.
30 29 20 10 0 10 20 30 40 50 60 70 80 90
20 15 10 5 0 5 10 15 20 25 30 35 n¬ 29
n¬ 25
30 20 10 0 10 20 29 30 40 50 60 70 80 90 93

20 15 10 5 0 5 10 15 20 25 30 35 Find the intersection.


n¬ 5 The range of values that fit all three inequalities
is 29  n  93.
20 15 10 5 0 5 10 15 20 25 30 35
32. Let the measure of the third side be n.
n¬ 5 30  30¬ n 30  n¬ 30
60¬ n or n  60 n¬ 0
20 15 10 5 0 5 10 15 20 25 30 35
Graph the inequalities on the same number line.
Find the intersection.
The range of values that fit all three inequalities 20 10 0 10 20 30 40 50 60 70 80 90
is 5  n  25. n¬ 60
29. Let the measure of the third side be n.
12  18¬ n 12  n¬ 18 18  n¬ 12 20 10 0 10 20 30 40 50 60 70 80 90
30¬ n or n  30 n¬ 6 n¬ 6 n¬ 0
Graph the inequalities on the same number line.
20 10 0 10 20 30 40 50 60 70 80 90
18 12 6 0 6 12 18 24 30 36 42 48 Find the intersection.
n¬ 30 The range of values that fit all three inequalities
is 0  n  60.
18 12 6 0 6 12 18 24 30 36 42 48 33. Let the measure of the third side be n.
n¬ 6 64  88¬ n 64  n¬ 88 88  n¬ 64
152¬ n or n  152 n¬ 24 n¬ 24
18 12 6 0 6 12 18 24 30 36 42 48 Graph the inequalities on the same number line.
n¬6
60 40 20 0 20 40 60 80 100 120 140152 160
18 12 6 0 6 12 18 24 30 36 42 48
n¬ 152
Find the intersection.
The range of values that fit all three inequalities 60 40 20 0 2024 40 60 80 100 120 140 160
is 6  n  30. n¬ 24
30. Let the measure of the third side be n.
21  47¬ n 21  n¬ 47 47  n¬ 21 60 40 24 20 0 20 40 60 80 100 120 140 160
68¬ n or n  68 n¬ 26 n¬ 26 n¬ 24
Graph the inequalities on the same number line.
60 40 20 0 20 24 40 60 80 100 120 140 152 160
30 20 10 0 10 20 30 40 50 60 6870 80 Find the intersection.
n¬ 68 The range of values that fit all three inequalities
is 24  n  152.

Chapter 5 138
34. Let the measure of the third side be n. Graph the inequalities on the same number line.
57  55¬ n 57  n¬ 55 55  n¬ 57
112¬ n or n  112 n¬ 2 n¬ 2 100 80 60 40 20 0 20 40 60 80 100 101 120
Graph the inequalities on the same number line. n¬ 101

10 0 10 20 30 40 50 60 70 80 90 100 110 112 120 100 97 80 60 40 20 0 20 40 60 80 100 120


n¬ 112 n¬ 97

10 2 0 10 20 30 40 50 60 70 80 90 100 110 120 100 80 60 40 20 0 20 40 60 80 97 100 120


n¬ 2 n¬ 97

10 0 2 10 20 30 40 50 60 70 80 90 100 110 120 100 80 60 40 20 0 20 40 60 80 97100 101120

n¬ 2 Find the intersection.


The range of values that fit all three inequalities
10 0 2 10 20 30 40 50 60 70 80 90 100 110 112 120
is 97  n  101.
Find the intersection. 38. Given: B  ACB
The range of values that fit all three inequalities Prove: AD  AB  CD
is 2  n  112.
B C
35. Let the measure of the third side be n.
75  75¬ n 75  n¬ 75
150¬ n or n  150 n¬ 0
Graph the inequalities on the same number line. A D
Proof:
20 0 20 40 60 80 100 120 140150 160 180 200
Statements Reasons
n¬ 150
1. B  ACB 1. Given
20 0 20 40 60 80 100 120 140 160 180 200 B
2. AA
C
 2. If two  are , the sides
n¬ 0 opposite the two  are .

20 0 20 40 60 80 100 120 140 150 160 180 200


3. AB  AC 3. Def. of  segments
Find the intersection. 4. AD  AC  CD 4. Triangle Inequality
The range of values that fit all three inequalities
is 0  n  150. 5. AD  AB  CD 5. Substitution
36. Let the measure of the third side be n.
78  5¬ n 78  n¬ 5 5  n¬ 78 39. Given: 
HEEG

83¬ n or n  83 n¬ 73 n¬ 73 Prove: HE  FG  EF
Graph the inequalities on the same number line. H E

75 60 45 30 15 0 15 30 45 60 75 83 90
G
n¬ 83 F
Proof:
75 73 60 45 30 15 0 15 30 45 60 75 90
Statements Reasons
n¬ 73
E
1. HE
G
 1. Given
75 60 45 30 15 0 15 30 45 60 73 75 90 2. HE  EG 2. Def. of  segments
n¬ 73
3. EG  FG  EF 3. Triangle Inequality
75 60 45 30 15 0 15 30 45 60 73 75 83 90 4. HE  FG  EF 4. Substitution
Find the intersection.
The range of values that fit all three inequalities
is 73  n  83.
37. Let the measure of the third side be n.
99  2¬ n 99  n¬ 2 2  n¬ 99
101¬ n or n  101 n¬ 97 n¬ 97

139 Chapter 5
40. Given: ABC 42. LM ¬[22  (2 4)]2  [20  (19 )]2
Prove: AC  BC  AB ¬ 4  1521
A
¬ 1525
¬39.1
MN ¬ [5  (22)] 2  ( 2
7  20)
D C B ¬ 289  729
Proof: ¬ 1018
Statements Reasons ¬31.9
1. Construct CD so 1. Ruler Postulate LN ¬ [5  (24 )]2  [
7   (19) ]2
that C is between B ¬ 361  144
and D and C D
 .
AC ¬ 505
2. CD  AC 2. Definition of  ¬22.5
? ?
3. CAD  ADC 3. Isosceles Triangle LM  MN¬ LN LM  LN¬ MN
? ?
Theorem 39.1  31.9¬ 22.5 39.1  22.5¬ 31.9
4. mCAD  mADC 4. Definition of  71¬ 22.5 ✓ 61.6¬ 31.9 ✓
?
angles LN MN¬ LM
?
5. mBAC  5. Angle Addition 22.5  31.9¬ 39.1
mCAD  mBAD Postulate 54.4¬ 39.1 ✓
All of the inequalities are true, so the coordinates
6. mBAC  6. Substitution can be the vertices of a triangle.
mADC  mBAD
43. XY ¬ (16  0)2 [12  (8 )]2
7. mADC  mBAD 7. Definition of ¬ 256  16
inequality ¬ 272
8. AB  BD 8. If an angle of a ¬16.49
triangle is greater YZ ¬ (28  16)2  [15 2
 (12)]
than a second angle,
¬ 144  9
then the side
opposite the greater ¬ 153
angle is longer than ¬12.37
the side opposite XZ ¬ (28  0)2 [15  (8 )]2
the lesser angle. ¬ 784  49
9. BD  BC  CD 9. Segment Addition ¬ 833
Postulate
¬28.86
10. AB  BC  CD 10. Substitution
XY  YZ¬ XZ
?

11. AB  BC  AC 11. Substitution ?


16.49  12.37¬ 28.86
(Steps 2, 10)
28.86¬ 28.86
41. AB ¬ (2  5
)2  (4  8)2 Because the sum of two measures equals the
¬ 9  144 third measure, the sides cannot form a triangle
and so the coordinates cannot be the vertices of
¬ 153
a triangle.
¬12.4
44. RS ¬ (3  1)2  [20  (4)]2
BC ¬ (3  2 )2 [1  (4)]2
¬ 16  2 56
¬ 25  9
¬ 272
¬ 34
¬16.5
¬5.8
RT ¬ (5  1)2  [1 2
2  (4)]
AC ¬ (3  5 )2 (1  8)2
¬ 16  2 56
¬ 64  81
¬ 272
¬ 145
¬16.5
¬12.0
? ? ST ¬ 
[5  (3)] 2
 [12  (20)]2
AB  BC¬ AC AB  AC¬ BC
? ? ¬ 64  1 024
12.4  5.8¬ 12.0 12.4  12.0¬ 5.8
¬ 1088
18.2¬ 12.0 ✓ 24.4¬ 5.8 ✓
? ¬33
AC  BC¬ AB
?
12.0  5.8¬ 12.4
17.8¬ 12.4 ✓
All of the inequalities are true, so the coordinates
can be the vertices of a triangle.

Chapter 5 140
?
RS  RT¬ ST 8; 2, 3, 7; 2, 4, 6; and 2, 5, 5. Check each of these
?
16.5  16.5¬ 33 triples using the Triangle Inequality.
219
33¬ 33 228
Because the sum of two measures equals the 237
third measure, the sides cannot form a triangle 246
and so the coordinates cannot be the vertices of 2  5  5 and 5  5  2, so there could be a
a triangle. triangle with side lengths 2, 5, and 5 units.
45. Consider all possible triples using the lengths Determine whether any triangle can have a side
3, 4, 5, 6, and 12. with 3 segments. If one side has length 3, then
? ? ?
3  4¬ 5 4  5¬ 3 3  5¬ 4 the possible triples of side lengths are 3, 1, 8; 3, 2,
7¬ 5 ✓ 9¬ 3 ✓ 8¬ 4 ✓ 7; 3, 3, 6; and 3, 4, 5. Check each of these triples
? ? ? using the Triangle Inequality.
3  4¬ 6 4  6¬ 3 3  6¬ 4
7¬ 6 ✓ 10¬ 3 ✓ 9¬ 4 ✓ 318
3  4¬ 12 327
7¬ 12 336
? ? ? 3  4  5 and 3  5  4 and 4  5  3, so there
4  5¬ 6 4  6¬
5 5  6¬ 4
could be a triangle with side lengths 3, 4, and
9¬ 6 ✓ 10¬ 5 ✓ 11¬ 4 ✓
? 5 units.
4  5¬ 12 Determine whether any triangle can have a side
9¬ 12 with 4 segments. If one side has length 4, then
? ? ?
3  5¬ 6 5  6¬ 3 3  6¬ 5 the possible triples of side lengths are 4, 1, 7; 4, 2,
8¬ 6 ✓ 11¬ 3 ✓ 9¬ 5 ✓ 6; 4, 3, 5; and 4, 4, 4. Check each of these triples
?
3  5¬ 12 using the Triangle Inequality. Note that we have
8¬ 12 already shown that there can be a triangle with
? sides 4, 3, and 5 units.
3  6¬ 12
9¬ 12 417
? 426
4  6¬ 12
4  4  4, so there could be a triangle with side
10¬ 12
? lengths 4, 4, and 4 units.
5  6¬ 12 By examining all of the triples we have
11¬ 12 considered to this point, we can see that all
Of all possible triples, 4 of them satisfy the possible triples have been listed and checked.
triangle inequality, so there are 4 possible Therefore, there are 3 triangles that can be
triangles. formed using the rope shown in the figure.
46. 3  4  5  12, which is divisible by 3 48. 14¬ m  17, so m is either 15 or 16 feet.
3  4  6  13, which is not divisible by 3 13¬ n  17, so n is 14, 15, or 16 feet.
4  5  6  15, which is divisible by 3 Check all possible triples using the triangle
3  5  6  14, which is not divisible by 3 inequality.
Carlota could make 2 different triangles with a ? ? ?
2  15¬ 14 2  14¬ 15 14  15¬ 2
perimeter that is divisible by 3.
17¬ 14 ✓ 16¬ 15 ✓ 29¬ 2 ✓
47. The rope has 13 knots that determine 12 ? ?
2  15¬ 15 15  15¬ 2
segments of the rope. We need to determine how
17¬ 15 ✓ 30¬ 2 ✓
many triangles there are whose perimeter is 12. ? ? ?
First determine whether any triangle can have a 2  15¬ 16 2  16¬ 15 15  16¬ 2
side with 1 segment. If one side has length 1, then 17¬ 16 ✓ 18¬ 15 ✓ 31¬ 2 ✓
?
the possible triples of side lengths are 1, 1, 10; 1, 2  16¬ 14 2  14¬ 16
2, 9; 1, 3, 8; 1, 4, 7; and 1, 5, 6. Check each of these 18¬ 14 ✓ 16¬ 16
? ? ?
triples using the Triangle Inequality. 2  16¬ 15 2  15¬ 16 15  16¬ 2
1  1  10 18¬ 15 ✓ 17¬ 16 ✓ 31¬ 2 ✓
129 ?
2  16¬ 16
?
16  16¬ 2
138 18¬ 16 ✓ 32¬ 2 ✓
147 The possible triangles that can be made from
156 sides with measures 2 ft, m ft, and n ft are (2 ft,
Therefore, there is no possible triangle with one 15 ft, 14 ft), (2 ft, 15 ft, 15 ft), (2 ft, 15 ft, 16 ft),
side of length 1 and perimeter 12. and (2 ft, 16 ft, 16 ft).
Determine whether any triangle can have a side
49. Of the 4 possible triangles listed in Exercise 48, 2
with 2 segments. If one side has length 2, then 2 1
the possible triples of side lengths are 2, 1, 9; 2, 2, are isosceles, so the probability is 4 or 2.

141 Chapter 5
50. Sample answer: The length of any side of a Page 266 Maintain Your Skills
triangle is greater than the differences between 54. Given: P is a point not on line .
the lengths of the other two sides.
Prove: 
PQ is the only line through P
Paragraph Proof:
perpendicular to .
By the Triangle Inequality Theorem, for ABC
with side measures a, b, and c, a  b  c, b 
c  a, and c  a  b. Using the Subtraction 1 Q
Property of Inequality, a  c  b, b  a  c, and P
c  b  a. 2 R
51. Sample answer: You can use the Triangle
Inequality Theorem to verify the shortest route Proof:
between two locations. Answers should include Statements Reasons
the following.  is not the only line 1. Assumption
• A longer route might be better if you want to 1. PQ
collect frequent flier miles. through P
perpendicular to .
• A straight route might not always be available.
2. 1 and 2 are right 2.  lines form 4 rt. .
52. D; If the perimeter is 29, the measure of the third angles.
side is 10.
?
7  10¬ 12
?
7  12¬ 10
?
10  12¬ 7 3. m1  90, m2  90 3. Def. of rt. 
17¬ 12 ✓ 19¬ 10 ✓ 22¬ 7 ✓ 4. m1  m2  4. The sum of  in a 
So, 7, 12, and 10 could be the sides of a triangle mQPR  180 is 180.
with perimeter 29. 5. 90  90  mQPR  5. Substitution
If the perimeter is 34, the measure of the third 180
side is 15. 6. mQPR  0 6. Subtraction Property
? ? ?
7  15¬ 12 7  12¬ 15 12  15¬ 7
22¬ 12 ✓ 19¬ 15 ✓ 27¬ 7 ✓ This contradicts the fact that the measure of an
So, 7, 12, and 15 could be the sides of a triangle  is the only line
angle is greater than 0. Thus, PQ
with perimeter 34. through P perpendicular to .
If the perimeter is 37, the measure of the third 55. mP  mQ  mR¬ 180
side is 18. 7x  8  8x  10  7x  6¬ 180
? ? ?
7  18¬ 12 7  12¬ 18 12  18¬ 7 22x  4¬ 180
25¬ 12 ✓ 19¬ 18 ✓ 30¬ 7 ✓ 22x¬ 176
So, 7, 12, and 18 could be the sides of a triangle x¬ 8
with perimeter 37. mP¬ 7(8)  8 or 64
If the perimeter is 38, the measure of the third mQ¬ 8(8)  10 or 54
side is 19. mR¬ 7(8)  6 or 62
?
7  12¬ 19 mP  mR  mQ, so the sides of PQR in
19¬ 19 order from longest to shortest are Q R, P
Q, P
R
.
So, 7, 12, and 19 cannot be the sides of a triangle. 56. mP  mQ  mR¬ 180
53. A; If the graphs of the equations do intersect, 3x  44  68  3x  x  61¬ 180
then we can solve the system of equations and x  173¬ 180
find the coordinates of any points of intersection. x¬ 7
Substitute x for y in the equation (x  5)2  mP  3(7)  44 or 65
( y  5)2  4 and solve for x. mQ  68  3(7) or 47
(x  5)2  [(x)  5]2¬ 4 mR  7  61 or 68
x  10x  25  x2  10x  25¬ 4
2 mR  mP  mQ, so the sides of PQR in
2x2  50¬ 4 order from longest to shortest are P Q
, QR, P
R
.
2x2¬ 46 57. JK ¬ (0  0)2  (0 5)2
x2¬ 23 ¬ 0  25 or¬5
Because there is no real number x whose square PQ ¬ (4  4
 )2  (3 8)2
is equal to 23, there are no points of
¬ 0  25¬or¬5
intersection.
KL ¬ (2  0)2 (0  0)2
¬ 4  0¬or¬2
QR ¬ (6  4)2  (3 3)2
¬ 4  0¬or¬2
JL ¬ (2  0)2 (0  5)2
¬ 4  25¬or  29
PR ¬ (6  4)2  (3 8)2
¬ 4  25¬or  29
The corresponding sides have the same measure
and are congruent. JKL  PQR by SSS.

Chapter 5 142
58. JK ¬ (1  6)2  (6  4)2 4. Given: 
MO¬ O
N
¬ 25  1 00 or 125 P
M ¬ N
P

Prove: MOP NOP
PQ   (5  0)2  (3  7)2
¬ 25  1 00 or 125 O
KL ¬ (9  1)2 [5  (
6)]2 M
¬ 100  121 or  221
P
QR ¬ (15  5)2  2
[8  (3)] N
¬ 100  121 or  221
Proof:
JL ¬ (9  6)2 (5  4)2
Step 1 Assume that MOP  NOP.
¬ 225  1 or 226
Step 2 We know that M OO N, and O
P
O P by
PR ¬ (15  0)2 (8  7)2 the Reflexive Property. If MOP  NOP,
¬ 225  1 or 226 then MOP  NOP by SAS. Then, M P
The corresponding sides have the same measure P
N  by CPCTC.
and are congruent. JKL  PQR by SSS. Step 3 The conclusion that MPN P
 contradicts
59. JK ¬ [1  (
6)]2  [5  (3)]2 the given information. Thus, the
¬ 49  6 4 or 113 assumption is false. Therefore, MOP
NOP.
PQ ¬ (5  2)2  [4  ( 11)]2
5. Given: mADC  mADB
¬ 9  49 or  58
D
Prove: A  is not an altitude of ABC.
KL ¬ (2  1)2  (2  5)2
A
¬ 1  49 or  50
QR ¬ (10  5)2 [10  (4)] 2
¬ 25  3 6 or 61
JL ¬ [2  (
6)]2  [2  (3 )]2 C D B
¬ 64  1 or  65 Proof:
PR ¬ (10  2)2 [10  (1 1)]2
Statements Reasons
¬ 64  1 or  65
1. 
AD is an altitude of 1. Assumption
The corresponding sides are not congruent, so the
ABC.
triangles are not congruent.
2. ADC and ADB are 2. Def. of altitude
60. 3x  54¬ 90
right angles.
3x¬ 36
x¬ 12 3. ADC  ADB 3. All rt  are .
61. 8x  14¬ 3x  19 4. mADC  mADB 4. Def. of  angles
5x  14¬ 19
5x¬ 33 This contradicts the given information that
x¬ 6.6 mADC  mADB. Thus, A D
 is not an altitude
62. 4x  7¬ 180 of ABC.
4x¬ 173 6. Check each inequality.
? ? ?
x¬ 43.25 7  24¬ 25 7  25¬ 24 24  25¬ 7
31¬ 25 ✓ 32¬ 24 ✓ 49¬ 7 ✓
All of the inequalities are true, so 7, 24, and 25
Page 266 Practice Quiz 2 can be the lengths of the sides of a triangle.
?
1. The number 117 is not divisible by 13. 7. 25  35¬ 60
2. mC mD 60¬ 60
Because the sum of two measures equals the
3. Step 1 Assume that x
8.
third measure, the sides cannot form a triangle.
Step 2 7x¬ 56
x¬ 8 8. Check each inequality.
? ? ?
Step 3 The solution of 7x  56 contradicts the 3  18¬ 20 18  20¬ 3 3  20¬ 18
assumption. Thus, x
8 must be false. Therefore, 21¬ 20 ✓ 38¬ 3 ✓ 23¬ 18 ✓
x  8. All of the inequalities are true, so 20, 3, and 18
can be the lengths of the sides of a triangle.
9. Check each inequality.
? ? ?
5  10¬ 6 5  6¬ 10 6  10¬ 5
15¬ 6 ✓ 11¬ 10 ✓ 16¬ 5 ✓
All of the inequalities are true, so 5, 10, and 6 can
be the lengths of the sides of a triangle.

143 Chapter 5
10. Let the measure of the third side be n. Also, the measure of any angle is always greater
57  32¬ n 57  n¬ 32 32  n¬ 57 than 0.
89¬ n or n  89 n¬ 25 n¬ 25 7x  4¬ 0
Graph the inequalities on the same number line. 7x¬ 4
x¬ 4

7
30 20 10 0 10 20 30 40 50 60 70 80 90
The two inequalities can be written as the
n¬ 89
compound inequality 47 x 7 .
 13
6

30 25 20 10 0 10 20 30 40 50 60 70 80 90 7. Given: P Q
S Q

n¬ 25 Prove: PR  SR
P S
30 20 10 0 10 20 25 30 40 50 60 70 80 90

n¬ 25 T
30 20 10 0 10 20 25 30 40 50 60 70 80 89 90

Find the intersection. Q R


The range of values that fit all three inequalities Proof:
is 25  n  89.
Statements Reasons
Q
1. PS
Q
 1. Given

5-5 R
2. QQ
R
 2. Reflexive Property
Inequalities Involving Two
3. mPQR  mPQS  3. Angle Addition
Triangles mSQR Postulate
4. mPQR  mSQR 4. Def. of inequality
Pages 270–271 Check for Understanding 5. PR  SR 5. SAS Inequality
1. Sample answer: A pair of scissors illustrates the
SSS inequality. As the distance between the tips 8. Given: 
TUUS
; U
S
S
V
of the scissors decreases, the angle between the Prove: ST  UV
blades decreases, allowing the blades to cut. S
2. The SSS Inequality Theorem compares the angle
between two sides of a triangle for which the two
sides are congruent and the third side is different.
The SSS Postulate states that two triangles that T U V
have three sides congruent are congruent.
3. In BDC and BDA, B CA D
, B
DB D
, and Proof:
mBDA  mCBD. The SAS Inequality Theorem Statements Reasons
allows us to conclude that AB  CD. U
1. TU
S
, U
S
S
V
 1. Given
4. In PQS and RQS, R QP Q
, Q
S
Q S
, and 2. mSUT  mUSV 2. Ext.  Inequality
PS  RS. The SSS Inequality Theorem allows us Theorem
to conclude that mPQS  mRQS.
3. ST  UV 3. SAS Inequality
5. The upper triangle is equilateral and so all angles
are 60 degrees. The SAS inequality allows us to 9. Sample answer: The pliers are an example of the
conclude that x  5  3x  7. SAS inequality. As force is applied to the handles,
x  5¬ 3x  7 the angle between them decreases causing the
5¬ 2x  7 distance between them to decrease. As the
12¬ 2x distance between the ends of the pliers decreases,
6¬ x more force is applied to a smaller area.
Also, the measure of any side is greater than 0.
3x  7¬ 0
3x¬ 7 Pages 271–273 Practice and Apply
x¬ 7

3 10. From the figure, AB  9 and FD  6, so AB  FD.
The two inequalities can be written as the 11. In BDC and FDB, F D
D C, B
DBD, and
compound inequality 7 3  x  6.
 BC  BF. The SSS Inequality allows us to
6. Because 12  8, the SSS Inequality allows us to conclude that mBDC  mFDB.
conclude 140  7x  4. 12. In BFA and DBF, A BBD, B
FBF, and
140¬ 7x  4 AF  FD. The SSS Inequality allows us to
136¬ 7x conclude that mFBA  mDBF.
13
 6
7 ¬ x 13. In ABD and CBD, B C
B A, B
D
B D, and
mABD  mCBD. The SAS Inequality allows
us to conclude that AD  DC.

Chapter 5 144
14. In ABO and CBO, A BC B
, O
BO B, and 20. Given: ABC, A
B
C
D

mCBO  mABO. The SAS Inequality allows Prove: BC  AD
us to conclude that OC  OA. B
15. In ABC, A BC B so ABC is isosceles with 2
base angles of measure 1 2 [180  (40  60)] or 40.

mAOB ¬180  (mOAB  mABO)
1
¬180  (40  60) A C
D
¬80
mAOD ¬180  mAOB Proof:
¬100 Statements Reasons
Therefore, mAOD  mAOB. 1. ABC, A
B
C
D
 1. Given
16. By the SAS Inequality, 10  3x  2. D
2. BB
D
 2. Reflexive Property
10¬ 3x  2
12¬ 3x 3. m1  m2 3. If an  is an ext.  of a
4¬ x or x  4 , then its measure is
greater than the
17. The triangle on the left is equilateral, so all
measure of either remote
angles have measure 60. Then by the SAS
int. .
inequality, x  2  2x  8.
x  2¬ 2x  8 4. BC  AD 4. SAS Inequality
2¬ x  8
10¬ x Q
21. Given: P R
S, QR  PS
The measure of any side is always greater than 0. Prove: m3  m1
2x  8¬ 0 Q R
2x¬ 8 1 2
x¬ 4
The two inequalities can be written as the 3
4
compound inequality 4  x  10. P S
C
18. M M C , so by the SSS Inequality m1  m2. Proof:
5x  20¬ 8x  100
Statements Reasons
20¬ 3x  100
120¬ 3x Q
1. PR
S
 1. Given
40¬ x 2. 
QSQS
 2. Reflexive Property
The measure of any angle is always greater 3. QR  PS 3. Given
than 0.
4. m3  m1 4. SSS Inequality
8x  100¬ 0
8x¬ 100
x¬ 12.5
The two inequalities can be written as the
compound inequality 12.5  x  40.
19. RTV  TRV, so RVT is isosceles and
V
R T V
. S
VS V and RS  ST, so by the SSS
Inequality, mRVS  mSVT.
15  5x¬ 10x  20
15¬ 5x  20
35¬ 5x
7¬ x
The measure of SVT is less than 180.
10x  20¬ 180
10x¬ 200
x¬ 20
The two inequalities can be written as the
compound inequality 7  x  20.

145 Chapter 5
R
22. Given: P P
Q
, SQ  SR 7. AE  AF 7. Def. of  segments
Prove: m1  m2
8. AE  EC  AE  FB 8. Add. Prop. of
P
Inequality
9. AE  EC  AF  FB 9. Substitution
S
1 2 10. AE  EC  AC, AF 10. Segment Add. Post.
R 4 3
Q  FB  AB
11. AC  AB 11. Substitution
Proof:
Statements Reasons
S
24. Given: R ¬ U
W

R
11. PPQ
 11. Given
T
S¬ WV
12. PRQ  PQR 12. If two sides of  RT¬ UV
are , the angles Prove: mS  mW
opposite the
sides are . R U
13. mPRQ  m1  13. Angle Add. Post.
m4, mPQR 
m2  m3 S T W V
14. mPRQ  mPQR 14. Def. of  angles
15. m1  m4  m2  15. Substitution Indirect Proof:
m3 Step 1 Assume mS
mW.
16. SQ  SR 16. Given Step 2 If mS
mW, then either mS  mW
or mS  mW.
17. m4  m3 17. If one side of a 
is longer than Case 1: If mS  mW, then RT  UV by the
another side, SAS Inequality.
then the  Case 2: If mS  mW, then RST  UWV by
opposite the SAS, and R T
U V
 by CPCTC. Thus
longer side is RT  UV.
greater than the Step 3 Both cases contradict the given RT  UV.
 opposite the Therefore, the assumption must be false,
shorter side. and the conclusion, mS  mW, must be
true.
18. m4  m3  x 18. Def. of inequality
25. As the door is opened wider, the angle formed
19. m1  m3  x 19. Substitution increases and the distance from the end of the
 m2  m3 door to the door frame increases.
10. m1  x  m2 10. Subtraction Prop. 26. By the SAS Inequality Theorem, if the tree
11. m1  m2 11. Def. of inequality started to lean, one of the angles of the triangle
formed by the tree, the ground, and the stake
23. Given: 
EDD F; m1  m2; D is the midpoint would change, and the side opposite that angle
B
of C; A
E
A F
. would change as well. However, with the stake in
Prove: AC  AB the ground and fixed to the tree, none of the sides
of the triangle can change length. Thus, none of
A the angles can change. This ensures that the tree
E will stay straight.
F
27. As the vertex angle increases, the base angles
2 1 decrease. Thus, as the base angles decrease, the
B C altitude of the triangle decreases.
D
0.78s 1.67
Proof: 28. v ¬ 
1.17
h
Statements Reasons 0.78(1.0)1.67
¬
0.851.17
1. D
ED F; D is the 1. Given
¬0.94 m/s
B
midpoint of D.
0.78s 1.67
2. CD  BD 2. Def. of midpoint v ¬ 
1.17
h
0.78(1.2) 1.67
D
3. CB
D
 3. Def. of  segments ¬ 
1.17
0.85
4. m1  m2 4. Given ¬1.28 m/s

5. EC  FB 5. SAS Inequality
E
6. AA
F
 6. Given

Chapter 5 146
29.
0.78s 1.67 D
39. Given: A bisects B
E
; A
B

D
E
.
Stride (m) v 
1.17 Velocity (m/s) Prove: ABC  DEC
h
0.78(0.25)1.67 B
0.25  0.07 D
1.11.17
0.78(0.50)1.67 C
0.50  0.22
1.11.17 A
E
0.78(0.75)1.67
0.75  0.43
1.11.17 Proof:
0.78(1.00)1.67
Statements Reasons
1.00  0.70
1.11.17 1. 
AD bisects B
E
; 1. Given
0.78(1.25)1.67 B
A 
DE.
1.25  1.01
1.11.17 C
2. BE
C
 2. Def. of seg. bisector
0.78(1.50)1.67
1.50  1.37 3. B  E 3. Alt. int.  Thm.
1.11.17
4. BCA  ECD 4. Vert.  are .
30. As the length of the stride increases, the angle
5. ABC  DEC 5. ASA
formed at the hip increases.
31. Sample answer: A backhoe digs when the angle M
40. Given: O bisects LMN; L
MM
N
.
between the two arms decreases and the shovel Prove: MOL  MON
moves through the dirt. Answers should include
the following. N
• As the operator digs, the angle between the O
arms decreases.
• The distance between the ends of the arms M L
increases as the angle between the arms
increases, and decreases as the angle decreases. Proof:
32. B; AD  BD by definition of a median. Statements Reasons
AC  BC since m1  m2 and by the SAS 1. 
OM bisects LMN; 1. Given
Inequality. M
LM N.
m1  mB because if an  is an
2. LMO  NMO 2. Def. of  bisector
ext.  of a , then its measure is greater than the
measure of either remote int. . M
3. OO
M
 3. Reflexive Prop.
mADC  m1 and mBDC  m2, and m1 
4. MOL  MON 4. SAS
m2 is given, so mADC  mBDC.
The correct answer is B.
41. EF ¬
(4  4
)2  (1
1  6)2
1(99.50  88.95  95.90  102.45)
33. B;
2

4  48.35 ¬
0  25 or 5
FG ¬
(9  4)
2  (6
 11)2
¬  or 50
25  25 
Page 273 Maintain Your Skills EG ¬
(9  4)
2  (6
 6)2
34. no; 1  21  25
? ? ? ¬25  0 or 5
35. yes; 16  6¬ 19 6  19¬ 16 16  19¬ 6 EFG is isosceles.
22¬ 19 ✓ 25¬ 16 ✓ 35¬ 6 ✓
42. EF ¬
[15 
(7)]2
 (0
 10)2
36. no; 8  7  15
D
37. A is a not a median of ABC. ¬
484 
100 or 584

38. The triangle is not isosceles. FG ¬
(2 
15)2 
(1 
0)2
¬
289 
1 or 290

EG ¬
[2 
(7)]2
 (
1  10
)2
¬  or 146
25  121 
EFG is scalene.
43. EF ¬
(7  1
6)2  
(6  1
4)2
¬  or 145
81  64 
FG ¬
(5 
7)2 
(14
 6)2
¬
144 
400 or 544

EG ¬
(5 
16)2 
(14
 14)2
¬441  784 or 35
EFG is scalene.

147 Chapter 5
44. EF ¬
(12 
9)2 
(14 
9)2 15. mQDR  mRDS ¬180
mQDR  110 ¬180
¬
9  25 or 34

mQDR ¬70
FG ¬
(14 
12)2  2
(6  14) mQRD  mQDR  mRQD ¬180
¬
4  64 or 68
 mQRD  70  73 ¬180
EG ¬
(14 
9)2 
(6  9)2 mQRD ¬37
The angle opposite D Q
 has a smaller measure
¬25  9 or 34 than the angle opposite D R, so DQ  DR.
EFG is isosceles.
16. mQRP ¬37 (see exercise 15)
45. Let p, q be the parts of the statement.
mQPR ¬27
p: it has to be special
In PQR, the angle opposite P Q
 has a greater
q: it has to be Wildflowers
measure than the angle opposite Q R
, so PQ  QR.
The statement p is true, so it follows that
Catalina should go to Wildflowers by the Law of 17. mSRQ ¬mSRD  mDRQ
Detachment. ¬34  37
¬71
The angle opposite S R
 has a greater measure
than the angle opposite S Q
, so SR  SQ.
Chapter 5 Study Guide and Review 18. 2 is a rational number.
19. The triangles are not congruent.
20. Assume that Miguel completed at most 20 passes
Page 274 Vocabulary and Concept Check in each of the five games in which he played. If
1. incenter we let p1, p2, p3, p4, and p5 be the number of
2. median passes Miguel completed in games 1, 2, 3, 4,
3. Triangle Inequality Theorem and 5, respectively, then
4. centroid p1  p2  p3  p4  p5 ¬the total number of
5. angle bisector passes Miguel completed
¬101.
6. perpendicular bisectors
Because we have assumed that he completed at
7. orthocenter most 20 passes in each of the five games,
p1
20 and p2
20 and p3
20 and p4
20 and
p5
20.
Pages 274–276 Lesson-by-Lesson Review Then, by a property of inequalities,
8. mACQ ¬mQCB p1  p2  p3  p4  p5
20  20  20  20  20
mQCB ¬1
 or 100 passes.
2 mACB But this says that Miguel completed at most
42  x ¬1
2 (123  x)
 100 passes this season, which contradicts the
42  x ¬61.5  2x information we were given, that he completed
101 passes. So our assumption must be false.
42  3
2 x ¬61.5

Thus, Miguel completed more than 20 passes in at
3x ¬19.5 least one game this season.
2
?
x ¬13 21. no; 7  5 ¬20
mACQ  42  13 or 55 12 ¬20
9. AR  RB Because the sum of two measures is less than the
3x  6 ¬5x  14 third measure, the sides cannot form a triangle.
6 ¬2x  14 22. Check each inequality.
? ? ?
20 ¬2x 16  20¬ 5 16  5¬ 20 20  5¬ 16
10 ¬x 36¬ 5 ✓ 21¬ 20 ✓ 25¬ 16 ✓
AB ¬AR  AB All of the inequalities are true, so 16, 20, and 5
¬3(10)  6  5(10)  14 can be the lengths of the sides of a triangle.
¬72 23. Check each inequality.
? ? ?
10. mAPC ¬90 18  20¬ 6 18  6¬ 20 20  6¬ 18
72  x ¬90 38¬ 6 ✓ 24¬ 20 ✓ 26¬ 18 ✓
x ¬18 All of the inequalities are true, so 18, 20, and 6
11. The side opposite DEF is longer than the side can be the lengths of the sides of a triangle.
opposite DFE, so mDEF  mDFE. 24. In BAM and DAM, A B
A D
, AM
A M, and
12. The side opposite GDF is shorter than the side BM  DM. The SSS Inequality allows us to
opposite DGF, so mGDF  mDGF. conclude that mBAC  mDAC.
13. The side opposite DEF is longer than the side 25. In BMC and DCM, B M
C D, MCM C , and
opposite FDE, so mDEF  mFDE. mBMC  mDCM. The SAS Inequality allows
14. R
The angle opposite S has a greater measure us to conclude that BC  MD.
D
than the angle opposite S, so SR  SD.

Chapter 5 148
26. Using the SSS Inequality, 54  28 so 41  x  20 But this says that Marcus spent less than one
or 21  x. x  20  0, so x  20. The two and one-half hours on a teleconference over the
inequalities can be written as the compound three days, which contradicts the information we
inequality 20  x  21. were given. So we must abandon our assumption.
27. In the upper triangle, the bottom angle has Thus, Marcus spent at least one half-hour on a
measure 90  60 or 30, so the upper left angle has teleconference, on at least one of the three days.
measure 180  (95  30) or 55. Then by the SAS 13. Let the measure of the third side be n.
Inequality, 5x  3  3x  17. 1  14¬ n 1  n¬ 14 14  n¬ 1
5x  3¬ 3x  17 15¬ n or n  15 n¬ 13 n¬ 13
2x  3¬ 17 Graph the inequalities on the same number line.
2x¬ 14
x¬ 7 15 12 9 6 3 0 3 6 9 12 15 18
n¬ 15

15 12 9 6 3 0 3 6 9 12 13 15 18

Chapter 5 Practice Test n¬ 13

15 13 12 9 6 3 0 3 6 9 12 15 18
Page 277 n¬ 13
1. b
15 12 9 6 3 0 3 6 9 12 13 15 18
2. c
3. a Find the intersection.
The range of values that fit all three inequalities
4. HP  PJ
is 13  n  15.
5x  16 ¬3x  8
2x  16 ¬8 14. Let the measure of the third side be n.
2x ¬24 14  11¬ n 14  n¬ 11 11  n¬ 14
x ¬12 25¬ n or n  25 n¬ 3 n¬ 3
HJ ¬HP  PJ Graph the inequalities on the same number line.
¬5(12)  16  3(12)  8
3 1 0 1 3 5 7 9 11 13 15 17 19 21 23 25
¬88
n¬ 25
5. mGJN ¬mNJH
6y  3 ¬4y  23 3 1 0 1 3 5 7 9 11 13 15 17 19 21 23 25
2y  3 ¬23 n¬ 3
2y ¬26
y ¬13 3 1 0 1 3 5 7 9 11 13 15 17 19 21 23 25

mGJH ¬mGJN  mNJH n¬ 3


¬6(13)  3  4(13)  23
3 1 0 1 3 5 7 9 11 13 15 17 19 21 23 25
¬150
Find the intersection.
6. mHMG ¬90
The range of values that fit all three inequalities
4z  14 ¬90
is 3  n  25.
4z ¬76
z ¬19 15. Let the measure of the third side be n.
13  19¬ n 13  n¬ 19 19  n¬ 13
7. By the Exterior Angle Theorem, m8  m7 and
32¬ n or n  32 n¬ 6 n¬ 6
m5  m8. By transitivity, m5  m7, so 5
Graph the inequalities on the same number line.
has the greatest measure.
8. By the Exterior Angle Theorem, m8  m7 and 8 4 0 4 8 12 16 20 24 28 32 36
m8  m6, so 8 has the greatest measure. n¬ 32
9. By the Exterior Angle Theorem, m1  m6 and
m1  m9, so 1 has the greatest measure. 8 4 0 4 6 8 12 16 20 24 28 32 36

10. 2n  1 is even. n¬ 6


11. Alternate interior angles are not congruent. 8 6 4 0 4 8 12 16 20 24 28 32 36
12. Assume that Marcus spent less than one half n¬ 6
hour on a teleconference every day. If we let t1, t2,
and t3 be the time spent on a teleconference on 8 4 0 4 6 8 12 16 20 24 28 32 36

days 1, 2, and 3, respectively, then t1  t2  t3  Find the intersection.


the total amount of time over the three days The range of values that fit all three inequalities
spent on the teleconference. is 6  n  32.
Because he spent less than a half hour every day 16. y  0, so y  1  y. Then 7  x and x  0, so
on a teleconference, t1  0.5 and t2  0.5 and 0  x  7.
t3  0.5.
Then, by a property of inequalities,
t1  t2  t3  0.5  0.5  0.5 or 1.5 hours.

149 Chapter 5
17. x  7¬ 11 7  11  20, so 7, 11, 20 cannot be the measures
x¬ 4 of the sides of the triangle.
7  11¬ 2x 9  13  26, so 9, 13, 26 cannot be the measures
18¬ 2x of the sides of the triangle.
9¬ x 9. The ramp rises 2 feet as it runs 24 feet, so the
4x9 slope is 2 1
24 or 
12.
18. In the two triangles, each has a side of measure
10. x  55¬ 90
12 and the shared side is congruent to itself. The
x¬ 35
hypotenuse of the right triangle has a length of
13.86, which is shorter than the third length of C
11. The point P is the midpoint of B with
8 
8 2  10
the other triangle, 14. So the SSS Inequality
allows us to conclude that x  90. In any triangle,
coordinates   
2 , 2  (8, 6).

an angle has measure less than 180, so we can 12. 


BC is vertical, so A
T
 is horizontal. The
write the compound inequality 90  x  180. y-coordinate of A is 4, and so the y-coordinate of T
C
is also 4. T is on B, so the x-coordinate of T is 8.
19. Let d be the distance between NewYork and
Therefore the coordinates of T are (8, 4).
Atlanta.
554  399¬ d 13. SSS Inequality
953¬ d or d  953 14a. From the points (0, 200) and (4, 120), the slope of
399  d¬ 554 (200  120)
the line is    20. Find slope again
(0  4)
d¬ 155 using one of the given points and (10, 0); the
Therefore, 155 mi  d  953 mi. (120  0)
(4  10)  20. Since the slope is the
slope is  
20. A; 3  8  11, so 3, 8, 11 cannot be the sides of a
triangle. same, (10, 0) must be on the original line.
Students may check by drawing an extension of
the line and will see that it goes through (10, 0).
14b. The point (10, 0) shows that on the tenth
Chapter 5 Standardized Test Practice payment Kendell’s balance will be $0, so the
amount will be paid in full.
Pages 278–279 15a.
y C (3, 4)
1. D; there are 36 inches in a yard, so there are 362
or 1296 square inches in a square yard. So there
are 1296(80) or 103,680 yarn fibers in a square A (3, 1)
yard.
2. B; 6  7  6  4  12  11  46 units x
3. B; the converse is false because if an angle is
acute it can have any measure between 0 and 90. B (0, 2)
4. B;
A B
15b. AB =¬ 
[0  (3)]2
 (2
 1)2
¬ 99
¬ 18
F C
¬4.2
BC ¬ (3  0)2  [4
 (2 )]2
¬ 9  36
E D
¬ 45
There are 6 people at the meeting. Let
¬6.7
noncollinear points A, B, C, D, E, and F represent
the 6 people. Connect each point with every other AC ¬ 
[3  (3)] 2 (4  1)2
point. Between every two points there is exactly ¬ 36  9
one segment. For the 6 points, 15 segments can be ¬ 45
drawn. Thus 15 exchanges are made. ¬6.7
5. B 15c. isosceles triangle because B C is congruent to A
C

6. B; use indirect reasoning 15d. According to the Isosceles Triangle Theorem, if
7. C; the shortest distance from a point to a line two sides of a triangle are congruent, then the
segment is a perpendicular segment angles opposite those sides are congruent. Since
8.
?
A; 2  9¬ 10
?
2  10¬ 9
?
9  10¬ 2 C
B A C
, A  B.
11¬ 10 ✓ 12¬ 9 ✓ 19¬ 2 ✓ 15e. If one side of a triangle is longer than another
5  8  13, so 5, 8, 13 cannot be the measures of side, the angle opposite the longer side has a
the sides of the triangle. greater measure than the angle opposite the
shorter side. Since B C
 is longer than AB
,
mA  mC.

Chapter 5 150
Chapter 6 Proportions and Similarity
Page 281 Getting Started
1. 2
6-1 Proportions
3 y  4 ¬6

2 y ¬10
3
y ¬3
 Pages 284–285 Check for Understanding
2 (10) or 15
1. Cross multiply and divide by 28.
5 x
 ¬  4
2. 2. Sample answer: 5 0 5 4
1
4  8 , 10  8
6 12  
x 
12  5
6 ¬12  12
  4
3. Suki; Madeline did not find the cross products
10 ¬x  4 correctly.
14 ¬x number of goals
9
4.    12 or 3 : 4
y2 number of games
3. 4 ¬
3 y1 height of replica
10 in
ches
5.  ¬ 10 feet
4(y  1) ¬3(y  2) height of statue
4y  4 ¬3y  6 ¬10 in
ches
y  4 ¬6 120 inches
y ¬10 1
¬
12
2y 32
4. 4 ¬y
 6. x ¬
11

5 35
2y2 ¬128 35x ¬5(11)
y2 ¬64 35x ¬55
y ¬8 11
x ¬ 
7
(y  y ) 2.3
2
5. m ¬ 1
 x
4 ¬ 3.7
 7. 
(x2  x1)
2.3(3.7) ¬4x
 5
¬ 013 8.51 ¬4x
 6
¬3
 2.1275 ¬x
¬2 x 
2 4
8.  2 ¬5
(y  y )
2
6. m ¬ 
1 5(x  2) ¬2(4)
(x2  x1) 5x  10 ¬8
3  (3) 5x ¬18
¬ 2  (
  6)
0 x ¬3.6
¬8
9. Rewrite 9 : 8 : 7 as 9x : 8x : 7x and use those
¬0 measures for the sides of the triangle. Write an
2 (y  y )
1 equation to represent the perimeter of the
7. m ¬ 
(x2  x1) triangle as the sum of the measures of its sides.
¬2  4 9x  8x  7x ¬144
2  (
3) 24x ¬144
¬6
5
 x ¬6
8. yes; congruent alternate exterior angles Use this value of x to find the measures of the
sides of the triangle.
9. yes; congruent alternate interior angles
9x  9(6) or 54 units
10. No; 5 and 3 do not have a relationship that 8x  8(6) or 48 units
could be used to determine whether the lines are 7x  7(6) or 42 units
parallel.
10. Rewrite 5 : 7 : 8 as 5x : 7x : 8x and use those
11. 21  2 measures for the angles of the triangle. Write an
22  4 equation to represent the sum of the angle
23  8 measures of the triangle.
24  16 5x  7x  8x ¬180
12. 12  2  1  2  1 20x ¬180
22  2  4  2  2 x ¬9
32  2  9  2  7 Use this value of x to find the measures of the
42  2  16  2  14 angles of the triangle.
13. 31  2  3  2  1 5x  5(9) or 45
32  2  9  2  7 7x  7(9) or 63
33  2  27  2  25 8x  8(9) or 72
34  2  81  2  79

151 Chapter 6
scale on map (cm) distance on map (cm) 21. Rewrite 3 : 4 : 5 as 3x : 4x : 5x and use those
11.  ¬  measures for the sides of the triangle.
distance represented (mi) actual distance (mi)
1.5 2.4 3x  4x  5x ¬72
 0 ¬x
20 12x ¬72
1.5x ¬200(2.4) x ¬6
1.5x ¬480 Use this value of x to find the measures of the
x ¬320 sides of the triangle.
The cities are 320 miles apart. 3x  3(6) or 18 in.
4x  4(6) or 24 in.
5x  5(6) or 30 in.
Pages 285–287 Practice and Apply 22. Rewrite 1
 1 1 x x x
2 : 3 : 5 as 2 : 3 : 5 and use those
number of hits 8
number of games  
12.  10 or 4 : 5 measures for the sides of the triangle.
number of boys 76 x  x  x ¬6.2
13.     or 76 : 89
165  76
2 3 5
number of girls
number of rands 208  
30 2x  3x  5x ¬30(6.2)
number of dollars  
14.  18 or 104 : 9 15x  10x  6x ¬186
44,125
number of students   31x ¬186
15. 
number of teachers  or about 25.3 : 1
1747 x ¬6
AC 20  0
16. 
BH 70  10
Use this value of x to find the measures of the
20 sides of the triangle.
 60 or 1 : 3
 x 6
2  2 or 3 cm
17. Rewrite 3 : 4 as 3x : 4x and use those measures
for the two lengths of cable. x  6 or 2 cm
3 3
3x  4x ¬42 x 6
5  5 or 1.2 cm
7x ¬42
height of the door 15
x ¬6 23.     3
Alice’s height in Wonderland 10 or 2
Use this value of x to find the measures of the two
height of the door
lengths of cable. 24. 
Alice’s height in Wonderland
3x  3(6) or 18 ft
4x  4(6) or 24 ft height of door in Alice’s normal world
¬ 
Alice’s normal height
18. Rewrite 2 : 5 : 3 as 2x : 5x : 3x and use those 15 x
 ¬
measures for the angles of the triangle. 10 50
2x  5x  3x ¬180 15(50) ¬10x
10x ¬180 750 ¬10x
x ¬18 75 ¬x
Use this value of x to find the measures of the The height of the door in Alice’s normal world
angles of the triangle. would be about 75 inches.
2x  2(18) or 36 Lincoln’s height in model 8 i
n.
25.  ¬
5x  5(18) or 90 Lincoln’s height in theater 6 ft 4 in.
3x  3(18) or 54 8 in.
¬
76 
in.
19. Rewrite 6 : 9 : 10 as 6x : 9x : 10x and use those ¬2 : 19
measures for the angles of the triangle. number of people in United States
6x  9x  10x ¬180 26. 
number of pounds of ice cream consumed
25x ¬180
number of people in Raleigh, NC
x ¬7.2 ¬ 
number of pounds of ice cream
Use this value of x to find the measures of the 255,082,000 276,000
angles of the triangle.  44,800 ¬x
4,183,3
6x ¬6(7.2) or 43.2 255,082,000x ¬1,154,603,165,000,000
9x ¬9(7.2) or 64.8 x ¬4,526,400
10x ¬10(7.2) or 72
The people of Raleigh, North Carolina, might
20. Rewrite 8 : 7 : 5 as 8x : 7x : 5x and use those consume 4,526,400 pounds of ice cream.
measures for the sides of the triangle.
number of people in United States
8x  7x  5x ¬240 27. 
number of pounds of ice cream consumed
20x ¬240
1 person
x ¬12 ¬ 
number of pounds of ice cream
Use this value of x to find the measures of the 255,082,000 1
sides of the triangle.  44,800 ¬x
4,183,3
8x  8(12) or 96 ft 255,082,000x ¬4,183,344,800
7x  7(12) or 84 ft x ¬16.4
5x  5(12) or 60 ft One person consumed about 16.4 pounds of ice
cream.

Chapter 6 152
28. 3 ¬5
x 38c. The ratio 4 : 5 : 4 : 5 indicates that there are four
8
3(5)  8x sides, and opposite sides are congruent. This
15 ¬8x description fits a rectangle or a parallelogram.
15 39. Sample answer: It appears that Tiffany used
8 ¬x
 
rectangles with areas that were in proportion as a
a ¬1
29. 
5.18 4 background for this artwork. Answers should
4a ¬5.18(1) include the following.
a ¬1.295 • The center column pieces are to the third
3x 48 column from the left pieces as the pieces from
30. 3 ¬
2 
92 the third column are to the pieces in the outside
3x(92) ¬23(48) column.
276x ¬1104 • The dimensions are approximately 24 inches by
x ¬4 34 inches.
13 2
6
49 ¬ 7x
31.   1.6
18 12
40.  1 ¬ x
 
13(7x) ¬49(26)
1.618x¬ 12
91x ¬1274
x ¬7.4 cm
x ¬14
41. D; the ratio of wheat to rice to oats is 3 : 1 : 2, so
2x  13 4
32.  28 ¬7 the ratio of wheat to oats is 3 : 2.
(2x  13)(7) ¬28(4) 3 ¬x
2 120
14x  91 ¬112
3(120) ¬2x
14x ¬21
360 ¬2x
x ¬3

2 180 ¬x
4x  3
33.  
12 ¬5

4
180 pounds of wheat will be used. The answer is D.
(4x  3)(4) ¬12(5)
16x  12 ¬60
16x ¬48 Page 287 Maintain Your Skills
x ¬3 42. always; m1  m2 ¬180
b 1 3x  50  x  30 ¬180
5
b  1 ¬ 6
34. 
4x  20 ¬180
(b  1)(6) ¬(b  1)(5) 4x ¬200
6b  6 ¬5b  5 x ¬50
b  6 ¬5 m1 ¬3(50)  50
b ¬11 ¬150  50 or 100
3x  1 2 m2 ¬50  30 or 80
2 ¬

35. x 
2
(3x  1)(x  2) ¬4 So, m1  m2, and by the SAS Inequality
3x2  6x  x  2 ¬4 LS  SN.
3x2  5x  2 ¬0 43. Always; PNO is an exterior angle of SNO, so
(3x  2)(x  1) ¬0 mPNO  m2. Then by the SAS Inequality,
3x  2 ¬0 or x10 OP  SN.
3x ¬2 x  1 44. never; m1  m2 ¬180
2
x ¬ 3
 3x  50  x  30 ¬180
4x  20 ¬180
36. The larger dimension of the photograph is
4x ¬200
27.5 cm, so reducing this dimension to 10 cm will
x ¬50
give the maximum dimensions of the reduced
photograph. 45. Let the measure of the third side be x.
27
.5 21
.3 16  31 ¬x 16  x ¬31 31  x ¬16
10 ¬ x
 
47 ¬x or x  47 x ¬15 x ¬15
27.5x ¬10(21.3) Graph the inequalities on the same number line.
27.5x ¬213
x ¬7.75 15 10 5 0 5 10 15 20 25 30 35 40 45 47 50

The maximum dimensions are 7.75 cm by 10 cm. x  47


reduced length 7.7
5
37.    21.3 15 10 5 0 5 10 15 20 25 30 35 40 45 50
original length
 0.36 or 36% x  15
38a. The ratio 2 : 2 : 3 indicates that there are three
15 10 5 0 5 10 15 20 25 30 35 40 45 50
sides, two of which have the same measure. This
description fits an isosceles triangle. x  15
38b. The ratio 3 : 3 : 3 : 3 indicates that there are four 15 10 5 0 5 10 15 20 25 30 35 40 45 47 50

sides with all the same measure. This


description fits a square or a rhombus.

153 Chapter 6
Find the intersection. YZ   (6  4)2  (3 3)2
The range of values that fit all three inequalities   22  02
is 15  x  47.  4  or 2
46. Let the measure of the third side be x. SU   (2  0)2 (0  5)2
26  40 ¬x 26  x ¬40 40  x ¬26   (2)2  (5)2
66 ¬x or x  66 x ¬14 x ¬14  29
Graph the inequalities on the same number line. XZ   (6  4)2  (3 8)2
  2  (
2 5) 2
20 10 0 10 20 30 40 50 60 66 70 80 90
 29
x  66
STU  XYZ by SSS.
20 10 0 10 14 20 30 40 50 60 70 80 90
49. Start at P(3, 4). Move up 3 units and then
x  14 move right 5 units. Draw the line through this
point and P.
20 14 10 0 10 20 30 40 50 60 70 80 90
y

x  14

20 10 0 10 14 20 30 40 50 60 66 70 80 90
O x
Find the intersection.
The range of values that fit all three inequalities
is 14  x  66. P( 3, 4)
47. Let the measure of the third side be x.
11  23 ¬x 11  x ¬23 23  x ¬11 .
50. Plot points A(5, 3) and B(1, 8). Draw line AB
34 ¬x or x  34 x ¬12 x ¬12 y
Graph the inequalities on the same numer line.
B
( 1, 8)
15 10 5 0 5 10 15 20 25 30 34 35 40
x  34

15 10 5 0 5 10 12 15 20 25 30 35 40
A(5, 3)
x  12
O x

51. Find the slope of 


15 12 10 5 0 5 10 15 20 25 30 35 40
JK.
x  12 (y  y )
2 1
m ¬ 
15 10 5 0 5 10 12 15 20 25 30 34 35 40
(x2  x1)
3 5
¬
4  ( 1)
Find the intersection.
2
The range of values that fit all three inequalities ¬ 5

is 12  x  34. The line through E(2, 2) also has slope 2

5.
48. Start at E(2, 2). Move down 2 units and then
12 y
X (4, 8) move right 5 units. Draw the line through this
8 point and E.
S (0, 5) y
4 Z (6, 3)
U (2, 0) Y (4, 3) x
12 8 4 T (0, 0) 4 8 12 E (2, 2)
4
O x
8
12
ST  
(0  0)2  (0 5)2
52. Find the slope of 
QR.
 
02  (5)2 (y  y )
 25
 or 5 2
m ¬ 
1
(x2  x1)
XY  
(4  4)2  (3 8)2 6  2
  
0  (5)
2 2 ¬ 
4  6
 25
 or 5 8 4
¬
1 0 or 5
TU (2  0)2 (0  0)2 The product of the slopes of two perpendicular
 
(2) 
2 0 2
lines is 1.
 4
 or 2
5  4   1, the slope of the line
Since 4 5
perpendicular to 
QR through S(8, 1) is 5.
4

Chapter 6 154
Start at S(8, 1). Move down 5 units and then 2. See students’ drawings. Sample counterexample:
move right 4 units. Draw the line through this A rectangle with consecutive sides of 4 in. and
point and S. 12 in. would not have sides proportional to a
y rectangle with consecutive sides of 6 in. and 8 in.
because 4 12
6 8 .
  
3. If two polygons are congruent, then they are
similar. All of the corresponding angles are
congruent, and the ratio of measures of the
corresponding sides is 1. Two similar figures have
S (8, 1) congruent angles, and the sides are in proportion,
O x
but not always congruent. If the scale factor is 1,
then the figures are congruent.
53. Yes; 100 km and 62 mi are the same length, so
4. PQR and GHI each have two angles with
AB  CD. By the definition of congruent
measure 60, so the third angle of each triangle
segments, AB
C D
.
must also have measure 60. Thus, P  Q  R
54. d ¬(x   
x )2 (y2 
y1)2 PQ QR
RP
2 1
 G  H  I and  3
GH  HI  IG  7 ,
  
AB ¬
(8  12)2  (3  3
)2 so PQR GHI.
¬
(20)2 0 2
5. From the diagram, A  E, B  F,
¬400
 or 20.0 C  G, and D  H.
55. d ¬
(x2  
x1)2  (y2 y1)2 AD CB
 4 2
EH  GF  6 or 3
 
CD ¬
(5  0)
2  (1
2  0)2 DC BA 3 or 2
G 
FE  

H 9 3
¬
5  12
2 2 2

¬169
 or 13.0 The ratios of the measures of the corresponding
sides are equal, and the corresponding angles are
56. d ¬
(x2  
x1)2 
(y2 y1)2
congruent, so parallelogram ABCD parallelogram

2  45
  2 
(1)

2 2
1 EFGH.
EF ¬ 
6. Use the congruent angles to write the
¬1.22 0.52
corresponding vertices in order: ACB DFE.
¬1.69
 or 1.3 Write a proportion to find x.
57. d ¬
(x2  
x1)2 
(y2 
y1)2 AC C
B
F ¬

D FE
  
2
GH ¬ (4  3)2  2
 3
7  7
21 27
x ¬ 18
   

¬1
 
2
 
2
5
7
21(18) ¬27x
378 ¬27x
 25
¬ 1  
49
14 ¬x
So, DF  14.
¬

74
 AC 21 3
The scale factor is 
DF 

49 14 or 2 .
¬1.2 7. Use the congruent angles to write the corres-
ponding vertices in order: polygon ABCD
polygon EFGH. Write a proportion to find x.
DA BA
Page 288 Spreadsheet Investigation: Fibonacci HE ¬ FE
  
Sequence and Ratios 10 14
3 ¬ x  5

x   
1. It increases also.
10(x  5) ¬(x  3)(14)
2. odd-odd-even
10x  50 ¬14x  42
3. It approaches 1.618. 10x  92 ¬14x
4. The increase in terms confirms the original 92 ¬4x
observations. 23 ¬x
5. As the number of terms increases, the ratio of EF  x  5
each term to its preceding term approaches the  23  5 or 28
golden ratio. EH  x  3
 23  3 or 20
GF EF
CB  AB
 
6-2 Similar Polygons GF 28
16  
 
14
G
 F
  2, so GF  2(16) or 32.
16
Pages 292–293 Check for Understanding DA 10 1
1. Both students are correct. One student has
The scale factor is 
HE  
20 or 2 .
inverted the ratio and reversed the order of the
comparison.

155 Chapter 6
8. Write proportions for finding side measures. DB 2 3
MP  513 or 8
  
new length → x 1

original length → 60 4 BC 4 3
PN  1023 or 8
 
4x ¬60
CD
 3
NM  8
x ¬15 
new height → y 1 The ratios of the measures of the corresponding

original height → 40 4 sides are equal, and the corresponding angles are
4y ¬40 congruent, so BCD PNM.
y ¬10 2
height of replica 3503 feet
The new length is 15 cm, and the new height is 15.    1052 feet
height of actual tower
10 cm.
 1

3
9. Write proportions for finding side measures.
new first side → x The scale factor is 1

3.
 5
original first side → 3 8
16. The first copy is 80% or 
10 of the original. The
x  15
second copy must be 100% or 1 of the original.
new second side → y
  5 second copy 1
original second side → 5    8
first copy 10
y  25 10
new third side → z
 8
 5  1.25
original third side → 4
z  20 Use a scale factor of 1.25 or 125%.
The new side lengths are 15 m, 25 m, and 20 m, 17. Use the congruent angles to write the
so the perimeter is 15 m  25 m  20 m  60 m. corresponding vertices in order: polygon ABCD
polygon EFGH. Write a proportion to find x.
10. See students’ drawings. The drawings will be
AB CD
EF ¬ GH
similar since the measures of the corresponding    
sides will be proportional and the corresponding x  1 x
8 ¬5  1
angles will be congruent.
(x  1)(5) ¬8(x  1)
5x  5 ¬8x  8
5 ¬3x  8
13 ¬3x
6.56 cm 13
3 ¬x

AB  x  1
13 1
6
 3  1 or 3


Figure is not shown actual size.


CD  x  1
13 1
0
3  1 or 3

1
6
A
B 3
2
EF  8 or 3 .
The scale factor is  
Pages 293–297 Practice and Apply
18. Use the congruent angles to write the
11. BCF  DCF, ABC  EDC, BAF  DEF,
corresponding vertices in order: ABC EDC.
and AFC  EFC; B C
D C, A
B
E D;
Write a proportion to find x.
F
A E F, and CF
C F. Therefore, ABCF is
AC C B

EC ¬
similar to EDCF since they are congruent.
CD
12. 1  2  3  4; 6  5 because if two x 7 4
12  x ¬ 6

angles of one triangle are congruent to two angles
of a second triangle, then the third angles are (x  7)(6) ¬(12  x)(4)
congruent. XYX WX Z
, and Y
WW Z, so the 6x  42 ¬48  4x
ratio of the corresponding sides is 1. Therefore, 10x  42 ¬48
XYW XWZ. 10x ¬6
13. ABC is not similar to DEF. From the lengths x ¬3

5
of the sides we can determine that A AC  x  7
corresponds to D, but A  D.
 3 3
5  7 or 7 5

14. B  P, D  M, and C  N because if
two angles of one triangle are congruent to two CE  12  x
angles of a second triangle, then the third angles  12  3 2
5 or 11 5
are congruent. CB
The scale factor is CD  4
 2
6 or 3 .

Chapter 6 156
19. Use the congruent angles to write the 22. The enlargement process E can be represented by
corresponding vertices in order: ABE ACD.
Write a proportion to find x.
the equation E  5
 5
4 4x .  
AB AE 23. 5
 5  25
4  4  16

AC ¬ AD
   
10 6.25 24. Explore: Every millimeter represents 1 meter.
10  x  2 ¬ 6.25  x  1
   
The dimensions of the field are about 69 meters
10
 ¬ 6.2 5 by 105 meters.
x  12 x  5.25
Plan: Create a proportion relating each
10(x  5.25) ¬(x  12)(6.25)
measurement to the scale to find the
10x  52.5 ¬6.25x  75
measurements in millimeters. Then make a scale
3.75x  52.5 ¬75
drawing.
3.75x ¬22.5
Solve:
x ¬6 millimeters → 1 x ← millimeters
BC  x  2 
meters → 1 69 ← meters
 6  2 or 8
69  x
ED  x  1
The width of the field should be 69 millimeters in
 6  1 or 5
AB AB 10 the drawing.
AC  AB  BC  10  8 
The scale factor is       millimeters → 1 y ← millimeters
10 
 5
18 or 9 . meters → 1 105 ← meters
20. Use the congruent angles to write the 105  y
corresponding vertices in order: RST EGF. The length of the field should be 105 millimeters
Write a proportion to find x. in the drawing.
R T ST 105 mm
EF ¬ GF
  
1 5
 ¬ 1 0 69 mm
11.25 x
15x ¬11.25(10) Figure is not shown actual size.
15x ¬112.5
x ¬7.5 Examine: The scale is 1 : 1, so it is clear that the
GF  x dimensions in the drawing are reasonable.
 7.5 25. Explore: Every 1
4 inch represents 4 feet. The
EG GF dimensions of the basketball court are 84 feet by
SR ¬ ST
 
50 feet.
EG 7.
5
 .7 ¬
20
Plan: Create a proportion relating each
10
measurement to the scale to find the
(EG)(10) ¬(20.7)(7.5)
measurements in inches. Then make a scale
(EG)(10) ¬155.25
drawing.
EG ¬15.525
ST 10 Solve:
4
GF  7.5 or 3 .
The scale factor is     inches → 1 x ← inches
4
21. Write proportions for finding side measures. 
feet → 4 84 ← feet
first new length → x 5
 1(84) ¬4x
original length → 2.5 4 4
21 ¬4x
4x ¬2.5(5) 5.25 ¬x
4x ¬12.5
The length of the court should be 5.25 inches in
x ¬3.125
second new length → y 5
the drawing.
 inches 1 y ← inches
first new length → 3.125 4 4

4y ¬3.125(5) feet 4 50 ← feet
4y ¬15.625 1(50) ¬4y
4
y ¬3.9 12.5 ¬4y
first new width → z 5
 3.125 ¬y
original width → 4 4 The width of the court should be 3.125 inches in
4z ¬4(5) the drawing.
4z ¬20
5 1–4 in.
z ¬5
second new width → w 5
 3 1–8 in.
first new width → 5 4
4w ¬5(5) Figure is not shown actual size.
4w ¬25
Examine: The scale is 1

4 : 4. The dimensions in
w ¬6.25
the drawing are reasonable.
After both enlargements the dimensions were
about 3.9 inches by 6.25 inches.

157 Chapter 6
26. Explore: Every 1
8 inch represents 1 foot. The 35. L ¬S
dimensions of the tennis court are 36 feet by 78 mL ¬mS
feet. 30 ¬x
Plan: Create a proportion relating each K ¬R
measurement to the scale to find the mK ¬mR
measurements in inches. Then make a scale mK ¬180  (mQ  mS)
drawing. y ¬180  (80  30)
Solve: y ¬180  110
inches → 1 x ← inches y ¬70
8
 ¬
feet → 1 36 ← feet AB BC
FE ¬ EH
36.    
1 (36) ¬x x  2 8
8  5 ¬
1 10
4.5 ¬x
(x  2)(10)  15(8)
The width of the court should be 4.5 inches in the 10x  20 ¬120
drawing. 10x ¬100
inches → 1 y ← inches
8
   x ¬10
feet → 1 78 ← feet DC BC

GH ¬
EH
1(78) ¬y y3
8 8
 ¬ 1
9.75 ¬y 5 0
The length of the court should be 9.75 inches in (y  3)(10) ¬5(8)
the drawing. 10y  30 ¬40
10y ¬70
9 3–4 in.
y ¬7
x 3 12
16 ¬8
4 1–2 in. 37. 
(x  3)(8) ¬16(12)
Figure is not shown actual size. 8x  24 ¬192
Examine: The scale is 1 
8 : 1. The dimensions in
8x ¬216
the drawing are reasonable. x ¬27
27. Always; the corresponding angles are congruent y1 12
 ¬8
and the ratios of the measures of the 10
corresponding sides are all 1. (y  1)(8) ¬10(12)
8y  8 ¬120
28. Always; all angles are right angles and so all are
8y ¬112
congruent, and the ratios of the measures of the
y ¬14
corresponding sides are all the same.
2x 20
2 ¬

1
29. Never; the number of angles and sides of the 38. 
15
figures must be the same for the figures to be 2x(15) ¬12(20)
compared. 30x ¬240
30. sometimes; true when corresponding angles are x ¬8
congruent and ratios of measures of y4 15
 ¬
20
corresponding sides are equal, false when one of 12
these does not hold (y  4)(20) ¬12(15)
20y  80 ¬180
31. sometimes; true when the ratios of the measures
20y ¬100
of the corresponding sides are equal, false when
y ¬5
they are not.
RS TS
32. sometimes; true when corresponding angles are 39. 
WU ¬VW
congruent and ratios of measures of x 49
corresponding sides are equal, false when one of

29 ¬
20
these does not hold 20x ¬29(49)
33. Always; all angles have measure 60 and are 20x ¬1421
congruent, and the ratios of the measures of the x ¬71.05
corresponding sides are all the same. RT TS

UV ¬
VW
34. G ¬L y3 49
 ¬ 
mG ¬mL 21 20
87 ¬x  4 (y  3)(20) ¬21(49)
91 ¬x 20y  60 ¬1029
J ¬O 20y ¬969
mJ ¬mO y ¬48.45
y  30 ¬60 AD 12 12 8
AG  12  4.5  7.5  5
40.     
y ¬30

Chapter 6 158
41. AG  AD  GD Sides opposite 23º angle.
 12  4.5 PR 12.5
    2.5
 7.5 NM 5
The ratios of the measures of the corresponding
DC AD
GF ¬ AG
42.    sides are equal, and the corresponding angles are
D C 1 2 congruent, so NMO PRS.
 ¬
14 7.
5 49. y
7.5(DC)  14(12) 8
7.5(DC)  168
C B
DC ¬22.4 4
43. mADC  mAGF D M N
 108 O A 4 8 12 x
BC AD 4
EF ¬ AG
44.   
B C 1
8 ¬ 2
7.
8
5
7.5(BC)  8(12)
7.5(BC)  96 DA  4 and MN  8 so the scale factor is 8 
4 or 2.
BC ¬12.8 To move from point A to point B, move up 4 units
and then move 2 units to the right. Because the
45. AB  BC  CD  AD  26  12.8  22.4  12  73.2
scale factor is 2, to move from N to L, move up 8
AE AG
AB ¬ AD
46.     units and move 4 units to the right. So the
AE 7.5 coordinates of L could be (16, 8). Similarly the
6 ¬

2

12 coordinates of P could be (8, 8)
12(AE) ¬26(7.5) Another similar polygon can be obtained by
12(AE) ¬195 moving down and to the right. To move from N to
AE ¬16.25 L, move down 8 units and move 4 units to the
AE  EF  FG  AG  16.25  8  14  7.5 right. So the coordinates of L could also be
 45.75 L(16, 8). The coordinates of P could be (8, 8).
73
.2 8 50.
47.  5  5
y
45.7 8
48. ABC IHG JLK and NMO PRS; B
A  I  J because each one measures 53º. N 4
B  H  L because each one is a right angle. A M C
C  G  K because each one measures 8 4 O 4 8x
90º-53º or 37º. 4
D
So all corresponding angles are congruent. Now
determine whether corresponding sides are 8
proportional.
Sides opposite 90º angle. AD  
[2 (7)]2
 (
4  1)2
AC 5 1 AC 5 IG 10  2 
5  (5)2
         4     8
IG 10 2 JK 1.25 JK 1.25  50
 or 52

  2  (3)
 
7  1
Sides opposite 53º angle. 2 2
BC 4 1
    
BC 4
    4
HG 8
    8
NM  11
2 
HG 8 2 LK 1 LK 1
  2 
  5
2 2
Sides opposite 37º angle. 5 2
AB 3 1 AB 3 IH 6
         4     8
IH 6 2 JL 0.75 JL 0.75  5
 0 5 
4 or 2 2
52
The ratios of the measures of the corresponding MN 2 
The scale factor is AD    1

2.
sides are equal, and the corresponding angles are 52 
congruent, so ABC IHG, ABC JLK, and To move from point A to point B, move up 4 units
IHG JLK. and then move 9 units to the right. Because the
N  P because each one measures 67º. scale factor is 1

2 , to obtain L from N move up 2
M  R because each one is a right angle. units and then move 4.5 units to the right. So the
O  S because each one measures 90º-67º or 23º.
So all corresponding angles are congruent. Now
 1
coordinates of L could be 1, 1

2 . Similarly, the
determine whether corresponding sides are coordinates of P could be 2, 3.
3
proportional.
Another similar polygon can be obtained by
Sides opposite 90º angle.
moving down and to the left. To move from N to L
PS 32.5
    2.5 move down 2 units and then move 4.5 units to the
NO 13
Sides opposite 67º angle.
RS 30
left. So the coordinates of L could be 10, 3

2 .  
    2.5
MO 12 1
Similarly, the coordinates of P could be 5
2 , 1 .  

159 Chapter 6
3 62. B; rewrite 5 : 3 as 5x : 3x.
1 in
ch 4 inch
et ¬
51.   5x  3x ¬32
24 fe x feet
8x ¬32
x ¬24 3

4   x ¬4
x ¬18 5x  5(4) or 20
5
 inch
3x  3(4) or 12
1 in
ch 8
There are 20  12 or 8 more girls than boys. The
24 feet ¬ y feet
 
answer is B.
y ¬24 5

8   63. D;
y ¬15 180  (51  85)  44
The living room has dimensions 18 feet by 15 feet. 180  (51  44)  85
Corresponding angles are congruent.
1 12 9.
3
14 inches
4  3 and 3.1  3, so the ratios of two pairs of
1 in
ch   
et 
52.  
24 fe x feet
corresponding sides are equal, so the triangles are
x  24(1.25) similar.
x  30 x ¬
9.
3
2.8 3.1
3
inch
1 in
ch 8
3.1x ¬2.8(9.3)
24 feet  y feet
 
3.1x ¬26.04
y  24 3

8   x ¬8.4
The answer is D.
y9
The deck has dimensions 30 feet by 9 feet. 64. Multiply each coordinate by 2.
53. The sides are in a ratio of 4 : 1, so if the length of A
has coordinates (0  2, 0  2)  (0, 0).
WXYZ is x then the length of ABCD is 4x, and if B
has coordinates (8  2, 0  2)  (16, 0).
the width of WXYZ is y then the width of ABCD is C
has coordinates (2  2, 7  2)  (4, 14).
(4x)(4y) 16xy 65.
4y. Then the areas are in a ratio of    y C
xy xy
or 16 : 1. 12
4(3)
54. 4 12 4
1  1(3)  3 or 1 . The ratio is still 4 : 1.
   
8
55. The ratio of the areas is still 16 : 1 since the ratio C
of the sides is still 4 : 1. 4
56. No; the corresponding sides are not in proportion. B B
The ratio of the widths is 1 to 1 but the ratio of AA 4 8 12 16x
the heights is 2 to 1.
57. The widths are the same but the height of the
36% rectangle is twice the height of the 18% 66. AB ¬(8  0)2  (0  0)2
2w ¬
8  0
2 2
rectangle, so the ratio of the areas is  w or 2 : 1. ¬64
 or 8
36
% 0.3
6
18%  0.18 or 2 : 1, so
The ratio of the percents is   A
B
 
(16  0)2 (0  0)2
the ratios are the same.  
16 
2 0 2

58. Sample answer: The difference between increase  256


 or 16
and decrease is 8%, so the level of courtesy is only BC ¬(2  8)2  (7  0)2
slightly decreased. ¬
(6)2 
 72
59. a  b  c  1 ¬85

3a 3b 3c 3
a  b c a  b c B
C
  
(4  16) 2(14  0)2
 1
3(a  b  c)  3
 
3a  3b  3c  
(12)2  142
a  6 c 6  340
 or 285 
3a  6 3c  6
60.  
The sides are no longer proportional, so the new AC ¬(2  0)2  (7  0)2
triangles are not similar.  
2  7
2 2

 53

61. Sample answer: Artists use geometric shapes in
patterns to create another scene or picture. The A
C
 
(4  0)2  (14  0)2
included objects have the same shape but are  
4  1
2 4 2

different sizes. Answers should include the  212


 or 253 
AB 8 1
B
 16 or 2
following. 67. 
A

• The objects are enclosed within a circle. The AC 
5 3
objects seem to go on and on. 
C
 2
A
   or 1

2
53
• Each “ring” of figures has images that are BC 8 5

C
 2
B
   or 1

2
approximately the same width, but vary in 85
number and design.

Chapter 6 160
68. You could use the slope formula to find that 83. m2  m5  180 (supplementary consecutive
C
B  B
C

. Thus, ABC  A
B
C
and interior )
ACB  A
C
B
because of corresponding 62  m5  180
angles. A  A
because of the Third Angle m5  118
Theorem. 84. mABD  m1 ( corresponding )
69. The sides are proportional and the angles are  118
congruent, so the triangles are similar. 85. m6  m4  180 (supplementary consecutive
interior )
Page 297 Maintain Your Skills m6  118  180
b 2
m6  62
7.8 ¬ 3
70.  86. m7  m6 ( alternate interior )
3b ¬7.8(2)  62
3b ¬15.6 87. m8  m4 ( corresponding )
b ¬5.2  118
c 2 5
c  3 ¬ 4
71. 
(c  2)(4) ¬(c  3)(5)
4c  8 ¬5c  15 6-3 Similar Triangles
c  8 ¬15
c ¬23
c ¬23 Page 298 Geometry Activity: Similar Triangles
4 FD 4
ST  7  0.57
2  1. 
72. 
4y  5 ¬ y
2y ¬(4y  5)(4) EF 2.
5
S 
4.4  0.57

R
2y ¬16y  20 ED 4.
3
RT  7.6  0.57
  
18y ¬20
10 All of the ratios equal about 0.57.
y ¬9

2. Yes, all sides are in the same ratio.
73. 
BC B A, B
OB O
, and mOBC  mOBA. By
3. Sample answer: Either all sides proportional or
the SAS Inequality, OC  AO.
two corresponding angles congruent.
74. ABC is isosceles with base angles
1[180  (68  40)]  36. Then mAOB  180 
2
(40  36)  104 and mAOD  180  104  76 so, Page 301 Check for Understanding
mAOD  mAOB. 1. Sample answer: Two triangles are congruent by
75. mABD  mADB because if one side of a the SSS, SAS, and ASA Postulates and the AAS
triangle is longer than another side, then the Theorem. In these triangles, corresponding parts
angle opposite the longer side has a greater must be congruent. Two triangles are similar by
measure than the angle opposite the shorter side. AA similarity, SSS Similarity, and SAS Similarity.
76. x  52  35  180 In similar triangles, the sides are proportional and
x  93 the angles are congruent. Congruent triangles are
77. x  32  57  180 always similar triangles. Similar triangles are
x  91 congruent only when the scale factor for the
proportional sides is 1. SSS and SAS are common
78. x  40  25  180
relationships for both congruence and similarity.
x  115
2. Yes; suppose RST has angles that measure 46°,
79. m1 ¬m2
54°, and 80°, ABC has angles that measure 39°,
10x  9 ¬9x  3
63°, and 78°, and EFG has angles that measure
x  9 ¬3
39°, 63°, and 78°. So ABC is not similar to
x ¬12
RST and RST is not similar to EFG, but
m1  10(12)  9 ABC is similar to EFG.
 120  9 or 111
3. Alicia; while both have corresponding sides in a
m2  9(12)  3 ratio, Alicia has them in proper order with the
 108  3 or 111 numerators from the same triangle.
80. m1  m4 ( alternate interior ) 4. A  D and F  C, so by AA Similarity,
 118) ABC DEF.
81. m2  m4  180 (supplementary consecutive DE FE
AB ¬ CB
  
interior )
x 3
m2  118  180  5 ¬
4 15
m2  62 15x ¬45(3)
82. m3  m4  180 (linear pair) 15x ¬135
x ¬9
m3  118  180
DE  x  9
m3  62

161 Chapter 6
5. A  D and B  E, so by AA Similarity, Pages 302–305 Practice and Apply
ABC DEF. 10. If the measures of the corresponding sides are
AB B C
E ¬
 proportional, then the triangles are similar.
D EF
QR QP
x 5   7 or 1,   
10 R
 or 1,  P 15
 or 1

x 4  3 NO 21 3 NM 30 3 OM 45 3
QR QP R P
3x ¬5(x  4)     , so by SSS Similarity,
NO NM OM
3x ¬5x  20 MNO PQR.
2x ¬20
x ¬10 11. If the measures of the corresponding sides are
AB  x  10 proportional, then the triangles are similar.
DE  x  4 QR SR
  7 1
QS
 3 1
U V  14 or 2 and TU  6 or 2

TV
 10  4 or 6
QR SR QS

U V  TU , so by SSS Similarity,
  
6. Triangles EFD and BCA are right triangles. To TV
determine whether corresponding sides are QRS TVU.
proportional, find AB. EF EG
12. F  J and   4
IJ  IA  5 , but SSA is not a

(AB)2 ¬(AC)2  (BC)2
valid justification for similarity. There is not
(AB)2 ¬102  52  100  25 enough information to determine whether the
AB ¬125
 or 55  triangles are similar.
D
E 8 and  EF 4
AB ¬ 5 C  5
 B 13. mR  mS  mT ¬180
5
8 4, so the triangles are not similar because mR  120  20 ¬180
55 5 mR ¬40
corresponding sides are not proportional.
mJ  mK  mL ¬180
7. If the measures of the corresponding sides are 40  120  mL ¬180
proportional, then the triangles are similar. mL ¬20
DF 25 S  K and R  J, so RST JKL by AA
9 DE  FE 2
1
AB  3 or 3, AC  81 or 3, BC  7 or 3
    
3 Similarity.
DF D E FE
AB  AC  BC , so by SSS Similarity,
   
14. ST  XV, UT  WV, and T  V,
DEF ACB. so STU XVW by SAS Similarity.
8. Triangles ABC and EDF are isosceles triangles AB
 3 1 BC 5 1
JK  9 or 3 and KL  15 or 3 , and B  K.
15.  
because they each have a pair of congruent sides.
ABC JKL by SAS Similarity.
Base angles of isosceles triangles are congruent,
so B  C and D  F. From the figure, 16. Since AE
 BD, EAC  DBC and AEC 
B  D, so by transitivity B  F and BDC because they are corresponding angles. By
D  C. So ABC EDF by AA Similarity. AA Similarity, AEC BDC.
9. tower 17. If the measures of the corresponding sides are
proportional, then the triangles are similar.
ST 6 3 SR 10
.5 3.5

BA 
20 or  and   
10 BC 30 or 10
ST SR
x ft post 
BA 
BC, so RST is not similar to CBA
because the sides are not proportional.
4 ft 6 in.
18.  D
AE C, so A  C and E  D because they
100 ft 3 ft 4 in.
are alternate interior angles. By AA Similarity,
Assuming that the sun’s rays form similar ABE CBD.
triangles, the following proportion can be written. AB EB
CB ¬ DB
   
height of tower (ft) tower shadow (ft)
   x  3 5
height of post (ft) post shadow (ft)  ¬
2x  8 3
Substitute the known values and let x be the 3(x  3) ¬5(2x  8)
height of the cell phone tower. 3x  9 ¬10x  40
x ¬ 
1
100
1 7x  9 ¬40
42 33 7x ¬49
 
31
3  
 x ¬ 41 (100)
2
x ¬7
AB  x  3
 1
33 x ¬450
 7  3 or 10
x ¬135
BC  2x  8
The cellphone tower is 135 feet tall.
 2(7)  8 or 6

Chapter 6 162
19. 
DC EB
, so ADC  AEB and ACD  ABE AB  62
2  1 2  180
 or 65
because they are corresponding angles. By AA BC  62  3  45
2  or 35

Similarity, ABE ACD. CA  |7  (8)|  15
AB AE
AC ¬ AD
    ST  |6  (4)|  10
x  2 8 5 TB  82  42  80
 or 45


x  2 6 ¬8
8(x  2) ¬(x  8)(3) BS  22  42  20
 or 25

8x  16 ¬3x  24 CA 15 6
5
3 AB  or 3, and
ST  10 or 2 , TB  45
   
5x  16 ¬24  2
BC
 3 
5
 3.
5x ¬8 or
BS 25 2
x ¬8
 C A A B BC
ST  TB  BS , ABC TBS by SSS
Since 
5    
AB  x  2 Similarity.
 8
 3
5  2 or 3 5 23. The perimeter of ABC is 65   35  15 or
AC  x  2  6 15  95. The perimeter of TBS is 45
 8 3
5  2  6 or 9 5
 25  10 or 10  65
.
15  
95
 3(5  35
)
20. ABD and FEC are right triangles with    or 3 

10  65 2(5  35
) 2
A  F and right angles B and E. Because all
right angles are congruent, ABD FEC by AA 24. False; this is not true for equilateral or isosceles
Similarity. triangles.
BD AB 25. True; similarity of triangles is transitive.
CE ¬ FE
  
x 1 26. QRS and STR are right angles, so
3
x  2 ¬ 8
 QRS  STR. Q  Q, so QRS QTR by
8(x  1) ¬3(x  2) AA Similarity. S  S, so QRS RTS by AA
8x  8 ¬3x  6 Similarity. Therefore, QTR RTS by
5x  8 ¬6 transitivity.
5x ¬14 27. AB FD, so BAE  AFC and ABE  ECF
14
x ¬5
because they are alternate interior angles. Then
BD ¬x  1 EAB EFC by AA Similarity. A D
 B C, so
14 9
ADF  ECF because they are corresponding
¬5  1 or 5 angles. F  F, so EFC AFD by AA
EC ¬x  2 Similarity. Then EAB AFD by transitivity.
14 24
¬5  2 or 5
 
28. PSY and PQR are right angles and so they are
21. ABC and ARS are right triangles with congruent. P  P, so PSY PQR by AA
A  A and right angles ASR and ACB. R
Similarity. P  W
X , so YWX  WYP because
Because all right angles are congruent, they are alternate interior angles.
ABC ARS by AA Similarity. WYX  YSP because they are right angles, so
A
 S
¬SR WYX YSP by AA Similarity.
AC CB
PY
  ¬PS
X
x 6

12 ¬ 9 XW Y
9x ¬72 P Y 3
0 ¬6

1
x ¬8
6(PY) ¬10(3)
AB ¬x  7
6(PY) ¬30
¬8  7 or 15
PY ¬5
AS ¬x
SY PS
¬8  ¬
YW X Y
22. y SY
  ¬3
8 6
C ( 2, 7) S(0, 6)
6(SY) ¬8(3)
6(SY) ¬24
B(4, 4) SY ¬4
PQ PR
 ¬ 
PS PY
PQ 5 
3 ¬ 5
5
O x
5(PQ) ¬3(10)
5(PQ) ¬30
PQ ¬6
T (0, 4) 29. 
PR KL, so RQM  LNM, PQM  KNM,
L  QRM, and K  QPM, since these are
all pairs of corresponding angles. LNM, KNM,
RQM, PQM, and LMK are all right angles,
A( 2, 8)
so each is congruent to the others.

163 Chapter 6
NM 9
Since LMK  MNK and K  K, 8 ¬6
LMK MNK by AA Similarity. 6(NM) ¬8(9)
LK MK

M K ¬
NK 7
NM ¬ 2
6 or 12
16  9  KP
2(KP) 
 KP ¬

2(KP)   Since L  QRM and LNM  RQM,
9
25 3(KP) QRM NLM by AA Similarity.
3(KP) ¬9 QR RM
 ¬
25(9) ¬9(KP)2 NL LM
25 ¬(KP)2 3
2
3 3
4
0

5 ¬KP 16 ¬ 
 LM
KM  KP  PM
 KP  2(KP)
32
 4
3 (LM) ¬16 3
 0
 
40 3
 3(KP) LM ¬16  3  32
 3(5) or 15 LM ¬20
PM  2(KP)  2(5) or 10 IJ HJ
Since L  QRM and K  QPM, 30. 
XJ 
YJ and J  J,
LKM RPM by AA Similarity. so IJH XJY by SAS Similarity.
LK KM mJXY  180  mWXJ
RP ¬ PM
   
 180  130
25 15

RP ¬
10  50
25(10) ¬15(RP) mJIH  mJXY by corr. 
25
0  50
15 ¬RP

50 mYIZ  mJIH by vert. 
3 ¬RP
  50
Since KNM  PQM and K  QPM, mJYX mYIZ + mWZG
KNM PQM by AA Similarity.  50  20
KN KM  70
PQ ¬ PM
   
because exterior angle  sum of remote interior
9 15
 ¬
PQ 10 angles
9(10) ¬15(PQ) mJHI  mJYX  70 by corr. 
90 ¬15(PQ) mJ  mJXY  mJYX ¬180
6 ¬PQ mJ  50  70 ¬180
RP  RQ  PQ mJ ¬60
50 mJHG  mJHI ¬180 Linear pair
3  RQ  6
 
mJHG  70 ¬180
50
3  6  RQ
  mJHG ¬110
32 31. RST is a right angle, so mRST  90.
3  RQ
 
mRTS  47.
mMQP  mQPM  mPMQ ¬180 mRTS  mR  mRST ¬180
90  mQPM  mPMQ ¬180 47  mR  90 ¬180
mQPM  mPMQ ¬90 mR ¬43
Since RMP is a right angle, UVT is a right angle, so mUVT  90.
mRMQ  mPMQ  90 mUVT  mTUV  mUTV ¬180
mQPM  mPMQ  mRMQ  mPMQ 90  mTUV  47 ¬180
mQPM  mRMQ, so QPM  RMQ. mTUV ¬43
Therefore, RQM MQP by AA Similarity. RUS is a right angle, so mRUS  90.
RQ MQ mRUS  mR  mRSU ¬180

MQ ¬
QP 90  43  mRSU ¬180
32

3 MQ mRSU ¬47
MQ ¬6
SUT is a right angle, so mSUT  90.
32
(MQ)2 ¬3 6
 mSUV  mTUV ¬90
(MQ)2 ¬64 mSUV  43 ¬90
MQ ¬8 mSUV ¬47
RM QM 32. Assuming that the sun’s rays form similar
MP ¬ QP
 
triangles, the following proportion can be written.
RM 8
0 ¬6
1 height of pyramid (ft)

height of staff (paces  3)
6(RM) ¬10(8)
RM ¬80
 40
 pyramid shadow length (paces  3)
6 or 3  
staff shadow length (paces  3)
Since KNM PQM,
NM KN
QM ¬ PQ
  

Chapter 6 164
Substitute the known values and let x be the P
35. Given: L M
N
 L P
height of the pyramid. LJ PJ J
Prove: 
JN 
JM
x ¬(125  
114)(3)
2(3) 3(3) Proof:
x 71
 ¬7 M N
6 9
9x ¬6(717) Statements Reasons
9x ¬4302
P
1. L M
N
 1. Given
x ¬478
The pyramid was about 478 feet tall at that time.
2. PLN  LNM, 2. Alternate Interior
D
33. x must equal y. If B A E, then CBD  CAE
LPM  PMN  Theorem
because they are corresponding angles and
C  C so BCD ACE by AA Similarity. 3. LPJ NMJ 3. AA Similarity
B C DC 2 x
Then AC  EC . Thus, 4  x  y . Cross multiply
 LJ PJ 4. Corr. sides of s
JN  JM
4.  
and solve for y. are proportional.
2 ¬x
4 xy 36. Given: A
EB C
, B
H
AE
,
2(x  y) ¬4x E
J
CA E

2x  2y ¬4x J
a. Prove: ABH DCB
2y ¬2x H D
y ¬x
34. Given: B  E; Q P BC
; Q
PE F
;
AB BC
 
DE   
EF A B C
Proof:
Prove: ABC DEF
AHB, AJC, and EBC are right angles
A because perpendicular lines form right angles.
D
Since all right angles are congruent,
Q P AHB  AJC  EBC. Since A  A by the
B C E F Reflexive Property, ABH ACJ, by AA
Proof: Similarity. Likewise, since C  C,
Statements Reasons ACJ DCB. By the Transitive Property,
ABH DCB.
1. B  E, Q
P
 B
C
; 1. Given BC
b. Prove:  BD
  
AB BC BE BA
P
QE
F
; 
DE 
EF Proof:
2. APQ  C 2. Corresponding From part a, A  CDB by definition of similar
 Postulate triangles. ABE  DBC because all right angles
AQP  B
are congruent. Thus, ABE DBC by AA
Similarity.
3. AQP  E 3. Transitive BC BD
BE  BA from definition of similar triangles.
   
Prop. of  
4. ABC AQP 4. AA Similarity 37. Given: BAC and EDF are right triangles.
AB AC
E 

AB BC 5. Def. of s D DF
5.  
AQ QP Prove: ABC DEF
6. AB  QP  AQ  BC 6. Cross products B
E
AB  EF  DE  BC
7. QP  EF 7. Def. of A C D F
 segments Proof:
8. AB  EF  AQ  BC 8. Substitution Statements Reasons
1. BAC and EDF are 1. Given
9. AQ  BC  DE  BC 9. Substitution right triangles.
10. AQ  DE 10. Div. Prop. 2. BAC and EDF are 2. Def. of rt. 
right angles.
Q
11. AD
E
 11. Def. of
 segments 3. BAC  EDF 3. All rt.  are .
AB AC
DE  DF
4. 
12. AQP  DEF 12. SAS    4. Given

13. APQ  F 13. CPCTC 5. ABC DEF 5. SAS Similarity

14. C  F 14. Transitive Prop.

15. ABC DEF 15. AA Similarity

165 Chapter 6
38. Reflexive Property of Similarity 40. If the side of DEF that is 36 cm corresponds to
Given: ABC the shortest side of ABC, then we can find the
Prove: ABC ABC lengths of the other sides of DEF using
F proportions.
36 x
4 ¬ 6
C 
D E
36(6) ¬4x
A B I 216 ¬4x
54 ¬x
36 y
4 ¬ 9
 
G H
36(9) ¬4y
Proof: 324 ¬4y
Statements Reasons 81 ¬y
1. ABC 1. Given The perimeter of DEF is 36  54  81 or
171 cm.
2. A  A, B  B 2. Reflexive Prop.
41. Assume the lines of sight create similar triangles.
3. ABC ABC 3. AA Similarity x 87
.6

1.92 ¬
0.4
Symmetric Property of Similarity 0.4x ¬1.92(87.6)
Given: ABC DEF 0.4x ¬168.192
Prove: DEF ABC x ¬420.48
Proof: The tower is about 420.5 m tall.
Statements Reasons 42. It is difficult to measure shadows within a city.
1. ABC DEF 1. Given 43. Assume that ADFE is a rectangle.
D G
2. A  D, B  E 2. Def. of polygons

3. D  A, E  B 3. Symmetric Prop. F

4. DEF ABC 4. AA Similarity

Transitive Property of Similarity


R
Given: ABC DEF and DEF GHI
Prove: ABC GHI A
Proof: A H
E
Statements Reasons
Let R be the point on EF
 where the vertical line
1. ABC DEF, 1. Given F
from D crosses E . Then ADR  DRF because
DEF GHI D
A  EF and alternate interior angles are
congruent. DFR  AHG because all right
2. A  D, B  E, 2. Def. of polygons angles are congruent. So AGH DRF by AA
D  G, E  H Similarity.
GH AH
3. A  G, B  H RF ¬ DF
3. Trans. Prop.   

4. ABC GHI 4. AA Similarity  ¬0


x 150
6 10
10x ¬6(1500)
39. MKO  MOP because they are both right 10x ¬9000
angles, and M  M, so MKO MOP by AA x ¬900 cm or 9 m
Similarity. OKP is a right angle because it forms So, the height of the tree is 9 m  1.75 m
a linear pair with right angle MKO. or 10.75 m.
OKP  MOP and P  P, so MOP
44. No; the towns are on different latitudinal lines, so
OKP by AA Similarity. Then MKO OKP by
the sun is at a different angle to the two
transitivity.
MK OK
buildings.
OK ¬ KP
 
45. y
1.
5 4.
5 8
4.5 ¬ KP
  A
B
1.5(KP) ¬4.5(4.5) 4
1.5(KP) ¬20.25 D
KP ¬13.5 8 O 4 x
The distance KP is 13.5 feet. C 4

Chapter 6 166
PS SR
46. If ABC ADE then  AB BC  ¬D
AD  
AD C
DE . Plot point E
PS 0.
7
C
so that B D
E
 and the proportion is true. 2.2 ¬ 1.4
  

AB ¬
[2  
(10)]2  (
4  6)2 1.4(PS) ¬2.2(0.7)
1.4(PS) ¬1.54
¬8
 2
2  (2)
PS ¬1.1
¬68
 or 217 SR 0.
7
The scale factor is  1
CD 1.4 or 2 .
AD ¬ 
[6  (10)]2  (2
 6)2
52. Use the congruent angles to write the
¬16
 2 
(4)2 corresponding vertices in order. EFG XYZ
¬272
 or 417 XY YZ
AB 2
 
17
or 1

EF ¬
FG
2 , so DE  2(BC). To get

AD 
4
17 22
.5 7.
5
6x ¬ 10
 
from point B to point C, move down 6 units and
then move left 2 units. To get from point D to 22.5(10) ¬6x(7.5)
point E, move down 12 units and then move left 225 ¬45x
4 units. Locate point E at (2, 10). 5 ¬x
EF  6x
47. ABC ACD by AA Similarity.
 6(5) or 30
ABC CBD by AA Similarity. XZ YZ
ACD CBD by AA Similarity. 
EG ¬
FG
XZ 7.
5
25 ¬

48. Sample answer: Engineers use triangles, some the
10
same shape, but different in size, to complete a
10(XZ) ¬25(7.5)
project. Answers should include the following.
10(XZ) ¬187.5
• Engineers use triangles in construction because
XZ ¬18.75
they are rigid shapes. FG 10 4
YZ  7.5 or 3 .
The scale factor is 
• With the small ground pressure, the tower does   
not sink, shift, lean, or fall over. 53. 1y ¬3
15
AE AB
49. A; ABE ACD by AA Similarity, so 
AD
 
AC . 15 ¬3y
10  4 x 2 5 ¬y
10 ¬ x  2  5
  

 ¬ xx 
6  2 54. 6
 7
8 ¬ b
10 3
6b ¬8(7)
6(x  3) ¬10(x  2)
6b ¬56
6x  18 ¬10x  20 28
18 ¬4x  20 b ¬ 3
38 ¬4x 20
 m
28 ¬ 21
55. 
9.5 ¬x
20(21) ¬28m
50. B;
x 3 x 420 ¬28m
 6 ¬ x 2 15 ¬m
(x  3)(x  2) ¬6x 16 9
x2  2x  3x  6 ¬6x 56.  7 ¬ s
x2  x  6 ¬6x 16s ¬7(9)
x  5x  6 ¬0
2 16s ¬63
(x  6)(x  1) ¬0 63
s ¬16

x  6 ¬0 or x  1 ¬0
x ¬6 x ¬1 57. Find the coordinates of T.
y
12
B
Page 306 Maintain Your Skills 8
51. Use the congruent angles to write the
corresponding vertices in order. 4
PQRS ABCD x
PQ SR
 ¬
AB DC
4 4 8 12
C
x ¬.
0 7 4
3.2 1.4
1.4x ¬3.2(0.7) 8
1.4x ¬2.24 A
x ¬1.6 12
BC CD 
AT is a median from A to B
C
, so T is the midpoint

QR ¬
RS

B C 1.4
C
of B . Then T has coordinates
 7 ¬
0.  5 
9 11  (1)
0.7
0.7(BC) ¬0.7(1.4)
 2 , 2  (7, 5).
1  11 12
BC ¬1.4 C
The slope of B is 9 5  4 or 3.

167 Chapter 6
5  (9) 14 PR QR
T
The slope of A is  3)  
7  (  7   ¬ 
10 or 5 . TR SR
 ¬

 
PT x 10 5
3 7
5 1, so A
 T
 is not perpendicular to x

10
40
PT  3 ¬
15

C
B. A
T
 is not an altitude.  4
0 10

3
58. p: you are at least 54 inches tall
q: you may ride the roller coaster  4
10 PT   0
  
4
0
3 ¬ 3 (15)

Adam is 5 feet 8 inches tall, or 5  12  8  40
10(PT)   0 60 0
3 ¬ 3

68 inches, so he can ride the roller coaster by the
200
Law of Detachment. 10(PT) ¬3
2 
9 15  11 20
59.  2 , 2 ¬2, 2
11 26 PT ¬ 3
¬(5.5, 13) scale on map (cm)
5.   
distance on map (cm)
4  4  (12) 2 8 distance on map (mi) actual distance (mi)
60.  
2 ,
2 
 
2 ¬ 2, 2  1 .5 29.2

100 ¬x
¬(1, 4)
1.5x ¬100(29.2)
7 8  (13)
0  7 5
61.  
2 , 2 ¬ 2, 2    1.5x ¬2920
x ¬1947
¬(3.5, 2.5)
The cities are about 1947 miles apart.

Page 306 Practice Quiz 1


1. yes; A  E, B  D, 1  3, 2  4 and 6-4 Parallel Lines and Proportional
AB BC AF FC

ED DC 
EF FC  1
 Parts
 5
6.5 6.
5 5
2. no; 6 5.5 and 5.5 6


3. D  B and AED  CEB, so ADE CBE Pages 311–312 Check for Understanding
by AA Similarity. 1. Sample answer: If a line intersects two sides of a
DE AD triangle and separates sides into corresponding
BE ¬ CB
  
segments of proportional lengths, then it is
3x  2
6 ¬ 10
15 parallel to the third side.
15(3x  2) ¬6(10) 2. Sample answer:
45x  30 ¬60 m n
45x ¬90 A B
x ¬2
AE
 AD
 ¬  D
CE CB
A E 10 C
12 ¬ 15
  
15(AE) ¬12(10)
15(AE) ¬120 m n
AE ¬8 A B
DE  3x  2
D
 3(2)  2 or 4
C
4. 
ST Q
P
, so TSR  PQR. R  R, so
PQR TSR by AA Similarity.
PQ QR 3. Given three or more parallel lines intersecting

TS ¬
SR two transversals, Corollary 6.1 states that the
25 10  5

TS ¬  parts of the transversals are proportional.
10
25(10) ¬(10  5)(TS) Corollary 6.2 states that if the parts of one
transversal are congruent, then the parts of every
250 ¬15(TS)
50 transversal are congruent.
3 ¬TS
 
W
4. L T
S
, so by the Triangle Proportionality
(SR)  (TR)2 ¬(TS)2
2
LT WS
0 2 Theorem,  L 
R
 
5 . Substitute the known
102  x2 ¬ 3
RW
measures.
2500
100  x2 ¬ 9 
5 6
5 ¬
9 
16 00 RW
x2 ¬9
4(RW) ¬5(6)
40
x ¬3 4(RW) ¬30
RW ¬7.5

Chapter 6 168
5. 
LW TS
, so by the Triangle Proportionality To find y:
T
L WS
Theorem, LR  RW . Substitute the known y ¬3
5y  2

measures. 2
y ¬2
5
8 
3 WS
 3 ¬6 y ¬5
6(5) ¬3(WS) 12. To find x:
30 ¬3(WS) 1x  2 ¬2x  4
3 3
10 ¬WS
1
3 x  2 ¬4

6. Use the Midpoint Formula to find the midpoints
B
of A and A
C
. 1
3 x ¬6

10  (2) 0  6 x ¬18

D 2,  
2  D(4, 3) To find y:
4  (2) 0  6
E2, 2  E(3, 3) 5y ¬7
3y  8


E
7. If the slopes of D and B
C
 are equal, D
E
 B
C
. 8y ¬8
3
E 33
 y ¬3
slope of D 3 
 4 or 0
00 13. The streets form a triangle cut by a Walkthrough
C
slope of B
10  
(4) or 0 that is parallel to the bottom of the triangle. Use
Because the slopes of D E
 and BC
 are equal, the Triangle Proportionality Theorem.

DE B C. Talbot Rd. Woodbury Ave.
8. First, use the Distance Formula to find BC and Entrance to Walkthrough Entrance to Walkthrough
  
DE. Walkthrough to Clay Rd. Walkthrough to Clay Rd.
8 8 0 x
BC ¬ [10  (4)]2
 (0 0)2 14 08 ¬
17
60
¬ 196  0 880(1760) ¬1408x
¬14 1,548,800 ¬1408x
1100 ¬x
DE ¬ (3  4)2 
(3  3)2
The distance from the entrance to the
¬ 49  0 Walkthrough along Woodbury Avenue is 1100
¬7 yards.
DE 7 1
BC  14 or 2

DE 1 1
BC  2 , then DE  2 BC.
If 
Pages 312–315 Practice and Apply
9. MQ ¬MR  RQ 14. 
MN YZ
, so by the Triangle Proportionality
12.5 ¬4.5  RQ M
Y N
Z
Theorem, XM  XN . Substitute the known

8 ¬RQ measures.
MY 9
4 ¬ 6
MP ¬MN  NP 
25 ¬9  NP 6(MY) ¬4(9)
16 ¬NP 6(MY) ¬36
N
In order to show R Q
P
, we must show that MY ¬6
MR MN XY  XM  MY
RQ  NP .
 
 4  6 or 10
MR 4.5 9
RQ ¬ 8 or 16
   15. 
MN Y Z , so by the Triangle Proportionality
M Y NZ
MN 9 Theorem,   . Substitute the known
NP ¬ 16
 XM XN
measures.
MR MN 9 10  t 
RQ  NP  16 . Since the sides have
Thus,     2
¬ 1
2 t2
N
proportional lengths, R Q
P
. 8(t  2) ¬2(t  1)
B
10. In order to show D A
E
, we must show that 8t  16 ¬2t  2
ED AB 6t  16 ¬2
DC  BC .
   
6t ¬18
ED 8 2 t ¬3
DC  20 or 5

AB 12 E
16. D B
C
, so by the Triangle Proportionality

BC 
25 D
B EC
Theorem, AD  AE . Substitute the known
ED AB  
  , so the sides do not have proportional
DC BC measures.
B
lengths and D is not parallel to A
E
. 24 18

AD ¬
3
11. To find x:
24(3) ¬18(AD)
20  5x ¬2x  6
72 ¬18(AD)
20 ¬7x  6
4 ¬AD
14 ¬7x
2 ¬x

169 Chapter 6
B
17. E D
C
, so by the Triangle Proportionality 6(x  4) ¬12(8)
ED BC 6x  24 ¬96
AE  AB . Substitute the known
Theorem,   
6x ¬120
measures.
x ¬20
2x  3 6
 3 ¬2 J
21. In order to have G F
K
, it must be true that
2(2x  3) ¬3(6) HG HJ
GF  JK .
 
4x  6 ¬18
4x ¬24 x 4 x  5
 18 ¬ 15
x ¬6
15(x  4) ¬18(x  5)
ED  2x  3 15x  60 ¬18x  90
 2(6)  3 or 9 3x  60 ¬90
18.  E
BC D, so by the Triangle Proportionality 3x ¬30
BE CD x ¬10
Theorem,  AB  AC . Substitute the known
  
measures. J
22. In order to have G F K
, it must be true that
20 x 
5 HG HJ
16 ¬ x  3
  
GF  JK .
 
20(x  3) ¬16(x  5) x  3.5 x  8.5

21  (x 3.5) ¬7  (x 
 8.5)
20x  60 ¬16x  80 x 3.5 x 8.5
4x  60 ¬80

17.5  x ¬15.5  x
4x ¬140 (x  3.5)(15.5  x) ¬(17.5  x)(x  8.5)
x ¬35 15.5x  x2  54.25  3.5x ¬17.5x  148.75
AC  x  3 ¬  x2  8.5x
 35  3 or 32 x  12x  54.25 ¬x2  26x  148.75
2
CD  x  5 12x  54.25 ¬26x  148.75
 35  5 or 40 54.25 ¬14x  148.75
19.  C
BF E and AC DF
, so by the Triangle 203 ¬14x
BC FE 14.5 ¬x
Proportionality Theorem, AB  AF and
  
A F C D T
23. In order to show Q R
S
, we must show that
FE  DE . Substitute the known measures.
  
PQ PT
R 

Q
10 
x x 
3 TS .
6 ¬ 8 PQ 9
R ¬

Q 30  9
 10
8x ¬6 x  3
  ¬9
21 or 3
7
8x ¬6x  20 PT 12
2x ¬20  ¬ 
TS 18  12
x ¬10 12
¬ 
6 or 2
y PQ PT
8 ¬ 
R 
Because   T  is not parallel to R
S
.
 1
x  3
0 2y  3 Q TS , Q
8 ¬
y
 T
24. In order to show Q R
S
, we must show that
 10 2y  3
10  3 PQ PT
R 

Q

y TS .
8 ¬

40 2y  3 PQ 65  22
R ¬

Q

3 22
40 43
8(2y  3) ¬ 
3 y ¬
22
40 Let TS  x. Then SP  3x and PT  3x  x or 2x.
16y  24 ¬ 
3 y
24 ¬8  PT x
2
TS  x  2
3y  
9 ¬y PQ PT
BC  x
Because 
QR  T
TS , Q is not parallel to R
S
.
 10 T
25. In order to show Q R
S
, we must show that
10
FE ¬x   PQ PT

QR 
3
10 TS .
¬10   1
3 or 13 3 Let RQ  x. Then PQ  2x.
CD  y x

PQ 1
9 
Q
2
R ¬ x or 2

DE  2y  3 PT 12.9  8.6
TS ¬
 2(9)  3 or 15    
8.6
J F
K
, it must be true that 4.3 1
20. In order to have G ¬
8.6 or 2
HG H
J
   . PQ PT
GF JK
R   1
6 8
Thus, 
Q TS  2 . Since the sides have
 ¬ x  4
12 T
proportional lengths, Q R
S
.

Chapter 6 170
T
26. In order to show Q R
S
, we must show that 30. Use the Distance Formula to find DE and AB.
PQ PT

QR  
TS . y
PQ 34.
88 43
6 A 8

QR ¬ 
18.32 or 229
PT 33.25  11.45 4
TS ¬
  
11.45
21
.8 43
6 x
¬
11.4 5 or 229 8 4 4 8
PQ PT 43
6
Thus, 
QR  TS  229 . Since the sides have
  B 4
T
proportional lengths, Q R
S
. C
8
27. 
DE is a midsegment of ABC and D
E
 BC
, so by
the Triangle Midsegment Theorem, DE  1

2 BC.
Then BC  2DE.

32  3
  4 2
2 2
DE ¬ 
 1
DE ¬ (4  1)2  (3
 1)2
¬ 94 
¬ 9 81
4  4
  
¬13
¬3
 
2 10
BC ¬2DE
¬213 AB ¬ [4  (1)]
2  (
3  6)2
¬ 4  13 or 52
 ¬9  81
M
28. If the slopes of W and T
S
 are equal, W
M
 T
S
. ¬310

12 
slope of WM   14 So, 3
10
  1
 ) and thus DE  1
2 (310

2 AB.
5  3 or 1
2
20 26 31.
slope of TS  
17  11 or 1 y
A (2, 12)
M
Because the slopes of W  and T
S
 are equal, C
M
W  T S.
M
W  is a midsegment of RST if W is the
T
midpoint of R and M is the midpoint of RS.
1  11 8  26
T
The midpoint of R is  
2, 2  (5, 17). 
These are not the coordinates of W.
1  17 8  20
S
The midpoint of R is  
2, 2  (8, 14). 
D B (5, 0) x
These are not the coordinates of M.
M
W  is not a midsegment because W and M are Graph 
AB. We can find segments of  AB with
not midpoints of their respective sides. lengths in a ratio of 2 to 1 by considering a second
29. 
DE is a midsegment of ABC. Use the Midpoint line and parallel lines that intersect this line and
Formula to find the coordinates of D and E. .
AB
Graph C(0, 12) and D(0, 0) and lines  CA and 
DB.
y  
CA and DB are horizontal lines and are parallel
A lines intersecting transversals CD  (the y-axis) and
8
. We can find P by finding a third parallel line
AB
4 intersecting 
CD and  AB so that this line

separates CD into two parts with a ratio of 2 to 1.
x
8 4 4 8 CD is 12 units, so if a horizontal line intersects
 at (0, 4) or (0, 8) then this line separates CD
CD 
B 4 into two parts with a ratio of 2 to 1. These
C horizontal lines intersect AB at (4, 4) and (3, 8).
8  into parts with a ratio of
These points cut off AB
2 to 1, so P could have coordinates (4, 4) or (3, 8).
1  6  (5)

D 
2 ,
7 
  1
2  D 3, 2
4  3  (5)
E 
2 ,
7 
2  E2, 4
3

E
Find the slopes of D and A
B
.
1
4  
E
slope of D 
3
2 or 3
3 
2
 36
B
slope of A
4 
(1) or 3
E
Both D  and A
B
 have slope 3, so D
E
 is parallel
B
to A.

171 Chapter 6
32. y N To find y:
2y  6 ¬3y  9
P
6 ¬y  9
15 ¬y
34. To find x:
2x  3 ¬6  x
R 3x  3 ¬6
3x ¬3
x x ¬1
To find y:
4y  1 ¬2y
3
1 ¬2
3y
3
 ¬y
2
35. The poles form parallel line segments and the
wires are transversals cutting through the ends of
the parallel segments.
L is on 
PN and M is on 
RN so graph N, P, and R D
and extend PN
 and R
N so that P
R
 divides N
L

LP 2
N
and M. 
PN  1 , and P
R
 divides N
L
 and M
N

MR 2
proportionally so RN  1 . Then LP  2(PN)
and MR  2(RN). Starting at N(8, 20), move to A
P(11, 16) by moving down 4 units and then right
50 ft
3 units. Locate L by moving from P down 8 units
C
and then right 6 units. The coordinates of L are
(17, 8). Now starting at N(8, 20), move to R(3, 8) 30 ft
by moving down 12 units and then left 5 units. a ft
Locate M by moving from R down 24 units and
then left 10 units. The coordinates of M are F
(7, 16). B E
x ft
Verify that LP  2(PN) and MR  2(RN). 40 ft
PN ¬ (11 8)2 (16  20)2
¬ 9  16
CFE is a right angle, so CFE  ABE and
¬25 or 5
CFB  DEB. Also, CEF  AEB, so
LP ¬(11  17)2 
(16 
8)2 AEB CEF by AA Similarity. CBF  DBE,
 36  64
 so CBF DBE by AA Similarity.
 100
 or 10 x a
0 ¬

So, LP  2(PN). 4 3
0
30x ¬40a
RN  
(8  3)
2  (2
0  8)2 30
40 x ¬a
 
25  144
 
 169
 or 13 40 
a ¬  x
50 40
MR   [3  (
7)]2 
 [8  2
(16)] 40a ¬50(40  x)
 100 
 
576 30
40 x ¬2000  50x
40 
 676
 or 26
So, MR  2(RN). 30x ¬2000  50x
80x ¬2000
33. To find x:
x ¬25
The sides of the large triangle are cut in equal
So, the distance from C to the taller pole is
parts by the segment whose length is labeled
25 feet.
x  2, so this segment is a midsegment and its
x 
length is half the length of the segment whose 36. a ¬ 40

30
length is labeled 5 30x ¬40a
3 x  11.

30(25) ¬40a
1
5
x  2 ¬2 3x  11  750 ¬40a
x  2 ¬5 11 18.75 ¬a
6x  2
  
The coupling is 18.75 feet above the ground.
1x  2 ¬ 11
6 2
1x ¬7
6 2
x ¬21

Chapter 6 172
37. Let y represent the length of the wire from the B
39. Given: D is the midpoint of A. E is the midpoint
smaller pole. C
of A.
302  402 ¬y2 E
Prove: D  B ; DE  1
C 
2 BC
900  1600 ¬y2
A
2500 ¬y2 E
D
50 ¬y
Let z represent the length of the wire from the B C
top of the smaller pole to the coupling. Proof:
40  Statements Reasons
z ¬ 25
50  z 25
25z ¬15(50  z) B
1. D is the midpoint of A . 1. Given
25z ¬750  15z C
E is the midpoint of A.
40z ¬750 D
2. A D B, A
E
E C 2. Midpoint
z ¬18.75 Theorem
The coupling is 18.75 feet down the wire from the 3. AD  DB, AE  EC 3. Def. of 
top of the smaller pole. segments
DB EC
AD  AE
38. Given:  4. AB  AD  DB,
  4. Segment
E
Prove: D BC
 AC  AE  EC Addition
Postulate
A
E 5. AB  AD  AD, 5. Substitution
D
AC  AE  AE
B C
Proof: 6. AB  2AD, AC  2AE 6. Substitution
Statements Reasons AB
7.  AC
  2,  2 7. Division Prop.
AD AE
1. 
DB EC 1. Given
AD  AE
  AB AC
8.    8. Transitive
AD D
B AE EC AD AE
Prop.
AD  AD  AE  AE
2.       2. Addition Prop.
9. A  A 9. Reflexive Prop.
AD 
DB AE 
EC
3.  AD  AE 3. Substitution
10. ADE ABC 10. SAS Similarity
4. AB  AD  DB, 4. Segment Addition
11. ADE  ABC 11. Def. of
AC  AE  EC Postulate
polygons
AB
5.  AC
 5. Substitution
AD AE E
12. D B
C
 12. If corr.  are ,
then the lines
6. A  A 6. Reflexive Prop.
are parallel.
7. ADE ABC 7. SAS Similarity BC AB 13. Def. of
DE  AD
13.   
8. ADE  ABC 8. Def. of polygons polygons
BC 14. Substitution
E
9. D B
C
 9. If corr.  are ,
14. 
DE 2
Prop.
then the lines
15. 2DE  BC 15. Mult. Prop.
are .
16. DE  1

2 BC
16. Division Prop.

40. Figure is not shown actual size.


A B C D

41. A B C D E

42. A B C

173 Chapter 6
a b
43. The total length of the lots along Lake Creek 47.  2 ¬18
Drive is 20  22  25  18  28 or 113 meters. a  b ¬36
The lines dividing the lots are perpendicular to a ¬36  b
Lake Creek Drive, so they are all parallel. Write a ¬5
proportions to solve for each variable. b 4
36 
b ¬5
b
total lake frontage 4

total length of lots along street 4(36  b) ¬5b
individual lake frontage 144  4b ¬5b
  144 ¬9b
individual length along street
135
.6 16 ¬b
u
113 ¬ 20

a  36  b
135.6(20) ¬113u  36  16 or 20
2712 ¬113u a  b  20  16 or 4
24 ¬u
48a. See students’ work. EF GH, F
G E H
, E
F
G H,
135
.6 w G
E H

113 ¬ 22
 F
135.6(22) ¬113w 48b. No, there will be an odd number of sides so it is
2983.2 ¬113w not possible to pair opposite sides.
26.4 ¬w
135
.6 x
113 ¬ 25
 Page 315 Maintain Your Skills
135.6(25) ¬113x 49. Yes, by AA Similarity; the parallel lines determine
3390 ¬113x congruent corresponding angles.
30 ¬x  9 
50. Yes, by SSS Similarity; 6 12
 3
y 8  12  16  4
135
.6
113 ¬ 18
  51. No; corresponding angles are not congruent. The
135.6(18) ¬113y third angles have measure 180  (72  66)  42
2440.8 ¬113y and 180  (66  38)  76.
21.6 ¬y x 7
52. 0 ¬
2 14
135
.6 z
113 ¬ 28
 14x ¬20(7)
135.6(28) ¬113z 14x ¬140
3796.8 ¬113z x ¬10
33.6 ¬z y 14
9 ¬
7
44. Use the Triangle Proportionality Theorem in 7y ¬126
DCA. y ¬18
AB DG DG DE
G
B A D
, so 
BC

GC . Also, in DCF, GC  EF .
  x 14
53. 
18 ¬

Using the Transitive Property of Equality, 21
AB DE 21x ¬252
BC  EF .
  
x ¬12
y 14
9 ¬
F 21
E 21y ¬126
D
y ¬6
G 54. If one side of a triangle is longer than another
A
B side, then the angle opposite the longer side has a
C greater measure than the angle opposite the
shorter side. In ADB, ADB is opposite a side
45. Sample answer: City planners use maps in their whose measure is 15, and ABD is opposite a
work. Answers should include the following. side whose measure is 12. 15  12, so
• City planners need to know geometry facts mADB  mABD.
when developing zoning laws. 55. If one side of a triangle is longer than another
• A city planner would need to know that the side, then the angle opposite the longer side has a
shortest distance between two parallel lines is greater measure than the angle opposite the
the perpendicular distance. shorter side. In ADB, ABD is opposite a side
12
 x
18 ¬ 42  x
46. B;  whose measure is 12, and BAD is opposite a
12(42  x) ¬18x side whose measure is 9. 12  9, so
504  12x ¬18x mABD  mBAD.
504 ¬30x
16.8 ¬x

Chapter 6 174
56. If one side of a triangle is longer than another 4. Let x represent the perimeter of WZX.
side, then the angle opposite the longer side has a WX perimeter of WZX
ST ¬ 
greater measure than the angle opposite the 
perimeter of SRT
shorter side. In BDC, BCD is opposite a side
5 ¬x
1
whose measure is 9, and CDB is opposite a side 6 5
whose measure is 13. 9  13, so 75 ¬6x
mBCD  mCDB. 12.5 ¬x
57. If one side of a triangle is longer than another The perimeter of WZX is 12.5 units.
side, then the angle opposite the longer side has a x 6.
5
greater measure than the angle opposite the
5. 
12 ¬
13
shorter side. In BDC, CBD is opposite a side 13x ¬78
whose measure is 10, and BCD is opposite a x ¬6
side whose measure is 9. 10  9, so 20 16
mCBD  mBCD.
6. x ¬
12
58. There are 6 equilateral triangles. 240 ¬16x
15 ¬x
59. There are 18 obtuse triangles in the figure, 3 in
x 
each equilateral triangle. 7. 9 ¬ 18

24
60. True; the hypothesis is false, so we cannot say 24x ¬162
that the statement is false. x ¬6.75
61. False; the hypothesis is true, but the conclusion is AB m
8. Given: ABC DEF and  E  n
D
false.
perimeter of ABC m
62. True; the hypothesis is false, so we cannot say Prove:    n
perimeter of DEF
that the statement is false.
63. True; the hypothesis is true and the conclusion is B
true. A
64. A D, B  E, C  F, A BD E,
C E
C
B E F
, A
CD F
D
65. R  X, S  Y, T  Z, R SX Y,
F
T
S Y Z
, R
T
X Z
 AB BC AC
Proof: Because ABC DEF, 
DE 
EF  DF .
 
66. P  K, Q  L, R  M, P QK L,
BC AC m
EF  DF  n . Cross products yield
So  
R
Q L M
, PR
K M

AB  DE m   m   m
n , BC  EF n , and AC  DF n .
  
Using substitution, the perimeter of
6-5 Parts of Similar Triangles ABC  DE m  
m  
m
n  EF n  DF n , or
  
m(DE  EF  DF). The ratio of the two
n
Page 319 Check for Understanding perimeters
1. ABC MNQ and A D and M R are altitudes, m(DE  EF  DF)
n
  m
angle bisectors, or medians. DE  EF  DF or n .
2. Sample answer: The perimeters are in the same height of Tamika distance from camera
24 9.  ¬ 
proportion as the side measures, which is   2 height of image length of camera
36 or 3 .
165
 ¬x
So if the smaller triangle has side lengths 6, 8, 5 10
and 10 (so that its perimeter is 6  8  10  24), 1650 ¬5x
then the larger triangle has side lengths 330 ¬x
3 (6), 3(8), and 3 (10) or 9, 12, and 15. Tamika should be 330 cm or 3.3 m from the
2 2 2
3. Let x represent the perimeter of DEF. The camera.
perimeter of ABC  5  6  7 or 18.
DE perimeter of DEF
AB ¬ 
 Pages 320–322 Practice and Apply
perimeter of ABC
10. Let x represent the perimeter of BCD. The
3 ¬x
1
5 8 perimeter of FDE  4  5  8 or 17.
54 ¬5x CD perimeter of BCD
DE ¬ 
 
10.8 ¬x perimeter of FDE
The perimeter of DEF is 10.8 units. 12 x

8 ¬17
204 ¬8x
25.5 ¬x
The perimeter of BCD is 25.5 units.

175 Chapter 6
11. Let x represent the perimeter of ADF. The 18. Let x represent EG.
perimeter of BCE  24  12  18 or 54. AD AC
 ¬ 
perimeter of ADF EH EG
DF
CE ¬ 
 15 1 7
perimeter of BCE  ¬
7.5 x
21
 x 15x ¬127.5
18 ¬ 54

x ¬8.5
1134 ¬18x
Thus, EG  8.5.
63 ¬x
The perimeter of ADF is 63 units. 19. Let x represent EH.
BG B
C
EH  EF
12. Let x represent the perimeter of CBH. The   
perimeter of FEH  11  6  10 or 27. ADEG is 4 
3
x  
2
D
a parallelogram, so A GE and BCH  HFE. 12
H
so C corresponds to FH
. 3 6
x  3
CH perimeter of CBH
FH ¬ 
 9  6x
perimeter of FEH
7 x 3  x

10 ¬
2
2
7
189 ¬10x Thus, EH  3

2.
18.9 ¬x FB
 FG
 ¬ 
20. SA ST
The perimeter of CBH is 18.9 units.
7  x
 ¬x5
13. Let x represent the perimeter of DEF. 2
DF perimeter of DEF 7x  x2 ¬10
FC ¬ 
 0 ¬x2  7x  10
perimeter of CBF
6 ¬
x 0 ¬(x  5)(x  2)
8 27 x  5 ¬0 or x  2  0
162 ¬8x x ¬5 x2
20.25 ¬x We must choose x  5 because otherwise FB  5
The perimeter of DEF is 20.25 units. and so FB  FG, but the hypotenuse must be
14. Let x represent the perimeter of ABC. The longer than either leg. Thus, x  5 and FB  2.
perimeter of CBD  3  4  5 or 12. DG D
C
JM  JL
perimeter of ABC 21.   
CB
B ¬ perimeter of CBD
 
D 6x
2
x  4
5 ¬x
3 12 8  6x  x2
60 ¬3x x2  6x  8  0
20 ¬x (x  4)(x  2)  0
The perimeter of ABC is 20 units. x40 or x20
15. Let x represent the perimeter of ABC. CBD is x4 x2
a right triangle, so use the Pythagorean Theorem We must choose x  2 because otherwise JM  4
to find BD. and so JM  JL, but the hypotenuse must be
(CD)2  (BD)2  (CB)2 longer than either leg. Thus, x  2 and DC  4.
122  (BD)2  31.22 12 x 5
32 ¬ 2x  3
22.  
144  (BD)2  973.44
(BD)2  829.44 24x  36 ¬32x  160
BD  28.8 36 ¬8x  160
The perimeter of CBD  31.2  28.8  12 or 72. 124 ¬8x
AB

perimeter of ABC
 ¬  15.5 ¬x
CB perimeter of CBD
11 20  x
23. 14 ¬ x
52 8.8 x   
 .2 ¬
31 7
2 11x ¬280  14x
33.8(72) ¬31.2x 25x ¬280
2433.6 ¬31.2x
78 ¬x x ¬11 1

5
The perimeter of ABC is 78 units. x 
3 x
24.  6  4
16. The original picture is similar to the enlarged
4x  12  6x
picture, so the dimensions and perimeters are
12  2x
proportional. The perimeter of the original picture
6x
is 2(18)  2(24) or 84 cm. The perimeter of the
enlarged picture is 0.30(84)  84 or 109.2 cm. The x 9
25. 8  2x
enlarged picture will take approximately 109.2 cm
2x2  72
of cord, so 110 cm will be enough.
x2  36
17. Yes, the perimeters are in the same proportion as x6
300
the sides,   1
600 or 2 . x represents a length, which must be positive. So,
x  6.

Chapter 6 176
26. 
TA and W
B
 are medians, so RA  AS and 31. Given: ABC RST, A D is a median of ABC.
UB  BV. Then RS  2(RA) and UV  2(UB). U
R is a median of RST.
TA RS AD AB
WB UV Prove: 
RU
RS
8  2(3)

3x  6  2(x 
2) A R
8 6
 
3x  6   
2x  4
16x  32  18x  36 T S
C B U
32  2x  36 D
68  2x Proof:
34  x Statements Reasons
UB  x  2
 34  2 or 36 1. ABC RST 1. Given
D
A is a median of
F
27. B bisects ABC, so by the Angle Bisector
ABC.
AF BA
Theorem, 
C F 
BC . Let x represent CF. Then U
R  is a median of
AF ¬9  x. RST.
9 x 6 2. CD  DB; TU  US
 x ¬
7.5
2. Def. of median
67.5  7.5x ¬6x AB CB 3. Def. of polygons
RS  TS
3.   
67.5 ¬13.5x
5 ¬x 4. CB  CD  DB; 4. Segment Addition
Thus, CF  5. TS  TU  US Postulate
C
A ED
, so BED  BFC. FBC  EBD, so AB CD DB
RS  TU  US
5.   5. Substitution
by AA Similarity, EBD FBC. Let y represent
AB DB DB B)
2(D
RS  US  US or 2(US)
6. 
BD.    6. Substitution
BD ED
C ¬
B  AB D
B
RS  US
FC 7.   7. Substitution
y 9
  ¬
7.5 5 8. B  S 8. Def. of polygons
5y ¬67.5
y ¬13.5 9. ABD RSU 9. SAS Similarity
Thus, BD  13.5 AD AB
RU  RS
10.  10. Def. of polygons
  
height of person distance from camera
28. 
height of image
¬  length of camera 32. Given: 
CD bisects ACB. By construction
x 7  12 E
A CD.
2 ¬

1

15 AD AC
Prove:    
B
15x ¬1008 DB C
x ¬67.2 E
The person is 67.2 inches or about 5 feet 7 inches C
tall.
29. xy  z2; ACD CBD by AA Similarity. Thus, 3 1 2
CD AD z x
BD  CD or y  z . The cross products yield xy  z .
   2
A D B
30. Given: ABC PQR Proof:
BD BA Statements Reasons
Prove: 
QS 
QP

1. 
CD bisects ACB. 1. Given
Q
B By construction,
E
A CD.
AD EC 2. Triangle Propor-
A
D
C P
S
R 2. 
DB 
BC

tionality Theorem
Proof: Since ABC PQR, A  P. BDA  3. 1  2 3. Definition of Angle
QSP because they are both right angles created Bisector
by the altitude drawn to the opposite side and all
4. Alternate Interior
right angles are congruent. Thus ABD PQS 4. 3  1
BD BA
Angle Theorem
by AA Similarity and QS  QP by the definition
  
5. 2  E 5. Corresponding
of similar polygons.
Angle Postulate
6. 3  E 6. Transitive Prop.
C
7. EA
C
 7. Isosceles  Th.
8. EC  AC 8. Def. of congruent
segments
AD AC
9. B 
D BC 9. Substitution

177 Chapter 6
33. Given: ABC PQR, B D is an altitude of 36. Given: 
RU bisects SRT.
ABC. Q
S
 is an altitude of PQR. U
V R T
QP QS SV SR
   Prove: 
Prove: BA BD VR RT
B Q S
V U

A C P R T
D R
S Proof:
Proof: A  P because of the definition of Statements Reasons
D
similar polygons. Since B and Q
S
 are
1. 
RU bisects SRT. 1. Given
C
perpendicular to A  and PR
, BDA  QSP. So,
QP QS U
V R T
ABD PQS by AA Similarity and BA  
BD
2. S  S 2. Reflexive Prop.
by definition of similar polygons.
34. Given: C  BDA 3. SUV  STR 3. Corresponding 
AC AD Postulate
Prove: 
DA 
BA

4. SUV STR 4. AA Similarity
C
B SV SR 5. Def. of s
VU  RT
5.   
A
D
6. URT  VUR 6. Alternate Interior
C BDA  Theorem
Given AC
— AD
— 7. VRU  URT 7. Def. of  bisector
ADB ACD DA BA
A A AA Similarity Def. of
polygons 8. VUR  VRU 8. Transitive Prop.
Reflexive Prop.
U
9. VV
R
 9. If 2  of a  are
35. Given:  bisects EFG.
JF , the sides opp.
H
E  FG
, EF
 H
G these  are .
EK G
J
KF  JF
Prove:   10. VU  VR 10. Def. of  segments
J SV SR

11. 
VR RT 11. Substitution

E K H 37. Given: RST ABC, W and D are midpoints of


S
T and C
B
, respectively.
F G
Prove: RWS ADB
Proof:
A
Statements Reasons R
1.  bisects EFG.
JF 1. Given B C
D
H
E  FG
, E
F H
G

2. EFK  KFG 2. Def. of  bisector S T
W
3. KFG  JKH 3. Corresponding 
Postulate Proof:
4. JKH  EKF 4. Vertical  are .
RST ABC S B
5. EFK  EKF 5. Transitive Prop. Given Def. of polygons
6. FJH  EFK 6. Alternate Interior 
Theorem RS TS W and D are
— —
7. FJH  EKF 7. Transitive Prop. AB CB midpoints.
Def. of polygons
8. EKF GJF 8. AA Similarity Given
EK G
J
KF  JF
9.  9. Def. of s
  RS
— 2WS
— 2WS TS
AB 2BD
2BD CB
Substitution
Def. of midpoint
RS WS

AB

BD RWS ADB
Def. of Division SAS Similarity

Chapter 6 178
38. Sample answer: The geometry occurs inside the PQ  2x  1
camera as the image is formed on the film.  2(7)  1 or 15
Answers should include the following. 46. The line goes through (3, 0) and (0, 3).
• 3 0
lens m 0  3 or 1
film object
y  mx  b
y  1x  (3)
• The triangles are similar because the SAS
yx3
Similarity Theorem holds. The congruent angles
are the vertical angles and the corresponding sides 47. y  y1  m(x  x1)
are the congruent sides of the isosceles triangles. y  (1)  2[x  (1)]
y  1  2x  2
39. Let x represent DF.
y  2x  1
AC
 AB
DF ¬ 
DE 48. 5 12 19 26 33
10
 .5 6.5    
x ¬ 8 7 7 7 7

84¬ 6.5x The numbers increase by 7.


12.9¬ x The next number will increase by 7. So, it will be
40. B; let x be one of the two numbers that are the 33  7 or 40. The next number will also increase
same. Then x  x  3x  180, so 5x  180 or by 7. So, it will be 40  7 or 47.
x  36. So, the numbers are 36, 36, and 108. The 49. 10 20 40
   
80 160
answer is B. 2 2 2 2
The numbers are multiplied by 2.
The next number will be multiplied by 2. So, it
Page 323 Maintain Your Skills will be 160 2 or 320. The next number will also
41. LO  LM  MO be multiplied by 2. So, it will be 320 2 or 640.
14  7  MO 50. 0 5 4 9 8 13
  
7  MO 5 1 5 1 5
LP  LN  NP The pattern is to add 5 then subtract 1.
16  9  NP The last number was obtained by adding 5, so the
7  NP next number is obtained by subtracting 1. So, it
N
In order to show M O
P
, we must show that will be 13  1 or 12. The next number is obtained
LM LN LM by adding 5, so it will be 12  5 or 17.
 7 LN
 9
MO  NP . MO  7 or 1. NP  7 . Since
    
LM LN
O  N


M

NP , the sides are not proportional and M
P
is not parallel to O. Page 323 Practice Quiz 2
DB EC
E
1. D B
C
, so 
AD  AE .
 
42. There is not enough information given to
N is parallel to O
P
. DB 18
determine whether M   
8 12
N
43. In order to show M O P, we must show that 12(DB)  144
LM LN LM 1
5 LN 12 DB  12
O 
NP . MO  5 or 3. NP  4 or 3. Thus,

M
       
LM L N AB  AD  DB
O 

M

NP . Since the sides have proportional  8  12 or 20
N
lengths, M O
P
. 2. AB  AD  DB
Z
44. V Y
X
, so Z  Y and V  X. So, VZW 20  4  DB
VW
XYW. Then  ZW 16  DB
XW  YW .
  
DB EC
3x  6 
DE BC, so 
AD  AE .
 
6
x4  5
 
16 m 4
4  m2
15x  30  6x  24   
9x  30  24 16m  32  4m  16
9x  54 12m  32  16
x6 12m  48
VW  3x  6 m4
 3(6)  6 or 12 3. XY  XV  VY
WX  x  4 30  9  VY
 6  4 or 10 21  VY
S
45. R Q
T
, so R  PQT and S  QTP. So, XZ  XW  WZ
PT
PQT PRS. Then   .
PQ 18  12  WZ
PS PR 6  WZ
 10
 2x 
 1
10  4 2x  1 6 Z
In order to show Y V
W
, we must show that
10 2x  1
14  2x  7
   XV XW XV
 9 3 
XW 1
2
VY  WZ . VY  21 or 7 . WZ  6 or 2. Thus,
      
20x  70  28x  14 XV XW
VY WZ so the sides are not proportional. Y
Z
 is
   
70  8x  14
56  8x W
not parallel to V.
7x

179 Chapter 6
4. XZ  XW  WZ Stage 4:
33.25  XW  11.45
21.8  XW
Z
In order to show Y V
W
, we must show that
XV XW XV 34.
88 43
6  XW 21.8 43
6
VY  WZ . VY  18.32 or 229 . WZ  11.45 or 229 .
          
XV XW 43
6
VY  WZ  229 . Since the sides have
Thus,     
Z
proportional lengths, Y VW
.
5. Let x represent the perimeter of DEF. The
perimeter of GFH  2  2.5  4 or 8.5.
EF perimeter of DEF

FH ¬ 
perimeter of GFH
6 ¬ x
4 8.5
51 ¬4x
12.75 ¬x
6. Let x represent the perimeter of RUW. The There are 81 nonshaded triangles at Stage 4.
perimeter of STV  12  18  24 or 54.
perimeter of RUW 2. Stage 0: 16  16  16 or 48 units
UW
VT  perimeter of STV
   Stage 1: 8  8  8 or 24 units
21
 x Stage 2: 4  4  4 or 12 units
18  54

Stage 3: 2  2  2 or 6 units
1134  18x Stage 4: 1  1  1 or 3 units
63  x
3. The perimeter will be divided by 2 from stage to
18  x 10
7.  x ¬
14 stage, getting smaller and smaller and
252  14x ¬10x approaching zero.
252 ¬24x 4. Yes, DFM is equilateral with 2 units on each
10.5 ¬x side. Yes, BCE, GHL, and IJN are
4 6 equilateral with 1 unit on each side.
8. x  
x 3
5. yes, by AA Similarity because each triangle has
4x  12  6x
all angles of measure 60
12  2x
6x 6. 3, each sharing a side with the shaded triangle in
Stage 1.
9. 25
x 10
¬x 7.
2x2 ¬50
x2 ¬25
x ¬5
Because x represents length, x cannot be negative.
So, x  5.
10. Let x represent the longest side of the second
garden.
53
.5 25 8. Three copies of the Stage 4 Sierpinski triangle
32.1 ¬ x
  
combine to form a Stage 5 triangle (just as three
53.5x ¬802.5 copies of a Stage 2 triangle combine to form a
x ¬15 Stage 3 triangle).
The longest side of the second garden is 15 feet.
9. 9 copies: 3 to make a Stage 5 triangle, and
3 copies of the Stage 5 triangle, so a total of 3  3
or 9 Stage 4 triangles
Page 324 Geometry Activity: Sierpinski Triangle
1. Stage 3:

6-6 Fractals and Self-Similarity

Page 328 Check for Understanding


1. Sample answer: irregular shape formed by
iteration of self-similar shapes
2. They can accurately calculate thousands of
iterations.
3. Sample answer: icebergs, ferns, leaf veins

Chapter 6 180
4. Stage 1: 2, Stage 2: 6, Stage 3: 14, Stage 4: 30 14. Stage Number of Dots Pattern
1 1 10
2 3 21
3 6 33
4 10 46
The formula is the stage number plus the number
of dots in the previous stage: An  n  An  1.
So, in the seventh stage, the number of dots is
Stage 3 Stage 4
A7  7  A6, where A6  6  A5 and A5  5  A4 
5. Stage Number of Branches Pattern 5  10 or 15. Then A6  6  15 or 21 and A7  7 
1 2 2(21  1) 21 or 28. So there are 28 dots.
2 6 2(22  1) 15. 1, 3, 6, 10, 15…; Each difference is 1 more than
3 14 2(23  1) the preceding difference.
4 30 2(24  1)
16. The triangular numbers are the numbers in the
The formula is twice the difference of 2 to a power
diagonal.
that is the stage number and 1: An  2(2n  1).
17. The result is similar to a Stage 3 Sierpinski
6. No, the base of the tree or segment of a branch triangle.
without an end does not contain a replica of the
entire tree. 1
1 1
7. 2  ¬1.4142… 1 0 1
  ¬1.1892…
2 1 1 1 1
1 0 0 0 1
8.  2   1.0905…; the results are getting closer 1 1 0 0 1 1
to 1, so the result after 100 repeats approaches 1. 1 0 1 0 1 0 1
1 1 1 1 1 1 1 1
9. Yes, the procedure is repeated over and over
again. 18. It is similar to a Stage 1 Sierpinski triangle.
10. First, write an equation to find the balance after
one year. 1
current balance  (current balance interest rate) 1 1
 new balance 1 2 1
4000  (4000  0.011)  4044 1 0 0 1
4044  (4044  0.011)  4088.48 1 1 0 1 1
1 2 1 1 2 1
4088.48  (4088.48  0.011)  4133.45
4133.45  (4133.45  0.011)  4178.92 1 0 0 2 0 0 1
After compounding interest four times, Jamir will
1 1 0 2 2 0 1 1
have $4178.92 in the account.
19. All of the numbers in the outside diagonal of
Pascal’s triangle are 1, so the sum of 25 1’s is 25.
Pages 328–331 Practice and Apply 20. The second diagonal consists of the natural
11. 9 holes
numbers, 1, 2, 3, 4, 5,…, 50. The sum is
1  2  3  4    46  47  48  49  50
 50  (1  49)  (2  48)  (3  47)  
 (24  26)  25
 50  24(50)  25
 1275

12. 73 holes

13. Yes, any part contains the same figure as the


whole, 9 squares with the middle shaded.

181 Chapter 6
21. Given: ABC is equilateral. CD  1

3 CB and
n
CE  1
CA
3
28. P  3 4
  
3 ; as the stages increase, the
perimeter increases and approaches infinity.
Prove: CED CAB
29. The original triangle and the new triangles are
A equilateral and thus, all of the angles are equal to
60. By AA Similarity, the triangles are similar.
30. x 12 3.46410… 1.8612… 1.3642… 1.168…
E
x
 3.46410… 1.8612… 1.3642… 1.168… 1.0807…
B C
D The numbers converge to 1.
Proof: 31. x 5 0.2 5 0.2 5
Statements Reasons
1
x 0.2 5 0.2 5 0.2
1. ABC is 1. Given
equilateral. The numbers alternate between 0.2 and 5.
CD  1
3 CB
32. x 0.3 0.6694… 0.8747… 0.9563… 0.9852…
1
CE  3CA 1
0.6694… 0.8747… 0.9563… 0.9852… 0.9950…
x3
C
2. AB
C
 2. Def. of equilateral  The numbers converge to 1.
3. AC  BC 3. Def. of  segments 33. x 0 1 2 4 16
4. 1 1
3 AC  3 CB
 4. Mult. Prop. 2x 1 2 4 16 65,536

5. CD  CE 5. Substitution The numbers approach positive infinity.


CD CE 34. x
CB  CB
6.    6. Division Prop. 1 3 7

CD CE 2x  1 3 7 15
CB  CA
7.    7. Substitution
35. x 5 0 5
8. C  C 8. Reflexive Prop.
9. CED CAB 9. SAS Similarity x5 0 5 10

22. 36. x 2 3 8
x2 1 3 8 63

37. x 4 6 24
3(2  x) 6 24 66

38. Write an equation to find the balance each month.

     
current current interest 100
balance
 balance rate  payment
23. Yes; the smaller and smaller details of the shape
 
new
have the same geometric characteristics as the
balance
original form.
1250  (1250 0.015)  100 ¬1168.75
24. Stage 1: 6 1168.75  (1168.75 0.015)  100 ¬1086.28
Stage 2: 36 1086.28  (1086.28 0.015)  100 ¬1002.57
25. Stage Number of Segments Pattern The balance after 3 months will be $1,002.57.
0 1 40 39. 0.200 0.64 0.201 0.6423…
1 4 41 0.64 0.9216… 0.6423… 0.9188…
2 16 42 0.9216… 0.2890… 0.9188… 0.2981…
3 64 43
0.2890… 0.8219… 0.2981… 0.8369…
The formula is 4 to the power of the stage
0.8219… 0.5854… 0.8369… 0.5458…
number: An  4n.
So in Stage 8 there are 48  65,536 segments. 0.5854… 0.9708… 0.5458… 0.9916…
26. Stage 0: 1 unit, Stage 1: 1
 1
3 unit, Stage 2: 9 unit,
0.9708… 0.1133… 0.9916… 0.0333…
1 0.1133… 0.4019… 0.0333… 0.1287…
Stage 3: 
27 unit; as the stages increase, the
0.4019… 0.9615… 0.1287… 0.4487…
length of the segments will approach zero.
0.9615… 0.1478… 0.4487… 0.9894…
27. Stage 0: 3 units, Stage 1: 3  4

3 or 4 units, Stage 2:
2 3
  
3 4
 4 4
3 3 3 3   or 5 1
  
4
3 units, Stage 3: 3 3 or Yes, the initial value affected the tenth value.
7 1
 40. A small difference in initial data can have a large
9 units effect in later data.

Chapter 6 182
41. Sample answer: The leaves in the tree and the Page 331 Maintain Your Skills
branches of the trees are self-similar. These self- 21 3x  6
14  x  4
48.   
similar shapes are repeated throughout the
painting. 21x  84  42x  84
42a. The flower and mountain are computer- 84  21x  84
generated; the feathers and moss are real. 168  21x
8x
42b. The fractals exhibit self-similarity and iteration.
x  16
17  20
49.  
43. See students’ work.
44. 20x  272
x  13 3

5
50. 36
x
 8x
3x2  48
x2  16
x  4
Reject x  4 because x represents length, which
is positive. So, x  4.
7 2x  1
51. x   
15
105  2x2  x
In Stage 1, the shaded triangle has legs 3 and
0  2x2  x  105
4 units. The hypotenuse has length c, where
(x  7)(2x  15)  0
c2  32  42, or c  9  16 or 5. So, the
x  7  0 or 2x  15  0
perimeter of the triangle is 3  4  5 or 12 units.
x7 2x  15
In Stage 2, there are three small shaded triangles
x  7.5
and the larger shaded triangle from Stage 1. Each
Reject x  7.5 because x represents length,
of the small shaded triangles has legs 2 and
which is positive. So, x  7.
1.5 units. The hypotenuse has length d, where
d2  22  1.52, or d  4  2.25
 or 2.5. So, the AK BL
JA  JB
52.   
perimeter of all shaded triangles in Stage 2 is 18 (JA) 9
3(2  1.5  2.5)  12 or 30 units.
 JA 
27  9
18
  (JA)
 9
45. Sample answer: Fractal geometry can be found in JA 18
the repeating patterns of nature. Answers should 324  18(JA)  9(JA)
include the following. 324  27(JA)
• Broccoli is an example of fractal geometry 12  JA
because the shape of the florets is repeated
JB AB
JL  KL
53. 
throughout; one floret looks the same as the   
stalk. 1
3 8

JL 
10
• Sample answer: Scientists can use fractals to
study the human body, rivers, and tributaries, 130  8(JL)
and to model how landscapes change over time. 16 1
4  JL

46. Suppose the 24-inch side of the larger triangle AK B
L
JA ¬ JB
54.   
corresponds to the smallest side of the smaller
10 1
4
25 ¬ JB
triangle. Let P represent the perimeter of the   
larger triangle.
24
10(JB) ¬350
 P
3  368
 JB ¬35
24(3  6  8)  3P 55. If one side of a triangle is longer than another
408  3P side, then the angle opposite the longer side has
136  P a greater measure than the angle opposite the
The maximum perimeter is 136 inches. shorter side. The sides of the triangle in order
47. C; Let x be the number of minutes the repair from least to greatest are 965, 1038, and 1042.
technician worked in excess of 30 minutes. So, the angles opposite these sides in the same
170 ¬80  2x order are arranged from least to greatest. That
90 ¬2x order is Miami, Bermuda, and San Juan.
45 ¬x 56. P  5s
The repair technician worked 30  45 or 60  5s
75 minutes. 12  s
Each side has measure 12 cm.

183 Chapter 6
4 x 16
57. P  2  2w 14. 3  x ¬ 25
  
54  2(2x  1)  2(x  2) 25(4  x) ¬16(3  x)
54  4x  2  2x  4 100  25x ¬48  16x
54  6x  6 100 ¬48  41x
48  6x 52 ¬41x
8x 52
41 ¬x
 
x  2  8  2 or 10
2x  1  2(8)  1 or 17 x  12 x 
7
15.  6 ¬4
The sides of the polygon have measures 10 feet,
4(x  12) ¬6(x  7)
10 feet, 17 feet, and 17 feet.
4x  48 ¬6x  42
58. P  n  2(n  2)  2n  7 48 ¬10x  42
57  n  2n  4  2n  7 6 ¬10x
57  5n  3 3 ¬x
60  5n 5
12  n number of total bases from hits
16. 
number of total at-bats
n  2  12  2 or 14 26
3
2n  7  2(12)  7 or 17  416
The sides of the polygon have measures 17, 14,  0.632
14, and 12 units. 17. Rewrite 2 : 7 as 2x : 7x and use those measures
as the lengths of the pieces of the board after
cutting it.
2x  7x  108
Chapter 6 Study Guide and Review 9x  108
x  12
2x  2(12) or 24
Page 332 Vocabulary and Concept Check 7x  7(12) or 84
1. true The two pieces have lengths 24 inches and
2. false, proportional 84 inches.
3. true 18. In similar polygons, corresponding sides are in
TU  6 or 2. But 
UV
4. false, sides proportion. 
VW 9 3 UV  1, so two of
5. false, iteration the sides are in a 2 : 3 ratio while two others are
6. false, one-half equal in length. Thus, the triangles are not
7. true similar.
8. true 19. The figures are rectangles, so all angles are
LK 24 3 LM 30 3
congruent.  16 or 2 and PQ  20 or 2 , so
    
9. false, parallel to RQ
the sides are in a 3 : 2 ratio. Opposite sides are
congruent, so all sides are in a 3 : 2 ratio. Thus,
the figures are similar.
Pages 332–336 Lesson-by-Lesson Review
3  x 20. ABCD AEFG
10. 4 12 AD AB
AG ¬ AE
3(12)  4x    
36  4x x x2
7.5 ¬

x  x  25
9x x  x 2
11. 7 28 x  7.5 ¬ x  3

3  z
  
x(x  3) ¬(x  7.5)(x  2)
7z  3(28) x2  3x ¬x2  2x  7.5x  15
7z  84 x2  3x ¬x2  5.5x  15
z  12 3x ¬5.5x  15
x 
2 14 2.5x ¬15
12.  5 ¬
10 x ¬6
10(x  2) ¬5(14)
AB  x  2
10x  20 ¬70
 6  2 or 4
10x ¬50
AG  x  7.5
x ¬5
 6  7.5 or 13.5
13. 3   7
y 
AD x 6 4
7 3 AG  x  7.5  13.5 or 9 .
The scale factor is 
3(y  3)  7(7)
3y  9  49
3y  58
58
y3


Chapter 6 184
21. 
PT SR, so P  R and T  S because they L
27. In order to show that G H
K
, we must show
are alternate interior angles. PQT  SQR IH IK
that  HG  KL .
  
because they are vertical angles. Thus, IH 2 1 3 IK 1
5
G  5
14 or 2 , and KL  9 or 3 .
PQT RQS. 
H     
PQ TQ Because the side lengths are not proportional,
  
RQ SQ L
G   H
K
.
6 x 3
6x  3x
 28. IL  IK  KL
(6  x)(3  x)  (6  x)(3) 36  28  KL
18  6x  3x  x2  18  3x 8  KL
18  3x  x2  18  3x In order to show that G L
 HK, we must show
IH IK
18  x2  18 that  
HG  
KL .
x2  0 IH 35 7 IK 28 7
x0

HG 
10 or 2 , and KL  8 or 2 .
    
IH IK
HG  KL , so GL H
K.
PQ  6  x   
 6  0 or 6 L
29. In order to show that G HK, we must show
QS  3  x IH IK
HG  KL . Let KL  x. Then IL  3x and
that  
 3  0 or 3
TQ
 3 3 IK  3x  x or 2x.
SQ  3  x  3  0 or 1.
The scale factor is 
IK x
2 IH 22 IK IH
KL  x or 2, and HG  11 or 2. KL  HG . Thus,
       
22. Triangles ABC and DFE are isosceles triangles.
A  D, and BA  CA and FD  ED so L
G H
K
.
BA C
A

FD  ED . Thus, ABC DFE by SAS 30. In order to show that GL H K
, we must show
IH IK
Similarity. that 
HG  
KL . Let HI  x. Then IG  3x, so
I J
23. H K
, so GHI  GJK and GIH  GKJ HG  3x  x or 2x.
because they are corresponding angles. Thus, IH x 1 IK 1
8
HG  2x or 2 , and KL  6 or 3. So the sides are
   
GHI GJK by AA Similarity.
24. mL  mQ  mLMQ ¬180 not proportional and G  H
L K
.
35  85  mLMQ ¬180 31. From the Triangle Proportionality Theorem,
mLMQ ¬60 BC ED
AB  AE .
   
LMQ  NMP, so mNMP  60.
Substitute the known measures.
mN  mP  mNMP  180
mN  40  60  180 4  
ED
9
6
mN  80 4(9)  6(ED)
LMQ is not similar to PMN because the 36  6(ED)
angles of the triangles are not congruent. 6  ED
25. Since A B
 D E, B  E and A  D because 32. From the Triangle Proportionality Theorem,
BC ED
AB  AE .
they are alternate interior angles. By AA    
Similarity, ABC DEC. Using the definition
BC AC Substitute the known measures.
EC  DC .
of similar polygons,     16  12 5
x3
 1
2 
AE
1
2  6

11x  4 5
6(x  3)  1(11x  2) 12 
AE
6x  18  11x  2 4(AE)  12(5)
18  5x  2 4(AE)  60
20  5x AE  15
4x E
33. Since B  C D
, ABE  ACD and AEB 
26. V  UST and T  T, so by AA Similarity, ADC by the corresponding angles postulate.
RVT UST. Using the definition of similar Then ABE ACD by the AA Similarity. Using
RT V
T
UT  ST .
polygons,    CD AD
BE  AE .
the definition of similar polygons,   
2x  4 3  x2
x2 
   Substitute the known measures.
4
2x  4 x5 CD 84

x 2  4 6  8

4(2x  4)  (x  2)(x  5) CD 12
8x  16  x2  5x  2x  10
6  8
8x  16  x2  7x  10 8(CD)  6(12)
16  x2  x  10 8(CD)  72
0  x2  x  6 CD  9
0  (x  3)(x  2)
x  3  0 or x  2  0
x3 x  2
Reject x  2 because otherwise UT  x  2
 2  2 or 0. So x  3.

185 Chapter 6
E
34. Since B  C D
, ABE  ACD and AEB  39. Stage 2 is not similar to Stage 1.
ADC by the corresponding angles postulate.
Then ABE ACD by the AA Similarity. Using
AC CD

the definition of similar polygons, 
AB 
BE .
Substitute the known measures. 40.
33 BC 32 x 2 4 60
 33  
24 x3 4 4 60 215,996
24(33  BC)  33(32)
792  24(BC)  1056 41. x 4 8 20
24(BC)  264
BC  11 3x  4 8 20 56

35. Let x represent the perimeter of DEF. The 42. x 10 0.1 10


perimeter of ABC  7  6  3 or 16.
perimeter of ABC
CB 1 0.1 10 0.1
FE   x

perimeter of DEF
6  
16
x 43.
9 x 30 6 9.6
6x  144
x 9 6 9.6 9.96
x  24 10
The perimeter of DEF is 24 units.
36. Let x represent the perimeter of QRS. The
perimeter of QTP  15  16  11 or 42.
TQ

perimeter of QTP
 ¬ 
Chapter 6 Practice Test
RQ perimeter of QRS
15 42
5 ¬ x
  
Page 337
15x ¬210 1. b
x ¬14 2. a
The perimeter of QRS is 14 units.
3. c
37. Let x represent the perimeter of CPD. CPD  x 1
BPA and D  A, so CPD BPA by AA 4. 
14  2
Similarity. Similar triangles have corresponding 2x  14
medians proportional to the corresponding sides, x7
so the corresponding medians are also 5. 43
x
10
x
8
proportional to the perimeters. 4x  324
2
BM perimeter of BPA x2  81
CN  
 
perimeter of CPD
x  9
13 12
  x k 
2 k2
7 ¬3
313 6. 
x13  3613 3(k  2) ¬7(k  2)
x  36 3k  6 ¬7k  14
38. Use the Pythagorean Theorem in PQM to find 6 ¬4k  14
PM. 20 ¬4k
(PM)2  (QM)2  (PQ)2 5 ¬k
(PM)2  122  132 7. Using corresponding angles, pentagon DCABE
(PM)2  169  144 CD 12
pentagon DGIHF. The scale factor is 
GD 
18 or
(PM)2  25
2 : 3.
PM  5
Since PQM PRQ, 8. Using corresponding angles, PQR PST.
PQ QR
PS  ST
PQ PM   
  
PR PQ
2x  2 2x
1
 3
 5  
2x  2 6 1
5
PR 13 2x  2 2x
5(PR)  13(13)    1
2x  8 5
5(PR)  169 15(2x  2)  2x(2x  8)
PR  33.8 30x  30  4x2  16x
PM QM 30  4x2  14x
PQ ¬ RQ
   
0  4x2  14x  30
5 1
 ¬ 2

13 RQ 0  2x2  7x  15
5(RQ) ¬13(12) 0  (x  5)(2x  3)
5(RQ) ¬156 x  5 ¬0 or 2x  3  0
RQ ¬31.2 x ¬5 x  3

2
3
The perimeter of PQR  13  33.8  31.2 or 78. Reject x  2, because otherwise QR  2x  3,
and length must be positive. So, x  5.
2
x QR
2(5
) 1
0
ST  15  15  15 or 2 : 3.
The scale factor is     

Chapter 6 186
9. Using corresponding angles, MAD MCB. 17. CMA  ACB and A  A, so
AM
  DM ABC ACM by AA Similarity.
CM BM (AM)2  62  102
25 3
x
 
x  20  
12 (AM)2  36  100
300  3x2  60x (AM)2  64
0  3x2  60x  300 AM  8
0  x2  20x  100 BC AC

CM AM 
0  (x  10)(x  10) B C 1 0
6  
x  10  0 8
x  10 8(BC)  60
DM 3
x 3(10) 30 BC  7.5
The scale factor is  BM  12  
    
12 12 or
5 : 2. B
C AB

CM ¬
AC

PQ 5 1
10. 
ML 10 or 2 7.5 8 MB
6 ¬ 10
  
PR 6 1

MN 12 or 2 75 ¬448  6(MB)
QR
  3 or 1
27 ¬6(MB)
LN 6 2 4.5 ¬MB
Corresponding sides are proportional, so the The perimeter of ABC is 10  7.5  8  4.5 or
triangles are similar by SSS Similarity. 30.
11. PTS  QTR since they are vertical angles. 18. x 3 12 87
mPTS  mQTR  90.
mPTS  mS  mP  180 5x  27 12 87 462
90  62  mP  180 height of fence shadow of fence
mP  28 19. 
height of backboard
 
shadow of backboard
mQTR  mR  mQ  180 4ft 20 
in.
90  66  mQ  180
x ¬ 65 in.
mQ  24 48 20
x ¬
65
Since only one pair of corresponding angles is 3120 ¬20x
congruent, the triangles are not similar. 156 ¬x
D
12. E  C B
 so AED  ACB and ADE  ABC The height of the top of the backboard is
because corresponding angles are congruent. Then 156 inches, or 13 feet.
the triangles are similar by AA Similarity. X Y
20. B; X  Y is spent, which is  X as part of the
GK GH weekly salary.
KJ  HI
13.   
8 
KJ 12
KJ  4

4(8  KJ)  12(KJ)
32  4(KJ)  12(KJ) Chapter 6 Standardized Test Practice
32  16(KJ)
2  KJ
GK GH
Pages 338–339
KJ  
14.   1. B; |8  2|  |6|  6
HI
GK 7
6 
 2. B; d  
[5 
(3)]2
 (2
 17)2
14  7
GK 7  (2)
 2
 (1
5)2

6  7
7(GK)  42  
4  225

GK  6  229
 miles
GK GH 3. A
KJ  HI
15.   
6 9 4. D
  
H
4 I 1.5 
cm 11.25cm
5. A;  2 ft ¬ x ft
6(HI)  36 
HI  6 1.5x ¬22.5
GI  GH  HI x ¬15 ft
 9  6 or 15 45  5
6. B; 63  7 , so the dimensions could be 7 inches by
16. Let x represent the perimeter of DEF. 5 inches.
CB perimeter of ACB
EF  
 
perimeter of DEF 7. C; us  pr because side UT corresponds to side RQ
10 7  10 13 and side ST corresponds to side PQ.
14 
   x
8. D
10 30
14  x
    9. counterexample
10x  420
x  42
The perimeter of DEF is 42 units.

187 Chapter 6
10. y  y1  m(x  x1) 13d. The new monthly rate will be $25 per month, so
y  2  3(x  2) the equation will be y  25x  40. The graph will
y  2  3x  6 have a less steep slope.
y  3x  4 14a. Using the Triangle Proportionality Theorem, if a
11. PQ  1

2 EF
line is parallel to one side of a triangle and
intersects the other two sides in two distinct
20  1
2 (3x  4)

points, then it separates these sides into
20  3
2x  2
 segments of proportional length, so  AE AD
AC  AB .
  
18  3

2x
Since the corresponding sides are proportional
12  x and the included angle, A, is the same, by SAS
Similarity Theorem we know that the triangles
20 
cm perimeter of model
40 m ¬ 
12.  23  40  46 are similar.
20
 x 14b. If B is between A and D and C is between A and
40 ¬ 109

AB AD
E, then BC  DE .
2180 ¬40x   
54.5 ¬x 3500 3500 
1500
1400 ¬
  
DE
The perimeter of the model is 54.5 cm. 3500 5000
1400 ¬ DE
   
(160  40)
13a. m   
(4  0) 3500(DE) ¬7,000,000
120 DE ¬2000
 4
 30 If D is between A and B and E is between A and
AB
 AD
13b. The slope represents the monthly flat rate, so C, then BC DE.
the company charges a flat rate of $30 per 3500 3500 
1500
1400 ¬
  
DE
month. 3500 2000
1400 ¬ DE
   
13c. y  y1  m(x  x1)
y  40  30(x  0) 3500(DE) ¬2,800,000
y  40  30x DE ¬800
y  30x  40 So, DE is 2000 feet or 800 feet.

Chapter 6 188
Chapter 7 Right Triangles and Trigonometry
Page 341 Getting Started
1. 3 12 7-1 Geometric Mean
4 ¬ a
  
3a ¬48
a ¬16 Page 343 Geometry Software Investigation
2. 5c ¬8

3 1. See students’ work.
3c ¬40 2. They are equal.
c ¬13.33
e 6 3. They are equal.
20 ¬ 5
3. 
4. They are similar.
5e ¬120
e ¬24
6 ¬f
5 10 Pages 345–346 Check for Understanding
60 ¬5f 1. Sample answer: 2 and 72.
12 ¬f Let x represent the geometric mean.
4. 4
 6
3 ¬ y 2 ¬x
x 72
4y ¬18 x2 ¬144
y ¬4.5 x ¬ 144 or 12
4 ¬1
z
3
2. C
4z ¬3
z ¬0.75
A
5. c2  52  122 D B
c2  25  144  169 For leg CB
, D
B is the segment of the hypotenuse
c  13 that shares an endpoint. Thus, it is the adjacent
6. c2  62  82 segment. The same is true for leg A C
 and segment
c2  36  64  100 D
A .
c  10 3. Ian; his proportion shows that the altitude is the
7. c2  152  152 geometric mean of the two segments of the
c2  225  225  450 hypotenuse.
c  21.21 4. Let x represent the geometric mean.
9 x
8. c2  142  272  ¬
x 4
c2  196  729  925 x ¬36
2

c  30.41 x ¬ 36 or 6
9.  8  42 5. Let x represent the geometric mean.
 4  
2  2
2 36 x
x ¬
4
9
10. 102  100 
52 25
x2 ¬1764
 75
   25  3
x ¬1764
 or 42
 25  
3  53 
6. Let x represent the geometric mean.
11. 39
 2 
362  
1521 
 129 6
 
225  15 6 ¬x
x 8
7 72 x2 ¬48
12.   
2
   2
2  x ¬ 48 or 4
3
7 
  
2 7 2
 
2
 x ¬6.9
4
13. x  44  38 ¬180 7. Let x represent the geometric mean.
x  82 ¬180 2
  ¬x
2
x ¬98 x 32

x2 ¬12
14. x  40  155
x  115 x ¬12
 or 23
x ¬3.5
15. x  2x  21  90 ¬180
3x  111 ¬180 8. Let x  CD.
3x ¬69 AD
 ¬ CD 
CD BD
x ¬23
2 ¬x
x 6
x2 ¬12
x ¬12
 or 23
x ¬3.5

189 Chapter 7
9. Let x  EH. 15. Let x represent the geometric mean.
GH EH 
45 x
x ¬ 80
  
EH ¬ FH
   

16  12
x ¬ x x2 ¬3600

12
x2 ¬48 x2 ¬60
x ¬48
 or 43 x ¬60 or 215
x ¬6.9 x ¬7.7
BD CD BA CA 16. Let x represent the geometric mean.
CD ¬ AD
10.  CA ¬ DA
     

28
 ¬x
x  
 y  
8 x
x ¬3
8 3
y ¬ 1372
3
x2 ¬24 y2 ¬33 x2 ¬38,41
6
x ¬24
 y ¬33
 x2 ¬196
x ¬26 x ¬196
 or 14
BD CD BA BC 17. Let x represent the geometric mean.
CD ¬ DA
11.  ¬
   
BC BD 3
x 2
 ¬ 3 x 2 y 
y ¬x
 5 x
23 2 x ¬1

6 2 y
2x ¬12  y ¬6 x2 ¬53
x ¬6 y2 ¬48
y ¬48
 or 43
x ¬ 3

5

3
x
12. Z 5

15
x ¬ 
5
x ¬0.8
18. Let x represent the geometric mean.
8 
3

¬x
5
x 

6 3

5
144
x2 ¬25
wall
x ¬ 
14
25
4

1  44
x ¬
Y 2 5
X 1
x ¬ 2
5 or 2.4
5 ft
19. Let x represent the geometric mean.
2 
2

6 x
12 ft W x ¬ 
5 
2

6
Draw a diagram. Let Y X
 be the altitude drawn 20
x2 ¬
36
from the right angle of WYZ.
WX YX
YX ¬ ZX
  
20
x ¬ 
36
5 1
 ¬ 2
 2  0
12 ZX x ¬
3  6
5ZX ¬144 2 
5
ZX ¬28.8 x ¬
6
Khaliah estimates that the wall is about 5  28.8  5
or 33.8 feet high. x ¬
3
x ¬0.7

Pages 346–348 Practice and Apply 20. Let x represent the geometric mean.
13

13. Let x represent the geometric mean. 7 x
x ¬ 
5
5 x
x ¬6

7
x2 ¬30 65
x2 ¬
49
x ¬ 30
x ¬5.5 
65
x ¬ 
49
14. Let x represent the geometric mean.

 

65 65
24 x x ¬ 
x ¬ 25 
49 7
x ¬600
2
x ¬1.2
x ¬ 600 or 106

x ¬24.5

Chapter 7 190
y
21. Let x ¬AD. x ¬
y 6
BD AD
 ¬
AD CD
y2  6x
5
x 5
y2  6 0
3 
x ¬9
x2 ¬45 y2  100
x ¬45 or 35
 y ¬100

x ¬6.7 y ¬10
22. Let x ¬EH. x ¬ z
x 
z 6
FH EH z2  x2  6x
 ¬
EH GH 2
12 x z2  3   6 3 
50 5
 0
x ¬
12

2500
x2 ¬144 z2   9  100
x ¬144
 or 12 16
z2   00
9
23. Let x ¬LM.
JM
 ¬
LM
LM
KM  
1600
z ¬   4
9 or 3
0

8 x 29. z2  52 ¬152
x ¬
16 z2  25 ¬225
x2 ¬128 z2 ¬200
x ¬128
 or 82 z ¬200 or 102

x ¬11.3 z ¬14.1
15 z
z ¬x
24. Let x  QS. 
PS Q S 15x ¬z2
S ¬

Q RS
15x ¬(10
2
2)
21 x
x ¬7
 15x ¬200
x2 ¬147 40
x ¬3
x ¬147
 15 5
5 ¬ y

x ¬12.1
15y  25
25. Let x  UW.
VW
  ¬ UW 
y ¬5

3
UW TW
2 x 30. x2  42 ¬102
x ¬
13 x2  16 ¬100
x2 ¬26 x2 ¬84
x ¬26
 x ¬84 or 221

x ¬5.1 x ¬9.2
y4 10
26. Let x  ZN.  ¬4
10
YN ZN
ZN ¬ XN
    4y  16 ¬100
2.5 x 4y ¬84
x ¬
1
0 y ¬21
x2 ¬25 zy4
x ¬25
 or 5 z  21  4  25
3 8 x 6x 36
27.  x ¬8
31. 6 ¬x
3
x2 ¬88 6x2 ¬1296
x ¬88
 or 222
 x2 ¬216
x ¬9.4 x ¬216 or 66

3 
8 y x ¬14.7
 y ¬3 6x  x y
 
y ¬
y2 ¬33 x
y2 ¬7x2
y ¬33

y2 ¬7(66)2
y ¬5.7
y ¬1512
2
8 z
z ¬3 y ¬1512 or 642

z2 ¬24 y ¬38.9
z ¬24
 or 26 6x  x
 
z ¬6z
x
z ¬4.9 z2 ¬42x2
6 ¬ 8 z2 ¬42(66)2
28. 8 x 6
z ¬9072
2
6x  36 ¬64
z ¬9072 or 367

6x ¬100
50 z ¬95.2
x ¬3


191 Chapter 7
12 x
32. x ¬8 40. Sample answer: The golden ratio occurs when the
x2 ¬96 geometric mean is approximately 1.62.
x ¬ 96 or 46
 41. Let x be the length of the brace. Let y be the
x ¬9.8 segment of the hypotenuse adjacent to the leg
12  8 y with measure 3 yards.
 y ¬8
y2 ¬32
y ¬32
 or 42
y ¬5.7
12 z
z ¬
12  8
z ¬48
2
5 yd 4 yd
z ¬48 or 43
z ¬6.9
a 17
33.  ¬  y x
17
 b
ab ¬17
3 yd
7b ¬17
17 5 ¬3
b ¬
7
3 y
5y ¬9
x 12
34.  ¬ 
12
 y y ¬9

5
xy ¬12 y x
 ¬
5 
x3 ¬12 x y
12 x2 ¬5y  y2
x ¬ or 43
 2

3 x2 ¬59
5  5
 9
x ¬6.9 81
x2 ¬9  
25
35. Never; let x and x  1 be consecutive positive 144
x  (x  1) x2 ¬
integers. The average of the numbers is   25
2
2x 
or  2
1 x ¬ 
14
4 1

25 or 5
2
2x  1
x ¬ 
2 x ¬2.4
 
2x  1
 x1 The brace is 2.4 yards long.
2 2  4x  1
4x
x  x ¬
2
4 42. Let x be the geometric mean.
The number of players from Indiana is 10, and
x2  x ¬x2  x  1 
4 the number of players from North Carolina is 7.
1 10
0 ¬4  x
x ¬ 7


Since 0 ¬1  x2 ¬70


4 , the first equation is never true.
x ¬70
The average of the numbers is not the geometric
x ¬8.4
mean of the numbers.
43. Let x be the geometric mean between UCLA and
36. Always; let a and b be positive integers, and let x
Clemson.
be the geometric mean of a2 and b2.
15 x
x ¬6
2
a ¬x
x b2
a2b2 ¬x2 x2 ¬90
(ab)2 ¬x2 x ¬90 
ab ¬x x ¬3 10
Because a and b are positive integers, their Let y be the geometric mean between Indiana and
product x is a positive integer. Virginia.
10 y
y  9
37. Sometimes; let a and b be positive integers, and
let x be their geometric mean. y2 ¬90
a ¬x y ¬ 90
x b
ab ¬x2 y ¬3 10
ab ¬x The geometric mean between Indiana and
x is an integer when ab is a perfect square. Virginia is the same as for UCLA and Clemson.
38. Sometimes; true when the triangle is a right
triangle, but not necessarily true otherwise.
39. FGH is a right triangle. O G
 is the altitude from
the vertex of the right angle to the hypotenuse of
that triangle. So, by Theorem 7.2, OG is the
geometric mean between OF and OH, and so on.

Chapter 7 192
AD CD
 ¬
44. 
CD BD
 47. Given: ADC is a right D
12 C D B
angle. D  is an
 ¬4
CD altitude of ADC.
(CD)2 ¬48 A
B AD A C
Prove: 
AD 
AC , B
CD ¬48 or 43 BC D C
(CB)2  (CD)2  (DB)2

DC 
AC
(CB)2  (43)2  42 Proof:
(CB)  48  16
2
Statements Reasons
(CB)2  64
1. ADC is a right 1. Given
CB   64 or 8 B
angle. D  is an
AB CB
CED  ACB, so  CD
DE. altitude of ADC.
12  4 8
 ¬ DE 2. ADC is a right 2. Definition of right
43
triangle. triangle
16DE ¬323 
3. ABD  ADC 3. If the altitude is
DE ¬23 
DBC  ADC drawn from the
45. Given: PQR is a ≈ right Q vertex of the rt.  to
angle. QS
 is an the hypotenuse of a
altitude of PQR. 2 rt. , then the 2 s
Prove: PSQ  PQR P 1 R formed are similar to
S
PQR  QSR the given  and to
PSQ  QSR each other.
Proof: AB AD 4. Definition of similar
4.   
AD AC ,
BC DC
polygons
Statements Reasons 
DC 
AC
1. PQR is a right 1. Given
angle. 48. Sample answer: The geometric mean can be used
S
Q  is an altitude of to help determine the optimum viewing distance.
PQR. Answers should include the following.
• If you are too far from a painting, you may not
S
2. Q R P
 2. Definition of altitude
be able to see fine details. If you are too close,
3. 1 and 2 are 3. Definition of you may not be able to see the entire painting.
right angles. perpendicular lines
• A curator can use the geometric mean to help
4. 1 PQR 4. All right  are . determine how far from the painting the roping
2 PQR should be.
5. P P 5. Congruence of angles 49. C; 6x ¬6
10
R R is reflexive.
10x ¬36
6. PSQ  PQR 6. AA Similarity x ¬3.6
PQR  QSR (statements 4 and 5) y 8
 ¬
7. PSQ  QSR 7. Similarity of triangles 8 10
is transitive. 10y ¬64
y ¬6.4
46. Given: ADC is a right D 50. B; 5x2  405 ¬1125
B
angle. D  is an 5x2 ¬720
altitude of ADC. x2 ¬144
AB D
B A C x ¬144

Prove:  B 
D CB B
x ¬12
Proof: It is given that ADC is a right angle and
B
D  is an altitude of ADC. ADC is a right
triangle by the definition of a right triangle. Page 348 Maintain Your Skills
Therefore, ADB  DCB, because if the altitude 51.
is drawn from the vertex of the right angle to the x 12 15 18
hypotenuse of a right triangle, then the two x3 15 18 21
triangles formed are similar to the given
AB DB 52.
x 4 14 44
triangle and to each other. So DB  CB by
definition of similar polygons. 3x  2 14 44 134
53.
x 3 7 47
x2 2 7 47 2207
54.
x 1 4 14
2(x  3) 4 14 34

193 Chapter 7
55. The smallest angle is opposite the side with the 66. c2  32  52
smallest measure, 20. Let x and 20  x be the c2  9  25
measures of the segments formed by the angle c2  34
bisector. By the Angle Bisector Theorem, c  34
x ¬24
 c  5.8
20  x 30 .
x 24 The length of the hypotenuse is about 5.8 inches.
 ¬
20  x 30
30x ¬480  24x
54x ¬480
Page 349 Geometry Activity: The Pythagorean
80 8
x ¬9 or 8 9 Theorem
10 0 1 1. yes
20  x ¬ 9 or 11 9
2. a2  b2  c2
The segments have measures 8 8 1
9 and 11 9 . 3. Sample answer: The sum of the areas of the two
56. By the Exterior Angle Inequality Theorem, smaller squares is equal to the area of the largest
m8  m6, m8  m3  m4, and square.
m8  m2. Thus, the measures of 6, 4, 2,
and 3 are all less than m8.
57. By the Exterior Angle Inequality Theorem,
m1  m5 and m1  m7. Thus, the 7-2 The Pythagorean Theorem and
measures of 5 and 7 are greater than m1. Its Converse
58. By the Exterior Angle Inequality Theorem,
m1  m7 and m6  m7. Thus, the
measures of 1 and 6 are less than m7. Pages 353–354 Check for Understanding
59. By the Exterior Angle Inequality Theorem, 1. Maria; Colin does not have the longest side as the
m2  m6, m7  m6, and m8  m6. value of c.
Thus, the measures of 2, 7, and 8 are all 2. Since the numbers in a Pythagorean triple satisfy
greater than m6. the equation a2  b2  c2, they represent the sides
60. y  mx  b of a right triangle by the converse of the
y  2x  4 Pythagorean Theorem.
8 0 3. A
61. m  02 3 5
8 3
2 or 4 C B
D 6
y  mx  b
y  4x  (8) 6 5
6
y  4x  8
06
62. m  
1 
2
F
12
E
6

3 or 2 Sample answer: ABC  DEF, A D,
y  y1 ¬m(x  x1) B E, and C F, A B corresponds to D
E,
y  6 ¬2(x  2) C
B  corresponds to EF , A
C
 corresponds to DF.
y  6 ¬2x  4 The scale factor is 2

1 . No; the measures of the
y ¬2x  2 sides do not form a Pythagorean triple since 6 5
and 3 5 are not whole numbers.
63. y  y1 ¬m(x  x1)
y  (3) ¬4[x  (2)] 4. x2  62 ¬102
y  3 ¬4(x  2) x2  36 ¬100
y  3 ¬4x  8 x2 ¬64
y ¬4x  11 x ¬64 or 8
2 2
64. c2 ¬32  42 5. x2  4
7  ¬ 7 
 5
c2 ¬9  16 16 25
c2 ¬25 x2  49 ¬ 49
  
9
c ¬ 25 or 5 x2 ¬
49
The length of the hypotenuse is 5 cm.
65. c2  52  122
x ¬  
9
49 or 7
3

c2  25  144 6. 202  37.52 ¬x2


c2  169 400  1406.25 ¬x2
c   169 or 13 1806.25 ¬x2
The length of hypotenuse is 13 feet. 1806.
25 ¬x
42.5 ¬x

Chapter 7 194
7. y Pages 354–356 Practice and Apply
K (1 , 6 ) 12. The altitude divides the side of measure 14 into
two congruent segments of measure 7 because the
L(3, 5) triangle is an isosceles triangle.
x2  72 ¬82
x2  49 ¬64
J (2 , 2 )
x2 ¬15
O x x ¬ 15
x ¬3.9
13. x2  42 ¬82
JK  
[1  (2)]2  (6
 2)2
x2  16 ¬64
 
12  42 or 17
 x2 ¬48
KL   
[3  (1)]2
 (5 
6)2 x ¬48 or 4 3
 
4  (
2 1) or 17
2  x ¬6.9
JL   
[3  (2)] 2
 (5 
2)2 14. 202  282 ¬x2
 
52  32 or 34
 400  784 ¬x2
JK 2  KL2 ¬JL2 1184 ¬x2
2 2 2 1184 ¬x
17
  17 ¬34 4 74 ¬x
17  17 ¬34 34.4 ¬x
34 ¬34
15. 402  322 ¬x2
Yes; JKL is a right triangle since the sum of the
1600  1024 ¬x2
squares of two sides equals the square of the
2624 ¬x2
longest side.
 2624 ¬x
8. Since the measure of the longest side is 39, 39 8 41 ¬x
must be c, and a or b are 15 and 36. 51.2 ¬x
a2  b2 ¬c2 16. x2  252 ¬332
152  362 ¬392 x2  625 ¬1089
225  1296 ¬1521 x2 ¬464
1521 ¬1521 x ¬ 464
These segments form the sides of a right triangle x ¬4 29
since they satisfy the Pythagorean Theorem. The x ¬21.5
measures are whole numbers and form a
17. x2  152 ¬252
Pythagorean triple.
x2  225 ¬625
9. Since the measure of the longest side is 21, 21 x2 ¬400
must be c, and a or b are 40 and 20. x ¬ 400 or 20
a2  b2 ¬c2 18. y
2
40
  202 ¬212
40  400 ¬441 R(1, 6)
440 ¬441
Since 400  441, segments with these measures
cannot form a right triangle. Therefore, they do
not form a Pythagorean triple.
10. Since the measure of the longest side is 108, O S(9, 0)
 108 must be c, and a or b are 44 and 8. Q(1, 0) x
a2  b2 ¬c2 QR   (1  1)2  (6
 0)2
2 2
 44  82 ¬ 108   02  62
44  64 ¬108   36 or 6
108 ¬108 RS   (9  1)2  (0
 6)2
Since 108  108, segments with these measures   82  (
6)2
form a right triangle. However the three numbers   100 or 10
are not all whole numbers. Therefore, they do not QS   (9  1)2  (0
 0)2
form a Pythagorean triple.   8  0
2 2

11. Let x represent the width of the screen.   64 or 8


11.52  x2 ¬192 QR2  QS2 ¬RS2
132.25  x2 ¬361 62  82 ¬102
x2 ¬228.75 36  64 ¬100
x ¬ 228.75 100 ¬100
x ¬15.1 Since the sum of the squares of two sides equals the
The screen is about 15.1 inches wide. square of the longest side, QRS is a right triangle.

195 Chapter 7
19. y RS  
[6 (4)]2 [9  (
4)]2
R (0, 6)  
(2) 
2  (5) or 29
2 
S (6, 6)
QS   [6 (9)]2 [ 9  (
2)]2
  3  (
2 7) or 58
2 
QR2  RS2 ¬QS2
Q (3, 2) (29)  (
2
29) ¬(
2
58)
2

29  29 ¬58
x 58 ¬58
QR  
(0  3)
2  (6
 2)2 Since the sum of the squares of two sides equals
 
(3)2 
 42 the square of the longest side, QRS is a right
triangle.
 
25 or 5
22. Since the measure of the longest side is 17, 17
RS  
(6  0)2  (6
 6)2
must be c, and a or b are 8 and 15.
 
6  0
2 2
a2  b2 ¬c2
 
36 or 6 82  152 ¬172
QS   (6  3)2  (6
 2)2 64  225 ¬289
  32  42 289 ¬289
  25 or 5 These segments form the sides of a right triangle
QR  QS2 ¬RS2
2 since they satisfy the Pythagorean Theorem. The
52  52 ¬62 measures are whole numbers and form a
25  25 ¬36 Pythagorean triple.
50 ¬36 23. Since the measure of the longest side is 25, 25
Since the sum of the squares of two sides is not must be c, and a or b are 7 and 24.
equal to the square of the longest side, QRS is a2  b2 ¬c2
not a right triangle. 72  242 ¬252
20. 49  576 ¬625
y
625 ¬625
R (2, 11) These segments form the sides of a right triangle
Q ( 4, 6) since they satisfy the Pythagorean Theorem. The
measures are whole numbers and form a
Pythagorean triple.
24. Since the measure of the longest side is 31, 31
x must be c, and a or b are 20 and 21.
S (4, 1) a2  b2 ¬c2
20  212 ¬312
2
QR   
[2  (4)]2
 (11
 6)2 400  441 ¬961
 
6  5 or 61
2 2  841 ¬961
RS  
(4  2)
2  (
1  11)2 Since 841  961, segments with these measures
 
2  (
2 12) or 148
2  cannot form a right triangle. Therefore, they do
not form a Pythagorean triple.
QS   
[4  (4)]2
 (1
 6)2
 
8  (
2 7) or 113
2  25. Since the measure of the longest side is 37, 37
must be c, and a or b are 12 and 34.
QR2  QS2 ¬RS2
2 2 2 a2  b2 ¬c2
61
  113
 ¬148
 122  342 ¬372
61  113 ¬148 144  1156 ¬1369
174 ¬148 1300 ¬1369
Since the sum of the squares of two sides is not Since 1300  1369, segments with these measures
equal to the square of the longest side, QRS is cannot form a right triangle. Therefore, they do
not a right triangle. not form a Pythagorean triple.
21. y
x
Q (9, 2)

R (4, 4)

S (6, 9)

QR  
[4 
(9)]2
 [
4  (
2)]2
 
5  (
2 2) or 29
2 

Chapter 7 196

26. Since the measure of the longest side is  7
4
, 33. Sample answer: They consist of any number of
35
 
74 1 1
similar triangles.
  must be c, and a or b are  and .
35 5 7 34. Yes; the measures of the sides are always
a2  b2 ¬c2 multiples of 5, 12, and 13.
2 2
7 2
1
   1
7  ¬ 35 
  4 35a. Find multiples of the triple 8, 15, 17.
5
1 1 74 8  2 ¬16

25 49 ¬ 
1225 15  2 ¬30
74
 ¬ 74
 17  2 ¬34
1225 1225
162  302 ¬342
These segments form the sides of a right triangle 256  900 ¬1156
since they satisfy the Pythagorean Theorem. 1156 ¬1156
However, the three numbers are not whole
8  3 ¬24
numbers. Therefore, they do not form a
15  3 ¬45
Pythagorean triple.
17  3 ¬51
35
27. Since the measure of the longest side is 
36 , 242  452 ¬512
35 3 2 576  2025 ¬2601
  
36 must be c, and a or b are 2 and 3 . 2601 ¬2601
a2  b2 ¬c2 Two triples are 16-30-34 and 24-45-51.

 2   32  ¬3356
3 2 2 2
 b. Find multiples of the triple 9, 40, 41.
9  2 ¬18
3  2 ¬
122
5 40  2 ¬80
4 9 1296
35 1225 41  2 ¬82
36 ¬ 1296
   
182  802 ¬822
35 122 5 324  6400 ¬6724
36  1296 , segments with these measures
Since   
6724 ¬6724
cannot form a right triangle. Therefore, they do
not form a Pythagorean triple. 9  3 ¬27
40  3 ¬120
28. Since the measure of the longest side is 1, 1
41  3 ¬123
must be c, and a or b are 3
 4
5 and 5 . 272  1202 ¬1232
a  b ¬c
2 2 2
729  14,400 ¬15,129
2 2
5
3
 4
5  ¬1
 2 15,129 ¬15,129
9 16 Two triples are 18-80-82 and 27-120-123.

25 
25 ¬1

c. Find multiples of the triple 7, 24, 25.
25
25 ¬1
 7  2 ¬14
24  2 ¬48
These segments form the sides of a right triangle
25  2 ¬50
since they satisfy the Pythagorean Theorem.
142  482 ¬502
However, the three numbers are not all whole
196  2304 ¬2500
numbers. Therefore, they do not form a
2500 ¬2500
Pythagorean triple.
10 1
0 7  3 ¬21
29. Since the measure of the longest side is   
7 , 7 24  3 ¬72
6
 8

must be c, and a or b are 7 and 7 . 25  3 ¬75
a2  b2 ¬c2 212  722 ¬752
2 2 2
441  5184 ¬5625
7
6
 8
 1
7  ¬ 7 
 0

36 64 100
5625 ¬5625
49  49 ¬ 49
    Two triples are 14-48-50 and 21-72-75.
100 100 36. d   (105  122)2
 (40
 38)2
49 ¬ 49
 
100 100   (17) 2  22

49  49 , segments with these measures


Since  
  289  4
form a right triangle. However, the three numbers   293
are not whole numbers. Therefore, they do not  17.1
form a Pythagorean triple. The distance from San Francisco to Denver is
30. about 17.1 degrees.
5 12 13
37. d   (105  115)2
 (40
 36)2
10 24 ? 26
  (10) 2 4 2
15 ? 36 39   100  16
? 20 48 52   116
31. 5-12-13  10.8
32. Sample answer: The triples are all multiples of The distance from Las Vegas to Denver is about
the triple 5-12-13. 10.8 degrees.

197 Chapter 7
38. Given: ABC with m 41. Let x be the hypotenuse of the triangle with
sides of measure a, A F height 26  12 or 38 and base 1
2 (9) or 4.5.
b, and c, where 4.5  38 ¬x
2 2 2
c2  a2  b2 b c b x
20.25  1444 ¬x2
Prove: ABC is a 1464.25 ¬x2
right triangle. C a B D a E  1464. 25 ¬x
The length of wire needed is 2x  2 1464.25 or
Proof: Draw DE on line  with measure equal to
about 76.53 feet.
a. At D, draw line m  DE. Locate point F on m
so that DF  b. Draw F E and call its measure x. 42. Let s represent the side of each square stone.
Because FED is a right triangle, a2  b2  x2. s2  (2s)2 ¬x2
But a2  b2  c2, so x2  c2 or x  c. Thus, ABC s2  4s2 ¬152
FED by SSS. This means C DE. 5s2 ¬225
Therefore, C must be a right angle, making s2 ¬45
ABC a right triangle. s ¬45 or 3
5
The area of each square stone is s2  45, so the
39. Given: ABC with right angle at C, AB  d
area of the walkway is 6  45 or 270 in2.
Prove: d   (x2  x1)2 (y2 y1)2
43. Let x represent the number of miles away from
y
the starting point.
62  122 ¬x2
A(x1, y1) 36  144 ¬x2
d 180 ¬x2
B (x2, y2) 180 ¬x
C (x1, y2) 13.4 ¬x
O x The trawler traveled about 13.4 miles out of the
way.
Proof: 44. AD  BC  6
AD2  AB2 ¬BD2
Statements Reasons
62  82 ¬BD2
1. ABC with right angle at 1. Given 36  64 ¬BD2
C, AB  d 100 ¬BD2
2. (CB)2  (AC)2  (AB)2 2. Pythagorean  100 ¬BD
Theorem 10 ¬BD
3.
x2  x1
 CB 3. Distance on a HD  BF  8

y2  y1
 AC number line
DM  1

2 (BD)
4.
x2  x1
2 
y2  y1
2  d2 4. Substitution
5. (x2  x1)2  (y2  y1)2  d2 5. Substitution  1
2 (10)
6. 
(x2  
x1)2 
(y2 
y1)2  d 6. Take the 5
square root HD2  DM2 ¬HM2
of each side. 82  52 ¬HM2
64  25 ¬HM2
7. d  
(x2  
x1)2 
(y2 
y1)2 7. Reflexive
Property 89 ¬HM2
89 ¬HM
9.4 ¬HM
40. First, use the Pythagorean Theorem to find the
length of the ladder, represented by y. HM  EM  FM  GM because HD  EA  FB 
122  162 ¬y2 GC and AC  DB so AM  BM  CM  DM.
144  256 ¬y2 45. Sample answer: The road, the tower that is
400 ¬y2 perpendicular to the road, and the cables form the
400 ¬y right triangles. Answers should include the
20 ¬y following.
The ladder is 20 feet long. • Right triangles are formed by the bridge, the
(2  12)2  x2 ¬202 towers, and the cables.
142  x2¬202 • The cable is the hypotenuse in each triangle.
196  x2 ¬400 46. A; let B(8, 0) be the vertex of right triangle ABE.
x2 ¬204 AB2  BE2 ¬AE2
x ¬204 82  h2 ¬102
x ¬251 64  h2 ¬100
x ¬14.3 h2 ¬36
The ladder reaches about 14.3 feet up the side of h ¬ 36
the house. h ¬6

Chapter 7 198
47. C; x2  36 ¬(9  x)2 57.
x2  36 ¬81  18x  x2 x 1 3 27
36 ¬81  18x
45 ¬18x 3x 3 27 7.6 1012
2.5 ¬x
48. 3-4-5, 6-8-10, 12-16-20, 24-32-40, 27-36-45 The sequence of numbers approaches positive
infinity.
49. 3  4  5  60
6  8  10  480 58.
x 4 2 1.41... 1.18... 1.09... 1.04...
 8  60
12  16  20  3840 1
x2
2 1.41... 1.18... 1.09... 1.04... 1.02...
 64  60
The sequence of numbers converges to 1.
24  32  40  30,720
59.
 512  60 x 4 0.25 4 0.25
27  36  45  43,740
 729  60 1 0.25 4 0.25 4
x
Yes, the conjecture holds true. The sequence of numbers alternates between 0.25
and 4.
60. No; 12  13  25, so the sides do not satisfy the
Page 356 Maintain Your Skills Triangle Inequality.
 7 
 
50. Let x represent the geometric mean. 3
61. 7   3 
3
3
 3

3 ¬x
x 12  
 2
18 2 18 2
x2 ¬36 62.    92

2
2
x ¬36  or 6 14 
2 
28
63.    2
51. Let x represent the geometric mean. 
2 2 
9 x 27
x ¬12 2  7

x ¬108
2
 3
3
11  3  33
33
x ¬ 108 or 63 64.     3 

3 3

x ¬10.4 24 
2 24   122
2
65.   
2 
52. Let x represent the geometric mean. 
2 2
66.     3 
11 x   12 3
x ¬ 7
 12 3   43

3 3 
x2 ¬77
2   
x ¬ 77 67.    
6  3
2
18

3  3 
3
x ¬8.8
6 
2
 3  22 

53. Let x represent the geometric mean.
6 x
x ¬9 15 
3 15 
3
  3  53

68.  
x2 ¬54 3
3
x ¬ 54 or 3 2 
8 2 
8
6 69.     8

8 8
x ¬7.3
42  

2
54. Let x represent the geometric mean.  8 2
2 x
x ¬7 25 
1 0 25 
10 5

10
70.   10  2
 
x2 ¬14 10
 
10
x ¬ 14
x ¬3.7
55. Let x represent the geometric mean.
7-3 Special Right Triangles
2 ¬x
x 5
x2 ¬10
x ¬ 10 Page 360 Check for Understanding
x ¬3.2 1. Sample answer: Construct two perpendicular
56. lines. Use a ruler to measure 3 cm from the point
x 5 10
 2.51... 2.24... 2.11... of intersection on one ray. Use the compass to
copy the 3 cm segment. Connect the two

2x 10
 2.51... 2.24... 2.11... 2.05... endpoints to form a 45°-45°-90° triangle with
sides of 3 cm and a hypotenuse of 3 2 cm.
The sequence of numbers converges to 2.

199 Chapter 7
2. Sample answer: Draw a line using a ruler. Then Graph A and B. A B
 lies on a vertical gridline of
use a protractor to measure a 90° angle. On one the coordinate plane. Since B D will be
ray mark a 2 cm length, and at that endpoint use perpendicular to A B
, it lies on a horizontal
the protractor to measure a 30° angle toward the gridline.
other ray. Where this ray intersects the other ray AB 
3  0
 3
should form a 60° angle, completing the 30°-60°- B
A  is the shorter leg. B
D is the longer leg.
90° triangle with sides 2 cm, 2 3cm, and a BD   3(AB)
hypotenuse of 4 cm. BD   3(3) or 33
3. The diagonal is twice as long as its width w, so Point D has the same y-coordinate as B. D is
the diagonal forms a 30°-60°-90° triangle with the located 3 3 units to the right of B or to the left of
length  and the width w. Then the length of the B. So, the coordinates of D are (8  3 3, 3) or
rectangle is 3 times the width, or    3w. about (13.20, 3) or (8  3 3, 3) or about (2.80, 3).
4. The triangle is a 45°-45°-90° triangle. The legs are 10. y
congruent, so x  3. The length of the hypotenuse D (2, 6 4 3)
is 2 times the length of the leg, so y  32.
5. The triangle is a 45°-45°-90° triangle. The legs are
congruent, so x  y. The length of the hypotenuse
is 2 times the length of the leg.
10 ¬x 2 B (2, 6) A(6, 6)
10
 ¬x
2
10  2
   ¬x
2
  2
10 2
 ¬x
2
52
 ¬x O x

So, x  52
 and y  52. Graph A and B. A B lies on a horizontal gridline of
the coordinate plane. Since B D
 will be
6. The triangle is a 30°-60°-90° triangle. y is the
perpendicular to A B, it lies on a vertical gridline.
measure of the hypotenuse and x is the measure
AB 
2  6
 4
of the longer leg.
B
A  is the shorter leg. BD is the longer leg.
y  2(8) or 16 BD   3(AB)
x  8 3 BD   3(4) or 4 3
7. ABC is a 30°-60°-90° triangle with hypotenuse c, Point D has the same x-coordinate as B. D is
shorter leg a and longer leg b. located 4 3 units above B. So, the coordinates of
c  2a D are (2, 6  4 3) or about (2, 12.93).
8  2a 11. The length of each leg of the 45°-45°-90° triangle
4a formed by homeplate, first base, and second base
b   3(a) is 90 feet. The distance d is the hypotenuse and is
b   3(4) or 4 3 2 times as long as a leg. Then d  902 or
8. ABC is a 30°-60°-90° triangle with hypotenuse c, about 127.28 feet.
shorter leg a and longer leg b.
b ¬3(a)
18 ¬ 3(a) Pages 360–362 Practice and Apply
18
 ¬a 12. The figure is a square, so each triangle is a
3

45°-45°-90° triangle. Thus, x  45. The length of
18 
3
   ¬a the hypotenuse is  2 times the length of a leg of
3
  3
18 
3
the triangle. Therefore, y  2(9.6) or 9.62.
 ¬a
3 13. The figure is a square, so each triangle is a
63 ¬a 45°-45°-90° triangle. Thus, y  45. the length of
c  2a the hypotenuse is  2 times the length of a leg of
c  2(63) the triangle.
c  123 17 ¬x 2
9. y 17
 ¬x
D (8 3 3, 3) B (8, 3) D (8 3 3, 3) 2 
17 
2
 ¬x
2  2
17
 2 ¬x
O A(8, 0) x 2

Chapter 7 200
14. The triangle is a 30°-60°-90° triangle where x is 18. In BCE, a is the measure of the hypotenuse and
the measure of the shorter leg and y is the E
C  is the longer leg.
measure of the longer leg. a ¬2(BE)
18 ¬2x 10 3 ¬2(BE)
9 ¬x 5 3 ¬BE
y ¬3(x) CE ¬ 3(BE)
y ¬3(9) or 9 3 CE ¬ 3 (5 3)
15. C CE ¬5  3 or 15
In CEA,  AE  is the longer leg.
y   3(CE)
y   3(15) or 15 3
y
12 19. In BEC, a is the measure of the hypotenuse and
x is the measure of the shorter leg.
60
a  2x
D a  2(73 )
a  143 
A x B CE  3 (x)
ABC is equilateral because C D bisects the base CE  3 (73 )
D
and C  is an altitude. Therefore ADC is a CE  7  3 or 21
30°-60°-90° triangle with hypotenuse y and In ACE, y is the measure of the longer leg and b
shorter leg 2x. is the measure of the hypotenuse.
x  3
 ¬12 y  3 (CE)
2 y  3 (21) or 213 
x ¬12  2 b  2(CE)
3

24 
3 b  2(21) or 42
x ¬
3
 3
 20. The altitude is the longer leg of a 30°-60°-90°
24 
3 triangle. Let x represent the length of the shorter leg.
x ¬ 
3 12  3 (x)
x ¬83
 12
x
3
y  2 2x
  12 
3
   x
yx 
3  3
y  83  12 
3
  x
3
16. x is the shorter leg and y is the longer leg of the 43x
30°-60°-90° triangle. Then the hypotenuse, which is a side of the
2x ¬11 equilateral triangle, has measure 2x  2(43 ) or
11   13.86 feet.
x ¬
2 or 5.5
83
y ¬3
(x) 21. The perimeter is 45, so each congruent side has
45
y ¬3
(5.5) or 5.53 measure  
3 or 15 cm. An altitude is the longer leg
17. F of a 30°-60°-90° triangle. Let x represent the length
15
of the shorter leg. Then x   
2 or 7.5. The altitude
5 has measure 3 (x)  7.53 or about 12.99 cm.
y
22. Each side of the square is a leg of a 45°-45°-90°
45
triangle. The length of the hypotenuse, 222  mm,
G is 2 times as long as a leg. So each leg has
D E
measure 22. Then the perimeter of the square is
x 4(22) or 88 mm.
In DEF, D E
 is the hypotenuse so x  52. In 23. The altitude is the longer leg of a 30°-60°-90°
FGE, F G
 is a leg and F
E
 is the hypotenuse. triangle. Let x represent the length of the shorter leg.
2(y) ¬5 7.4  3 (x)
7.4
y ¬
5   x
2
 3
7.4
 
3
5 
y ¬   
2  x
3  3
2
 
2
7.4 
3
5
2   x
y ¬2
3
Then the hypotenuse, which is a side of the
equilateral triangle, has measure 2x  2  
7.4

3

3

3
14.8 
or  3 . Thus, the perimeter of the equilateral


triangle is 3  3

14.8 3
 14.83 or about 25.63 m.

201 Chapter 7
24. c d 4 4 5  (1)
30
30

through the midpoint of G H


,   
2, 2 or (4, 2).
60
6 6 60
GH 
5  (1)
 6
b The diagonal through P also has measure 6. Point
e 60
60

a 30
P has the same y-coordinate as the midpoint of
60

120
H
G . P is located 3 units to the left or right of the
6 6 midpoint of GH. So, the coordinates of P are
f 60
60
(4  3, 2)  (1, 2) or (4  3, 2)  (7, 2).
29. y
The diagonals determine equilateral triangles D(3, 7)
with sides equal to half the length of each
diagonal, or 6 inches. So e  f  6, and e  f  3.
Then a  3 (e) or 33, so c  d  33 . Then the
perimeter of the rectangle is 2(6)  2(33   33 )
or 12  123   32.78 inches.
25. Each side has measure  
256
4 or 8. The length of a
x
diagonal is  2 times as long as a side of the square.
So the measure of a diagonal is 82   11.31.
26. 8  2x
4x
y  3 (x) P(10.5, 6) C (3, 6)
y  3 (4) or 43 
z6 Graph C and D. C D lies on a vertical gridline of
Each leg of the 45°-45°-90° triangle has length x, the coordinate plane. Since P C
 will be
or 4. Then CB  42 . So, the perimeter of ABCD perpendicular to CD, it lies on a horizontal
is 43  6  4  42   6  8  43   42  gridline.
24, or about 36.59 units. CD 
7  (6)
 13
27. y P (4, 8) D
C  is the longer leg. P
C is the shorter leg. So,
CD  3 (PC).
13  3(PC)
13
  PC
3
13 

3
  PC
3  3
13 
3
  PC
3
A (3, 1 ) B ( 4, 1)
Point P has the same y-coordinate as C. P is
x 13 
3
located  
3 units to the left of C. So, the
Graph A and B. A B
 lies on a horizontal gridline of coordinates of P are 3  13 
3

3 , 6 or about

the coordinate plane. Since P B
 will be (10.51, 6).
perpendicular to AB
, it lies on a vertical gridline. 30. y
AB 
4  (3)
 7
AB and PB are congruent, so PB  7. Point P has
the same x-coordinate as B. P is located 7 units
P (8, 1.5) x
above B. So, the coordinates of P are (4, 1  7) or
(4, 8).
28. y

H (4, 5 ) C (2, 5) D (10, 5)

P P Graph C and D. C D lies on a horizontal gridline of


the coordinate plane. mC  30, so mD  60.
Let Q be the point on C D where the altitude from
x P intersects CD. Then mCPQ  60.
G ( 4 , 1 ) CD 
10  2
 8
P
C  is the longer leg, and PD
 is the shorter leg.
Graph G and H. If GH is the diagonal of a square 2(PD)  CD
with vertices P, G, and H then the other diagonal 2(PD)  8
is perpendicular to and bisects GH. G
H
 lies on a PD  4
vertical gridline of the coordinate plane, so the 3(PD)  CP
other diagonal through P is horizontal and goes 3(4)  CP
43   CP

Chapter 7 202
CP is the hypotenuse of 30°-60°-90° triangle CPQ. d¬ 3 (c)
Q
C  is the longer side and PQ is the shorter side. d¬ 3 (3 )
2(PQ)  CP d¬ 3
2(PQ)  43  d¬ 2e
PQ  23  3¬ 2e
3(PQ)  CQ 1.5¬ e
3(23 )  CQ f¬ 3 (e)
2(3)  CQ f¬ 1.53 
6  CQ f¬ 2g
Q is on CD 6 units to the right of C, so Q has 1.53¬ 2g
coordinates (2  6, 5)  (8, 5). P is 23  units 0.753 ¬ g
above Q, so P has coordinates (8, 5  23 ). x¬ 3 (g)
31. T
S is the shorter leg of a 30°-60°-90° triangle. x¬ 3 (0.753)
SP  2(ST) x¬ 0.75  3  2.25
63  2(ST) 39. The hexagon consists of six equilateral triangles.
33  ST So, mUXY  60. WY bisects a 60° angle, so
Then c  33 . PT is a vertical line segment, so mXYW  30. WY is twice the length of the
a  c  33 . longer side of a 30°-60°-90° triangle.
PT  3 (ST)
PT  3 (33 )
1
WY ¬2  2

2 3
 
PT  3  3 or 9 ¬123
Then b  9. PQ S R
, so PQ is a horizontal line ¬20.78 cm
segment. Thus d  b  9. 40. Find CB.
32. 12 triangles AB  2 (CB)
33. The smaller angle is rotated, so it is the 30° angle. 347  2(CB)
34
7
34. There are no gaps because when a 30° angle is   CB
2
rotated 12 times, it rotates 360°. 347  
2
    CB
35. Sample answer: 2  2

347 2
  CB
2
2
347
The center fielder is standing  2 or about
245.4 feet from home plate.
41. 24
A E B
60

8
36. 2x  8
45

x4
D F C
y  3(x)
y  3(4) or 43
 Draw altitudes C E and AF
. BEC is a
y  2(z) 30°-60°-90° triangle and DAF is a
  2
43 (z) 45°-45°-90° triangle.
4 
3 In BEC,  CB is the hypotenuse and EC
 is the
  z
2 longer leg.
4 
3  
2
    z CB  2(EB)
2 2
4 
6 8  2(EB)
  z 4  EB
2
26z EC  3 (EB)
37. BD  3 (DH) EC  43 
  3
83 (DH) AF  EC, so AF  43 .
8  DH DF  AF, so DF  43 .
BH  2(DH) AD  2 (DF)
BH  2(8) AD  2 (43)
BH  16 AD  46 
FC  AE
38. 4¬ 2a a
c  AB  EB
2¬ a
 24  4
b¬ 3 (a) 30
e  20
b¬ 23  b 30
d
4 The perimeter of ABCD is AB  BC  DF  FC 
b¬ 2c
30
f AD  24  8  43   20  46 or 52  43 
23¬ 2c g
30
46 units.
3¬ c
x

203 Chapter 7
42. Sample answer: Congruent triangles of different 48. a2  b2¬ c2
color can be arranged to create patterns. Answers 202  482¬ 522
should include the following. 400  2304¬ 2704
• 5, 9, 15, and 17; 3, 4, 6, 7, 10, 11, and 12 2704¬ 2704
• Placing 45° angles next to one another forms Since 2704  2704, the measures satisfy the
90° angles, which can be placed next to each Pythagorean Theorem. So the sides can be the
other, leaving no holes. sides of a right triangle. All side lengths are whole
43. C; 2x  4x  90 numbers, so the measures form a Pythagorean
6x  90 triple.
x  15 49. a2  b2¬ c2
mA  4x 72  242¬ 252
 4(15) 49  576¬ 625
 60 625¬ 625
mB  2x Since 625  625, the measures satisfy the
 2(15) Pythagorean Theorem. So the sides can be the
 30 sides of a right triangle. All side lengths are whole
BC  3 (AC) numbers, so the measures form a Pythagorean
6  3(AC) triple.
6  AC 50. a2  b2¬ c2
3
6  
3 122  342¬ 372
    AC 144  1156¬ 1369
3  3
6 
3 1300¬ 1369
3  AC
Since 1300  1369, the measures do not satisfy
23   AC the Pythagorean Theorem. So the sides cannot be
AB  2(AC) the sides of a right triangle. The measures do not
AB  2(23)  43
 form a Pythagorean triple.
  
2 2
44. (3 ★ 4)(5 ★ 3)¬ 3 5 z 10
42 32 51. 1
0 ¬ 4
52 4z¬ 100
¬ 42
z¬ 25
25
¬ 
16 y¬ z  4
y¬ 25  4 or 21
y x
¬ 4
x
Page 363 Maintain Your Skills 4y¬ x2
45. a2  b2¬ c2 4(21)¬ x2
32  42¬ 52 84¬ x2
9  16¬ 25 84¬ x
25¬ 25 221
¬ x
Since 25  25, the measures satisfy the 9.2¬ x
12
Pythagorean Theorem, so the sides can be the 52. ¬ x
x 8
sides of a right triangle. All side lengths are whole
96¬ x2
numbers, so the measures form a Pythagorean
96
¬ x
triple.
46
¬ x
46. a2  b2¬ c2 9.8¬ x
92  402¬ 412 8¬ 
y
81  1600¬ 1681 y 12  8
1681¬ 1681 y2¬ 32
Since 1681  1681, the measures satisfy the y¬ 32

Pythagorean Theorem. So the sides can be the y¬ 42 
sides of a right triangle. All side lengths are whole y¬ 5.7
numbers, so the measures form a Pythagorean 12
 z
z ¬ 12  8
triple.
z2¬ 48
47. a2  b2¬ c2 z¬ 48

202  212¬ 312 z¬ 43 
400  441¬ 961 z¬ 6.9
841¬ 961
Since 841  961, the measures do not satisfy the
Pythagorean Theorem. So the sides cannot be the
sides of a right triangle. The measures do not
form a Pythagorean triple.

Chapter 7 204
15 5 13
53. 
5 ¬ y 62. 0.2  g
15y¬ 25 0.2g  13
y¬ 5
 g  65
3
7
x¬ 15  y 63. n  0.25
x¬ 15  5
 7  0.25n
3
40 28  n
x¬   m
3 64. 9   0.8
15 z
z¬ x 7.2  m
15x¬ z2 24
65. x  0.4
 
40
15 
3 ¬ z
2
24  0.4x
200¬ z2 60  x
200
¬ z 35
66. y¬ 0.07
102
¬ z 35¬ 0.07y
14.1¬ z 500¬ y
54. In ALK and ALN, A L
 A L,  N
KL L
, and
AK  AN. Then mALK  mALN by the SSS
Inequality.
Page 363 Practice Quiz 1
55. In ALK and NLO, A L OL, 
KL N
L
, and 1. Let x represent the measure of the altitude.
AK  NO. So, mALK  mNLO by the SSS 21 x
 x¬ 7
Inequality.
x2¬ 147
56. In OLK and NLO, L O LO, 
KL N
L, and
x¬ 147 or 73 
KO  NO. So, mOLK  mNLO by the SSS
x¬ 12.1
Inequality.
2. Let x represent the measure of the altitude.
57. In KLO and ALN, K L NL, 
LO A
L
, and
9 x
N
A  K O
. KLO ALN by SSS Congruence. x¬ 5
So mKLO¬ mALN by CPCTC and the x2¬ 45
definition of congruent angles. x¬ 45  or 35 
58. JK   [1  (3)]2 (5
 2)2 x¬ 6.7
 2 2  3
2 3. AB   (4  2)2  (0 1)2
 13
  2  2 
 (1) 2

RS   [4  (6)]2
 (3
 6)2  5 
 2 2 
 (3) 2 BC   (5  4)2  (7 0)2
 13  1  2  7
2

KL   [4  (
1)]2 
 (4 
5)2  50 
 5 2 
 (1) 2 AC   (5  2)2  (7 1)2
 26  3 2  6
2

ST   
[1  (4)] 2
 (4 
3)2  45 
 5 2  12 AB2  AC2¬ BC2
 26 (5 )2  (45 )2¬ (50 )2
JL   [4  (3)]2 
 (4 
2)2 5  45¬ 50
 7 2  2
2 50¬ 50
 53
 The triangle is a right triangle because the side
lengths satisfy the Pythagorean Theorem.
RT   [1  (
6)]2 
 (4 
6)2
4. x  3
 72)
2  ( 2
y  (2 )3 or 32 
 53

5. x  2(6) or 12
JKL RST by SSS.
y  3 (6) or 63 
59. 5¬ 3x
15¬ x
60. 9x  0.14
x  1.26 7-4 Trigonometry
10
61. 0.5¬ k
0.5k¬ 10 Page 365 Geometry Activity: Trigonometric
k¬ 20 Ratios
1. They are similar triangles because corresponding
sides are proportional.

205 Chapter 7
opposite leg
2. In AED In AGF In ABC 6. sinA   
hypotenuse
DE FG BC  ac
AD  0.6114 AF  0.6114 AC  0.6114
sin A   
8

17  0.47
AE AG
  0.7913  AB
cos A  AF  0.7913 AC  0.7913
AD    adjacent leg
cosA   
hypotenuse
DE FG BC
AE  0.7727 AG  0.7727 AB  0.7727
tanA   bc
   
15
3. Sample answer: Regardless of the side lengths, 
17  0.88

the trigonometric ratio is the same when opposite leg
tanA  
adjace
nt leg
comparing angles in similar triangles.
 a

b
4. mA is the same in all triangles.
8

15  0.53
opposite leg
sinB   
hypotenuse
Pages 367–368 Check for Understanding
1. The triangles are similar, so the ratios remain the  bc
same. 15

17  0.88

2. Sample answer: adjacent leg
C cosB   
hypotenuse
15  ac
35 8
B A 
17  0.47
mB  90, mC  55, b  26.2, c  21.4 tanB  
opposite leg

adjacent leg
3. All three ratios involve two sides of a right triangle. b
The sine ratio is the measure of the opposite side  a
divided by the measure of the hypotenuse. The 15
8  1.88
cosine ratio is the measure of the adjacent side
divided by the measure of the hypotenuse. The 7. KEYSTROKES: SIN 57 ENTER
tangent ratio is the measure of the opposite side sin57°  0.8387
divided by the measure of the adjacent side. 8. KEYSTROKES: COS 60 ENTER
4. The tan is the ratio of the measure of the opposite cos60°  0.5000
side divided by the measure of the adjacent side for 9. KEYSTROKES: COS 33 ENTER
a given angle in a right triangle. The tan1 is the
cos33°  0.8387
measure of the angle with a certain tangent ratio.
opposite leg 10. KEYSTROKES: TAN 30 ENTER
5. sinA   
hypotenuse tan30°  0.5774
 ac 11. KEYSTROKES: TAN 45 ENTER
14
 tan45°  1.0000
50  0.28

adjacent leg 12. KEYSTROKES: SIN 85 ENTER
cosA   
hypotenuse sin85°  0.9962
 bc 13. KEYSTROKES: 2nd [TAN1] 1.4176 ENTER
48 mA  54.8
50  0.96

opposite leg 14. KEYSTROKES: 2nd [SIN1] 0.6307 ENTER
tanA  adjacent leg
a
mB  39.1
 b 15. y
14
 48  0.29

C(0, 6)
opposite leg
sinB   
hypotenuse
b
 c
48
 50  0.96
 A(6, 0)
B(4, 2)
adjacent leg
cosB   
hypotenuse x
a
 c
 14 Explore: You know the coordinates of the vertices
50  0.28

of a right triangle and that C is the right angle.
opposite leg
tanB  adjacent leg You need to find the measure of one of the angles.
b Plan: Use the Distance Formula to find the measure
 a
of each side. Then use one of the trigonometric ratios
48
 14  3.43
 to write an equation. Use the inverse to find mA.

Chapter 7 206
Solve: AB  (4  6)2  (2  0)2 KEYSTROKES: 2nd [TAN1] 1 2 ENTER
 100
 4 mB  26.56505118
The measure of B is about 26.6.
 104
 or 226 
Examine: Use the sine ratio to check the answer.
BC  [0  (
4)] 2  (6  2)2 AC
sinB   
 16
6  1 AB
 32
 or 42  sinB  4 or 1
4
5 
5
AC  (0  6)
 36
6  3
2  (6  0)2
5
B  sin 1 1
KEYSTROKES: 2nd [SIN1] 1 2nd [2 ] 5
 72
 or 62 
ENTER
Use the tangent ratio.
BC mB  26.56505118
tanA  
AC The answer is correct.
4

2
tanA   or 2
 17. Let x be Maureen’s distance from the tower in

62 3
feet.
A  tan12
3
 1815
tan31.2   
x
KEYSTROKES: 2nd 2 3 ENTER
[TAN1] 1815
x
tan 3
mA  33.69006753 1.2
The measure of A is about 33.7. KEYSTROKES: 1815 TAN 31.2 ENTER
Examine: Use the sine ratio to check the answer. Maureen is about 2997 feet from the tower.
BC
sinA  
AB

4

2
sinA  
2
26 Pages 368–370 Practice and Apply
2
4
A  sin 1  
2
26
18. sinP  
opposite leg

hypotenuse
KEYSTROKES: 2nd [SIN1] 4 2nd [2
] 2 ) p
 r
( 2 2nd [2 ] 26 ) ) ) 12
 37  0.32

mA  33.69006753
adjacent leg
The answer is correct. cosP   
hypotenuse
16. q
y
B(7 , 5 )  r
35
 37  0.95

opposite leg
tanP   adjace
nt leg
p
 q
x 12
 35  0.34

opposite leg
sinQ   
hypotenuse
A(3 , 3 ) C(7 ,  3 ) q
 r
Explore: You know the coordinates of the vertices
35
of a right triangle and that C is the right angle.  37  0.95

You need to find the measure of one of the angles. adjacent leg
cosQ   
hypotenuse
Plan: Use the Distance Formula to find the measure
p
of each side. Then use one of the trigonometric  r
ratios to write an equation. Use the inverse to 12
 37  0.32

find mB.
opposite leg
Solve: AB   (7  3)2  [5
 (3)]2 tanQ   adjace
nt leg
 164  6 q
p 
 80 or 45  35

12  2.92

BC  (7 )  732  (  5)2
 0   64 or 8
AC   (7  3)2  [
3  ( 3)]2
 160   or 4
Use the tangent ratio.
AC
tanB  
BC
tanB  4
 1
8 or 2
B  tan12
1

207 Chapter 7
opposite leg opposite leg
19. sinP   
hypotenuse
sinQ   
hypotenuse
p q
 r  r
  
 

6 2 3
 3
  
2
32  2  3
 
3  3
 3  0.58 2  0.87
adjacent leg
cosP    adjacent leg
hypotenuse cosQ   
hypotenuse
a
 r p
 r
2  
 3
  
2
3
32  2 
 
2
3
 
6
 3  0.82
opposite leg  1
2  0.50

tanP  adjace nt leg opposite leg
p tanQ  
adjace
nt leg
 q q
 p
  
 6
  
3
23  3  3
 
3
2
 
2  
 2  0.71 3

2

sinQ  
opposite leg

 3
  1.73
hypotenuse opposite leg
q
 r 21. sinP   
hypotenuse
p
 2  
3
  
2  r
32  2 
2
3
 6  
 3  0.82 
33
adjacent leg  2
3  0.67

cosQ   
hypotenuse adjacent leg
p cosP   
 r hypotenuse
q
    r
 6
  
2
32  2  15
 

3


 3 33  3 
 3  0.58
 5
opposite leg  3  0.75
tanQ   adjace nt leg opposite leg
q tanP  adjace
 p nt leg
p
2    q
 3
  
6
6  6  2  
 2
  1.41  3
 15
 15  15
opposite leg 2 5
20. sinP   
hypotenuse
 5  0.89

p opposite leg
 r sinQ  hypotenuse

3 q
 
2  r
3
15  
3
 
 1
2  0.50
 33  3 
adjacent leg  5
cosP     3  0.75
hypotenuse
 ar cosQ  
adjacent leg

hypotenuse
 
3
3 p
 
2  r
3
2 
3
3   
2  0.87 33 
2
tanP  
opposite leg  3  0.67
adjace
nt leg
opposite leg
p
 q tanQ  adjace nt leg
q
3
2
 p
 
3 
3
 15   
3
2  
 
3 23  3 
3  0.58  
5
 2  1.12


Chapter 7 208
opposite leg
22. KEYSTROKES: SIN 6 ENTER 36. tanx°  
adjace
nt leg
sin6°  0.1045 25
 
5
23. KEYSTROKES: TAN 42.8 ENTER
 5.0000
tan42.8°  0.9260
37. KEYSTROKES: 2nd [SIN1] 0.7245 ENTER
24. KEYSTROKES: COS 77 ENTER mB  46.42726961
cos77°  0.2250 The measure of B is about 46.4.
25. KEYSTROKES: SIN 85.9 ENTER 38. KEYSTROKES: 2nd [COS1] 0.2493 ENTER
sin85.9°  0.9974 mC  75.56390633
26. KEYSTROKES: TAN 12.7 ENTER The measure of C is about 75.6.
tan12.7°  0.2254 39. KEYSTROKES: 2nd [TAN1] 9.4618 ENTER
27. KEYSTROKES: COS 22.5 ENTER mE  83.96691253
cos22.5°  0.9239 The measure of E is about 84.0.
opposite leg
28. sinA¬  
40. KEYSTROKES: 2nd [SIN1] 0.4567 ENTER
hypotenuse mA  27.17436867
26 The measure of A is about 27.2.
¬  
26
¬ 0.1961 41. KEYSTROKES: 2nd [COS1] 0.1212 ENTER
opposite leg mD  83.03863696
29. tanB  
adjace
nt leg The measure of D is about 83.0.
5
26
 or 5 42. KEYSTROKES: 2nd [TAN1] 0.4279 ENTER
126 1
 5.0000 mF  23.16608208
adjacent leg The measure of F is about 23.2.
30. cosA¬  
hypotenuse x
43. tan24°  
19

26
5
¬ 26 19tan24°  x
¬ 0.9806 KEYSTROKES: 19 TAN 24 ENTER
opposite leg x  8.5
31. sinx°¬  
hypotenuse 12
25 44. sinx°  
17
¬  
526 x°  sin1
17 
12

25
 
26
¬  
526  26 KEYSTROKES: 2nd [SIN1] 12 17
25 
26 ENTER
¬ 5(26) x  44.9
5
26
¬  cos62°  x
26 45. 60
¬ 0.9806 60cos62°  x
adjacent leg KEYSTROKES: 60 COS 62 ENTER
32. cosx°¬  
hypotenuse x  28.2
5 x
¬  cos31°  
526 46. 34
5
¬   
2
6 34cos31°  x
 
526 26 KEYSTROKES: 34 COS 31 ENTER
26 x  29.1
¬  
26
6.6
¬ 0.1961 47. sin17°  x
6.
6
33. tanA  
opposite leg x
adjace
nt leg sin 17°

2 6 KEYSTROKES: 6.6 SIN 17 ENTER

x  22.6
5
26
 1

5
15
48. tanx°  
18
 0.2000 x°  tan1
18 
15

adjacent leg
34. cosB¬  
hypotenuse KEYSTROKES: 2nd [TAN1] 15 18
26 ENTER
¬  
26 x  39.8
¬ 0.1961
opposite leg
35. siny°¬  
hypotenuse
1
¬ 
26


¬ 1  2
6
26
 
26

¬ 226
 6

¬ 0.1961

209 Chapter 7
49. Let x represent the vertical change of the plane
after climbing at a constant angle of 3° for 60 41
5
J  sin 1 


ground miles. KEYSTROKES: 2nd [SIN1] 5 2nd [2
] 41
x
tan3°  60
ENTER
60tan3°  x mJ  51.34019175
The answer is correct.
KEYSTROKES: 60 TAN 3 ENTER
x  3.1 53.
D(1 , 2 ) y
The plane is about 3.1  1 or 4.1 miles above
sea level.
50. Let x represent the maximum height. x
x
sin75°¬ 20
20sin75°¬ x
KEYSTROKES: 20 SIN 75 ENTER C(6 , 5 )
x  19.32 B(1 ,  5 )
The ladder can reach a maximum height of about
19.32 feet.
Explore: You know the coordinates of the vertices
51. Let x represent the distance from the base of the
of a right triangle and that B is the right angle.
ladder to the building.
x You need to find the measure of one of the angles.
cos75°  20 Plan: Use the Distance Formula to find the measure
20cos75°  x of each side. Then use one of the trigonometric ratios
KEYSTROKES: 20 COS 75 ENTER to write an equation. Use the inverse to find mC.
x  5.18 Solve: BC   [6  (1)]2  [ 2
5  (5)]
The base of the ladder is about 5.18 feet from the  25  0 or 5
building.
BD   [1  (1)]2  [2 2
 (5)]
52. y  0   49 or 7
J(2, 2) CD   [1  (6)]2  [2 2
 (5)]
 259  4
 74
x BD
tanC  
B
C
C(2, 2) L(7 ,  2 ) tanC  7

5
C  tan17
5

KEYSTROKES: 2nd [TAN1] 7 5 ENTER
Explore: You know the coordinates of the vertices mC  54.46232221
of a right triangle and that C is the right angle. The measure of C is about 54.5.
You need to find the measure of one of the angles. Examine: Use the sine ratio to check the answer.
Plan: Use the Distance Formula to find the BD
measure of each side. Then use one of the sinC  
CD
trigonometric ratios to write an equation. Use the 7
sinC  
inverse to find mJ. 74

Solve: JC   (2  2)
2  (
2  2)2 74
7
C  sin 1 

 0  16 or 4 KEYSTROKES: 2nd [SIN1] 7 2nd [2
] 74
CL   (7  2
)2  [ 2
2  (2)] ENTER
 25
  0 or 5 mC  54.46232221
The answer is correct.
JL   (7  2
)2  (
2  2)2
 25
6  1 54. y
 41
 Z(0 , 35)
CL
tanJ  JC
tanJ  5

4
J  tan15
4

X(5 , 0 ) Y(7 , 0 )
KEYSTROKES: 2nd [TAN1] 5 4 ENTER
x
mJ  51.34019175
The measure of J is about 51.3.
Examine: Use the sine ratio to check the answer. Explore: You know the coordinates of the vertices
CL of a right triangle and that Z is the right angle.
sinJ  
JL
You need to find the measure of one of the angles.
sinJ  5
41


Chapter 7 210
6 
5
Plan: Use the Distance Formula to find the tany°   
24
measure of each side. Then use one of the 
5
tany°  4
trigonometric ratios to write an equation. Use the

inverse to find mX. y°  tan14 
5

Solve: XY   
[7  (5)]2 (0 
0)2 KEYSTROKES: 2nd [TAN1] 2nd [2
] 5 )
 144
  0 or 12 4 ENTER
YZ   (0  7)
2  (  0
35 )2 y  29.2
x
 49
5  3 57. tan55°¬ 
1
2
 84
 or 221  12tan55°¬ x
XZ   
[0  (5)] 2
 (35  0)2
 KEYSTROKES: 12 TAN 55 ENTER
 25
5  3 x  17.1
 60
 or 215  sin47°  xy
XZ x
cosX  
XY y
sin 47°
cosX   
15
2 15

 KEYSTROKES: 12 TAN 55 ) SIN 47
12 or 6
ENTER
15

X  cos16 
 y  23.4
KEYSTROKES: 2nd [COS1] 2nd [2 ] 15 ) 24
58. tan32°  x
6 ENTER
24
mX  49.79703411 x
tan
32°
The measure of X is about 49.8. KEYSTROKES: 24 TAN 32 ENTER
Examine: Use the sine ratio to check the answer. x  38.4
YZ y
sinX  
XY
cos32°  x

21
2 21
 xcos32°  y
sinX  12 or 6 KEYSTROKES: 24 TAN 32 ) COS 32
X  sin 1 
21

6 ENTER
KEYSTROKES: 2nd [SIN1] 2nd [2
] 21 ) y  32.6
6 ENTER 59. Let d represent the distance between Alpha
mX  49.79703411 Centauri and the sun.
1
The answer is correct. tan0.00021  d
1
55. A d
tan0.00021
KEYSTROKES: 1 TAN 0.00021 ENTER
d  272,837
35

20 The distance is about 272,837 astronomical units.


60. The stellar parallax would be too small.
61. Let E be the point where DB
 intersects AC
.
C B EAB is a 45°-45°-90° triangle because it is an
isosceles right triangle.
CB
sin35°   2
0 AB  2 (AE)
20sin35°  CB 8  2 (AE)
KEYSTROKES: 20 SIN 35 ENTER 8  AE
CB  11.5 2
cos35°¬  AC
 
8   
2
20  AE
2 2 
20cos35°¬ AC   AE
42
KEYSTROKES: 20 COS 35 ENTER
AE
AC  16.4 sinx°  
AD
The perimeter is about 20  11.5  16.4 or 4 
2
sinx°   
10
47.9 inches.
2 
2
24
56. sinx°   sinx°   5
36
62. Sample answer: Surveyors use a theodolite to
x°  sin1
36 
24

measure angles to determine distances and
KEYSTROKES: 2nd [SIN1] 24 36 ENTER heights. Answers should include the following.
x  41.8 • Theodolites are used in surveying, navigation, and
Let a represent the side of the smaller triangle meteorology. They are used to measure angles.
opposite the angle of measure y. • The angle measures from two points, which are
(2a)2  242¬ 362 a fixed distance apart, to a third point.
4a2  576¬ 1296
4a2¬ 720
a2¬ 180
a¬ 180
 or 65 

211 Chapter 7
63. C; (AC)2  32  52 73. a2  b2¬ c2
(AC)2  9  25 52  122¬ 132
(AC)2  16 25  144¬ 169
AC  16  or 4 169¬ 169
AC Since the side measures satisfy the Pythagorean
cosC  BC

Theorem, they can be the sides of a right triangle.
cosC  4

5 The measures are all whole numbers, so they do
64. B; x2  152  242  15(24) form a Pythagorean triple.
x2  225  576  360 74. a2  b2¬ c2
x2  441 92  122¬ 152
x  441
 81  144¬ 225
x  21 225¬ 225
65. cscA  5 5 4 5
3 ; secA  4 ; cotA  3 ; cscB  4 ; Since the side measures satisfy the Pythagorean
secB  5 3
3 ; cotB  4
 Theorem, they can be the sides of a right triangle.
The measures are all whole numbers, so they do
13 1
3 5 1
3
66. cscA  
12 ; secA  5 ; cotA  12 ; cscB  5 ;
   form a Pythagorean triple.
13
secB   12 75. a2  b2¬ c2
12 ; cotB  5
  
 82  122¬ 162
67. cscA  8
4 or 2; secA 
 8 or 
2
3
3 ; 64  144¬ 256
4
3

3
4 
cotA   ; cscB  8 or 
4 or 3
2
3
3 ;
208¬ 256
4
3 Since 208  256, the measures cannot be the

secB  8 4 or 
4 or 2; cotB  4

3
3
sides of a right triangle because they do not
3
68. cscA  4 or 2 ; secA  4 or 2
; satisfy the Pythagorean Theorem. The measures
22  22 do not form a Pythagorean triple.
2 
2 4
cotA   or 1; cscB   or 2 ; 76. Rewrite 4 : 11 as 4x : 11x and use those values for
22  22
4 2 
2 the number of minutes of commercials and actual
secB   or 2 ; cotB   or 1
22  22 show.
4x  11x¬ 30
15x¬ 30
Page 370 Maintain Your Skills x¬ 2
69. b  3a 4x  4(2) or 8
b  3(4) or 43 8 minutes are spent on commercials.
c  2a 77. m15  117 vertical 
c  2(4) or 8 78. m7  30 corresponding 
70. b  3a 79. m3  30  180 linear pair
3  3a m3  150
3  a
3 80. m12  117  180 linear pair
3  
3
    a m12  63
3 3  81. m11  m12 alternate interior 
3a
m11  63
c  2a
c  23  82. m4  30  180 linear pair
m4  150
71. c¬ 2a
5¬ 2a
2.5¬ a
b¬ 3 a 7-5 Angles of Elevation and
b¬ 3 (2.5) or 2.53

Depression
72. a2  b2¬ c2
42  52¬ 62
16  25¬ 36 Page 373 Check for Understanding
41¬ 36 1. Sample answer: ABC
Since 41  36, the measures cannot be the sides A
B
of a right triangle because they do not satisfy the
Pythagorean Theorem. The measures do not form
a Pythagorean triple.
C

Chapter 7 212
2. Sample answer: An angle of elevation is called 7. The angle of depression between the top of the
that because the angle formed by a horizontal line tower and the horizontal is 12°. Find the distance
and a segment joining two endpoints rises above along the ground from the plane to the tower.
the horizontal line. D A
12

3. The angle of depression is FPB and the angle of 150 ft


elevation is TBP. C B
4. B Because D A
 and C B are horizontal, they are
parallel. Thus, DAC ACB. So mACB  12.
150
tan12°¬  CB
10,000 ft
CBtan12°¬ 150
plane x
150
A C CB¬  tan 12°
CB¬ 706
10,000 ft The plane is about 706 feet from the base of the
tower.

50 mi

AC is 50 miles or 50(5280)  264,000 feet. Pages 374–376 Practice and Apply


Let x represent mCAB. 8. P
D
CB 7
12.5

tanx°¬ 
AC
10,0
00
tanx°¬ 
264,000 75 m
x¬ tan1 10,0
264,0 00 
00

x¬ 2.2
C B A
The angle of elevation should be about 2.2°.
5. sun The parasailer is at P and the boats are at A and
B. PCB and PCA are right triangles. The
A
distance between the boats is AB or AC  BC.
Because PD and CA are horizontal lines, they are
7.6 m parallel. Thus, DPA PAC and DPB
PBC because they are alternate interior angles.
This means that mPAC  7 and mPBC  12.5.
x
75
B C tan7°¬ AC

18.2 m
ACtan7°¬ 75
Let x represent mACB. 75
AC¬ tan7°
AB
tanx°¬ 
BC
 AC¬ 610.83
75
tanx°¬ 7.6 tan12.5°¬  
18.2 BC
BCtan12.5°¬ 75
x¬ tan1 .2 
7.6
18 75
BC¬ tan 1
2.5°
x¬ 22.7
The angle of elevation is 22.7°. BC¬ 338.30
AC  BC  610.83  338.30 or about 273.
6. The angle of depression between the ship and the
The distance between the boats is about 273 m.
horizontal is 13.25°. Find the length along the
ocean floor. 9. The angle of elevation between the green and the
horizontal is 12°.
A B
ship 13.25

Let d represent the distance from the tee to the


40 m hole.
diver 36
sin12°¬  d
D C
dsin12°¬ 36
36
The ocean floor and the horizontal level with the d¬  
sin 12°
ship are parallel, creating congruent alternate
d¬ 173.2
interior angles BAC and ACD.
40 The distance is about 173.2 yards.
tan13.25°¬  DC
DC tan13.25°¬ 40
40
DC¬  tan13.25°
DC¬ 169.9
The diver must walk about 169.9 meters along
the ocean floor.

213 Chapter 7
60
10. H G tan8°¬ 
AF
500 m AFtan8°¬ 60
P
Q 60
11 km AF¬  
tan8°
AF¬ 426.92
The angle of depression between the horizontal 60
and the flight of the helicopter H to the landing tan11°¬ 
AM
pad P is GHP. AMtan11°¬ 60
60
The ground and the horizontal from the helicopter AM¬¬ 
tan11°
are parallel. Therefore, mGHP  mHPQ since AM¬ 308.67
they are alternate interior angles. Let AF  AM  426.92  308.67 or about 118.2 yards.
x  mHPQ. The merry-go-round and the Ferris wheel are
0.
5
tanx°¬ 
11 about 76.4 yards apart.
x¬ tan1
11 
0.
5 vertical rise140
horizontal distance  2000
14.  
x¬ 2.6  0.07 or 7%
The angle of depression is about 2.6°. The grade of the highway is 7%.
11. A D 15. A
300 yd
27.6 yd
x C 140 ft x
B C
B 2000 ft
The angle of depression between the horizontal
Let x represent mACB.
and the sledding run is DAC. 140
The horizontals from the top of the run and the tanx° ¬2000
bottom of the run are parallel. Therefore, x ¬tan1140
2000 
mDAC  mACB since they are alternate
interior angles. Let x  mACB. x ¬4.00
27
.6 The angle of elevation is about 4°.
sinx°¬ 
300 16. A
x¬ sin1
300 
27
.6

x¬ 5.3
The angle of depression is about 5.3°. 1100 ft
12. A
24.4

635 ft B C
369.39 ft 1100
sin24.4° ¬ 
AC
x ACsin24.4° ¬1100
B C
1100
AC ¬
sin 2
4.4°
Let x represent mACB.
369
.39 AC ¬2663¬
sinx°¬ 635 The ski run is about 2663 feet.
x¬ sin1 635 
369
.39
17. S
x¬ 35.6
The angle of elevation (incline) is about 35.6°.
13. R 175 ft
B
8

11
K E
60 yd 6 ft
F 200 ft G
A M F 
KE and F
G are parallel, so KF  EG. Since SG is
175 feet and EG is 6 feet, SE is 169 feet. Let
RAM and RAF are right triangles. The
x represent mSKE.
distance between the merry-go-round M and the
16
9
Ferris wheel F is MF or AF  AM. tanx° ¬
200
Because RB and AF are horizontal lines, they are 16
x ¬tan1 9
200 
parallel. Thus, BRF RFA and BRM
RMA because they are alternate interior angles. x ¬40.2¬
This means that mRFA  8 and mRMA  11. The angle of elevation is about 40.2°.

Chapter 7 214
18. S The raised end of the treadmill is about 8.3 inches
off the floor.
22. R
123 ft

K 37
E F
P
6 ft
G RP  10 and RF  48. Let x  mRFP.
F
RP
sinx° ¬
 and F
KE G are parallel, so KF  EG. Since SG is RF
10
123 feet and EG is 6 feet, SE is 117 feet. sinx° ¬
48
11
7
tan37° ¬ 10
x ¬sin1
48 ¬
KE 
KEtan37° ¬117 x ¬12¬
11
7
KE ¬
tan 37° ¬ The incline of the treadmill is about 12°.
KE ¬155.3¬ 23. Let xi represent the rise at each stage i, i  1, 2,
KE  FG, so the distance between kirk and the 3, 4, 5. The length of the treadmill is 48 inches.
geyser is about 155.3 feet. Suppose the incline at the beginning of the exam
19. D is 10°.
x1
Stage 1: sin10°  
48
x1  48sin10°
x1  8.3351
60
30
x
Stage 2: sin12°  
2

C A B 48
200 ft x2  48sin12°
x2  9.9798
DC
tan60° ¬ x
AC Stage 3: sin14°  
3

ACtan60° ¬DC 48
x3  48sin14°
DC
tan30° ¬
BC x3  11.6123
ACta
n60° x
tan30° ¬ BC Stage 4: sin16°  
4
48
BCtan30° ¬ACtan60° x4  48sin16°
AC ta
n 60° x4  13.2306
BC ¬ tan 30° x
Stage 5: sin18°  
5


tan60°
AC  200 ¬AC tan 30°  48
x5  48sin18°
AC  200 ¬AC(3) x5  14.8328
200 ¬3AC  AC x2  x1  9.9798  8.3351
200 ¬2AC  1.6447
100 ¬AC x3  x2  11.6123  9.9798
BC ¬AC  200  1.6325
¬100  200 or 300 x4  x3  13.2306  11.6123
The observers are 100 feet and 300 feet from the  1.6183
base of the tree. x5  x4  14.8328  13.2306
20. Let x represent the distance from the spotlight to  1.6026
the base of the cloud formation. No, the end of the treadmill does not rise the same
x
tan62.7° ¬ 8
3 distance each time. The changes in the rise of the
83tan62.7° ¬x treadmill between stages are only approximately
160.8 ¬x the same, about 1.6 inches.
The ceiling is about 160.8  1.5 or 162.3 meters. 24. D
21. R

15.85

25.6

P F C B 0.5 km A
RF  48 and mRFP  10. Find RP.
RP
sin10° ¬
RF
D
tan25.6° ¬ C
BC
RP
sin10° ¬
48 BCtan25.6° ¬DC
48sin10° ¬RP¬ D
tan15.85° ¬ C
AC
8.3 ¬RP¬

215 Chapter 7
ACtan15.85° ¬DC tan
AD ¬DC 52°
tan 33° 
ACtan15.85° ¬BCtan25.6°
tan
7  DC ¬DC 52°
tan 33° 
AC ¬BC tan 25.6°
.85° 
tan 15
Change AC to meters.
tan
7 ¬DC 52°
tan 33°   DC
0.5 km  500 m tan
7 ¬DC
tan 33°  1
52°


tan 25.6°
500  BC ¬BC  tan 15 .85°  DC  
tan52°
7
tan 33°  1
¬

t a n 25 .6°
500 ¬BC  tan 15 .85°  BC 
 
7

tan 25.6°
500 ¬BC  tan 15 .85°  1  BD ¬ 
tan
 52°
1
tan52°
tan 33°
500
BC ¬  BD ¬3.0

tan 25.6°
tan 15.85°  1
The balloon is about 3.0 miles above the ground.
BC ¬727 27. Answers should include the following.
DC ¬BCtan25.6° • Pilots use angles of elevation when they are
¬348 ascending and angles of depression when
Ayers Rock is about 348 meters high. descending.
25. E A • Angles of elevation are formed when a person
16
looks upward and angles of depression are
29
formed when a person looks downward.
3 mi
28. B; let x represent the distance from the ship to
the foot of the tower.
D C n B 12
tan25° ¬ 0
x
f xtan25° ¬120
Find f  n. 120
x ¬
tan
A
E  is parallel to D
B
, so EAD ADB and 25°
EAC ACB because they are alternate x ¬257.3
interior angles. Then mADB  16 and The ship is about 257.3 meters from the tower.
mACB  29. y
 ¬
x
29. A; 
28 1
6
tan16° ¬3f 16y ¬28x
f tan16° ¬3 161
2  ¬28x

3
f ¬tan16° 8 ¬28x
3 8
tan29° ¬ n
 
28 ¬x
ntan29° ¬3 2
 ¬x
7
3
n ¬
tan29°
3 3
f  n ¬tan16°  
tan29°
Page 376 Maintain Your Skills
f  n ¬5.1
30. cosA ¬0.6717
The crater is about 5.1 miles across.
A ¬cos1(0.6717)
26. B A ¬47.8
31. sinB ¬0.5127
B ¬sin1(0.5127)
B ¬30.8
32. tanC ¬2.1758
C ¬tan1(2.1758)
A
33
52

C C ¬65.3
D 33. cosD ¬0.3421
7 mi
D ¬cos1(0.3421)
BD
tan33° ¬ D ¬70.0
AD
ADtan33° ¬BD 34. sinE ¬0.1455
BD E ¬sin1(0.1455)
tan52° ¬
DC E ¬8.4
DCtan52° ¬BD 35. tanF ¬0.3541
ADtan33° ¬DCtan52° F ¬tan1(0.3541)
F ¬19.5

Chapter 7 216
28
 7
36. 12 ¬x2
 47. 15 ¬ x
12
  ¬x 28x ¬15(7)
2
28x ¬105
12
2
 x x ¬3.75
2 2 
122 x 3
 
2
¬x 48. 
40 ¬
26
62 ¬x 26x ¬40(3)
y ¬x 26x ¬120
60
y ¬62  x ¬
13
37. x  143 
y  2(14) or 28
38. 20  2y 7-6 The Law of Sines
10  y
x ¬y3 
x ¬103  Pages 380–381 Check for Understanding
39. Let  be the length of the model. 1. Felipe; Makayla is using the definition of the sine
length of model wingspan of model ratio for a right triangle, but this is not a right
length of plane  
 wingspan of plane triangle.
 36
78 ¬
90 2. F
90 ¬78(36)
90 ¬2808
e d
 ¬31.2
The length of the model is 31.2 cm.
40a. 1 2 D E
f
40b. AAS
Sample answer: Let mD  65, mE  73, and
X
40c. F  G X sin 65°
d  15. Then  
15 is the fixed ratio or scale
40d. CPCTC
factor for the Law of Sines extended proportion.
40e. 4 3
sin 65° sin 73°
The length of e is found by using  15 
 
40f. Isosceles Triangle Theorem e.
x  The mF is found by evaluating 180  (mD 
41. 6 ¬ 35

42 mE). In this problem mF  42. The length of f
42x ¬6(35) sin 65° sin 42°
is found by using  15 
 
42x ¬210 f.
x ¬5 3. In one case you need the measures of two sides
and the measure of an angle opposite one of the
3
x ¬
5
42. 45 sides. In the other case you need the measures of
3(45) ¬5x two angles and the measure of a side.
135 ¬5x sinX sin Y
x ¬y
4. 
27 ¬x
sin 37°
 sin 68°
 ¬
y
12 24
17 ¬ x
43.    3
12x ¬17(24) ysin37° ¬3sin68°
12x ¬408 3 sin68°
y ¬sin 37°
x ¬34
y ¬4.6
24
 x
36 ¬ 15
44.  5. mX  mY  mZ ¬180
24(15) ¬36x 57  mY  72 ¬180
360 ¬36x mY ¬51
10 ¬x X
sin sin Y
 x ¬y
12 48
13 ¬ x
45.    sin 57° sin 5 1°
 x ¬ 
12.1
12x ¬13(48)
12.1sin57° ¬xsin51°
12x ¬624
12.1 si
n 57°
sin 51° ¬x
x ¬52 
x 5 13.1 ¬x
46. 
18 ¬8
8x ¬18(5)
8x ¬90
x ¬11.25

217 Chapter 7
Y
sin sin Z
6.  y ¬z 10. mP  mQ  mR ¬180
sin
Y sin 37° 33  mQ  58 ¬180
7 ¬ 11
  
mQ  91 ¬180
11sinY ¬7sin37° mQ ¬89
7sin
37° sinP sinQ
sinY ¬ 11  p ¬q
7sin
Y ¬sin1 37°
11 
sin 33° sin 89°
 p ¬ 22

Y ¬23°
22sin33° ¬psin89°
sinY sin Z
7.  y ¬¬z 22 sin
33°
sin 89° ¬p

sin 92° sinZ
17 ¬ 14
  
12.0 ¬p
14sin92° ¬17sinZ
R
sin sin Q
14sin92°
¬sinZ
 r ¬q
17 sin 58° sin 89°
sin114sin 92°  r ¬ 
 ¬Z
17  22
22sin58° ¬rsin89°
55° ¬Z
22 sin
58°
sin 89° ¬r

8. mP  mQ  mR ¬180
mP  59  66 ¬180 18.7 ¬r
mP  125 ¬180 sinP sin Q
mP ¬55
11.  p ¬q
sin 1
20° sinQ
sinP sin Q  ¬
p ¬q
 28 22
sin 55° sin 59° 22sin120° ¬28sinQ
72 ¬ q
   
22sin120°

28 ¬sinQ
qsin55° ¬72sin59°
72 sin
59°
22si
sin1 n
 ¬Q120°

q ¬sin 55°
28
43° ¬Q
q ¬75.3
mP  mQ  mR ¬180
sinP sin R
p ¬r 120  43  mR ¬180

sin 55° sin 66° 163  mR ¬180
72 ¬ r
   
mR ¬17
rsin55° ¬72sin66° P
sin sin R
p ¬r

72 sin
66°
r ¬sin 55° sin 1
20° sin 17°

28 ¬ r
r ¬80.3
rsin120° ¬28sin17°
sinP sin R
p ¬r
9.  28 sin
r ¬ 17°
sin 120°
sin 1
05° sinR

32 ¬11 r ¬9.5
11sin105° ¬32sinR 12. mP  mQ  mR ¬180
11sin105°
¬sinR 50  65  mR ¬180
32
115  mR ¬180
sin111sin
 ¬R
32
105°
 mR ¬65
19° ¬R sinP sin Q
 p ¬q
mP  mQ  mR ¬180 sin 50° sin 65°
12 ¬ q
105  mQ  19 ¬180    
mQ  124 ¬180 qsin50° ¬12sin65°
mQ ¬56 12 sin
65°
q ¬sin 50°
P
sin sinQ
 p ¬q q ¬14.2
sin 1
05° sin 56°

32 ¬ q sinP sin R
 p ¬r
qsin105° ¬32sin56°
sin 50° sin 65°
12 ¬ r
32 sin56°    
q ¬sin 105°
rsin50° ¬12sin65°
q ¬27.5
12 sin
65°
r ¬sin 50°
r ¬14.2

Chapter 7 218
Q
sin sin R sinK sinM
13.  q ¬r 18.  k ¬m
sin 110.7° sin
R sin 73° sin
M

17.2 ¬9.8

18.2 ¬ 10.5
 
9.8sin110.7° ¬17.2sinR 10.5sin73° ¬18.2sinM
9.8sin 110.7° 10.5 sin 73°

17.2 ¬sinR 
18.2 ¬sinM
9.8si
sin1
 ¬Rn
17.2
110.7°

10.5
sin1 s
 ¬Mi
18.2
n 73°

32° ¬R 33° ¬M
mP  mQ  mR ¬180 mM ¬33
mP  110.7  32 ¬180 sinK sinM
mP  142.7 ¬180
19.  k ¬m
sin 96° sin M
10 ¬ 4.8
mP ¬37   
sinP sin Q 4.8sin96° ¬10sinM
 p ¬q
4.8sin96°
sin 37° sin 11
0.7°  ¬sinM
 p ¬ 17.2 10
17.2sin37° ¬psin110.7°
4.8s
sin1 i
 ¬M
10
n96°

17.2 sin 37° 29° ¬M
sin 110.7° ¬p
 
mM ¬29
11.1 ¬p
14. 
AD BC, so DAC BCA because they are 20. mK  mL  mM ¬180
alternate interior angles. Thus, mDAC  88. 31  88  mM ¬180
sin 32° sin88° 119  mM ¬180
  ¬
6 D
C mM ¬61
DCsin32° ¬6sin88° sinL sinM
 ¬m

6 sin88°
DC ¬ sin 32° sin 88° sin 61°
DC ¬11.3
  ¬ 
5.4
5.4sin88° ¬sin61°
ABCD is a parallelogram, so A D
 B
C and
5.4 si
n 88°
sin 61° ¬
C
D  A B. The perimeter of ABCD is 
2(6)  2(11.3), or 34.6 units. 6.2 ¬
15. mA  mB  mC ¬180 sinM sin L
m ¬
21. 
55  mB  62 ¬180
mB  117 ¬180 sin 59° sinL
14.8 ¬ 8.3
  
mB ¬63
8.3sin59° ¬14.8sinL
sin 63° sin 62°
240 ¬ AB
    8.3 sin 59°
 
14.8 ¬sinL
ABsin63° ¬240sin62°
240 si
n 62° sin1 8.3 sin
 ¬L
14.8
59°

AB ¬ sin 63°
29° ¬L
AB ¬237.8 feet mL ¬29
sinX sin Y
x ¬y
22. 
sin 41° sin 71°
Pages 381–382 Practice and Apply x ¬
 
7.4
K
sin sinL
k ¬ 
16.   7.4sin41° ¬xsin71°
sin 63° sin 45° 7.4 si
n 41°
sin 71° ¬x

 k ¬ 
22
22sin63° ¬ksin45° 5.1 ¬x
22 sin
63° mW  mX  mY ¬180
sin 45° ¬k

mW  41  71 ¬180
27.7 ¬k
mW  112 ¬180
sinK sin L mW ¬68
17.  k ¬
W
sin sin Y
 w ¬y
sin 70° sin 52°
3.2 ¬ 
   
sin 68° sin 71°
 sin70° ¬3.2sin52°  w ¬ 
7.4
3.2 si
n 52°
 ¬ sin 70° 7.4sin68° ¬wsin71°
7.4 si
n 68°
 ¬2.7 
sin 71° ¬w
7.3 ¬w

219 Chapter 7
sinY sin X sinW sin X
23.  y ¬x  w ¬x
sin 96° sinX sin 38° sin27°
23.7 ¬ 10.3 8.5 ¬ x
      
10.3sin96° ¬23.7sinX xsin38° ¬8.5sin27°
10.3 si
n 96° 8.5 si
n 27°

23.7 ¬sinX x ¬ sin 38°
sin110.3 si n 96° x ¬6.3
 ¬X
23.7  sinW sin Y
25.6° ¬X  w ¬y
sin 38° sin115°
mW  mX  mY ¬180 
8.5 ¬
  y 
mW  25.6  96 ¬180 ysin38° ¬8.5sin115°
mW  121.6 ¬180 8.5 sin
115°
mW ¬58.4 y ¬ sin 38°
W
sin sin Y y ¬12.5
 w ¬y
sin5
8.4° sin 96° 27. mW  mX  mY ¬180
 ¬ 
w 23.7 36  mX  62 ¬180
23.7sin58.4° ¬wsin96° mX  98 ¬180
23.7 sin
 58.4°
¬w mX ¬82
sin 96°
sinW sin X
20.3 ¬w  w ¬x
24. mW  mX  mY ¬180 sin 36° sin 82°
3.1 ¬ x
   
52  25  mY ¬180
xsin36° ¬3.1sin82°
77  mY ¬180
3.1 si
n 82°
mY ¬103 x ¬ sin 36°
W
sin sin Y x ¬5.2
w ¬y

sin 52° sin 1
03° sinW sin Y
w ¬y

w ¬

15.6
15.6 sin52° ¬wsin103° sin 36° sin 62°
3.1 ¬ y
   
15.6 si
n 52°
sin 103° ¬w
 ysin36° ¬3.1sin62°
12.6 ¬w 3.1 si
n 62°
y ¬ sin 36°
sinX sin Y y ¬4.7
x ¬y

W
sin sin Y
w ¬y
sin 25° sin 1
03° 28. 
x ¬

15.6
sin 1
07° sinY
15.6sin25° ¬x sin103° 
30 ¬9.5
15.6 si
n 25° 9.5sin107° ¬30sinY
sin 103° ¬x

9.5sin 107°
6.8 ¬x 
30 ¬sinY
X
sin sin Y 9.5si
sin1
 ¬Yn 107°

x ¬y
25.  30
sin
X sin 1
12° 17.6° ¬Y
20 ¬ 56
 
56sinX ¬20sin112° mW  mX  mY ¬180
20sin
112° 107  mX  17.6 ¬180
sinX ¬ 56 mX  124.6 ¬180
20sin
X ¬sin1 112° mX ¬55.4
56 
W
sin sinX
X ¬19.3°  w ¬  x
mW  mX  mY ¬180 sin 1
 07° sin5
¬ 5.4°
30 x
mW  19.3  112 ¬180
xsin107° ¬30sin55.4°
mW  131.3 ¬180
30 sin
55.4°
mW ¬48.7 x ¬sin 107°
W
sin sin Y x ¬25.8
 w ¬y
sinW sin X
 w ¬

sin4
8.7° sin 1
12° 29.
 w ¬ 56 x
sin 88° sin X
21 ¬ 16
56sin48.7° ¬wsin112°   
56 sin
48.7° 16sin88° ¬21sinX
sin 112° ¬w

16sin88°
45.4 ¬w 
21 ¬sinX
26. mW  mX  mY ¬180 16s
sin1 in
 ¬X
21
88°

38  mX  115 ¬180
49.6° ¬X
mX  153 ¬180
mX ¬27

Chapter 7 220
mW  mX  mY ¬180 33. P
88  49.6  mY ¬180
137.6  mY ¬180 b a
mY ¬42.4
sinW sin Y
 w ¬y A
43
48

B
sin 88° sin42.4° 20 mi
21 ¬
   y
mA  mB  mP ¬180
y sin88° ¬21sin42.4°
43  48  mP ¬180
21 sin
42.4°
y ¬ sin 88° 91  mP ¬180
y ¬14.2 mP ¬89
sinP sinA
20 ¬ a
30. Let x be the measure of each base angle.  
x  x  44 ¬180 sin 89° sin 43°
20 ¬ a
   
2x  44 ¬180
2x ¬136 asin89° ¬20sin43°.
20 sin
43°
x ¬68 a ¬sin 89°
Let y be the measure of each of the congruent a ¬13.6
sides. sinP sinB
20 ¬ b
 
sin 44° sin 68°
46 ¬ y
    sin 89° sin 48°
20 ¬ b
   
y sin44° ¬46sin68° bsin89° ¬20sin48°
46 sin
68°
y ¬¬sin 44° b ¬20 sin
48°
sin 89°
y ¬61.4 b ¬14.9
The perimeter of the triangle is about The first station is about 14.9 miles from the
46  61.4  61.4 or 168.8 cm. plane, and the second station is about 13.6 miles
sin 28° sin 40° from the plane.
12 ¬ AB
31.    
34. T
ABsin28° ¬12sin40°
12 sin40°
AB ¬ 
sin28°
AB ¬16.43

AB 
DC and 
AD 
BC, so the perimeter of ABCD
is 2(16.43)  2(12) or 56.9 units.
32. Z

b d a

x y

78
44
80
85

Y 26 ft X A B
315 ft
mX  mY  mZ ¬180 mA  mB  mT ¬180
44  78  mZ ¬180 80  85  mT ¬180
122  mZ ¬180 165  mT ¬180
mZ ¬58 mT ¬15
sin 44° sin 58°
 x ¬  sinT sinA
315 ¬ a
26  
26sin44° ¬xsin58° sin 15° sin 80°
315 ¬ a
   
26 sin
44°
sin 58° ¬x

asin15° ¬315sin80°
21.3 ¬x 315 si
a ¬ n 80°
sin 15°
sin 78° sin 58°
 y ¬ 26
 a ¬1198.6
26sin78° ¬ysin58° B
sin sin 90°
 d ¬a
26 sin
78°
sin 58° ¬y
 sin 85° sin 90°
 d ¬
1198.6
30 ¬y 1198.6sin85° ¬dsin90°
The length of fence needed is about 30  21.3  1198.6sin 85°

sin 90° ¬d
26 or 77.3 feet.
1194.0 ¬d
The distance across the gorge is about 1194 feet.

221 Chapter 7
35. The plot of land is an isosceles triangle. Let x 39. See art for Exercise 38.
represent the measure of one of the base angles. B
sin sinC
b ¬ 60
 
x  x  85 ¬180
sin124° sin 29°
2x  85 ¬180 
b ¬ 
60
2x ¬95 60sin124° ¬bsin29°
x ¬47.5 60 sin124°
sin 29° ¬b

Let y represent the length of the base of the
triangle. 102.6 ¬b
sin 4
7.5° sin 85° 60  a  60  56.2 or 116.2

160 ¬ y

116.2  102.6  13.6
y sin47.5° ¬160sin85° Keisha added about 13.6 miles to the flight.
160 si
n85° A
sin C
sin
y ¬sin 47.5° 40. Yes; in right ABC,  a  c where C is the

y ¬216 asi
nC
right angle. Then sinA   c . Since mC ¬90,
The perimeter of the property is about 160  asin90°
160  216 or 536 feet, so 536 feet of fencing then sinA   c . Since sin90°  1, then
material is needed. sinA  ac, which is the definition of the sine ratio.
36. P 41. Sample answer: Triangles are used to determine
distances in space. Answers should include the
j k following.
• The VLA is one of the world’s premier
40
27
astronomical radio observatories. It is used to
K 1433 m J make pictures from the radio waves emitted by
mK  mJ  mP ¬180 astronomical objects.
40  27  mP ¬180 • Triangles are used in the construction of the
67  mP ¬180 antennas.
mP ¬113 42. mX  mY  mZ ¬180
sin
P sinJ 48  112  mZ ¬180
1433 ¬ j
 
sin 1
13° sin 27° 160  mZ ¬180
1433 ¬ j
   mZ ¬20
jsin113° ¬1433sin27° sinX sin Y
x ¬y

1433s in 27°
j ¬ sin 48° sin1 12°
12 ¬
sin 113°   
y
j ¬706.8 ysin48° ¬12sin112°
Kayla and Paige are about 706.8 meters apart. 12 sin112°
y ¬ sin 48°
37. See art for Exercise 36.
y ¬15.0
sin
P sin K
 3 ¬k sinX sin Z
x ¬z
143 
sin 1
13° sin 40°
1433 ¬ k
   sin 48° sin 20°
12 ¬ z
   
ksin113° ¬1433sin40° zsin48° ¬12sin20°
1433s in 40°
k ¬ sin 113° 12 sin
z ¬ 20°
sin 48°
k ¬1000.7 z ¬5.5
Jenna and Paige are about 1000.7 meters apart. 9  8 77 31
43. A; Metropolis Grill:  4 4  7.75

38. b A 10  8  35 26
C Le Circus:  4
 4  6.5
27

8  9 46 27
Aquavent:  4 4  6.75

a 60 mi 7  9 47 2
7
124
Del Blanco’s:  4 4  6.75
B Metropolis Grill has the best average rating of the
four restaurant choices.
mA  mB  mC ¬180
27  124  mC ¬180
151  mC ¬180
mC ¬29 Page 383 Maintain Your Skills
A
sin sinC 44. Find x so that the angle of elevation is 73.5°.
 a ¬60 6 1
sin 27° sin 29°
tan73.5° ¬ x
 a ¬ 
60 xtan73.5° ¬7
60sin27° ¬asin29° 7
x ¬
tan73.5°
60 sin
27°
sin 29° ¬a

x ¬2.07
56.2 ¬a The overhang should be about 2.07 feet long.
Keisha must fly about 56.2 miles.

Chapter 7 222
c2  a
2  b2 102  7
2  82
45. Let y represent the amount of the window that 53.  2ab ¬
2(7)(8)
will get direct sunlight. 100  4
9  64
7y ¬ 112
tan26.5° ¬ 
2.07 13 13
¬ 12 ¬
1 
2.07tan26.5° ¬7  y 112
y ¬7  2.07tan26.5° c2  a
2  b2 62  4
2  92
54.  2ab ¬ 2(4)(9)
y ¬5.97 36  16  81
¬ 
72
About 5.97 feet of the window will get direct
6
1 6
1
sunlight. ¬
72 ¬ 72


cosJ  k
j j
46. sinJ   tanJ   c2  a
55. 
2  b2 102  5
¬
2  82
k 2ab 2(5)(8)
8 15 8
sinJ   cosJ   tanJ   100  2
5  64 1
1
17 17 15 ¬ 80 ¬
80
sinJ  0.47 cos J  0.88 tan J  0.53
c2  a
2  b2 132  1
62  42
56.  ¬
sinL  k tanL  j
j 2ab 2(16)(4)
cosL  
k
169  256  16
15 8 15 ¬ 
128
sinL  17
 cosL  
17 tanL  8
1
03 10
3
sinL  0.88 cos L  0.47 tanL  1.88 ¬
128 ¬ 128


cosJ  k
j j c2  a
2  b2 92  32
 102
47. sinJ   tanJ   57.  ¬
k  2ab 2(3)(10)
20 21 20 81  9
 100
sinJ  
29
 cosJ  
29 tanJ  
21
 ¬ 60
sinJ  0.69 cos J  0.72 tan J  0.95 2
¬ 8 7
60 ¬ 15
sinL  k tanL  j
j
cosL   c2  a
2  b2 112  5
2  72
k 58.  ¬
21 20 21 2ab 2(5)(7)
sinL    cosL    tanL   
29 29 20 121  2
5  49 47
¬ 70 ¬
70
sinL  0.72 cos L  0.69 tanL  1.05
cosJ  k
j j
48. sinJ   tanJ  
k 
sinJ  12 12
cosJ    
3
tanJ  12
 Page 383 Practice Quiz 2
24 24 123 16
 
1. tanx° ¬
10
sinJ  1
2
 cosJ  2
3
tanJ  3
3
16
x ¬tan1
10 

sinJ  0.50 cos J  0.87 tan J  0.58
x ¬58.0
sinL  k tanL  j
j
cosL   x
k 2. cos17° ¬
9.7
12 
3 12 12 
3
sinL   
24 cosL  24
 tanL   
12 9.7cos17° ¬x
sinL   
3
cosL  1 tanL  3
 9.3 ¬x
2 2 32
sinL  0.87 cos L  0.50 tanL  1.73 3. cos53° ¬x
xcos53° ¬32
cosJ  k
j j
49. sinJ   tanJ  
k 32
x ¬ 
7 
2 7 
2 7

2 cos 53°
sinJ   cosJ    tanJ   x ¬53.2
14 14 7
2
 
2 
2
sinJ   2 cosJ  2 tanJ  1.00 4.
5m
sinJ  0.71 cos J  0.71
sinL  k tanL  j
j
cosL  
k
7
2 7 
2 7

2
sinL   cosL   tanL   15

14 14 
72 15


2  
2
sinL  2 cosL   2 tanL  1.00
sinL  0.71 cos L  0.71 500 m
50. m1  54  120 Exterior Angle Theorem
m1  66
51. m2  54 alternate interior 
52. m2  m3  36 ¬180 Angle Sum Theorem
54  m3  36 ¬180 x x
m3  90 ¬180 x
tan15° ¬
500 
5
m3 ¬90
495tan15° ¬x
132.6 ¬x
The distance is about 132.6 meters.

223 Chapter 7
D
sin sinF
5. EF ¬DE 5. a2 ¬b2  c2  2bccosA
sin
D sin 82° a2 ¬1072  942  2(107)(94)cos105°
8 ¬ 12
  
a2 ¬20,285  20,116 cos105°
12sinD ¬8sin82° a ¬20,28
5 ,116
 20os105c°
8sin
82°
sinD ¬ 12 a ¬159.7
8sin
D ¬sin1 s2 ¬r2  t2  2rtcosS
12 
82° 6.
652 ¬332  562  2(33)(56)cos S
D ¬41°
4225 ¬4225  3696 cosS
mD  mE  mF ¬180 0 ¬3696 cosS
41  mE  82 ¬180 0 ¬cosS
mE  123 ¬180 S ¬cos1(0)
mE ¬57 S ¬90°
E
sin sinD r2 ¬s2  t2  2stcosR
DF ¬
7.
EF
sin5 7° sin 41° 2.22 ¬1.32  1.62  2(1.3)(1.6)cos R
 DF ¬8 4.84 ¬4.25  4.16 cosR
8sin57° ¬DF sin41° 0.59 ¬4.16 cosR
8 sin57° 0.5
9
4.16 ¬cosR

sin 41° ¬DF

10.2 ¬DF R ¬cos1
4.16 
0.5
9

R ¬98°
8. We know the measures of three sides (SSS), so
Page 384 Geometry Software Investigation: use the Law of Cosines.
The Ambiguous Case of the Law x2 ¬y2  z2  2yzcosX
of Sines 52 ¬102  132  2(10)(13)cos X
1. BD, AB, and mA 25 ¬269  260 cosX
2. Sample answer: There are two different triangles. 244 ¬260 cosX
2
44
260 ¬cosX
3. Sample answer: The results are the same. In each 
case, two triangles are possible. 2
X ¬cos1 44
260 
4. Sample answer: Circle B intersects 
AC at only one
point. See students’ work. X ¬20°
5. Yes; sample answer: There is no solution if circle X
sin sin Y
B does not intersect 
AC.
 x ¬y
sin 20°
 sin
 ¬ Y
5 10
10sin20° ¬5sinY
10sin20°
7-7 The Law of Cosines 
5
 ¬sinY

sin110sin20°
 ¬Y
5 
Pages 387–388 Check for Understanding 43° ¬Y
1. Sample answer: Use the Law of Cosines when you mX  mY  mZ ¬180
have all three sides given (SSS) or two sides and 20  43  mZ ¬180
the included angle (SAS). mZ ¬117
R Y 9. We know the measures of two sides and the
6 8 10 included angle (SAS), so use the Law of Cosines.
Q S X 38
Z 2 ¬k2  m2  2kmcosL
12 15 2 ¬202  242  2(20)(24)cos47°
2. If you have all three sides (SSS) or two sides and 2 ¬976  960 cos47°
the included angle (SAS) given, then use the Law  ¬976
 os47°
960 c
of Cosines. If two angles and one side (ASA or  ¬17.9
AAS) or two sides with angle opposite one of the sinL sin K
sides (SSA) are given, then use the Law of Sines.
  ¬k
sin 47° sinK
17.9 ¬ 20
3. If two angles and one side are given, then the   
Law of Cosines cannot be used. 20sin47° ¬17.9sin K
4. b2 ¬a2  c2  2accosB 20 sin47°
¬sinK
17.9
b2 ¬52  (2 )2  2(5)(2
)cos45°
b ¬27  102
2  cos45° sin120 sin
 ¬K
17.9
47°

b ¬ 27  102  c
os45° 55° ¬K
b ¬4.1 mK  mL  mM ¬180
55  47  mM ¬180
mM ¬78

Chapter 7 224
10. Let n, d, and q be the measures of the sides 16. e2 ¬f2  g2  2fgcosE
opposite N, D, and Q, respectively. 142 ¬192  322  2(19)(32)cos E
n  1 1 196 ¬1385  1216cos E
2 (10)  2 (24) or 17 mm

1189 ¬1216cos E
d  1
 1
2 (24)  2 (22) or 23 mm 11
89
1216 ¬cosE

q  1 1
2 (10)  2 (22) or 16 mm
 118
E ¬cos1
1216 
9
n2 ¬d2  q2  2dqcosN
E ¬12°
172 ¬232  162  2(23)(16)cos N
289 ¬785  736cos N 17. f 2 ¬e2  g2  2egcosF
496 ¬736cos N 1982 ¬3252  2082  2(325)(208)cos F
4
96 39,204 ¬148,889  135,200cos F
736 ¬cosN

109,685 ¬135,200cos F
4
N ¬cos1 96
736  109,685
 ¬cosF
135
,200
N ¬47.6° 109,685
F ¬cos1 200 
135,
q2 ¬n2  d2  2ndcosQ
162 ¬172  232  2(17)(23)cos Q F ¬36°
256 ¬818  782 cosQ 18. g2 ¬e2  f 2  2efcosG
562 ¬782 cosQ 102 ¬21.92  18.92  2(21.9)(18.9)cos G
5
62 100 ¬836.82  827.82cosG
782 ¬cosQ

736.82 ¬827.82cosG
5
Q ¬cos1 62
782  736.82
827.82 ¬cosG
 
Q ¬44.1° 736
G ¬cos1 .82
827.82 
mQ  mD  mN ¬180
44.1  mD  47.6 ¬180 G ¬27°
mD ¬88.3 H
sin sin F
19.  h ¬0f
sin
H sin 40°
8 ¬ 10
  
Pages 388–390 Practice and Apply 10sinH ¬8sin40°
11. u2 ¬t2  v2  2tvcosU 8sin
40°
sinH ¬
u2 ¬9.12  8.32  2(9.1)(8.3)cos32° 10
u2 ¬151.7  151.06 cos32° 8sin
H ¬sin1¬ 40°
10 
u ¬151.7
.06  151s32°co
H ¬31°
u ¬4.9
12. v2 ¬t2  u2  2tucosV mF  mG  mH ¬180
v2 ¬112  172  2(11)(17)cos78° 40  mG  31 ¬180
v2 ¬410  374 cos78° mG ¬109
v ¬410
 os78°
374 c G
sin F
sin
g ¬0 f
 
v ¬18.2 sin109° sin 40°
 ¬ 
13. t2 uu2  v2  2uvcosT g 10
t2 ¬112  172  2(11)(17)cos105° 10sin109° ¬gsin40°
t2 ¬410  374 cos105° 10 sin109°
sin 40° ¬g

t ¬410
 os105
374 c°
t ¬22.5 14.7 ¬g
14. t2 uu2  v2  2uvcosT 20. um2  q2  2mqcosP
p2
t2 ¬172  112  2(17)(11)cos59° p2 ¬112  102  2(11)(10)cos38°
t2 ¬410  374 cos59° p2 ¬221  220 cos38°
t ¬410
 os59°
374 c p ¬221
 os38°
220 c
t ¬14.7 p ¬6.9
sinM P
sin
f 2 ¬e2  g2  2egcosF m ¬0 p
15.   
8.32 ¬9.12  16.72  2(9.1)(16.7)cosF sin
M sin 38°
11 ¬ 6.9
  
68.89 ¬361.7  303.94 cosF
292.81 ¬303.94 cos F 6.9sin M ¬11sin38°
292.81 11sin
sinM ¬ 38°
303.94 ¬cosF
  6.9
292
F ¬cos1
11sin
M ¬sin1 38°

303.94 
.81
6.9
F ¬16° M ¬79°
mM  mP  mQ ¬180
79  38  mQ ¬180
mQ ¬63

225 Chapter 7
sinA sin B
21. b2 ¬c2  d2  2cdcosB  a ¬b
182 ¬152  112  2(15)(11)cos B sin 30° sinB
15 ¬ 19
  
324 ¬346  330cos B
22 ¬330cos B 19sin30° ¬15sinB
22 19sin30°
330 ¬cosB
  ¬sinB
15
B ¬cos12
2
330  sin119sin
 ¬B30°

15
B ¬86° 39° ¬B
sinB C
sin mA  mB  mC ¬180
b ¬0 c
 
sin 86° sinC 30  39  mC ¬180
18 ¬ 15
   mC ¬111
15sin86° ¬18sinC A
sin sin C
15sin86°
25.  a ¬c
 ¬sinC sin
18 a53° sin 2
 ¬  8°
14.9
sin115sin

18
86°

¬C 14.9sin53° ¬asin28°
56° ¬C 14.9 si
n 53°
sin 28° ¬a

mB  mC  mD ¬180
25.3 ¬a
86  56  mD ¬180
mD ¬38 mA  mB  mC ¬180
53  mB  28 ¬180
A
sin sinC
a ¬ c
22.   mB ¬99
sin 42° sin 77° B
sin sin C
b ¬c

 a ¬6
6sin42° ¬asin77° sin 99° sin 28°
 b ¬ 
14.9
6 sin42°
sin 77° ¬a
 14.9 sin 99° ¬b sin 28°
4.1 ¬a 14.9 si
n 99°
sin 28° ¬b

mA  mB  mC ¬180 31.3 ¬b
42  mB  77 ¬180
26. Find mDAB and mBCD.
mB ¬61
B
sin sinC DB2 ¬AB2  AD2  2(AB)(AD)cos(DAB)
b ¬ c
 
2
sin 61° sin 77° 723 ¬52  52  2(5)(5)cos(DAB)
 b ¬6 52
9
9 ¬50  50cos(DAB)

6sin61° ¬bsin77°
79
9 ¬50cos(DAB)
6 sin61° 
sin 77° ¬b

5.4 ¬b 
5  
1 79
0 9 ¬cos(DAB)
23. c2 ¬a2  b2  2abcosC
c2 ¬10.32  9.52  2(10.3)(9.5)cos37°
mDAB ¬cos1  1 
5
79
0 9   
c2 ¬196.34  195.7cos37° mDAB ¬100
c ¬ 196.3
4  19
5.7cos 
37° BD2 ¬BC2  DC2 
c ¬6.3 2(BC)(DC)cos(BCD)
2
 A
sin sin
a ¬ c
 C 723 ¬82  82  2(8)(8)cos(BCD)
sin
A sin 37° 529
9 ¬128  128cos(BCD)

10.3 ¬ 6.3
  
6.3sin A ¬10.3sin37° 623
9 ¬128cos(BCD)

10.3 si
n37°
sinA ¬ 6.3 1 
128  623
9 ¬cos(BCD)
 
A ¬sin1
10.3 si
n37°

A ¬80°
6.3
mBCD ¬cos1  1 
128
62
9
3
  
mBCD ¬57
mA  mB  mC ¬180
80  mB  37 ¬180 sinL sinM
27.   ¬m
mB ¬63 sin 23° sin
M
54 ¬ 44
  
24. a2 ¬b2  c2  2bccosA
152 ¬192  282  2(19)(28)cos A 44sin23° ¬54sinM
225 ¬1145  1064cos A 44sin23°
 ¬sinM
920 ¬1064cos A 54
920 sin144sin
 ¬M23°

1064 ¬cosA
 54
18.6° ¬M
A ¬cos1 92
 0
1064 
A ¬30°

Chapter 7 226
sinM sin N
mL  mM  mN ¬180  m ¬n
23  18.6  mN ¬180 sin 46° sin 79°
mN ¬138.4
 m ¬ 16

N
sin sin L 16sin46° ¬msin79°
 n ¬ 16 sin
46°
sin 79° ¬m
sin138.4° sin 23° 
 n ¬ 54

11.7 ¬m
54sin138.4° ¬nsin23° 31. 2 ¬m2  n2  2mncosL
54sin 1
38.4°

sin 23° ¬n 4232 ¬2562  2882  2(256)(288)cosL
91.8 ¬n 178,929 ¬148,480  147,456cos L
28. m2 ¬2  n2  2ncosM 30,449 ¬147,456cos L
30,449
,456 ¬cosL
18 ¬242  302  2(24)(30)cos M
2 
147
L ¬cos1 ,456 
324 ¬1476  1440cos M 30,449
147
1152 ¬1440cos M L ¬101.9°
11
52
1440  cosM
 sinL sinM
  ¬m
M ¬cos1 115
 2
1440  sin 10
 1.9° sin
¬ M
423 256
M ¬36.9° 256sin101.9° ¬423sin M
sinM sin L
 m ¬
256sin101.9°

423 ¬sinM
sin 3
6.9° sinL
 ¬ sin1
18 24 256sin
 ¬M
423
101.9°

24sin36.9° ¬18sinL 36.3° ¬M
24sin36.9°

18 ¬sinL mL  mM  mN ¬180
sin1 101.9  36.3  mN ¬180
24sin
 ¬L
18
36.9°
 mN ¬41.8
53.2° ¬L
32. m2 ¬2  n2  2ncosM
mL  mM  mN ¬180
m2 ¬6.32  6.72  2(6.3)(6.7)cos55°
53.2  36.9  mN ¬180
m2 ¬84.58  84.42cos55°
mN ¬89.9
m ¬ 84.58 84.42cos 
55°
29. 2 ¬m2  n2  2mncosL m ¬6.0
2 ¬192  282  2(19)(28)cos49°
sinM sin L
m ¬
2 ¬1145  1064cos49° 
 ¬ 1145 1064 
cos49° sin 55°
 sin
 ¬ L
 ¬21.1 6 6.3
6.3sin55° ¬6sinL
sinL sinM
 ¬m
 6.3sin55°

6 ¬sinL
sin 49° sin
M
21.1 ¬ 19
  
sin1
6.3si
 ¬L
6
n55°

19sin49° ¬21.1sin M
59.3° ¬L
19 sin49°
21.1 ¬sinM
 
mL  mM  mN ¬180
sin1 19 sin

21.1
49°
 ¬M 59.3  55  mN ¬180
mN ¬65.7
42.8° ¬M
33. m2 ¬2  n2  2ncosM
mL  mM  mN ¬180
m2 ¬52  102  2(5)(10)cos27°
49  42.8  mN ¬180
m2 ¬125  100cos27°
mN ¬88.2
m ¬ 125  100cos27°
30. mL  mM  mN ¬180 m ¬6.0
55  46  mN ¬180 sin
L sinM
mN ¬79
  ¬m
sin
L sin 27°
5 ¬ 6
sinL sin N   
  ¬n
sin 55° sin 79° 6sinL ¬5sin27°
  ¬ 16

5sin27°
16sin55° ¬ sin79° sinL ¬ 6
16 sin
55° L ¬sin1
5sin27°

sin 79° ¬
 6
13.4 ¬ L ¬22.2°
mL  mM  mN ¬180
22.2  27  mN ¬180
mN ¬130.8

227 Chapter 7
sinL sin N
34. 2 ¬m2  n2  2mncosL 37.   ¬n
142 ¬202  172  2(20)(17)cos L sin 51° sinN
40 ¬ 35
  
196 ¬689  680cos L
493 ¬680cos L 35sin51° ¬40sinN
4
93 35sin
 51°
¬sinN
680 ¬cosL
 40
L ¬cos1
680 
49
3 sin1
35sin
 ¬N
40
51°

L ¬43.5° 42.8° ¬N
sinL sinM mL  mM  mN ¬180
 ¬ m
  
sin 43.5° sin M 51  mM  42.8 ¬180
 ¬ mM ¬86.2
14 20
20sin43.5° ¬14sinM sinL sinM
  ¬m
20sin43.5° sin 51° sin 86.2°
 ¬sinM
40 ¬
14    m
sin120sin
 ¬M
14
43.5°
 msin51° ¬40sin86.2°
40 sin
86.2°
79.5° ¬M m ¬ sin 51°
mL  mM  mN ¬180 m ¬51.4
43.5  79.5  mN ¬180 38. 2 ¬m2  n2  2mncosL
mN ¬57.0 102 ¬112  122  2(11)(12)cos L
35. m2 ¬2  n2  2ncosM 100 ¬265  264cos L
m2 ¬142  212  2(14)(21)cos60° 165 ¬264cos L
m2 ¬637  588cos60° 1
65
264 ¬cosL

m ¬ 637 588co
s60°
m ¬18.5 L ¬cos1
264 
16
5
sin
L sinM
  ¬m L ¬51.3°
sinL sinM
 ¬m
sin
L sin 60° 
14 ¬ 18.5
  
sin 51.3° sin
M
18.5sinL ¬14sin60°  ¬
10 11
14 sin
60°
sinL ¬ 18.5 11sin51.3° ¬10sinM
11sin 51.3°
L ¬sin1 14 sin

18.5
60°
   ¬sinM
10
L ¬40.9° sin1
11sin

10
51.3°
¬M 
mL  mM  mN ¬180 59.1° ¬M
40.9  60  mN ¬180 mL  mM  mN ¬180
mN ¬79.1 51.3  59.1  mN ¬180
36. 2 ¬m2  n2  2mncosL mN ¬69.6
142 ¬152  162  2(15)(16)cos L 39. BC2 ¬BP2  PC2  2(BP)(PC)cos(BPC)
196 ¬481  480cos L 2 2
285 ¬480cos L BC2 ¬1
2  214   2  188 
 1
2
85
480 ¬cosL
 21
2  214 2  188 cos70°
 1

L ¬cos1
480 
28
5 BC2 ¬20,285  20,116 cos70°
L ¬53.6° BC ¬20,28
5 ,116
 20os70°
c
sinL sinM BC ¬115.8
 ¬ m
  
AB2 ¬AP2  BP2  2(AP)(BP)cos(APB)
sin 5
3.6° sin
M
 ¬ 2 2
14 15 AB2 ¬1
2  188   2  214 
 1
15sin53.6° ¬14sinM
15sin53.6° 21
2  188 2  214 cos(180  70)°
 1

14 ¬sinM
AB2 ¬20,285  20,116 cos110°
sin115sin
 53.6°
14 
¬M
AB ¬20,28
5 ,116
 20os110c°
59.6° ¬M AB ¬164.8
mL  mM  mN ¬180 AB  DC and AD  BC, so the perimeter of ABCD
53.6  59.6  mN ¬180 is 2(115.8)  2(164.8) or 561.2 units.
mN ¬66.8 40. QS2 ¬PQ2  PS2  2(PQ)(PS)cosP
QS2 ¬7212  7562  2(721)(756)cos58°
QS2 ¬1,091,377  1,090,152cos58°
QS ¬1,091
,377
 ,152
1,090os58°
c
QS ¬716.7

Chapter 7 228
sin(PQS) ¬sin P
  ¬
PS Q
S 242 ¬302  222  2(30)(22)cos A
sin(PQS)
5
¬sin 58°
¬ 576 ¬1384  1320cos A
7 6 716 .7
808 ¬1320cos A
716.7 sin(PQS) ¬756sin58° 808
1320 ¬cosA
756 sin 58° 
sin(PQS) ¬716
.7
A ¬cos1
1320 
80
8
mPQS ¬sin1 716.7 
756 si
n 58°
A ¬52.3°
mPQS ¬63.5 Adam has a greater angle, which is 52.3°.
QS2 ¬QR2  RS2  2(QR)(RS)cos R 43a. Pythagorean Theorem
716.72 ¬5472  5932  2(547)(593)cosR 43b. Substitution
513,658.89 ¬650,858  648,742 cos R 43c. Pythagorean Theorem
137,199.11 ¬648,742 cos R 43d. Substitution
137,199.11
 ¬cosR 43e. Def. of cosine
648,742
43f. Cross products
R ¬cos1
648,742 
137,199.11

43g. Substitution
R ¬77.8° 43h. Commutative Property
41. A 44. AB  
[10  (6)]2  [4  (8)]2
 
162  42
 
272
174 ft 180 ft BC  
(6  10)2   2
[8  (4)]
 
(4) 
2  12 2

 
160
AC  
[6  (6)]2 
 [8  (8)]2
C
186 ft
B  
12 
2 16 2

a2 ¬b2  c2  2bccosA  
400 or 20
1862 ¬1742  1802  2(174)(180)cosA AC2 ¬BC2  AB2  2(BC)(AB)cos B
2 2
34,596 ¬62,676  62,640cos A 202 ¬160
  272
 
28,080 ¬62,640 cos A
28,080
¬2160
272
cosB
 ¬cosA
62,640 400 ¬432  243,52
0 cosB
A ¬cos1
62,640 
28,080

32 ¬243,52
0 cosB
A ¬63.4°  3
2
¬cosB
sinA sin B 2
43,52
0
a ¬b

sin 6
 3.4° sin
¬ B  2
B ¬cos1 32


43,520
186 174
B ¬85.6°
174sin63.4° ¬186sin B So, mABC  85.6°.
174sin63.4°

186 ¬sinB CB2 ¬AB2  AC2  2(AB)(AC)cos A
2 2
sin1 174sin

186
63.4°
¬B  160
 ¬272
  202  2272
(20)cosA
56.8° ¬B 160 ¬672  40272
 cosA
mA  mB  mC ¬180
63.4  56.8  mC ¬180 512 ¬40272
 cosA
mC ¬59.8 512
 ¬cosA
40272
42. Let C represent Carlos’s angle and A represent
Adam’s angle.  402
A ¬cos1 51
2
72 
242 ¬402  502  2(40)(50)cos C A ¬39.1°
576 ¬4100  4000cos C mDCA  mB  mA
3524 ¬4000cos C  85.6  39.1 or 124.7
35
24 45. Sample answer: Triangles are used to build
4000 ¬cosC

supports, walls, and foundations. Answers should
C ¬cos1
4000 
352
4
include the following.
C ¬28.2° • The triangular building was more efficient with
the cells around the edge.
• The Law of Sines requires two angles and a side
or two sides and an angle opposite one of those
sides.

229 Chapter 7
46. B; d2 ¬e2  f 2  2efcosD B
53. To show that A

C
D
, we must show that
d2 ¬122  152  2(12)(15)cos75° AB
  AE.
d2 ¬369  360cos75° CD CE
AB 8 AE
   or 2, and    9. Since the side lengths
d ¬369  360cos75° 
CD 4 CE 4
d ¬16.6 B
are not proportional, A is not parallel to C
D
.
47. C; earnings  base salary  commission
B
54. To show that A

C
D
, we must show that
Let s represent her sales for the month.
4455 ¬1280  0.125s AB
  BE
CD DE.
3175 ¬0.125s AB
    or 9, and 
5.4 BE
E 
D
18 9
25,400 ¬s CD 3 5 10 or 5 .
Her sales that month were $25,400. AB BE
Thus, 
CD D E . Since the sides have proportional
B
lengths, A

C
D
.
Page 390 Maintain Your Skills 55. Given: JFM  EFB
X
sin sinY LFM  GFB
x ¬ y
48.  
sin 22° sin 49°
Prove: JFL  EFG
  ¬ 
x 4.7 H G
4.7sin22° ¬x sin49° L
4.7 sin 22° E J
 ¬x F
sin 49°
2.3 ¬x M C
D
sinX Y
sin
x ¬ y
49.  
A B
sin 50° sin
Y
14 ¬ 10
   Proof:
Since JFM  EFB and LFM  GFB, then
10sin50° ¬14sinY
10sin50°
by the definition of similar triangles,

14 ¬sinY JF M
F MF LF
F BF and BF  GF . By the Transitive

E
  
sin1 10sin 50°
 1
4 
¬Y JF LF
33° ¬Y
Property of Equality, 
EF 
GF . F F by the
Reflexive Property of Congruence. Then, by SAS
50. Similarity, JFL  EFG.
x 56. Given: J
M

E
B

23
M
L

G
B
1.55 m Prove: JL


E
G
100 m H G
x
L
tan23° ¬ E J
100 F
100tan23° ¬x M C
D
42.45 ¬x
x  1.55 ¬42.45  1.55 or 44.0 A B
The height of the building is about 44.0 meters. Proof:
B
51. To show that A 

C
D
, we must show that M
Since J 

E
B and L
M

G
B
, then MJF BEF
AC BD and FML FBG because if two parallel lines
CE  DE .
   
are cut by a transversal, corresponding angles are
AC
   or 7, and 
8.4 BD 6.
3 congruent. EFB EFB and BFG BFG
DE  4.5
 
CE 6 5
by the Reflexive Property of Congruence. Then
or 7
 AC
  BD
5 . Thus, CE  DE . Since the sides have
 EFB  JFM and FBG  FML by AA
B


C
D
. JF MF MF LF
Similarity. Then 
EF  BF , BF  GF by the
 
proportional lengths, A
B


C
D
, we must show that JF LF
definition of similar triangles. 
EF G
52. To show that A
F by the
AC BD

CE 
DE .
Transitive Porperty of Equality and
EFG EFG by the Reflexive Property of
CE  AE  AC  15  7 or 8.
AC 7
Congruence. Thus, JFL  EFG by SAS
So, 
CE  8. Similarity and FJL FEG by the definition of
DE  BE  BD  22.5  10.5 or 12. L
similar triangles. J

E
G because if two lines are
BD 10
.5 cut by a transversal so that the corresponding
7 AC BD
DE  12 or 8 . Thus, CE  DE . Since the
So,      
angles are congruent, then the lines are parallel.
B
sides have proportional lengths, A

C
D
.

Chapter 7 230
57. y y  4 ¬4x  8
Z(0 , 1 2 ) y ¬4x  12
Solve a system of equations to find the point of
intersection of the medians.
Y(4, 8 ) x  8 ¬4x  12
3x  8 ¬12
3x ¬4
x ¬4

3
X(8 , 0 ) x
Replace x with 4

3 in one of the equations to find
the y-coordinate.
Find an equation of the altitude from X to Y Z
.
Z
The slope of Y  is 12 
 8 y  4
3 8

0  (4) or 1, so the slope of
the altitude is 1. 20
y3
y  y1 ¬m(x  x1)
y  0 ¬1(x  8)

The coordinates of the centroid are 4
  2
0
3 , 3 or 
y ¬x  8 about (1.3, 6.7).
Find an equation of the altitude from Z to X Y
. 59. y
80
Y
The slope of X  is   or 2, so the slope of Z(0 , 1 2 )
4  8 3
the altitude is 3

2.
y  y1 ¬m(x  x1) Y(4 , 8 )
y  12 ¬3
2 (x  0)

3
y ¬2 x  12
Solve a system of equations to find the point of X(8 , 0 ) x
intersection of the altitudes.
x  8 ¬3
2 x  12
 Find an equation of the perpendicular bisector
Z
of Y.
8 ¬5
2 x  12

5
Z
The midpoint of Y is 4 

2 ,
8 
0 
2
12

or (2, 10).
4 ¬2x 12  8

The slope of YZ is  
0  (4) or 1, so the slope of
8
5 ¬x
 the perpendicular bisector is 1.
Replace x with 8  y  y1  m(x  x1)
5 in one of the equations to find
the y-coordinate. y  10  1[x  (2)]
y  10  x  2
 
y   85 8

y  x  8
48
y
5
Find an equation of the perpendicular bisector of
The coordinates of the orthocenter are 8 
  48

5 , 5 or Y
X .
The midpoint of XY 8  (4)
 is 
,  0 8

or (2, 4).
(1.6, 9.6). 2 2
80
Y
 is  2
4  8 or  3 , so the slope of
58. y The slope of X 
3
Z(0 , 1 2 ) the perpendicular bisector is 2.
y  y1  m(x  x1)

Y(4, 8 ) y  4  3
2 (x  2)


y  3
2x  3  4


y  3
2x  1

X(8 , 0 ) x Solve a system of equations to find the point of
intersection of the perpendicular bisectors.
Find an equation of the median from X to Y Z
. x  8¬ 3
2x  1

Z
The midpoint of Y is 4 
2
 ,
0 8 
2
12

or (2, 10).
8¬ 5
2x  1

10  0
2  8 or 1.
Then the slope of the median is  
7¬ 5

2x
y  y1 ¬m(x  x1)
1
 4
y  0 ¬1(x  8) 5 ¬ x
y ¬x  8 Replace x with 14
5 in one of the equations to find
Y
Find an equation of the median from Z to X. the y-coordinate.
Y
The midpoint of X 8  (4)
 is  2
,  0 
2
8

or (2, 4). y  14
5 8
4 12 2
y 6
Then the slope of the median is 2  0 or 4. 5
y  y1 ¬m(x  x1) The coordinates of the circumcenter are 5 , 5 
14 26

y  4 ¬4 (x  2) or (2.8, 5.2).

231 Chapter 7
Page 391 Geometry Activity: Trigonometric 9. Let x represent the geometric mean.
Identities 4  x
x 81
1. Sample answer: It is of the form a2  b2  c2, x2  324
where c  1. x  324
1 1 x  18
cos  sec ; tan  cot
2. 
10. Let x represent the geometric mean.

sin
cos  tan
3.  Original equation 20
  x
y x ¬ 35
y y

r  x sin  r, cos  xr, tan  yx x2¬ 700
x x¬ 700

r
y y
rxr  x Multiply by the reciprocal of xr. x¬ 26.5
y y 11. Let x represent the geometric mean.
   Multiply. 18
x x   x

cos x ¬ 44
4. 
sin  cot Original equation x2¬ 792
sin  yr, cos  xr, cot  xy
x

r  

x x¬ 792

y y

r x¬ 28.1
x r x
ry  y Multiply by the reciprocal of yr. 12. Let x¬ RS.
PS
   RS
x  x Multiply. RS ¬ QS
y y
8 x
¬ 
5. tan2  1  sec2 Original equation x 14
y 2 2 y x2¬ 112
x  1  xr tan  x, sec  xr
y2 2 x¬ 112
 or 47

  1  r2 Evaluate exponents. x¬ 10.6
x2 x
2 y2

2
x 2  1  (x2)xr2 Multiply each side by x2. So RS¬ 10.6.
x
13. 152  202¬ x2
y2  x2  r2 Simplify.
225  400¬ x2
r2  r2 Substitution; y2  x2  r2 625¬ x2
6. cot  1  csc2
2 Original equation 625
¬ x
2 2
x
y  1  yr cot  xy, sec  yr 25¬ x
2 2
14. x2   7  ¬ 
17 
xy  1  yr
5 13
1 
2 2
2 2 Evaluate exponents.
25 169
x2  289 ¬ 
y xy  1  (y )yr
2 2 289
2 2 Multiply each side by y2.
2 2
14
4
x2¬ 
x2  y2  r2 Simplify. 289
r2  r2 Substitution; x2  y2  r2 x¬ 
14
 4
289
12
x¬ 
17
15. x2  132¬ 212
Chapter 7 Study Guide and Review x2  169¬ 441
x2¬ 272
Page 392 Vocabulary and Concept Check x¬ 272
 or 417

1. true x¬ 16.5
2. false; opposite; adjacent 16. x  9
3. false; a right y  92 
4. true 17. 13  x2
13
 x
5. true 
2
6. false; 45°-45°-90° 13

2
 x
2 2
7. false; depression
132

2 x
 
yx
Pages 392–396 Lesson-by-Lesson Review 132
y 
2
8. Let x represent the geometric mean. 18. x  2(6) or 12
4  x y  63
x 16
x2  64 19. z  183 
x  64  a  2z
x8 a  2(183 ) or 363


Chapter 7 232
20. 14  z3
 26. B
14
 z
3 x
60 ft
A
14
3
 z 500 yd C
3 3 
143 Let x represent mBAC.
3 z
 
60
tanx°¬  
500  3
a  2z
60
a2  
143

3

or 
283

3
 tanx°¬  
1500
z  y3
 x¬ tan11500 
60

143 x¬ 2.3
3  y3
 
The angle of elevation must be greater than 2.3°.
143 1
    y 27. A D
3 3
14
3 y

b  2y 100 ft

b  23  or 3
14
 28
 x

C 240 ft B
21. sinF  ac
 9 3 Let x represent mABC.
15 or 5 The ground and the horizontal level with the top
 0.60
of the escalator are parallel. Therefore,
cosF  bc mDAB  mABC since they are alternate
12 4
 
15 or 5
interior angles.
 0.80 10
0
tanx°¬ 
240
tanF  a

b x¬ tan1
240 
10
0
9

12 or 3

4 x¬ 22.6
 0.75 The angle of depression is about 22.6°.
sinG  bc 28. A D
12 4
 
15 or 5 50 ft
x

 0.80
C 1000 ft
B
cosG  ac
9 3 Let x represent mABC.
 15 or 5
 0.60 The ground and the horizontal level with the
b initial point of the balloon are parallel. Therefore,
tanG ¬a
mDAB  mABC since they are alternate
12 4
¬ 9 or 3 interior angles.
¬1.33 tanx°¬ 50

1000
22. sinF  ac cosF  bc tanF¬¬a
7
 24 
b
7
¬ 24


x¬ tan1 50

1000 
25 25 x¬ 2.9
 0.28  0.96 ¬0.29
The angle of depression is about 2.9°.
sinG  bc cosG  ac b
tanG ¬a
29. A
24 7 24
 
25 25 ¬
7
 0.96  0.28 ¬3.43
23. sinP  0.4522
30 yd
P  sin1(0.4522)
KEYSTROKES: 2nd [SIN1] 0.4522 ENTER
mP  26.9
24. cosQ  0.1673 44

Q  cos1(0.1673) C x B
KEYSTROKES: 2nd [COS1] 0.1673 ENTER
mQ  80.4 Let x represent the length of the shadow of the
building, BC.
25. tanR  0.9324 30
R  tan1(0.9324) tan44°¬  x
KEYSTROKES: 2nd [TAN1] 0.9324 ENTER xtan44°¬ 30
mR  43.0 30
x¬ 
tan44°
x¬ 31.1
The shadow is about 31.1 yards long.

233 Chapter 7
sin
 A sin B
30. C 35. a ¬ b
30 ft sin 2
 9° sin
 B
x
4.8 ¬ 8.7
A 400 ft B 8.7sin29°¬ 4.8sin B
Let x represent mCAB. 8.7si
 n 29°
4.8 ¬ sinB
3
0
tanx°¬ 400
x¬ tan1  3
0
 
sin1 8.7s i
 ¬ B
4.8
n 29°

400 61°¬ B
x¬ 4.3
mA  mB  mC  180
The angle of elevation of the track is about 4.3°.
29  61  mC  180
sin
 F sin
 G mC  90
31. f ¬ g
 C
sin sin A
c ¬ a
sin 82°
   sin 4
8°
f ¬ 16
sin 90°
 sin 29°
16sin82°¬ fsin48° c¬  
4.8
16 sin
 82° 4.8sin90°¬ csin29°
sin 48° ¬ f
4.8 si
 n 90°
21.3¬ f sin 29° ¬ c
 H
sin sin
 G 9.9¬ c
32. h ¬ g
sin
H sin 65° 36. mA  mB  mC  180
  
10.5 ¬ 13 29  64  mC  180
13sinH¬ 10.5sin65° mC  87
10.5 si
n65° sin
 A sin
 B
sinH¬  a ¬ b
13
sin 29° sin 6 4°

10.5 si
H¬ sin1  13
n65°
  a¬  
18.5
18.5sin29°¬ asin64°
H¬ 47°
18.5 si
 n 29°
sin
 A  B
sin sin 64° ¬ a
33. a ¬ b 10.0¬ a
sin 6
 4° sin
 B sinB sin C
15 ¬ 11 
b ¬ c
11sin64°¬ 15sinB sin 6
¬  4° sin 87°
11sin64° 18.5 c
 ¬ sinB csin64°¬ 18.5sin87°
15
sin1 11sin 64°
 ¬ B
15  18.5 si
c¬  n 87°
sin 64°
41°¬ B c¬ 20.6
mA  mB  mC  180 37. z2¬ x2  y2 2xycosZ
64  41  mC  180 z2¬ 7.62  5.42  2(7.6)(5.4)cos51°
mC  75 z2¬ 86.92  82.08cos51°
sin C
c¬ asin A z¬ 86.92
08cos
 82.51° 
sin 75° sin 64° z¬ 5.9

c¬ 

15 38. y2¬ x2  z2  2xzcosY
15sin75°¬ csin64° y2¬ 212  162  2(21)(16)cos73°
15 sin
 75° y2¬ 697  672cos73°
sin 64° ¬ c
y¬ 697
 s73°
672co
16.1¬ c
sinC sinA
y¬ 22.4
34.  
c ¬ a 39. a2¬ b2  c2  2bccosA
sin 6
 7° sin 55°
 a2¬ 132  182  2(13)(18)cos64°
12 ¬ a
asin67°¬ 12sin55° a2¬ 493  468cos64°
12 sin
55° a¬ 493
 s64°
468co
a¬ sin 67° a¬ 17.0
 A
sin sin B
a ¬ b
a¬ 10.7
mA  mB  mC¬ 180 sin 64° sin
B
  
55  mB  67¬ 180 17 ¬ 13
mB¬ 58 13sin64°¬ 17sinB
sin B sin
b ¬ c C 13sin
 64°
17 ¬ sinB
sin 58°
 sin 6
b¬  12
7°
sin1 13sin 64°
 ¬ B
17 
12sin58°¬ bsin67° 43°¬ B
12 sin
 58° mA  mB  mC  180
sin 67° ¬ b
11.1¬ b 64  43  mC  180
mC  73

Chapter 7 234
 B
sin sin C
40. b ¬ c 10. 19  x2

sin
 B sin 5
 3° 19
 x
5.2 ¬ 6.7 2
6.7sin B¬ 5.2sin53° 19  
2
    x
5.2si
n 53° 
2 2 
sinB¬  6.7 192
2 x
 
B¬ sin1  
5.2si

n 53°
6.7 yx
B¬ 38° 192
y 
2
mA  mB  mC¬ 180
11. This is a 30°-60°-90° triangle with hypotenuse of
mA  38  53¬ 180
length 12, shorter leg with length of y and longer
mA¬ 89
A
sin B
sin leg with length of x.
 
a ¬ b 12  2y
sin 89° sin 38°
 a¬  
5.2 6y
5.2sin89°¬ asin38° x  y3
5.2 si
n 89° x  63

sin 38° ¬ a 12. x2  82 162
8.4¬ a
x2  64  256
x2  192
x  192

Chapter 7 Practice Test x  83
8
siny°  
16
Page 397 y  sin18
16
1.    2abcosC
c2 a2 b2 y  30
2. Yes; two perfect squares can be written as a  a BC
13. cosB   
and b  b. Multiplied together, we have a  a  b  b. AB
15
 5
Taking the square root, we have ab, which is 
21  7
rational. BC
14. tanA  
3. Sample answer: 2, 23 , 4 AC
15
4. Let x represent the geometric mean. 
16
7 x
x  
63 BC
15. sinA  
AB

x2  441 15
 5
x  441  or 21 21  7
5. Let x represent the geometric mean. sin
 F sin G
16. f ¬ g
6 x
x  
24 sin 5
 9° sin 71°

x  144
2 13 ¬ g
x  144  or 12 gsin59°¬ 13sin71°
13 sin
71°
6. Let x represent the geometric mean. g¬ sin 59°
10 x
x  

50 g¬ 14.3
x2  500 sinF sin H
f¬ ¬h
17. 
x  500 or 105
sin 5 2° sin
H
7. x  5 ¬ 6
2 2 2 
10
 ¬
12.5
x2  25¬ 36 12.5sin52° ¬10sinH
x2¬ 11 12.5 si
n52°
 ¬sinH
x¬ 11 10
x¬ 3.32 sin1
12.5 si
10
 ¬H
n52°

8. 72  132¬ x2 80.1° ¬H


49  169¬ x2 18. f 2¬ g2  h2  2ghcosF
218¬ x2 f 2¬ 152 132  2(15)(13)cos48°
218
¬ x f 2¬ 394  390cos48°
14.8¬ x f¬ 394
 s48°
390co
2
9. x2  2  ¬ 9
12 2 f¬ 11.5
x2  36¬ 81 19. h2¬ f2  g2  2fgcosH
x2¬ 45 h2¬ 13.72  16.82  2(13.7)(16.8)cos71°
x¬ 45 h2¬ 469.93  460.32cos71°
x¬ 35  h¬ 469.9
3 0.32co
 46s71° 
x¬ 6.7 h¬ 17.9

235 Chapter 7
20. c2¬ a2  b2  2abcosC since they are alternate interior angles.
c2¬ 152  172  2(15)(17)cos45° Let x represent CD, the horizontal distance to the
c2¬ 514  510cos45° city.
c¬ 514
 s45°
510co 0.5
tan9°¬ x

c¬ 12.4 xtan9°¬ 0.5
 C
sin sin
 A
c ¬ a x¬ 0.5

sin 4 5° sin
A tan 9°
  
12.4 ¬ 15 x¬ 3.2
15sin45°¬ 12.4sinA The horizontal distance to the city is about
15 sin45° 3.2 miles.
12.4 ¬ sinA
24.
sin1 12.4 ¬ A
15 sin45° C
x
59°¬ A 10

A
mA  mB  mC¬ 180 5 mi B
59  mB  45¬ 180
Let x represent CB, the height of the incline.
mB¬ 76
A
sin sinB tan10°¬ 5x
21.  
a ¬ b 5tan10°¬ x
sin
 A sin 4
 8°
12.2 ¬ 10.9 0.9¬ x
10.9 sin A¬ 12.2sin48° The height of the incline is about 0.9 mile.
12.2 si
sinA¬  n 48° 25. D; Let y represent the unknown side length in the
10.9 triangle.
A¬ sin1
12.2 si
10.9
n 48°
 52  y2¬ 132
A¬ 56° 25  y2¬ 169
mA  mB  mC¬ 180 y2¬ 144
56  48  mC¬ 180 y¬ 144
mC¬ 76 y¬ 12
12
sin
 C sin B tanX¬ 
c ¬ b
5
sin 76°
 sin 48°
c¬  
10.9
10.9sin76°¬ csin48°
10.9 si
n 76° Chapter 7 Standardized Test Practice

sin 48° ¬ c
14.2¬ c Pages 398–399
22. a2¬ b2  c2 2bccosA 1. C; there is no information to support choices A, B,
192¬ 23.22  212 2(23.2)(21)cos A or D. 1 and 4 are vertical angles, and 2 and
361¬ 979.24  974.4cos A 3 are vertical angles.
618.24¬ 974.4cos A 2. D; AD¬ CD
618
 .24
974.4 ¬ cosA 3x  5¬ 5x  1
A¬ cos1974.4 
618
.24 5¬ 2x  1
6¬ 2x
A¬ 51°
sinA sin B 3¬ x
 a ¬ b AC  AD  CD
sin 5
 1° sin
 B AC  3x  5  5x  1
19 ¬ 23.2
23.2sin51°¬ 19sinB AC  3(3)  5  5(3)  1
23.2 si
n51° AC  28
 ¬ sinB
19 SR PT
3. B; 
DC AE
sin 
1 23.2 ¬ B
sin51°
S R 6
19 8  
11
72°¬ B
mA  mB  mC¬ 180 11(SR)  48
48 4
51  72  mC¬ 180 SR   
11 or 4 11
mC¬ 57 A B A C
4. C;   
AC ¬ AD
23. 12
 
 3 AC
A B
AC ¬ 
12
9
(AC)2¬ 180
0.5 mi
AC¬ 180

AC¬ 13.4
5. B; mRTS  180  135 or 45.
C x D RT  (ST)2

 52

The ground and the horizontal level with the
plane are parallel. Therefore, mBAD  mADC

Chapter 7 236
6. D; the height of the original tower is AB  BC. 10. y  y1¬ m(x  x1)
60
sin36°¬ 
BC y  32¬ 9
5 (x  0)

BCsin36°¬ 60 y¬ 9
5 x  32

60
BC¬ sin 36° YZ XZ
BC¬ 102
11. 
UV 
TV
6
AB  BC¬ 60  102 or 162 feet.  10 or 3

5
sin
 R sin S
7. C; r ¬ s 12. Let x represent Dee’s height above the water.
x
sin 3
 4° sin
 S sin41°¬  50
14 ¬ 21 0
21sin34°¬ 14sinS 500sin41°¬ x
21sin34° 328¬ x
 ¬ sinS
14 Dee is about 328 feet above the water.
sin1
21s ¬ S
in
14
34°
13. Since Sasha is equidistant from Toby and Rani,
57°¬ S T
S  and SR are congruent and STR is an
isosceles triangle. According to the Isosceles
8. mC  mBDC  mDBC¬ 180
Triangle Theorem, T and R are also
90  mBDC  55¬ 180
congruent. S X is perpendicular to T
R
, so SXT
mBDC¬ 35
and SXR are both right angles and congruent.
mADB  mBDC¬ mADC
Two corresponding angles and the corresponding
mADB  35¬ 61
nonincluded sides are congruent (AAS Theorem),
mADB¬ 26
so STX and SRX are congruent triangles.
mA  mABD  mADB  180
Since these triangles are congruent, the
69  mABD  26  180
corresponding sides T X
 and R
X are congruent and
mABD  85
mABC  mABD  mDBC have equal length; therefore when Sasha is
 85  55 jumping at Point X she will be at the midpoint
 140 between Toby and Rani.
y y
9. m  
x 
2
x
1
2 1
50 32
10  0
18 9

10 or 5

237 Chapter 7
Chapter 8 Quadrilaterals
y y
Page 403 Getting Started 9. The slope is given by m  
2
1
.
x2  x1
1. The angles measuring x° and 50° are 0 
a
m
b0
supplementary. Find x.
mx  50  180  a

b
mx  130
The slope is a
b.
So, x is 130.
y2  y1
2. x° is the measure of the exterior angle of the 10. The slope is given by m   .
x2  x1
triangle so its measure is the sum of the two ac
m ¬
c  (a)
remote interior angles or 25  20. So, x  45.
ac
3. The measure of an internal angle of an ¬
c  a
equilateral triangle is 60. The angle measuring x° a c
¬
a  c or 1
is supplementary to one of the angles. Find x.
mx  60  180 The slope is 1.
mx  120
So, x is 120.
y y
4. The slope is given by m  
2
1
. 8-1
x2  x1 Angles of Polygons
10  3 7
S
R: 
1  4 ¬ 5

10 
20 1
0
S
T: 
1  13 ¬ 14
 Page 406 Geometry Activity: Sum of the
¬5
 Exterior Angles of a Polygon
7
S
R  and T
S are perpendicular since their slopes 1. sum of
number of
are opposite inverses. measure of
Polygon exterior
y y exterior
5. The slope is given by m  
2
1
. angles angles
x2  x1
8 
6 2 triangle 3 360
S
R : 
3  (9)  12
quadrilateral 4 360
 1

6
8 
20 12 pentagon 5 360
S
T: 31  2
 
hexagon 6 360
 6
S
R  and T
S
 are perpendicular since their slopes heptagon 7 360
are opposite inverses.
2. The sum of the measures of the exterior angles is
y y
6. The slope is given by m  
2
1
. 360.
x2  x1
3  (
1) 4
S
R: 
5  (6)  11
5 3 2
S
T: 
2  5  3 Page 407 Check for Understanding

RS and T
S are not perpendicular since their 1. A concave polygon has at least one obtuse angle,
slopes are not opposite inverses. which means the sum will be different from the
y y formula.
7. The slope is given by m  
2
1
.
x2  x1 2. Yes; an irregular polygon can be separated by the
84 4
S
R:  (6)  3
3  diagonals into triangles so the theorems apply.
82 6 3. Sample answer:
S
T :   5  8
3 
regular quadrilateral:
 3
4
S
R and TS
 are perpendicular since their slopes
are opposite inverses.
y y
8. The slope is given by m  
2
1
. The sum of the interior angles is 360°.
x2  x1
d  d2
m  quadrilateral that is not regular:
c  2c
d

2
 
32c

 3d
c
The slope is 3d
c.
The sum of the interior angles is 360°.

Chapter 8 238
4. Use the Interior Angle Sum Theorem. 10. The sum of the measures of the exterior angles is
S  180(n2) 360. There are 6 congruent exterior angles.
 180(5  2) 6n  360
 540 n  60
The sum of the measures of the interior angles of The measure of each exterior angle is 60. Since
a pentagon is 540. each exterior angle and its corresponding interior
5. Use the Interior Angle Sum Theorem. angle form a linear pair, the measure of the
S  180(n  2) interior angle is 180  60 or 120.
 180(12  2) 11. The sum of the measures of the exterior angles is
 1800 360. There are 18 congruent exterior angles.
The sum of the measures of the interior angles of 18n  360
a dodecagon is 1800. n  20
6. Use the Interior Angle Sum Theorem to write an The measure of each exterior angle is 20. Since
equation to solve for n, the number of sides. each exterior angle and its corresponding interior
S ¬180(n  2) angle form a linear pair, the measure of the
(60)n ¬180(n  2) interior angle is 180  20 or 160.
60n ¬180n  360 12. Use the Interior Angle Sum Theorem.
0 ¬120n  360 S  180(n  2)
360 ¬120n  180(5  2)
3 ¬n  540
The polygon has 3 sides. The sum of the measures of the interior angles of
7. Use the Interior Angle Sum Theorem to write an the base of the fish tank is 540.
equation to solve for n, the number of sides.
S ¬180(n  2)
(90)n ¬180(n  2) Pages 407–409 Practice and Apply
90n ¬180n  360 13. Use the Interior Angle Sum Theorem.
0 ¬90n  360 S  180(n  2)
360 ¬90n  180(32  2)
4 ¬n  5400
The polygon has 4 sides. The sum of the measures of the interior angles of
8. Since n  4, the sum of the measures of the a 32-gon is 5400.
interior angles is 180(4  2) or 360. Write an 14. Use the Interior Angle Sum Theorem.
equation to express the sum of the measures of S  180(n  2)
the interior angles of the polygon.  180(18  2)
360  mT  mU  mV  mW  2880
360  x  (3x  4)  x  (3x  4) The sum of the measures of the interior angles of
360  8x  8 an 18-gon is 2880.
368  8x 15. Use the Interior Angle Sum Theorem.
46  x S  180(n  2)
Use the value of x to find the measure of each  180(19  2)
angle.  3060
mT  46, mU  3  46  8 or 134, mV  46, The sum of the measures of the interior angles of
and mW  3  46  8 or 134. a 19-gon is 3060.
9. Since n  6, the sum of the measures of the 16. Use the Interior Angle Sum Theorem.
interior angles is 180(6  2) or 720. Write an S  180(n  2)
equation to express the sum of the measures of  180(27  2)
the interior angles of the polygon.  4500
720  mJ  mK  mL  mM  mN  The sum of the measures of the interior angles of
mP a 27-gon is 4500.
720  2x  (9x  30)  (9x  30)  2x 
17. Use the Interior Angle Sum Theorem.
(9x  30)  (9x  30)
S  180(n  2)
720  40x  120
 180(4y  2)
600  40x
 720y  360
15  x
 360(2y  1)
Use the value of x to find the measure of each
The sum of the measures of the interior angles of
angle.
a 4y-gon is 360(2y  1).
mJ  30, mK  9  15  30 or 165,
mL  9  15  30 or 165, mM  30, 18. Use the Interior Angle Sum Theorem.
mN  9  15  30 or 165, and mP  9  15  30 S  180(n  2)
or 165.  180(2x  2)
 360(x  1)
The sum of the measures of the interior angles of
a 2x-gon is 360(x  1).

239 Chapter 8
19. Use the Interior Angle Sum Theorem. 26. Use the Interior Angle Sum Theorem to write an
S  180(n  2) equation to solve for n, the number of sides.
 180(8  2) S ¬180(n  2)
 1080 (176.4)n ¬180(n  2)
The sum of the measures of the interior angles of 176.4n ¬180n  360
the octagonal garden is 1080. 0 ¬3.6n  360
20. Use the Interior Angle Sum Theorem. 360 ¬3.6n
S  180(n  2) 100 ¬n
 180(6  2) The polygon has 100 sides.
 720 27. Since n  4, the sum of the measures of the
The sum of the measures of the interior angles of interior angles is 180(4  2) or 360. Write an
the hexagonal gazebos is 720. equation to express the sum of the measures of
21. Use the Interior Angle Sum Theorem to write an the interior angles of the polygon.
equation to solve for n, the number of sides. 360  mM  mP  mQ  mR
S ¬180(n  2) 360  x  4x  2x  5x
(140)n ¬180(n  2) 360  12x
140n ¬180n  360 30  x
0 ¬40n  360 Use the value of x to find the measure of each
360 ¬40n angle.
9 ¬n mM  30, mP  4  30 or 120, mQ  2  30
The polygon has 9 sides. or 60, and mR  5  30 or 150.
22. Use the Interior Angle Sum Theorem to write an 28. Since n  5, the sum of the measures of the
equation to solve for n, the number of sides. interior angles is 180(5  2) or 540. Write an
S ¬180(n  2) equation to express the sum of the measures of
(170)n ¬180(n  2) the interior angles of the polygon.
170n ¬180n  360 540  mE  mF  mG  mH  mJ
0 ¬10n  360 540  x  (x  20)  (x  5)  (x  5)  (x  10)
360 ¬10n 540  5x  30
36 ¬n 510  5x
The polygon has 36 sides. 102  x
23. Use the Interior Angle Sum Theorem to write an Use the value of x to find the measure of each
equation to solve for n, the number of sides. angle.
S ¬180(n  2) mE  102, mF  102  20 or 122,
(160)n ¬180(n  2) mG  102  5 or 107, mH  102  5 or 97,
160n ¬180n  360 and mJ  102  10 or 112.
0 ¬20n  360 29. Since n  4, the sum of the measures of the
360 ¬20n interior angles is 180(4  2) or 360. Since a
18 ¬n parallelogram has congruent opposite angles, the
The polygon has 18 sides. measures of angles M and P are equal, and the
24. Use the Interior Angle Sum Theorem to write an measures of angles N and Q are equal. Write an
equation to solve for n, the number of sides. equation to express the sum of the measures of
S ¬180(n  2) the interior angles of the parallelogram.
(165)n ¬180(n  2) 360  mM  mN  mP  mQ
165n ¬180n  360 360  10x  20x  10x  20x
0 ¬15n  360 360  60x
360 ¬15n 6x
24 ¬n Use the value of x to find the measure of each
The polygon has 24 sides. angle.
25. Use the Interior Angle Sum Theorem to write an mM  10  6 or 60, mN  20  6 or 120,
equation to solve for n, the number of sides. mP  10  6 or 60, and mQ  20  6 or 120.
S ¬180(n  2) 30. Since n  4, the sum of the measures of the
(157.5)n ¬180(n  2) interior angles is 180(4  2) or 360. Write an
157.5n ¬180n  360 equation to express the sum of the measures of
0 ¬22.5n  360 the interior angles of the isosceles trapezoid.
360 ¬22.5n 360  mT  mW  mY  mZ
16 ¬n 360  20x  20x  30x  30x
The polygon has 16 sides. 360  100x
3.6  x
Use the value of x to find the measure of each
angle.
mT  20  3.6 or 72, mW  20  3.6 or 72,
mY  30  3.6 or 108, and mZ  30  3.6 or 108.

Chapter 8 240
31. Since n  10, the sum of the measures of the 37. The sum of the measures of the exterior angles is
interior angles is 180(10  2) or 1440. The sum of 360. A regular nonagon has 9 congruent exterior
the given measures is 10x  440. Find x. angles.
1440  10x  440 9n  360
1000  10x n  40
100  x The measure of each exterior angle is 40. Since
The measures of the interior angles of the each exterior angle and its corresponding interior
decagon are 105, 110, 120, 130, 135, 140, 160, 170, angle form a linear pair, the measure of the
180, and 190. interior angle is 180  40 or 140.
32. Since n  5, the sum of the measures of the 38. The sum of the measures of the exterior angles is
interior angles is 180(5  2) or 540. Write an 360. A regular octagon has 8 congruent exterior
equation to express the sum of the measures of angles.
the interior angles of the polygon. 8n  360
540  mA  mB  mC  mD  mE n  45
540  6x  (4x  13)  (x  9) (2x  8)  (4x  1) The measure of each exterior angle is 45. Since
540  17x  13 each exterior angle and its corresponding interior
527  17x angle form a linear pair, the measure of the
31  x interior angle is 180  45 or 135.
Use the value of x to find the measure of each 39. Since n  11, the sum of the measures of the
angle. interior angles is 180(11  2) or 1620. A regular
mA  6  31 or 186, mB  4  31  13 or 137, 11-gon has 11 congruent interior angles. Let the
mC  31  9 or 40, mD  2  31  8 or 54, and measure of one of these angles be x.
mE  4  31  1 or 123. 1620  11x
33. Sample answer: Since n  4, the sum of the 147.3  x
measures of the interior angles is 180(4  2) or To the nearest tenth, the measure of each interior
360. Write an equation to express the sum of the angle of the 11-gon is 147.3. Since each interior
measures of the interior angles of the quadrilateral. angle and its corresponding exterior angle form a
360  x  2x  3x  4x linear pair, the measure of the exterior angle is
360  10x about 180  147.3 or 32.7.
36  x 40. Since n  7, the sum of the measures of the
The measures of the interior angles of the interior angles is 180(7  2) or 900. A regular
quadrilateral are 36, 2  36 or 72, 3  36 or 108, 7-gon has 7 congruent interior angles. Let the
and 4  36 or 144. measure of one of these angles be x.
34. Since n  4, the sum of the measures of the 900  7x
interior angles is 180(4  2) or 360. Write an 128.6  x
equation to express the sum of the measures of To the nearest tenth, the measure of each interior
the interior angles of the quadrilateral. angle of the 7-gon is 128.6. Since each interior
360  x  (x  10)  (x  20)  (x  30) angle and its corresponding exterior angle form a
360  4x  60 linear pair, the measure of the exterior angle is
300  4x about 180  128.6 or 51.4.
75  x 41. Since n  12, the sum of the measures of the
The measures of the interior angles of the interior angles is 180(12  2) or 1800. A regular
quadrilateral are 75, 75  10 or 85, 75  20 or 95, 12-gon has 12 congruent interior angles. Let the
and 75  30 or 105. measure of one of these angles be x.
35. The sum of the measures of the exterior angles is 1800  12x
360. A regular decagon has 10 congruent exterior 150  x
angles. The measure of each interior angle of the 12-gon
10n  360 is 150. Since each interior angle and its
n  36 corresponding exterior angle form a linear pair,
The measure of each exterior angle is 36. Since the measure of the exterior angle is about 180 
each exterior angle and its corresponding interior 150 or 30.
angle form a linear pair, the measure of the 42. Consider the sum of the measures of the exterior
interior angle is 180  36 or 144. angles, N, for an n-gon.
36. The sum of the measures of the exterior angles is N  sum of measures of linear pairs  sum of
360. A regular hexagon has 6 congruent exterior measures of interior angles
angles.  180n  180(n  2)
6n  360  180n  180n  360
n  60  360
The measure of each exterior angle is 60. Since So, the sum of the exterior angle measures is 360
each exterior angle and its corresponding interior for any convex polygon.
angle form a linear pair, the measure of the
interior angle is 180  60 or 120.

241 Chapter 8
9y
43. Since n  5, the sum of the measures of the 48. Since   9, y  2x. Substitute this result into
2x
interior angles is 180(5  2) or 540. A regular
pentagon has 5 congruent interior angles. Let the the other equation relating x and y. Then solve
measure of one of these angles be x. for x.
540  5x 6x  3y ¬48
108  x 6x  3(2x) ¬48
The measure of each interior angle of the 6x  6x ¬48
Pentagon is 108. Since each interior angle and its 12x ¬48
corresponding exterior angle form a linear pair, x ¬4
the measure of the exterior angle is 180  108
or 72.
44. Yes; both the dome and the architectural elements Page 409 Maintain Your Skills
are based upon a regular octagon.
49. Use the Law of Cosines to find mC since the
Since n  8, the sum of the measures of the
measures of all three sides are known.
interior angles is 180(8  2) or 1080. A regular
c2 ¬a2  b2  2ab cosC
octagon has 8 congruent interior angles. Let the
measure of one of these angles be x. 112 ¬62  92  2(6)(9)cos C
1080  8x 121 ¬36  81  108 cosC
135  x 4 ¬108 cosC
4
The measure of each interior angle is 135. Since 
108 ¬cosC
each interior angle and its corresponding exterior C ¬cos11
27 
angle form a linear pair, the measure of the
exterior angle is about 180  135 or 45. C ¬92.1
180(n 2) 180n  360 To the nearest tenth, the measure of angle C is
45.  n  
n 92.1.
n
180 360
 n  n 50. Use the Law of Cosines to find mB since the
360 measures of all three sides are known.
 180   n
The two formulas are equivalent. b2 ¬a2  c2  2ac cos B
23.62 ¬15.52  25.12  2(15.5)(25.1)cos B
46. Sample answer: The outline of a scallop shell is 556.96 ¬240.25  630.01  778.1 cos B
a convex polygon that is not regular. The lines
313.3 ¬778.1 cos B
in the shell resemble diagonals drawn from one
31
3.3
778.1 ¬cos B
vertex of a polygon. These diagonals separate the 
polygon into triangles. Answers should include
the following.
B ¬cos1  
313
.3
778.1 
• The Interior Angle Sum Theorem is derived B ¬66.3
from the pattern between the number of sides To the nearest tenth, the measure of angle B is
in a polygon and the number of triangles. The 66.3.
formula is the product of the sum of the
measures of the angles in a triangle, 180, and 51. Use the Law of Cosines to find mA since the
the number of triangles the polygon contains. measures of all three sides are known.
a2 ¬b2  c2  2ab cos A
• The exterior angle and the interior angle of a
polygon are a linear pair. So, the measure of an 472 ¬532  562  2(53)(56) cos A
exterior angle is the difference between 180 and 2209 ¬2809  3136  5936 cos A
the measure of the interior angle. 3736 ¬5936 cos A
37
36
5936 ¬cos A
47. B; since the unknown polygon is regular, its 
interior angles are congruent. The sum of the
measures of the interior angles of the square,
A ¬cos1 
46
7
742 
pentagon, and unknown regular polygon is 360. A ¬51.0
Let the measure of each interior angle of the To the nearest tenth, the measure of angle A
unknown polygon be x. Find x using the fact that is 51.0.
the measures of the interior angles of squares and
52. Use the Law of Cosines to find mC since the
regular pentagons are 90 and 108, respectively.
measures of all three sides are known.
360  x  90  108
162  x c2 ¬a2  b2  2ab cos C
The sum of the measures of the interior angles is 162 ¬122  142  2(12)(14)cos C
given by S  180(n  2), the Interior Angle Sum 256 ¬144  196  336 cos C
Theorem. This is equal to 162n, where, in both 84 ¬336 cos C
cases, n is the number of sides. Solve for n. 84
 36 ¬cos C
3
162n ¬180(n  2)
162n ¬180n  360 C ¬cos1 1

4 
0 ¬18n  360 C ¬75.5
360 ¬18n To the nearest tenth, the measure of angle C is
20 ¬n 75.5.
The polygon has 20 sides.

Chapter 8 242
53. Use the Law of Sines since we know the measures 56. Use the Law of Sines since we know the measures
of two sides and an angle opposite one of the of two sides and an angle opposite one of the
sides. sides.
sinH sinG
h ¬ g
G
sin sinF  
g ¬ f
 
sinH sin 65°
32.4 ¬ 30.7
sin
G sin 54°   
17 ¬ 15
  
32.4 si
n 65°
17sin
sin G ¬ 54° sin H ¬ 30.7
15
G ¬sin1 17sin
15
54°
 H ¬sin1  
32.4 si
30.7 
n 65°

G ¬66° H ¬73°
Use the Angle Sum Theorem to find mH. Use the Angle Sum Theorem to find mF.
mF  mG  mH ¬180 mF  mG  mH ¬180
54  66  mH ¬180 mF  65  73 ¬180
mH ¬60 mF ¬42
Use the Law of Sines to find h. Use the Law of Sines to find f.
F
sin H
sin
f ¬ h
  sinG sin F
g ¬ f
 
sin 5 4° sin 60°
15 ¬ h
    sin 6 5° sin 42°
30.7 ¬ f
   
15 sin
60°
h ¬ 30 .7 si
n 42°
sin 54° f ¬ sin 65°
h ¬16.1
f ¬22.7
Therefore, mG  66, mH  60, and h  16.1.
Therefore, mH  73, mF  42, and f  22.7.
54. Use the Angle Sum Theorem to find mG.
mF  mG  mH ¬180 L
57. Given: J K
M
, J
K
L
M

47  mG  78 ¬180 Prove: JKL  MLK
mG ¬55 J K
Use the Law of Sines to find f and h.
sinG sin F
 g ¬f
sin 55° sin 47° L M
31 ¬ f
   
31 sin
47° Proof:
f ¬sin 55° Statements Reasons
f ¬27.7
1.  K
JL M
, J
K
 L
M
 1. Given
sinG sin H
 g ¬h 2. MKL  JLK, 2. Alt. int.  are .
sin 55° sin 78° JKL  MLK
31 ¬ h
   
31 sin
78°
L
3. K KL
 3. Reflexive Property
h ¬sin 55° 4. JKL  MLK 4. ASA
h ¬37.0 58. Line b is the transversal that forms 3 and 11
Therefore, mG  55, f  27.7, and h  37.0. where it intersects lines m and n. 3 and 11 are
55. Use the Angle Sum Theorem to find mF. corresponding angles.
mF  mG  mH ¬180 59. Line m is the transversal that forms 6 and 7
mF  56  67 ¬180 where it intersects lines b and c. 6 and 7 are
mF ¬57 consecutive interior angles.
Use the Law of Sines to find f and h. 60. Line c is the transversal that forms 8 and 10
sinG sin F
 g ¬f where it intersects lines m and n. 8 and 10 are
sin 56° sin 57° alternate interior angles.
63 ¬ f
   
61. Line n is the transversal that forms 12 and 16
63 sin 57°
f ¬ 
sin 56 ° where it intersects lines b and c. 12 and 16 are
alternate exterior angles.
f ¬63.7
62. 1 and 4, 1 and 2, 2 and 3, and 3 and
sinG sin H
 g ¬h 4 are consecutive interior angles.
sin 56° sin67° 63. 3 and 5, and 2 and 6 are alternate interior
63 ¬ h
  
angles.
63 sin
67°
h ¬sin 56° 64. 1 and 5, and 4 and 6 are corresponding
h ¬70.0 angles.
Therefore, mF  57, f  63.7, and h  70.0. 65. None; there are no pairs of alternate exterior
angles.

243 Chapter 8
Page 410 Spreadsheet Investigation: Angles of 6. Since consecutive angles in parallelograms are
Polygons supplementary, TSR is supplementary to STQ
1. For a regular polygon, the measure of each and SRQ.
interior angle in the polygon can be found by 7. MJK  KLM because opposite angles in a
dividing the sum of the measures of the interior parallelogram are congruent. Find mKLM.
angles by the number of sides of the polygon. mKLM  mKLR  mMLR
So, the formula to find the measure of each  70  30  100
interior angle in the polygon is “C2/A2”. So, mMJK  100.
2. For a regular polygon, the sum of the measures of 8. Consecutive angles in a parallelogram are
the exterior angles of the polygon can be found by supplementary. So, mJML  180  mKLM.
multiplying the number of sides by the measure mJML  180  100 or 80.
of the exterior angles. So, the formula to find the 9. Consecutive angles in a parallelogram are
sum of the measures of the exterior angles of the supplementary. So, mJKL  180  mKLM.
polygon is “A2*E2”. mJKL  180  100 or 80.
3. The formula for the sum of the measures of the 10. KJL  JLM because they are alternate
interior angles is “(A2-2)*180”, which gives interior angles. The measure of JLM is 30. So,
180 for 1 side and 0 for 2 sides. mKJL  30.
4. No, a polygon is a closed figure formed by 11. Opposite sides of a parallelogram are congruent,
coplanar segments. so their measures are equal.
Find a.
5. A 15-sided polygon has 13 triangles.
JM ¬KL
6. The measure of the exterior angle of a 15-sided 3a ¬21
polygon is 24. a ¬7
7. The measure of the interior angle of a 110-sided 12. Opposite sides of a parallelogram are congruent,
polygon is about 176.7. so their measures are equal. Find b.
8. Each interior angle measures 180. This is not JK ¬ML
possible for a polygon. 2b  3 ¬45
2b ¬42
b ¬21
13. Given: VZRQ and WQST
8-2 Parallelograms Prove: Z  T
Q
R
Pages 411–412 Geometry Activity: Properties of S
Parallelograms W
1. 
FGH J
P Q
R S and F
J
G HP S
Q R
. T
2. F  P  H  R and J  G  Q  S. V
Z
3. Opposite angles are congruent; consecutive angles
Proof:
are supplementary.
Statements Reasons
1. VZRQ and 1. Given
Page 414 Check for Understanding WQST
1. Opposite sides are congruent; opposite angles are 2. Z  Q, 2. Opp.  of a  are
congruent; consecutive angles are supplementary; Q  T .
and if there is one right angle, there are four right 3. Z  T 3. Transitive Prop.
angles.
14. Given: XYRZ, WZ
W
S

2. Diagonals bisect each other; each diagonal forms
Prove: XYR  S
two congruent triangles in a parallelogram.
3. Sample answer: W

x X Y

2x
Z R S
4. 
SVV Q because diagonals of parallelograms Proof: Opposite angles of a parallelogram are
bisect each other. congruent, so Z  XYR. By the Isosceles
5. Since diagonals bisect each other and opposite Triangle Theorem, since WZ
W S, Z  S. By
sides of parallelograms are congruent, VRS  the Transitive Property, XYR  S.
VTQ by SSS.

Chapter 8 244
J
15. C; G  and H
K are the diagonals. Since the 30. The diagonals of a parallelogram bisect each
diagonals of a parallelogram bisect each other, the other, so MQ  QP. Find w.
J
intersection point is the midpoint of G and H
K. MQ ¬QP
Find the intersection of the diagonals by finding 4w  3 ¬11.1
J
the midpoint of G . 4w ¬14.1
x1  x2 y1  y2 w ¬3.5
2, 2  
3 

2 ,
3 
2
4  (5)
31. The diagonals of a parallelogram bisect each
 (0, 0.5) other, so RQ  QN. Find z.
The diagonals intersect at (0, 0.5). RQ ¬QN
3z  3 ¬15.4
3z ¬18.4
Pages 415–416 Practice and Apply z ¬6.1
16. DAB  BCD because opposite angles of a
32. The diagonals of a parallelogram bisect each
parallelogram are congruent.
other, so EJ  JG. Find x.
17. ABD  CDB because alternate interior angles EJ ¬JG
are congruent. 2x  1 ¬3x
B
18. A  DC
 because opposite sides of a parallelogram 1 ¬x
are parallel. EG  EJ  JG
G
19. B G D
 because the diagonals of a  [2(1)  1]  3(1)
parallelogram bisect each other. 6
20. ABD  CDB because the diagonal B D
 33. The diagonals of a parallelogram bisect each
separates the parallelogram into two congruent other, so HJ  JF. Find y.
triangles. HJ ¬JF
21. ACD  BAC because alternate interior angles 1y  2 ¬y  1
2 2
are congruent.
2  1 1
2 ¬y  2 y

22. RNP  NRM because they are alternate
interior angles. So, mRNP  38. Find mMNP. 5 ¬1 y
2 2
mMNP  mMNR  mRNP
5 ¬y
 33  38
FH  HJ  JF
 71
23. NRP  MNR because they are alternate ¬ 1
   1
2 (5)  2  5  2 
interior angles. So, mNRP  33. ¬9
24. RNP  MRN because they are alternate
34. Since the diagonals of a parallelogram bisect each
interior angles. So, mRNP  38.
other, the drawer pulls are at the intersection
25. RMN is supplementary to MNP. Find point of the diagonals.
mRMN.
35.
mRMN  mMNP ¬180 A B
mRMN  71 ¬180 3a
 2b
mRMN ¬109 18 a
26. MQN is supplementary to PQN. Find mMQN.
180  mMQN  mPQN P
1
180  mMQN  83 3 b 12a
97  mMQN
27. MQR  PQN because they are vertical angles. D C
So, mMQR  83. Since ABCD is a parallelogram, the diagonals
28. Opposite sides of a parallelogram are congruent, bisect each other. So AC  2(AP)
so their measures are equal. Find x. AC  12a and AP  3a  18
MN ¬RP 12a ¬2(3a  18)
3x  4 ¬20 12a ¬6a  36
3x ¬24 6a ¬36
x ¬8 a ¬6
29. Opposite sides of a parallelogram are congruent, DP  PB
so their measures are equal. Find y. 3b  1  a  2b
NP ¬MR 3b  1  6  2b
2y  5 ¬17.9 b5
2y ¬12.9 DB  2(3b  1)
y ¬6.45  2(3  5  1)
 2(16)  32
So, a  6, b  5, and DB  32.

245 Chapter 8
36. A 2x  5 B 40. They are all congruent parallelograms. Since A, B
2y and C are midpoints, A C
, A
B
, and BC
 are
120
midsegments. The midsegment is parallel to the
21 third side and equal to half the length of the third
side. So, each pair of opposite sides of ACBX,
ABYC, and ABCZ are parallel.
41. Given: PQRS
Prove: P QR S

R
Q S P

D 21 C P Q
1 3
Opposite sides of a parallelogram are congruent, 2
4
B
so A C D
 and AB  CD. S R
Find x. Proof:
AB ¬CD Statements Reasons
2x  5  21
2x ¬16 1. PQRS 1. Given
x ¬8 2. Draw an auxiliary 2. Diagonal of PQRS
Consecutive angles in a parallelogram are segment P R
 and
supplementary, so B is supplementary to label angles 1, 2, 3,
BAD and mBAD  mCAD  mBAC. and 4 as shown.
Find y. Q
3. P SR, P
S
Q R 3. Opp. sides of  are .
mBAD  mB ¬180 4. 1  2, and 4. Alt. int.  are .
mCAD  mBAC  mB ¬180 3  4
21  2y  120 ¬180 R
5. P P R 5. Reflexive Prop.
2y ¬39
6. QPR  SRP 6. ASA
y ¬19.5
Q
7. PR S and 7. CPCTC
37. The Distance Formula is
R
Q S P

d  (x 2 x1)2 (y2 y1)2.
The diagonals bisect each other if EQ  QG and 42. Given: GKLM
HQ  QF. Prove: G and K are supplementary.
Find these measures. K and L are supplementary.
EQ   (3  0) 2  (1 5)2 L and M are supplementary.
5 M and G are supplementary.
QG   (6  3) 2  (3  1)2 G K
5
HQ   (3  0) 2  [1 2
 (1)]
M L
  13
Proof:
QF   (6  3) 2  (3 1)2
  13 Statements Reasons
The diagonals do indeed bisect each other. 1. GKLM 1. Given
38. If EG  FH, the diagonals are congruent. Use the K
2. GM
L
, 
GMK
L
 2. Opp. sides of 
Distance Formula to find EG and FH. are .
d   (x2  x1)2 (y2 y1)2 3. G and K are 3. Cons. int.  are
EG   (6  0) 2  (3  5)2 supplementary. suppl.
 10 K and L are
FH   (0  6) 2  (1  3)2 supplementary.
  52 L and M are
 2  13 supplementary.
M and G are
No, the diagonals are not congruent, since EG  FH.
supplementary.
H
39. E  is vertical, so its slope is undefined. Find the
F
slope of E .
y y
m ¬
2
1
x2  x1
3  5
¬
60
¬1
3

No, the consecutive sides are not perpendicular
because the slopes of the sides are not opposite
reciprocals of each other.

Chapter 8 246
43. Given: MNPQ 4. HFG and DJK 4.  lines form four rt.
M is a right angle. are rt. . .
Prove: N, P and Q are right angles 5. HFG and DJK 5. Def. of rt. 
M N are rt. s.
6. HFG  DJK 6. HA
Q P
47. Given: BCGH, HDF
D

N
Proof: By definition of a parallelogram, M Q P.
Prove: F  GCB
Since M is a right angle, M
QM N. By the
Perpendicular Transversal Theorem, M QQ P. H
B
Q is a right angle, because perpendicular lines
G
form a right angle. N  Q and M  P
because opposite angles in a parallelogram are D F
C
congruent. P and N are right angles, since all Proof:
right angles are congruent.
Statements Reasons
44. Given: ACDE is a parallelogram.
Prove: EC bisects A
D
. A
D
 bisects E
C. 1. BCGH 1. Given

HDFD

A C
B 2. F  H 2. Isosceles Triangle
Theorem
E D 3. H  GCB 3. Opp.  of  are .
Proof: It is given that ACDE is a parallelogram.
4. F  GCB 4. Congruence of angles
Since opposite sides of a parallelogram are is transitive.
congruent, E
AD C. By definition of a
parallelogram, EA
D C. AEB  DCB and 48. MSR  PST because they are vertical angles.
EAB  CDB because alternate interior angles NMP  MPQ because they are alternate
are congruent. EBA  CBD by ASA. E BB C interior angles. So, MSR is similar to PST.
and AB
B D by CPCTC. By the definition of N
M Q P because opposite sides of a
C
segment bisector, E  bisects A
D and AD bisects parallelogram are congruent. TP  1 
2 QP is given,
C
E .
so TP  1

2 MN.
45. Given: WXYZ
M.
S
Prove: WXZ  YZX Find 
SP
MS MR
SP  TP
W X  
1
MN
 
4
1
Z Y MN
2
Proof:
 1

2
Statements Reasons
The ratio of MS to SP is 1

2.
1. WXYZ 1. Given
49. Sample answer: The graphic uses the illustration
X
2. WZY
, W
Z
XY
 2. Opp. sides of  are . of wedges shaped like parallelograms to display
3. ZWX  XYZ 3. Opp.  of  are . the data. Answers should include the following.
4. WXZ  YZX 4. SAS • The opposite sides are parallel and congruent,
the opposite angles are congruent, and the
46. Given: DGHK is a parallelogram.
consecutive angles are supplementary.
H
FGD
• Sample answer:
J
D H
K

100
Prove: DJK  HFG 79%
80
Use the Web
Percent That

G F D 60%
60
40 29%
H J K 20
Proof: 0
1998 1999 2000
Statements Reasons Year
1. DGHK is a 1. Given 50. Consecutive angles of a parallelogram are
parallelogram. supplementary. Find x
H
FG D, D
JH
K
 (3x  42)  (9x  18) ¬180
2. G  K 2. Opp.  of  are . 12x  24 ¬180
H
3. G D K 3. Opp. sides of  are . 12x ¬156
x ¬13
3(13)  42  81
9(13)  18  99
The measures of the angles are 81 and 99.

247 Chapter 8
51. B; the perimeter p is equal to the sum of the 57. Since the measures of two sides and an angle
measures of the sides. Find y. opposite one of the sides are known, use the
2x  2y ¬p Law of Sines.
y
2  2y ¬p  C
sin sin
c ¬ b
 B
5
2y  2y ¬p sin
C sin 57°
14 ¬ 12.5
  
5
12y 14 sin
57°
 ¬p sinC ¬ 12.5
5
5p
y ¬
12 C ¬sin1 12.5 
14 sin
57°

mC ¬69.9
Use the Angle Sum Theorem.
Page 416 Maintain Your Skills mA  mB  mC ¬180
52. Use the Interior Angle Sum Theorem. mA  57  69.9 ¬180
S  180(n  2) mA ¬53.1
 180(14  2) sinB A
sin
b ¬ a
 2160  
The sum of the measures of the interior angles of sin 57° sin53.1°
12.5 ¬
  
a
a 14-gon is 2160.
12.5 sin
53.1°
53. Use the Interior Angle Sum Theorem. a ¬ sin 57°
S  180(n  2) a ¬11.9
 180(22  2)
58. Since the measures of two sides and the included
 3600
angle are known, use the Law of Cosines.
The sum of the measures of the interior angles of
c2  a2  b2  2ab cos C
a 22-gon is 3600.
c2  212  242  2(21)(24) cos 78°
54. Use the Interior Angle Sum Theorem. c ¬ 212 242 2(21)(24) cos
78°
S  180(n  2) c ¬28.4
 180(17  2)
 2700 a2 ¬b2  c2  2bc cos A
The sum of the measures of the interior angles of 212 ¬242  28.42  2(24)(28.4) cos A
a 17-gon is 2700. 941.56 ¬1363.2 cos A
941 .56
1363.2 ¬cos A
55. Use the Interior Angle Sum Theorem.  
S  180(n  2)

941
.56
 180(36  2) cos1  
1363.2 ¬A
 6120 46.3 ¬mA
The sum of the measures of the interior angles of
Use the Angle Sum Theorem.
a 36-gon is 6120.
mA  mB  mC ¬180
56. Since the measures of two sides and the included 46.3  mB  78 ¬180
angle are known, use the Law of Cosines. mB ¬55.7
a2  b2  c2  2bc cos A
59. The numbers of the outside diagonals are all ones,
a2  112  132  2(11)(13) cos 42°
so the first thirty numbers sum to 30.
a  112  132  
2(11)(13) cos
42°
a  8.8 60. The second diagonal consists of the natural
numbers, 1, 2, 3, 4, 5, . . . , 70.
b2 ¬a2  c2  2ac cos B
The sum is 1  2  3  4  . . .  67  68  69  70
112 ¬8.82  132  2(8.8)(13) cos B
¬(1  70)  (2  69)  (3  68)  . . .  (35 + 36)
125.44 ¬2(8.8)(13) cos B
¬35(71)
125 .44
228.8 ¬cos B
  ¬2485
cos1228.8  ¬B
125
.44 The sum of the first 70 numbers is 2485.

56.8 ¬mB
Use the Angle Sum Theorem.
mA  mB  mC ¬180
42  56.8  mC ¬180
mC ¬81.2

Chapter 8 248
61. y 5. Opposite sides are parallel and congruent,
8 opposite angles are congruent, or consecutive
B 4 angles are supplementary.

8 4 O 4 8x Pages 420–421 Check for Understanding


A C
4 1. Both pairs of opposite sides are congruent; both
pairs of opposite angles are congruent; diagonals
D bisect each other; one pair of opposite sides is
parallel and congruent.
 is a side. Find the slope of A
AB B
. 2. Sample answer:
y y
m
2
1
B C
x2  x1
5  (2)
 
2  (5) A D
7
 3 3. Shaniqua; Carter’s description could result in a
62. y shape that is not a parallelogram.
8 4. No; one pair of opposite sides is not parallel and
B 4 congruent.
5. Yes; the missing angle measure of the
parallelogram is 180  102 or 78, so each pair of
8 4 O 4 8x opposite angles is congruent.
A C
4 6. Opposite sides of a parallelogram are congruent.
Find x and y.
D 2x  5  3x  18
13  x
D
B is a diagonal. Find the slope of B
D
. 2y  12 ¬5y
y y
m
2
1
12 ¬3y
x2  x1
4 ¬y
9  5

1  (2) 7. Opposite angles of a parallelogram are congruent.
 14 Find x and y.
3x  17  2x  24
63.
8
y x  41
5y  6 ¬y  58
B 4 4y ¬64
y ¬16
8 4 O 4 8x
8. Yes;
A C y
4

E D
D
D
C is a side. Find the slope of C
D
.
y y
m
2
1
C
x2  x1
9  (2) B O x
 1  2
 7
3

If the opposite sides of a quadrilateral are
parallel, then it is a parallelogram.
y y
C
slope of B
2
1

8-3 x2  x1
Tests for Parallelograms 1 
0
 40

Page 417 Geometry Activity: Testing for a  1


4


Parallelogram 4 5
E
slope of D  26
1. They appear to be parallel.
 1
4

2. The quadrilaterals formed are parallelograms.
4 
0
3. The measures of pairs of opposite sides are equal. E
slope of B 20
4. Opposite angles are congruent, and consecutive 2
5 
1
angles are supplementary. D
slope of C64
2

249 Chapter 8
C
Since opposite sides have the same slope, B  D
E
 11. Given: P
TT
R

and BE
 CD. Therefore, BCDE is a parallelogram TSP  TQR
by definition. Prove: PQRS is a parallelogram.
9. Yes; P Q
y T

C S R
D Proof:
Statements Reasons
B 1. 
PTT
R
, 1. Given
A O x TSP  TQR
2. PTS  RTQ 2. Vertical angles are
congruent.
First use the Distance Formula to determine 3. PTS  RTQ 3. AAS
whether the opposite sides are congruent.
S
4. P Q
R 4. CPCTC
AD  
(x2  
x1)2 
(y2 
y1)2 S
5. P Q
R
 5. If alternate interior
  [6  (4)]2  (3 0)2 angles are congruent,
 13 lines are parallel.
BC   (1  3
)2  (4 1)2 6. PQRS is a 6. If one pair of opposite
parallelogram. sides is parallel and
  13
congruent, then the
Since AD  BC, A DB C
.
quadrilateral is a
Next, use the Slope Formula to determine
parallelogram.
whether AD B
C.
y y 12. If one pair of opposite sides is congruent and
D
slope of A
2
1
x2  x1 parallel, the quadrilateral is a parallelogram.
30
6  (4)
 3
2 Pages 421–423 Practice and Apply
4 
slope of B 11
C 
3 13. Yes; each pair of opposite angles is congruent.
3 14. Yes; the diagonals bisect each other.
 2

AD and BC have the same slope, so they are 15. Yes; opposite angles are congruent.
parallel. Since one pair of opposite sides is 16. No; none of the tests for parallelograms are
congruent and parallel, ABCD is a parallelogram. fulfilled.
10. No; 17. Yes; one pair of opposite sides is parallel and
y
congruent.
G 18. No; none of the tests for parallelograms are
H fulfilled.
19. Opposite sides of a parallelogram are congruent.
x Find x and y.
O
2x ¬5x  18
F 18 ¬3x
E 6 ¬x
3y ¬96  y
If the midpoints of the diagonals are the same, 4y ¬96
the diagonals bisect each other. If the diagonals of y ¬24
a quadrilateral bisect each other, then the 20. Diagonals of a parallelogram bisect each other.
quadrilateral is a parallelogram. Find x and y.
Find the midpoints of EG and F
H. 2x  3 ¬5x
x x y y 3  (3) 3 ¬3x
G
E: 
1
 2 1 2 4  2

2, 2 ¬2, 2 1 ¬x
¬(1, 0) 4y ¬8y  36
x1  x2 y1  y2 4  (6) 1  2 36 ¬4y
H
F:   
2, 2 ¬ 2, 2  9 ¬y
¬1, 1
2

The midpoints of EG
 and F
H
 differ, so EFGH is
not a parallelogram.

Chapter 8 250
21. Opposite sides of a parallelogram are congruent. 25. Yes;
Find x and y. G y E
y  2x ¬4
y ¬2x  4
5y  2x ¬3y  2x
2y ¬4x O x
y ¬2x
Find x.
B C
2x ¬2x  4
4x ¬4 If the opposite sides of a quadrilateral are
x ¬1 parallel, then it is a parallelogram.
So, y  2(1) or 2. y y
C
slope of B
x 
2
x
1
22. Since the opposite sides of a parallelogram are 2 1
parallel, there are two pairs of alternate interior 3  (3)
¬ 
2  (6)
angles formed by the diagonal of the
parallelogram. Find x and y. ¬0
4 4
25x ¬100 G
slope of E
4  4
x ¬4 ¬0
10y ¬40 4  (3)
G
slope of B 
4  (6)
y ¬4
¬7

2
23. Since the opposite sides of a parallelogram are
4  (3)
parallel, there are two pairs of alternate interior E
slope of C 
42
angles formed by the diagonal of the
parallelogram. Find y in terms of x. ¬7

2
1 y ¬x  12 C
Since opposite sides have the same slope, B EG

2 G
and B C E. Therefore, BCEG is a parallelogram
y ¬2x  24 by definition.
Opposite angles of a parallelogram are congruent. 26. No;
Find another equation for y in terms of x.
y
3y  4 ¬4x  8 8
S
3y ¬4x  4 4
y ¬4 4
3x  3
 T R

Find x by setting the two expressions for y equal 8 4 O x


to each other 4
4
4x  4 ¬2x  24 Q
3 3 8
68
  ¬2x
3 3
If the opposite sides of a quadrilateral are
34 ¬x
parallel, then it is a parallelogram.
So, y  2(34)  24 or 44. y y
R
slope of Q
x 
2
x
1
24. Diagonals of a parallelogram bisect each other. 2 1
2  (6)
Find y in terms of x. ¬ 
2  (3)
3y  4 ¬x
¬8

3y ¬x  4 5
26
y ¬1 4 T
slope of S 5 
3x  3
 (1)
¬1
4y ¬2

3x R
The opposite sides, Q  and S
T
, do not have the
y ¬1

6x same slope, so they are not parallel. Therefore,
Find x by setting the two expressions for y equal QRST is not a parallelogram.
to each other. 27. Yes;
1x ¬1x  4 y C
6 3 3
4 ¬1x D
3 6
8 ¬x
So, y  1
 1
6 (8) or 1 3 . O x

A
If both pairs of opposite sides of a quadrilateral
are congruent, then the quadrilateral is a

251 Chapter 8
parallelogram. Use the Distance Formula to 30. No;
determine whether the opposite sides are y
congruent. 8
H
AD ¬  (x2  
x1)2 (y2 y1)2 4
¬ [4  (5)]2  [2 2
 (4)] J
¬ 37 O 4 8 x
BC ¬  (4  3
)2  [4  (2)]2 4 L
K
¬ 37 8
AB ¬  [3  (
5)]2 
 [2 2
 (4)]
¬ 68 If both pairs of opposite sides of a quadrilateral
are congruent, then the quadrilateral is a
CD ¬  (4 4)2  (2  4
)2
parallelogram. Use the Distance Formula to
¬ 68 determine whether the opposite sides are
Since the measures of both pairs of opposite sides congruent.
are equal, ABCD is a parallelogram. HJ ¬  (x2  
x1)2 (y2 y1)2
28. Yes; ¬  (9  5
)  (0
2  6)2

y ¬  52
KL ¬  (3  8
)2  [2  (5)]2
OY x
¬  34
Z
Since HJ  KL, H J
K L
. Therefore, HJKL is not
a parallelogram.
X 31. Yes;
W
S y T

If the midpoints of the diagonals are the same,


the diagonals bisect each other. If the diagonals of
a quadrilateral bisect each other, then the
W V
quadrilateral is a parallelogram.
Find the midpoints of WY and X
Z.
x1  x2 y1  y2 5  (1) O x
Y
W:  
  ¬ 
2 , 2
6 

2 ,
0  
2    If the midpoints of the diagonals are the same,
¬(3, 3) the diagonals bisect each other. If the diagonals of
x1  x2 y1  y2 1  (5) 4  (2) a quadrilateral bisect each other, then the
Z
X:  
2, 2 ¬ 2, 2    quadrilateral is a parallelogram.
¬(3, 3) Find the midpoints of SV and W
T.
x1  x2 y1  y2
Y
The midpoints of W  and X
WXYZ is a parallelogram.
Z are the same, so V
S:     1  6 9  2
2, 2 ¬ 2, 2 
29. No; ¬5
2, 2 
 1
1

x x y y
G 8 y : 2, 2 ¬
T
W 1
2 , 2 
2 3 
2 3 
8 1 2

4 H ¬5
2, 2 
11
 
V
The midpoints of S and W
T
 are the same, so
STVW is a parallelogram.
4 O 4 8 12 x
J 32. Yes;
4
y
K
8 C
D
First use the Distance Formula to determine
whether the opposite sides are congruent. O x
GH ¬  (x2  
x1)2 
(y2 
y1)2
¬ 
[4  (
2)]2 
 (4 
8)2 G F
¬  52
JK ¬  (1 6)2 [7 (3)]2 First use the Distance Formula to determine
whether the opposite sides are congruent.
¬  65
CD ¬  (x2  
x1)2 
(y2 y1)2
Since GH  JK, G H

J K. Therefore, GHJK is not
a parallelogram. ¬ [3  (7)]
2  (2
 3)2
¬ 17

Chapter 8 252
FG ¬  (4  0)2 
[3 (4)]2 
slope of MR  slope of N
P

¬  17 8 y  (1)
1 ¬ x  (1)

 
Since CD  FG, C DFG
. 8 y1
Next, use the Slope Formula to determine
1 ¬ x 1
whether CD
F G. Again, x  0 and y  9.
y2  y1 So, move P to (0, 9).
D
slope of C x2  x1
2  3 N
Move R: slope of M  slope of P
R
¬
3  (7) y  (4)
7
5 ¬ x  (
2)
¬1
4
 y4
7
5 ¬ x  2
3  (4) 
 
G
slope of F ¬4  0

1
¬ 4
 x  7 and y  3.
D
C  and FG have the same slope, so they are R
slope of M  slope of N
P

parallel. Since one pair of opposite sides is y6 3
 
x  (6) ¬1
congruent and parallel, CDFG is a parallelogram. y6  3
33. y

x 6 ¬1
M
Again, x  7 and y  3.
So, move R to (7, 3).
34. y
Q
O x
S
N
R x
O
P
W
Sample answer: Hold N, P, and R fixed. Let M be T
N
M(x, y). Find the slopes of M and MR so that
they equal those of PR
 and NP
, respectively.
N
slope of M   slope of P
R Hold S, T, and W fixed. Let Q be Q(x, y). Find the
1  y 2  (4) S
slopes of Q  and Q
W so that they equal those of
  x ¬
1  
5  (2) W
T  and ST, respectively.
1y 2

1 x ¬
3 S
slope of Q  slope of TW
1y 1  (2)
If x  4 and y  1, then M
N
P
R
. 
4 x ¬ 
5  (1)
R
slope of M  slope of N
P 1y 1
2  y 4  (1)

4 x ¬
4

  x ¬
5  
2  (1) 1y  1
2y

4 x ¬4
3

5 x ¬1 If x  0 and y  2, then Q
S
T
W
.
If x  4 and y  1, then M
RN P. W
slope of Q  slope of ST

Move M to (4, 1) to make both pairs of opposite 1  y 2  1
  x ¬
5  
1  4
sides parallel. Then MNPR is a parallelogram. 1y  3
Perform the same process for the other vertices.

5 x ¬5
1y 3
Move N: slope of MN  slope of P
R 
5 x ¬5
y6 2
 
x  (6) ¬
3 If x  0 and y  2, then Q
W
S T. Move Q to (0, 2)
y6  2 to make both pairs of opposite sides parallel. Then

x 6 ¬3 QSTW is a parallelogram.
x  3 and y  4. Perform the same process for the other vertices.
R
slope of M  slope of N
P
 S
Move S: slope of Q   slope of T
W

2  6 4  y y3 1
 (6) ¬
5  2 
x  
x  (3) ¬4
4 y y3 1
3 ¬4
8 
1 ¬ 2  x
 x 
Again, x  3 and y  4. x  1 and y  2.
So, move N to (3, 4). W
slope of Q  slope of S
T

1  3 2  y
N
Move P: slope of M  slope of P
R
  (3) ¬
5  1 
x
1  6 2  y 4 2y
 ¬ 
 (6) ¬
1  5 
x 2 1x
 7 2y Again, x  1 and y  2.
5 ¬ 
5x
So, move S to (1, 2).
x  0 and y  9.

253 Chapter 8
S
Move T: slope of Q  slope of T
W
 D
slope of C  slope of A
B

1 
3 1  y y  (1) 5 
4
3) ¬ 5  x 4 ¬

4  (   x  71
2 1y y1
7 ¬ 5 
1
x x 4 ¬6
x  2 and y  3. For (10, 0) and (2, 2), CDA B.
W
slope of Q  slope of S
T
 C
slope of B  ¬2
y1
4 ¬
x  Suppose D is (2, 2).
2 4
y1 4  (2)
 4
 ¬  slope of 
DA ¬ 
2 x4 1  (2)
¬2
Again, x  2 and y  3.
So 
DAB C.
So, move T to (2, 3)
So, (2, 2), (4, 10), and (10, 0) are the
S
Move W: slope of Q  slope of T
W
 possibilities for the fourth vertex. Any of these
2 y  (2) values results in both pairs of opposite sides being
7 ¬ x  (
1)
y2 parallel, and thus, the four points form a
2

7 ¬x1 parallelogram.
Again, x  8 and y  0. 36. y
W
slope of Q  slope of S
T

y3 2  1
 
x  (3) ¬ 
1  4 Q
y3 3 R

x 3 ¬
5
x  8 and y  0. O x
So, move W to (8, 0). S
35. y
B
A The fourth vertex can have one of three possible
positions to complete the parallelogram.
Let T(x, y) be the fourth vertex.
Find the slopes of ST
 and RT so that they equal
R
those of Q  and QS, respectively.
O x T
slope of S   slope of Q
R
C y  (1) 1 
2
 1) ¬
x  ( 1  (2)
The fourth vertex can have one of three possible y1  1

x 1 ¬3
positions to complete the parallelogram.
Let D(x, y) be the fourth vertex. For (2, 2) and (4, 0), S
TQ
R
.
1  2
Find the slopes of BD
 and CD
 so that they equal S
slope of Q ¬ 
1  (2)
C
those of A  and A
B, respectively. ¬3
D
slope of B  slope of AC
 Suppose T is (2, 2).
y5 1  4
 7 ¬
x  4 1 T
slope of R 2  1
 ¬ 
21
y5 5
 7 ¬3
x  ¬3
D
For (10, 0) and (4, 10), BA
C
. S
So QR
T
.
54 1  1
B
slope of A ¬  S
slope of R  ¬ 
71 1  1
¬1 ¬1
6
Suppose D is (10, 0). Suppose T is (4, 0).
0  (1) 02

slope of CD ¬  T
slope of Q ¬ 
10  4 4  (2)
1
¬ ¬1
6
B
So AC
D
. T
So QR
S.
1  5 T
slope of R  slope of Q
S


slope of BC ¬ 
47
y1 1  2
¬2  1 ¬
x  1 (2)
Suppose D is (4, 10). y1  3
10  4
 1 ¬1
x 
D
slope of A ¬ 
41
¬2
D
So AB
C
.

Chapter 8 254
For (2, 2) and (0, 4), R
TQ S. 41. Given: 
ABD
C
1  (1) B
A D
C


slope of S R ¬ 
1  (1)
¬1 Prove: ABCD is a parallelogram.
Suppose T is (0, 4). A 1 B
4 2
T
slope of Q  ¬0  (2) 2
¬1 D C
So S
RQ T. Proof:
So, (2, 2), (4, 0), and (0, 4) are the possibilities Statements Reasons
for the fourth vertex. Any of these values results B
1. AD C, A
B
D
C
 1. Given
in both pairs of opposite sides being parallel, and
C
2. Draw A 2. Two points determine
thus, the four points form a parallelogram.
a line.
37. JKLM is a parallelogram because K M and JL
 are
diagonals that bisect each other. 3. 1  2 3. If two lines are
parallel, then
38. If both pairs of opposite sides are parallel and
alternate interior
congruent, then the watchbox is a parallelogram.
angles are congruent.
39. Given: A D
B C  A B
3 4. 
ACA C 4. Reflexive Property
B
AD C  2
Prove: ABCD is a
1
4 5. ABC  CDA 5. SAS
D C 6. D
A B C 6. CPCTC
parallelogram.
Proof: 7. ABCD is a 7. If both pairs of
Statements Reasons parallelogram. opposite sides are
congruent, then the
D
1. AB C
, A
B
D
C
 1. Given quadrilateral is a
B
2. Draw D. 2. Two points determine parallelogram.
a line.
3. 
DBDB
 3. Reflexive Property 42. This theorem is not true. ABCD is a
4. ABD  CDB 4. SSS 
parallelogram with diagonal BD, ABD
CBD.
5. 1  2, 3  4 5. CPCTC A D
6. D
ABC
, A
BDC
 6. If alternate interior
angles are congruent, B C
lines are parallel. 43. Given: ABCDEF is a regular hexagon.
7. ABCD is a 7. Definition of Prove: FDCA is a parallelogram.
parallelogram. parallelogram A B
40. Given: 
AEE
C, D
EE B
F C
Prove: ABCD is a parallelogram.
A B
E E D
1 3 2
4 Proof:
D C Statements Reasons
Proof: 1. ABCDEF is a 1. Given
Statements Reasons regular hexagon.
1. 
AEE C, D
E
EB
 1. Given B
2. A D E
, B
C
E F 2. Definition of a
2. 1  2 2. Vertical  are . E  B, FACD regular hexagon
3  4 3. ABC  DEF 3. SAS
3. ABE  CDE 3. SAS C
4. A D F
 4. CPCTC
ADE  CBE 5. If both pairs of
5. FDCA is a .
B
4. A D C 4. CPCTC opposite sides are
D
A B C congruent, then the
5. ABCD is a 5. Definition of quadrilateral is a
parallelogram. parallelogram parallelogram.
44. Sample answer: The roofs of some covered bridges
are parallelograms. The opposite sides are
congruent and parallel. Answers should include
the following.
• We need to know the length of the sides, or the
measures of the angles formed.
• Sample answer: windows or tiles

255 Chapter 8
45. B; 51. Use the Interior Angle Sum Theorem to write an
y equation to solve for n, the number of sides.
S ¬180(n  2)
(135)n ¬180(n  2)
(8, 2) 135n ¬180n  360
(2, 2)
0 ¬45n  360
O x 360 ¬45n
8 ¬n
( 1 , 6 ) The polygon has 8 sides.
52. Use the Interior Angle Sum Theorem to write an
equation to solve for n, the number of sides.
By plotting choices A, C, and D on the graph, it is S ¬180(n  2)
obvious that they cannot be the fourth vertex. (144)n ¬180(n  2)
Choice B is the only possibility. 144n ¬180n  360
(2, 2) and (8, 2) have the same y-component. 0 ¬36n  360
These are the endpoints of a horizontal segment 360 ¬36n
10 units long. 10 ¬n
(11, 6) and (1, 6) have the same y-component. The polygon has 10 sides.
These are also the endpoints of a horizontal
53. Use the Interior Angle Sum Theorem to write an
segment 10 units long.
equation to solve for n, the number of sides.
These two sides of the quadrilateral are
S ¬180(n  2)
congruent and parallel, so the quadrilateral is a
(168)n ¬180(n  2)
parallelogram.
168n ¬180n  360
46. C; use the Distance Formula to find the distance 0 ¬12n  360
between X and Y. 360 ¬12n
XY ¬ (x 2 x1)2 (y2 
y1)2 30 ¬n
¬ 
(3 
5)2 
(4 
7)2 The polygon has 30 sides.
¬ 185
 54. Use the Interior Angle Sum Theorem to write an
equation to solve for n, the number of sides.
S ¬180(n  2)
(162)n ¬180(n  2)
Page 423 Maintain Your Skills 162n ¬180n  360
47. The diagonals of a parallelogram bisect each 0 ¬18n  360
other, so ML  LQ. 360 ¬18n
Find w. 20 ¬n
ML ¬LQ The polygon has 20 sides.
w ¬12
55. Use the Interior Angle Sum Theorem to write an
48. Opposite sides of a parallelogram are congruent, equation to solve for n, the number of sides.
so their measures are equal. S ¬180(n  2)
Find x. (175)n ¬180(n  2)
NQ ¬MR 175n ¬180n  360
3x  2 ¬4x  2 0 ¬5n  360
4 ¬x 360 ¬5n
49. Opposite sides of a parallelogram are congruent, 72 ¬n
so their measures are equal. The polygon has 72 sides.
Find x. 56. Use the Interior Angle Sum Theorem to write an
NQ ¬MR equation to solve for n, the number of sides.
3x  2 ¬4x  2 S ¬180(n  2)
4 ¬x (175.5)n ¬180(n  2)
So, NQ is 3(4)  2 or 14 units. 175.5n ¬180n  360
50. Opposite sides of a parallelogram are congruent, 0 ¬4.5n  360
so their measures are equal. 360 ¬4.5n
Find y. 80 ¬n
QR ¬MN The polygon has 80 sides.
3y ¬2y  5 57. The legs of this right triangle have equal lengths,
y ¬5 so it is a 45°-45°-90° triangle. The hypotenuse of a
So, QR is 3(5) or 15 units.
45°-45°-90° triangle is  2 times the length of the
legs. Therefore, x  45 and y  12  2.
58. The hypotenuse of this right triangle is twice the
length of one of its legs, so it is a 30°-60°-90°
triangle. The longer leg is  3 times the length of
the shorter leg. Therefore, x  10  3 and y  30.

Chapter 8 256
59. Two angles are given, 60° and 90°, so this is a 2. Opposite sides of a parallelogram are congruent,
30°-60°-90° triangle. The shorter leg is half the so their measures are equal. Find x.
length of the hypotenuse, and the longer leg is WZ ¬XY
3 times the length of the shorter leg. Therefore, x2 ¬42  x
x  16  3 and y  16. 0 ¬x2  x  42
0 ¬(x  6)(x  7)
60. Determine the slopes of AB and B
C.
y y 0  x  6 or 0  x  7
B
slope of A ¬
x 
2
x
1
6  x 7x
2 1
3 
¬ 5 x  6 or 7, so x2  36 or 49.
62 So, WZ is 36 or 49.
¬1

2 3. YXZ  XZW because they are alternate
7 
3 interior angles. So, mYXZ is 54 and mWXY is
C
slope of B
86
54  60 or 114. WXY and XYZ are
2
supplementary, so mXYZ is 180  114 or 66.
B
The product of the slopes of A and BC
 is 1, so
4. Opposite angles of a parallelogram are congruent.
B
AB C. Find x and y.
B
61. Determine the slopes of A  and B
C. 5x  19 ¬3x  9
y y
B
slope of A ¬
x 
2
x
1 2x ¬28
2 1
7 
2 x ¬14
¬
0  (1) 6y  57 ¬3y  36
¬5 3y ¬93
1 
7 y ¬31
C
slope of B 40
5. Opposite sides of a parallelogram are congruent.
 3

2 Find x and y.
B
The product of the slopes of A and B C
 is not 1, 2x  4 ¬x  4
B
so A is not perpendicular to BC
. x ¬8
B
62. Determine the slopes of A  and BC. 4y  8 ¬3y  2
y y y ¬6
B
slope of A ¬
x 
2
x
1
2 1
7 
4
¬
50
¬3

5 8-4 Rectangles
3 
7
C
slope of B
85
 4

3 Pages 427–428 Check for Understanding
B
The product of the slopes of A and B C
 is not 1, 1. If consecutive sides are perpendicular or
B
so A is not perpendicular to BC
. diagonals are congruent, then the parallelogram
B
63. Determine the slopes of A  and BC. is a rectangle.
y y 2. Sample answer:
B
slope of A ¬
x 
2
x
1
2 1
3  (5)
¬ 
1  (2)
2
¬3
0  (3)
C
slope of B 
1  1 Yes, the opposite sides are congruent and parallel,
3 consecutive sides are perpendicular.
 2
3. McKenna is correct. Consuelo’s definition is
B
The product of the slopes of A and B
C
 is 1, so correct if one pair of opposite sides is parallel and
B
AB C. congruent.
4. The diagonals of a rectangle are congruent, so
C
A B D.
Page 423 Practice Quiz 1 C
A  ¬B D

1. Use the Interior Angle Sum Theorem to write an AC ¬BD
equation to solve for n, the number of sides. 30  x ¬4x  60
S ¬180(n  2) 90 ¬5x
18 ¬x
147131n ¬180(n  2) 5. The diagonals of a rectangle bisect each other and
1620

11 n ¬180n  360
 are congruent, so NP  1 
2 NR and MP N P.
36
0 ¬ 0
11 n  360
NP ¬1

2 NR
360 ¬360 2x  30 ¬1
2 (2x  10)

11 n
11 ¬n x ¬35
The polygon has 11 sides. So, MP  2(35)  30 or 40.

257 Chapter 8
6. QRT  RTS because they are alternate 11. The diagonals of a rectangle bisect each other, so
interior angles. Q
N Q K.
QRT ¬RTS Q
N  ¬Q K
Q
mQRT ¬mRTS NQ ¬QK
x2  1 ¬3x  11 2x  3 ¬5x  9
x  3x  10 ¬0
2
12 ¬3x
(x  5)(x  2) ¬0 4 ¬x
x  5  0 or x  2  0 So, NQ  2(4)  3 or 11. J Q
N Q, so JQ  11.
x5 x  2
12. Opposite sides of a rectangle are congruent, so
So, x  5 or 2.
K
J N M.
7. QRT and SRT are complementary.
K
J ¬NM
mQRT  mSRT ¬90 JK ¬NM
Use the values of x found in Exercise 6.
x2  1 ¬8x  14
[(2)2  1]  mSRT ¬90
x2  8x  15 ¬0
5  mSRT ¬90
(x  3)(x  5) ¬0
mSRT ¬85
x  3  0 or x  5  0
or x3 x5
52  1  mSRT ¬90 So, x  3 or x  5. JK  32  1 or 10 or
26  mSRT ¬90 JK  52  1 or 26.
mSRT ¬64 13. The sum of the measures of NJM and KJM is
SRT  RSQ and the sum of the measures of 90. Find x.
the interior angles of a triangle is 180. mNJM  mKJM ¬90
Find mRPS. 2x  3  x  5 ¬90
mRPS  mSRT  mRSQ ¬180 3x ¬88
mRPS  2mSRT ¬180 x ¬291

3
mRPS  2(64) ¬180 14. KMN is a right triangle, so the sum of the
mRPS ¬52 measures of NKM and KNM is 90. Find x.
or mNKM  mKNM ¬90
mRPS  2(85) ¬180 x2  4  x  30 ¬90
mRPS ¬10 x2  x  56 ¬0
8. The diagonals of a rectangle are congruent. Use (x  8)(x  7) ¬0
the Distance Formula to determine whether the x  8  0 or x  7  0
diagonals of quadrilateral EFGH are congruent. x  8 x7
EG ¬  (x2  
x1)2 (y2 
y1)2 So, x  8 or x  7. mKNM  8  30 or 22 or
mKNM  7  30 or 37. KNM  JKN because
¬ 
[2  (
4)]2 
 [3 
(3)]2 they are alternate interior angles, so mJKN 
¬ 
72 22 or 37.
HF ¬ 
(x2  
x1)2 
(y2 
y1)2 15. The sum of the measures of JKN and NKM is
90. Find x.
¬ 
[3  (
5)]2 
 (1
 1)2
mJKN  mNKM ¬90
¬ 68 2x2  2  14x ¬90
The lengths of the diagonals are not equal, so 2x2  14x  88 ¬0
EFGH is not a rectangle. x2  7x  44 ¬0
9. The framer can make sure that the angles (x  4)(x  11) ¬0
measure 90 or that the diagonals are congruent. x  4  0 or x  11  0
x4 x  11
Since mNKM  14x, and the measure of an
angle must be positive, discard x  11.
Pages 428–430 Practice and Apply So, x  4.
10. The diagonals of a rectangle bisect each other and 16. m1  30.
Q
are congruent, so N Q M
.
17. 2 is complementary to 1 because consecutive
Q
N  ¬Q
M sides of a rectangle are perpendicular. So, m2
NQ ¬QM  90  30 or 60.
5x  3 ¬4x  6 18. The diagonals of a rectangle are congruent and
x ¬9 bisect each other. So, the triangles formed by the
So, NQ  5(9)  3 or 42. NK is twice NQ or 84. diagonals of a rectangle are isosceles. Therefore,
2  3. From Exercise 17, m3  60.
19. 4 is complementary to 3 because consecutive
sides of a rectangle are perpendicular.
m4  m3 ¬90
m4  60 ¬90
m 4 ¬30

Chapter 8 258
20. 5  1 because they are alternate interior To determine if DFGH is a rectangle, the points must
angles. So, m5  30. be connected in the order given. When the points are
21. WXZ is a right triangle, so the sum of the plotted and connected, it is clear that DFGH is not a
measures of 1 and 6 is 90. Therefore, quadrilateral. So DFGH is not a rectangle.
m6  90  30 or 60. 29. If the opposite sides of a quadrilateral are parallel
22. 7  3 because they are alternate interior and the diagonals of the quadrilateral are
angles. From Exercise 18, m7  60. congruent, then the quadrilateral is a rectangle.
H
Find the slopes of D , FG, GH, and DF
.
23. 8 is complementary to 7.
m8  m7 ¬90 y
m8  60 ¬90 F 8
G
m8 ¬30
24. The sum of the interior angles of a triangle is 180. 4
m9  m7  m6 ¬180 x
m9  60  60 ¬180
8 4 4 8
m 9 ¬60 H
D 4
25. The contractor can measure the opposite sides
and the diagonals to make sure they are
congruent. y y
H
slope of D
x 
2
x
1
2 1
26. Use the Pythagorean Theorem to find the 2  (3)
measure of the diagonal of the television screen. ¬ 
7  (4)
c ¬ a2  b2 1
¬
11
¬ 21
 2 362  441 1296 G
slope of F  9 
8
6  (5)
¬ 1737 1

¬42 11
9
2 
The measure of the diagonal is about 42 in. H
slope of G
76
27. Opposite sides of a rectangle are parallel. Find  11
the slopes of DH
 and F
G. 8  (3)
F
slope of D
5 
(4)
y
8
F  11
G 4 H
So, DF G
 and G
H
D F
. Use the Distance
Formula to determine whether the diagonals of
H x
quadrilateral DFGH are congruent.
8 4 O 4 8
D DG ¬ 
(x2  
x1)2 
(y2 
y1)2
4
¬  
[6  (4)]2
 [9 
(3)]2
8
¬ 244

y2  y1
H
slope of D x2  x1 FH ¬ 
(x2  
x1)2 
(y2 
y1)2
1  (1)
¬ 6  9
  ¬ 
[7  (
5)]2 
 (2
 8)2
¬ 2 ¬ 244
15
y2  y1 G
So, DF H, D
HF
G, and G
H
 DF. Therefore,
G
slope of F x2  x1 DFGH is a rectangle.
¬5   5 30. Use the Distance y
6  9 Formula to find 8
¬0 WY and XZ. W Z
H
The slopes are not equal. Therefore, D and FG 4
are not parallel. So, DFGH is not a rectangle. X x
28. The diagonals of a rectangle are congruent. Use 8 4 4 8
the Distance Formula to determine whether the 4
diagonals of quadrilateral DFGH are congruent.
Y
y 8
8
G
4 WY ¬ 
(x2  
x1)2 
(y2 
y1)2
D H
x ¬ 
(1 
2)2 
(7 
4)2
4 12 ¬ 130

F
4
XZ ¬ (x
 2
x1)2 
(y2 
y1)2
8 ¬ 
(3  0
)2  [9
 (2
)]2
¬ 130


259 Chapter 8
31. See the figure in Exercise 30. Find the So by transitivity of parallel lines and
Y
coordinates of the midpoints of W  and X
Z
. G
substitution, HE F and HG  EF. So GHEF is
x x y y
   2  (1) 4  (7)

a parallelogram. Since H and E are midpoints, by
Y
W: 
1
 2
2 ,

1
 2
 2, 2
2 the Triangle Midsegment Theorem, HE  1 2
(BD)
 1 
3
2, 2
  and GF  12
(BD). Since AC  BD, HE  1(AC)
2
x1  x2 y1  y2 and GF  21(AC). Therefore, HE  GH  GF  EF.
Z
X:     
2 , 2  
2  9 
2 , 2
0 
3
 EFGH is a parallelogram with all sides congruent.
 7
 3
2, 2   35. To make the diagonals the same, either A C must
32. The midpoints of the diagonals are not the same be shortened or BD must be lengthened. This can
(Exercise 31), so the diagonals do not bisect be accomplished by moving L and K until the
each other. Therefore, WXYZ is not a rectangle. length of the diagonals is the same.
36. Find the ratio of the length to the width of the
33. Consecutive sides of a rectangle are
rectangle.
B
perpendicular. Find the slopes of A, B
C
, C
D
, and
19.
42
12.01  1.617
A
D. 
y Since 1.617 is close to 1.618, the rectangle is a
C golden rectangle. Use the Pythagorean Theorem
to find the length of the diagonal.
c  a  
2  b2
D x
 19.422 2
 12.01
O
B  22.83
The length of the diagonal is about 22.83 ft.
37. See students’ work.
A
y y 38. Parallelograms have opposite sides congruent and
B
slope of A
x 
2
x
1
bisecting diagonals, so the minimal requirements
2 1
1  (4) to justify that a parallelogram is a rectangle are
 
2  (4) that diagonals are congruent or the parallelogram
 1

2
has one right angle.
3  (1) 39. Sample answer:
C
slope of B 
02 A B
 2
03 D C
D
slope of C
6 
0 
ACBD
 but ABCD is not a rectangle.
 1

2 Y
40. Given: WXYZ is a rectangle with diagonals W
A4  0 Z
and X.
slope of D 4 
(6) Y
Prove: W X Z

 2
X Y
The consecutive sides are perpendicular.
Therefore, ABCD is a rectangle.
W Z
34. y Proof:
C
G Statements Reasons
F 1. WXYZ is a rectangle 1. Given
D O x Y
with diagonals W 
and XZ
.
B
H X
2. W Z Y
 2. Opp. sides of  are
E .
A Z
3. W W Z 3. Reflexive Property
Draw diagonals AC and B
D. ABCD is a rectangle. 4. XWZ and YZW 4. Def. of rectangle
C
So AB D and AC  BD. Since G and H are are right angles
midpoints, by the Triangle Midsegment Theorem 5. XWZ  YZW 5. All right  are .
G
HA and HG  1
C 
2 (AC) 6. XWZ  YZW 6. SAS
Since E and F are midpoints, Y
7. W X Z
 7. CPCTC
F
EA and EF  1
C 
2 (AC)

Chapter 8 260
X
41. Given: WYZ, X
YW Z, and W
Y
X
Z
 43. Given: DEAC and FEAB are rectangles.
Prove: WXYZ is a rectangle. GKH  JHK
J
G and H
K
 intersect at L.
W X Prove: GHJK is a parallelogram.

Z Y MA N
E
Proof:
F
Statements Reasons BC D
K G
1. 
WX Y Y
Z, XW Z
, 1. Given
and WYX Z J
L H
X
2. W W X 2. Reflexive Property
Proof:
3. WZX  XYW 3. SSS
Statements Reasons
4. ZWX  YXW 4. CPCTC
1. DEAC and FEAB 1. Given
5. mZWX  mYXW 5. Def. of 
are rectangles.
6. WXYZ is a 6. Def. of parallelogram GKH  JHK
parallelogram J
G  and HK
 intersect
7. ZWX and YXW 7. Consec.  of  are at L.
are supplementary suppl. E
2. D A C
 and 2. Def. of parallelogram
8. mZWX  8. Def. of suppl. E
FA B

mYXW  180 3. plane   plane  3. Def. of parallel plane
9. ZWX and YXW 9. If 2  are  and 4. G, J, H, K, L are in 4. Def. of intersecting
are right angles suppl., each  is a the same plane. lines
rt. .
H
5. G K J
 5. Def. of parallel lines.
10. WXYZ is a rectangle 10. Def. of rectangle
G
6. K H J
 6. Alt. int.  are .
42. Given: PQST is a rectangle. 7. GHJK is a 7. Def. of parallelogram
R
QVT parallelogram.
R
Prove: P V
S
 44. Explore: We need to find the number of
Q R S rectangles that can be formed using four of the
twelve points as corners.
Plan: Count the number of rectangles with each
possible set of dimensions. Arrange the data in a
table
P V T Solve:
Proof: Dimensions Number
Statements Reasons length  width of rectangles
1. PQST is a rectangle. 1. Given 32 1
R
Q V T
 31 2
2. PQST is a 2. Def. of rectangle
22 2
parallelogram.
S
3. T P Q 3. Opp. sides of  are 21 4
. 12 3
4. T and Q are 4. Definition of 11 6
rt. s. rectangle
There are 18 rectangles formed by using the rows
5. T  Q 5. All rt.  are .
and columns as sides.
6. RPQ  VST 6. SAS There are 2 additional rectangles formed as
R
7. P V S 7. CPCTC shown.

Examine: The table covers all possible


dimensions. The figure shows the only rectangles
that can be formed using rectangles at an angle to
the rows and columns. So the answer is
reasonable.

261 Chapter 8
45. No; there are no parallel lines in spherical 57. Opposite sides of a parallelogram are congruent,
geometry. so their measures are equal. Find x.
46. AC appears to be shorter than TR, so AC TR. AB ¬CD
5x ¬25
47. Since the sides would not be parallel in spherical
x ¬5
geometry, a rectangle cannot exist.
58. 
ST is the altitude to side Q
R
 in right triangle
48. Sample answer: The tennis court is divided
QSR so by Theorem 7.2, its measure is the
into rectangular sections. The players use the
geometric mean between the two segments of the
rectangles to establish the playing area. Answers
hypotenuse. Let x  ST.
should include the following.
18 x
x ¬ 34

• Not counting overlap, there are 5 rectangles on
each side of a tennis court. x2 ¬612
• Measure each diagonal to make sure they are x ¬ 612

the same length and measure each angle to
x ¬24.7
make sure they measure 90.
59. 
NP is the altitude to side M
O
 in right triangle
B
49. A; since AC E, ADE  BAD and
MNO so by Theorem 7.2, its measure is the
BDC  ABD because they are alternate
geometric mean between the two segments of the
interior angles. It is given that ADE  BAD, so
hypotenuse.
BAD  ABD. Therefore, ABD is isosceles and
DB  DA, so DB is 6. Let x  NP.
11 x
50. D; since s is the shorter side of the playground, x ¬
2
7
s  10 is the longer side. The perimeter of the x2 ¬297
fence is 80 feet, and the perimeter of a rectangle
is equal to twice the width plus twice the length. x ¬ 297

Therefore, the equation to find s is x ¬17.2
2(s  10)  2s  80. B
60. The measure of the altitude to A  is the
geometric mean between the two segments of the
hypotenuse of ABC.
Page 430 Maintain Your Skills Let x  the measure of the altitude.
24 x
51. There are 31 parallelograms: 11 individual x ¬
1
4
parallelograms, 12 using two others, 6 using three x2 ¬336
others, and 2 using four others.
x ¬ 336
52. The sum of the measures of the internal angles of x ¬18.3
a triangle is 180. Find mAFD.
61. Use the Distance Formula to find the distance
mAFD  mFDA  mDAF ¬180 betweeen the given points.
mAFD  34  49 ¬180
mAFD ¬97 d ¬ 
(x2  
x1)2 
(y2 
y1)2

53. ACD  BAC because they are alternate ¬ 


(3 
1)2  2
[1  (2)]
interior angles. mACD  54. So, mBAC  54. ¬5
BAD is supplementary to ADC. Find mCDF. The distance between the points is 5 units.
mCDF  mFDA  mDAF  mBAC ¬180 62. Use the Distance Formula to find the distance
mCDF  34  49  54 ¬180 between the given points.
mCDF ¬43
d ¬ 
(x2  
x1)2 
(y2 
y1)2
54. FBC  ADF because they are alternate
interior angles. mADF  34. So, mFBC  34. ¬ 
[5  (
5)]2 
 (12
 9)2

55. BCF  DAF because they are alternate ¬ 109



interior angles. mDAF  49. So, mBCF  49. ¬10.4
56. Opposite sides of a parallelogram are congruent, The distance between the points is 109
 or about
so their measures are equal. Find y. 10.4 units.
BC ¬AD 63. Use the Distance Formula to find the distance
3y  4 ¬29 between the given points.
3y ¬33 d ¬ 
(x2  
x1)2 
(y2 
y1)2
y ¬11
¬ 
(22 
1)2 
(24 
4)2
¬29
The distance between the points is 29 units.

Chapter 8 262
QM ¬ 
(3  0
)2  (0
 3)2
8-5 Rhombi and Squares ¬ 
9  9 ¬3 2

Use slope to determine whether the consecutive
Page 434 Check for Understanding sides are perpendicular.
03
1. Sample answer: N
slope of M
3 
0
Square 1
(rectangle 3  0
P
slope of N
0  (
3)
with 4
sides)  1
0
3 
Rectangle Rhombus
Q
slope of P 
03
( with ( with 4 1
1 right ) sides) 0 
3
M
slope of Q
30
 1
Parallelogram
(opposite sides || ) Since the slopes of MN and PQ
 are opposite
P
reciprocals of the slopes of N  and Q
M
,
2. Sample answer: consecutive sides are perpendicular. The lengths
of the four sides are the same, so the sides are
congruent. MNPQ is a rectangle, a rhombus, and
a square.
3. A square is a rectangle with all sides congruent. 9. y
4. The sides of a rhombus are congruent. Find x. N
AB ¬BC P
2x  3 ¬5x
3 ¬3x
M O x
1 ¬x
The value of x is 1. Q

5. From Exercise 1, x  1.
So, BC  5(1) or 5. AD is congruent to BC, so
AD  5. If the diagonals are congruent, then
6. The diagonals of a rhombus are perpendicular, so parallelogram MNPQ is either a rectangle or a
mAEB  90. square. If the diagonals are perpendicular, then
7. Consecutive angles of a rhombus are MNPQ is a square or a rhombus.
supplementary. Find mBCD. Use the distance formula to compare the lengths
mBCD  mABC ¬180 of the diagonals.
mBCD  83.2 ¬180 MP  
(4 
2)2 
(0  2)2
mBCD ¬96.8  
36  4  40

8. y NQ  
(3 
1)2  2
[3  (1)]
M     32
16  16 
Use slope to determine whether the diagonals are
perpendicular.
N O Q x 02
P
slope of M
4 
2
 1

3
P 1  3
Q
slope of N
1  (
3)
 1
If the four sides are congruent, then
parallelogram MNPQ is either a rhombus or a The diagonals are not congruent or perpendicular.
square. If consecutive sides are perpendicular, MNPQ is not a rhombus, a rectangle, or a square.
then MNPQ is a rectangle or a square.
Use the distance formula to compare the lengths
of the sides.
MN ¬ 
(3 
0)2 
(0  3
)2
¬ 
9  9  3 2

NP ¬  
[0  (3)]2
 (3
 0)2
¬ 
9  9  3 2

PQ ¬ [3
]  0
2  [03)]
 (2
¬ 
9  9  3 2


263 Chapter 8
10. Given: KGH, HJK, GHJ, and JKG are 16. The diagonals of a rhombus are perpendicular, so
isosceles. mYVZ  90. The measure of the interior angles
Prove: GHJK is a rhombus. of a triangle is 180. Find mYZV.
G H mYVZ  mYZV  mWYZ ¬180
90  mYZV  53 ¬180
mYZV  143 ¬180
mYZV ¬37
K J
17. The diagonals of a rhombus bisect the angles, so
Proof: XYW  WYZ and mXYW  53.
Statements Reasons 18. The diagonals of a rhombus bisect each other, so
1. KGH, HJK, 1. Given V
XZ V. Find a.
GHJ, and JKG V
X ¬Z
V

are isosceles. XV ¬ZV
G
2. KG H
, HJK
J
, 2. Def. of isosceles  5a  1
2a  2 ¬ 
H
G H J, K
G
KJ
 4
8a  8 ¬5a  1
G
3. KH J
, GH
KJ
 3. Transitive Property
G
4. KG H
, HJK
J
 3a ¬9
4. Substitution
5. GHJK is a rhombus. 5. Def. of rhombus a ¬3
So, XV  2(3)  2 or 4 and XZ is twice XV or 8.
11. If the measure of each angle is 90 or if the
19. From Exercise 18, XV  4. VW  3.
diagonals are congruent, then the floor is a square.
The diagonals of a rhombus are perpendicular, so
mWVX  90 and WVX is a right triangle.
W
X is the hypotenuse of WVX. Use the
Pages 434–437 Practice and Apply Pythagorean Theorem.
12. Consecutive angles in a rhombus are (VW)2  (XV)2 ¬(XW)2
supplementary, so DAB and ADC are 32  42 ¬(XW)2
25 ¬(XW)2
supplementary. Find mDAB.
5 ¬XW
mDAB  mADC ¬180 20. y
mDAB  1
2 mDAB ¬180
 H
12
E
3 mDAB ¬180
2
8
mDAB ¬120
Opposite angles of a rhombus are congruent, so 4
BCD  DAB. The diagonals of a rhombus
F G
bisect the angles, so mACD  1

2 mBCD or 60. 4 O 4 8 12 x
13. Consecutive angles in a rhombus are
supplementary, so DAB and ADC are If the diagonals are congruent, then
supplementary. Find mDAB. parallelogram EFGH is either a rectangle or a
square. If the diagonals are perpendicular, then
mDAB  mADC ¬180 EFGH is a square or a rhombus.
mDAB  1
2 mDAB ¬180
 Use the Distance Formula to compare the lengths
of the diagonals.
3 mDAB ¬180 EG ¬  (1  7
)2  (10  2)2
2
mDAB ¬120 ¬  36  64   100
14. By definition, a rhombus has four congruent ¬10
A
sides, so DC B
 and DA  6. FH ¬  (4  12)2  (0  12)2
15. Consecutive angles in a rhombus are ¬  256  144   400
supplementary, so DAB and ADC are ¬20
supplementary. Find mADC. Use slope to determine whether the diagonals are
mDAB  mADC ¬180 perpendicular.
10  2
2(mADC)  mADC ¬180 G
slope of E  ¬17


3(mADC) ¬180 ¬4



3
mADC ¬60 0 12
H
slope of F  ¬
4  12
The diagonals of a rhombus bisect the angles, so
 3

mADB  1

2 (mADC) or 30.
4

Chapter 8 264
The diagonals are not congruent. Since the slopes 23. y G
G
of E and F H
 are opposite reciprocals of each
other, the diagonals are perpendicular. EFGH is a F
rhombus.
21. H y G H
O x
E

E F
If the diagonals are congruent, then
parallelogram EFGH is either a rectangle or a
O x square. If the diagonals are perpendicular, then
EFGH is a square or a rhombus.
If the diagonals are congruent, then Use the Distance Formula to compare the lengths
parallelogram EFGH is either a rectangle or a of the diagonals.
square. If the diagonals are perpendicular, then EG ¬  (2  1)2 
(1  5)2
EFGH is a square or a rhombus. ¬ 9  36   45
Use the Distance Formula to compare the lengths FH ¬  (4  3)2 
(3  1
)2
of the diagonals. ¬ 49  4   53
EG ¬  (7  1)2 (3  7
)2 Use slope to determine whether the diagonals are
¬ 64  16   80 perpendicular.
FH ¬  [2  (4)]
2  (3 7)2 G
slope of E 1 
 ¬  5
2  1
¬ 4  16   20
¬2
Use slope to determine whether the diagonals are 31
perpendicular. H
slope of F  ¬
4 
3
37
G
slope of E  ¬
7 1 ¬2

7
¬1

2
The diagonals are not congruent or perpendicular.
37 EFGH is not a rhombus, a rectangle, or a square.
H
slope of F ¬ 2 
(4)
¬2 24. Sample answer:
The diagonals are not congruent. Since the slopes
G
of E and F H
 are opposite reciprocals of each
other, the diagonals are perpendicular. EFGH is a 3 cm
rhombus.
22. y H 3 cm
G
25. Sample answer:

E F
5 cm

O x 26. Every square is a parallelogram, but not every


If the four sides are congruent, then parallelogram is a square. The statement is
parallelogram EFGH is either a rhombus or a sometimes true.
square. If consecutive sides are perpendicular, 27. Every square is a rhombus. The statement is
then EFGH is a rectangle or a square. always true.
It is obvious from the figure that each side has 28. Every rectangle is a parallelogram. The statement
measure 5, so the sides are congruent. It is also is always true.
obvious that HE and GF are vertical segments 29. If a rhombus is a square, then the rhombus is
and H G
 and EF are horizontal, so consecutive also a rectangle. Otherwise the rhombus is not
sides are perpendicular. Thus, EFGH is a square, a rectangle. The statement is sometimes true.
a rectangle, and a rhombus. 30. Every square is a rhombus, but not every rhombus
is a square. The statement is sometimes true.
31. Every square is a rectangle. The statement is
always true.

265 Chapter 8
32. The width of the square base is 153

4 in. Since the 37. The width and length of the rectangular court are
width of the smaller boxes is one half the width of 6400 mm and 9750 mm, respectively. The
the square base, the dimensions of the smaller measure of the diagonal can be found using the
boxes are 77
 7
8 in. by 7 8 in. Pythagorean Theorem with width and length as
33. The diagonals of a rhombus bisect each other and the legs of a right triangle.
are perpendicular, so along with the sides, they diagonal measure  6400 2  97502
form four congruent right triangles with legs 12
 ¬11,662.9
2 No; the diagram is not correct. The correct
16

or 6 cm and 2 or 8 cm. Find the length of one of
measure is about 11,662.9 mm.
the four congruent sides–the hypotenuse of one of
the right triangles–by using the Pythagorean 38. The side length of the square service boxes is
Theorem. 1600 mm. The length of the diagonal can be found
side length ¬  62  82 using the Pythagorean Theorem with the side
¬  36  64  10 length of the square service boxes as the legs of a
right triangle.
The side length is 10 cm, so the perimeter of the
diagonal length   16002  160
02
rhombus is 4(10) or 40 cm.
¬2263
34. ABCD is a rhombus; EFGH and JKLM are
The length of the diagonal of the square service
congruent squares.
boxes is about 2263 mm or 2.263 m.
35. Given: ABCD is a parallelogram.
39. The flag of Denmark contains four red rectangles.
C
A B D

The flag of St. Vincent and the Grenadines
Prove: ABCD is a rhombus.
contains a blue rectangle, a green rectangle, a
A B yellow rectangle, a blue and yellow rectangle, a
E yellow and green rectangle, and three green
rhombi. The flag of Trinidad and Tobago contains
two white parallelograms and one black
D C parallelogram.
Proof: We are given that ABCD is a 40. Given: WZY  WXY
parallelogram. The diagonals of a parallelogram WZY and XYZ are isosceles.
E
bisect each other, so A E C
. B
EB E because Prove: WXYZ is a rhombus.
congruence of segments is reflexive. We are also W X
given that ACB D. Thus, AEB and BEC are P
right angles by the definition of perpendicular
lines. Then AEB  BEC because all right Z Y
angles are congruent. Therefore, AEB  CEB
Proof:
by SAS. ABB C by CPCTC. Opposite sides of
parallelograms are congruent, so A B
C D
 and Statements Reasons
C
B A D
. Then since congruence of segments is 1. WZY  WXY 1. Given
B
transitive, A C DB CA D. All four sides of WZY and XYZ are
ABCD are congruent, so ABCD is a rhombus by isosceles.
definition. Z
2. WW
X
, Z
Y
X
Y
 2. CPCTC
36. Given: ABCD is a rhombus.
Prove: Each diagonal bisects a pair of Z
3. WZ
Y
, W
X
X
Y
 3. Def. of isosceles
opposite angles. triangle
A B Z
4. WW
X
Z
Y
X
Y
 4. Substitution Property
6 3
5 4 5. WXYZ is rhombus. 5. Def. of rhombus

1 2 8 41. Given: TPX  QPX  QRX  TRX


7
Prove: TPQR is a rhombus.
D C
P Q
Proof: We are given that ABCD is a rhombus. By
definition of rhombus, ABCD is a parallelogram. X
Opposite angles of a parallelogram are congruent,
T R
so ABC  ADC and BAD  BCD.
B
AB CC DD A because all sides of a Proof:
rhombus are congruent. ABC  ADC by SAS. Statements Reasons
5  6 and 7  8 by CPCTC. BAD  1. TPX  QPX  1. Given
BCD by SAS. 1  2 and 3  4 by CPCTC. QRX  TRX
By definition of angle bisector, each diagonal
bisects a pair of opposite angles. P
2. T  P
QQ
R
T
R
 2. CPCTC

3. TPQR is a rhombus. 3. Def. of rhombus

Chapter 8 266
42. Given: LGK  MJK • Sample answer: Since the angles of a rhombus
GHJK is a parallelogram. are not all congruent, riding over the same road
Prove: GHJK is a rhombus. would not be smooth.
K J 46. B; the side length of the square is 36
  6 units.
M So, the perimeter of the square is 4(6) or
L 24 units. Since rectangle ABCD is contained
within the square, which itself is a rectangle,
G H ABCD cannot have a greater perimeter than the
square. Therefore, the perimeter of rectangle
Proof:
ABCD is less than 24 units.
Statements Reasons
47. C; test all four values.
1. LGK  MJK 1. Given
GHJK is a parallelogram. x
1 
 x
x2
G
2. KK
J
 2. CPCTC
0
1

2
J
3. KG
H
, K
G
J
H
 3. Opp. sides of
1
2
 are .
3
4
G
4. KJ
H
G
H
J
K
 4. Substitution
4
5  21
Property 2 2
5. GHJK is a rhombus. 5. Def. of rhombus 3
has the greatest value.
43. Given: QRST and QRTV are rhombi.
Prove: QRT is equilateral.
Q R Pae 437 Maintain Your Skills
48. The diagonals of a rectangle bisect each other and
are congruent, so PJL J
.
V S J
P ¬LJ
T
Proof: PJ ¬LJ
Statements Reasons 3x  1 ¬2x  1
x ¬2
1. QRST and QRTV are 1. Given
So, x  2.
rhombi.
49. The interior angles of a rectangle are 90°. Find
V
2. Q VT T
R
QR
, 2. Def. of rhombus
mMLK.
T
Q TSRS
Q
R mMLK  mPLM ¬mPLK
T
3. QT
R
Q
R
 3. Substitution mMLK  90 ¬110
Property mMLK ¬20
4. QRT is equilateral. 4. Def. of equilateral Diagonals of a rectangle are congruent and bisect
triangle each other. So JL
J M
. Then LKMJ is a rhombus
because opposite sides of a parallelogram are
44. Hexagons 1, 2, 3, and 4 have 3, 12, 27, and 48
congruent. Since each diagonal of a rhombus
rhombi, respectively. Note that the numbers of
bisects a pair of congruent opposite angles,
rhombi are 3  3(1)  3(12), 12  3(4)  3(22),
KML  MLK. The sum of the measures of the
27  3(9)  3(32), and 48  3(16)  3(42). So, the
interior angles of a triangle is 180. Find mLKM.
number of rhombi are given by 3x2, where x is the
mMLK  mLKM  mKML ¬180
hexagon number.
20  mLKM  20 ¬180
Number of mLKM ¬140
Hexagon
rhombi So, mLKM  140.
1 3 50. MJN is supplementary to PJN. Find mPJN.
2 12 mMJN  mPJN ¬180
3 27 35  mPJN ¬180
4 48 mPJN ¬145
5 75 Since the diagonals of a rectangle are congruent
6 108 and bisect each other, PJN is isosceles with
x 3x2 J
sides P  and JN
 congruent. Since PJN is
isosceles, MPN  PNL. The sum of the
45. Sample answer: You can ride a bicycle with
measures of the interior angles of a triangle is
square wheels over a curved road. Answers should
180. Find mMPN.
include the following.
mMPN  mPNL  mPJN ¬180
• Rhombi and squares both have all four sides
mMPN  mMPN  145 ¬180
congruent, but the diagonals of a square are
2(mMPN) ¬35
congruent. A square has four right angles and
mMPN ¬17.5
rhombi have each pair of opposite angles
So, mMPN  17.5.
congruent, but not all angles are necessarily
congruent.

267 Chapter 8
51. Since the diagonals of a rectangle are congruent 54. No;
and bisect each other, LJ
J N
. Since the sides of y
a rhombus are congruent, L JM K. Therefore, H
K
M J N
.
Find x.
K
M  ¬J N
 J
MK ¬JN G
x
6x ¬14  x
7x ¬14 F
x ¬2
Once again, the sides of a rhombus are congruent, If both pairs of opposite sides of a quadrilateral
K
so M K L. Find y. are congruent, then the quadrilateral is a
K
M  ¬KL parallelogram. Use the Distance Formula to
MK ¬KL determine whether the opposite sides are
6x ¬3x  2y congruent.
3x ¬y HJ ¬  (3  2)2 (4  1)2
2
Substituting the value of x from above, ¬ 34
3(2) ¬y FG ¬  
[1  (4)] 2
 (1  1)2
2
3 ¬y ¬ 29
So, x  2 and y  3. Since HJ  FG, H J

F G
. Therefore, FGHJ is not
52. Since mLMP  mPMN, the diagonal P M
 a parallelogram.
bisects LMN. If the diagonals of a rectangle 55. No;
bisect its interior angles, the rectangle must be a y
square. The diagonals of a square are 8
M
perpendicular, so mPJL  90.
4
53. Yes; L
y
N
Q 8 4 4 8x

P 4
K
8
O R x
If the opposite sides of a quadrilateral are
S parallel, then it is a parallelogram. The slope of
N
K  is undefined, since it is vertical. L
M
 is clearly
not vertical. The opposite sides, KN
 and LM, do
If both pairs of opposite sides of a quadrilateral not have the same slope, so they are not parallel.
are congruent, then the quadrilateral is a Therefore, KLMN is not a parallelogram.
parallelogram. Use the Distance Formula to 56. Yes;
determine whether the opposite sides are y
congruent. 8
C
PQ ¬  (0  6)2  (2
 4)2
4
¬ 40 D
RS ¬  [4  (
2)]2  [0 (2)]2
8 A 4 8x
¬ 40
QR ¬  (6  4)2  (4
 0)2 B
¬ 20 8
PS ¬  [0  (
2)]2  [2 (2)]2
If the opposite sides of a quadrilateral are
¬ 20 parallel, then it is a parallelogram.
Since the measures of both pairs of opposite sides 1  (5)
are equal, PQRS is a parallelogram. B
slope of A ¬
4 (2)
¬2
7 
3
D
slope of C ¬
13
¬2
1  7
C
slope of A ¬ 
4  1
¬8

5
5  3
D
slope of B ¬ 
2  3
¬8

5

Chapter 8 268
B
Since opposite sides have the same slope, A CD
 Page 438 Geometry Activity: Kites
C
and A B D. Therefore, ABDC is a parallelogram 1. The diagonals intersect at a right angle.
by definition. 2. QRS  QTS
57. From the Triangle Proportionality Theorem, 3. R
See students’ work; NT N, but Q
N
N S
.
PS QP
  . 4. 3 pairs: QRN  QTN, RNS  TNS,
ST RT
Substitute the known measures. QRS  QTS
PS 24 5. J
9 ¬
16
PS(16) ¬24(9)
K M
16(PS) ¬216 P
PS ¬13.5
58. From the Triangle Proportionality Theorem, L
PS QS
  .
PT
The diagonals intersect in a right angle; JKL 
QR
Substitute the known measures. JML; K PP M
, JP

P L; 3 pairs:
y2 16 JPK  JPM, KPL  MPL, JKL  JML.
 ¬
16 
y3 12 6. One pair of opposite angles is congruent. The
4(y  2) ¬16(y  3) diagonals are perpendicular. The longer diagonal
4y  8 ¬16y  48 bisects the shorter diagonal. The short sides are
56 ¬12y congruent and the long sides are congruent.
42
3 ¬y
59. From the Triangle Proportionality Theorem,
TS RT
PS  QP .
   8-6 Trapezoids
Substitute the known measures.
TS 15
8 ¬
 
TS  21 Page 441 Geometry Activity: Median of a
21(TS) ¬15(TS  8) Trapezoid
6(TS) ¬120 1. See students’ work.
TS ¬20
2. The median is the average of the lengths of the
60. G
If A A C
, ACG is isosceles. Then AGC  bases.
ACG. So, MN  1
2 (WX  ZY).

61. J
If A AH, AJH is isosceles. Then AJH 
AHJ.
62. If AFD  ADF, ADF is isosceles. Then A F
 Page 442 Check for Understanding
D
A . 1. Exactly one pair of opposite sides is parallel.
63. If AKB  ABK, ABK is isosceles. Then A K
 2. Properties Trapezoid Rectangle Square Rhombus
B
A .
diagonals are only
64. Solve for x. yes yes no
congruent isosceles
1
(8x  6x  7) ¬5
2 diagonals are
no no yes yes
8x  6x  7 ¬10 perpendicular
2x ¬17 diagonals
x ¬8.5 bisect each no yes yes yes
65. Solve for x. other
1(7x  3x  1) ¬12.5 diagonals
2 no no yes yes
7x  3x  1 ¬25 bisect angles
10x ¬24
3. Sample answer: The median of a trapezoid is
x ¬2.4
parallel to both bases.
66. Solve for x.
1(4x  6  2x  13) ¬15.5
2
4x  6  2x  13 ¬31
6x ¬12 trapezoid
x ¬2
67. Solve for x.
1(7x  2  3x  3) ¬25.5 isosceles trapezoid
2
7x  2  3x  3 ¬51
10x ¬50
x ¬5

269 Chapter 8
4. y 8. The perspective makes it appear that the
R S buildings are formed by trapezoids and
parallelograms.

Pages 442–445 Practice and Apply


Q T
9a. y
x
A D
A quadrilateral is a trapezoid if exactly one pair
of opposite sides is parallel. Use the Slope
x
Formula.
2  2 O
T
slope of Q  ¬
3  6 B C
¬0
6  6
S
slope of R  ¬
1  4
¬0 A quadrilateral is a trapezoid if exactly one pair
2  6 of opposite sides is parallel. Use the Slope
R
slope of Q  ¬
3  (1) Formula.
¬2 3 3
6 2 D
slope of A ¬3  2
slope of ST ¬
46
¬0
¬2
1 
(1)
T
Exactly one pair of opposite sides is parallel, Q C
slope of B ¬4  5
and RS
. So, QRST is a trapezoid. ¬0
3  (1)
5. See graph in Exercise 4. B
slope of A ¬
3  (4)
Use the Distance Formula to show that the legs ¬4
are congruent. 3
1 
D
slope of C ¬
52
QR ¬  [3  (1)]2  (2 6)2
¬  4  16   20 ¬4
3


ST ¬  (4  6
)2  (6  2)2 D
Exactly one pair of opposite sides is parallel, A
C
and B. So, ABCD is a trapezoid.
¬  4  16   20
9b. y
Since the legs are congruent, QRST is an isosceles
trapezoid.
A D
6. Given: CDFG is an isosceles C D
trapezoid with bases
D
C  and FG. x
Prove: DGF  CFG O
Proof: G F
B C
CDFG is an isosceles
trapezoid with CG DF
Use the Distance Formula to determine whether
bases CD and GF. Def. isos. trap.
the legs are congruent.
Given
AB ¬  [3  (4)]2
 [3 (
1)]2
CF DG GF GF ¬  1  16
Diag. of isos. Reflex. Prop. ¬  17
trap. are . CD ¬  (5  2
)2  (
1  3)2
CGF DFG ¬  9  16
SSS ¬5
Since the legs are not congruent, ABCD is not an
DGF CFG isosceles trapezoid.
CPCTC
7. The median of a trapezoid is parallel to the bases,
and its measure is one-half the sum of the
measures of the bases. Find x.
YZ ¬1
2 (EF  HG)


13 ¬1
2 [(3x  8)  (4x  10)]

26 ¬7x  2
28 ¬7x
4 ¬x

Chapter 8 270
0  ( 10)
10a.
J E
slope of F  ¬ 
6  (4)
y ¬1
H 3  (10)
E
slope of D  ¬5  (4)
¬7
K 10
x F
slope of C ¬
1 
6
O
¬1

7
C
Exactly one pair of opposite sides is parallel, D
E
and F . So, CDEF is a trapezoid.
G
11b. 8 y
A quadrilateral is a trapezoid if exactly one pair
of opposite sides is parallel. Use the Slope Formula. 4
4  4
H
slope of G  ¬ 
5  5 C F
¬4

5 8 4 O 4 8x
5  1 D 4
J
slope of K  ¬0  (5)
¬4
5
 8
1  (4)
K
slope of G ¬ 
5  (5) E
5
¬0 or undefined Use the Distance Formula to show that the legs
4  5 are congruent.
J
slope of H  ¬50
1 DE ¬  [5  (4)]
2  [ 2
3  (10)]
¬5
J
Exactly one pair of opposite sides is parallel, K ¬  1  49
H
and G . So, GHJK is a trapezoid. ¬  50
10b. CF ¬  (1  6)2 (1  0)2
J
y ¬  49  1
H ¬  50
Since the legs are congruent, CDEF is an isosceles
K trapezoid.
x
O 12a. y
8
R
4
S
G Q
12 8 4 O x
Use the Distance Formula to determine whether
the legs are congruent. 4
T
GK ¬  [5  (5)]2
 ( 4  1)2
8
¬  0  25
¬5 A quadrilateral is a trapezoid if exactly one pair
JH ¬  (0  5)
2  (5
 4)2 of opposite sides is parallel. Use the Slope
Formula.
¬  25  1 14
R
slope of Q ¬12  (9)
¬  26
Since the legs are not congruent, GHJK is not an ¬1
4  3
isosceles trapezoid. S
slope of T ¬
11 (4)
11a. 8 y ¬1
1  (4)
T
slope of Q ¬
12  (11)
4
¬5
C F 43
8 4 O 4 8x S
slope of R ¬
9 
(4)
D 4 ¬1
5
R
Exactly one pair of opposite sides is parallel, Q
8
S
and T. So, QRST is a trapezoid.
E
A quadrilateral is a trapezoid if exactly one pair
of opposite sides is parallel. Use the Slope
Formula.
3  1
C
slope of D ¬5  (1)
¬1

271 Chapter 8
12b. y So, AB  16.
8 Q and S are supplementary to T and R,
R
4 respectively. So, mQ  180  120 or 60 and
S mS  180  45 or 135.
Q
12 8 4 O x 17. The median of a trapezoid is parallel to the bases,
4
and its measure is one-half the sum of the
T measures of the bases. First find AB.
8 AB ¬1
2 (RS  QT)

Use the Distance Formula to show that the legs ¬1
2 (54  86)

are congruent. ¬70
QT ¬  
[1  (4)]2 [1 2
2  (11)] So, AB  70.
¬  25  1 Find GH.
¬  26 GH ¬12 (RS  AB)

RS ¬  (4  3
)2  [
9  (4)]2
¬1
2 (54  70)

¬  1  25
¬  26 ¬62
Since the legs are congruent, QRST is an isosceles So, GH  62.
trapezoid. 18. The median of a trapezoid is parallel to the bases,
13. The median of a trapezoid is parallel to the bases, and its measure is one-half the sum of the
and its measure is one-half the sum of the measures of the bases. Find AB.
measures of the bases. Find DE. From Exercise 17, AB  70.
Find JK.
XY ¬1
2 (DE  HG)

JK ¬1
2 (QT  AB)

20 ¬1
2 (DE  32)

¬1
2 (86  70)

40 ¬DE  32
¬78
8 ¬DE
So, JK  78.
14. The median of a trapezoid is parallel to the bases,
19. The median of a trapezoid is parallel to the bases,
and its measure is one-half the sum of the
and its measure is one-half the sum of the
measures of the bases. Find VT.
measures of the bases. Find RP.
AB ¬1
2 (VT  RS)

RP ¬1
2 (JK  LM)

15 ¬1
2 (VT  26)

30 ¬VT  26
5  x ¬1
 1 
2 2(x  3)  2 x  1
10  2x ¬2x  6  1
2x  1

4 ¬VT 1
5 ¬2x
15. The median of a trapezoid is parallel to the bases,
and its measure is one-half the sum of the 10 ¬x
measures of the bases. Find the length of the x  10, so RP  5  10 or 15.
median and let it be x. 20. Since the two octagons are regular polygons with
x ¬1
2 (WZ  XY)
 the same center, the quadrilaterals are trapezoids
with one pair of opposite sides parallel.
¬1
2 (8  20)

21. Sample answer: triangles, quadrilaterals,
¬14 trapezoids, hexagons
The length of the median is 14. 22. A trapezoid must have exactly one pair of
Both base pairs of an isosceles trapezoid are opposite sides parallel. A parallelogram must
congruent, so mX  70 and mW  mZ. The have both pairs of opposite sides parallel. A
sum of the measures of the interior angles of a square must have all four sides congruent and
quadrilateral is 360. consecutive sides perpendicular. A rhombus must
Find mW and mZ. have all four sides congruent. A quadrilateral has
mW  mZ  mX  mY ¬360 four sides.
mW  mW  70  70 ¬360 Use the Slope Formula to determine whether the
2(mW) ¬220 opposite sides are parallel.
mW ¬110 C
slope of B ¬2 4
mZ ¬110 14
16. The median of a trapezoid is parallel to the bases, ¬2

3
and its measure is one-half the sum of the 1  (1)
E
slope of D ¬ 
52
measures of the bases. Find AB.
AB ¬1 ¬2

2 (TS  QR)
 3
4 
1
¬1 D
slope of C ¬
2 (12  20)
 45
¬16 ¬3

Chapter 8 272
2  (1) 1  (2)
E
slope of B ¬ 
12 P
slope of M ¬
3 (2)
¬3 ¬3
Use the distance formula to compare the lengths 3  (1)
O
slope of N ¬ 
13
of the sides.
¬2
BC ¬  (1  4
)2  (2 4)2
All four sides have different slopes. Therefore,
¬  94
opposite sides are not parallel and the figure is a
¬  13 quadrilateral.
DE ¬  (5  2)2  [1
 (1)]2 25. A trapezoid must have exactly one pair of
¬  94 opposite sides parallel. A parallelogram must
¬  13 have both pairs of opposite sides parallel. A
CD ¬  (4  5)2  (4
 1)2 square must have all four sides congruent and
¬  19 consecutive sides perpendicular. A rhombus must
have all four sides congruent. A quadrilateral has
¬  10
four sides.
BE ¬  (1  2
)2  [2 2
 (1)]
Use the Slope Formula to determine whether the
¬  19 opposite sides are parallel.
¬  10 R  ¬03
slope of Q 3 0
Opposite sides are parallel. Since the slopes of ¬1
consecutive sides are not negative reciprocals, 0  ( 3)
T
slope of S  ¬ 30

consecutive sides are not perpendicular; there are
no right angles. Opposite sides are congruent, but ¬1
0  (3)
consecutive sides are not congruent. BCDE is a slope of QT ¬3  0
parallelogram. ¬1
3  0
23. A trapezoid must have exactly one pair of slope of RS ¬
03
opposite sides parallel. A parallelogram must ¬1
have both pairs of opposite sides parallel. A Use the distance formula to compare the lengths
square must have all four sides congruent and of the sides.
consecutive sides perpendicular. A rhombus must
QR ¬  (3  0)2 (0  3)2
have all four sides congruent.
A quadrilateral has four sides. ¬  9  9   18
Use the Slope Formula to determine whether the ST ¬  (3  0
)2  [0 (3)]2
opposite sides are parallel. ¬  9  9   18
22 QT ¬  (3  0) 
2 [0  (
3)]2
H
slope of G ¬
2 
4
¬0 ¬  9  9   18
K
slope of J 1 
 ¬ (1) RS ¬  (0  3
)2  (3 0)2
6  (4)
¬  9  9   18
¬0
All four sides have the same length and are
2  (1)
K
slope of G ¬ 
2  (4) congruent. Opposite sides are parallel. Because
the slopes of consecutive sides are opposite
¬3

2 reciprocals, consecutive sides are perpendicular.
2  (1)
J
slope of H ¬ 
46
QRST is a square.
26. A quadrilateral is a trapezoid if exactly one pair
¬32

of opposite sides is parallel. Use the Slope
Exactly one pair of opposite sides is parallel, so Formula.
GHJK is a trapezoid. 4  1
R
slope of Q  ¬
04
24. A trapezoid must have exactly one pair of
opposite sides parallel. A parallelogram must ¬3

4
have both pairs of opposite sides parallel. A 3  (
3)
S
slope of P  ¬4  4
square must have all four sides congruent and
consecutive sides perpendicular. A rhombus must ¬3

4
have all four sides congruent. A quadrilateral has 34
Q
slope of P ¬
4 
0
four sides.
Use the Slope Formula to determine whether the ¬1

4
opposite sides are parallel. 1  ( 3)
S
 ¬ 4
4  4  0 or undefined
13 slope of R 
N
slope of M  ¬3 1 R
Exactly one pair of opposite sides is parallel, Q
¬1

2 and PS
. So, PQRS is a trapezoid.
1 (2)
P
slope of O ¬
3  (2)
Use the Distance Formula to determine whether
the legs are congruent.
¬1

5

273 Chapter 8
PQ ¬  (4  0)2 (3  4
)2 31. The median of a trapezoid is parallel to the bases,
and its measure is one-half the sum of the measures
¬ 16  1
of the bases. So, the length of 
W
V is given by
¬ 17 WV  21(DG  EF). Since DG and EF are vertical,
RS ¬  (4  4
)2  [1 2
 (3)] their lengths are given by y1  y2 . Find WV.
¬ 0  16 WV ¬1
2 (DG  EF)

¬4
¬1
2 [2  (2)  5  (3)]

Since PQ  RS, PQRS is not an isosceles
trapezoid. ¬6
27. Use the Midpoint Formula to find the coordinates So, WV  6.
Q
of the midpoints of P  and RS
. 32. Given: HJ
GK
, HGK  JKG

 
4  3 
0  4 Prove: GHJK is an isosceles trapezoid.
Q
P:  
2 , 2 ¬(2, 3.5)
H J
: 
S
R 2 , 2 ¬(4, 1)
4 
4 1  (3)
G K
Q
The coordinates of the midpoints of P  and R
S
 Proof:
are A(2, 3.5) and B(4, 1), respectively. HJ GK HGK JKG
Given Given
28. The median of a trapezoid is parallel to the bases,
and its measure is one-half the sum of the
measures of the bases. So, the length of AB
 is GHJK is a trapezoid. HG JK
given by AB  12 (QR  PS). To find the lengths of
 Def. of trapezoid CPCTC
the bases, recognize that the bases are the
hypotenuses of two special right triangles, 3-4-5
and 6-8-10. Find AB. GHJK is an isosceles trapezoid.
AB ¬1
2 (QR  PS)
 Def. of isosceles trapezoid

33. Given: TZX  YXZ


¬1
2 (5  10)

Prove: XYZW is a trapezoid.
¬7.5
So, AB  7.5. T W 1 Z
29. A quadrilateral is a trapezoid if exactly one pair
2
of opposite sides is parallel. Use the Slope X Y
Formula. Proof:
TZX YXZ
25
E
slope of D ¬
2 
5 Given
¬3

7
3 (2) 1 2 WZ || XY
G
slope of F  ¬
5  (2) CPCTC If alt. int.
¬1 are , then the
7 lines are ||.
The slopes of DG and EF
 are undefined. They are
both vertical; therefore, they are parallel.
XYZW is a trapezoid.
G
Exactly one pair of opposite sides is parallel, D
Def. of trapezoid
and EF
. So, DEFG is a trapezoid.
Use the Distance Formula to determine whether 34. Given: ZYXP is an isosceles X
trapezoid. Y W
the legs are congruent.
DE ¬  (2  5)2 (2  5
)2 Prove: PWX is isosceles. P
¬  49  9
Proof: Z
¬  58
ZYXP is an isosceles trapezoid.
GF ¬  
[5  (2)] 2
 [3  (2)]2
Given
¬  49  1
¬  50 Y Z WY WZ XY PZ
Since DE  GF, DEFG is not an isosceles Base of an If 2 of a Def. of
isos. trap. are . are , the sides isos. trap.
trapezoid. opp. are .
30. Use the Midpoint Formula to find the coordinates WX XY WY,
of the midpoints of DE and GF
. WP PZ WZ WY WZ XY PZ

E
D 
2 
:  
2 , 2
2 
5  5

¬(1.5, 3.5)
Segment
Addition
Def. of Def. of

:   ¬(1.5, 2.5)
2  2 
5  (3) WX XY WP PZ WX WP
F
G 
2 , 2 Substitution Subtraction
The coordinates of the midpoints of DE and GF
are W(1.5, 3.5) and V(1.5, 2.5), respectively. PWX is isosceles. WX WP
Def. of isos. Def. of

Chapter 8 274
D
35. Given: E and C are midpoints of A and D
B
. 39. 6
D
AD B R S
Prove: ABCE is an isosceles trapezoid.
D
M P
1 3
E C
2 4
X Y
A B
Proof:
V T
E and C are midpoints 3
of AD and DB. AD DB
Given Given

EC || AB A B
A segment joining the Isos. Th.
midpoints of two sides
of a triangle is parallel
to the third side. A
Extend R V and ST to intersect at A. Since RSTV
2 is suppl. to A, 2 4 is an isoceles trapezoid, R  S. So RSA is
4 is suppl. to B. suppl. to isosceles. Then RA . RA  RX  XA and
SA
Consec. are suppl. are . SA  SY  YA. Since RA  SA and RX  SY,
XA  YA. By the Converse of the Triangle
Proportionality Theorem, X YRS. Since R
SVT,
ABCE is an isos.
trapezoid. Y
X V T
. Let M and P be the midpoints of R X and
1
Def. of isos. trapezoid Y
S , respectively. Draw M P. RX  2(XV), so 2RX
1
 XV. RM  MX  2RX. So RM  MX  XV.
36. Given: ABCD is an isosceles trapezoid.
C
B A
D
 Similarly, SP  PY  YT.
B
AC D
 By an argument similar to the one above,  MP R
S,
Prove: A  D P
M XY, M
PV T. So XY
 is the median of
ABC  DCB isosceles trapezoid MPTV. And M P is the median
of isosceles trapezoid RSYX.
A 1 1
F B So, MP  2(RS  XY) and XY  2(MP  VT)
1 1
MP  2(6  XY) XY  2(MP  3)
E C 1
MP  3  2XY
D
Substitute this expression for MP into the second
Proof: Draw auxiliary segments so that B FA D
 equation.
E
and C A D
. Since BC
A D
 and parallel lines 1 1
are everywhere equidistant, B FC E. XY  2[(3  2XY)  3]
Perpendicular lines form right angles, so BFA 1 1
XY  2(6  2XY)
and CED are right angles. BFA and CED are 1
right triangles by definition. Therefore, XY  3  4XY
3
XY  3
BFA  CED by HL. A  D by CPCTC. 4
Since CBF and BCE are right angles and all XY  4
right angles are congruent, CBF  BCE.
40. It is not possible. Since pairs of base angles of an
ABF  DCE by CPCTC. So, ABC  DCB
isosceles trapezoid are congruent, if two angles
by angle addition.
are right, all four angles will be right. Then the
37. Sample answer: quadrilateral would be a rectangle, not a
D C trapezoid.
41. Sample answer: Trapezoids are used in
A B monuments as well as other buildings. Answers
38. Sample answer: should include the following.
• Trapezoids have exactly one pair of opposite
2 cm sides parallel.
• Trapezoids can be used as window panes.
42. Quadrilateral WXYZ is a trapezoid because
X
exactly one pair of opposite sides is parallel, W 
and Y Z
.

275 Chapter 8
43. B; points in the shaded region have the following 54. Solve the proportion for y.
characteristics: x 0, y
0, and y 5
3 x  5. The
 5 ¬
20

y4 28
only choice that satisfies all three requirements is
B, (1, 3). 5(28) ¬20(y  4)
140 ¬20y  80
60 ¬20y
Page 445 Maintain Your Skills 3 ¬y
44. Opposite angles of a rhombus are congruent, so 55. Solve the proportion for y.
2y 52
QLM ¬QPM. Find x.  ¬
9 36
QLM ¬QPM
mQLM ¬mQPM 36(2y) ¬52(9)
2x2  10 ¬8x 72y ¬468
2x2  8x  10 ¬0 13
x2  4x  5 ¬0 y ¬2
(x  5)(x  1) ¬0 56. Find the slope of a segment given the endpoints
x  5  0 or x  1  0 (0, a) and (a, 2a).
x5 x  1 y y
slope of segment ¬
2
1

x must be greater than zero (otherwise mQPM x2  x1


is negative), so x  5. 2a  a
¬ 
a  0
The diagonals of a rhombus bisect the angles, so ¬1
mLPQ  1 2 mQPM, and mQPM  8(5) or 40.

57. Find the slope of a segment given the endpoints
Therefore, mLPQ  20.
(a, b) and (a, b).
45. By definition, a rhombus has four congruent y2  y1
slope of segment ¬ 
L
sides, so QM P and QL  10. x2  x1
b b
46. Consecutive angles in a rhombus are ¬
a  ( a)
supplementary, so QLM and LQP are ¬0
supplementary. From Exercise 44, x  5. So
mQLM  2(5)2  10 or 40. Find mLQP. 58. Find the slope of a segment given the endpoints
mQLM  mLQP ¬180 (c, c) and (c, d).
y2  y1
40  mLQP ¬180 slope of segment ¬ 
x2  x1
mLQP ¬140 d c
47. The diagonals of a rhombus bisect the angles, so ¬
cc

d c
mLQM  1 1
2 mLQP. mLQM  2 (140) or 70.
 ¬ 
0 , which is undefined
48. By definition, a rhombus has four congruent 59. Find the slope of a segment given the endpoints
sides, so the perimeter is 4(10) or 40. (a, b) and (2a, b).
y y
49. Use the Distance Formula to find RS and TV. slope of segment ¬
2
1
x2  x1
RS ¬  (7  0)2 
(3  4)2 b  ( b)
¬  49  4
9  7  2 ¬ 
2a  a
2b
TV ¬  [3  (
4)]2 
 [1 (7)]2 ¬a
¬  49  6
4   113 60. Find the slope of a segment given the endpoints
T
50. Find the coordinates of the midpoints of R and (3a, 2b) and (b, a).
V
S  using the Midpoint Formula. y y
slope of segment ¬
2
1
x2  x1
T
R 
7 
:  
2 ,
3 
3 
2
 1

 (2, 1)
a 
¬ 2b
b 3a
: 
V
S 2, 2  2, 2
0  (4) 4  (7) 3 a 
¬ 2b
3a  b
51. No; RSTV is not a rectangle because opposite 61. Find the slope of a segment given the endpoints
sides are not congruent (Exercise 49) and the (b, c) and (b, c).
diagonals do not bisect each other (Exercise 50). y y
slope of segment ¬
2
1
x2  x1
52. Solve the proportion for y.
c  c
16 24 ¬b 
38 ¬ y
   b
16y ¬24(38) ¬bc
16y ¬912
y ¬57
53. Solve the proportion for y.
y 17
 ¬
30
6
30y ¬17(6)
30y ¬102
17
y ¬5


Chapter 8 276
Page 445 Practice Quiz 2 Opposite sides are parallel. Since one diagonal is
1. BAD is a right angle, so BAC and CAD are vertical and the other is horizontal, they are
complementary. Find x. perpendicular. Since the slopes of consecutive
mBAC  mCAD ¬90 sides are not opposite reciprocals, they are not
(2x  1)  (5x  5) ¬90 perpendicular. So, MNPQ is a rhombus.
7x ¬84 4. The median of a trapezoid is parallel to the bases,
x ¬12 and its measure is one-half the sum of the
2. ABD  BDC because they are alternate measures of the bases. Find MN.
interior angles. Find y. MN ¬1
2 (TR  VS)

ABD ¬BDC ¬1
2 (44  21)

mABD ¬mBDC
¬32.5
y2 ¬3y  10
5. The median of a trapezoid is parallel to the bases,
y2  3y  10 ¬0
and its measure is one-half the sum of the
(y  5)(y  2) ¬0
measures of the bases. Find VS.
y  5  0 or y  2  0
y5 y  2 MN ¬1
2 (TR  VS)

If y  5, mABD  52 or 25. If y  2, mABD 25 ¬1
2 (32  VS)

 (2)2 or 4. ABD  BAC because they are
base angles of an isosceles triangle, so their 50 ¬32  VS
measures are equal. 18 ¬VS
From Question 1, x  12.
So, mBAC  2(12)  1 or 25, therefore, y is 5.
Reject y  2 because it leads to a contradiction. Page 446 Reading Mathematics: Hierarchy of
3. Polygons
y
4 1. False; all jums are mogs and some mogs are jums.
N
x
Mogs is of a higher class than jums in the
hierarchy.
12 8 4 4
M Q 2. False; both jebs and jums are mogs, but no jebs
4
are jums. Jebs and jums are distinct members of
8 the same class.
P 3. True; all lems are jums because every element of
12 a class is contained within any class linked above
it in a hierarchy diagram.
If opposite sides are parallel, then MNPQ can be
4. True; some wibs are jums and some wibs are jebs.
a rhombus, a rectangle, or a square. If diagonals
Jums and jebs are members of the same class in
are perpendicular, then MNPQ can be a rhombus
the hierarchy. Wibs is a member of the class below
or a square. If consecutive sides are
and is directly linked to both jums and jebs.
perpendicular, then MNPQ is a rectangle or a
square. 5. True; all mogs are bips because every element of a
Use the Slope Formula to determine whether class is contained within any class linked above it
opposite sides are parallel and consecutive sides in a hierarchy diagram.
are perpendicular. 6. All triangles are polygons. Both isosceles and
3  3 scalene triangles are triangles. All equilateral
N
slope of M ¬ 
5  (2) triangles are isosceles triangles.
¬2 Polygons
9 
(3)
Q
slope of P ¬2  1
¬2 Triangles
3  (
3)
Q
slope of N ¬2  1 Isosceles Scalene
¬2
3 (9) Equilateral
P
slope of M ¬
5  (2)
¬2
Use the Slope Formula to determine whether the
diagonals are perpendicular. 8-7 Coordinate Proof With
3  (9) Quadrilaterals
P
slope of N ¬ 
2  (2)
12
¬0 , which is undefined Page 448 Geometry Software Investigation:
3  (3) Quadrilaterals
Q
slope of M  ¬ 5  1
¬0 1. See students’ work.
2. A parallelogram is formed by the midpoints since
the opposite sides are congruent.

277 Chapter 8
Pages 449–450 Check for Understanding 8. Given: D(195, 180), E(765, 180), F(533, 0),
1. Place one vertex at the origin and position the G(195, 0)
figure so another vertex lies on the positive x-axis. Prove: DEFG is a trapezoid.
2. Sample answer: y
y D (195, 180) E(765, 180)
D
C

B O G(195, 0) F(533, 0) x
O A x
Proof:
180 180
3. y E
Slope of D
765  195 or 0
C (a, a b) 0 0
D (0, a b) F
Slope of G
533  195 or 0
0
180  4
5
F
Slope of E  
765  533 or 58
0
180 
G
Slope of D  195  195 or undefined
E
D and G F
 have the same slope, so exactly one
O A(0, 0) B(a, 0) x pair of opposite sides are parallel. Therefore,
4. The quadrilateral is a square. Opposite sides of a DEFG is a trapezoid.
square are congruent and parallel, and its interior
angles are all 90°. So, the x-coordinate of C is a
and the y-coordinate is a. Pages 450–451 Practice and Apply
The coordinates of C are (a, a). 9. y
5. The quadrilateral is a parallelogram. Opposite D (b, c) C (a b, c)
sides of a parallelogram are congruent and
parallel. So, the y-coordinate of D is b.
The length of A B is a, and the length of D
C
 is a.
So, the x-coordinate of D is (a  c)  a or c. O A(0, 0) B(a 2b, 0) x
The coordinates of D are (c, b). 10. y
6. Given: ABCD is a parallelogram. D (a, b) C (c a, b)
Prove: AC and DB bisect each other.
y
D (b, c) C (a b, c) O A(0, 0) B(c, 0) x
11. The quadrilateral is a parallelogram. Opposite
sides of a parallelogram are congruent and
O A(0, 0) B(a, 0) x parallel. So, the y-coordinate of B is c.
Proof: The length of H G is a  b , and the length of B
C

C
The midpoint of A 0  (a
 ¬  2
 b) 

, 0
2
c is a  b. So, the x-coordinate of B is a  (a  b)
or b.
¬ 2 , 2
a b c The coordinates of B are (b, c).
12. The quadrilateral is a square. Opposite sides of a
  
B
The midpoint of D 2 , 2
a b 0c
square are congruent and parallel, and its interior
angles are all 90°. So, the x-coordinate of A is b
¬ 2 , 2
a b c
and the y-coordinate is b, and the
C
A and D
B bisect each other. x-coordinate of E is b and the y-coordinate is b.
The coordinates of A and E are (b, b) and
7. Given: ABCD is a square.
(b, b), respectively.
C
Prove: AD B
y
13. The quadrilateral is a parallelogram. Opposite
C (a, a) sides of a parallelogram are congruent and
D (0, a)
parallel. So, the y-coordinates of E and G are c
and 0, respectively.
The length of H G is a, and the length of E
F
 is a.
O A(0, 0) B(a, 0) x So, the x-coordinate of G is a, and the x-coordinate
of E is (a  b)  a or b.
Proof: The coordinates of G and E are (a, 0) and (b, c),
0 a
B
Slope of D
a  0 or 1
respectively.
0  a 14. The quadrilateral is an isosceles trapezoid. The
C
Slope of A  0  a or 1 top and bottom sides of the trapezoid are parallel,
C
The slope of A  is the negative reciprocal of the so the y-coordinate of M is c.
B
slope of D, so they are perpendicular. The length of L M
 is a  2b, so the x-coordinate of
M is (a  b)  (a  2b) or b.
The coordinates of M are (b, c).

Chapter 8 278
15. The quadrilateral is a rectangle. Opposite sides of D
19. Given: isosceles trapezoid ABCD with AB
C

a rectangle are congruent and parallel, and its D
Prove: B A C
interior angles are all 90°. So, the y-coordinates of y
T and W are c and c, respectively. D (b, c) C (a b, c)
The origin is at the center of the rectangle, so the
x-coordinates of T and W are both 2a (the
opposites of the x-coordinates of U and V). O A(0, 0) B(a, 0) x
The coordinates of T and W are (2a, c) and
Proof:
(2a, c), respectively.
BD ¬  (a  b
)2  (0 c)2  (a  b)2  c2
16. The quadrilateral is an isosceles trapezoid. The
right and left sides of the trapezoid are parallel, AC ¬  ((a  
b)  0
)2  (c 0)2  (a  b)2  c2
so the x-coordinates of T and S are 0 and a, BD ¬AC and B DA C
respectively. 20. Given: ABCD is a trapezoid with median X Y .
The length of QT is a, so the y-coordinate of T is Y
Prove: X A B
 and XYD C
1
2 S
a. The length of R  is 2a  2c, so the y
y-coordinate of S is (a  c)  (2a  2c) or a  c. A( b, c) B(a b, c)


The coordinates of T and S are 0, 1
 
2 a and
X Y
(a, a  c), respectively. O D(0, 0) C (a, 0) x
17. Given: ABCD is a rectangle. D
Proof: The midpoint of A is X. The coordinates
C
Prove: ADB
y are 
b c
 C
2 , 2 . The midpoint of B
2a 
 is Y  
b c
2, 2 . 
A(0, b) B(a, b) The slope of A B
  0, the slope of X
Y
  0 and the
C
slope of D   0. Thus, XY
A B
 and XYDC.
21. Given: ABCD is a rectangle.
O D(0, 0) C (a, 0) x Q, R, S, and T are midpoints of their
respective sides.
Proof: Use the Distance Formula to find Prove: QRST is a rhombus.
AC  a  and BD  
2  b2 a2  b2. A
C
 and D
B

y C(a, b)
have the same length, so they are congruent. D(0, b) R
18. Given: ABCD and A C
B D

Prove: ABCD is a rectangle. Q S
O
y
A(0, 0) T B(a, 0) x
D (b, c) C (a b, c)
Proof:

O A(0, 0) B(a, 0) x
Midpoint Q is  
0 
0 b0 b
2 , 2 or 0, 2 .   
Midpoint R is  2 , 2 or 2, 2 or 2, b
a  0 bb a 2b a
Proof:
AC   (a  b 0)2 (c 
Midpoint S is  2 , 2 or 2, 2 or a, 2.
0)2 a  a b0 2a b b
BD   (b  a)  (c
2  0) 2

But AC  BD and Midpoint T is  2 , 2 or 2, 0.


a  0 00 a

 (a  b  0)2 (c 0)2 ¬  (b  a)


2  (c
 0)2.
QR  a2  0   b 2
 ¬ a  b

2 2 2 2
(a  b  0)  (c  0) ¬(b  a)  (c  0)2
2 2 2  b 2 2
(a  b)2  c2 ¬(b  a)2  c2
RS  a  a2
  b2  b 
2 2
a  2ab  b2  c2 ¬b2  2ab  a2  c2
2

2ab ¬2ab
¬  2  b2 or  2  b2
2 2 2 2
4ab ¬0 a a
a  0 or b ¬0
ST  a  a2
   2  0 ¬  2  b2
2 2 2 2
Because A and B are different points, a  0. Then b a
b  0. The slope of A D
 is undefined and the slope
QT   2  0   0 2

2 2
B
of A   0. Thus A D
A B. DAB is a right angle a  b
and ABCD is a rectangle.
¬ a2  b2 or  2  b2
2 2 2 2
a

QR  RS  ST  QT
R
QR S
S T
QT
QRST is a rhombus.

279 Chapter 8
22. Given: RSTV is a quadrilateral. 24. y
D(c, d)
S
A, B, C, and D are midpoints of sides R,
T
S, T
V, and V
R
, respectively. C(a 4.5, b)
Prove: ABCD is a parallelogram.
y
T (2d, 2b)
S(2a, 2e) O
B A(0, 0) B(a, 0) x
C
A 25. No, there is not enough information given to
O prove that the sides of the tower are parallel.
R(0, 0) D V(2c, 0) x
26. From the information given, we can approximate
Proof: Place quadrilateral RSTV on the the height from the ground to the top level of the
coordinate plane and label coordinates as shown. tower.
(Using coordinates that are multiples of 2 will 27. Sample answer: The coordinate plane is used in
make the computation easier.) By the Midpoint coordinate proofs. The Distance Formula, Midpoint
Formula, the coordinates of A, B, C, and D are Formula and Slope Formula are used to prove

 
2a theorems. Answers should include the following.
 2e
A 2 , 2 ¬(a, e); • Place the figure so one of the vertices is at the
B
2d 
2 , 2e 
2a  
2
2b
 ¬(d  a, e  b); origin. Place at least one side of the figure on
the positive x-axis. Keep the figure in the first
C 2d 
2
2c   ¬(d  c, b); and
, 22b quadrant if possible and use coordinates that
will simplify calculations.
D22 , 2  ¬(c, 0).
c 0 • Sample answer: Theorem 8.3: Opposite sides of
a parallelogram are congruent.
B
Find the slopes of A and D
C
. 28. D; ABCD is a parallelogram, so opposite sides are
B
Slope of A C
Slope of D parallel. BCAD and B
C  lies along the
y y y y x-axis, so AD
 is parallel to the x-axis and is
m ¬2

x x
1
m2

x x
1
2 1 2 1 horizontal. Therefore, D must have the same
(e  b)  e 0 b y-coordinate as A.
¬ 
(d  a)  a 
c  (d  c)
Opposite sides of a parallelogram are congruent,
b 
b b D
so A B C
 and AD  BC. According to the
¬d  d or d
The slopes of AB and DC
 are the same so the Distance Formula, BC  (x  2 x1)2 
(y2 
y1)2
segments are parallel. Use the Distance Formula   (c  b
)  0  c  b. So, the length of AD is
2

to find AB and DC. c  b. AD is horizontal, the x-coordinate of D must


AB ¬  ((d  
a)  a)
2  ((
e  b) e)2 be greater than that of A, and c  b
0, therefore,
¬ d 2  b
2 the x-coordinate of D must be c  b.
The coordinates of point D are (c  b, a).
DC ¬  ((d  
c)  c
)2  (b 0)2
29. A; if p  5, then 5  p2  p  5  (5)2  (5)
¬ d 
2  b2
or 15.
Thus, AB  DC. Therefore, ABCD is a
parallelogram because if one pair of opposite sides
of a quadrilateral are both parallel and
congruent, then the quadrilateral is a
Page 451 Maintain Your Skills
parallelogram. N
30. Given: MNOP is a trapezoid with bases M and
P
O, M
N
Q O
23. Sample answer:
Prove: MNOQ is a parallelogram.
y
P
Q O
B (a, b) c C (a  c, b)
M N
Proof:
x Statements Reasons
A (0, 0) a X c Y a D (2a  c, 0)
1. MNOP is a trapezoid with 1. Given
N
bases M and O
P.
Graph points A(0, 0) and B(a, b). If ABCD is an N
MQ O

C
isosceles trapezoid, BAD. So B
C is horizontal. P
2. OM
N
 2. Def. of trapezoid
Let b be the y-coordinate of C. Since ABCD is 3. MNOQ is a parallelogram. 3. If one pair of
isosceles AB  CD. AX  a. Let XY  c. Then opp. sides are 
YD  a because ABX  DCY by HL. and , the
The coordinates of C and D are C(a  c, b) and quad. is .
D(2a  c, 0).

Chapter 8 280
31. Opposite angles of a rhombus are congruent and Chapter 8 Study Guide and Review
the diagonals of a rhombus bisect opposite angles,
so RMP  MPR and their measures are equal.
Since RMP  JMK and mJMK  55,
Page 452 Vocabulary and Concept Check
mMPR  55. 1. true
32. KJM and MLK are right angles and JKM 2. true
and KLM are right triangles because the 3. false; rectangle
interior angles of a rectangle are right angles. 4. true
Opposite sides of a rectangle are parallel, so 5. false; trapezoid
LKM  JMK and their measures are equal 6. false; rhombus
because they are alternate interior angles.
7. true
Therefore, mLKM  55. The sum of the
measures of the interior angles of a triangle is 8. true
180. Find mKML.
mKLM  mLKM  mKML ¬180
90  55  mKML ¬180 Pages 452–456 Lesson-by-Lesson Review
mKML ¬35 9. Use the Interior Angle Sum Theorem.
33. KLM is a right angle because the interior angles S  180(n  2)
of a rectangle are right angles. Opposite angles of  180(6  2)  720
a rhombus are congruent, so MLP  MRP and The measure of an interior angle of a hexagon is
their measures are equal. Therefore, mMLP  70. 72
 0
6 or 120.
The measure of KLP is the sum of the measures
10. Use the Interior Angle Sum Theorem.
of KLM and MLP. Find mKLP.
S  180(n  2)
mKLP ¬mKLM  mMLP
 180(15  2)  2340
mKLP ¬90  70
mKLP ¬160 The measure of an interior angle of a regular
234 0
34. Let x represent the geometric mean. 15-gon is  
15 or 156.
7 x 11. Use the Interior Angle Sum Theorem.
x ¬
1
4 S  180(n  2)
x2 ¬98
 180(4  2)  360
x ¬ 98 
The measure of an interior angle of a square is
x ¬9.9 36
0

The geometric mean of 7 and 14 is 98  or about 4 or 90.
9.9. 12. Use the Interior Angle Sum Theorem.
35. Let x represent the geometric mean. S  180(n  2)
2   ¬x
5  180(20  2)  3240

x 
6 5 The measure of an interior angle of a regular
x2 ¬60 324
20-gon is  0
20 or 162.
x ¬ 60
 13. Since n  4, the sum of the measures of the
x ¬7.7 interior angles is 180(4  2) or 360. Write an
The geometric mean of 2 5  and 6 5 is equation to express the sum of the measures of
60
 or about 7.7. the interior angles of the polygon.
36. VXY is an exterior angle of WVX. So 360  mW  mX  mY  mZ
mWVX mVXY.
37. VZ and XZ are equal, so V ZX Z
 and VXZ is 1

360  2a  8  a  (a  28)  (a  2)
an isosceles triangle. The base angles of an 7
360  2a  18
isosceles triangle are congruent, so XVZ 
VXZ and mXVZ  mVXZ. 378  7
2a
38. WYV  XYV because they are the same angle. 108  a
VXY and XYV are two interior angles of Use the value of a to find the measure of each angle.
VXY. The side of VXY opposite VXY
measures 6  6 or 12. The side of VXY opposite mW  1
2 (108)  8 or 62, mX  108, mY 

XYV measures 8. According to Theorem 5.9, if 108  28 or 80, and mZ  108  2 or 110.
one side of a triangle is longer than another side,
then the angle opposite the longer side has a
greater measure than the angle opposite the
shorter side. Therefore, mXYV mVXY.
39. Z
X X Z
 and VZY Z. XY VX, so
mXZY mXZV by the SSS Inequality
Theorem.

281 Chapter 8
14. Since n  5, the sum of the measures of the AD ¬ [2  (1)]2
 [5 2
 (2)]
interior angles is 180(5  2) or 540. Write an ¬ 
50
equation to express the sum of the measures of
BC ¬ (4  6
)2  [4 2
 (3)]
the interior angles of the polygon.
¬ 
53
540  mA  mB  mC  mD  mE
540  (x  27)  (1.5x  3)  (x  25)  AD  BC, so ABCD is not a parallelogram.
(2x 22)  x 22. Yes;
540  6.5x  33 y
507  6.5x
12
78  x
Use the value of x to find the measure of each 8
angle. J K
mA  78  27 or 105, mB  1.5  78  3 or 4
120, mC  78  25 or 103, mD  2  78  22 or H L
x
134, and mE  78.
4 8 12
15. BCD  BAD because opposite angles in a
parallelogram are congruent. Find mBAD. If the midpoints of the diagonals are the same,
mBAD  mCAD  mBAC the diagonals bisect each other. If the diagonals of
 20  32 a quadrilateral bisect each other, then the
 52 quadrilateral is a parallelogram.
So, mBCD  52. Find the midpoints of HK and J
L.
16. The diagonals of a parallelogram bisect each
other, so AF and CF are equal. Therefore, AF  7.
K
H 0 
:  
5 46
  
5
2 , 2 ¬ 2, 5

17. Consecutive angles in a parallelogram are : 


L
J 2 , 2  ¬ 2 , 5
4 
 6 
9  4 5
supplementary. So, mADC  180  mBCD.
From Exercise 15, mBCD  52. K
The midpoints of H  and J
L
 are the same, so
So, mADC  180  52 or 128. HKJL is a parallelogram.
ADB  CBD because they are alternate 23. Yes;
interior angles. So mADB  43.
y
mBDC  mADC  mADB
V
 128  43 12
 85
So, mBDC  85. 8
W
18. Opposite sides of a parallelogram are congruent, T
4
so their measures are equal and BC  AD.
Therefore, BC  9. x

19. Opposite sides of a parallelogram are congruent, 12 8 4 S 4


so their measures are equal and CD  AB.
If the opposite sides of a quadrilateral are
Therefore, CD  6.
parallel, then it is a parallelogram.
20. Consecutive angles in a parallelogram are 1  5
supplementary. So, mADC  180  mBAD. T
slope of S ¬ 
2  2
Find mBAD. ¬3

2
mBAD  mCAD  mBAC
13  7
 20  32 W
slope of V ¬
10 (14)
 52 ¬3

2
So, mADC  180  52 or 128. 5 
13
V
slope of T ¬
2  (10)
21. No;
A y ¬2

3
B 1  7
W
slope of S ¬
2  
(14)
¬2

3
x T
Since opposite sides have the same slope, S VW

V
and T S W. Therefore, STVW is a parallelogram
by definition.
D
C
If both pairs of opposite sides of a quadrilateral
are congruent, then the quadrilateral is a
parallelogram. Use the Distance Formula to
determine whether the opposite sides are
congruent.

Chapter 8 282
24. The diagonals of a rectangle bisect each other and S
So, RT V
 and S 
TV R. Use the Distance
are congruent, so AF  1

2 AC.
Formula to determine whether the diagonals of
quadrilateral RSTV are congruent.
AF ¬1

2 AC RT ¬  (0  4 )2  (0
 7)2
2x  7 ¬1

2 (26) ¬  16  4 9
2x ¬13  7 ¬  65
x ¬3 SV ¬  
[6  (2)] 2 (3 4)2
So, AF  2(3)  7 or 13. ¬  64  1
25. The diagonals of a rectangle are congruent and ¬  65
bisect each other. So, the triangles formed by the Since the opposite sides are parallel and the
diagonals of a rectangle are isosceles. Therefore, diagonals are congruent, RSTV is a rectangle.
2  1 and m2  52.
30. The diagonals of a rhombus bisect the angles, so
26. The diagonals of a rectangle bisect each other and 1  2. Find x.
are congruent, so CF
D F. Find x. 1 ¬2
 ¬D
CF F m1 ¬m2
CF ¬DF 2x  20 ¬5x  4
4x  1 ¬x  13 24 ¬3x
3x ¬12 8 ¬x
x ¬4 31. The diagonals of a rhombus bisect each other, so
27. The interior angles of rectangles are right angles, AF  1 1
2 AC  2 (15) or 7.5.

and the sum of the measures of the interior 32. The diagonals of a rhombus are perpendicular, so
angles of a triangle is 180. So, the sum of the m3 is 90. Find y.
measures of 2 and 5 is 90. Find m5. m3 ¬90
m2  m5 ¬90 y2  26 ¬90
(70  4x)  (18x  8) ¬90 y2 ¬64
14x ¬28 y ¬8 or 8
x ¬2
33. C
B AD, so ADY and BCY are supplementary.
So, m5  18(2)  8 or 28.
Find mBCY.
S
28. Find the slopes of R , S
T
, T
V, and VR
. mBCY  mADY ¬180
5 
(5)
S
slope of R ¬3  0 mBCY  78 ¬180
¬0 mBCY ¬102
5  4 Both pairs of base angles of an isosceles trapezoid
T
slope of S ¬ 
0  (3) are congruent, so XBC  BCY and mXBC
¬3  102.
4 
4 34. The median of a trapezoid is parallel to the bases,
V
slope of T ¬
30
and its measure is one-half the sum of the
¬0 measures of the bases. Find JM.
4  (
5)
R
slope of V ¬ AB ¬1
2 (KL  JM)
0  (3) 
¬3
57 ¬1
2 (21  JM)

The slopes of consecutive sides are not negative
reciprocals, so consecutive sides are not 114 ¬21  JM
perpendicular. Therefore, RSTV is not a rectangle. 93 ¬JM

(Note: S T  V
R
 so RSTV is not even a 35. Given: ABCD is a square.
parallelogram. So it is not a rectangle.) C
Prove: A B D

29. If the opposite sides of a quadrilateral are parallel y
and the diagonals of the quadrilateral are C(a, a)
D(0, a)
congruent, then the quadrilateral is a rectangle.
Find the slopes of RS
, S
T, T
V
, and V
R
.
0 
3
S
slope of R ¬
06 O A(0, 0) B(a, 0) x
¬1

2 Proof:
7 
4 a 0
V
slope of T ¬ C
Slope of A
4  (2) a  0 or 1
¬1
 a 0
2 D
Slope of B
0  a or 1
3 
7 C
 is the negative reciprocal of the
T
slope of S ¬
64
The slope of A
D
slope of B. Therefore, A
C
B D
.
¬2
40
R
slope of V ¬
2 
0
¬2

283 Chapter 8
36. Given: ABCD is a y If the midpoints of the diagonals are the same,
parallelogram. D(b, c) C (a b, c) the diagonals bisect each other. If the diagonals of
Prove: ABC  CDA a quadrilateral bisect each other, then the
quadrilateral is a parallelogram.
Find the midpoints of AC and B
D.
Proof:
O A(0, 0) B(a, 0) x
C
A 
4 
:  4 
2 ,
3  (8)
2   
  4, 5
2
AB  (a  0
)2  (0
 0)2  a2  02 or a
: 
D
B 6 
2 
2 ,
  4, 5
0  (5)
DC  
[(a  
b)  b]
2  (c
 c)2  
a2  02 or a 2 2

AD  
(b  0
)2  (c
 0)2  
b2  c2 The midpoints of AC
 and BD
 are the same, so the
diagonals bisect each other and ABCD is a
BC  
[(a  
b)  a
]2  (c
 0)2  
b2  c2
parallelogram.
AB and DC have the same measure, so A B
D C. 9. Yes;
AD and BC have the same measure, so A D
B C.
y
C
A A C by the Reflexive Property. Therefore, 12
ABC  CDA by SSS. T
37. The quadrilateral is an isosceles trapezoid. The 8 V
top and bottom sides of the trapezoid are parallel, S
4
so the y-coordinate of P is c.
The length of M N is 4a, so the x-coordinate of P is W x
4a  a or 3a. 8 4
O
4 8
The coordinates of P are (3a, c). 4
38. The quadrilateral is a parallelogram. Opposite
sides of a parallelogram are congruent and First use the Distance Formula to determine
parallel. So, the y-coordinate of U is c. whether the opposite sides are congruent.
The length of V W
 is b  (a) or a  b. So the ST ¬  (2  2)2 
(6  11)2
U
length of T  is also a  b. So, the x-coordinate of ¬  41
U is (a  b)  0 or a  b.
VW ¬  [3  (
1)]2 
 (8 3)2
The coordinates of U are (a  b, c).
¬  41
Since ST  VW, S TV W.
Next, use the Slope Formula to determine
Chapter 8 Practice Test whether S T
V W
.
6 
11
T
slope of S  ¬
2  2
Page 457
1. true ¬5

4
8 
3
2. false; 3. false; W
slope of V ¬
3  (1)
¬5

4

ST and V
W have the same slope, so they are
parallel. Since one pair of opposite sides is
congruent and parallel STVW is a parallelogram.
4. 
HKF G
 because opposite sides of parallelograms 10. No;
are congruent. y
8
5. FKJ  HGJ because alternate interior angles
are congruent. 4 H
J
6. FKH  FGH because opposite angles of x
parallelograms are congruent. O
8 12
H
7. G F K
 because opposite sides of parallelograms 4
G
are parallel. F
8. Yes; 8
y
If both pairs of opposite sides of a quadrilateral
4 are congruent, then the quadrilateral is a
A
B x parallelogram. Use the Distance Formula to
O determine whether the opposite sides are
4 8 12
4 D congruent.

8
C

Chapter 8 284
FG ¬  (7  4
)2  [ 2
3  (2)] 14. y
8
¬ 10 D
HJ ¬  (6  1
2)2  
(4  2
)2 4
¬ 40 C A
GH ¬  (4  6)
2  (
2  4)2 O 4 8 12 x
¬ 40 4
FJ ¬  (7  1
2)2  
(3  2)2
8 B
¬ 50
Since the measures of both pairs of opposite sides If the four sides are congruent, then
are not equal, they are not congruent. Therefore, parallelogram ABCD is either a rhombus or a
FGHJ is not a parallelogram. square. If consecutive sides are perpendicular,
11. Yes; then ABCD is a rectangle or a square.
y Y Use the Distance Formula to compare the lengths
X of the sides.
AB ¬  (12  6)2  2
[0  (6)]
¬  36  36
Z ¬6  2
W BC ¬  (6  0)2  (
6  0)2
x
¬  36  36
O
¬6  2
CD ¬  (0  6)2  (0
 6)2
If both pairs of opposite sides of a quadrilateral
are congruent, then the quadrilateral is a ¬  36  36
parallelogram. Use the Distance Formula to ¬6  2
determine whether the opposite sides are AD ¬  (12  6)2 (0  6
)2
congruent. ¬  36  36
WX ¬  [4  (3)]2  (2 6)2 ¬6  2
¬ 17 Use the Slope Formula to determine whether the
YZ ¬  (2  1)2  (7  3)2 consecutive sides are perpendicular.
¬ 17 B
slope of A 0  (
 ¬  6)
12  6
XY ¬  (3  2)2  (6  7)2
¬1
¬ 26
0
6 
WZ ¬  (4  1)2  (2  3)2 C
slope of B ¬60
¬ 26 ¬1
Since the measures of both pairs of opposite sides 0 
6
D
slope of C ¬
06
are equal, they are congruent. Therefore, WXYZ is
a parallelogram. ¬1
12. The diagonals of a rectangle bisect each other, so 06
D
slope of A ¬ 
12  6
P
Q P S. Find x.
P
Q  ¬P
S ¬1
QP ¬PS Since the slopes of AB
 and CD are negative
3x  11 ¬4x  8 C
reciprocals of the slopes of B and AD
,
3 ¬x consecutive sides are perpendicular. The lengths
Find QS. of the four sides are the same, so the sides are
QS  QP  PS congruent. Therefore, ABCD is a rectangle, a
 (3x  11)  (4x  8) rhombus, and a square.
 7x  19
 7(3)  19
 40
So, QS  40.
13. Opposite sides of a rectangle are parallel, so
QTR  SRT because they are alternate
interior angles. Find x2.
QTR ¬SRT
mQTR ¬mSRT
2x2  7  x2 ¬18
x2 ¬25
So, mQTR  2(25)  7 or 43.

285 Chapter 8
15. y 17. y
12

8 D(?, ?) C(?, ?)
B
A C

D
O x A(0, 0) x
4 8 12
4 b X a Y b B(a  2b, 0)

If the four sides are congruent, then


parallelogram ABCD is a square or a rhombus. If Sample answer: To find the y-coordinates of C and
the diagonals are congruent, then ABCD is a D
D notice that C  is parallel to the x-axis. So C
D
 is
square or a rectangle. If the diagonals are a horizontal segment, and C and D both have the
perpendicular, then ABCD is a square or a same y-coordinate. Call it c.
rhombus. To find the x-coordinate of D, notice that AX  b
Use the Distance Formula to compare the lengths so the x-coordinate of D is b. The x-coordinate of c
of the sides. is the same as the x-coordinate of Y or a  b.
AB ¬  (2  5)2  (4  6)2 So the coordinates of D and C are D(b, c) and
¬  49  4   53 C(a  b, c)
BC ¬  (5  12)2  
(6  4)2 18. Given: trapezoid WXYZ with median S T
Prove: W X
S TYZ
¬  49  4   53
y
CD ¬  (12  5)2 (4  2)2
W(0, 2d) X(b, 2d)
¬  49  4   53
AD ¬  (2  5)2  (4  2)2 S( a, d) T(a b, d)
¬ 494     53
Use the Distance Formula to compare the lengths Z( 2a, 0) O Y(2a b, 0) x
of the diagonals. Proof:
AC ¬  (2  12)2  (4  4)2 To prove lines parallel, show their slopes equal.
¬  196  0  14 2d  2d
The slope of WX is  b  0 or 0.
BD ¬  (5  5
)2  (6 2)2 d d
The slope of ST is 
(a  b)  (a) or 0.
¬  0  16  4
00
Use the Slope Formula to determine whether the The slope of YZ is 
(2a  b) 
 (2a) or 0.
diagonals are perpendicular. Since WX, ST, and YZ all have zero slope, they
4 4 are parallel.
C
slope of A ¬
2  12
19. The median of a trapezoid is parallel to the bases,
¬0 and its measure is one-half the sum of the
6 
2 measures of the bases. Find the length of the mid-
D
slope of B ¬
55
chord of the keel.
¬4

0 , which is undefined length of mid-chord  1
2 (length of root chord 

C
A  is horizontal and B D
 is vertical, so the length of tip chord)
diagonals are perpendicular, but not congruent
since AC  BD. The lengths of the four sides are ¬1
2 (9.8  7.4)

the same, so the sides are congruent. Therefore, ¬1

2 (17.2)
ABCD is a rhombus.
16. The quadrilateral is a parallelogram. Opposite ¬8.6
sides of a parallelogram are congruent and The length of the mid-chord is 8.6 ft.
parallel. So, the y-coordinate of P is c. 20. C; use the Interior Angle Sum Theorem to write
The length of M Q  is b  (a) or a  b and the an equation to solve for n, the number of sides.
P
length of N  is a  b. So, the x-coordinate of P is S ¬180(n  2)
(a  b)  0 or a  b. (108)n ¬180(n  2)
The coordinates of P are (a  b, c). 108n ¬180n  360
0 ¬72n  360
360 ¬72n
5 ¬n
The polygon has 5 sides.

Chapter 8 286
Chapter 8 Standardized Test Practice 8. Set y equal to zero to find the x-coordinate at
which the graph crosses the x-axis.
y ¬4x  5
Pages 458–459 0 ¬4x  5
1. C; the length of the ramp is the hypotenuse of a 4x ¬5
right triangle with legs measuring 3 meters and 5
x ¬5

4
meters. Use the Pythagorean Theorem to find the
length of the hypotenuse. The point at which the graph crosses the x-axis is
c ¬ a  
2  b2
54, 0.
¬  3  52
2
9. If one side of a triangle is longer than another
¬  34 side, then the angle opposite the longer side has a
¬6 greater measure than the angle opposite the
To the nearest meter, the length of the ramp shorter side. The side representing the path from
should be 6 m. Candace’s house to the theater is opposite a 55°
angle, and the side representing the path from
2. D; the contrapositive of the statement “If an
Julio’s house to the theater is opposite a 40°
astronaut is in orbit, then he or she is weightless”
angle. Since 55
40, Julio’s house is closer to the
is “If an astronaut is not weightless, then he or
theater.
she is not in orbit.”
D
10. C  is the altitude of right triangle ABC so, by
3. B; for the two rectangles to be similar, the
Theorem 7.2, its measure is the geometric mean
measures of their corresponding sides must
between the segments of the hypotenuse.
proportional. The ratio of the length to the width
Let x  CD
of QRST is 7

4 or 7 : 4. The ratios of the choices, 4
x ¬
x
25
28 21
 7  14
 7 7
14  2, 12  4 , 4  2 , and 8 .
from A to D, are:  
11. The sides of a rhombus are congruent, so A C
The dimensions, 21 cm by 12 cm, could be the separates rhombus ABCD into two isosceles
dimensions of a rectangle similar to QRST. triangles. The base angles of an isosceles triangle
4. C; the ladder, wall, and ground form a 30°-60°-90° are congruent, so ACD  CAD.
right triangle. The ladder is the hypotenuse, the CDE is an exterior angle of ACD so mCDE 
wall is the longer leg, and the ground is the mCAD  mACD.
shorter leg. In a 30°-60°-90° triangle, the length of Substituting mACD for mCAD,
the hypotenuse is twice the length of the shorter 116  mACD  mACD
leg, and the length of the longer leg is  3 times 116  2(mACD)
the length of the shorter leg. So, the shorter leg is 58  mACD
24 12a. MNR and PQR are both right angles and all
2 or 12 ft, and the longer leg is 12 
  3 ft. The
ladder reaches 12  3 ft up the side of the house. right angles are congruent, so MNR  PQR.
Since congruence of angles is reflexive, R 
5. B; the diagonals of a rectangle are congruent, so
R. MNR is similar to PQR because two
L
J K M. Find x.
angles are congruent (AA Similarity).
L
J  ¬K
M
JL ¬KM 12b. The ratios of corresponding sides of similar
2x  5 ¬4x  11 MR PR
polygons are the same, so 
MN 
QP.
16 ¬2x
a
400  40
0
The proportion is  126  120 . Solve for a.
8 ¬x 
6. C; the diagonals of a rhombus bisect each other a
400  40
0
126 ¬ 120
 
but are not necessarily congruent.
7. A; the bases of a trapezoid are parallel, so AB is
40
400  a ¬126  0
120 
D
parallel to C . a ¬420  400
a ¬20
The distance across the sand trap, a, is 20 yards.

287 Chapter 8
c0 c
13a. Given: quadrilateral ABCD C
13b. The slope of A is a  b 0 or 
a b.
Prove: ABCD is a parallelogram c  0 c
D
The slope of B  is 
b a or 
b a.
y
c c
D (b, c) C (a b, c) The product of the slopes is a b  ba
c2
b2  a2 . Since c  a  b , the product of the
2 2 2

a 
2 2
b
O A(0, 0) B(a, 0) x b2  a2 or 1, so the diagonals of ABCD
slopes is 
Proof: are perpendicular.
c 0 c
D
The slope of A  is  C
b  0 or b . The slope of B is 13c. Since the diagonals are perpendicular, ABCD is a
c  0 c rhombus.
b D and B C
 have the same slope
 or . A
aba
so they are parallel.
AD   (b  0)
2  (c 0)2  b .
2  c2

BC   (a  b a) 
2  (c 
0)  b
2  .
2  c2

Since one pair of opposite sides are parallel and


congruent, ABCD is a parallelogram.

Chapter 8 288
Chapter 9 Transformations
Page 461 Getting Started 9. sin A ¬2
3

1. y A ¬sin1 2
3
 
A ¬41.8°
B( 1, 3) A(1, 3) The measure of angle A is approximately 41.8.
10. sin A ¬4
5

O x A ¬sin14
5

A ¬53.1°
2. y
The measure of angle A is approximately 53.1.
C( 3, 2) 9
11. cos A ¬12
O x A ¬cos1 9
12 
A ¬41.4°
D( 3, 2) The measure of angle A is approximately 41.4.
15
3. y 12. cos A ¬17

A ¬cos117 
15
E( 2, 1) A ¬28.1°
The measure of angle A is approximately 28.1.
O x

F( 1, 2)
13. 
0 1
1 1
  
5
5 1
4

0(4)  1(1)
0(5)
1(5)  (1)(5) 1(4)  (1)(1)

 1(5)

5 1

5
4. y
10
G(2, 5)
1 0
14. 
1 1 2 3
0 2


1(0)  0(2) 1(2)  0(3)



x 1(0)  1(2) 1(2)  1(3)
O

2 1
0 2
H(5, 2)

15. 
1 0 2 5 1
5. J( 7, 10) y 0 1 3 4 5

8
K( 6, 7)

1(3)0(2) 1(4)0(5) 1(5)0(1)
0(3)1(2) 0(4)1(5) 0(5)1(1)
4
2 5 1

12 8 4 O x 3 4 5

6. y
16. 10 10 13 3 3 2
1 2 1 
1( 1) 0(3) 1( 3) 0( 1) 1( 3) 0( 2) 1(2) 0(1)
¬
0( 1) ( 1)(3) 0( 3) ( 1)( 1) 0( 3) ( 1)( 2) 0(2) ( 1)(1)

O x
3 3 2
L(3, 2) 31 1 2 1 
M(6, 4)
Page 462 Geometry Activity: Transformations
7. tan A ¬3

4
1. Rotation; the figure has been turned around a
A ¬tan1 34 point.
A ¬36.9° 2. Dilation; the figure has been enlarged.
The measure of angle A is approximately 36.9. 3. Reflection or rotation; the figure has either been
8. tan A ¬5

8
flipped over a line or turned around a point.
A ¬tan15
8
 4. Translation; the figure has been slid down and to
the left.
A ¬32.0°
The measure of angle A is approximately 32.0. 5. Dilation; the figure has been reduced.
6. Reflection; the figure has been flipped over a line.
7. Translation; the figure has been slid down and to
the left.

289 Chapter 9
8. Reflection or rotation; the figure has either been 8. y A
flipped over a line or turned around a point. B
9. Reflection; the figure has been flipped over a line.
10. Reflection; the figure has been flipped over a line.
11. Rotation, reflection, and translation result in an O x
image that is congruent to its preimage. They are
isometries.
B
A
9. A y A
9-1 Reflections

Page 467 Check for Understanding


1. Sample Answer: The centroid of an equilateral O x
triangle is not a point of symmetry. B
B
2. Sample answer: W(3, 1), X(2, 3), Y(3, 3) and
C C
Z(3, 1) with reflected image W(1, 3), X(3, 2),
Y(3, 3), Z(1, 3)
y Z
10. y
X Y
D
W Z E
F
O x
O x
X F
W E
D
3. Angle measure, betweenness of points,
collinearity, and distance are four properties that 11. y I
are preserved in reflections.
4.
m J H
G
H I
5. 2; The figure has 2 lines of symmetry, each
passing through opposite vertices (at the tips). O J x
Yes; the figure has point symmetry with respect G
to its center.
12. 1; The butterfly has 1 line of symmetry through
6. 3; The figure has 3 lines of symmetry, each its head and tail.
passing through the center, a vertex, and the No; the butterfly has no common point of
midpoint of the side opposite the vertex. reflection and, thus, no point symmetry.
No; the figure has no common point of reflection
13. 4; The leaf has 4 lines of symmetry: two passing
and, thus, no point symmetry.
between the leaves and two passing through their
7. 6; The figure has 3 lines of symmetry at the tips centers.
of the 5-sided figures and 3 lines of symmetry Yes; the figure has point symmetry with respect
between the 5-sided figures. The figure has point to its center.
symmetry with respect to its center.

14. Looking at the tiger directly from the front, its


face has one line of symmetry that goes down the
center of the face vertically.
No, the tiger face has no common point of
reflection and, thus, no point symmetry.

Chapter 9 290
Pages 467–469 Practice and Apply 25. Draw perpendiculars from P, R, S, T, and U to
15. X is on line , so it is its own reflection. Y is the line . Locate P, R, S, T, and U so that line  is
image of W under a reflection in line . So, X Y
 is the perpendicular bisector of P P
, R
R
, S
S
, T
T
,
the image of W X under a reflection in line . and UU. P, R, S, T, and U are the respective
images of P, R, S, T, and U. Connect vertices P,
16. Z is on line , so it is its own reflection. Y is the
R, S, T, and U.
image of W under a reflection in line . So, Y Z
 is
the image of W Z under a reflection in line . 
17. X and Z are on line , so they are their own S S
reflections. W is the image of Y under a reflection R R
in line . So, XZW is the image of XZY under a T T
reflection in line .
P U U P
18. T is on line m, so it is its own reflection.
19. U is on line m, so it is its own reflection. V is the 26. Since W is on line , W is its own reflection. Draw
image of Y under a reflection in line m. So, V U is segments perpendicular to line  from X, Y, and Z.
the image of U Y under a reflection in line m. Locate X, Y, and Z so that  is the
20. V is the image of Y, Y is the image of V, and X is perpendicular bisector of XX, Y
Y
, and Z
Z
. X,
the image of W under a reflection in line m. Y, and Z are the respective images of X, Y, and
So, VYX is the image of YVW under a reflection Z. Connect vertices W, X, Y, and Z.
in line m.
X Y
21. T is the image of U under a reflection in point Z.
22. U is the image of T, V is the image of X, and Z is
its own image under a reflection in point Z. So,
UVZ is the image of TXZ under a reflection in W
point Z. Z

23. W is the image of Y, T is the image of U, and Z is X


its own image under a reflection in point Z. So, Z
WTZ is the image of YUZ under a reflection in
point Z. Y
24. Draw perpendiculars from A, B, C, D, E, and F to
27. Plot rectangle MNPQ. Since Q Q passes through
line . Locate A, B, C, D, E, and F so that line
the origin, use the horizontal and vertical
 is the perpendicular bisector of A A
, B
B, C
C,
distances from Q to the origin to find Q. From Q

DD, E
E, and F
F. Points A, B, C, D, E, and F
to the origin is 2 units to the right and 3 units
are the respective images of A, B, C, D, E, and F.
down. Q is located by repeating that pattern from
Connect vertices A, B, C, D, E, and F.
the origin. Two units to the right and 3 units
C D down yields Q(2, 3).
Q(2, 3) → Q(2, 3) N(2, 3) → N(2, 3)
P(2, 3) → P(2, 3) M(2, 3) → M(2, 3)
E
B y
QN P M
A F

O x
A F

PM Q N
B E

C D

291 Chapter 9
28. Plot rectangle GHIJ. Since G G
 passes through y yx
the origin, use the horizontal and vertical B
C
distances from G to the origin to find G. From G
to the origin is 2 units up and 2 units to the right.
G is located by repeating that pattern from the
origin. Two units up and 2 units to the right O B
yields G(2, 2). x
G(2, 2) → G(2, 2) I(3, 3) → I(3, 3) D
D C
H(2, 0) → H(2, 0) J(2, 4) → J(2, 4)
J y
32. Plot KLM. Use the vertical grid lines to find a
I
corresponding point for each vertex so that the
G
line y  2 is equidistant from each vertex and its
H image. For K(4, 0), the vertical distance to the line
O H x y  2 is 2. To plot K, move up 2 units on the
vertical gridline so the image of K is K(4, 4).
G K(4, 0) → K(4, 4)
I L(2, 4) → L(2, 0)
J M(2, 1) → M(2, 3)
29. Plot square QRST. Use the vertical grid lines to y
find a corresponding point for each vertex so that L K
the x-axis is equidistant from each vertex and its
image. M
Q(1, 4) → Q(1, 4) S(3, 2) → S(3, 2) M
R(2, 5) → R(2, 5) T(0, 1) → T(0, 1) O K x
L
y R
Q

S 33. Plot FGH. To find FGH, use the horizontal


T grid lines to find a corresponding point for each
OT x vertex so that the y-axis is equidistant from each
S vertex and its image.
F(1, 4) → F(1, 4) H(3, 2) → H(3, 2)
Q G(4, 2) → G(4, 2)
y
R F F
30. Plot trapezoid DEFG. Use the horizontal grid
lines to find a corresponding point for each vertex
so that the y-axis is equidistant from each vertex G G
and its image. O x
E(2, 4) → E(2, 4) F(2, 1) → F(2, 1)
D(4, 0) → D(4, 0) G(4, 3) → G(4, 3) H H
E y E
34. Plot XYZ. Use the horizontal grid lines to find
a corresponding point for each vertex so that the
D D line x  1 is equidistant from each vertex and
O x its preimage. For X(1, 4), the horizontal distance
F F to the line x  1 is 2. To plot X, move 2 units to
the left of the line x  1 so the preimage of X is
G G (3, 4).
X(1, 4) → X(3, 4)
31. Plot BCD and the line y  x. The slope of y  x Y(2, 2) → Y(4, 2)
B
is 1. B is perpendicular to y  x, so its slope is Z(2, 3) → Z(0, 3)
1. From B to the line y  x, move up 2 units and Notice that Z is to the left of x  1 so its
to the left 3 units. From the line y  x, move up preimage is to the right of the line x  1.
2 units and to the left 3 units to B(0, 5)
B(5, 0) → B(0, 5) D(2, 1) → D(1, 2)
C(2, 4) → C(4, 2)

Chapter 9 292
y 41. Undo the reflections in turn. A(4, 7), B(10, 3),
X X and C(6, 8). Reflection in the origin: multiply
both coordinates by 1: (a, b) → (a, b):
A(4, 7), B(10, 3), and C(6, 8). Reflection in
Y Y the y-axis: multiply the x-coordinate by 1:
(a, b) → (a, b): A(4, 7), B(10, 3), and
O x
C(6, 8).
Z Z Reflection in the x-axis: multiply the y-coordinate
by 1 (a, b) → (a, b): A(4, 7), B(10, 3), and
C(6, 8).
35. 2; The figure has two lines of symmetry, each Undoing the transformations results in the
passing through opposite vertices. triangle ABC.
Yes; the figure has point symmetry with respect
42.
to its center.
36. 8; The figure has eight lines of symmetry, each cue ball
passing through opposite vertices.
Yes; the figure has point symmetry with respect eight ball
to its center.
37. 1; The figure has one line of symmetry, which reflected
passes horizontally through the center. pocket
No; the figure has no common point of reflection
and, thus, no point symmetry.
38. The preimage and final image have the same
shape and the same orientation.
m n 43. Consider point (a, b). Upon reflection in the
origin, its image is (a, b). Upon reflection in
the x-axis and then the y-axis, its image is
(a, b) → (a, b). The images are the same.
44. The diamond has numerous lines of symmetry,
39. The preimage and final image have the same including vertical and horizontal lines of
shape, but the final image is turned or rotated symmetry. It has a point of symmetry at the
with respect to the preimage. center.
n 45. The diamond has a vertical line of symmetry,
m
passing through its center.
46. The diamond has a vertical line of symmetry,
passing through its center.
47. The diamond has a vertical line of symmetry and
a horizontal line of symmetry. It has a point of
40. Apply the reflections in turn. D(1, 4), E(2, 8), symmetry at the center.
F(6, 5), and G(3, 1). Reflection in the x-axis: 48. Sample answer: Reflections of the surrounding
Multiply the y-coordinates by 1: (a, b) → vistas can be seen in bodies of water. Answers
(a, b): D(1, 4), E(2, 8), F(6, 5), and should include the following.
G(3, 1). Reflection in the line y  x: Interchange • Three examples of line symmetry in nature are
the x- and y- coordinates (a, b) → (b, a): the water’s edge in a lake, the line through the
D(4, 1), E(8, 2), F(5, 6), and G(1, 3). middle of a pin oak leaf, and the lines of a four
y E yx leaf clover.
8
F • Each point above the water has a
D F corresponding point in the image in the lake.
E G The distance of a point above the water appears
G x
the same as the distance of the image below the
8 O G 8 water.
D
D
F
8
E

293 Chapter 9
49. D; x-axis reflection: (2, 5) → (2, 5) and 53. BE is the measure of the median of the trapezoid.
y-axis reflection: (2, 5) → (2, 5) AF  CD
BE ¬ 2
or reflection in the origin: (2, 5) → (2, 5) 32 48
¬
50. B; a  c ¬2a  b  2c 2
25  45 ¬2(25)  18  2(45) ¬40
25  45 ¬158 AF 
54. BE ¬ CD
2
32 48
¬ 2
Page 469 Maintain Your Skills ¬40
51. Given: Quadrilateral LMNP BE CD
XY ¬ 2
X, Y, Z, and W are midpoints of 40 48
¬
their respective sides. 2
W
Prove: Y and X Z bisect each other. ¬44
AF CD
y 55. BE ¬ 2
M(2d, 2e) 32 48
Y ¬ 2
N(2a, 2c)
X ¬40
Z AF BE
WZ ¬ 2
32  40
L(0, 0) W P(2b, 0) x ¬ 2
Proof: ¬36
 is 2d 
, 2e  56. mF  mG  mH ¬180
2 or
2a  2c
Midpoint Y of M N 2 mF  53  71 ¬180
(d  a, e  c). mF ¬56
2a 
Midpoint Z of N  is 
P 2 , 2c 2
2b   0
 or (a  b, c). Use the Law of Sines to write a proportion
0 2b 0  0
 is 
L
Midpoint W of P 2 , 2 or (b, 0).
to find g.
sinF sinG
0  2d 0  2e f ¬g
 is 2, 
M
Midpoint X of L 2 or (d, e).
dab ec0 ¬fsinG
Midpoint of W is 2, 
Y 2 or
g ¬
sinF
abdce
2, 2.
48
g ¬ sin
53°
sin 56°
d  ab 
 is 
Z
Midpoint of X 2 , e
2 or
c g ¬46.2
a  bd  Use the Law of Sines again to find the measure of
 2 , c
2. X
e
Z
 and W
Y
 bisect each other. the third side.
52. Given: Isosceles trapezoid sinF sinH
f ¬h
D
A B C y fsinH
H, J, K, and G D(b, c) J C(a b, c) h ¬
sinF
are midpoints 48sin71°
of their
H K h ¬ sin56°
respective h ¬54.7
A(0, 0) G B(a, 0) x
sides. Therefore, mF  56, g  46.2, and h  54.7.
Prove: GHJK is a rhombus. 57. mF  mG  mH ¬180
Proof: 59  45  mH ¬180

 a 2 2
b
 
2  2ab  a
2  c2
mH ¬76
HJ  b2  2  2  c ¬
c  2 ;
Use the Law of Sines to write a proportion
to find f.
GK   2
 2c
2a  b 2 2
2  a   0 sinG sinF
g ¬f

b
 
2  2ab  a2
 c2 gsinF
¬  2 ; f ¬
sinG

 b2  a2
   2  0

b 
 2
ab  a 21sin
59°
2 2 2 c
2 2
f ¬
HG  c
 ¬  2 ; sin45°
f ¬25.5
KJ   2
 2  2  c
2a   b a 2 c 2


b
 
2  2ab  a2
 c2
¬  2 ;

HJ  GK  HG  KJ, so HJ
G
K
H
G
K
J

and GHJK is a rhombus.

Chapter 9 294
Use the Law of Sines again to find the measure of
the third side. 9-2 Translations
G
sin sinH
g ¬ h
 
gsinH Page 472 Check for Understanding
h ¬
sinG 1. Sample answer: A(3, 5) and B(4, 7); start at 3,
21
h ¬ sin
76° count to the left to 4, which is 7 units to the left
sin45° or 7. Then count up 2 units from 5 to 7 or 2.
h ¬28.8 The translation from A to B is (x, y) →
Therefore, mH  76, f  25.5, and h  28.8. (x  7, y  2).
58. We know two sides and the measure of the angle 2. The properties that are preserved include
opposite one of the sides. Use the Law of Sines to betweenness of points, collinearity, and angle and
find the measure of the second angle. distance measure. Since translations are
composites of two reflections, all translations are
F
sin sinH
f ¬ h
  isometries. Thus, all properties preserved by
fsinH reflections are preserved by translations.
sinF ¬
h 3. Allie; counting from the point (2, 1) to (1, 1) is
fsinH
F ¬sin1 right 3 and down 2 to the image. The reflections
h
would be too far to the right. The image would be
F ¬sin  13.n261° 
1 14.5si
 reversed as well.
F ¬74° 4. Yes; GHI is a translation of ABC. DEF is the
So, mF  74. image of ABC when ABC is reflected in line
Use the Angle Sum Theorem to find the measure m, and GHI is the image of DEF when DEF
of angle G. is reflected in line n.
mF  mG  mH ¬180 5. No; quadrilateral WXYZ is oriented differently
74  mG  61 ¬180 than quadrilateral NPQR.
mG ¬45 6. This translation moved every point of the
Use the Law of Sines and a proportion to find g. preimage 1 unit right and 3 units up.
H
sin sinG D(3, 4) → D(3  1, 4  3) or D(2, 1)
 h ¬g
hsin G E(4, 2) → E(4  1, 2  3) or E(5, 5)
g ¬ 
sinH Graph D and E and connect. Graph D and E and
13.2sin 45° connect.
g ¬ 
sin61°
y E
g ¬10.7
Therefore, mF  74, mG  45, and g  10.7.
d ¬
(x
 E
59. 2
x1)2 
(y2 
y1)2
EF ¬

(2  3
)2  [0
 (1
)]2
EF ¬
(1)
 2
 12 O x
D
EF ¬
2

60. d ¬
(x
 2
x1)2 
(y2 
y1)2
D
FG ¬

(3  2
)2  (3
 0)2
7. This translation moved every point of the
FG ¬
1
 
2  32
preimage 3 units to the left and 4 units down.
FG ¬
10
 K(5, 2) → K(5  3, 2  4) or K(2, 6)
61. d ¬
(x
 2
x1)2 
(y2 
y1)2 L(3, 1) → L(3  3, 1  4) or L(6, 5)
GH ¬

(5  3)
2  (4
 3)2 M(0, 5) → M(0  3, 5  4) or M(3, 1)
GH ¬

22  12 Graph K, L, and M and connect to form KLM.
GH ¬
5
 Graph K, L, and M to form KLM.
y
62. d ¬
(x
 2
x1)2 
(y2 
y1)2 8
HE ¬

(3  5)
2  (
1  4)2 M
4
HE ¬

(2)2 
 (5
)2 M
O
HE ¬
29
 8 4 4 8x
L K
4
L K
8

295 Chapter 9
8. 1 → 2  (x, y  3) 18. This translation moved every point of the
2 → 3  (x  4, y) preimage 2 units to the right and 1 unit down.
3 → 4  (x  4, y) E(0, 4) → E(0  2, 4  1) or E(2, 5)
F(4, 4) → F(4  2, 4  1) or F(2, 5)
G(0, 2) → G(0  2, 2  1) or G(2, 1)
Pages 472–475 Practice and Apply Plot the vertices of the preimage and the image
9. Yes; it is one reflection after another with respect and connect the respective vertices to form the
to the two parallel lines. preimage and the image.
10. No; it is a reflection followed by a translation. y
11. No; it is a reflection followed by a rotation. G
12. No; it is a reflection followed by a translation. G
O x
13. Yes; it is one reflection after another with respect
to the two parallel lines.
14. No; it is a reflection followed by a translation. F E
15. For each endpoint of P Q
 move left 3 units and up F
4 units to find the image. Connect P and Q. E
y
8 19. This translation moved every point of the
Q preimage 5 units to the left and 3 units up.
4 Q P(1, 4) → P(1  5, 4  3) or P(4, 7)
P Q(1, 4) → Q(1  5, 4  3) or Q(6, 7)
4 O 4 8 x
R(2, 4) → R(2  5, 4  3) or R(7, 1)
4 S(2, 4) → S(2  5, 4  3) or S(3, 1)
P
Plot the vertices of the preimage and the image
8
and connect the respective vertices to form the
B
16. For each endpoint of A move right 4 units and preimage and the image.
down 2 units. Connect A and B. Q P y
y
8
A
Q P
4 A

8 4 O 4 8x
4 O x
B B R S
8

17. This translation moved every point of the R S


preimage 1 unit to the right and 4 units up.
20. This translation moved every point of the
M(2, 2) → M(2  1, 2  4) or M(1, 2) preimage 4 units to the right and 3 units down.
J(5, 2) → J(5  1, 2  4) or J(4, 6) V(3, 0) → V(3  4, 0  3) or V(1, 3)
P(0, 4) → P(0  1, 4  4) or P(1, 8) W(3, 2) → W(3  4, 2  3) or W(1, 1)
Plot the vertices of the preimage and the image X(2, 3) → X(2  4, 3  3) or X(2, 0)
and connect the respective vertices to form the Y(0, 2) → Y(0  4, 2  3) or Y( 4, 1)
preimage and the image. Z(1, 0) → Z(1  4, 0  3) or Z(3, 3)
y Plot the vertices of the preimage and the image
P and connect the respective vertices to form the
J preimage and the image.
y
X
P W Y
J
M O X
V Z x
O x
W Y
M V Z

Chapter 9 296
21. As a translation, the bishop moves left 3 squares y
and down 7 squares. 8
W
22. Sample answers: pawn: up two squares; rook: left
4 T
four squares; knight: down two squares, right y=1 Y
1 square; bishop: up three squares, right three Y
squares; queen: up five squares; king: right 8 4 T 4 8 x
1 square W
4
y = 4
23. Four triangle lengths is equivalent to a T
translation of 48 in. right. 8
Y
24. Two triangle lengths left and four triangle lengths
up and left (60° angle above horizontal) form one Examine: Notice that the image of W(7, 4) is
leg and the hypotenuse of a right triangle. Use W(7, 6) or point D. The line y  1 is equidistant
the Pythagorean Theorem to find the other leg from these points. The image of Y(9, 0) is Y(9, 2)
(direction up). or G. Again, the line y  1 is equidistant from
c2 ¬a2  b2 these points. So, two possible parallel lines are
(4  12)2 ¬a2  (2  12)2 y  4 and y  1.
2304 ¬a2  576 27. This translation moved every point of the
1728 ¬a2 preimage 2 units to the right and 4 units down.
24
 3 ¬a
P(3, 2) → P(3  2, 2  4) or P(1, 6)
41.6 ¬a
Q(1, 4) → Q(1  2, 4  4) or Q(1, 0)
The translation is 24
 3  41.6 in. up and 24 in. R(2, 2) → R(2  2, 2  4) or R(4, 6)
left.
Q y
25. The red line represents a translation of six
triangle lengths right and four triangle heights
down. Use the Pythagorean Theorem to find the
triangle height. Q
c2 ¬a2  b2 O x
2
122 ¬a2  2 
12 R
P
144 ¬a2  36
108 ¬a2
6
3 ¬a
Six lengths: 6(12)  72 P R
Four heights: 4(6
3 )  24
3   41.6
The translation is 72 in. right and 28. First reflect RST in the line y  2. Use the
24
 3  41.6 in. down. vertical grid lines to find images of the vertices
that are the same distance from y  2 as the
26. Sample answer:
preimage.
Explore: We are looking for two parallel lines
R(4, 1) → R(4, 5) R 
such that TWY is reflected over each line to
S(1, 3) → S(1, 1) S, T y
result in the image BDG.
T(1, 1) → T(1, 3)
Plan: Once we choose any line to be the first
parallel line, there is only one possible line for the
second parallel line. x
Solve: Choose y  4 for the first parallel line. T, S
Use the vertical grid lines to determine the R
vertices of the image that are the same distance
from y  4 as the preimage. Since the
y-coordinate of W is 4, W is its own image.
T(3, 7) → T(3, 1)
Next reflect RST S y
Y(9, 8) → Y(9, 0)
over the line y  2
Now we need to find a line such that a reflection to get RST.
over that line has an image of BDG. We are R(4, 5) → R(4, 9)
looking for a line that is equidistant between S(1, 1) → S(1, 5) T
TWY and BDG. We are only looking at the T(1, 3) → T(1, 7) O x
y-coordinates. R
T(3, 1) → B(3, 3). The distance between the
y-coordinates is 1  3  4 so the parallel line S
that lies halfway between is 2 units above the line
y  1 or y  1.
T
R

297 Chapter 9
80
%
29. To find the image, “undo” the translation. To undo 33. 9  8.89% per person
(x  4, y  5), add 4 to the x-coordinate and 78
%
8  9.75% per person

subtract 5 from the y-coordinate.
A(8, 5) → A(8  4, 5  5) or A(4, 0) 70
%
7  10% per person

B(2, 7) → B(2  4, 7  5) or B(6, 2) 62
%
7  8.86% per person

C(3, 1) → C(3  4, 1  5) or C(7, 4)
y No; the percent per figure is different in each
category.
8 B
34. Sample answer: Every time a band member takes
A a step, he or she moves a fixed amount in a
4 B certain direction. This is a translation. Answers
A should include the following.
8 4 O C 8x • When a band member takes a step forward,
4 backward, left, right, or on a diagonal, this is a
C translation.
• To move in a rectangular pattern, the band
30. In order to find the coordinates of H and N we member starting at (0, 0) could move to (0, 5).
find the transformation from vertex F to vertex M. Moving from (0, 5) to (4, 5), from (4, 5) to (4, 0)
F(3, 9) → M(4, 2) and from (4, 0) back to the origin, the band
The translation in the x-direction is 1 unit to member would have completed the rectangle.
the right. The translation in the y-direction is
35. Translations and reflections preserve the
7 units down.
congruences of lengths and angles. The
G(1, 4) → N(x1, y1)
composition of the two transformations will
→ N(1  1, 4  7) or N (0, 3)
preserve both congruences. Therefore, a glide
H(x2, y2) → P(6, 3)
reflection is an isometry.
Undo the transformation by subtracting one and
adding 7. So the coordinates of H are 36. First, find the vertices of the image after the
H(6  1, 3  7) or H(5, 4). translation (x, y) → (x, y  2).
D(4, 3) → (4, 3  2) or (4, 1)
y F E(2, 2) → (2, 2  2) or (2, 4)
F(0, 1) → (0, 1  2) or (0, 1)
Now reflect the image in the y-axis. Use the
formula (a, b) → (a, b).
G (4, 1) → D(4, 1)
H (2, 4) → E(2, 4)
M (0, 1) → F(0, 1)
y
O x D

D F
N P F
x
The coordinate form of the translation
is (x, y) → (x  1, y  7). E
31. The categories that show a boy-girl-boy unit
translated within the bar are “more brains” and E
“more free time”.
32. “More friends” and “more athletic ability” are the
categories that show a boy-girl-boy unit reflected
(about a vertical line) within the bar.

Chapter 9 298
37. First, find the vertices of the image after the 41. Draw perpendiculars from P, R, S, T, and U
translation (x, y) → (x  3, y). to line m. Locate P, R, S, T, and U so that line
A(3, 2) → (3  3, 2) or (0, 2) m is the perpendicular bisector of P P
, R
R
, S
S
,
B(1, 3) → (1  3, 3) or (2, 3) T
T , and U
U. Points P, R, S, T, and U are the
C(2, 1) → (2  3, 1) or (5, 1) respective images of P, R, S, T, and U. Connect
Now reflect the image over the line y  1. vertices P, R, S, T, and U.
Use the horizontal grid lines to find the vertices S
of ABC such that each vertex of the image is
the same distance from the line y  1 as its T
preimage.
U
(0, 2) → A(0, 4)
(2, 3) → B(2, 5) R
(5, 1) → C(5, 3) P
m
y B P
A R
C
y=1 U
T
C x

A S
B
42. Since D is on line m, D is its own reflection.
Draw perpendiculars from E, F, G, H, and I
38. C; X(5, 4) → X(3, 1)  X(5  2, 4  3) to line m. Locate E, F, G, H, and I so that line
So, (x, y) → (x  2, y  3). m is the perpendicular bisector of E E
, F
F
, G
G,
Y(3  2, 1  3) ¬Y(1, 4) H
H , and II. Points E, F, G, H, and I are the
Z(0  2, 2  3) ¬Z(2, 1) respective images of E, F, G, H, and I. Connect
y y vertices D, E, F, G, H, and I.
39. A; m ¬
2
1
x2  x1
H
1  5
¬ 
2  (2)
6 G
¬4 I m
F
¬3
2


E I

Page 475 Maintain Your Skills D


40. Draw perpendiculars from A, B, C, D, and E G
to line m. Locate A, B, C, D, and E so that line H
m is the perpendicular bisector of A A, B
B
, C
C
, E
D
D , and E
E. Points A, B, C, D, and E are the F
respective images of A, B, C, D, and E. Connect 43. The top and bottom segments of the trapezoid are
vertices A, B, C, D, and E. parallel, so the y-coordinate of Q is the same as
B that of R, c.
A The x-coordinate of Q plus the x-coordinate of R
equals the x-coordinate of S.
A C
E x  b ¬a
x ¬a  b
E D
B So, Q(?, ?)  Q(a  b, c).
T is at the origin, so T(?, ?)  T(0, 0).
D 44. Opposite sides of a parallelogram are congruent
C
m
and parallel. So, the y-coordinate of B is b and
that of D is 0.
The x-coordinate of B plus the x-coordinate of D
equals the x-coordinate of C.
x  d ¬a  d
x ¬a
So, B(?, ?)  B(a, b) and D(d, ?)  D(d, 0).

299 Chapter 9
45. Find the opposite side of the triangle, y. x  4 ¬x  2
y 6 ¬2x
tan 45° ¬
6 3 ¬x
6tan 45° ¬y
y ¬(3)  4
6 ¬y
¬3  4
Six yards is 18 feet. So, the height of the tree is
¬1
5  18 or 23 ft.
46. A certain shopper is not greeted when he walks The point of intersection is (3, 1).
through the door. Find the distance between the two points of
47. You did not fill out an application. intersection, (0, 4) and (3, 1).
48. y6 d ¬
(x2  
x1)2 (y2 y1)2
49. The two lines are not parallel. ¬
 (3  0) 
2 [1  (4)]2
50. x  2 and x  5 are vertical lines. The distance ¬
 99
between them can be measured along any ¬3
 2
horizontal segment connecting them. The distance The distance between the lines is 3
 2.
is 5  (2)  7. 54.
51. y  6 and y  1 are horizontal lines. The 30
distance between them can be measured along
any vertical segment between them. The distance 55.
is 6  (1)  5.
45
52. Let line  be y  2x  3 and line m be y  2x  7.
The slope of the parallel lines is 2. The slope of a 56.
line perpendicular to the parallel lines is the
1
opposite reciprocal of 2, or 2. Use the y-intercept
52
of line m, (0, 7), as one of the endpoints of the
perpendicular segment, P. Find P. 57.
P: y  y1 ¬m(x  x1)
1
y  (7) ¬2(x  0) 60
y  7 ¬1
2x
1 58.
y ¬2x  7
105
Solve the system of the equations of lines P and 
to find where they intersect.
59.
1
2 x  7 ¬2x  3
 150
x  14 ¬4x  6
5x ¬20
x ¬4
y ¬1(4)  7
2
¬2  7 9-3 Rotations
¬5
The point of intersection is (4, 5).
Page 477 Geometry Software Investigation:
Find the distance between the two points of
intersection, (0, 7) and (4, 5).
Reflections in Intersecting Lines
1. See students’ figures.
d ¬
 (x2  
x1)2 
(y2 y1)2
2. The transformation is a rotation about P.

 (4  0)2 [5 (7)]2
3. See students’ work.

 16  4 4. See students’ work. The angle measure should be
¬2
 5 twice the measure of the acute angle formed by
The distance between the lines is 2
5 . the intersecting lines.
53. Let line  be y  x  2 and line m be y  x  4. 5. See students’ work. The angle measures should be
The slope of the parallel lines is 1. The slope of a the same as mAPA in Exercise 4.
line perpendicular to the parallel lines is the
6. Sample answer: The measure of the angle of
opposite reciprocal of 1, or 1. Use the y-intercept
rotation is twice the measure of the acute angle
of line m, (0, 4), as one of the endpoints of the
formed by the intersecting lines.
perpendicular segment, P. Find P.
P: y  y1 ¬m(x  x1)
y  (4) ¬1(x  0)
y  4 ¬x
y ¬x  4
Solve the system of the equations of lines P and 
to find where they intersect.

Chapter 9 300
Pages 478–479 Check for Understanding 5. First reflect parallelogram ABCD in line . Next,
1. Sample answer: reflect the image in line m. Parallelogram
y ABCD is the image of parallelogram ABCD
C under reflections in lines  and m.
B m 
A
O 90 x
A C B
A

B D C
B
ABC has vertices A(1, 0), B(4, 2), and C(2, 3).
A
ABC has vertices A(0, 1), B(2, 4), and C
C(3, 2). D
For a clockwise rotation of 90 degrees about the
origin, (x, y) → (y, x).
B C
B y
C
B A D
C 90
A 6. First reflect quadrilateral DEFG in line . Since E
O A x and F are on , each point is its own image. Next,
reflect the image in line m. Quadrilateral
DEF G is the image of quadrilateral DEFG
under reflections in lines  and m.
ABC has vertices A(1, 0), B(4, 2), and C(2, 3).
 D
ABC has vertices A(0, 1), B(2, 4), and
E
C(3, 2). For a counterclockwise rotation of 90
degrees about the origin, (x, y) → (y, x).
G m
2. A rotation image can be found by reflecting the
image in a line, then reflecting that image in a D
second of two intersecting lines. The second F
method is to rotate each point of the given figure G
using the angle of rotation twice the angle formed G
between the intersecting lines. Use the D
intersection point of the two lines as the point of
rotation. F

3. Both translations and rotations are made up of E


two reflections. The difference is that translations
reflect across parallel lines and rotations reflect
across intersecting lines.
7. First graph XY. Draw a segment from the origin
4. Draw a segment from G to B. Use a protractor to O to point X. Use a protractor to measure a 45°
measure a 60° angle counterclockwise with G B as angle clockwise with OX as one side. Draw OR.
. Use a compass to copy G
one side. Draw GX B . Name the
Use a compass to copy O X onto OR
. Name the segment G
onto GX B
. segment OX . Repeat with point Y. X
Y
 is the
Repeat with points C and D. BCD is the image of XY under a 45° clockwise rotation about
image of BCD under a 60° counterclockwise the origin.
rotation about point G.
y
C D X
X ( 5 , 8 )

B
G
C 60 Y(0, 3)
Y
45
O x

B

301 Chapter 9
8. First graph PQR. Draw a segment from the
origin O to point P. Use a protractor to measure a E
90° angle counterclockwise with OP as one side. B R F
Draw OX. Use a compass to copy OP onto 
OX. 110
Name the segment O P. Repeat with points Q and
C F D
R. PQR is the image of PQR under a 90°
counterclockwise rotation about the origin. B

P(1, 8) y C
D E

4
Q (2, 4) 13. Draw a segment from Q to M. Use a protractor to
measure a 180° angle counterclockwise with QM
P (8, 1) 90 as one side. Draw 
QX. Use a compass to copy QM

12 4 O x
4 8
onto 
QX. Name the segment Q M. Repeat with
Q (4, 2)
4 points N and P. MNP is the image of MNP
R (7, 4)
R(4, 7) under a 180° counterclockwise rotation about
8 point Q.
M
9. The regular hexagon has rotational symmetry of N P
order 6 because there are 6 rotations less than Q
360° (including 0 degrees) that produce an image
indistinguishable from the original. 180˚
360°
magnitude ¬ ord
er
P N
M
360°
¬ 6 14. First graph XYZ and point P. Draw a segment
¬60°
from point P to point X. Use a protractor to
The magnitude of the symmetry is 60°.
measure a 90° angle counterclockwise with P X
 as
10. The regular octagon has rotational symmetry of one side. Draw 
PR. Use a compass to copy P X
order 8 because there are 8 rotations less than . Name the segment P
onto PR X
. Repeat with
360° (including 0 degrees) that produce an image points Y and Z. XYZ is the image of XYZ
indistinguishable from the original. under a 90° counterclockwise rotation about
360°
magnitude ¬ order point P.
36
0° y Z
¬8 Y
¬45°
The magnitude of the symmetry is 45°. Z
11. The left, center, and right fans have rotational P X Y
symmetry of orders 5, 4, and 3, respectively, 90
because there are 5, 4, and 3 rotations less than o x
360° (including 0 degrees) that produce images X
indistinguishable from the originals.
The magnitude of the symmetry is given by 15. First graph RST and point P. Draw a segment
360°
order . So, the magnitudes of the symmetries from point P to point R. Use a protractor to
for the left, center, and right fans are 72°, 90°, and measure a 90° angle clockwise with PR as one
120°, respectively. . Use a compass to copy P
side. Draw PX R
 onto
. Name the segment P
PX R
. Repeat with points
S and T. RST is the image of RST under a
Pages 479–481 Practice and Apply 90° clockwise rotation about point P.
12. Draw a segment from R to B. Use a protractor to y T
measure a 110° angle counterclockwise with RB
as one side. Draw  B
RX. Use a compass to copy R  R 90
onto 
RX. Name the segment R B
. Repeat with
points C, D, E, and F. Pentagon BCDEF is the T
image of pentagon BCDEF under a 110° R S
O x
counterclockwise rotation about point R.
S

Chapter 9 302
16. The Ferris wheel has rotational symmetry of 23. M y L
order 20 because there are 20 rotations less than
360° (including 0 degrees) that produce an image M
indistinguishable from the original.
360°
magnitude ¬
order O K x
36

¬20 L
¬18°
The magnitude of the symmetry is 18°.
K
17. From Exercise 16, the magnitude of the
symmetry is 18°. Seat 1 is moved 4 positions, Reflection in line y  x: K(5, 0) → K(0, 5)
or 4(18°)  72°. L(2, 4) → L(4, 2)
18. From Exercise 16, the magnitude of the symmetry M(2, 4) → M(4, 2)
is 18°. Divide 144° by the magnitude of the Reflection in x-axis: K(0, 5) → K(0, 5)
rotational symmetry. L(4, 2) → L(4, 2)
M(4, 2) → M(4, 2)
144°
18°  8 positions
 The angle of rotation is 90° clockwise.
Seat 1 is moved 8 positions, or to the original 24. y
Z Y
position of seat 9.
19. m
Y t
X
Z X' O X x
X Z'
Y'
Y Z
Reflection in line y  x: X(5, 0) → X(0, 5)
20. S' P' Y(3, 4) → Y(4, 3)
m t Z(3, 4) → Z(4, 3)
R' Q' R S Reflection in line y  x: X(0, 5) → X(5, 0)
Y(4, 3) → Y(3, 4)
Q P Z(4, 3) → Z(3, 4)
The angle of rotation is 180°.
25. y
21. J K t ( 3 , 1)
N L
M
30˚
x
(2, 0)
M' N'
L' J'
K'
m
22. y x  2cos 30°
U 
3
V y  2sin 30°
1
T , 1).
The coordinates of the image are (
3
O T x
V

U

Reflection in y-axis: T(4, 0) → T(4, 0)


U(2, 3) → U(2, 3)
V(1, 2) → V(1, 2)
Reflection in x-axis: T(4, 0) → T(4, 0)
U(2, 3) → U(2, 3)
V(1, 2) → V(1, 2)
The angle of rotation is 180°.

303 Chapter 9
26. The CD changer has rotational symmetry of order 44. Reflection is an indirect isometry because the
5 because there are 5 rotations less than 360° image of the transformed figure cannot be found
(including 0 degrees) that produce an image by moving it intact within the plane.
indistinguishable from the original. 45. Translation is a direct isometry because the
360°
magnitude  
ord
er
image of the transformed figure is found by
moving it intact within the plane.
36

¬5 46. Rotation is a direct isometry because the image of
¬72° the transformed figure is found by moving it
The magnitude of the symmetry is 72°. intact within the plane.
27. Yes; it is a proper successive reflection with
respect to the two intersecting lines.
28. Yes; it is a proper successive reflection with Page 482 Maintain Your Skills
respect to the two intersecting lines. 47. Yes; it is one reflection after another with respect
29. Yes; the teacups are rotating. to the two parallel lines.
30. Yes; the scrambler is rotating. 48. No; it is a rotation followed by a reflection with
31. No; the roller coaster is not rotating. respect to line a.
32. The letters H, I, N, O, S, X, and Z produce the 49. Yes; it is one reflection after another with respect
same letter after being rotated 180°. to the two parallel lines.
36
0° 50. C is the image of A in a reflection across line p.
40°/reflection  9 reflections
33. 
G is its own image. So, the image of A G
 reflected
34. Angles of rotation with measures 90 or 180 would
G
across line p is C .
be easier on a coordinate plane because of the
grids used in graphing. 51. C is the image of F reflected across point G.
35. In each case, y-coordinates become x-coordinates 52. H is the image of E reflected across line q. G is its
and the opposite of the x-coordinates become own image. So, the image of G E
 reflected across
y-coordinates, or (x, y) → (y, x). H
line q is G .
36. The 80° clockwise rotation and then 53. A and F are the images of C and D, respectively,
150° counterclockwise rotation about the origin is in a reflection across line p. G is its own image.
equivalent to a 70° counterclockwise rotation. So, the image of CGD reflected across line
p is AGF.
37. Any point on the line of reflection is invariant.
38. The center of rotation is the only invariant point. R
54. Q  P S because opposite sides are parallel.
39. There are no invariant points. Every point is T
55. P T R because diagonals bisect each other.
translated a units in the x-direction and b units 56. SQR  QSP because alternate interior angles
in the y-direction. are congruent.
40. Sample answer: The Tilt-A-Whirl sends riders 57. QPS  QRS because opposite angles are
tipping and spinning on a circular track. Answers congruent.
should include the following. 58. Let y be the opposite side of the right triangle
• The Tilt-A-Whirl shows rotation in two ways. formed by the eye of the surveyor, the top of the
The cars rotate about the center of the ride as building, and the side of the building at the eye
the cars go around the track. Each car rotates level of the surveyor (100 meters from the eye).
around a pivot point in the car.
tan 23° ¬y
• Answers will vary but the Scrambler, Teacups, 100
and many kiddie rides use rotation. 100tan 23° ¬y
360° 42.45 ¬y
41. B; the central angle of the octagon is 8  45°.
So, the height of the building is about
The triangle is moved three positions clockwise, or 42.45  1.55  44.0 m.
3(45)°  135°.
59. Use the triangle inequality.
5 y and y  3 z, so x  5  3 z  15 z, or
42. D; x  2
 1 2 1 2 ?
6  8  16
15 15 14
16 no
z 2 x  2 (6)  45.
  
43. 60. Use the triangle inequality.
Transformation reflection translation rotation ?
12  17  20
angle measure yes yes yes 29  20 yes
61. Use the triangle inequality.
betweenness ?
yes yes yes 22  23  37
of points
45  37 yes
orientation no yes no 62. 180a  360
36
0
collinearity yes yes yes a ¬
180
distance a ¬2
yes yes yes
measure

Chapter 9 304
63. 180a  90b ¬360 Page 482 Practice Quiz 1
2a  b ¬4 1. For a reflection in the origin, (a, b) → (a, b)
b ¬2a  4 D(1, 1) → D(1, 1)
Use a table. E(1, 4) → E(1, 4)
a b F(3, 2) → F(3, 2)
0 4 y E

1 2
2 0 F
D
Three values are (0, 4), (1, 2), and (2, 0). O x
D
64. 135a  45b ¬360 F
3a  b ¬8
b ¬3a  8
Use a table. E
a b 2. For a reflection in the line y  x, (a, b) → (b, a)
0 8 A(0, 2) → A(2, 0)
B(2, 2) → B(2, 2)
1 5
C(3, 0) → C(0, 3)
2 2 D(1, 1) → D(1, 1)
Three values are (0, 8), (1, 5), and (2, 2). y
65. 120a  30b ¬360 C
B
4a  b ¬12 A B
b ¬4a  12
D C
Use a table. O x
A
a b D
0 12
1 8
2 4 3. The translation moved each endpoint 3 units to
3 0 the left and 4 units up.
P(1, 4) → P(1  3, 4  4) or P(2, 0)
Four values are (0, 12), (1, 8), (2, 4), and (3, 0).
Q(4, 1) → Q(4  3, 1  4) or Q(1, 3)
66. 180a  60b ¬360
y
3a  b ¬6
b ¬3a  6 Q
Use a table.
a b O
P x
0 6
Q
1 3
2 0
P
Three values are (0, 6), (1, 3), and (2, 0).
4. The translation moved each vertex 1 unit to the
67. 180a  30b ¬360
right and 4 units down.
6a  b ¬12
K(2, 0) → K(2  1, 0  4) or K(1, 4)
b ¬6a  12
L(4, 2) → L(4  1, 2  4) or L(3, 2)
Use a table.
M(0, 4) → M(0  1, 4  4) or M(1, 0)
a b y
0 12 M
1 6 L
2 0 O M
Three values are (0, 12), (1, 6), and (2, 0). K x
L

305 Chapter 9
5. The 36-horse carousel has rotational symmetry of 4. Let m1 represent one interior angle of the
order 36 because there are 36 rotations less than regular decagon. Use the Interior Angle Formula.
360° (including 0 degrees) that produce an image 180(n  2)
m1   n

indistinguishable from the original.
180(10 2)
36
magnitude ¬ 0° ¬ 10
order
36
0° ¬144
¬36 Since 144 is not a factor of 360, a decagon will not
¬10° tessellate the plane.
The magnitude of the symmetry is 10°. 5. Let m1 represent one interior angle of the
regular decagon. Use the Interior Angle Formula.
180(n  2)
m1   n

9-4 Tessellations 180(30
¬  2)
30
¬168
Page 483 Geometry Activity: Tessellations of Since 168 is not a factor of 360, a decagon will not
Regular Polygons tessellate the plane.
1. equilateral triangle, square, and hexagon 6. Yes; Use the algebraic method to determine
whether a semi-regular tessellation can be
2. The measure of an interior angle of an equilateral
created using squares and triangles of side length
triangle is 60; of a square, 90; of a hexagon, 120.
1 unit.
The sum of the measures of the angles at each
Each interior angle of a square measures 90°, and
vertex must be 360. The expressions are:
each interior angle of a triangle measures 60°.
6(60)  360; 4(90)  360; 3(120)  360.
Find whole number values for h and t so that
3. The measure of an interior angle of a pentagon is 90h  60t  360.
108; of a heptagon, about 128.57; of an octagon, Let h  2.
135. 90(2)  60t ¬360
36
0 1  360 36
0 2 180  60t ¬360
108  3 3 ; 128.57  2.8; 135  2 3
 
60t ¬180
108, 128.57, and 135 are not factors of 360, so the
t ¬3
pentagon, the heptagon, and the octagon do not
When h  2 and t  3, there are two squares with
tessellate the plane.
three triangles at each vertex.
Regular Measure of Does It
Polygon One Interior Angle Tessellate? 60
60 60
triangle 60 yes 90 90
square 90 yes
pentagon 108 no 7. Yes; Use the algebraic method to determine
hexagon 120 yes whether a semi-regular tessellation can be
created using squares and octagons of side length
heptagon 128.57 no
1 unit.
octagon 135 no Each interior angle of a square measures 90°, and
each interior angle of an octagon measures
4. If a regular polygon has an interior angle with a 180(8  2)
 or 135°.
measure that is a factor of 360, then the polygon 8
will tessellate the plane. Find whole number values for h and t so that
90h  135t  360.
Pages 485–486 Check for Understanding Let h  1.
90(1)  135t ¬360
1. Semi-regular tessellations contain two or more
90  135t ¬360
regular polygons, but uniform tessellations can be
135t ¬270
any combination of shapes.
t ¬2
2. Sample answer: When h  1 and t  2, there is one square with
two octagons at each vertex.

90
135 135

3. The figure used in the tessellation appears to be a


trapezoid, which is not a regular polygon. Thus,
the tessellation cannot be regular.

Chapter 9 306
8. Yes; the pattern is a tessellation because at the 16. No; let m1 represent one interior angle of the
different vertices the sum of the angles is 360°. regular 36-gon. Use the Interior Angle Formula.
The tessellation is uniform because at every 180(n  2)
m1   n

vertex there is the same combination of shapes
180(36  2)
and angles. ¬ 
36
9. Yes; the pattern is a tessellation because at the ¬170
different vertices the sum of the angles is 360°. Since 170 is not a factor of 360, a 36-gon will not
The tessellation is not uniform because the tessellate the plane.
number of angles at the vertices varies. 17. No; A rhombus is not a regular polygon, so a semi-
10. Each “postage stamp” is a square that has been regular tessellation cannot be created from
tessellated and 90 is a factor of 360. It is a regular octagons and rhombi.
regular tessellation since only one polygon is 18. Yes; Use the algebraic method to determine
used. whether a semi-regular tessellation can be
created using regular dodecagons and equilateral
triangles of side length 1 unit.
Pages 486–487 Practice and Apply Each interior angle of a dodecagon measures
180(12  2)
11. No; let m1 represent one interior angle of the  or 150°, and each interior angle of an
12
regular nonagon. Use the Interior Angle Formula. equilateral triangle measures 60°.
180(n  2)
m1   n
Find whole number values for h and t so that
150h  60t  360.
180(9  2)
¬ 9 Let h  2.
¬140 150(2)  60t ¬360
Since 140 is not a factor of 360, a nonagon will 300  60t ¬360
not tessellate the plane. 60t ¬60
t ¬1
12. Yes; let m1 represent one interior angle of the
When h  2 and t  1, there are two dodecagons
regular nonagon. Use the Interior Angle Formula.
with one triangle at each vertex.
180(n  2)
m1   n
180(6  2)
¬ 6
¬120 60 150
Since 120 is a factor of 360, a hexagon will
tessellate the plane. 150
13. Yes; let m1 represent one interior angle of the
equilateral triangle. Use the Interior Angle
Formula.
180(n  2)
m1   n
19. Yes; Use the algebraic method to determine
whether a semi-regular tessellation can be
180(3  2)
¬ 3 created using regular dodecagons, squares, and
¬60 equilateral triangles of side length 1 unit.
Since 60 is a factor of 360, an equilateral triangle Each interior angle of a dodecagon measures
180(12  2)
will tessellate the plane.  or 150°, and each interior angle of
12
14. No; let m1 represent one interior angle of the squares and equilateral triangles measures
regular dodecagon. Use the Interior Angle 90° and 60°, respectively.
Formula. Find whole number values for h, s, and t so that
180(n  2) 150h  90s  60t  360.
m1   n Let h  1 and s  1.
180(12  2) 150(1)  90(1)  60t ¬360
¬ 1
2
¬150 150  90  60t ¬360
60t ¬120
Since 150 is not a factor of 360, a dodecagon will
t ¬2
not tessellate the plane.
When h  1, s  1, and t  2, there are one
15. No; let m1 represent one interior angle of the dodecagon, one square, and two triangles at each
regular 23-gon. Use the Interior Angle Formula. vertex.
180(n  2)
m1   n 60
180(23  2) 60
¬ 2
3
90
¬164.3 150
Since 164.3 is not a factor of 360, a 23-gon will
not tessellate the plane.

307 Chapter 9
20. No; Use the algebraic method to determine 23. yes; tessellation:
whether a semi-regular tessellation can be
created using regular heptagons, squares, and
equilateral triangles of side length 1 unit.
Each interior angle of a heptagon measures
180(7  2) 900
7  7 or approximately 128.6°, and
each interior angle of squares and equilateral
triangles measures 90° and 60°, respectively.
Find whole number values for h, s, and t so that
The pattern is a tessellation because at the
90
0
7 h  90s  60t  360.
 different vertices the sum of the angles is 360°.
Let s  1 and t  1. The tessellation is not uniform because the
90
0 number of angles at the vertices varies.
7 h  90(1)  60(1)  360

24. No; use the algebraic method to determine
90
0
7 h  90  60 ¬360
 whether the combination of a regular pentagon
90
0 and a square with equal side length tessellates
7 h ¬210

the plane.
h ¬1.63
Each interior angle of a square measures 90°, and
Let s  1 and t  2. each interior angle of a pentagon measures
90
0
7 h  90(1)  60(2)  360
 180(5  2)
5 or 108°.
90
0
7 h  90  120 ¬360
 Find whole number values for h and t so that
90
0 90h  108t  360.
7 h ¬150

Let h  1.
h ¬1.17
90(1)  108t  360
Let s  2 and t  1. 90  108t ¬360
90
0
7 h  90(2)  60(1)  360 108t ¬270

90
0 t ¬2.5
7 h  180  60 ¬360

Let h  2.
90
0
7 h ¬120
 90(2)  108t  360
h ¬0.93 180  108t ¬360
There are no more reasonable possibilities. So, a 108t ¬180
semi-regular tessellation cannot be created from t ¬1.67
regular heptagons, squares, and equilateral Let h  3.
triangles. 90(3)  108t  360
21. yes; tessellation: 270  108t ¬360
108t ¬90
t ¬0.83
No combination of a regular pentagon and a
square with equal side length can be formed such
that the total of the measures of the angles at a
vertex is 360°. So, the combination does not
The pattern is a tessellation because at the tessellate the plane.
different vertices the sum of the angles is 360°. 25. Yes; the pattern is a tessellation because at the
The tessellation is uniform because at every different vertices the sum of the angles is 360°.
vertex there is the same combination of shapes The tessellation is not uniform because the
and angles. number of angles at the vertices varies.
22. yes; tessellation: 26. Yes; the pattern is a tessellation because at the
different vertices the sum of the angles is 360°.
The tessellation is not uniform because the
number of angles at the vertices varies.
27. Yes; the pattern is a tessellation because at the
different vertices the sum of the angles is 360°.
The tessellation is uniform because at every
vertex there is the same combination of shapes
and angles. The tessellation is also regular since
The pattern is a tessellation because at the it is formed by only one type of regular polygon.
different vertices the sum of the angles is 360°. 28. Yes; the pattern is a tessellation because at the
The tessellation is uniform because at every different vertices the sum of the angles is 360°.
vertex there is the same combination of shapes The tessellation is uniform because at every
and angles. vertex there is the same combination of shapes
and angles. The tessellation is also semi-regular
since more than one regular polygon is used.

Chapter 9 308
360(12  2) 180 180(10) 180
29. The tessellation is semi-regular since more than 41. A;   12    12
2(12) 12
one regular polygon is used. The tessellation is 1800  180
also uniform because at every vertex there is the ¬ 
12
same combination of shapes and angles. 1620
¬ 
12
30. Always; the sum of the measures of the interior
angles of a triangle is 180°. If each angle is used ¬135
twice at each vertex, the sum of the angles is
360°.
31. Never; semi-regular tessellations have the same Page 488 Maintain Your Skills
combination of shapes and angles at each vertex 42. First graph ABC and point P. Draw a segment
like uniform tessellations. The shapes for semi- from point P to point A. Use a protractor to
regular tessellations are just regular. measure a 90° angle counterclockwise with P A
 as
. Use a compass to copy P
one side. Draw PR A
32. Sometimes; when the combination of shapes are . Name the segment P
onto PR A
. Repeat with
regular polygons, then the uniform tessellation
points B and C. ABC is the image of ABC
becomes semi-regular.
under a 90° counterclockwise rotation about
33. Always; the sum of the measures of the angles of point P.
a quadrilateral is 360°. So if each angle of the
y
quadrilateral is rotated at the vertex, then that A  (1, 12)
equals 360 and the tessellation is possible.
34. Never; the measure of an interior angle is 8
180(16  2) B  (6, 6)
  or 157.5, which is not a factor of 360.
16 90 4 A
35. Yes; the measure of each angle is 90°. PO x
36. None of these; the tessellation is not uniform 4 C 8
C (2, 0)
because the number of angles at the vertices
varies. It is not regular since more than one
B
polygon is used. It is not semi-regular since not
all of the polygons are regular. 43. First graph DEF and point P. Draw a segment
37. The tessellation is uniform because at every from point P to point D. Use a protractor to
vertex there is the same combination of shapes measure a 90° angle clockwise with PD as one
. Use a compass to copy P
side. Draw PR D
 onto
and angles. The tessellation is also regular since

PR. Name the segment P D
. Repeat with points E
only one regular polygon is used.
and F. DEF is the image of DEF under a
38. Sample answers:
90° clockwise rotation about point P.
y F
D
90 x
135 135 O F  (8, 1)
90
90 90 P
E

E  (2, 5) D  (7, 5)

44. First graph parallelogram GHIJ and point P.


The measures are 90°, 90°, 90°, 135°, and 135°. Draw a segment from point P to point G.
The tessellation is not regular since the Use a protractor to measure a 90° angle
counterclockwise with PG as one side. Draw PR.
pentagons are not regular and it is not uniform
Use a compass to copy P G . Name the
 onto PR
since the number of angles at the vertices varies.
segment PG. Repeat with points H, I, and J.
39. Sample answer: Tessellations can be used in art Parallelogram GHIJ is the image of GHIJ under
to create abstract art. Answers should include the a 90° counterclockwise rotation about point P.
following.
G y
• The equilateral triangles are arranged to form
hexagons, which are arranged adjacent to one x
another. J O
• Sample answers: kites, trapezoids, isosceles G  (7, 2) 90
triangles P
180(n 2) H
40. C; interior angle   J
n (4, 4) H
180(9 2) (2, 4)
¬ 9 I I  (1, 6)
¬140

309 Chapter 9
45. First graph rectangle KLMN and point P. Draw a 50. 12, 16, 20
segment from point P to point K. Use a protractor Since the measure of the longest side is 20,
to measure a 90° angle counterclockwise with PK
 20 must be c, and a and b are 12 and 16.
. Use a compass to copy P
as one side. Draw PR K
 a2  b2 ¬c2
. Name the segment P
onto PR K
. Repeat with 122  162 ¬202
points L, M, and N. Rectangle KLMN is the 144  256 ¬400
image of KLMN under a 90° counterclockwise 400 ¬400
rotation about point P. These segments form the sides of a right triangle
y since they satisfy the Pythagorean Theorem. The
M  (2, 9) measures are whole numbers and form a
8 Pythagorean triple.
L 4 L N  (6, 3)
51. 9, 10, 15
(5, 5) Since the measure of the longest side is 15,
PO M x 15 must be c, and a and b are 9 and 10.
4 4 8 a2  b2 ¬c2
90 K 
4 (3, 1) 92  102 ¬152
K 81  100 ¬225
8 181 ¬225
N
Since 181 225, segments with these measures
46. The move is a translation 15 feet out from the cannot form a right triangle. Therefore, they do
wall and 21 feet to the left, then a rotation of not form a Pythagorean triple.
90° clockwise. 52. 2.5, 6, 6.5
47. Opposite sides of a parallelogram are congruent. Since the measure of the longest side is 6.5,
Set the expressions for opposite sides equal and 6.5 must be c, and a and b are 2.5 and 6.
solve. a2  b2 ¬c2
y  y2 2.52  62 ¬6.52
0  y2  y 6.25  36 ¬42.25
0  y(y  1) 42.25 ¬42.25
0  y or 0  y  1 Since 42.25  42.25, segments with these
y0 y1 measures form a right triangle. However, only one
Since y represents a side length, which must be of the three numbers is a whole number.
positive, y  1. Therefore, they do not form a Pythagorean triple.
6x ¬4x  8 53. 14, 14
 3, 28
2x ¬8 Since the measure of the longest side is 28,
x ¬4 28 must be c, and a and b are 14 and 14
3 .
So, when x is 4 and y is 1, the quadrilateral is a a2  b2 ¬c2
parallelogram. 142  (14
3 )2 ¬282
48. Opposite sides of a parallelogram are congruent. 196  588 ¬784
Set the expressions for opposite sides equal and 784 ¬784
solve. Since 784  784, segments with these measures
5y ¬2y  36 form a right triangle. However, only two of the
3y ¬36 three numbers are whole numbers. Therefore,
y ¬12 they do not form a Pythagorean triple.
6x  2 ¬64 54. 14, 48, 50
6x ¬66 Since the measure of the longest side is 50,
x ¬11 50 must be c, and a and b are 14 and 48.
So, when x is 11 and y is 12, the quadrilateral is a a2  b2 ¬c2
parallelogram. 142  482 ¬502
49. Opposite angles of a parallelogram are congruent 196  2304 ¬2500
and the sum of its interior angles is 360°. 2500 ¬2500
2x  8 ¬120 These segments form the sides of a right triangle
2x ¬112 since they satisfy the Pythagorean Theorem. The
x ¬56 measures are whole numbers and form a
120  120  2(5y) ¬360 Pythagorean triple.
10y ¬120
y ¬12
So, when x is 56 and y is 12, the quadrilateral is a
parallelogram.

Chapter 9 310
55. 1
 1 1
2, 3, 4 59. y Q
Since the measure of the longest side
is 1
 1 1 1
2 , 2 must be c, and a and b are 3 and 4 .
a2  b2 ¬c2 R
2 2 2 P
13  14 ¬12
1   1
1
1
6 ¬4
x
9
25 1

14 4 ¬4
S
25 1 From Exercise 58, slope of PQ
  1 slope of
Since  144 4, segments with these measures S
R   1, slope of Q
R
  1, and slope of P
S  1.
cannot form a right triangle. Therefore, they do Since the product of the slopes of adjacent sides
not form a Pythagorean triple. of quadrilateral PQRS is 1, the adjacent sides
56. By the Triangle Midsegment Theorem, are perpendicular.
B
A E and AB  1
F  C
2 EF, B D and BC  1
F 
2 DF,
60. y Q


and AC DE and AC  1
 2 DE. So, EF  30,

DF  22, DE  26, and the perimeter
is 30  22  26  78. R
P
57. By the Triangle Midsegment Theorem,
AB  FC, AC  1
2 DE, and BC  DA. So, AB  7,
 x

BC  10, and AC  9. S
58. y Q Use the distance formula to find the length of
each side.
d ¬
(x
 2 x1)2 (y2  y1)2
R PQ ¬
 (1  5)2  (6  2)2

P ¬
(4)
 2 42
x

32
QR ¬
 (3  1)2  (2  6)2
S ¬
(4)
 2  (4 )2

Q
P and R
S
 are opposite sides. ¬
32
y y RS ¬
 [1  (3)]2 
 (2  2)2
Q
slope of P ¬
x 
2
x
1


4 2  (
2 1
6 
2 4)2
¬
15 ¬
32
4
¬
4 PS ¬
 (1  5)2  ( 2  2)2
¬1 ¬
(4)
 2  (4 )2

2  2 ¬
32
S
slope of R ¬
1  (
3)
4 61. The measures of the sides of quadrilateral PQRS
¬4 are equal. Opposite sides are parallel and
¬1 adjacent sides are perpendicular. PQRS is a
R
Q and P
S
 are opposite sides. square.
26
R
slope of Q ¬
3 
1
62. Corresponding sides of similar polygons are
4 proportional.
¬
4 AB
scale factor ¬
WX
¬1
8
2 2 ¬
S
slope of P ¬15
12
4 ¬2

¬
4
3
¬1 The scale factor is 2

3.
The opposite sides of quadrilateral PQRS have
the same slopes. Therefore, the sides are parallel.

311 Chapter 9
AB 8 2
63. The scale factor is 
WX 12  3.
The ratios of the measures of the corresponding 9-5 Dilations
sides of similar polygons are equal.
AB BC

WX ¬XY
 Pages 493–494 Check for Understanding
2
 ¬10
X
1. Dilations only preserve length if the scale factor is
3 Y
3 1 or 1. So for any other scale factor, length is not
XY ¬ 2 (10)
 preserved and the dilation is not an isometry.
XY ¬15 2. Sample answer:
AB 8 2
64. The scale factor is 
WX 12  3. A B
The ratios of the measures of the corresponding
sides of similar polygons are equal. A B
AB CD A B

WX ¬YZ

2 10
 ¬
3 YZ D C
YZ ¬32
 (10) D C

YZ ¬15 D C
AB 8 2
65. The scale factor is 
WX 12  3. 3. Trey; Desiree found the image using a positive
The ratios of the measures of the corresponding scale factor.
sides of similar polygons are equal.
4. Since |4| 1, the dilation is an enlargement.
AB AD , CW
, CV
, and CU. Since r is positive,
X ¬

W W Z Draw CX
2 ¬ 15
 X, W, V, and U will lie on the continuation of
3 WZ the sides of the quadrilateral.
3
WZ ¬ 2 (15)
 Locate X, W, V, and U so that CX  4(CX),
CW  4(CW), CV  4(CV), and CU  4(CU).
WZ ¬22.5
Draw quadrilateral XWVU.

Page 489 Geometry Activity: Tessellations and


Transformations
1. Yes; whatever space is taken out of the square is
then added onto the outside of the square. The
area does not change; only the shape changes.
2. Modify the bottom of the unit to be like the right
side of the triangle. Erase the bottom and right C
original sides of the triangle.

5. Since 1
5 1, the dilation is a reduction. Draw

, CR
CP , CS , CT
, CU
, CV
. Locate P, R, S, T, U,

3. and V so that CP  1 1


5 (CP), CR  5 (CR),

CS  1 1 1
5 (CS), CT  5 (CT), CU  5 (CU), and


CV  1

5 (CV). Draw hexagon PRSTUV.
R

4.

P S

R S
5. P T
C
V U

V T

Chapter 9 312
6. Since |2|  1, the dilation is an enlargement. 10. Find K, L, and M using the scale factor, r  3.
Draw C E, C
D
, C
G, and CF. Since r is negative,
Preimage Image
E, D, G, and F will lie on rays that are opposite (x, y) (3x, 3y)
to 
CE, 
CD, 
CG, and 
CF, respectively. Locate E,
D, G, and F so that CE  2(CE), CD  2(CD), K(5, 8) K(15, 24)
CG  2(CG), and CF  2(CF). Draw L(3, 4) L(9, 12)
quadrilateral EDGF.
E M(1, 6) M(3, 18)
D y K
24
L
12 K
L
F
24 12 12 24 x
12
M
G
M
C
G 24
F
11. For ease, compare diagonals of the figures.
D image length
E scale factor ¬ 
preimage length
7. AB  3, r  4 8 un
r ¬ its
Use the Dilation Theorem. 4 units
AB  |r|(AB) r ¬2
AB  (4)(3) Since the scale factor is greater than 1, the
AB  12 dilation is an enlargement.
8. AB  8, r 2

5
12. Compare the vertical sides of the triangles.
image length
Use the Dilation Theorem. scale factor ¬ 
preimage length
AB ¬|r|(AB) 4 un
its
r ¬
8 ¬2

5 (AB)
6 units
r ¬2

20 ¬AB 3
9. Find P and Q using the scale factor, r  1
 Since 0 |r| 1, the dilation is a reduction.
3.
13. C; The drawing and the garden are similar.
Image 12ft x
18 ft ¬ 8 in.
Preimage 
y
(x, y) x3, 
3 2 ¬ x
3 8 in.
P(9, 0) P(3, 0) 5 1
3 in. ¬x

Q(0, 6) Q(0, 2)
y
Pages 494–496 Practice and Apply
Q
14. Since |3| 1, the dilation is an enlargement.
Draw CX, 
CY, and 
CZ. Since r is positive, X, Y,
Q and Z will lie on the continuation of the sides of
the triangle.
Locate X, Y, and Z so that CX  3(CX),
O P P x CY  3(CY), and CZ  3(CZ). Draw XYZ.

X X
C

Y Z

Y Z

313 Chapter 9
15. Since |2| 1, the dilation is an enlargement. 18. Since |1| 1, the dilation is a congruence
Draw CT, 
CS, 
CR, and  CP. Since r is positive, T, transformation. Draw  CE, 
CD, 
CA, and CB. Since
S, R, and P will lie on the continuations of the r is negative, E, D, A, and B will lie on rays
sides of the quadrilateral. Locate T, S, R, and that are opposite to 
CE, 
CD, 
CA, and CB. Locate
P so that CT  2(CT), CS  2(CS), CR  2(CR), E, D, A, and B so that CE  CE, CD  CD,
and CP  2(CP). Draw quadrilateral TSRP. CA  CA, and CB  CB. Draw quadrilateral
T S EDAB.
A
T S E

C B
D
D
B C
P R
P R
E
16. Since 21 1, the dilation is a reduction. Draw C
K
, A
, C
CL M
, and CN. Since r is positive, the reduction
will have the same orientation. Locate 19. Since 1 4 1, the dilation is a reduction. Draw

K, L, M, and N so that CK  1 
2 (CK), L
C, CM
, and C N
. Since r is negative, L, M, and
CL 1 1 1 , CM
, and
2 (CL), CM  2 (CM), and CN  2 (CN).
 N lie on rays that are opposite to CL

CN, respectively. Locate L, M, and N so that
Draw quadrilateral KLMN.
N CL  1 1 1
4 (CL), CM  4 (CM), and CN  4 (CN).

Draw LMN.

K N M
K
C
L
M
L M

C N
L N
L
17. Since 2
5 1, the dilation is a reduction. Draw
 M
20. ST  6, r  1
R
C, CS
, and C T. Since r is positive, the reduction
Use the Dilation Theorem.
will have the same orientation. Locate R, S, and
ST  |r|(ST)
T so that CR 2 2
5 (CR), CS  5 (CS), and

2 ST  (1)(6)
CT  5(CT). Draw RST. ST  6
R 21. ST  4 3
5, r  4

T R Use the Dilation Theorem.
T C ST  |r|(ST)
S
S ST  3
4  5 
 4

ST  3

5
22. ST  12, r  2

3
Use the Dilation Theorem.
ST ¬|r|(ST)
12 ¬2
3 (ST)

18 ¬ST
12 3
23. ST  5 , r  5
Use the Dilation Theorem.
ST ¬|r|(ST)
1
 2
 
3
5 ¬ 5 (ST)
4 ¬ST

Chapter 9 314
24. ST  32, r  5

4 Find X, Y, and Z using the scale factor, r  1

2.
Use the Dilation Theorem.
Preimage Image
ST  |r|(ST)
ST  5
(x, y) 12x, 12y
4 (32)

ST  40 X(1, 2) X1
2 , 1

25. ST  2.25, r  0.4
Y(4, 3) Y2, 3
2

Use the Dilation Theorem.
ST  |r|(ST)
Z(6, 1) Z3, 1
2

ST  (0.4)(2.25)
ST  0.9
y
26. Find F, G, and H using the scale factor, r  2. x
OX Z
Preimage Image Z
Y
(x, y) (2x, 2y)
F(3, 4) F(6, 8) X
G(6, 10) G(12, 20)
Y
H(3, 5) H(6, 10)
Find F, G, and H using the scale factor, r  1

2. 28. Find P, Q, R, and S using the scale factor, r  2.

Image Preimage Image


Preimage (x, y) (2x, 2y)
(x, y) 12x, 12y P(1, 2) P(2, 4)
F(3, 4) F3
2 , 2
 Q(3, 3) Q(6, 6)
G(6, 10) G(3, 5) R(3, 5) R(6, 10)
S(1, 4) S(2, 8)
H(3, 5) H3
2, 2
 5
Find P, Q, R, and S using the scale factor,
y G r  1

2.
20

Preimage Image
16
(x, y) 12x, 12y
12
P(1, 2) P1
2 , 1

H 8
F
Q(3, 3) Q3
2, 2
 3
4
G
H
F R(3, 5) R3
2, 2
 5
4 O 4 8 12 x
S(1, 4) S1
2 , 2

27. Find X, Y, and Z using the scale factor, r  2.
Preimage Image y R
(x, y) (2x, 2y)
S
X(1, 2) K(2, 4)

Y(4, 3) Y(8, 6) Q


P
Z(6, 1) Z(12, 2)
S R
Q
P
O x

315 Chapter 9
29. Find K, L, M, and N using the scale factor, r  2. 33. Compare QT and QT. Use the Pythagorean
Theorem to find the side lengths.
Preimage Image image length
(x, y) (2x, 2y) scale factor ¬
preimage length
K(4, 2) K(8, 4)

4 
r ¬ 
3
2 2
13 units
L(4, 6) L(8, 12)

42  12 units
M(6, 8) M(12, 16)
17
 
3
r ¬ 
N(6, 10) N(12, 20)
17
1
Find K, L, M, and N using the scale factor, r ¬ 3

r  21. Since 0 |r| 1, the dilation is a reduction.
34. Compare YZ and YZ. Note that the scale factor
Preimage Image is negative since the image appears on the
(x, y) 12x, y opposite side of the center with respect to the
preimage. Use the Pythagorean Theorem to find
K(4, 2) K(2, 1) the side lengths.
L(4, 6) L(2, 3) image length
scale factor ¬
preimage length
M(6, 8) M(3, 4)
N(6, 10) N(3, 5)

1
r ¬ 
 
23 2
4  units


42  32 units
L y
25

12 4
r ¬ 

25
8 1
r ¬4
K
4 Since 0 |r| 1, the dilation is a reduction.
L K
35. Compare BD and BD. Note that the scale factor
12 8 4 O 4 8 12 x is negative since the image points appear on the
4 opposite side of the center with respect to the
M N preimage points. Use the Pythagorean Theorem to
8 find the side lengths.
image length
scale factor ¬
12 preimage length


42  12 units
16 r ¬ 
M

2  
2
2
1 2
units
20
N
17

r ¬ 


 17
30. Compare sides of the squares. 2
image length r ¬2
scale factor ¬
preimage length
Since |r|  1, the dilation is an enlargement.
6 un
its
r ¬
2 units 36. Determine the width in inches of the actual
r ¬3 wingspan of the SR-71.
12 
in.
Since the scale factor is greater than 1, the (55 ft)1 ft  7 in.  660 in.  7 in.
dilation is an enlargement.  667 in.
31. Compare the vertical sides of the triangles. Find the scale factor by dividing the wingspan of
image length the model by the wingspan of the plane.
scale factor ¬ 14 1 1
preimage length 

667   7 
66 48
2 un
its 14
r ¬4 units 1
The scale factor is about  .
r ¬1

2
48
37. Each dimension is reduced by a factor of 0.75.
Since 0 |r| 1, the dilation is a reduction.
0.75(10)  7.5
32. By inspection, we see that the scale factor is 1. 0.75(14)  10.5
This is a congruence transformation. The new dimensions are 7.5 in. by 10.5 in.

Chapter 9 316
38. Find the ratio of the area of the image to that of 44. The width and height of the photograph are
the preimage. increased by 150%, or a factor of 1.5.
area of image 0.75(10)  0.75(14) 1.5(480)  720
 ¬  10  14
area of preimage 1.5(640)  960
¬0.752 The dimensions of the image are 960 pixels by
¬0.5625 720 pixels.
9 45. The original width is 640 pixels. To reduce it
¬
16
9 to 32 pixels, Dinah must use a scale factor
The area of the image is 16 that of the preimage. 3
2 1
640  20 .
of 
39. Each side of the rectangle is lengthened by a
factor of 4, so the perimeter is four times the 46. The original height is 480 pixels. To enlarge it to
600 5
original perimeter. 600 pixels, Dinah used a scale factor of 480  4 .
40. Find the ratio of the areas. 47. The dimensions of the photograph are 10 cm by
area of image (4b)(
4h)
   bh
12 cm. The space available is 6 cm by 8 cm.
area of preimage
6  0.6 and 8  0.67, so a scale factor of 0.6 is
 16 10 12
required for the photograph to be as large as
The area is 16 times the original area.
possible on the page. So, she should save the
41. Given: dilation with center C and scale factor r image file at 60%.
Prove: ED  r(AB)
48. Use the distance formula to find the length of
A E each side.
C
d 
 (x2  
x1)2 
(y2 y1)2
B CD 
 (3  7)2  (8
 7)2
D 
16  1

17

Proof:
BC 
 (7  5)2  [7
 (1)]2
CE  r(CA) and CD  r(CB) by the definition of
CE CD CE CD 
4  64
CA  r and CB  r. So, CA  CB
a dilation.       
 2
17
by substitution. ACB  ECD, since congruence AB 
 [5  (
1)]2  (1 1)2
of angles is reflexive. Therefore, by SAS

36  4
Similarity, ACB is similar to ECD. The
 2
10
corresponding sides of similar triangles are
ED CE CE AD 
 [3  (
1)]2  (8 1)2
proportional, so  AB  CA . We know that CA  r,
    

16  49 
E D
so AB  r by substitution. Therefore, ED  r(AB)
 
65

by the Multiplication Property of Equality. Find the perimeter.
42. First dilation: (x, y) → (rx, ry) P ¬
17
  2
17   2
10  
65 
Second dilation: (rx, ry) → (r2x, r2y) ¬26.8
Find r2 using the x-values of A and A. The perimeter is about 26.8 units.
3  r2(12) 49. Find A, B, C, and D using the scale factor,
1
  r2 r  2.
4
1  r Preimage (x, y) Image (2x, 2y)
2
So, the scale factor is 1. A(1, 1) A(2, 2)
2 B(5, 1) B(10, 2)
43. Use the distance formula to find XY and XY.
C(7, 7) C(14, 14)
d 
(x2  x1)2 (y2 y1)2
D(3, 8) D(6, 16)
XY 
 (0  4
)2  (5 2)2

 16  9 y D
5 8 C
XY 
 (12 6)2  (11 3)2
4

 36  64 B
 10 A
Find the absolute value of the scale factor. 12 8 4 O 4 B 8x
image length A
r    4
preimage length
10
 5
 8
2
12
The absolute value of the scale factor is 2.
C 16
D

317 Chapter 9
50. Use the distance formula to find the length of Dilation with scale factor of 1

3:
 
each side. x y
(x, y) → 3, 3
dz 
(x2x1)2 (y2 
y1)2
AB 
(10
  2)2 
 [2 
(2)]2 (10, 4) → T 1

0 4
3 , 3 

144  16
 4
10

(7, 7) → U 7
 7
3, 3 
BC 
 [14  
 (10)] 2 (14 2)2

16  256

(3, 1) → V 1, 1
3
 
 4
17
 y
CD 
 [6  
(14)] 2  [
16  (14
)]2

64  4
 2
17

AD 
 (6  2)2 [16  (2)]2 U

64  196
 2
65 V T x
Find the perimeter.
P ¬4
10  4
17   2
17   2
65
¬53.5
The perimeter of quadrilateral ABCD is about
53.5 units, so it is twice the perimeter of
quadrilateral ABCD.
51. T(6, 5), U(3, 8), V(1, 2)
Reflection in the x-axis:
(x, y) → (x, y) 52. Translate the points so that the center is the
T(6, 5) → (6, 5) origin.
U(3, 8) → (3, 8) (x, y) → (x  3, y  5)
V(1, 2) → (1, 2) G(3, 5) → (0, 0)
H(7, 4) → (4, 9)
y
I(1, 0) → (4, 5)
y
12

8
G
4
x
I x
V 12 8 4 4 8 12
4 H
T 8

12

U 16
Translation with (x, y) → (x  4, y  1):
(6, 5) → (10, 4) 20
(3, 8) → (7, 7)
(1, 2) → (3, 1) Dilate the figure using the scale factor 2.
(x, y) → (2x, 2y)
y
(0, 0) → (0, 0)
(4, 9) → (8, 18)
(4, 5) → (8, 10)

Chapter 9 318
y 55. A; find the slope of 3x  5y  12.
12 3x  5y¬ 12
5y¬ 3x  12
8 y¬ 3
5x  5
 1
2

4 The slope is 3


5.
x The slopes of perpendicular lines are opposite
reciprocals of each other.
12 8 4 4 8 12 1  5
4  3 3
5
8
The slope is 5

3.

12
Page 497 Maintain Your Skills
16 56. No; use the algebraic method to determine
whether a semi-regular tessellation can be
20
created using equilateral triangles and regular
Translate the points back so that the center is at pentagons of side length 1 unit.
(3, 5). Each interior angle of an equilateral triangle
(x, y) → (x  3, x  5) measures 60°, and each interior angle of a regular
180(5 2)
(0, 0) → G(3, 5) pentagon measures  5
or 108°.
(8, 18) → H(11, 13) Find whole number values for h and t so that
(8, 10) → I(5, 5) 60h  108t  360.
y Let h  1.
12 60(1)  108t¬ 360
60  108t¬ 360
8 108t¬ 300
G
t¬ 2.8
4
Let h  2.
x 60(2)  108t¬ 360
12 8 4 4 8 12 120  108t¬ 360
4 108t¬ 240
I t¬ 2.2
8 Let h  3.
60(3)  108t¬ 360
12 180  108t¬ 360
H 108t¬ 180
16
t¬ 1.7
20 Let h  4.
60(4)  108t¬ 360
The coordinates of the vertices of the image are 240  108t¬ 120
G(3, 5), H(11, 13) and I(5, 5). 108t¬ 120
53. Sample answer: Yes; a cut and paste produces an t¬ 1.1
image congruent to the original. Answers should Let h  5.
include the following. 60(5)  108t¬ 360
• Congruent figures are similar, so cutting and 300  108t¬ 360
pasting is a similarity transformation. 108t¬ 60
• If you scale both horizontally and vertically by t¬ 0.6
the same factor, you are creating a dilation. There are no more reasonable possibilities. So, a
semi-regular tessellation cannot be created from
54. B; the pentagons are similar. Find the scale factor
equilateral triangles and regular pentagons.
by dividing the length of the radius of the larger
pentagon by that of the smaller pentagon. 57. No; use the algebraic method to determine
6 6 whether a semi-regular tessellation can be
Scale factor   6
12
created using regular octagons and hexagons of
 6 side length 1 unit.
2 Each interior angle of a regular octagon measures
So, the perimeter of the larger pentagon is twice 180(8
  2)
8
or 135°, and each interior angle of a
that of the smaller pentagon (5n). The perimeter 180(6  2)
regular hexagon measures  6
or 120°.
of the larger pentagon is 10n.

319 Chapter 9
Find whole number values for h and t so that 60. First graph ABC and point P. Draw a segment
135h  120t  360. from point P to point A. Use a protractor to
Let t  1. measure a 90° angle counterclockwise with P A
 as
135h  120(1)¬ 360 one side. Draw 
PR. Use a compass to copy P A
135h  120¬ 360 . Name the segment P
onto PR A
. Repeat with
135h¬ 240 points B and C. ABC is the image of ABC
t¬ 1.8 under a 90° counterclockwise rotation about
Let t  2. point P.
135h  120(2)¬ 360 y A
135h  240¬ 360
135h¬¬120
t¬ 0.9 B
There are no more reasonable possibilities. So, a
semi-regular tessellation cannot be created from
regular octagons and hexagons. C
58. Yes; use the algebraic method to determine C
whether a semi-regular tessellation can be
P 90
created using squares and equilateral triangles of
side length 1 unit.
Each interior angle of a square measures 90°, and
each interior angle of a triangle measures 60°. O B x
Find whole number values for h and t so that A
90h  60t  360.
61. First graph parallelogram DEFG and point P.
Let h  2.
Draw a segment from point P to point D. Use a
90(2)  60t  360
protractor to measure a 90° angle clockwise with
180  60t  360
P as one side. Draw 
D PR. Use a compass to copy
60t  180
P onto 
D PR. Name the segment P D. Repeat with
t3
points E, F, and G. Parallelogram DEFG is the
When h  2 and t  3, there are two squares with
image of DEFG under a 90° clockwise rotation
three triangles at each vertex.
about point P.
60 y
60 60 E
90 90 F

O x
59. No; use the algebraic method to determine
whether a semi-regular tessellation can be D
created using regular hexagons and dodecagons of
side length 1 unit. 90 G
Each interior angle of a regular hexagon D
180(6 2) P E
measures  6
or 120°, and each interior
180(12  2)
angle of a regular dodecagon measures  12
or 150°.
Find whole number values for h and t so that
120h  150t  360.
Let h  1. G
F
120(1)  150t¬ 360
120  150t¬ 360 62. Yes; the opposite sides of a rectangle are
150t¬ 240 congruent, and the diagonals are congruent.
t¬ 1.6 63. Given: J  L
Let h  2. B is the midpoint of JL
.
120(2)  150t¬ 360 Prove: JHB  LCB
240  150t¬ 360 J
150t¬ 120
t¬ 0.8 B H
C
There are no more reasonable possibilities. So, a
semi-regular tessellation cannot be created from
regular hexagons and dodecagons. L
Proof: It is known that J  L. Since B is the
midpoint of JL
, J
B
LB
 by the Midpoint
Theorem. JBH  LBC because vertical angles
are congruent. Thus, JHB  LCB by ASA.

Chapter 9 320
64. Use the tangent ratio. 5. Find A, B, and C using the scale factor, r  1

2.
BC
tan A   y
AC
2
Preimage (x, y) Image x
2, 2
tan A  3
A(10, 2) A(5, 1)
A  tan1 2
3

B(1, 6) B(1
2 , 3)

Use a calculator to find mA.
mA  33.7 C(4, 4) C(2, 2)
65. Use the tangent ratio. y
BC 8
tan A  
AB
 B
28 C
tan A  7
4

A tan1 4 A
Use a calculator to find mA. A O 4 8 x
mA  76.0 C
B
66. Use the cosine ratio.
AC
cos A  
AB

20
cos A  
32
A  cos1 5
8

9-6 Vectors
Use a calculator to find mA.
mA  51.3
Page 501 Geometry Activity:
Comparing Magnitude and
Page 497 Practice Quiz 2 Components of Vectors
1. Yes; the pattern is a tessellation because at the 1. See students’ work.
different vertices the sum of the angles is 360°. 2. The components of b are twice the components of a .
The tessellation is uniform because at every 3. The components of b are three times the
vertex there is the same combination of shapes components of a .
and angles. 4. Sample answer: The magnitude is n times greater
The tessellation is semi-regular because it is than the magnitude of x, y, and the direction is
composed of more than one type of regular the same.
polygon.
2. Yes; the pattern is a tessellation because at the
different vertices the sum of the angles is 360°.
Pages 502–503 Check for Understanding
The tessellation is uniform because at every
1. Sample answer: 7, 7
vertex there is the same combination of shapes
and angles. y

3. Since 3
4 1, the dilation is a reduction. 4, 4 3, 3
Draw CD, C
F
, and C
E. Locate D, F, and E on
D
C, C
F
, and CE so that CD  3
4 (CD), O x
CF  3
(CF), and CE  3(CE). Draw DFE.
4 4 2. Two equal vectors must have the same magnitude
D
D and direction, but parallel vectors have the same
direction. The magnitude of parallel vectors can
C be different.
E 3. Sample answer: Using a vector to translate a
F F figure is the same as using an ordered pair
E
because a vector has horizontal and vertical
4. Since 2  1, the dilation is an enlargement. components which can be represented by ordered
Since r is negative, P, R, S, T, and U will lie pairs.
on rays that are opposite to 
CP, CR, 
CS, 
CT, and 4. Find the change in x-values and the

CU respectively. Locate P, R, S, T, and U so corresponding change in y-values.
that CP  2(CP), CR  2(CR), CS,  2(CS), AB  x2  x1, y2  y1
CT  2(CT), and CU  2(CU). Draw PRSTU.
 1  (4), 3  (3)
R  5, 6
T U P
S
C S
R P
U
T

321 Chapter 9
y y
5. Find the change in x-values and the tan A  
x 
2
x
1
2 1
corresponding change in y-values. 4  0
CD  x2  x1, y2  y1  12  (6)
 0  (4), 1  4  2

3
 4, 3
mA  tan1 2

3
6. Find the magnitude using the distance formula.
 33.7
AB ¬
(x
 2
x1)2 
(y2 
y1)2 A vector in standard position that is equal to AB

(3  2)2 (3  7
)2 forms a 33.7° angle with the negative x-axis in

41
 the third quadrant. So it forms a 180  33.7 or
¬6.4 213.7° angle with the positive x-axis.
Graph AB to determine how to find the direction. Thus, AB has a magnitude of 2
 13 or about
Draw a right triangle that has AB as its 7.2 units and a direction of about 213.7°.
hypotenuse and an acute angle at A. 8. Find the magnitude using the Pythagorean
y Theorem.
A v 
x2  y2

 2
82  (15)
 17
Vector v lies in the fourth quadrant. Find the
B direction.
y
tan  x
x
15
8

tan A  
y y m  tan1 8 
15
x 
2 1
2x 1
3 7  61.9

3  2 v forms a 61.9° angle with the positive x-axis in
 4

5 the fourth quadrant. So, it forms a 360  61.9 or
mA  tan1 4 
5
298.1° angle with the positive x-axis.
 38.7 Thus, v has a magnitude of 17 units and a
A vector in standard position that is equal to AB direction of about 298.1°.
forms a 38.7° angle with the negative x-axis in 9. First, graph JKL. Next, translate each vertex
the third quadrant. So it forms a 180  38.7 or by t , 1 unit left and 9 units up. Connect the
218.7° angle with the positive x-axis. vertices to form JKL.
Thus, AB has a magnitude of
 41 or about L y
6.4 units and a direction of about 218.7°. 16

7. Find the magnitude using the distance formula. 12


AB 
(x2 x1)2 
(y2 
y1)2
L J

[12
 
 (6)] 2  (
4  
0)2 8
K
 2
13 4
 7.2
O
Graph AB to determine how to find the direction. x
8 4 4
Draw a right triangle that has AB as its J
K
hypotenuse and an acute angle at A.
y
10. First, graph trapezoid PQRS. Next, translate each
vertex by u, 3 units right and 3 units down.
Connect the vertices to form trapezoid PQRS.
y
4 Q
P
P Q R
A(6, 0) O S x
8 12 16
x R
4
S

Chapter 9 322
11. First, graph WXYZ. Next, translate each can be found by adding the two vectors. Find the
vertex by e , 1 unit left and 6 units up. Finally, components of the current vector.
translate each vertex by f , 8 units right and x, y  3cos(30°), 3sin(30°)
5 units down. Connect the vertices to form
WXYZ. 
 3

2
3
2 
 , 3

y Find the resultant vector.


Z
x
 3

10     3 
2 , 0   2   10  1.5
3
3
, 1.5
4 Z O 4 8 12 Use the Pythagorean Theorem to find the
Y
4 magnitude.
Y W
W 10  1.5
3, 1.5
8 X

 (10 1.5
 
)  (1.5
X 2
3 )2
12. Find the resultant vector.  12.7
g  h  4  0, 0  6 The resultant vector lies in the fourth quadrant.
Find the direction.
 4, 6 y
Find the magnitude to the resultant vector. tan  x
1
.5
g  h 
x2  y2 
10  1.5
3


42  62
m  tan1  
1.5
 2
 13 10  1.5
3 
 6.8
 7.2
The speed of the boat is about 12.7 knots, at a
The resultant vector lies in the first quadrant.
direction of about 6.8° south of due east.
Find the direction.
y
tan  x
 6

4 Pages 503–505 Practice and Apply
 3

2
15. Find the change in x-values and the
corresponding change in y-values.
m  tan1   3
2 AB  x2  x1, y2  y1
 56.3
 3  1, 3  (3)
The resultant vector forms a 56.3° angle with the
 2, 6
positive x-axis in the first quadrant. Thus, g  h
has a magnitude of 2
 13 or about 7.2 units and a 16. Find the change in x-values and the
direction of about 56.3°. corresponding change in y-values.
CD  x2  x1, y2  y1
13. Find the resultant vector.
t  u  0  12, 9  9  3  (2), 4  0
 12, 18  1, 4
Find the magnitude of the resultant vector. 17. Find the change in x-values and the
t  u 
 x2  y2 corresponding change in y-values.

 122  (18)2 EF  x2  x1, y2  y1
 6
 13  3  4, 1  3
 21.6  7, 4
The resultant vector lies in the fourth quadrant. 18. Find the change in x-values and the
Find the direction. corresponding change in y-values.
y
tan  x GH  x2  x1, y2  y1
1
 8  2  (3), 4  4
12
 5, 0
 3

2 19. Find the change in x-values and the
m  tan1 3 2
 corresponding change in y-values.
 56.3 LM  x2  x1, y2  y1
The resultant vector forms a 56.3° angle with  1  4, 3  (2)
the positive x-axis in the fourth quadrant. So, it  3, 5
forms a 360  56.3 or 303.7° angle with the 20. Find the change in x-values and the
positive x-axis. corresponding change in y-values.
Thus, t  u has a magnitude of 6
 13 or about NP  x2  x1, y2  y1
21.6 units and a direction of about 303.7°.
 1  (4), 1  (3)
14. The initial path of the boat is due east, so a vector  3, 2
representing the speed of the boat lies on the
21. Find the magnitude using the distance formula.
positive x-axis and is 10 units long. The current is
flowing 30° south of east, so a vector representing CD 
(x 2x1)2 
(y2 
y1)2
the speed of the current will be 30° below the 

(9  4
)2  (2
 2)2
positive x-axis 3 units long. The resultant speed 5

323 Chapter 9
Since CD  5, 0, it is along the positive x-axis. Thus, CD has a magnitude of 2
5  or about
Thus, CD has a magnitude of 5 units and a 4.5 units and a direction of 296.6°.
direction of 0°. 24. Find the magnitude using the distance formula.
22. Find the magnitude using the distance formula. CD 

(x2  
x1)2 
(y2 
y1)2
CD 
(x
 2 x1)2 
(y2 y1)2 
(2  0)2 [4 (7)]2

 
[2  (2)]2
 (5 
1)2 
13

 4
2   3.6
 5.7 Graph CD to determine how to find the direction.
Graph CD to determine how to find the direction. Draw a right triangle that has CD as its
Draw a right triangle that has CD as its hypotenuse and an acute angle at C.
hypotenuse and an acute angle at C. y
y x
D(2, 5)

C
x C

y y
tan C ¬
x 
2
x
1
y2  y1 2 1
tan C ¬
x2  4  (7)
x1 ¬2 
0
5 
1 ¬ 1.5
¬
2  (2)
¬1 mC  tan1 (1.5)
mC  tan1 1  56.3
 45 A vector in standard position that is equal to CD
A vector in standard position that is equal to CD forms a 56.3° angle with the negative x-axis in
forms a 45° angle with the positive x-axis in the the second quadrant. So it forms a 180  56.3
first quadrant. or 123.7° angle with the positive x-axis.
Thus, CD has a magnitude of 4
 2 or about Thus, CD has a magnitude of
 13 or about
5.7 units and a direction of 45°. 3.6 units and a direction of 123.7°.
25. Find the magnitude using the distance formula.
23. Find the magnitude using the distance formula.
CD 
(x
CD 
(x
 2 x1)2 (y2 y1)2
2x1)2 (y2 y1)2

[6  (
8)]2 
 [0  (7)]2

[3  (5)]2 (6 10)2
 7
5
 2
5   15.7
 4.5
Graph CD to determine how to find the direction.
Graph CD to determine how to find the direction.
Draw a right triangle that has CD as its
Draw a right triangle that has CD as its
hypotenuse and an acute angle at C.
hypotenuse and an acute angle at C.
C y
y
D x

D
C

x y y
tan C ¬
x 
2
x
1
y2  y1 2 1
tan C ¬
x2 x1 0  (7)
¬  
6  (8)
6  10
¬
3  (5) ¬0.5
¬2 mC  tan1 (0.5)
mC  tan1 (2)  26.6
 63.4 A vector in standard position that is equal to CD
A vector in standard position that is equal to CD forms a 26.6° angle with the positive x-axis in the
forms a 63.4° angle with the positive x-axis in the first quadrant.
fourth quadrant. So it forms a 360  63.4 or Thus, CD has a magnitude of 7
 5 or about
296.6° angle with the positive x-axis. 15.7 units and a direction of 26.6°.

Chapter 9 324
y
26. Find the magnitude using the distance formula. tan θ  x
CD 
(x
 2
x1)2 
(y2 y1)2 15

1
2


(2 
10)2 
[2 
(3)]2
 5


145
 4
 12.0 mθ  tan1 5 4

Graph CD to determine how to find the direction.  51.3
Draw a right triangle that has CD as its u forms a 51.3° angle with the negative x-axis in
hypotenuse and an acute angle at C. the second quadrant. So, it forms a 180  51.3
y or 128.7° angle with the positive x-axis.
Thus, u has a magnitude of 3
41  or about
x 19.2 units and a direction of about 128.7°.
D 29. Find the magnitude using the Pythagorean
C Theorem.
v 
x2  y2

(25)
220)  (2
 5
41

 32.0
y y Vector v lies in the third quadrant. Find the
tan C ¬
x 
2
x
1
direction.
2 1
2  (3) y
¬   tan θ  x
2  10
1 2
0
¬  
25
12
mC  tan1  1
12    4

5
 4.8 mθ  tan1 45

A vector in standard position that is equal to CD  38.7
forms a 4.8° angle with the negative x-axis in the v forms a 38.7° angle with the negative x-axis in
second quadrant. So it forms a 180  4.8 or 175.2° the third quadrant. So, it forms a 180  38.7 or
angle with the positive x-axis. 218.7° angle with the positive x-axis.
Thus, CD has a magnitude of
 145 or about Thus, v has a magnitude of 5
41  or about
12.0 units and a direction of 175.2°. 32.0 units and a direction of about 218.7°.
27. Find the magnitude using the Pythagorean 30. Find the magnitude using the Pythagorean
Theorem. Theorem.
t 
x2  y2 w 
x 
2  y2


72  242 
36
 2 (15)2
 25  39
Vector t lies in the first quadrant. Find the
Vector w lies in the fourth quadrant. Find the
direction.
y direction.
tan θ  x y
tan θ  x
24
  1
5
7 
36
mθ  tan1 7 
24

5
 
 73.7 12
v forms a 73.7° angle with the positive x-axis in mθ  tan1  5
12
the first quadrant.  22.6
Thus, v has a magnitude of 25 units and a w forms a 22.6° angle with the positive x-axis in
direction of about 73.7°. the fourth quadrant. So, it forms a 360  22.6 or
28. Find the magnitude using the Pythagorean 337.4° angle with the positive x-axis.
Theorem. Thus, w has a magnitude of 39 units and a
direction of about 337.4°.
u 
x 
2  y2
31. Find the magnitude using the Distance Formula.

(12)
2  152
 3
41
MN 
(x 2x1)2 (y2 y1)2
 19.2 
[9
 (3)]
2  (9 3)2
Vector u lies in the second quadrant. Find the  6
2 
direction.  8.5
Graph MN to determine how to find the
direction. Draw a right triangle that has MN as
its hypotenuse and an acute angle at M.

325 Chapter 9
N Graph MN to determine how to find the direction.
y
Draw a right triangle that has MN as its
hypotenuse and an acute angle at M.
y

M M

x x
N
y y
tan M ¬
x 
2
x
1
2 1
9 3
¬
9  (3) y y
¬1 tan M ¬
x 
2
x
1
2 1

mM  tan1 (1) 2  2


¬
12
0
 45
¬1

3
A vector in standard position that is equal to MN
forms a 45° angle with the negative x-axis in the mM  tan1 13

second quadrant. So it forms a 180  45 or 135°  18.4
angle with the positive x-axis. A vector in standard position that is equal to MN
 has a magnitude of 6
2
Thus, MN  or about forms an 18.4° angle with the negative x-axis in
8.5 units and a direction of 135.0°. the third quadrant. So it forms a 180  18.4 or
32. Find the magnitude using the Distance Formula. 198.4° angle with the positive x-axis.
MN 
(x Thus, MN has a magnitude of 4
10  or about
2x1)2 (y2 y1)2
12.6 units and a direction of 198.4°.

 (2  8
)2  (5
 1)2
34. Find the magnitude using the Distance Formula.
 2
13
 7.2
MN 
(x 2x1)2 (y2 y1)2
Graph MN to determine how to find the direction. 
[6
1)]
 ( 2 (8 7)2
Draw a right triangle that has MN as its 
274

hypotenuse and an acute angle at M.  16.6
y Graph MN to determine how to find the
direction. Draw a right triangle that has MN as
its hypotenuse and an acute angle at M.
N
y
M

M
x x

y y
tan M ¬
x 
2
x
1
2 1 N
51
¬
2  8
2 y y
¬3 tan M ¬
x 
2
x
1
2 1

mM  tan1 2


3
 ¬8 
 7
6  (1)
 33.7 15
¬7
A vector in standard position that is equal to MN
forms a 33.7° angle with the negative x-axis in mM  tan1 7 
15
the second quadrant. So it forms a 180  33.7 or  65.0
146.3° angle with the positive x-axis. A vector in standard position that is equal to MN
Thus, MN has a magnitude of 2
13  or about forms an 65.0° angle with the positive x-axis in
7.2 units and a direction of 146.3°. the fourth quadrant. So it forms a 360  65.0 or
33. Find the magnitude using the Distance Formula. 295.0° angle with the positive x-axis.
MN 
(x 2x1)2 (y2 y1)2 Thus, MN has a magnitude of
274  or about

(12
 
0)2 
(2
 2)2 16.6 units and a direction of 295.0°.
 4
10

 12.6

Chapter 9 326
35. Find the magnitude using the Distance Formula. 37. First, graph ABC. Next, translate each vertex by
MN 
(x
 2
x1)2 
(y2 y1)2 a , 6 units down. Connect the vertices to form
ABC.

[11)]
 ( 2 2(1) 102
y A
 2
122

 22.1 4
Graph MN to determine how to find the C A
direction. Draw a right triangle that has MN as 8 4 O 4 8x
its hypotenuse and an acute angle at M.
4
y C
B
12 8
M
12
B
4 x 38. First, graph DEF. Next, translate each vertex by
b, 3 units left and 9 units down. Connect the
vertices to form DEF.
N
y E
D
y2  y1 4
tan M ¬
x2 x1
12  10
¬ 
1  (1) 12 8 4 O 4 x
¬11 4 E F
D
mM  tan1 (11)
 84.8 8
A vector in standard position that is equal to MN
forms an 84.8° angle with the positive x-axis in 12
F
the fourth quadrant. So it forms a 360  84.8 or
275.2° angle with the positive x-axis.3 39. First, graph square GHIJ. Next, translate each
Thus, MN has a magnitude of 2
122  or about vertex by c , 3 units right and 8 units down.
22.1 units and a direction of 275.2°. Connect the vertices to form square GHIJ.
36. Find the magnitude using the Distance Formula. J y
4
MN 
(x 2x1)2 (y2 y1)2 I G
O

 [6  (4)]2
 (4  0)2 12 8 4 x
J
 2
5  H 4
 4.5
Graph MN to determine how to find the direction. I 8
G
Draw a right triangle that has MN as its
H
hypotenuse and an acute angle at M.
40. First, graph quadrilateral KLMN. Next, translate
y
M each vertex by x, 10 units left and 2 units up.
x Connect the vertices to form quadrilateral
KLMN.
K y
N L N K
N 8
L
4

y y 12 8 4 O 4 x
tan M ¬
x 
2
x
1
M M
2 1
4  0
¬6  (4)
¬2
mM  tan1 (2)
 63.4
A vector in standard position that is equal to MN
forms a 63.4° angle with the negative x-axis in
the third quadrant. So it forms a 180  63.4 or
243.4° angle with the positive x-axis.
Thus, MN has a magnitude of 2
5  or about
4.5 units and a direction of 243.4°.

327 Chapter 9
41. First, graph pentagon OPQRS. Next, translate 45. First, graph quadrilateral EFGH. Next, translate
each vertex by y, 5 units left and 11 units up. each vertex by p, 6 units left and 10 units up.
Connect the vertices to form pentagon Finally, translate each vertex by q, 1 unit right
OPQRS. and 8 units down. Connect the vertices to form
S y quadrilateral EFGH.
12
O G 16 y
K G
8 12
P F
Q S F
4 8
O
R
O x 4
8 4 4
4
P E H
Q 12 8 4 O 4 x
H
42. First, graph hexagon TUVWXY. Next, translate E
each vertex by z , 18 units left and 12 units up. 46. First, graph pentagon STUVW. Next, translate
Connect the vertices to form hexagon each vertex by p, 4 units left and 5 units up.
TUVWXY. Finally, translate each vertex by q, 12 units right
y and 11 units up. Connect the vertices to form
V W 18 pentagon STUVW.
U
y T U
12 24
T X
Y 6 V 20 V
U W S W
12 6 O T 6 x 16
X
6 Y 12
T U
43. First, graph ABCD. Next, translate each 8
vertex by p, 11 units right and 6 units up. Finally, V
4
translate each vertex by q, 9 units left and 3 S W
units down. Connect the vertices to form
ABCD. O 4 8 12 x
y 47. Find the resultant vector.
a  b  5  0, 0  12
8 4 O 4 8x  5, 12
A
Find the magnitude of the resultant vector.
D A B
a  b 
x2  y2
D C B 
52  122
C 12  13
The resultant vector lies in the first quadrant.
Find the direction.
44. First, graph XYZ. Next, translate each vertex by y
p, 2 units right and 2 units up. Finally, translate tan θ  x
each vertex by q, 4 units left and 7 units down. 12
5
Connect the vertices to form XYZ.
mθ  tan1  5 
12
y Y
4  67.4
The resultant vector forms a 67.4° angle with the
Y
O x positive x-axis in the first quadrant.
4 8 12
Z Thus, a  b has a magnitude of 13 units and a
4 direction of about 67.4°.
X
Z 48. Find the resultant vector.
8
c  d  0  (8), 8  0
X  8, 8
Find the magnitude of the resultant vector.
c  d 
x2  y2

(8)2  (8)2
 8
2
 11.3

Chapter 9 328
The resultant vector lies in the third quadrant. The resultant vector lies in the first quadrant.
Find the direction. Find the direction.
y y
tan θ  x tan θ  x
 8
 8  2

4
1
 1

mθ  tan1 (1) 2
 45 mθ  tan1 1 2

The resultant vector forms a 45° angle with the  26.6
negative x-axis in the third quadrant. So it forms The resultant vector forms a 26.6° angle with the
a 180  45 or 225° angle with the positive x-axis. positive x-axis in the first quadrant.
Thus, c  d has a magnitude of 8
 2 or about Thus, w  x has a magnitude of 2
5  or about
11.3 units and a direction of 225°. 4.5 units and a direction of about 26.6°.
49. Find the resultant vector. 52. Find the resultant vector.
e  f  4  7, 0  (4) y  z  9  (10), 10  (2)
 3, 4  1, 12
Find the magnitude of the resultant vector. Find the magnitude of the resultant vector.
e  f 
 x2  y2 y  z 
x2  y2

 32  (
4)2 
(1)2  (12)2
5 
145
The resultant vector lies in the fourth quadrant.  12.0
Find the direction. The resultant vector lies in the third quadrant.
y Find the direction.
tan θ  x y
 4 tan θ  x
 3 1
2

mθ  tan1 4 3
 1
 53.1  12
The resultant vector forms a 53.1° angle with the mθ  tan1 (12)
positive x-axis in the fourth quadrant. So it forms  85.2
a 360  53.1 or 306.9° angle with the positive The resultant vector forms an 85.2° angle with
x-axis. the negative x-axis in the third quadrant. So it
Thus, e  f has a magnitude of 5 units and a forms a 180  85.2 or 265.2° angle with the
direction of about 306.9°. positive x-axis.
50. Find the resultant vector. Thus, y  z has a magnitude of
 145 or about
u  v  12  0, 6  6 12.0 units and a direction of about 265.2°.
 12, 12 53. The first path of the freighter is due east, so a
Find the magnitude of the resultant vector. vector representing the path lies on the positive
u  v 
 x2  y2 x-axis and is 35 units long. The second path of the
freighter is due south, so a vector representing

 122  122
this path begins at the tip of the first vector and
 12
 2 stretches 28 units in the negative y-direction. Add
 17.0 the two vectors, 35, 0 and 0, 28.
The resultant vector lies in the first quadrant. 35  0, 0  (28)  35, 28
Find the direction. Use the Pythagorean Theorem to find the
y
tan θ  x magnitude.
12 35, 28 
 352 (28)2
 
12  44.8
1 The resultant vector lies in the fourth quadrant.
mθ  tan1 (1) Find the direction.
 45 y
tan θ  x
The resultant vector forms a 45° angle with the
2
8
positive x-axis in the first quadrant. 
35
Thus, u  v has a magnitude of 12
2  or about  4

17.0 units and a direction of 45°. 5
mθ  tan1 4
5

51. Find the resultant vector.
w  x  5  (1), 6  (4)  38.7
 4, 2 The distance the freighter traveled is about 44.8 mi,
Find the magnitude of the resultant vector. at a direction of about 38.7° south of due east.
w  x 
 x2  y2

4 2  22
 2
 5
 4.5

329 Chapter 9
54. Let the initial direction of the swimmer be the 59. Sample answer: Quantities such as velocity are
positive x-direction, and the direction that the vectors. The velocity of the wind and the velocity
current flows by the positive y-direction. The of the plane together factor into the overall flight
speed and direction of the swimmer can be plan. Answers should include the following.
represented by the vector 4.5, 0, and those of the • A wind from the west would add to the velocity
current can be represented by the vector 0, 2. contributed by the plane resulting in an overall
Add the vectors. velocity with a larger magnitude.
4.5  0, 0  2  4.5, 2 • When traveling east, the prevailing winds add
Use the Pythagorean Theorem to find the to the velocity of the plane. When traveling
magnitude. west, they detract from it.
4.5, 2 
4.52 
22 60. B; find the sum of the vectors.
 4.9
  r
q   5, 10  3, 5
The resultant vector lies in the first quadrant.
Find the direction.  8, 15
y Use the Pythagorean Theorem to find the
tan θ  x
magnitude of the vector sum.
2

4.5 q
  r 
82  1
52
 4
  17
9
The magnitude is 17.
mθ  tan1 4
9

61. D; 5b  125
 24
5b  53
The swimmer is traveling about 4.9 mph at an
So, b  3.
angle of 24°.
4b  3  43  3
55. Add the vectors representing the velocities of the  64  3
wind and jet.  192
100, 0  450, 450  100  450, 0  450
 350, 450
The resultant vector for the jet is 350, 450 mph. Page 505 Maintain Your Skills
56. From Exercise 55, the resultant vector is 62. AB  8, r  2
350, 450. Use the Dilation Theorem.
Use the Pythagorean Theorem to find the AB  r (AB)
magnitude. AB  (2)(8)
350, 450 
(350)2  4502 AB  16
 570.1 63. AB  12, r  1
2
The magnitude of the resultant is about 570.1 mph. Use the Dilation Theorem.
57. From Exercise 55, the resultant vector is
AB  r (AB)
350, 450.
Find the direction. AB  1
2 (12)

y AB  6
tan θ  x
 450 64. AB  15, r  3
350 Use the Dilation Theorem.
 9

7 AB  r (AB)
mθ  tan1 9
7
 15  (3)(AB)
 52.1 5  AB
The direction of the resultant is about 52.1° north 65. AB  12, r  1

4
of due west. Use the Dilation Theorem.
58. Sample answer: Let one vector be 1, 0. Then AB  r (AB)
the x-components of the other two vectors must
12  1
4 (AB)

sum to 1, the y-components must cancel, and
the magnitudes of the other two vectors must 48  AB
be 1. Try 1
2 for the x-components. Find the 66. Yes; the pattern is a tessellation because at the
y-components. different vertices the sum of the angles is 360°.

 2
2 The tessellation is uniform because at every
1 ¬ 1
 y 2
vertex there is the same combination of shapes
1 ¬1 and angles. The tessellation is also semi-regular
4 y
 2
since more than one regular polygon is used.
3
 ¬y2
4


 
3
2 ¬y
Three vectors with equal magnitude, the sum of


which is 0, 0, are 1, 0, 1, 
2

3
2 
, and
12, 
23 .
Chapter 9 330
2 2 2 8 8 8
  
67. Yes; the pattern is a tessellation because at the
74. 
different vertices the sum of the angles is 360°. 7 2 5 1 1 1
The tessellation is not uniform because the
number of angles at the vertices varies. 2  8 2  8 2  8
7  1 2  1 5  1
68. The opposite angles of the rhombus are
10 6 10

6 1 4
congruent.
WZY  WXY and XYZ  XWZ.
mXYZ  mXWZ  mWZY  mWXY ¬360
2mXYZ  2mWZY ¬360
75. 3 99 51 15  3(9)
3(9)
3(5) 3(1)
3(1) 3(5) 
2mXYZ  21 5 mXYZ ¬360

12
 27
27
15 3
3 15
5 mXYZ ¬360


 
mXYZ ¬150 1(4) 1(5) 1(0) 1(2)
4 5 0 2
69. The measures of all sides of the rhombus are equal. 76. 1

2 
4 4 6 0
  2
1(4)
2
1(4)
2
1(6)
2
1(0)
So, WX  YZ  12. 2 2 2 2
70. mXYZ ¬5mWZY
150 ¬5mWZY
 22 2.52 
0 1
3 0
30 ¬mWZY
mXZY ¬1 
2 mWZY
77. 42 42  238 74
¬1

2 (30)
¬15
 42 42  2(3) 2(7)
2(8) 2(4)

42 42  6 14


71. The opposite angles of the rhombus are 16 8
congruent. 
WXY  WZY
mWXY  mWZY   42166 4  8
2  14 
mWXY  30 (From Exercise 70, mWZY  30.)
72. 30  4
12 4
12 
78. 11 11  22 3
4
 (1)(2) 1(3)  (1)(4)
30 30
  1(2)
1(2)  1(2) 1(3)  1(4) 
x y
  22  22 3  4
34 
25
30  44 11
The diagonals bisect each other perpendicularly.
They also bisect the interior angles.
The lengths of the diagonals are 2x and 2y.
2x  2(30 cos 25°)
 54.4 9-7 Transformations with Matrices
2y  2(30 sin 25°)
 25.4
To the nearest tenth, the lengths of the diagonals Pages 508–509 Check for Understanding
are 25.4 cm and 54.4 cm. 1. A(3, 3)  A(3, 3)
B(4, 1) and B(1, 4) imply (x, y) → (y, x).
73. 5
3
5
2 

1 8
7 6

5  1 5  8
 
3  7 2  6  C(1, 1) and C(1, 1) imply (x, y) → (y, x).
(x, y) → (y, x) occurs with reflection in the line
4 3 y  x.

10 4  
0 1
The reflection matrix is .
1 0
2. Sample answer: The format used to represent the
transformation is different in each method, but
the result is the same. Positive values move a
figure up or right, and negative values move a
figure down or left.

331 Chapter 9
 2 2 2 2

9. The reflection matrix for a reflection in the y-axis
3. Sample answer:
1 1 1 1
is 10 0
1
. 
4. The vertex matrix for ABC is  5 3
4 1
0
2
.  The vertex matrix for HIJK is

The translation matrix is  2 2 2


.   54 1
1 6 3
3 7
.
1 1 1
Multiply the vertex matrix for HIJK by the
Find the vertex matrix for the image.
reflection matrix to find the vertex matrix of the
 5
4 1
3 0
2

2 2 2
1 1 1   
3 1 2
3 2 1
.    image.
1 0 5 1 3 7
The coordinates of the vertices of ABC are  0 1   
4 1 6 3

5 1
 
3 7
4 1 6 3 
A(3, 3), B(1, 2), and C(2, 1). The coordinates of the vertices of the image are
5. The vertex matrix for rectangle DEFG is H(5, 4), I(1, 1), J(3, 6), and K(7, 3).

13 5
3
3 3
0 0
.  10. The rotation matrix for a counterclockwise
0 1
The translation matrix is  06 0 0 0
. 
rotation of 90° is
1 0. 
6 6 6
Find the vertex matrix for the image. M
The vertex matrix for L is
3
5
. 21 
1 5 3 3 1 5 13 3
 3 3 0 0

0 0 0 0
6 6 6 6  

9 9 6 6    M
Multiply the vertex matrix for L  by the rotation
matrix to find the vertex matrix of the image.
The coordinates of the vertices of the image are
0 1 2 3
D(1, 9), E(5, 9), F(3, 6), and G(3, 6).  1 0   
1 5

1 5
2 3   
6. The vertex matrix for XYZ is
3 6 3
4 10 5
.   The coordinates of the vertices of the image are
L(1, 2) and M(5, 3).
Multiply the vertex matrix by the scale factor to 11. The rotation matrix for a counterclockwise
find the vertex matrix of the image.
6 12 6 rotation of 270° is
0 1

. 
2 3 6 3
4 10 5
  
8 20 10  1 0

The coordinates of the vertices of the image are The vertex matrix for PQR is 63 6 2
7 7
. 
X(6, 8), Y(12, 20), and Z(6, 10).
Multiply the vertex matrix for PQR by the
7. The vertex matrix for ABCD is
1 3 3 1
2 3 5 4
.   rotation matrix to find the vertex matrix of the
image.
Multiply the vertex matrix by the scale factor to
find the vertex matrix of the image.  0 1
1 0  3 7 7  
6 6 2
  
3 7
6 6 2
7

 
1 3 3 1
4  4  4 4
 The coordinates of the vertices of the image are
1
4
1 3 3 1
2 3 5 4 
1

3 5
2  4  4 1

P(3, 6), Q(7, 6), and R(7, 2).
12. The vertex matrix for quadrilateral STUV is
The coordinates of the vertices of the image are
4 2 0 2
A1      . 
4 ,  2 , B  4 ,  4 , C  4 ,  4 , and
 1 3 3 3 5 1 2 1 2
Multiply the vertex matrix by the scale factor to
D 1 
4 , 1 .
  find the vertex matrix of the image.
4 2 0 2 8 4 0 4
8. The reflection matrix for a reflection in the x-axis 2 1 2 1 2
  
2 4 2 4 
is  1 0
0 1 
. The rotation matrix for a counterclockwise
0 1
rotation of 90° is  . 
F
The vertex matrix for E is
5
1
. 24  1 0
Multiply the vertex matrix of the image due to
F
Multiply the vertex matrix for E  by the reflection dilation to find the vertex matrix of the final
matrix to find the vertex matrix of the image. image.
2 5 2 5
 1 0
0 1  4 1  
4 1    0 1
1 0

8 4 0 4
 
2 4 2 4

2 4 2 4
8 4   0 4 
The coordinates of the vertices of the image are The coordinates of the vertices of the image are
E(2, 4) and F(5, 1). S(2, 8), T(4, 4), U(2, 0) and V(4, 4).

Chapter 9 332
13. The rose bed must be dilated by a scale factor Find the vertex matrix for the image.
of 1
 6 2 6 6 6 6
2.
The vertex matrix of the rose bed is  23 6
2 1 1
  
2 2 2 2 
13 37 75 51. 
0
41
0 8
0 3 1 
Multiply the vertex matrix by the scale factor to The coordinates of the vertices of the image are
find the vertex matrix of the new rose bed plan. R(4, 1), S(0, 0), T(0, 3), and U(8, 1).

 
3 7 5 1
1 
3 7 5 1
2 1 3 7 5  
2 2 2 2 19. The vertex matrix for ABC is  65 4 3
5 7
.
1 3 7
2 2 2 5
2

Multiply the vertex matrix by the scale factor to
The new coordinates are (1.5, 0.5), (3.5, 1.5), find the vertex matrix of the image.
(2.5, 3.5), and (0.5, 2.5).
14. All dimensions have been reduced by 1 
2 , so the
2  65 4 3
5 7
  
12 8 6
10 10 14 
coordinates of the center after the changes have The coordinates of the vertices of the image are
been made will be 12 (4, 4)  (2, 2).
 A(12, 10), B(8, 10), and C(6, 14).
1 0 2
20. The vertex matrix for DEF is
4 1 3
.  
Pages 509–511 Practice and Apply Multiply the vertex matrix by the scale factor to
4 1
F
15. The vertex matrix for E is .   find the vertex matrix of the image.

 
1 3  0 3 1 2
1
The translation matrix is 25 25.
1

3  2
3 4

0
1
3
4 1
3 3 1
The coordinates of the vertices of the image are
Find the vertex matrix for the image.
4 1 2 2 6 3 D 1
  4  
1 
2
3 ,  3 , E 0,  3 , and F  3 , 1 .  
 1 3

5 5
 
6 8    21. The vertex matrix for quadrilateral GHIJ is
The coordinates of the vertices of the image are 4 4 6
E(6, 6) and F(3, 8). 2 6 8 10
6
. 
3 4 7
16. The vertex matrix for JKL is
5 8 5
.   Multiply the vertex matrix by the scale factor to
find the vertex matrix of the image.
The translation matrix is
3 3
4 4
.  3
4  1

2  42 46 6
8
6
10

2 2
 
1 3
3 3
4 5 
Find the vertex matrix for the image.
The coordinates of the vertices of the image are
35 4
8  
7
5

3 3 3
4 4 4

6
1   1
4
4
1  G(2, 1), H(2, 3), I(3, 4), and J(3, 5).
22. The vertex matrix for pentagon KLMNO is
The coordinates of the vertices of the image are
J(6, 1), K(1, 4), and L(4, 1).
2 2 2 2 21 3 6 4 3
1 1 3 3
. 
17. The vertex matrix for MNOP is
7 9 7 5
.   Multiply the vertex matrix by the scale factor to
The translation matrix is
3 3 3 3
6 6 6 6
.   find the vertex matrix of the image.

Find the vertex matrix for the image. 4 21 3 6 4 3


1 1 3 3 
27 2
9
2 2
7 5
  
3 3 3 3
6 6 6 6   84 12 24 16 12
4 4 12 12 
  1
1
5
3
5 1
1 1  The coordinates of the vertices of the image are
K(4, 8), L(12, 4), M(24, 4), N(16, 12),
The coordinates of the vertices of the image are and O(12, 12).
M(1, 1), N(5, 3), O(5, 1), and P(1, 1). 23. The reflection matrix for a reflection in the y-axis
1 0
18. The vertex matrix for trapezoid RSTU is
6 2
is 
0 1
. 
 23 6
2 1 1
. 
6 6 6 6
The vertex matrix for XY
 is
2 4
2 1
.  
The translation matrix is  2 2 2 2
.  Multiply the vertex matrix for XY by the
reflection matrix to find the vertex matrix of the
image.

333 Chapter 9
10   2 4
  
0 2 4 Find the vertex matrix for the image.

1  2 1 2 1 3 4 4 4 7 3 1
The coordinates of the vertices of the image
 3
1
3 2
3
  
1 1 1

2 2 3  
are X(2, 2), and Y(4, 1). The coordinates of the vertices of the image are
24. The reflection matrix for a reflection in the line V(7, 2), W(3, 2), and X(1, 3).

 
0 1 29. The rotation matrix for a counterclockwise
y  x is .
0 1
1 0
5 0 1
rotation of 90° is
1 0
. 
The vertex matrix for ABC is  3 5 3
. 
Multiply the vertex matrix for ABC by the
The vertex matrix for VWX is 33 1 3
3 2.

reflection matrix to find the vertex matrix of the Multiply the vertex matrix for VWX by the
image. rotation matrix to find the vertex matrix of the
0 1 3 5 3 image.
 01 10  35     0 1 3 1 3 3 2
5 3 5 0 1
The coordinates of the vertices of the image are
1 0
 3 3 2
3

3  
1 3 
A(3, 5), B(5, 0), and C(3, 1). The coordinates of the vertices of the image are
25. The reflection matrix for a reflection in the x-axis V(3, 3), W(3, 1), and X(2, 3).

is
1

0 1
0

. 30. The reflection matrix for a reflection in the line

The vertex matrix for quadrilateral DEFG is y  x is 


0 1
1 0
. 
4 2 3 3
 5 6 1 4
.  The vertex matrix for VWX is 33 1 3
3 2.
Multiply the vertex matrix for quadrilateral Mutiply the vertex matrix for VWX by the
DEFG by the reflection matrix to find the vertex reflection matrix to find the vertex matrix of the
matrix of the image. image.
4 2 3 3 4 2 3 3
1 0
0 1    5 6 1 4

5 6 1   4  0 1
1 0 
3 1
 3 3 2
3

3 3 2
 
3 1 3 
The coordinates of the vertices of the image are The coordinates of the vertices of the image are
D(4, 5), E(2, 6), F(3, 1), and G(3, 4). V(3, 3), W(3, 1), and X(2, 3).
26. The reflection matrix for a reflection in the y-axis 31. The vertex matrix for polygon PQRST is
1 0
is 0 1
.  1 4 2 0 2
1 1 4 4 1
. 
The vertex matrix for quadrilateral HIJK is

 9 2 4 2
. 
The translation matrix is 23 3 3 3 3
2 2 2 2
. 
1 6 3 4
Find the vertex matrix for the image.
Multiply the vertex matrix for quadrilateral HIJK
1 4 2 0 2
by the reflection matrix to find the vertex matrix
of the image.
1 1 4 4 1

3
 3 3 3 3
2 2 2 2 2 
1 0
  9 2 4 2
0 1  1 6 3 4

9 2 4 2
 
1 6 3 4   32 1
1
1
2
3 5
2 1 
The coordinates of the vertices of the image are The coordinates of the vertices of the image are
H(9, 1), I(2, 6), J(4, 3), and K(2, 4). P(2, 3), Q(1, 1), R(1, 2), S(3, 2),
3 1 and T(5, 1).
27. The vertex matrix for VWX is
3 3 2
3
.   32. The vertex matrix for polygon PQRST is
1 4 2 0 2
Multiply the vertex matrix by the scale factor to
find the vertex matrix of the image. 1 1 4 4 1
. 
 
2 2
 2 Multiply the vertex matrix by the scale factor to
3
2
3 3
1 3
3 2

2
 3
4
2 3
find the vertex matrix of the image.
1 4 2 0 2 0 6
The coordinates of the vertices of the image are 3 
1 1 4 4 1
  
3 12 6
3 3 12 12 3 
V(2, 2), W 2   4
3 , 2 , and X 2,  3 .
   The coordinates of the vertices of the image are
P(3, 3), Q(12, 3), R(6, 12), S(0, 12),
28. The vertex matrix for VWX is 33 1 3
3 2. and T(6, 3).

The translation matrix is 4


1
4 4
1 1
. 

Chapter 9 334
33. The reflection matrix for a reflection in the y-axis 37. The rotation matrix for a counterclockwise
1 0 0 1
is 0 1
.  rotation of 90° is
1 0
.  
 21 
The vertex matrix for polygon PQRST is 6 5 1
The vertex matrix for STUV is .
1 4 2 0 2 1 3 3
1 1 4 4 1
.  Multiply the vertex matrix for STUV by the
Multiply the vertex matrix for PQRST by the rotation matrix to find the vertex matrix of the
reflection matrix to find the vertex matrix of the iamge.
0 1 1 1 3 3
     
image. 2 6 5 1

1 0 1 4 2 0 2 0  1 1 3 3
   
1 2 6 5 1
0 1  1 1 4 4 1 The coordinates of the vertices of the image are
0 2 S(1, 2), T(1, 6), U(3, 5), and V(3, 1).
 11 4
1
2
4 4 1  38. The rotation matrix for a counterclockwise
The coordinates of the vertices of the image are
P(1, 1), Q(4, 1), R(2, 4), S(0, 4),
rotation of 270° is
0 1
1 0
.  
and T(2, 1). The vertex matrix for pentagon ABCDE is
1 4 4 5
34. The rotation matrix for a counterclockwise
1 0
 1
6
0 8 10 3
. 
rotation of 180° is
0 1  .  Multiply the vertex matrix for pentagon ABCDE
The vertex matrix for polygon PQRST is by the rotation matrix to find the vertex matrix of
the image.
1 4 2 0 2
1 1 4 4 1
.   0 1 1
 
6 4 4 5

1 0  1 0 8 10 3
Multiply the vertex matrix for PQRST by the
8 10 3
rotation matrix to find the vertex matrix of the
image.
  11 0
6
4 4 5 
10 1 4 2
  
0 0 2 The coordinates of the vertices of the image are
 1 1 4 A(1, 1), B(0, 6), C(8, 4), D(10, 4),
1 4 1
and E(3, 5).
4 2 0 2
  11
1 4 4 1  39. The vertex matrix for polygon ABCDEF is
3 2 2 3 2 2
The coordinates of the vertices of the image are
P(1, 1), Q(4, 1), R(2, 4), S(0, 4),
 1 4 4 1 2 2
. 
and T(2, 1). Multiply the vertex matrix by the scale factor to
find the vertex matrix of the image.
35. The rotation matrix for a counterclockwise
3 2 2 3 2 2
rotation of 90° is
0 1
1 0  .  1
3  1 4 4 1 2 2 

 
3 1 2 2 2 2
N
The vertex matrix for M is
10
. 121  

3 3 1 3 3
1 4
 4   2
1 2
N
Multiply the vertex matrix for M  by the rotation 3 3 3 3 3 3
matrix to find the vertex matrix of the image. The reflection matrix for a reflection in the x-axis
0 1 12 3 1 10
1  
0  1 10
  
12 3  is  10
1
0
. 
The coordinates of the vertices of the image are Multiply the vertex matrix of the image by the
M(1, 12), and N(10, 3). reflection matrix to find the vertex matrix of the
36. The rotation matrix for a counterclockwise final image.

 
1 0 1 2 2 2 2
rotation of 180° is
0 1
.   1 0
0 1  

3 3 1 3 3
1 4 4 1 3 2
2  
3 3 3 3 3
The vertex matrix for PQR is
5
 1 1
. 
 
1 2 4 1 3 2
2
3
 1 2
3 23

Multiply the vertex matrix for PQR by the 
1

3 4 4
3 3 3
 1 2
3
2
3
rotation matrix to find the vertex matrix of the
image. The coordinates of the vertices of the final image
1 0 5 1 1
 5 1
 
0 1  1 2 4
1

1 2  
4  
are A 1, 1   2 4   
2 4
3 , B  3 ,  3 , C 3 ,  3 ,


D1, 1
3 , E 3 , 3 , and F 3 , 3 .
The coordinates of the vertices of the image are
 2 2 2 2
P(5, 1), Q(1, 2), and R(1, 4).

335 Chapter 9
40. The vertex matrix for polygon ABCDEF is Multiply the vertex matrix for ABCDEF by the
rotation matrix to find the vertex matrix of the
31 2
4
3 2 2
2
1 2 2
4
.  image.

The translation matrix is 10 10   31 2


4
2
4
3 2 2
1 2 2 
5 5 5 5 5 5
 .  2 2 3 2 2
 
2 2 2 2 2 2 3
 1 4 4 1 2 2
Find the vertex matrix for the image.
3 2 2 3 2 2 5 5 5 5 5 5 Multiply the vertex matrix of the image by the
 1 4 4 1 2 2
  
2 2 2 2 2 2  scale factor to find the vertex matrix of the final
image.
 83 76 36 23 30 70 2  3 2 2 3 2 2
1 4 4 1 2 2 
The rotation matrix for a counterclockwise

rotation of 90° is
0 1
.   62 4
8
4
8
6 4 4
2 4 4 
1 0
Multiply the vertex matrix of the image by the The coordinates of the vertices of the final
rotation matrix to find the vertex matrix of the image are A(6, 2), B(4, 8), C(4, 8), D(6, 2),
final image. E(4, 4), and F(4, 4).
0 1 8 7 3 2 3 7
 1 0   3 6 6 3 0 0  43. Each footprint is reflected in the y-axis, then
translated up two units.

  3
6 6 3 0 0
8
7 3 2 3 7  44. Reflect in the y-axis using the matrix 10 0
1.

The coordinates of the vertices of the final image Translate up 2 units using
0
2
. 
are A(3, 8), B(6, 7), C(6, 3),
D(3, 2), E(0, 3), and F(0, 7). Combine the two operations into
1 0
41. The reflection matrix for a reflection in the line
 0 1  y
x
  

0
2
.
y  x is
0 1
1 0
.  45. Imagine the y-axis in the middle of the plan. Then
a reflection in the y-axis could be used to create a
The vertex matrix for polygon ABCDEF is
floor plan with the garage on the left. The
3 2 2 3 2 2
 1 4 4 1 2 2
.  reflection matrix is
1 0
 . 
0 1
Multiply the vertex matrix for ABCDEF by the
46. A counterclockwise rotation of 90° could be used
reflection matrix to find the vertex matrix of the
to create a floor plan with the house facing east.
image.
0 1
  
0 1
1 0 
3 2 2 3 2 2
1 4 4 1 2 2  The rotation matrix is
1 0
.  
47. A reflection in the line y  x transforms (x, y)
1 2 2
  31 4
2
4
2 3 2 2  into (y, x). The matrix that performs this
0 1
operation is 
1 0
. 
The translation matrix is  14 1 1 1 1 1
4 4 4 4 4
.  48. Matrices make it simpler for movie makers to
Find the vertex matrix for the final image. move figures. Answers should include the
4 4 1 2 2 following.
 1
3 2 2 3 2 2

1 1 1 1 1 1
4 4 4 4 4 4    • By using a succession of matrix
transformations, an object will move about in a
2 1 1
  21 5
2
5
6 7 6 2  scene.
• Sample answer: programming the animation in
The coordinates of the vertices of the final image a screen saver
are A(2, 1), B(5, 2), C(5, 6), D(2, 7), E(1, 6), 49. The rotation matrix for a 90° clockwise rotation is
and F(1, 2). equivalent to a 270° counterclockwise rotation, or

 10 
42. The rotation matrix for a counterclockwise 1
.
1 0
rotation of 180° is
0 1  .  0
50. B; since 26% are action movies and 14% are
The vertex matrix for polygon ABCDEF is comedies, 60% of the movie titles are neither
3 2 2 3 2 2 action movies nor comedies.
 1 4 4 1 2 2
.  0.60(2500)  1500
So, 1500 movie titles are neither action movies
nor comedies.

Chapter 9 336
180(n  2) 180(8  2)
Page 511 Maintain Your Skills  ¬
n 8
51. First, graph ABC. Next, translate each vertex by ¬135
, 1 unit left and 5 units down. Connect the
v 57. The measure of an exterior angle of a regular
vertices to form ABC. 360
polygon is given by n. The measure of an
y B 360
8 exterior angle of a 10-sided polygon is  
10 or 36.
Use the Interior Angle Formula to find the
4
B interior angle of a regular 10-sided polygon.
A 180(n  2) 180(10  2)
 ¬
8 4 O 4 8x n 10
A ¬144
4 C
58. The two right triangles are similar. So, using the
DE CD
AB  BC .
definition of similar polygons, 
8 
C
Solve for DE.
DE CD
AB  BC
 
52. First, graph quadrilateral DEFG. Next, translate
CD
, 7 units left and 8 units up.
each vertex by w DE  (AB)
BC
Connect the vertices to form quadrilateral 34
DE (1.75) .5
DEFG. 0.75
DE  80.5
E y
The tree is 80.5 m tall.
8
F
D 4
E
G Chapter 9 Study Guide and Review
8 4 O 4 x
F
4 D Vocabulary and Concept Check
1. false; center
G
2. true
53. Compare WX and WX. Note that the scale factor 3. false; component form
is negative since the image appears on the
4. false; magnitude
opposite side of the center with respect to the
preimage. 5. false; center of rotation
image length 6. true
scale factor ¬preimagelength 7. false; scale factor
2un
its
r ¬ 4units
 8. false; resultant vector
r ¬1

2
Lesson-by-Lesson Review
Since 0 r 1, the dilation is a reduction. 9. Use the vertical grid lines to find images of each
54. The measure of an exterior angle of a regular vertex of ABC so that each vertex of the image
360
polygon is given by  n. The measure of an is the same distance from the x-axis as the
360 vertex of the preimage or use (a, b) → (a, b)
exterior angle of a 5-sided polygon is  5 or 72.
Use the Interior Angle Formula to find the A(2, 1) → A(2, 1)
interior angle of a regular 5-sided polygon. B(5, 1) → B(5, 1)
180(n  2) 180(5  2) C(2, 3) → C(2, 3)
 ¬
n 5 Draw triangles ABC and ABC.
¬108
y C
55. The measure of an exterior angle of a regular
360
polygon is given by  n. The measure of an B
36
0
exterior angle of a 6-sided polygon is 6 or 60.
A
Use the Interior Angle Formula to find the O A x
B
interior angle of a regular 6-sided polygon.
180(n  2) 180(6  2)
 ¬ C
n 6
¬120
56. The measure of an exterior angle of a regular
360
polygon is given by  
n . The measure of an
36
0
exterior angle of a 8-sided polygon is 8 or 45.
Use the Interior Angle Formula to find the
interior angle of a regular 8-sided polygon.

337 Chapter 9
10. Use the transformation (a, b) → (b, a). 14. This translation moved each vertex 1 unit to the
W(4, 5) → W(5, 4) Y(3, 3) → Y(3, 3) right and 3 units down.
X(1, 5) → X(5, 1) Z(6, 3) → Z(3, 6) X(2, 5) → X(2  1, 5  3) or X(3, 2)
Draw parallelograms WXYZ and WXYZ. Y(1, 1) → Y(1  1, 1  3) or Y(2, 2)
y Z(5, 1) → Z(5  1, 1  3) or Z(6, 2)
W X
Draw XYZ and XYZ.
y X
Z Y

X
O X x
Y Z
Y O x

W Y Z
15. y
Z B
11. Use the vertical grid lines to find images of each
vertex of rectangle EFGH such that each vertex of D C
the image is the same distance from the line x  1
as the vertex of the preimage.
E(4, 2) → E(6, 2) G(0, 4) → G(2, 4) O x
F(0, 2) → F(2, 2) H(4, 4) → H(6, 4)
Draw rectangles EFGH and EFGH. C D
y x 1

x
B
O
E F F E Reflection in x-axis: B(3, 5) → (−3, −5)
C(3, 3) → (3, 3)
H D(5, 3) → (5, 3)
H G G
Reflection in the y-axis: (3, 5) → B(3, 5)
12. This translation moved each vertex 4 units to the
(3, 3) → C(3, 3)
left and 4 units down.
(5, 3) → D(5, 3)
E(2, 2) → E(2  4, 2  4) or E(2, 2)
The angle of rotation is 180°.
F(6, 2) → F(6  4, 2  4) or F(2, 2)
G(4, 2) → G(4  4, 2  4) or G(0, 6) 16. y y x
H(1, 1) → H(1  4, 1  4) or H(3, 5) F
Draw quadrilaterals EFGH and EFGH.
y E
H
F O G
G x
H
OH x
F
E
F G
Reflection in line y  x: F(0, 3) → (3, 0)
H G(1, 0) → (0, 1)
G H(4, 1) → (1, 4)
13. This translation moved each vertex 2 units to the Reflection in line y  x: (3, 0) → F(0, 3)
right and 4 units up. (0, 1) → G(1, 0)
S(3, 5) → S(3  2, 5  4) or S(1, 1). (1, 4) → H(4, 1)
T(1, 1) → T(1  2, 1  4) or T(1, 3) The angle of rotation is 180°.
Draw ST and S
T
.
y T

T O x
S

Chapter 9 338
17. y 24. No; let m1 represent one interior angle of the
N regular pentagon. Use the Interior Angle
M Formula.
180(n  2)
m1 ¬n
N
180(5  2)
M ¬5
L L
¬108
O x Since 108 is not a factor of 360, a pentagon will
not tessellate the plane.
25. Yes; the measure of an interior angle of an
equilateral triangle is 60, which is a factor of 360,
so an equilateral triangle will tessellate the plan.
26. No; let m1 represent one interior angle of the
regular decagon. Use the Interior Angle Formula.
Reflection in line y  x: L(2, 2) → (2, 2) 180(n  2)
m1 ¬n
M(5, 3) → (3, 5)
180(10  2)
N(3, 6) → (6, 3) ¬10 
Reflection in the x-axis: (2, 2) → L(2, 2) ¬144
(3, 5) → M(3, 5) Since 144 is not a factor of 360, a decagon will not
(6, 3) → N(6, 3) tessellate the plane.
The angle of rotation is 90° counterclockwise. 27. CD  8, r  3
18. The figure has rotational symmetry of order 9 Use the Dilation Theorem.
because there are 9 rotations less than 360° CD  r (CD)
(including 0 degrees) that produce an image CD  (3)(8)
indistinguishable from the original. CD  24
magnitude ¬360°
order
28. CD  23 , r  6

36 0° Use the Dilation Theorem.
¬9
CD  r (CD)
¬40°
CD  (6)23

The magnitude of the symmetry is 40°.
36

CD  4
19. magnitude ¬ order 29. CD  24, r  6
360°
¬ 9
Use the Dilation Theorem.
¬40° CD ¬ r (CD)
The magnitude of the symmetry is 40°. 24 ¬(6)(CD)
4 ¬CD
Vertex 2 is moved 5 positions, or 5(40°)  200°. 10
30. CD  60, r   3
360°
20. magnitude ¬ order Use the Dilation Theorem.
360° CD ¬ r (CD)
¬ 9
¬40° 1
60 ¬ 0
3 (CD)
The magnitude of the symmetry is 40°.
18 ¬CD
Divide 280° by the magnitude of the rotational
31. CD  12, r  5
6

symmetry.
280° Use the Dilation Theorem.
40°  7 positions

CD  r (CD)
Vertex 5 is moved 7 positions, or to the original
position of vertex 7. CD  5
6 (12)

21. Yes; the pattern is a tessellation because at the CD  10
different vertices the sum of the angles is 360°. 32. CD  55 5
The tessellation is not uniform because the 2 ,r 4
Use the Dilation Theorem.
number of angles at the vertices varies.
CD ¬ r (CD)
22. Yes; the pattern is a tessellation because at the
different vertices the sum of the angles is 360°.
5
 5
 
5
2 ¬ 4 (CD)
The tessellation is uniform because at every 22 ¬CD
vertex there is the same combination of shapes
33. Find P, Q, and R using the scale factor, r  2.
and angles. The tessellation is also regular since
it is formed by only one type of regular polygon. Preimage (x, y) Image (2x, 2y)
23. Yes; the pattern is a tessellation because at the P(1, 3) P(2, 6)
different vertices the sum of the angles is 360°. The Q(2, 2) Q(4, 4)
tessellation is uniform because at every vertex there
R(1, 1) R(2, 2)
is the same combination of shapes and angles.

339 Chapter 9
34. Find E, F, G, and H using the scale factor, Graph AB to determine how to find the direction.
r  2. Draw a right triangle that has AB as its
Preimage (x, y) Image (2x, 2y) hypotenuse and an acute angle at A.
E(3, 2) E(6, 4) y
F(1, 2) F(2, 4)
A
G(1, 2) G(2, 4)
H(3, 2) H(6, 4)
35. Find the change in x-values and the x
corresponding change in y-values. B
AB ¬x2  x1, y2  y1
¬0  (3), 2  (2)
¬3, 4 y y
36. Find the change in x-values and the tan A ¬
x 
2
x
1
2 1
corresponding change in y-values. 2  5
¬ 
5  8
CD ¬x2  x1, y2  y1 7
¬
13
¬4  4, 2  (2)
7
¬8, 4 mA ¬tan1 
13
37. Find the change in x-values and the ¬28.3
corresponding change in y-values. A vector in standard position that is equal to AB
EF ¬x2  x1, y2  y1 forms a 28.3° angle with the negative x-axis in
the third quadrant. So it forms a 180  28.3 or
¬1  1, 4  (4)
208.3° angle with the positive x-axis.
¬0, 8
Thus, AB has a magnitude of
218  or about
38. Find the magnitude using the Distance Formula. 14.8 units and a direction of about 208.3°.
AB ¬
 (x2  
x1)2 
(y2 y1)2 40. Find the magnitude using the Distance Formula.

 [9  (6)]
2  (3  4)2 AB ¬
(x2  
x1)2 
(y2 y1)2

58 ¬
[15  (14
)]2  ( 2
5  2)
¬7.6 ¬
890

Graph AB to determine how to find the direction. ¬29.8
Draw a right triangle that has AB as its
Graph AB to determine how to find the direction.
hypotenuse and an acute angle at A.
Draw a right triangle that has AB as its
y
hypotenuse and an acute angle at A.
A
y

10
x x
A
10 20
10 B
B

y y
tan A ¬
x 
2
x
1
2 1 y y
3  4 tan A ¬
x 
2
x
1
¬
9 
(6) 2 1
5  2
¬
15  (
¬7

3
14)
7
¬
mA ¬tan1 7 
3
29
¬66.8 mA ¬tan1 7
29 
A vector in standard position that is equal to AB ¬13.6
forms a 66.8° angle with the negative x-axis in A vector in standard position that is equal to AB
the third quadrant. So it forms a 180  66.8 or forms a 13.6° angle with the positive x-axis in
246.8° angle with the positive x-axis. the fourth quadrant. So it forms a 360  13.6 or
Thus, AB has a magnitude of
 58 or about 346.4° angle with the positive x-axis.
7.6 units and a direction of about 246.8°. Thus, AB has a magnitude of
890  or about
39. Find the magnitude using the Distance Formula. 29.8 units and a direction of about 346.4°.
AB ¬
(x2  
x1)2 
(y2 y1)2 41. Find the magnitude using the Distance Formula.
AB ¬
(x
 2x1)2 
(y2  y1)2

(5 
8)2 
(2 
5)2

218
 ¬
(45
  16)2
 (0
 40)2
¬14.8 ¬
5321

¬72.9

Chapter 9 340
Graph AB to determine how to find the direction.
Draw a right triangle that has AB as its
The vertex matrix for DEF is 3
2
0 2
5 4
. 
hypotenuse and an acute angle at A. Multiply the vertex matrix for DEF by the
y A rotation matrix to find the vertex matrix of the
20 image.
3 0 2 2 5 4
10  0 1
1 0
  
2 5 4
  
3 0 2 
The coordinates of the vertices of the image are
B 10 10 20x
D(2, 3), E(5, 0), and F(4, 2).
10

20
46. The vertex matrix for PQR is
9 1 4
2 1 5 . 
y2  y1
tan A ¬
x2 x1
The translation matrix is 52 2 2
5 5
. 
0 40 Find the vertex matrix of the image.
¬
45  16
40
¬
61  92 1
1
4
5
 
2 2 2
5 5 5

11 3
3 6   6
0 
40
mA ¬tan1  61
 The reflection matrix for a reflection in the x-axis
¬33.3
A vector in standard position that is equal to AB
is 
1 0
0 1
. 
forms a 33.3° angle with the negative x-axis in Multiply the vertex matrix of the image by the
the third quadrant. So it forms a 180  33.3 or reflection matrix to find the vertex matrix of the
213.3° angle with the positive x-axis. final image.
Thus, AB has a magnitude of
5321 or about
72.9 units and a direction of about 213.3°.  10 1
0
  
11 3 6
3 6 0
 
11 3 6
3 6 0 
3 0 2
42. The vertex matrix for DEF is 2 5 4 .   The coordinates of the vertices of the final image
are P(11, 3), Q(3, 6), and R(6, 0).
The translation matrix is
3 3
6 6
.3
6  47. The rotation matrix for a counterclockwise
1 0
Find the vertex matrix for the image.
rotation of 180° is  0 1
. 
3 0 2 3 3 3 6 3 1 The vertex matrix for WXYZ
  
2 5 4  6 6 6  8 1 10    8 2 1 6
The coordinates of the vertices of the image are
is  1 3 0 3 
.

D(6, 8), E(3, 1), and F(1, 10). Multiply the vertex matrix for WXYZ by the
3 0 2 rotation matrix to find the vertex matrix of the
43. The vertex matrix for DEF is 2 5 4 .   image.
1 0 8 2 1 6
Multiply the vertex matrix by the scale factor to
find the vertex matrix of the image.
 0 1   
1 3 0 3  1 3 0 3  
8 2 1 6

Multiply the vertex matrix of the image by the

 
12 8
3   0
 
0 2 5 5 scale factor to find the vertex matrix of the final
4 
5 2 5 4 8 16 image.

5 4 5
16 4 2 12
   
8 2 1 6
The coordinates of the vertices of the image are 2 1 3 0 3 
2 6 0 6
D 5 ,  5 , E(0, 4), and F 5 ,  5 .
12
 8 8 1
 6
The coordinates of the vertices of the final image
44. The reflection matrix for a reflection in the line are W(16, 2), X(4, 6), Y(2, 0), and
Z(12, 6).
y  x is  
0 1
1 0
.

3 0 2
The vertex matrix for DEF is 2 5 4 .   Chapter 9 Practice Test
Multiply the vertex matrix for DEF by the
reflection matrix to find the vertex matrix of the Page 517
image. 1. isometry 2. uniform
3 0 2 2 5 4
  
0 1
1 0

2 5 4

3 0 2    3. scalar 4. E
C
5. D  6. BCA
The coordinates of the vertices of the image are
D(2, 3), E(5, 0), and F(4, 2).
45. The rotation matrix for a counterclockwise
rotation of 270° is
0 1
1 0 
. 
341 Chapter 9
7. First, graph PQR. Next, translate each vertex 11. y A
right 3 units and up 1 unit. Connect the vertices C B
to form PQR.
P(3, 5) → P(3  3, 5  1) or P(0, 6) x
Q(2, 1) → Q(2  3, 1  1) or Q(1, 2) B C
R(4, 2) → R(4  3, 2  1) or R(1, 3) A

P y Reflection in line y  x: A(3, 2) → (2, 3)


P B(1, 1) → (1, 1)
C(3, 1) → (1, 3)
R Reflection in line y  x: (2, 3) → A(3, 2)
(1, 1) → B(1, 1)
R Q (1, 3) → C(3, 1)
Q
x The angle of rotation is 180°.
O
12. y R

8. First, graph parallelogram WXYZ. Next, translate


each vertex up 5 units and left 3 units. Connect
the vertices to form WXYZ.
W(2, 5) → W(2  3, 5  5) or W(5, 0)
X(1, 5) → X(1  3, 5  5) or X(2, 0) S
Y(2, 2) → Y(2  3, 2  5) or Y(1, 3) OS x
Z(1, 2) → Z(1  3, 2  5) or Z(4, 3) R
T
Z Y y
T
Reflection in y-axis: R(1, 6) → (1, 6)
S(1, 1) → (1, 1)
W X O x T(3, 2) → (3, 2)
Z Y Reflection in line y  x: (1, 6) → R(6, 1)
(1, 1) → S(1, 1)
(3, 2) → T(2, 3)
The angle of rotation is 90° clockwise or 270°
W X counterclockwise.
G
9. First, graph F. Next, translate each vertex left 13. Yes; the pattern is a tessellation because at the
4 units and down 1 unit. Connect the vertices to different vertices the sum of the angles is 360°.
form FG
. The tessellation is uniform because at every
F(3, 5) → F(3  4, 5  1) or F(1, 4) vertex there is the same combination of shapes
G(6, 1) → G(6  4, 1  1) or G(2, 2) and angles.
y F 14. Yes; the pattern is a tessellation because at the
F different vertices the sum of the angles is 360°.
The tessellation is uniform because at every
vertex there is the same combination of shapes
and angles. The tessellation is also semi-regular
O x since more than one regular polygon is used.
G 15. Yes; the pattern is a tessellation because at the
G different vertices the sum of the angles is 360°.
10. y The tessellation is not uniform because the
number of angles at the vertices varies.
L K
16. MN  5, r  4
J Use the Dilation Theorem.
MN  r (MN)
O x MN  (4)(5)
J MN  20
17. MN  8, r  14

K L Use the Dilation Theorem.
MN  r (MN)
Reflection in y-axis: J(1, 2) → (1, 2)
K(3, 4) → (3, 4) MN  1
4 (8)

L(1, 4) → (1, 4) MN  2
Reflection in x-axis: (1, 2) → J(1, 2)
(3, 4) → K(3, 4)
(1, 4) → L(1, 4)
The angle of rotation is 180°.

Chapter 9 342
18. MN  36, r  3 24. The distance Gunja must travel can be found by
Use the Dilation Theorem. multiplying the distance measured on the map by
MN ¬ r (MN) the scale factor.
36 ¬(3)(MN) 150
2.25 in.   mi
1 in.  337.5 mi
12 ¬MN
Gunja must travel 337.5 mi.
19. MN  9, r  1
5 25. A; A reflection of (3, 4) in the x-axis gives (3, 4).
Use the Dilation Theorem. A reflection of (3, 4) in the origin gives (3, 4).
MN  r (MN) A reflection of (3, 4) in the y-axis gives (3,4).
MN  1
5 (9)
 A reflection of (3, 4) in the origin gives (3, 4).
Only choice A, a reflection in the x-axis,
MN  9

5 gives (3, 4).
20. MN  20, r  2

3
Use the Dilation Theorem.
MN ¬ r (MN)
20 ¬2 Chapter 9 Standardized Test Practice
3 (MN)


30 ¬MN Pages 518–519


29 3
21. MN   5 , r  5
 1. D
Use the Dilation Theorem. 2. A; both CE
 and D F are at an angle of 70° from
MN ¬ r (MN) the line 
AB, so they are parallel.
5 ¬ 5 (MN)
29
  3 3. D; the congruence of a single pair of opposite
29 sides is not sufficient proof that QRST is a
3 ¬MN
 
parallelogram.
22. Find the magnitude of the resultant vector.
4. B; in a reflection in the y-axis, x-coordinates
 ¬

v x2  y2 become their opposite, so (4, 2) → (4, 2).

(3)2 
 22 5. D; parallelogram JKLM can be thought of as

13
 parallelogram ABCD transformed by having all of
¬3.6 its points moved the same distance in the same
The resultant vector lies in the second quadrant. direction, or translated.
Find the direction. 6. B; congruence transformations preserve angle and
y
tan ¬x distance measure, collinearity, and betweenness of
¬2
 points. Orientation is not necessarily preserved.
3
7. A; a 180° rotation of ABC about point C on line
m ¬tan12 3

m maps ABC to ABC.
¬33.7 8. The top and bottom sides of the hexagon must be
The resultant vector forms a 33.7° angle with the parallel and have the same length. So, the bottom
negative x-axis in the second quadrant. So it side must have length 2 and slope 0. The
forms a 180  33.7 or 146.3° angle with the coordinates of the missing vertex are (1, 2).
positive x-axis. 9. The triangle is isosceles, so the missing angle is
Thus, v has a magnitude of
13
 or about x°. The sum of the interior angles of a triangle
3.6 units and a direction of about 146.3°. is 180°. Find x.
23. Find the magnitude of the resultant vector. 2x°  x°  x° ¬180°
 ¬
x
w 
2  y2 4x° ¬180°

 (6)2  (8)2 x° ¬45°
¬10 So, x  45.
The resultant vector lies in the third quadrant. 10. The two right triangles are similar. So, using the
AC
 B
C
Find the direction. definition of similar polygons,  CE CD .
y
tan ¬x Use the Pythagorean Theorem to find CE.

¬ 8 CE ¬
 (CD)2  (DE )2
6

100
 2 452
¬4
3

¬5
481

m ¬tan14 3

Find x.
¬53.1 AC BC
The resultant vector forms a 53.1° angle with the 
CE ¬
CD
negative x-axis in the third quadrant. So it forms x 300
 ¬
5

481 100
a 180  53.1 or 233.1° angle with the positive
x ¬329
x-axis.
To the nearest meter, the length of the cable
Thus, w has a magnitude of 10 units and a
is 329 m.
direction of about 233.1°.

343 Chapter 9
11. Given: 
ABA
C
, A 12b. Find Q, R, S, and T using the scale factor,
D
A A
E r  2.
Prove: ABD  ACE Preimage (x, y) Image (2x, 2y)
Proof: Q(2, 2) Q(4, 4)
B D E C R(2, 4) R(4, 8)
Statements Reasons S(3, 2) S(6, 4)
B
1. AA
C
, A
D
A
E
 1. Given T(3, 4) T(6, 8)
2. ABD  ACE 2. Isos.  Thm. R y
8
ADE  AED
Q
4
3. ADB and ADE 3. If 2  form a linear
are supplementary. pair, then they are
AEC and AED suppl. 8 4 O 4 8x
are supplementary. 4
S
4. ADB  AEC 4.  suppl. to   8
are . T
12c. Multiply the x- and y-coordinates of each vertex
5. ABD  ACE 5. AAS by the scale factor; Q(2, 2) becomes (2  2, 2  2)
or Q(4, 4).
12a. y
8 12d. Enlargements and reductions preserve the shape
of the figure. Congruence transformations
R 4
Q preserve collinearity, betweenness of points, and
angle and distance measures.
8 4 O 4 8x
S 4
T
8

Chapter 9 344
Chapter 10 Circles
Page 521 Getting Started 10. x2  4x ¬10
x2  4x  10 ¬0
1. 4p ¬72
9 Use the Quadratic Formula.
 
9 4p ¬9(72)
4 9 4
a  1, b  4, c  10
b 
b 
 4ac 2
p ¬162 x ¬ 2a
2. 6.3p ¬15.75 (4)  
(4)2  )
 4(1)(10
x ¬ 
2(1)
6.3p 15.
75
  ¬
6.3 6.3 4 
56
x ¬ 
2
p ¬2.5
4  2
14

3. 3x  12 ¬8x x ¬ 2
3x  12  3x ¬8x  3x x ¬2  14

12 ¬5x x ¬5.7, 1.7
12 5x 11. 3x2  2x  4  0
5 ¬ 5
 
Use the Quadratic Formula.
12
5 ¬x or x ¬2.4
 a  3, b  2, c  4
4. 7(x  2) ¬3(x  6) b  b
x 
 
 4ac
2

7x  14 ¬3x  18 2a
7x  3x ¬18  14 (2)  
(2)  
 4(3)(4)
2
x   2(3)
4x ¬32
32 2 
52
4x x 6
4 ¬ 4
 
13
2  2
x 
x ¬8 6
5. C ¬2pr 1
13
x 3
2pr
C ¬  x  1.5, 0.9
2p 2p
C
¬r 12. x2 ¬x  15
2p x2 x  15 ¬0
C
6. r ¬
6.28
Use the Quadratic Formula.
a  1, b  1, c  15
6.28r ¬6.28C
6.28  b  b 
 4ac
2
x 
6.28r ¬C 2a
7. c2 ¬a2  b2 (1)  
(1) 
2 )
4(1)(15
x   2(1)
172 ¬82  x2
289 ¬64  x2 1
61
x 2
225 ¬x2
x  4.4, 3.4
 ¬
225 x2
13. 2x2  x ¬15
15 ¬x
2x2  x  15 ¬0
8. c2 ¬a2  b2 Use the Quadratic Formula.
102 ¬62  x2 a  2, b  1, c  15
100 ¬36  x2
b  b 
 4ac
2
64 ¬x2 x 
2a
 ¬x
64 2 1   5)
1  4(2)(1
2 
x   2(2)
8 ¬x
c2 ¬a2  b2 1  
121

9. x 4
(6x)2 ¬722  722
1  11
36x2 ¬5184  5184 x 4
36x2 ¬10,368 x  2.5, 3
2
36x 10,368
36 ¬ 36
 
x2 ¬288
x ¬288
 10-1 Circles and Circumference
x ¬17.0

Page 524 Geometry Activity:


Circumference Ratio
1. See students’ work.
2. Each ratio should be near 3.1.
3. C  3.14d

345 Chapter 10
Pages 525–526 Check for Understanding 18. Two chords are shown: B E or C
D.
1. Sample answer: The value of  is calculated by E
19. B  is the only chord that goes through the center,
dividing the circumference of a circle by the E
so B  is a diameter.
diameter. A
20. F is a radius not contained in a diameter.
2. d  2r, r  1

2d 21. The circle has its center at R, so it is named circle
3. Except for a diameter, two radii and a chord of a R, or R.
circle can form a triangle. The Triangle Inequality 22. Six radii are shown: RT, R
U
, RV, R
W, R
X, or R
Z
.
Theorem states that the sum of two sides has to
23. Three chords are shown: Z V, T
X
, or W
Z .
be greater than the third. So, 2r has to be greater
than the measure of any chord, but 2r is the X
24. T  or W
Z are the chords that go through the
measure of the diameter. So the diameter has to X
center, so T  and W
Z are diameters.
be longer than any other chord of the circle. U
25. R  and RV are radii not contained in a diameter.
4. The circle has its center at E, so it is named circle 26. d  2r
E, or E. d  2(2) or 4 ft
5. Four radii are shown: EA, E
B
, EC
, or ED. 27. r  1

2d
6. Three chords are shown: A B, A
C, or BD
.
r  1

2 (5) or 2.5 ft
C
7. A  and BD
 are chords that go through the center,
C
so A  and BD are diameters. 28. TR  1

2 (TX)
8. r  1

2d TR  1

2 (120) or 60 cm
r  1
 29. ZW  2RZ
2 (12) or 6
ZW  2(32) or 64 in. or 5 ft 4 in.
The radius is 6 mm.
R
30. U  and R
V are both radii.
9. d  2r
RV  UR
d  2(5.2) or 10.4
RV  18 in.
The diameter is 10.4 in.
T
31. X is a diameter and UR is a radius.
10. Since the radius of Z is 7, XZ  7.
Z
Y  is part of radius X
Z
. UR  1

2 XT
XY  YZ ¬XZ UR  1

2  YZ ¬7 2 (1.2) or 0.6 m
YZ ¬5 32. AZ  CW
AZ  2
11. Since the radius of W is 4, IW  4 and WY  4.
IX
 is part of diameter IY
. X
33. A is a radius of A, and Z
X is part of A
X
.
IX  XY ¬IW  WY AZ  ZX ¬AX
IX  2 ¬4  4 2  ZX ¬1

2 (10)
IX ¬6
ZX ¬3
12. IC  IW  WY  XZ  ZC  XY
34.  is a radius of B, and B
BZ X
 is part of B
Z
.
IC  4  4  7  7  2
ZX  BX ¬BZ
IC  20
13. d  2r 3  BX ¬1

2 (30)
d  2(5) or 10 m 3  BX ¬15
C  d
BX ¬12
C  (10) or about 31.42 m
35. BY  BX
14. C ¬d
BY  12
2368 ¬d
2368 36. YW  ZX
 ¬d
 
YW  3
753.76 ¬d 37. AC  AZ  ZW  WC
d ¬753.76 ft AC  2  30  2 or 34
r ¬1

2d 38. FG  GH
FG  10
r ¬1

2 (753.76) or 376.88 ft 39. FH  FG  GH
15. B; C  d FH  10  10 or 20
C ¬(9) or 9 mm
40. GL  GH
GL  10
L
41. G is a diameter of J, so GL  10.
Pages 526–527 Practice and Apply J
G is a radius of J.
16. The circle has its center at F, so it is named circle
F, or F. GJ  1

2 (GL)
17. Three radii are shown: FA, F
B
, or FE. GJ  1

2 (10) or 5

Chapter 10 346
L
42. J is a radius of J. 53. The diameter of the circle is the same as the
hypotenuse of the right triangle.
JL  1

2 (10) or 5
d2  32  42
43.  is a diameter of K, so JL  5.
JL
d2  25
K
J is a radius of K.
d  25  or 5 ft
JK  1

2 JL C  d
JK  1

2 (5) or 2.5 C  (5) or 5 ft
44. d ¬2r 54. The diameter of the circle is the same as the
d ¬2(7) or 14 mm hypotenuse of the right triangle.
C ¬d d2  102  102
C ¬(14) or about 43.98 mm d2  200
45. r ¬1

2d d  200 or 102  in.
r ¬1
 C  d
2 (26.8) or 13.4 cm
C ¬d C  (102 ) or 102  in.
C ¬(26.8) or about 84.19 cm 55. The diameter of the circle is the same as the
hypotenuse of the right triangle.
46. C ¬d
26 ¬d d2  (4 2)2  (4 2)2
26 ¬d or d  26 mi d  64
2

r ¬1
 d  64  or 8 cm
2d
C  d
r ¬1

2 (26) or 13 mi C  (8) or 8 cm
47. C ¬d
56. 1; This description is the definition of a radius.
76.4 ¬d
76
.4
57. 0; The longest chord of a circle is the diameter,
 ¬d
 which contains the center.
24.32  d or d  24.32 m 58. C  2r
r  1

2d
C  2(800) or about 5026.5 ft
59. 800  200  600
r  1

2 (24.32) or 12.16 m 800  300  500
48. r  1

2d
The range of values for the radius of the explosion
circle is 500 to 600 ft.
2 12 2  or 6 4 yd
r  1
 1 1
60. C  2r
C  d C  2(500) or about 3142
C  121
2  or about 39.27 yd
 C  2(600) or about 3770
49. d  2r The least and maximum circumferences are
3142 ft and 3770 ft, respectively.
d  263
4  or 13 2 in.
 1 61. Let r  the radius of O.
C  d x2  y2 ¬r2
p2  t2 ¬r2
C  131
2  or about 42.41 in.

x2  y2  p2  t2 ¬288
50. r  1

2d Substitute r2 for x2  y2 and r2 for p2  t2.
r2  r2 ¬288
r  1

2 (2a) or a 2r2 ¬288
C  d r2 ¬144
C  (2a) or about 6.28a r ¬12
51. d  2r C  2r
C  2(12) or 24 units
d  2a
6 or about 0.33a 62. Sample answer: about 251.3 feet. Answers should
C  2r include the following.
C  2a
6 or about 1.05a
• The distance the animal travels is
approximated by the circumference of the circle.
52. The diameter of the circle is the same as the
hypotenuse of the right triangle. • The diameter for the circle on which the animal
is located becomes 80  2 or 78. The
d2  162  302
circumference of this circle is 78. Multiply by
d2  1156 22 to get a total distance of 22(78) or
d  1156
 or 34 m 5391 feet. This is a little over a mile.
C  d
C  (34) or 34 m

347 Chapter 10
63. Let r  the radius of C. Then 2r  the radius mC ¬tan1 1

2
of B, and 4r  the radius of A.
¬27
sum of circumferences ¬2r  2(2r)  2(4r)
42 ¬2r  4r  8r The magnitude is about 44.7 and the direction is
42 ¬14r about 27°.
3 ¬r 70. AB  |k|(AB)
AC  r  2(2r)  4r AB  6(5)
AC  3  2(2  3)  4(3) AB  30
AC  3  12  12 or 27 71. AB  |k|(AB)
100d
64. A;  AB  1.5(16)
k% of gasoline has been pumped. AB  24
65. Small circle: C  2r
72. AB ¬|k|(AB)
C  2(5) or 10
23
¬1
 2
Medium circle: r  5  5 or 10
C  2r ¬1

3
C  2(10) or 20
Q
73. Given: R bisects SRT R
Large circle: r  5  5  5 or 15 Prove: mSQR mSRQ
C  2r
C  2(15) or 30
The circumferences from least to greatest are S Q T
10, 20, and 30.
Proof:
Statements Reasons
1.  bisects SRT.
RQ 1. Given
Page 528 Maintain Your Skills 2. SRQ  QRT 2. Def. of  bisector
| ¬1
66. |AB  
2  42
3. mSRQ  mQRT 3. Def. of  
¬17
 4. mSQR 4. Exterior Angle
¬4.1  mT  mQRT Theorem
mA ¬tan1 4
 5. mSQR mQRT 5. Def. of Inequality
1
6. mSQR mSRQ 6. Substitution
¬76
The magnitude is about 4.1 and the direction is
about 76°.
| ¬4
67. |V  
2  92 F
74. a is the midpoint of O . Therefore, the missing
coordinates are (2a, 0).
¬97

75. x  2x ¬180 Linear pair
¬9.8
3x ¬180
mV ¬tan1 9

4 x ¬60
¬66 76. 2x  3x ¬90 Lines are perpendicular.
The magnitude is about 9.8 and the direction is 5x ¬90
about 66°. x ¬18
| ¬
68. |AB (7  4
)2  (2
2  2)2 77. (3x  x)  2x ¬180 Linear pair
¬ 3  2
2 0 2 6x ¬180
x ¬30
¬409

78. 3x  5x ¬180 Linear pair
¬20.2
8x ¬180
22  2
tanA ¬
74
 x ¬22.5
20
¬  79. 3x ¬90  lines form 2 rt .
3
20 x ¬30
mA ¬tan1  
3 80. x  x  x ¬360
¬81 3x ¬360
The magnitude is about 20.2 and the direction is x ¬120
about 81°.
| ¬
69. |CD (40 
0)2  2
(0  (20))
¬ 40 
2 20 2

¬2000

¬44.7
0  (20)
tanC ¬ 
40  0
20 1
¬
40 or 2

Chapter 10 348
12. C  2r
10-2 Angles and Arcs C  2(12) or 24
Let   arc length.
60 
Pages 532–533 Check for Understanding  0 ¬
36 24

60

360 (24) ¬
1. Sample answer: A
4 ¬
The length of 
90 110
C TR is 4 units or about 12.57 units.
160 B
13. Sample answer:
 25%(360°)  90°, 23%(360°)  83°,
AB, 
BC, 
AC, 
ABC, 
BCA,    110,
CAB; mAB 28%(360°)  101°, 22%(360°)  79°,
  
mBC  160, mAC  90, mABC  270, 2%(360°)  7°
  250, mCAB  200
mBCA
2. A diameter divides the circle into two congruent
arcs. Without the third letter, it is impossible to
Pages 533–535 Practice and Apply
know which semicircle is being referenced.
14. AGC and CGB are a linear pair.
3. Sample answer: Concentric circles have the same mAGC  mCGB ¬180
center, but different radius measures; congruent 60  mCGB ¬180
circles usually have different centers but the mCGB ¬120
same radius measure.
15. AGC and BGE are vertical angles.
4. MCN and NCL are a linear pair. mBGE  mAGC
mMCN  mNCL ¬180 mBGE  60
60  mNCL ¬180
16. AGD is a right angle.
mNCL ¬120
mAGD  90
5. MCR and RCL are a linear pair.
17. One way to find mDGE is by using AGD and
mMCR  mRCL ¬180
BGE. AGB is a straight angle.
(x  1)  (3x  5) ¬180
mAGD  mDGE  mBGE ¬mAGB
4x  4 ¬180
90  mDGE  60 ¬180
4x ¬176
mDGE ¬30
x ¬44
Use the value of x to find mRCL. 18. CGD is composed of adjacent angles, CGA and
AGD.
mRCL ¬3x  5
mCGD  mCGA  mAGD
¬3(44)  5
mCGD  60  90
¬132  5 or 137
mCGD  150
6. Use the value of x to find mRCM.
19. AGE is composed of adjacent angles, AGD and
From Exercise 5, x ¬44.
DGE.
mRCM ¬x  1
mAGE  mAGD  mDGE
¬44  1 or 43
mAGE  90  30
7. RCN is composed of adjacent angles, RCM mAGE  120
and MCN.
20. ZXV and YXW are vertical angles.
mRCN  mRCM  mMCN
mZXV ¬mYXW
 43  60 or 103
2x  65 ¬4x  15
8. 
BC is a minor arc, so mBC  mBAC.
50 ¬2x
BAC ¬EAD 25 ¬x
mBAC ¬mEAD Use the value of x to find mZXV.
 ¬mEAD
mBC
 ¬42 mZXV  2x  65
mBC  2(25)  65

9. CBE is a semicircle.  50  65 or 115
  180
mCBE 21. Use the value of x to find mYXW.
 is by using 
10. One way to find mEDB EDC and CB. From Exercise 20, x  25.

EDC is a semicircle. mYXW  4x  15
  mEDC   mCB 
mEDB  4(25)  15

mEDB  180  42 or 222  100  15 or 115
 is by using 
11. One way to find mCD CDE and  DE. 22. ZXY and ZXV are a linear pair.

CDE is a semicircle. mZXY  mZXV ¬180
  mDE  ¬mCDE
mCD mZXY  115 ¬180

mCD  42 ¬180 mZXY ¬65
 ¬138
mCD 23. ZXY and VXW are vertical angles.
VXW ¬ZXY
mVXW ¬mZXY
mVXW ¬65

349 Chapter 10
24.    mBOC. 33. WZV and UZY are vertical angles.
BC is a minor arc, so mBC
B
Since A  is a diameter and AOC is a right WZV ¬UZY
angle, mAOB  180 and mAOC  90. mWZV ¬mUZY
 ¬mUY 
mBOC  mAOC ¬mAOB mWV

mBOC  90 ¬180 mWV ¬76
mBOC ¬90 
34. mWX  mWZX
 ¬90   2x
mBC mWX
   mAOC. From Exercise 32, x 26.
25. AC is a minor arc, so mAC
  2(26) or 52
AOC is a right angle. mWX
 ¬mAOC
mAC 35. XZY ¬WZX
 ¬90
mAC mXZY ¬mWZX
 ¬mWX
26.    mAOE. mXY
AE is a minor arc, so mAE

AOE and BOC are vertical angles. mXY ¬52
AOE ¬BOC    mXY 
36. mWUY  360  mWX
mAOE ¬mBOC 
mWUY  360  52  52 or 256
 ¬mBOC
mAE   360  mWX   mXY

 ¬90 37. mYVW
mAE 
mYVW  360  52  52 or 256
27.    mEOB.   360  mXY 
EB is a minor arc, so mEB 38. mXVY
EOB and AOC are vertical angles. 
mXVY  360  52 or 308
EOB ¬AOC   360  mWX 
39. mWUX
mEOB ¬mAOC 
 ¬mAOC mWUX  360  52 or 308
mEB
 ¬90
mEB 40. C  d
C  (32) or 32
B
28. Since A  is a diameter, mAOB  180.
  mAOB Let   arc length.
mACB
  180 10
0 
360 ¬ 32

mACB
10
0
360 (32) ¬
29. Since BOD  DOE  EOF  FOA, 
mBOD  mDOE  mEOF  mFOA. 
80
9 ¬

Since mAOB  180, each of the four angles The length of  
80
DE is 9 units or about
18
0
measures 4 or 45. 27.93 units.

AD is composed of adjacent arcs, 
DE, 
EF, and 
FA. 41. C  d
    C  32
mAD  mDE  mEF  mFA
  mDOE  mEOF  mFOA   360  mDCE
mAD mDHE
  45  45  45 or 135
mAD   360  90 or 270
mDHE
30. 
CBF is composed of adjacent arcs, 
CB, 
BD, 
DE, Let   arc length.
and 
EF. 27
0 
360 ¬ 32
  mCB   mBD   mDE   mEF  
mCBF
 27
0
360 (32) ¬
mCBF  mCOB  mBOD  mDOE 
 mEOF 24 ¬
  90  45  45  45 or 225
mCBF The length of 
DHE is 24 units or about
  360  mAC  75.40 units.
31. mADC
   360  mHCF
mADC  360  mAOC 42. mHDF
  360  90 or 270
mADC   360  125 or 235
mHDF
32. Find the value of x. Let   arc length.
VY is a diameter, 
Since  VUY is a semicircle. 
VUY 23
5 
360 ¬ 32

 and UY. VU is a
is composed of adjacent arcs, VU 23
5
  mVZU. UY
minor arc, so mVU  is a minor arc, 360 (32) ¬


so mUY  mUZY. 188
9 ¬
 
  mUY   mVUY 
mVU The length of  188
HDF is  
9 units or about
mVZU  mUZY  180 65.62 units.
4x  (2x  24)  180
43. Let   arc length.
6x  24  180
45 
6x  156 
360 ¬
32

x  26 4 5
.

36 0 (32) ¬
Use the value of x to find mUY 4 ¬

mUY  2x  24
  2(26)  24 The length of 
HD is 4 units or about 12.57 units.
mUY
  52  24 or 76 44. Sample answer: 76%(360°)  273°,
mUY
16%(360°)  58°, 5%(360°)  18°, 3%(360°)  11°

Chapter 10 350
45. The first category is a major arc, and the other 56. Sample answer: The hands of the clock form
three categories are minor arcs. central angles.
46. How many free files Answers should include the following.
have you collected? • The hands form acute, right, and obtuse angles.
101 to 500 • Some times when the angles formed by the
16% minute and hour hand are congruent are 1:00
500 to 1000 and 11:00, 2:00 and 10:00, 3:00 and 9:00, 4:00
5%
and 8:00, and 5:00 and 7:00. They also form
100 or less more than 1000
76% congruent angles many other times of the day,
3%
such as 3:05 and 8:55.
47. always 57. B; C  2r or about 6.3r
P  2  2w
48. Sometimes; the central angle of a minor arc can
P  2(2r)  2r or 6r
be greater than 90°.
Since 6.3r 6r, the circumference of the circle is
49. Never; the sum of the measures of the central greater than the perimeter of the rectangle.
angles of a circle is always 360.
58. Rewrite 3:5:10 as 3x:5x:10x and use these
50. always measures for the measures of the central angles
51. Let m1  2x, then m2  3x and m3  4x. of the circle.
m1  m2  m3 ¬360 3x  5x  10x ¬360
2x  3x  4x ¬360 18x ¬360
9x ¬360 x ¬20
x ¬40 The measures of the angles are 3(20) or 60, 5(20)
Therefore, m1  2(40) or 80, m2  3(40) or or 100, and 10(20) or 200.
120, and m3  4(40) or 160.
52. C  d
C  (12) or 12 in. Page 535 Maintain Your Skills
The measure of the angle from the minute hand 59. d  2r
to the hour hand at 2:00 is 60. d  2(10) or 20
Let   arc length. C  d
6
0 
360 ¬ 12 C  (20) or about 62.83

6
0 60. r  1

2d
360 (12) ¬

2 ¬ r  1

2 (13) or 6.5
The arc length is 2 in. or about 6.3 in. C  d
53. C  2r C  (13) or about 40.84
C  2(12) or 24 61. C ¬d
Let   arc length. 28 ¬d
27
0  28  d or d  28
360 ¬ 24

27
0 r  1

2d
360 (24) ¬

18 ¬ r  1

2 (28) or 14
The length of the arc is 18 ft or about 56.5 ft. 62. C ¬d
54. Given: BAC  DAE B C 75.4 ¬d
Prove: 
BC   DE D 75
.4
 ¬d
A
E d ¬24.00
r  1

2d
Proof: r  1

2 (24.00) or 12.00
Statements Reasons 63. magnitude  (72)  2 
(45)2 or about 84.9
45
1. BAC  DAE 1. Given direction  tan1 
72 or about 32
2. mBAC  mDAE 2. Def. of   The magnitude is about 84.9 newtons and the
  mDE 3. Def. of arc measure direction is about 32° northeast.
3. mBC
  Def. of  arcs 12 10
4. BC  DE 4. 64.  
18  x ¬x
55. No; the radii are not equal, so the proportional 12x ¬10(18  x)
part of the circumferences would not be the same. 12x ¬180  10x
Thus, the arcs would not be congruent. 22x ¬180
2
x ¬8
11

351 Chapter 10
26
.2 17.3
65. x ¬
24.2 2 75. 2x  2x  x ¬180
26.2(24.22) ¬17.3x 5x ¬180
634.564 ¬17.3x x ¬36
36.68 ¬x 76. 3x ¬180
66. Construct a line perpendicular to the line with x ¬60
the equation y  7  0 through point Q(6, 2).
y
y70
10-3 Arcs and Chords

Page 538 Geometry Activity: Congruent Chords


and Distance
x 1. 
SU and S
X are perpendicular bisectors of V
T
 and
Y
W , respectively.
2. VT  WY, SU  SX
Q 3. Sample answer: When the chords are congruent,
The line y  7  0 is a horizontal line, so the line they are equidistant from the center of the circle.
perpendicular to the given line is vertical. The
desired distance is 2  7 or 9.
The distance is 9 units. Pages 539–540 Check for Understanding
67. First, write the equation of a line p perpendicular 1. Sample answer: An inscribed polygon has all
to both lines. The slope of each of the given lines vertices on the circle. A circumscribed circle
is 1. So the slope of p is 1. Use the y-intercept means the circle is drawn around so that the
of the first line, (0, 3). polygon lies in its interior and all vertices lie on
y  y1 ¬m(x  x1) the circle.
y  3 ¬1(x  0) 2. Sample answer:
y  3 ¬x
y ¬x  3
110 90
Next, find the point of intersection using the 60
equations y  x  3 and y  x  4. 100
x  3 ¬x  4
2x ¬7
x ¬7
 None of the sides are congruent.
2
3. Tokei; to bisect the chord, it must be a diameter
y  7 1
2  4 or y   2
 and be perpendicular.
The point of intersection is 7
2 ,  2 .
 1 4. Given: X, U VW Y
Finally, find the distance between (0, 3) and Prove: UV   WY
U
2, 2.
7 1

V
d ¬
(x2  
x1)2 
(y2 
y1)2
X

2  0
  
2  3
2 2
7 1

W
¬24.5
 Y
The distance between the lines is 24.5
 units. Proof: Because all radii are congruent,
68. 90  57.5 ¬32.5 U
X X VX W
X Y
. You are given that
180  57.5 ¬122.5 V
U W Y, so UVX  WYX, by SSS. Thus,
The measures of the complement and supplement UXV  WXY by CPCTC. Since the central
are 32.5 and 122.5, respectively. angles have the same measure, their intercepted
69. If ABC has three sides, then ABC is a triangle. arcs have the same measure and are therefore,
70. Both are true. congruent. Thus, UV  WY.
 bisects    1mAB.
71. x  42 since the triangle is isosceles. Y
5. O AB, so mAY 2
 1
mAY  2mAB
72. x  x  30  180 by the Angle Sum Theorem
2x ¬150   1(60) or 30
mAY 2
x ¬75 Y
6. O bisects A, so AX  1
B 
2 (AB).
73. 40  40  x ¬180
AX  1

2 (AB)
x ¬180  80 or 100
74. x  x  90 ¬180 AX  1

2 (10) or 5
2x ¬90
x ¬45

Chapter 10 352
7. Draw radius O A. Radius OA
 is the hypotenuse of 22. 
TZ bisects X
Y
, so XV  VY.
OXA. XY  XV  VY
(OX)2  (AX)2 ¬(OA)2 XY  5  5 or 10
(OX)2  52 ¬102   mBC   mCD   mDE
  mEF 
23. mAB
(OX)2  25 ¬100   mGH   mHA  36
0
 mFG or 45
(OX)2 ¬75 8
OX ¬ 75 or 53   mMJ
  mJK
  mKL
36
0
24. mLM 4 or 90
B
8. A  and CE
 are equidistant from P, so AB
C E
. 25. 2x  x  2x  x ¬360
QE  1
2
(CE), so CE  2(20) or 40. 6x ¬360
B
A C E, so AB  40. x ¬60
9. (PE)2 ¬(PQ)2  (QE)2 2x ¬120
  mRQ   120;
(PE)2 ¬102  202 mNP
   60
(PE)2 ¬100  400 mNR  mPQ
(PE)2 ¬500 26. (LK)  (FL)2 ¬(FK)2
2

PE ¬10 5 or about 22.36 (LK)2  82 ¬172


10. 
AB  BC   CA (LK)2  64 ¬289
Each arc measures  36
0° (LK)2 ¬225
3 or 120°.
LK ¬15
L
27. F  bisects K
M
, so LK  LM.
Pages 540–543 Practice and Apply KM  KL  LM
KM  15  15 or 30
Y
11. X bisects A, so AM  1
B 
2 (AB). G
28. J and K M are equidistant from the center,
AM  1

2 (AB) G
so J K M
.
AM  1

2 (30) or 15 JG  KM
12. MB  AM JG  30
MB  15 H
29. F  bisects J
G, so JH  1

2 (JG).
Z
13. X bisects C, so CN  1
D 
2 (CD). JH  1

2 (JG)
CN  1
2 (CD) JH  1

2 (30) or 15
1
CN  2(30) or 15 30. 
DF bisects B C
, so FB  CF.
14. ND  CN FB  CF
ND  15 FB  8
 bisects    mCZ. F bisects B C
, so BC  FB  CF.
Z
15. X CD, so mDZ 31. D
  BC  FB  CF
mDZ  mCZ
  40 BC  8  8 or 16
mDZ
 bisects    1mCD
.
Z
16. X CD, so mCZ 2 B
32. A  and B C are equidistant from the center,
 
mCD  2mCZ B
so A B C
.
  2(40) or 80
mCD AB  BC
  mCD. AB  16
B
17. AC D , so mAB
 
mAB  mCD 33. (FD)2  (CF)2 ¬(DC)2
  80
mAB (FD)2  82 ¬102
 bisects    1mAB
. (FD)2  64 ¬100
Y
18. X AB, so mYB 2
  (FD)2 ¬36
YB  1 mAB
2 FD ¬6

YB  1

2 (80) or 40 Since ED  FD, ED  6.
19. (QR)2(PR)2 ¬(PQ)2 34. Since XY  and S
T  are equidistant from the center,
(QR)2  32 ¬52 Y
X S T.
(QR)2  9 ¬25 XY  ST
(QR)2 ¬16 4a  5 ¬5a  13
QR ¬4 9a ¬18
R
20. P  bisects Q
S
, so QS  2(QR). a ¬2
QS  2(QR) ST  5a  13
QS  2(4) or 8 ST  5(2)  13
21. TV  13  1 or 12 ST  10  13 or 3
TX  TW since both are radii. SQ  1 1
2 (ST)  2 (3) or 1.5

(XV)2  (TV)2 ¬(TX)2 35. Since AC  20, then BC  1 
2 (20) or 10.
(XV)2  (12)2 ¬(13)2 Since mACE  45 and mBDC ¬90, then
(XV)2  144 ¬169 mCBD  180  (90  45) or 45.
(XV)2 ¬25
XV ¬5

353 Chapter 10
Therefore CD  5x. 3. AN  1
 3. Def. of bisector
(CD)2  (BD)2  (BC)2 2 MN;
(5x)2  (5x)2 ¬102 BQ  1

2 PQ
25x2  25x2 ¬100 4. MN  PQ 4. Def. of  segments
50x2 ¬100
5. 1 1
2 MN  2 PQ
 5. Mult. Prop.
x2 ¬2
x ¬ 2 or about 1.41 units 6. AN  BQ 6. Substitution
36a. Given N
7. AB Q
 7. Def. of  segments
36b. All radii are congruent. N
8. O  
OQ 8. All radii of a circle
36c. Reflexive Property are .
36d. Definition of perpendicular lines 9. AON  BOQ 9. HL
36e. ARP  BRP A
10. OO
B 10. CPCTC
36f. CPCTC
39. Let x  width of largest square. Use the
36g. If central angles are congruent, intercepted arcs
Pythagorean Theorem.
are congruent.
x2  x2 ¬42
37. Given: O, O S RT
,
R S T 2x2 ¬16
V
O UW ,
x2 ¬8
S
OO V O
W x ¬ 8 or about 2.82
Prove: R T
U W The width is about 2.82 in.
V
Proof: U 40. C

Statements Reasons 34 m
30 m
T
1. OO
W
 1. All radii of a  A xm B
are . 30 m
2. 
OS RT, 
O
V
U
W, 34 m
2. Given
S
O O V
3. OST, OVW are 3. Def. of  lines D
right angles. Let x  distance from the center to the chord. Use
4. STO  VWO 4. HL the Pythagorean Theorem.
T
5. S V W
 5. CPCTC x2  302 ¬342
6. ST  VW 6. Definition of  x2  900 ¬1156
segments x2 ¬256
7. 2(ST)  2(VW) 7. Multiplication x ¬16
Property The chord is 16 m from the center of the circle.
8. 
OS bisects R
T
; 8. Radius  to a chord 41. B
24 in.
V
O  bisects U
W. bisects the chord. 30 in.
9. RT  2(ST), 9. Def. of seg. bisector
UW  2(VW) A
E x F C
10. RT  UW 10. Substitution
30 in.
T
11. RU W 11. Definition of 
segments D
Since the diameter is 60 inches, the radius is 30
38. Given: O, M
N
P Q
 M N inches. Let x  distance from the center to the
N
O and O
Q are radii. A
chord. Use the Pythagorean Theorem.
A
OMN; O
B
P Q
 O Q x2  242 ¬302
A
Prove: O OB
 B x2  576 ¬900
x2 ¬324
Proof: P x ¬18
The chord is 18 in. from the center of the circle.
Statements Reasons
1. O, MNP Q
, O
N
 1. Given
Q
and O  are radii,
A
O  M N,
B
O  P Q

A
2. O  bisects MN
; O
B 2. 
OA and OB
 are
Q
bisects P. contained in radii.
A radius  to a chord
bisects the chord.

Chapter 10 354
42. A 51. Sample answer: The grooves of a waffle iron are
chords of the circle. The ones that pass
10 cm horizontally and vertically through the center are
48 cm
diameters. Answers should include the following.
C

x cm
A F B
B
Let r  the radius of the circle. Use the E
Pythagorean Theorem. C D
102  242 ¬r2
100  576 ¬r2
676 ¬r2
26 ¬r • If you know the measure of the radius and the
The radius is 26 cm. distance the chord is from the center, you can
use the Pythagorean Theorem to find the length
43.
of half of the chord and then multiply by 2.
• There are four grooves on either side of the
16 yd 11 yd diameter, so each groove is about 1 in. from the
x yd center. In the figure, EF  2 and EB  4
because the radius is half the diameter. Using
the Pythagorean Theorem, you find that
FB  3.464 in. so AB  6.93 in. Approximate
lengths for other chords are 5.29 in. and
Since the diameter is 32 yards, the radius is 16
7.75 in., but exactly 8 in. for the diameter.
yards. Let x  half of the length of the chord. Use
the Pythagorean Theorem. B
52. C; D  bisects A
C and OA  OC
x2  112 ¬162 53. Bridgeworth population in 2010  1.2(204,000) or
x2  121 ¬256 244,800
x2 ¬135 Sutterly population in 2010  1.2(216,000) or
x ¬135 259,200
The length of the chord is 2135 or about 23.24 yd. In 2010, 259,200  244,800 or 14,400 more people
44. The line through the midpoint bisects the chord will live in Sutterly than in Bridgeworth.
and is perpendicular to the chord, so the line is a
diameter of the circle. Where two diameters meet
would locate the center of the circle. Page 543 Maintain Your Skills
45. Let r be the radius of P. Draw radii to points D 54. 
KTR is a semicircle.
  mKTR   mTR 
and E to create triangles. The length, DE, is r 3 mKT

r
3 1 mKT  180  mTSR
and AB  2r; 2r
2.   180  42 or 138
mKT
46. Inscribed regular hexagon; the chords and the 
55. ERT is a semicircle.
radii of the circle are congruent by construction.   180
mERT
Thus, all triangles formed by these segments are  is by using mKT .
56. One way to find mKRT
equilateral triangles. That means each angle of  
mKRT  360  mKT
the hexagon measures 120°, making all angles of   360  138 or 222
mKRT
the hexagon congruent and all sides congruent.
U
57. S is a chord that is not a diameter.
47. Inscribed equilateral triangle; the six arcs making
up the circle are congruent because the chords 58. MD is a radius and RI is a diameter.
intercepting them were congruent by RI  2(MD)
construction. Each of the three chords drawn RI  2(7) or 14
intercept two of the congruent chords. Thus, the M
59. All radii are congruent: R, AM
, D
M
, IM

three larger arcs are congruent. So, the three 1x ¬120
60. 2
chords are congruent, making this an equilateral
triangle. x ¬240
  mCD  1x ¬25
48. mAB 61. 2
49. No; congruent arcs must be in the same circle or x ¬50
congruent circles, but these are in concentric
62. 2x ¬1
2 (45  35)

circles.
B
50. A C D; in the smaller circle, O
X
OY because 2x ¬1

2 (80)
they are radii. This means that in the larger 2x ¬40
B
circle, A and C D
 are equidistant from the center,
x ¬20
making them congruent chords.

355 Chapter 10
63. 3x ¬1
2 (120  60)
 SC  1

2 CT
3x ¬1

2 (60) SC  1

2 AC
3x ¬30 SC  1

2 (42) or 21
x ¬10 2
10. 1
2 x  5 ¬13
 2 2
64. 45 ¬1
2 (4x  30)

2
x4  25 ¬169
45 ¬2(4x)  1
1 
2 (30) 2
x4 ¬144
45 ¬2x  15
30 ¬2x x2 ¬576
15 ¬x x ¬24
65. 90 ¬1
2 (6x  3x)


90 ¬1

2 (9x) 10-4 Inscribed Angles
90 ¬4.5x
20 ¬x
Page 544 Geometry Activity: Measure of
Inscribed Angles
Page 543 Practice Quiz 1 1. See students’ work.
  2(mXYZ)
C
1. B, B
D
, and B
A
 are radii. 2. mXZ
3. The measure of an inscribed angle is one-half the
2.  and C
BD B
 are radii, so BD  CB.
measure of its intercepted arc.
BD ¬CB
3x ¬7x  3
4x ¬3
x ¬3
 Pages 548–549 Check for Understanding
4
1. Sample answer: C
   2BD
AC is a diameter, so AC
AC  2BD D
AC  2(3x)
AC  6x B
AC  63
4  or 4.5
 A
   180. 2. The measures of an inscribed angle and a central
3. ADC is a semicircle, so mADC
   angle for the same intercepted arc can be
mAD  mADC  mCD
  180  mCBD calculated using the measure of the arc. However,
mAD
  180  85 or 95
mAD the measure of the central angle equals the
measure of the arc, while the measure of the
4. C  2r
inscribed angle is half the measure of the arc.
C  2(3)  and mNP . mQP   mMN  120
C  6 or about 18.8 in. 3. First find mQP
and mNP  mMQ   60 because the central angles
5. The degree measure of an arc connecting two
are congruent vertical angles.
360
consecutive rungs is 40 or 9.   1  60  30
m1  1mNP
2 2
6. C  2r
m2  1  1
C  2(3) or 6 2 mQP  2  120  60
  mCA   mAD   1  120  60
mCAD m3  1mMN
 2 2
mCAD  180  mABD 1  1
  180  150 or 330
mCAD m4  2mNP  2  60  30
Let   arc length m5  1 1
2 MQ  2  60  30
33
0 
360 ¬ 6

m6  1 1
2 MN  2  120  60
33
0
360 (6) ¬

m7  1 1
2 QP  2  120  60
5.5 ¬
m8  1 1
The length of 
CAD is 5.5 or about 17.3 units. 2 MQ  2  60  30
7. mCAM  mNTM 4. Given: Quadrilateral ABCD B C
mCAM  28 is inscribed in P.
mC  1 mB A
8. mHMN  180  2(40) or 100  22(mDAB ) P
  mHN  Prove: mCDA
mES

mES  mHMN D
  100
mES
Proof: Given mC  1

2 (mB) means that
9. CT  AC
mB  2(mC). Since mB  1  
2 (mCDA)

Chapter 10 356
 
and mC  1 m1  1  1
2 (mDAB), the equation becomes 2 mQR  2  120  60
)  2 1(mDAB
) . Multiplying each side   1  45  22.5
1(mCDA
2
2 m2  1mPS
2 2
   
by 2 results in mCDA  2(mDAB). m3  1 1
2 mSR  2  75  37.5
5. Angle PTS is a right angle because it intercepts a m4  1  1
semicircle. 2 mPQ  2  120  60
  1  45  22.5
m1  m2 mPTS ¬180 m5  1mPS
2 2
(6x  11)  (9x  19)  90 ¬180 
m6  2mQR  1
1
2  120  60

15x  120 ¬180
15x ¬60 1  1
m7  2mSR  2  75  37.5
x ¬4
m8  1  1
Use the value of x to find the measures of 1 2 mPQ  2  120  60
  2(mBDC)
and 2. 9. mBC
  2(25) or 50
m1  6x  11 m2 ¬9x  19 mBC
  mBC   mCD
  mAD ¬360
 6(4)  11 ¬9(4)  19 mAB

120  50  130  mAD ¬360
 35 ¬55
 ¬60
Because PQ     mRS
RS, mPQ . mAD
m3  m4 because they are inscribed angles 1 
m1 ¬2mAD
intercepting congruent arcs, PQ and RS.
¬1

2 (60) or 30
4y  25 ¬3y  9
y ¬16 m2  m1 or 30
Use the value of y to find the measures of 3 
m3 ¬1 mBC
2
and 4.
m3  4y  25 m4 ¬3y  9 ¬1

2 (50) or 25
 4(16)  25 ¬3(16)  9 10. m1  m6  1  
 39 ¬39 2 mXZ

6.  1
2 (100) or 50
X
, XS  

Y
Since X S T SY, and m8  m11.
m1  m11 ¬90
50  m11 ¬90
m11 ¬40
W
m8 ¬40
m2  m8 ¬90
V m2  40 ¬90
m2 ¬50
W
Since ZS , 
T ZT   TW, and m9  m10.
Y m10  m6 ¬90
m10  50 ¬90
If a quadrilateral is inscribed in a circle, then its m10 ¬40
opposite angles are supplementary. V and X m9 ¬40
are opposite angles, so m9  m4 ¬90
mV  mX  180 40  m4 ¬90
mV  28  180 m4 ¬50
mV  152 XZW is a right angle because it intercepts a
W and Y are opposite angles, so semicircle.
mW  mY  180 m3  m4 ¬90
110  mY  180 m3  50 ¬90
mY  70 m3 ¬40
7. Since XZY is a semicircle, XZY is a right angle. YXZ is a right angle because it intercepts a
So, the probability is 1. semicircle.
m2  m5 ¬90
50  m5 ¬90
Pages 549–551 Practice and Apply m5 ¬40
 and mQR. mPQ
 ¬mQR
. m5  m3  m7 ¬180
8. First find mPQ 40  40  m7 ¬180
   
mPS  mSR  mQR  mPQ ¬360 m7 ¬100
  mPQ
45  75  mPQ  ¬360
 ¬360
120  2mPQ
 ¬240
2mPQ
 ¬120
mPQ
 ¬120
mQR

357 Chapter 10
11. Given: 
AB  
DE, 
AC     mAB   120, and mBC
 .
CE B D mAD ¬mCD
   
Prove: ABC  EDC mAB  mAD  mBC  mCD ¬360
1 2   mBC
120  120  mBC 
A E ¬360

2mBC ¬120

mBC ¬60
C 
mCD ¬60
Proof: 1 
m4  2 mCD

Statements Reasons  1
2 (60) or 30
1. 
AB  DE, AC  
CE 1. Given 1
m7  mBC
    mCE
 2. Def. of  arcs
2
2. mAB  mDE, mAC
 ,  1

2 (60) or 30
3. 1mA
2 AB  1mDE
2
3. Mult. Prop.
  m4  30
1mAC 1mCE m4  m5 ¬90
2 2
4. mACB  1   4. Inscribed  30  m5 ¬90
2 mAB,
Theorem m5 ¬60
mECD  1  
2 mDE, m6 ¬60
, m2  1mCE
m1  21mAC  15. KPR is a right angle because 
KPR is a
2
semicircle.
5. mACB  mECD, 5. Substitution
m2 ¬90
m1  m2
mR  mK  mP ¬180
6. ACB  ECD, 1  2 6. Def. of   mR  mK  90 ¬180
B
7. ADE 7.  arcs have mR  mK ¬90
 chords.
1x  5  1x ¬90
8. ABC  EDC 8. AAS 3 2
5x ¬85
6
12. Given: P A
Prove: AXB CXD x ¬102
C 1 m3  mR  1
3x  5

2 X
B  1
3 (102)  5 or 39
P 

D m1  mK  1


2x
Proof: 1
 2(102) or 51
Statements Reasons 16. By the definition of a rhombus,
Q
PQ RR SS P.
1. P
Therefore, 
PQ  QR   RS  
1. Given
SP.
2. A  C 2. Inscribed  intercepting  360
same arc are . mSP   or 90
4
3. 1  2 3. Vertical  are .  
mQRP  1
2 (mPQ)
4. AXB CXD 4. AA Similarity
 1
2 (90) or 45
13. Inscribed angles that intercept the same arc are 17. m1 m2 since D EE C
.
congruent. m1 m2  90
m1 ¬m2 m1 m2  45
x ¬2x  13 m3is a right angle because 
ABC is a semicircle.
13 ¬x m1 m3  m4  180
m1  x or 13 45  90  m4 ¬180
m2  13 m4 ¬45
 
14. m8  1mAB
2
1
m5  mCF
2
 1

2 (120) or 60  1

2 (60) or 30
m2  m8 ¬90 m7 is a right angle because 
AFC is a semicircle.
m2  60 ¬90 m5  m6  m7 ¬180
m2 ¬30 30  m6  90 ¬180

BC  
CD because   so
BD  AC m6 ¬60
m1  m2   2(m6)
mAF
m1  30  2(60) or 120
m3  m8
m3  60

Chapter 10 358
 
23. mPSR  1 
2 (mPQ  mQR)
18. W
 1
2 (72  72) or 72

1   mST   mTP
)
24. mPQR  2(mRS
 1
2 (72  72  72) or 108

  
25. mPTS  mPT  mTS
 72  72 or 144
  mBZA or 104
26. mBA
  360  (mBA   mCB)
R 27. mADC
Z
 360  (104  94) or 162
T 
28. mBDA  1mBA2
If a quadrilateral is inscribed in a circle, then its
opposite angles are supplementary. W and T  1

2 (104) or 52
  mCB

are opposite angles, so 29. 2(mZAC)  180 ¬mBA
mW  mT ¬180 2(mZAC)  180 ¬104  94
45  mT ¬180 2(mZAC)  180 ¬198
mT ¬135 2(mZAC) ¬18
R and Z are opposite angles, so mZAC ¬9
mR  mZ ¬180   2mABC
30. mAC
100  mZ ¬180  2(50) or 100
mZ ¬80 
mDEF  1mDBF
2
19.
 1

2 (128) or 64
  40, then mPQS
  360  40 or 320. If T
31. If mPS
A D 
is located in PQS, then

mPTS  1mPS 2
 1

2 (40) or 20.
B C The probability that mPTS  20 is the same
as the probability that T is contained in 
PQS,
Since ABCD is a trapezoid A D
 B C
. A and B
32
 0 8
are consecutive interior angles so they are 360 or 9 .
supplementary.   110, then mPQR   360  110 or 250.
32. If mPSR
mA  mB ¬180 
If T is located in PQR, then
60  mB ¬180 
mB ¬120 mPTR  1mPSR 2
If a quadrilateral is inscribed in a circle, then its  1

2 (110) or 55.
opposite angles are supplementary. A and C
The probability that mPTR  55 is the same
as the probability that T is contained in 
are opposite angles, so
PQR,
mA  mC ¬180
25
 0 25

60  mC ¬180 360 or 36 .
mC ¬120 33. No matter where T is selected, mSTQ  1 
2 (180)
B and D are opposite angles, so or 90 because 
SPQ is a semicircle. Therefore, the
mB  mD ¬180 probability that mSTQ  90 is 1.
120  mD ¬180 
360.
34. mPTQ can never equal 180 since mPQ
mD ¬60 Therefore, the propability that mPTQ  180 is 0.
20. Sample answer: P Q is a diagonal of PDQT and a
diameter of the circle.
21. Sample answer: E F is a diameter of the circle and
a diagonal and angle bisector of EDFG.
22. Since pentagon PQRST is equilateral,
Q
P Q R
R S
S TT P
 and

PQ   QR  RS  ST   TP.
 36
0
mQR or 72
5

359 Chapter 10
35. Given: T lies inside PRQ. K 37. Given: inscribed MLN and N
K
R is a diameter of T. P Q CED,  CD  
MN D
 M
Prove: mPRQ  1mPKQ Prove: CED  MLN
2 O
T C

L E
Proof: R Proof:
Statements Reasons Statements Reasons
1. mPRQ  mPRK  1.  Addition Th. 1. MLN and CED are 1. Given
mKRQ
  mPK
  mKQ inscribed; 
CD  
MN
2. mPKQ 2. Arc Addition ;
2. mMLN  1 2
mMN 2. Measure of an
Theorem
  1mPK
 1  inscribed  
3. 1mPKQ 3. Multiplication Prop. mCED  2mCD
2 2 half measure of
1 
mKQ intercepted arc.
2  
3. mCD  mMN 3. Def. of  arcs
 
4. mPRK  1 4. The measure of an
2 mPK, 4. 1   1mMN
mCD 
inscribed  whose 4. Mult. Prop.
 
2 2
mKRQ  1
2 mKQ side is a diameter is 5. mCED  mMLN 5. Substitution
half the measure of
6. CED  MLN 6. Def. of  
the intercepted arc
(Case 1). 
38. Given: PQR is a semicircle. P Q
 
5. 1
2 mPKQ  mPRK  5. Subst. (Steps 3, 4) Prove: PQR is a right angle.
mKRQ
C
 
6. 1
2 mPKQ  mPRQ 6. Substitution
S
(Steps 5, 1) R
36. Given: T lies outside PRQ. K P Proof: Since PSR is a semicircle, 
PSR is also
  180. PQR is an
K
R  is a diameter of T. Q a semicircle and mPSR
Prove: mPRQ  1   inscribed angle, and mPQR  1  
2 mPQ 2 (mPSR ) or 90,
T making PQR a right angle.
39. Given: quadrilateral A
ABCD inscribed
Proof: R in O B
Prove: A and C are O
Statements Reasons supplementary.
B and D are D
1. mPRQ  1. Angle Addition C
mKRQ  mPRK Theorem, Subtraction supplementary.
Property
  mKQ
  mKP  Proof: By arc addition and the definitions of arc
2. mPQ 2. Arc Addition measure and the sum of central angles,
Theorem,   mDAB   360. Since mC  1mDAB 
mDCB 2
Subtraction Property
  1(mKQ
 , mC  mA  1(mDCB 
3. 1
mPQ 3. Division Property and mA  1mDCB
2 2
2 2
 ), but mDCB
  mDAB
  360, so
mKP)  mDAB
4. mPRK  1  4. The measure of an mC  mA  1
2 (360) or 180. This makes C

2 mKP,
inscribed  whose
 
mKRQ  1 and A supplementary. Because the sum of the
2 mKQ side is a diameter is
measures of the interior angles of a quadrilateral
half the measure of
is 360, mA  mC  mB  mD  360. But
the intercepted arc
mA  mC  180, so mB  mD  180,
(Case 1).
making them supplementary also.

5. mPRQ  21mKQ 5. Subst. (Steps 1, 4) 36
0
40. There are 8 congruent arcs, so each measures 

1mKP
8
2 or 45.

6. mPRQ  21(mKQ 6. Distributive 41. Isosceles right triangle because sides are congruent
) Property radii making it isosceles and AOC is a central
mKP
1  angle for an arc of 90°, making it a right angle.
7. mPRQ  2mPQ 7. Substitution 42. Square because each angle intercepts a semicircle,
(Steps 6, 3) making them 90° angles. Each side is a chord of
congruent arcs, so the chords are congruent.

Chapter 10 360
43. Square because each angle intercepts a 51. C  2r
semicircle, making them 90° angles. Each side is a C  2(12) or 24
chord of congruent arcs, so the chords are Let   length of 
QR.
congruent. 60 

360 ¬¬
24

44. Use the properties of trapezoids and inscribed 60
quadrilaterals to verify that ABCD is isosceles. 
360 (24) ¬
4 ¬
The length of 
A
QR is 4 units.
B
52. C  2r
D C  2(16) or 32
C Let   length of 
QR.
mA  mD  180 (same side interior angles 90 
 180)

360 ¬
32

mA  mC  180 (opposite angles of inscribed 90

360 (32) ¬
quadrilaterals  180) 8 ¬
mA  mD  mA  mC (Substitution) The length of  QR is 8 units.
mD  mC (Subtraction Property)
53. always
D  C (Def. of  s)
Trapezoid ABCD is isosceles because the base 54. sometimes
angles are congruent. 55. sometimes
45. Sample answer: The socket is similar to an 56. Use the converse of the Pythagorean Theorem.
inscribed polygon because the vertices of the a2  b2 ¬c2
hexagon can be placed on a circle that is 42  52 ¬62
concentric with the outer circle of the socket. 16  25 ¬36
Answers should include the following. 41
¬36
• An inscribed polygon is one in which all of its It is not a right triangle.
vertices are points on a circle. 57. Use the converse of the Pythagorean Theorem.
• The side of the regular hexagon inscribed in a a2  b2 ¬c2
circle 3
 3
4 inch wide is 8 inch.
32  82 ¬102
9  64 ¬100
46. C; mAOB  2mACB, so the ratio is 1 : 2.
73
¬100
47. There are 18 even-numbered pages and 18 It is not a right triangle.
odd-numbered pages, so there are 18  6  18  7
58. Use the converse of the Pythagorean Theorem.
or 234 articles.
a2  b2 ¬c2
282  452 ¬532
784  2025 ¬2809
Page 551 Maintain Your Skills 2809 ¬2809
48. Since AB  60, CD  30. Since DE  48, FD  24. It is a right triangle.
(FD)2  (CF)2 ¬(CD)2
242  (CF)2 ¬302
(CF)2 ¬324
CF ¬18 10-5 Tangents
49. Draw a line from C to E. Since AB  32, CE  16.
Draw CE. Use the Pythagorean Theorem.
(FE)2  (FC)2  (CE)2 Page 552 Geometry Software Investigation:
(FE)2  112 ¬(16)2 Tangents and Radii
FE2 ¬135 1. 
WX is a radius.
FE ¬135 2. WX WY
FE ¬11.62 3. It doesn’t, unless Y and X coincide.
50. Since DE  60, FD  30. Use the Pythagorean 4. X
W   XY
Theorem.
5. Sample answer: The shortest distance from the
(FD)2  (FC)2 ¬(CD)2
center of a circle to the tangent is the radius of
302  162 ¬(CD)2
the circle, which is perpendicular to the tangent.
1156 ¬(CD)2
34 ¬CD
Since AB  2(CD), AB  2(34) or 68.
Page 555 Check for Understanding
1a. Two; from any point outside the circle, you can
draw only two tangents.
1b. None; a line containing a point inside the circle
would intersect the circle in two points. A
tangent can only intersect a circle in one point.

361 Chapter 10
1c. One; since a tangent intersects a circle in exactly 10. Determine whether JGH is a right triangle.
one point, there is one tangent containing a point (JG)2  (GH)2 ¬(JH)2
on the circle. 52  122 ¬142
2. If the lines are tangent at the endpoints of a 169
¬196
diameter, they are parallel and thus, not Because the converse of the Pythagorean Theorem
intersecting. did not prove true in this case, JGH is not a
3. Sample answer: right triangle.
polygon circumscribed about a circle No; GH  is not tangent to J.
11. Determine whether KLM is a right triangle.
(KL)2  (LM)2 ¬(KM)2
2
102  62 ¬ 136
136 ¬136
Because the converse of the Pythagorean Theorem
is true, KLM is a right triangle with right angle
polygon inscribed in a circle KLM and L MK L
.
L
Yes; K  is tangent to M.
For Exercises 12–15, use the Pythagorean
Theorem.
12. (NO)2  (NP)2 ¬(OP)2
62  x2 ¬102
36  x2 ¬100
4. Triangle MPO is a right triangle with hypotenuse x2 ¬64
O
M . Use the Pythagorean Theorem. x ¬8
(MP)2  (PO)2 ¬(MO)2 13. (QR)  (RS) ¬(QS)2
2 2
162  x2 ¬202 122  x2 ¬(12  8)2
256  x2 ¬400 144  x2 ¬400
x2 ¬144 x2 ¬256
x ¬12 x ¬16
5. If PRO is a right triangle, then 14. (WU)2  (UV)2 ¬(WV)2
R
P  is tangent to O. Use the converse of the 122  72 ¬x2
Pythagorean Theorem. 144  49 ¬x2
(PR)2  (PO)2 ¬(RO)2 193 ¬x2
52  122 ¬132 193
 ¬x
25  144 ¬169 15. (AC)2  (AB)2 ¬(BC)2
169 ¬169 82  x2 ¬172
R
Yes, P  is tangent to O. 64  x2 ¬289
6. 4(3)  4x ¬32 x2 ¬225
4x ¬20 x ¬15
x ¬5 16. DE ¬DF
7. Each side of the square is 2(72) or 144 feet. The x  2 ¬14
total length of fence is 4(144) or 576 feet. x ¬16
17. HJ ¬HN
HJ ¬2
Pages 556–558 Practice and Apply HK ¬HJ  JK
8. Determine whether ABC is a right triangle. 5 ¬2  JK
(AB)2  (BC)2 ¬(AC)2 3 ¬JK
162  302 ¬342 KL ¬JK
1156 ¬1156 KL ¬3
Because the converse of the Pythagorean Theorem 18. RS  RQ
is true, ABC is a right triangle with right angle 6
ABC and A BB C. ST ¬TU
C
Yes; B  is tangent to A. 4
9. Determine whether DEF is a right triangle. RT ¬RS  ST
(EF)2  (DF)2 ¬(DE)2 x ¬6  4 or 10
32  42 ¬52 AC
25 ¬25 19. sin B ¬
BC
Because the converse of the Pythagorean Theorem 15
sin 30° ¬ x

is true, DEF is a right triangle. Since 
DE is the
15
longest side, F is the right angle. E is not a x ¬
sin 
30° or 30
right angle, so 
DE is not perpendicular to 
EF.
No; DE is not tangent to F.

Chapter 10 362
20. (DG)2  (DE)2 ¬(EG)2 26. CF ¬CE
x2  162 ¬(12  x)2 6(3  x) ¬3x
x2  256 ¬144  24x  x2 18  6x ¬3x
112 ¬24x 18 ¬9x
2 ¬x
4 2
3 ¬x

CE  3x CF ¬6(3  x)
21. See students’ work.  3(2) or 6 ¬6(3  2) or 6
22. Given:   AB B BE ¬BD
B
A  is a radius of A. 4y ¬12y  4
Prove:  is tangent to A. A 4 ¬8y
1 ¬y
2
Proof: Assume  is not tangent to A. Since
 touches A at B, it must touch the circle in BE  4y BD ¬12y  4
another place. Call this point C. Then AB  AC. ¬41
2  or 2
 ¬121
2   4 or 2

B
But if A  , AB must be the shortest distance AF ¬AD
between A and . There is a contradiction. 10(z  4) ¬2z
Therefore,  is tangent to A. 10z  40 ¬2z
23. Let r  the radius of M. 40 ¬8z
(PL)2  (ML)2 ¬(PM)2 5 ¬z
102  r2 ¬(r  2)2 AF  10(z  4) AD ¬2z
100  r2 ¬r2  4r  4  10(5  4) or 10 ¬2(5) or 10
96 ¬4r CE  CF  BE  BD  AF  AD  6  6 
24 ¬r 2  2  10  10 or 36
PL  ML  MN  NP  10  24  24  2 or 60 The perimeter is 36 units.
The perimeter is 60 units. 27. Given: AB is tangent B
24. R to X at B.
x C
A  is tangent X A
x to X at C.
Prove: ABA C
 C
5 13 Proof:
O
Statements Reasons
5 5
1. 
AB is tangent to X 1. Given
S 5 13
T C
at B, A  is tangent to
X at C.
18
2. Draw B X, C
X, and A
X
. 2. Through any two
Use the Pythagorean Theorem to write an points, there is one
equation for RST. line.
(x  5)2  182 ¬(x  13)2 B
3. A B X, A
CC X
 3. Line tangent to a
x  10x  25  324 ¬x2  26x  169
2
circle is  to the
180 ¬16x radius at the pt. of
11.25 ¬x tangency.
11.25  5  18  13  11.25 ¬58.5 4. ABX and ACX are 4. Def. of  lines
The perimeter is 58.5 units. right angles.
25. d  5, r  2.5, GY  EG  2.5. Since C
B
 is a X
5. B C X
 5. All radii of a circle
tangent, AEB is a right angle and GEB is a are .
right triangle. Use the Pythagorean Theorem.
X
6. A A X
 6. Reflexive Prop.
(EG)2  (EB)2 ¬(GB)2
(2.5)2  (EB)2 ¬(2.5  2.5)2 7. ABX  ACX 7. HL
6.25  (EB)2 ¬25 B
8. A A C
 8. CPCTC
(EB)2 ¬18.75
28. Let a  the radius of the roll of film, b  the
EB ¬ 18.75
amount of film exposed, and c  the distance from
EB  EC  DC  DA  FA  FB
the center of the roll to the intake of the holding
6EB  6 18.75 or 15 3
chamber. Since the diameter of the roll of film is
The perimeter is 15 3 units.
25, a  12.5. Use the Pythagorean Theorem.
a2  b2 ¬c2
12.52  b2 ¬1002
156.25  b2 ¬10,000
b2 ¬9843.75
b ¬99
About 99 millimeters of film would be exposed.

363 Chapter 10
E
29. A and B
F
  
mAFC ¬1
2 mAC
D
30. A and B
C ¬1
2 (90) or 45
31. 12;  
BD ¬AC
G N
mBED ¬mAFC
4 4
mBED ¬45
5Q L
13 38. Connect L to J to form a right triangle. Use the
P Pythagorean Theorem.
(LJ)2 ¬x2  52
102 ¬x2  52
Draw PG, N
L, and P
L
. Construct LQ
G P, thus 100 ¬x2  25
LQGN is a rectangle. GQ  NL  4, so QP  5. 75 ¬x2
Using the Pythagorean Theorem, (QP)2  (QL)2 53 ¬x or x  8.7
 (PL)2. So, QL  12. Since GN  QL, GN  12. 39. KB  5  2 or 3
32. Sample answer: Many of the field events have the Use the Pythagorean Theorem.
athlete moving in a circular motion and releasing 32  x2 ¬52
an object (discus, hammer, shot). The movement of 9  x2 ¬25
the athlete models a circle and the path of the x2 ¬16
released object models a tangent. x ¬4
Answers should include the following. 40. Connect O to A to form a right triangle; OA  8.
• The arm of the thrower, the handle, the wire, Let x  1
2 AP.
and hammer form the radius defining the circle
Use the Pythagorean Theorem.
when the hammer is spun around. The tangent
42  x2 ¬82
is the path of the hammer when it is released.
16  x2 ¬64
• The distance the hammer was from the athlete
x2 ¬48
was about 70.68 meters.
x ¬4 3
33. A AP  24 3 or 8
3  13.9
x x 41. Sample answer:
R Given: ABCD is a rectangle. E is the midpoint
S
x B
of A .


y y
19

D(0, b) C(2a, b)
B
D
y
A(0, 0) E(a, 0) B(2a, 0) x
P Q
6 y Prove: CED is isosceles.
 Proof: Let the coordinates of E be (a, 0). Since E
14
C is the midpoint and is halfway between A and B,
PB  RB ¬19  x the coordinates of B would be (2a, 0). Let the
CP  CQ ¬14  y coordinates of D be (0, b), so the coordinates of
PB  CP ¬6 C would be (2a, b) because it is on the same
(19  x)  (14  y) ¬6 horizontal as D and the same vertical as B.
33  x  y ¬6 ED   (a  0)2  (0 b)2
27 ¬x  y  a  b
2 2
AD ¬27 EC   
(a  2a) 2 (0  b )2
34. B; 2  12  22      92  470  a  b
2 2
102  112  122      192  1470
Since ED  EC, E D
E C. DEC has two




congruent sides, so it is isosceles.


902  912  922      992  9470 42. x  3  1
2 [(4x  6)  10]

470  1470  2470      9470  

49,7
00 1
100 100 or 497 x  3  2(4x  4)
D
35. A and B
C x  3  2x  2
E
36. A and B
F
 or A
C
 and B
D
 5x
43. 2x  5  1
2 [(3x  16)  20]

1
2x  5  2(3x  4)
Page 558 Maintain Your Skills
 ¬mFA   mAC
 2x  5  32x  2
37. mFAC 

180 ¬90  mAC 1x ¬3
 2
90 ¬mAC
x ¬6

Chapter 10 364
44. 2x  4  1
2 [(x  20)  10]

1 10-6 Secants, Tangents, and Angle
2x  4  2(x  10)
Measures
2x  4  1
2x  5

3
x ¬1
2 Page 564 Check for Understanding
x ¬2
3
1. Sample answer: A tangent intersects the circle in
1 only one point and no part of the tangent is in the
45. x  3  2[(4x  10)  45] interior of the circle. A secant intersects the circle
x  3  1
2 (4x  35)
 in two points and some of its points do lie in the
interior of the circle.
35
x  3  2x   
2 2. Sample answer:
41
2 ¬x or x  20.5
A T

Page 560 Geometry Activity: Inscribed


and Circumscribed Triangles C
1. See students’ work. Angle TAC is a right angle; There are several
2. See students’ work. reasons: (1) If the point of tangency is the
3. See students’ work. endpoint of a diameter, then the tangent is
4. The incenter is equidistant from each side. The perpendicular to the diameter at that point. (2)
perpendicular to one side should be the same The arc intercepted by the secant (diameter) and
length as it is to the other two sides. the tangent is a semicircle. Thus the measure of
the angle is half of 180 or 90.
5. The incenter is equidistant from all the sides. The
radius of the circle is perpendicular to the tangent 3. m1  180  1
2 (38  46)

sides and all radii are congruent, matching the 1
¬180  2(84)
distance from the incenter to the sides.
6. The circumcenter is equidistant from all three ¬180  42 or 138
vertices, so the distance from the circumcenter to 4. m2  1
2 (360  100)

one vertex is the same as the distance to each of
¬1

2 (260) or 130
the others.
7. The circumcenter is equidistant from the vertices 5. x  1
2 [84  (180  84  52)]

and all of the vertices must lie on the circle. So,
¬1
2 (84  44)

this distance is the radius of the circle containing
the vertices. ¬1

2 (40) or 20
36
0
8. 6  60 6. x  1
2 [128  (360  148  128)]

9. 1
30 ¬2(128  84)
r 3
¬1

2 (44) or 22
60 7. 55 ¬1
2 [x  (360  x)]

r
1
55 ¬2(2x  360)
55 ¬x  180
Suppose all six radii are drawn. Each central 235 ¬x
angle measures 60°. Thus, six 30°-60°-90°  
8. mCAS ¬1
2 mSA
triangles are formed. Each triangle has a side
which is a radius r units long. Using 30°-60°-90° ¬1

2 (46) or 23

side ratios, the segment tangent to the circle 9. mSAK  2mSLK
has length r 3, making each side of the  2(78) or 156
   mSA 
circumscribed triangle 2r 3. If all three sides mAK  mSAK
have the same measure, then the triangle is  156  46 or 110

equilateral. mQAK  1mAK
2
10. The incenter is the point from which you can
construct a circle “in” the triangle. Circum means  1

2 (110) or 55
around. So the circumcenter is the point from
which you can construct a circle “around” the
triangle.

365 Chapter 10
A
10. Since S LK, ASL and SLK are 23. 25 ¬1
2 (90  5x)

supplementary consecutive interior angles. 50 ¬90  5x
So mASL  180  78 or 102 and mAKL 
40 ¬5x
 2(mASL)  2(102) or 204. 8 ¬x
  mAK   mKL
mAKL
 24. 360  (160  34  106) ¬60
204 ¬110  mKL (See Exercise 9.)
 x ¬1
2 (60  34)
94 ¬mKL 
    mSL¬360
11. mSA  mAK  mKL x ¬1

 2 (26) or 13
46  110  94  mSL¬360
¬360 25. x ¬1
2 (7x  20)
250  mSL 
¬110
mSL 2x ¬7x  20
20 ¬5x
4 ¬x
Pages 564–567 Practice and Apply 26. x ¬1
2 (10x  40)

12. m3 ¬1
2 (100  120)

x ¬5x  20
1 20 ¬4x
 2(220) or 110
5 ¬x
13. m4  1
2 (45  75)

27. x  2.5 ¬1
2 [(4x  5)  50]

1
 2(120) or 60 2x  5 ¬4x  45
14. m5  1 50 ¬2x
2 [360  (110  150)]

25 ¬x
1
¬2(360  260) 28. 90  60 ¬30
¬1 30 ¬1
2 (105  5x)

2 (100) or 50
15. 5a  3a  6a  4a ¬360 60 ¬105  5x
18a ¬360 5x ¬45
a ¬20 x ¬9
m6  1(5a  6a) 29. 50 ¬1
2 [(360  x)  x]

2
¬1
 100 ¬360  2x
2 (11a) 2x ¬260
¬1
2 (11  20)
 x ¬130
¬1
 30. 30 ¬1
2 [x  (360  x)]

2 (220) or 110
60 ¬2x  360
16. m7  1

2 (196) or 98 420 ¬2x
17. m8  1
 210 ¬x
2 (180) or 90
18. m9  1 31. 3x  1
2 [(4x  50)  30]

2 (360  120)

1
 1
 3x  2(4x  20)
2 (240) or 120
3x  2x  10
19. m10 ¬1
2 [360  (100  160)]
 x ¬10
1
¬2(360  260) 32. 40 ¬1
2 [(x  12x)  (x  2x)]
 2 2

¬1

2 (100) or 50
1
40 ¬2(10x)
 
20. 65 ¬1 40 ¬5x
2 (mAC  72)
 8 ¬x
130 ¬mAC  72
 33. mC  1
2 (116  38)
58 ¬mAC 

21. x  1
2 (90  30)
 1
 2(78) or 39
1   mBH )
¬2(60) or 30 mC ¬1(mBAH
2
1  
22. x ¬1
2 [20  (180  20  150)]
 39 ¬ 2 [(360  mBH)  mBH]


1
¬2(20  10) 39 ¬1
2 (360  2mBH)


¬1 39 ¬180  mBH
2 (10) or 5 
mBH ¬141

Chapter 10 366
  2mEFB
34. mBE 40b. Given: 
DG is a tangent to the circle. 
DF is a
 2(30) or 60 secant to the circle.
Prove: mFDG  1   
mFGE  180  52 or 128 2 mFG  mGE
mFGE  mEFB  mGEF ¬180 F
128  30  mGEF ¬180
E
mGEF ¬22
  2mGEF
mCF D G H
 2(22) or 44 Proof:
  360  (mAB   mBE
  mFE  mCF
)
mAC
 360  (108  60  118  44) Statements Reasons
 360  330 or 30  is a tangent to the 1. Given
1. DG
  44 (See Exercise 34.)
35. mCF  is a secant
circle. DF
36. mEDB ¬1  
2 (mBE  mAC)
to the circle.
¬1 2. mDFG  1   2. The meas. of an
2 (60  30)
 2 mGE,
 inscribed   1 
¬1
 mFGH  1 mFG 2
2 (30) or 15 2
the meas. of its
37. mAMB ¬23.5 intercepted arc.
mAMB ¬1   
2 (mBC  mAB) 3. mFGH  3. Exterior 
 
23.5 ¬1
2 (mBC  71) mDFG  mFDG Theorem
47 ¬mBC  71   1mGE

 4. 1mFG
2 2
4. Substitution
118 ¬mBC
   mFDG
38. mABC  mAB  mBC
  
5. 1 1 
2 mFG  2 mGE
mABC  71  118 or 189 5. Subtraction Prop.
No, its measure is 189, not 180.  mFDG
39. C  d  
6. 1 
2 mFG  mGE
6. Distributive Prop.
C  (100)  314.2 feet
11
8  mFDG
360  314.2  103

You would walk about 103 ft. 40c. Given: 
HI and 
HJ are tangents to the circle.
 and AT
 are secants to the circle. Prove: mIHJ  1   
40a. Given: AC 2 (mIXJ  mIJ)
  mBR )
Prove: mCAT  1(mCT 2 I X
C
B
A
R T H J K
Proof: Proof:
Statements Reasons
Statements Reasons
1. 
HI and 
HJ are 1. Given
1. 
AC and 
AT are 1. Given
tangents to the circle.
secants to the circle.
2. mCRT  1   2. mIJK  1   2. The measure of an
2 mCT,
2. The meas. of an 2 mIXJ,
 1  1
inscribed   2
mACR  1mBR inscribed   1  mHIJ  2mIJ
2 2
the meas. of its the measure of its
intercepted arc. intercepted arc.
3. mCRT  3. Exterior  3. mIJK  mHIJ  3. Ext.  Th.
mACR  mCAT Theorem mIHJ
  1mIJ

  1mBR
4. 1mCT  4. Substitution 4. 1mIXJ
2 2 4. Substitution
2 2
mCAT mIHJ
  1   
5. 1 1 
5. 1
2 mCT  2 mBR
5. Subtraction Prop. 2 mIXJ  2 mIJ 5. Subtr. Prop.

 mCAT  mIHJ
    
6. 1 
6. 1
2 mCT  mBR
6. Distributive Prop. 2 (mIXJ  mIJ) 6. Distrib. Prop.
 mIHJ
 mCAT

367 Chapter 10
41. The diagonals of a rhombus are perpendicular.   1mCA
1mDC  by substitution.
2 2
Let x  side length of rhombus.
mDAC  1  
52  122 ¬x2 2 mDC since DAC is inscribed, so
25  144 ¬x2  
substitution yields 1 1 
2 mDC  mCAB  2 mDC
169 ¬x2 . By Subtraction Prop., mCAB  1mCA
.
13 ¬x  21mCA 2
y  is a tangent to O.
43b. Given: AB
 is a secant to O. CAB is obtuse.
AC
r Prove: mCAB  1  
5 2 mCDA
D
12 5 C O

x E A B
Let r  radius of inscribed circle. Proof: CAB andCAE form a linear pair, so
The radius forms two right triangles. One triangle mCAB  mCAE  180. Since CAB is
has a hypotenuse of 5. Let y  measure of the obtuse, CAE is acute and Case 1 applies, so
portion of the side of the rhombus that is a leg of . mCA   mCDA
  360, so
mCAE  1mCA
2
this triangle. The second right triangle has a
hypotenuse of length 12. Its second leg has measure   1mCDA
1mCA   180 by Division Prop., and
2 2
13  y.   180 by substitution. By
1mCAE  1mCDA
Use the Pythagorean Theorem to write an 2 2
equation for each triangle. the Transitive Prop., mCAB  mCAE 
r2  (13  y)2  122
r2  y2 ¬52 } mCAE  1mCDA
2
, so by Subtraction Prop.,

r2 ¬25  y2 mCAB  1  
2 mCDA.
Substituting for r2,
25  y2  (13  y)2 ¬144 44. Let x  the measure of the angle of view.
25  y2  169  26y  y2 ¬144 360  54  306
194  26y ¬144 x  1
2 (306  54)

26y ¬50 1
x  2(252) or 126
y ¬1.9
r2  25  1.92 , m1  1mRQ
 so
45. 3, 1, 2; m3  mRQ 2
r2  21.39   mTP
) 
r  4.6 m3 m1, m2  1(mRQ
2
The radius of the inscribed circle is approximately   1mTP
1mRQ , which is less than 1mRQ
,
2 2 2
4.6 cm.
so m2 m1.
42. Let x  measure of intercepted arc.
46. Sample answer: Each raindrop refracts light from
86.5 ¬1
2 [(360  x)  x]

the sun and sends the beam to Earth. The
173 ¬360  2x raindrop is actually spherical, but the angle of the
2x ¬187 light is an inscribed angle from the bent rays.
x ¬93.5 Answers should include the following.
43a. Given: AB is a tangent to O. • C is an inscribed angle and F is a secant-
 is a secant to O. CAB is acute.
AC secant angle.
 • The measure of F can be calculated by finding
Prove: mCAB  1mCA 2  and the
the positive difference between mBD
D
measure of the small intercepted arc containing
point C.
O C
47. A; let x  the measure of the common intercepted
arc of A and B.
A B
mA ¬1
2 (x  15)

Proof: DAB is a right  with measure 90, and
 10 ¬1
2 (x  15)
DCA is a semicircle with measure 180, since if a 
line is tangent to a , it is  to the radius at the 20 ¬x  15
point of tangency. Since CAB is acute, C is in 35 ¬x
the interior of DAB, so by the Angle and Arc
mB  1
2 (95  35)

Addition Postulates, mDAB  mDAC 
  mDC   mCA . By 1
mCAB and mDCA ¬2(60) or 30
substitution, 90  mDAC  mCAB and
48. C; the set of data can be represented by the linear
    1 
180  mDC  mCA. So, 90  1 2 mDC  2 mCA equation y  12 x  1.

by Division Prop., and mDAC  mCAB 

Chapter 10 368
Page 568 Maintain Your Skills 57. x2  6x  40 ¬0
49. The tangent forms a right angle with the radius. (x  4)(x  10) ¬0
Use the Pythagorean Theorem. x  4  0 or x  10 ¬0
(2x)2  242 ¬(16  24)2 x ¬4 x  10
4x2  576 ¬1600 58. 2x2  7x  30 ¬0
4x2 ¬1024 (2x  5)(x  6)  0
x2 ¬256 2x  5  0 or x  6  0
x ¬16 x  5
 x  6
2
50. Both tangents to the circle on the left have the 1
x  22
same measure. Both tangents to the circle on the
right have the same measure. Both tangents to 59. 3x2  24x  45 ¬0
the circle in the middle have the same measure. 3(x2  8x  15)  0
Because the left tangent to the middle circle is 3(x  3)(x  5) ¬0
also tangent to the left circle and the right x  3  0 or x  5  0
tangent to the middle circle is also tangent to the x3 x5
right circle, the tangents to the left and right
circles have the same measure. Page 568 Practice Quiz 2
12x  10 ¬74  4x 1. Each central angle has a measure of 36
0
8 or 45.
16x ¬64 Therefore, the remaining 2 angles in each triangle
x ¬4 180  45
each measure  2
 or 67.5.
51. EGN is an inscribed angle.
 2. Inscribed angles of the same arc are congruent.
mEGN  1mEN
2 m1  m2  1 
2 (68) or 34
 1
2 (66) or 33 3. x  2(6) or 12
 ¬mGE   mEN
52. mGEN
 4. x  1
2 (60  34)

180 ¬mGE  66

114 ¬mGE  1

2 (26) or 13

mGME  1mGE
2 5. 360  129  231
1
 2 (114) or 57
 x  1

2 (231) or 115.5
  89
53. mGM
GNM is an inscribed angle that intercepts 
GM.
 
1
mGNM  (mGM)
2 10-7 Special Segments in a Circle
 1
2 (89) or 44.5
vertical rise Page 569 Geometry Activity: Intersecting
54. slope   
horizontal run
1 Chords
slope  
12 1. PTS  RTQ (vertical  are );
1
The slope is 
12. P  R ( intercepting same arc are );
55. 30 feet  30  12 or 360 in. S  Q ( intercepting same arc are )
Let x  the height of the ramp. 2. They are similar by AA Similarity.
(12x)2  x2  3602 PT ST
RT  TQ or PT  TQ  RT  ST
3.  
145x2 ¬129,600
x2 ¬893.8

x ¬30
Pages 571–572 Check for Understanding
The ramp is about 30 in. high.
1. Sample answer: The product equation for secant
56. Given: ACB F
 segments equates the product of exterior segment
Prove: AB  CF measure and the whole segment measure for each
F secant. In the case of secant-tangent, the product
B C involving the tangent segment becomes (measure
A of tangent segment)2 because the exterior
Proof: By definition of congruent segments, segment and the whole segment are the same
AC  BF. Using the Segment Addition Postulate, segment.
we know that AC  AB  BC and BF  BC  2. Latisha; the length of the tangent segment
CF. Since AC  BF, this means that AB  BC  squared equals the product of the exterior secant
BC  CF. If BC is subtracted from each side of segment and the entire secant, not the interior
this equation, the result is AB  CF. secant segment.

369 Chapter 10
3. Sample answer: x  2 ¬0 or x  10 ¬0
B x ¬2 x ¬10
A Disregard the negative value.
x ¬2
C 12. x2 ¬3(9  3)
x2 ¬36
D x ¬6
4. 9x ¬3(6) 13. 42 ¬2(x  2)
9x ¬18 16  2x  4
x ¬2 12  2x
5. 312 ¬20(20  x) 6 ¬x
961 ¬400  20x 14. 162 ¬x(x  x  16)
561 ¬20x 256 ¬x(2x  16)
28.1 ¬x 256 ¬2x2  16x
6. x(10  x) ¬3(3  5) 0 ¬2x2  16x  256
10x  x2 ¬24 0 ¬x2  8x  128
x  10x  24 ¬0
2 0 ¬(x  16)(x  8)
(x  2)(x  12) ¬0 0 ¬x  16 or 0 ¬x  8
x  2 ¬0 or x  12 ¬0 16 ¬x 8 ¬x
x ¬2 x ¬12 Since the length of a segment cannot be negative,
Since x represents a length, it must be positive. reject x ¬16. So x ¬8.
Reject the negative value. So x  2. 15. (9.8)2  7.1(2x  7.1)
7. Draw a model using a circle. Let x represent the 96.04  14.2x  50.41
unknown measure of the segment of diameter  AB. 45.63  14.2x
Use the products of the lengths of the intersecting 3.2  x
chords to find the length of the diameter. 16. 4(4  2)  3(3  x)
A 24 ¬3(3  x)
8 ¬3  x
3 5 ¬x
E 17. x(5  x) ¬3(3  9)
D C 5x  x2 ¬36
3.5 3.5
x  5x  36 ¬0
2

x (x  4)(x  9) ¬0
x  4 ¬0 or x  9 ¬0
x ¬4 x ¬9
B
Disregard the negative value.
AE  EB ¬DE  EC x ¬4
3x ¬3.5  3.5 18. x(x  5  x)  5(5  5  x)
x ¬4.08 x(5  2x) ¬5(10  x)
AB ¬AE  EB 5x  2x2 ¬50  5x
AB ¬3  4.08 or 7.08 2x2 ¬50
7.08
The radius of the circle is about  
2 or 3.54. x2 ¬25
The ratio of the arch width to the radius of the x ¬5
circle is about 7:3.54.
19. x(x  3x) ¬8(8  x  2)
x(4x) ¬8(10  x)
4x2 ¬80  8x
Pages 572–574 Practice and Apply 4x  8x  80 ¬0
2
8. 2x ¬4(5) 4(x  2x  20) ¬0
2x ¬20 (2)  (2)
 
2 )
4(1)(20
x ¬10 x   2(1)
9. 6  6  x  9 2   84
36 ¬9x x  
2
4 ¬x x  5.6 or x  3.6
10. 7  2 ¬3  x Disregard the negative value. So x  5.6.
14 ¬3x
14
3 ¬x or x  4.7

11. x(x  8) ¬5  4
x2  8x ¬20
x2  8x  20 ¬0
(x  2)(x  10)  0

Chapter 10 370
20. Let r  the radius of the circle. 2x(2x  24) ¬y(y  12.25)
2(3)(2  3  24) ¬y2  12.25y
180 ¬y2  12.25y
0 ¬y2  12.25y  180
2
12.25  (12.25
) 80)
2  4(1)(1
y ¬ 
2(1)
4.25 x
2 y ¬8.6 or y ¬20.9
Disregard the negative value. So y ¬8.6.
27. 3(3  x) ¬4(4  9)
Let x represent the unknown measure of the 9  3x ¬52
segment of the diameter. Use the products of the 3x ¬43
lengths of the intersecting chords to find the x ¬14.3
length of the diameter. 28. 102 ¬y(6  y)
4.25x ¬2  2 100 ¬6y  y2
x ¬0.94 0 ¬y2  6y  100
d ¬4.25  0.94 or 5.19 mm 6  6 00)
2  4(1)(1
y ¬ 
2(1)
r ¬1 1
2 d ¬ 2 (5.19) or 2.6 mm

y ¬7.4 or y ¬13.4
21. Given: 
WY and ZX
 W Disregard the negative value. So y ¬7.4.
intersect at T. X 29. Let r  the radius of the circle. Draw a model
Prove: WT  TY  ZT  TX using a circle. Let x represent the unknown
T
measure of the segment of the diameter. Use the
Y products of the lengths of the intersecting chords
Proof: Z
to find the diameter.
Statements Reasons 60x ¬100  100
a. W  Z, a. Inscribed angles that 
x ¬166.6
X  Y intercept the same
arc are congruent. 60
b. WXT ZYT b. AA Similarity
100 100
WT TX c. Definition of similar
ZT  TY
c.  
triangles x
d. WT  TY  ZT  TX d. Cross products

22. 4(9) ¬(8  x)(8  x)


36 ¬64  x2
  60 or 226.6 cm
d ¬166.6
x2 ¬28 r ¬1 1  
2 d ¬ 2 (226.6) or 113.3 cm

x ¬5.3 
The radius is 113.3 cm.
23. 2(5  3) ¬y2 30. Given:  and E
EC B are secant segments.
16 ¬y2 Prove: EA  EC  ED  EB
4 ¬y
24. 12 ¬6(6  y  3)
2 B D
144 ¬54  6y
90 ¬6y E
15 ¬y C A

2x ¬3y Proof:
2x ¬3(15)
2x ¬45 Statements Reasons
x ¬22.5 1. 
EC and E
B are 1. Given
25. 10(10  8) ¬9(9  x) secant segments.
180 ¬81  9x 2. DEC  AEB 2. They name the same
99 ¬9x angle. (Reflexive Prop.)
11 ¬x 3. ECD  EBA 3. Inscribed  that
26. 92 ¬x(x  24) intercept the same
81 ¬x2  24x arc are .
0 ¬x2  24x  81 4. ABE DCE 4. AA Similarity
0 ¬(x  3)(x  27) EA EB
0 ¬x  3 or 0 ¬x  27 5. 
ED 
EC 5. Definition of similar
3 ¬x 27 ¬x triangles
Disregard the negative value. So x ¬3. 6. EA  EC  ED  EB 6. Cross Products

371 Chapter 10
S
31. Given: tangent R and secant 
US Page 574 Maintain Your Skills
Prove: (RS)2  US  TS R 36. 360  102  258
S m1  1

2 (258) or 129
37. m2  1
2 (85  230)

T  1

2 (315) or 157.5

Proof: U 38. m3  1


2 (28  2  12)


 1

2 (52) or 26
Statements Reasons
39. x  7
S
1. tangent R  and 1. Given
S
secant U  40. Connect the center with the point of tangency,
 
forming a right triangle. Use the Pythagorean
2. mRUT  1 2 mRT
2. The measure of an
inscribed angle Theorem.
equals half the (12  x)2 ¬122  162
measure of its 144  24x  x2 ¬144  256
intercepted arc. x2  24x  256 ¬0
(x  8)(x  32) ¬0
 
3. mSRT  1
2 mRT 3. The measure of an
x  8 ¬0 or x  32 ¬0
angle formed by a
x ¬8 x ¬32
secant and a tangent
Disregard the negative value. So x ¬8.
equals half the
measure of its 41. x  36
intercepted arc. 42. tan 67° ¬5x
4. mRUT  mSRT 4. Substitution 5 tan 67° ¬x
5. SUR  SRT 5. Definition of 12 ¬x
congruent angles The distance across the stream is about 12 feet.
6. S  S 6. Reflexive Prop. 43. No two sides are congruent, and one angle is
greater than 90. The triangle is scalene and obtuse.
7. SUR SRT 7. AA Similarity
RS TS 44. Two sides are congruent and one angle is a right
8. 
US 
RS
 8. Definition of similar angle. The triangle is isosceles and right.
triangles
45. All of the sides are congruent and all three angles
9. (RS)2  US  TS 9. Cross Products congruent. The triangle is equilateral, acute or
equiangular.
32. ZY ¬XY
(WX)2 ¬XY  XZ 46. d  (x 2 x1)2 
(y2 y1)2
(WX)2 ¬XY(XY  ZY) d  
(10 (2))2
 (1
2  7)2
(WX)2  XY(2XY) d  
144  25
(WX)2 ¬2(XY)2
d  
169 or 13
WX ¬ 2(XY)2
WX ¬ 2  XY 47. d  (x
 2
x1)2 
(y2 
y1)2
33. Sample answer: The product of the parts of one d  
(3  1)
2  (4
 7)2
intersecting chord equals the product of the parts d  
4  9 or 13
of the other chord. Answers should include the
following. 48. d  (x
 2
x1)2 
(y2 
y1)2
•AF, 
FD, 
E
F, F
B d  
(15 9)2 
(2 
(4))2
• AF  FD  EF  FB
d  
36  4 or 40

34. D; x2 ¬20  x
x2  x  20 ¬0
(x  4)(x  5) ¬0
x  4  0 or x  5 ¬0
x ¬4 x ¬5
35. C; let x  time working together.
x x
15 
30 ¬1

30  x
5 
1 
x
0 ¬30(1)
3
2x  x ¬30
3x ¬30
x ¬10
It will take them 10 minutes working together.

Chapter 10 372
The center is at (3, 0), and the radius is 4.
Equations of Circles y

Pages 577–578 Check for Understanding


1. Sample answer: (3, 0)
y O x

O x
8. Explore: You are given three points that lie on a
circle.
Plan: Graph NMQ. Construct perpendicular
bisectors of two sides to locate the center. Find the
length of the radius. Use the center and radius to
2. A circle is the locus of all points in a plane write an equation.
(coordinate plane) a given distance (the radius) y
from a given point (the center). The equation of a
circle is written from knowing the location of the Q(2, 2)
given point and the radius.
3. (x  h)2  (y  k)2 ¬r2 O x
[x  (3)]2  (y  5)2 ¬102
(x  3)2  (y  5)2 ¬100
4. (x  h)2  (y  k)2 ¬r2 M(2, 2)
(x  0)2  (y  0)2 ¬( 7)2 N(2, 2)
x2  y2 ¬7
5. First find the length of the diameter and radius. Solve: The center is at (0, 0).
d  (x  r  
(0  2
)2  [0
 (2
)]2 or 8

2 x1)2 (y2 
y1)2
d  
(6 
2)2 
(15 
7)2 Write an equation.
(x  0)2  (y  0)2  (8
)2
d    or 82
64  64  x2  y2  8
8 
2
r   or 42  y
2
The center is the midpoint of the diameter: Q (2, 2)

2
C 2(6)
, 7 
2
15

or C(2, 11)
O
(x  h)2  (y  k)2 ¬r2 x
[x  (2)]2  (y  11)2 ¬(42 )2
(x  2)  (y  11) ¬32
2 2 M N(2, 2)
( 2, 2)
6. (x  5)2  (y  2)2  9
(x  h)2 ¬(x  5)2 (y  k)2 ¬(y  2)2
x  h ¬x  5 y  k ¬y  2 Examine: Verify the location of the center by
h ¬5 k ¬2 finding the equations of the two bisectors and
h ¬5 k ¬2 solving a system of equations.
r2  9, so r  3  bisector of M N: M
N is horizontal so its 
The center is at (5, 2), and the radius is 3. bisector is vertical and goes through the midpoint
y N
of M : (0, 2).
Its equation is x  0.
 bisector of Q N: Q
N is vertical so its  bisector
is horizontal and goes through the midpoint of
( 5, 2) N
Q : (2, 0).
Its equation is y  0.
The intersection of x  0 and y  0 is the point
O x (0, 0).
So the center is correct.
9. The center is at (0, 0) and the radius is 4(10) or 40.
7. (x  3)2  y2  16 (x  h)2  (y  k)2 ¬r2
Write the equation in standard form. (x  0)2  (y  0)2 ¬402
(x  3)2  (y  0)2  42 x2  y2 ¬1600

373 Chapter 10
Pages 578–580 Practice and Apply 22. If d  12, r  6.
10. (x  h)2  (y k)2 ¬r2 The center is at (0  18, 0  7) or (18, 7).
(x  0)2  (y  0)2 ¬32 (x  h)2  (y  k)2 ¬r2
x2  y2 ¬9 [x  (18)]2  [y  (7)]2 ¬62
11. (x  h)2  (y  k)2 ¬r2 (x  18)2  (y  7)2 ¬36
[x  (2)]2  [y  (8)]2 ¬52 23. Sketch a drawing of the two tangent lines.
(x  2)2  (y  8)2 ¬25 y
12. (x  h)2  (y  k)2 ¬r2 (7, 8)
(x  1)2  [y  (4)]2 ¬( 17)2 5 units
(x  1)  (y  4) ¬17
2 2
5 units
13. If d  12, r  6.
(x  h)2  (y  k)2 ¬r2
(x  0)2  (y  0)2 ¬62 y3
x2  y2 ¬36
14. (x  h)2  (y  k)2 ¬r2 x
(x  5)2  (y  10)2 ¬72 O
(x  5)2  (y  10)2 ¬49 x2
15. If d  20, r  10. The line x  2 is perpendicular to a radius. Since
(x  h)2  (y  k)2 ¬r2 x  2 is a vertical line, the radius lies on a
(x  0)2  (y  5)2 ¬102 horizontal line. Count 5 units to the right from
x2  (y  5)2 ¬100 x  2. Find the value of h.
16. If d  16, r  8. h  2  5 or 7
(x  h)2  (y  k)2 ¬r2 Likewise, the radius perpendicular to the line
[x  (8)]2  (y  8)2 ¬82 y  3 lies on a vertical line. The value of k is 5
(x  8)2  (y  8)2 ¬64 units up from 3.
17. If d  24, r  12. k  3  5 or 8
(x  h)2  (y  k)2 ¬r2 The center is at (7, 8) and the radius is 5.
[x  (3)]2  [y  (10)]2  122 (x  h)2  (y  k)2  r2
(x  3)2  (y  10)2 ¬144 (x  7)2  (y  8)2  52
(x  7)2  (y  8)2  25
18. The distance between the center and the endpoint
24. x2  y2  25
of the radius is r   [0  (
3)]2 
 (6 6)2 or 3.
Write the equation in standard form.
(x  h)  (y  k) ¬r
2 2 2
(x  0)2  (y  0)2  52
[x  (3)]2  (y  6)2 ¬32 The center is at (0, 0), and the radius is 5.
(x  3)2  (y  6)2 ¬9 y
19. The midpoint of the diameter is   2  (2)
2
,  2 
2
 
2 8

or (0, 0). The distance from (0, 0) to either endpoint, 4


say (2, 2) is
r   (2  0)2  (2  0)2 or 
8. 8 4 O 4 8x
(x  h)  (y  k)  r2
2 2
4
(x  0)2  (y  0)2  ( 8)2
x2  y2  8 8
20. Find the center, which is the midpoint of the
diameter: 
7 

2 , 22
15   6

or (11, 2). Then find 25. x2  y2  36
r, the distance from (11, 2) to (7, 2). Write the equation in standard form.
(x  0)2  (y  0)2  62
r  [7  
(11)]2  (
2  2)2 or 32
. The center is at (0, 0), and the radius is 6.
(x  h)  (y  k) ¬r
2 2 2
y
[x  (11)]2  (y  2)2 ¬(32)2 8
(x  11)2  (y  2)2 ¬32
4
21. The distance between the center and the endpoint
of the radius is r   
[1  (2)] 2
 (0 
1)2 or 10

8 4 O 4 8x
(x  h)  (y  k) ¬r
2 2 2

[x  (2)]2  (y  1)2 ¬( 10)2 4


(x  2)2  (y  1)2 ¬10
8

Chapter 10 374
26. x2  y2  1  0 29. (x  1)2  (y  2)2  9
Write the equation in standard form. Compare each expression in the equation to the
x2  y2  1 standard form.
(x  0)2  (y  0)2  12 (x  h)2 ¬(x  1)2 (y  k)2 ¬(y  2)2
The center is at (0, 0), and the radius is 1. x  h ¬x  1 y  k ¬y  2
y h ¬1 k ¬2
h ¬1 k ¬2
r2  9, so r  3.
The center is at (1, 2), and the radius is 3.
y
O x

O x
( 1, 2)

27. x2  y2  49  0
Write the equation in standard form.
x2  y2  49
(x  0)2  (y  0)2  72
The center is at (0, 0), and the radius is 7. 30. Explore: You are given three points that lie on a
y circle.
8
Plan: Graph ABC. Construct the perpendicular
4 bisectors of two sides to locate the center. Find the
length of the radius. Use the center and radius to
write the equation.
8 4 O 4 8x
4 y

28. (x  2)2  (y  1)2  4 B(4, 2)


A(0, 2)
Compare each expression in the equation to the
x
standard form.
(x  h)2 ¬(x  2)2 (y  k)2 ¬(y  1)2 C(2, 0)
x  h ¬x  2 y  k ¬y  1
h ¬2 k ¬1
r2  4, so r  2. Solve: The center is at (2, 2). Find r by using the
The center is at (2, 1), and the radius is 2. center and a point on the circle, (2, 0).
y r  (2  2 )2  (0 2)2 or 2
Write an equation.
(x  2)2  (y  2)2  22
(x  2)2  (y  2)2  4
(2, 1) Examine: Verify the location of the center by
finding the equations of the two bisectors and
O x solving a system of equations.
The perpendicular bisector of A B
 is the vertical
line with equation x  2. Next find the equation of
0 2
the 1 bisector of A 
C. The slope of A 2  0  1.
C is 
The slope of the  bisector is 1. The midpoint of

A C  (1, 1). Use the point-slope form.
y  1  1(x  1)
yx
Substitute x  2 to find the intersection.
y2
The intersection is (2, 2) so the center is correct.

375 Chapter 10
31. Explore: You are given three points that lie on a so the slope of its bisector is 2

9 . The midpoint of
circle. C
B  is (2, 3). Use the slope and the midpoint to
Plan: Graph ABC. Construct the perpendicular write an equation for the bisector of B C:
bisectors of two sides to locate the center. Find the y  2 2
3
9 x  9 . Solving the system of equations,
 
length of the radius. Use the center and radius to
write the equation. y  4x  5 and y  2
 23
9 x  9 , yields (2, 3),

y which is the circumcenter. Let (2, 3) be D, then
DA  DB  DC  85 .
C(3, 3)
35. The center is at (0, 0) and the radius is 7.
(x  h)2  (y  k)2  r2
B(0, 0) (x  0)2  (y  0)2  72
x2  y2  49
A(6, 0) x
36. concentric circles
26
37. The radius is  2 or 13.
38. (x  6)2  (y  2)2  36
Solve: The center is at (3, 0). Find r by using
the center and a point on the circle, (0, 0). y
8
r  (0  0)2  [0  (3)]2 or 3 y = 2x  2
write an equation. 4
[x  (3)]2  (y  0)2 ¬32 x
(x  3)2  y2 ¬9 12
4 4 8
Examine: Verify the location of the center by
finding the equations of the two bisectors and 4
solving a system of equations.
The  bisector of AB
 is the vertical line with 8
equation x  3. Solve algebraically for the intersection of the two
Next find the equation of the  bisector of A C
. graphs.
3 0
C
The slope of A  3  (6)  1. So the slope of Substitute 2x  2 for y in the equation of the circle.
the  bisector is 1. (x  6)2  (2x  2  2)2 ¬36
Find the midpoint of A C
: 3 
2  , 3
(6) 
2
0
 x2  12x  36  (2x)2 ¬36

 9
 3
2, 2  5x2  12x ¬0
x(5x  12) ¬0
Use the point-slope form. x  0 or 5x  12  0
y  3
 
9
2 ¬1 x  2  y  2x  2
x  2.4

y  3 9
2 ¬x  2
 y  2(0)  2 or y  2(2.4)  2
2y  3 ¬2x  9  2  2.8
2x  2y ¬6 The line is a secant because it intersects the circle
x  y ¬3 at (0, 2) and (2.4, 2.8).
Substitute x  3 to find the intersection. 39. x2  4x  y2  8y ¬16
3  y  3 (x2  4x  4)  (y2  8y  16) ¬16  4  16
y0 (x  2)2  (y  4)2 ¬36
The intersection is (3, 0) so the center is correct. (x  2)2  [y  (4)]2 ¬62
The center is at (2, 4), and the radius is 6.
32. (x  2)2  (y  2)2  r2
The center is at (2, 2). 40. The center is at (58, 55), and the radius is 80.
Find r by using the Distance Formula with the (x  h)2  (y  k)2 ¬r2
center and the point (2, 5). [x  (58)]2  (y  55)2 ¬802
(x  58)2  (y  55)2 ¬6400
r  
(2  2
)2  (5
 2)2
41. See students’ work.
 9 or 3 42. The center is at (0, 0), and the radius is
33. (x  5)2  (y  3)2  r2 185  1740 or 1925.
The center is at (5, 3). (x h)2  (y  k)2 ¬r2
Find r by using the Distance Formula with the (x  0)2  (y  0)2 ¬(1925)2
center and the point (5, 1). x2  y2 ¬3,705,625
r  
(5  5
)2  (1
 3)2
 4  or 2
34. The slope of A  is 1
C 
4 , so the slope of its bisector
C
is 4. The midpoint of A  is (0, 5). Use the slope
and the midpoint to write an equation for the
C
bisector of A: y  4x  5. The slope of B is 9
C 
2,

Chapter 10 376
43a. y 44. Sample answer: Equations of concentric circles;
(0, 3) 2 2 answers should include the following.
x y  9
• (x  h)2  (y  k)2  r2
• x2  y2  9, x2  y2  36, x2  y2  81,
x2  y2  144, x2  y2  225
(3, 0) x 45. B; x2  y2  4x  14y  53 ¬81
(x2  4x)  (y2  14y) ¬28
y x  3 (x  4x  4)  (y2  14y  49) ¬28  4  49
2

(x  2)2  (y  7)2 ¬81


[x  (2)]2  (y  7)2 ¬92
Substitute x  3 for y in the equation of the circle. The center is at (2, 7) and the diameter is 2(9)
x2  (x  3)2 ¬9 or 18.
x  x2  6x  9 ¬9
2
46. D; there are 2 pints in one quart and 4 quarts in one
2x2  6x ¬0
gallon, so there are 8 pints in one gallon. One of the
2x(x  3) ¬0
8 pints is gone, so 7 of the 8 pints are left, or 7

8.
2x  0 or x  3 ¬0
x0 x ¬3
yx3
y  0  3 or y  3  3 Page 580 Maintain Your Skills
3 0 47. AX  EX or 24
The intersection points are (0, 3) and (3, 0). 48. (DX)2  (DE)2  (EX)2
43b. (DX)2  72  242
x 2y 2 = 25
y (DX)2  625
DX  25
49. DX ¬QX  QD
x 2y 2 = 9 25 ¬QX  7
x
18 ¬QX
50. TX  DX  DT
TX  25  7 or 32
51. 360  (117  125)  118
x  1

2 (118) or 59
Since x2  y2  25 and x2  y2  9, 25  9. This is 52. 360  (100  120  90)  50
never true, so there are no intersection points.
43c. x  1
2 (120  50)

y
 1

2 (70) or 35
(x3)2y 2 = 9
53. 180  (45  130)  5
x  1
2 (45  5)


x  1

2 (40) or 20
(0, 0)
54. A(3, 2) → A(3  3, 2  4) or A(6, 6)
B(4, 1) → B(4  3, 1  4) or B(1, 3)
(x3)2y 2 = 9 C(0, 4) → C(0  3, 4  4) or C(3, 0)
55. A(3, 2) → A(3, 2)
B(4, 1) → B(4, 1)
Since (x  3)2  y2  9 and (x  3)2  y2  9, C(0, 4) → C(0, 4)
(x  3)2  y2 ¬(x  3)2  y2
56. Each child needs 2(12)  2(10)  44 inches plus
(x  3)2 ¬(x  3)2
1 inch overlap, or 45 inches total. The teacher
x  6x  9 ¬x2  6x  9
2
1125
6x ¬ 6x needs 25 45  1125 inches, or 36  31.25 yards.
12x ¬0
x ¬0
Substitute x ¬0 into the equation of either circle. Chapter 10 Study Guide and Review
(0  3)2  y2 ¬9
32  y2 ¬9
9  y2 ¬9 Page 581 Vocabulary and Concept Check
y2 ¬0 1. a 2. j
y ¬0
3. h 4. i
The intersection point is (0, 0).
5. b 6. f
7. d 8. g
9. c 10. e

377 Chapter 10
Pages 581–586 Lesson-by-Lesson Review 20. 
BCA is a semicircle.
  180
11. r  1

2d
mBCA
   mAGB.
21. AB is a minor arc, so mAB
r  1

2 (15) or 7.5 in.

mAB  mAGB
  30
C  d mAB
C  (15) 22. mAGC  mCGD ¬90
C  47.12 in. mAGB  mBGC ¬mAGC
12. d  2r 30  mBGC ¬90
d  2(6.4) or 12.8 m mBGC ¬60
 ¬60
C  d mBC
C  (12.8)    mFGD.
23. FD is a minor arc, so mFD
C  40.21 m Vertical angles are congruent.
13. C ¬2r FGD ¬AGB
68 ¬2r mFGD  mAGB
68 mFD ¬mAGB
2 ¬r
 
mFD ¬30
10.82 yd ¬r
24. CDF is composed of adjacent arcs, 
 
d  2r CD and DF.
  mCD   mDF 
mCDF
68
d2 
2    90  30 or 120
mCDF
25. BCD is composed of adjacent arcs, 
 
d  21.65 yd BC and CD.
  mBC
mBCD   mCD 
14. r  1

2d
  60  90 or 150
mBCD
r  1

2 (52) or 26 cm 26. 
FAB is a semicircle.
  180
C  2r mFAB
C  2(26) 27. C  2r
C  163.36 cm C  2(6) or 12
15. C ¬2r mDIG  180  2(mDGI)
138 ¬2r mDIG  180  2(24) or 132
13
8
Let   arc length.
2 ¬r
 13
2 
360 ¬ 12

21.96 ft ¬r 13
2
360 (12) ¬

d  2r
 
22
5  ¬
13
d2  8 
2
d  43.93 ft The length of  2
DG is  2
5  units.
16. d  2r 28. WN  IW  5 and IW is a radius.
d  2(11) or 22 mm C  2r
C  d C  2(5) or 10
C  (22) Since IWN is equilateral, mWIN  60.
C  69.12 mm Let   arc length.
mBPY  mYPC  mCPA ¬180 60 
0 ¬
17. 
36 10

3x  (3x  3)  (2x  15) ¬180 60
8x  12 ¬180 
360 (10) ¬
8x ¬168 5 ¬
3
x ¬21
   mYPC.
YC is a minor arc, so mYC The length of 
WN is 5
3  units.

  mYPC 29. SV  1

mYC 2 SU
  3x  3
mYC
  3(21)  3 or 60 SV  1

2 (20) or 10
mYC
18.    mBPC.
BC is a minor arc, so mBC 30. WZ  1

2 YW

BC is composed of adjacent arcs, 
BY and 
YC. WZ  1

2 (20) or 10
  
mBC  mBY  mYC
  mBPY  mYPC 31.  bisects S
RT U
, so UV  SV.
mBC
  3x  (3x  3) UV  SV
mBC
  6x  3 UV  10
mBC
 bisects 
  6(21)  3 or 123   2mYX .
mBC X
32. R YW, so mYW
 
  360  mBCX mYW  2mYX
19. mBX   2(45) or 90
 mYW
mBX  360  (3x  3x  3  2x  15  3x)
  360  (11x  12)
mBX 33. SU  YW and R X bisects   bisects 
T
YW, R SU.
  360  [11(21)  12]  
mBX mST  mYX
  360  243 or 117   45
mBX mST

Chapter 10 378
  mYW. 45. 26 ¬12 (89  x)
34. Since SU  YW, mSU 
 
mSU  mYW 52 ¬89  x
  90
mSU x ¬37
35. m1  1  46. 33 ¬1 2 (x  51)
2 (96) or 48 
36. 2 is a right angle because it intercepts a 66 ¬x  51
semicircle. 117 ¬x
m2  90 47. 7(7  x) ¬132
37. Inscribed angles of the same arc are congruent. 49  7x ¬169
m3  32 7x ¬120
 x ¬17.1
38. m3  1 mGH
2
48. 8.1x ¬10.3(17)
 1

2 (78) or 39 8.1x ¬175.1
m2  m3 ¬90 x ¬21.6
m2  39 ¬90 49. x(15  x) ¬8(8  12)
m2 ¬51 15x  x2 ¬160
m1  m2 ¬90 x2  15x  160 ¬0
m1  51 ¬90 b  b  
2  4ac
x  
2a
m1 ¬39
15  (15)2  
4(1)(160)
39. m2  m3 ¬90 x  
2(1)
2x  x ¬90 15 29.4
x
3x ¬90 2
x ¬30 x  7.2 or x  44.4
m2  2x Disregard the negative value. So x  7.2.
 2(30) or 60 50. (x  h)2  (y  k)2 ¬r2
m3  x or 30 (x  0)2  (y  0)2 ¬(5 )2
x2  y2 ¬5
m1  m2 ¬90
m1  60 ¬90 51. If d  6, r  3.
m1 ¬30 (x  h)2  (y  k)2 ¬r2
[x  (4)]2  (y  8)2 ¬32
 
40. m2  1
2 mJH (x  4)2  (y  8)2 ¬9
 1

2 (114) or 57 52. The center is at the midpoint of the diameter.
m2  m3 ¬90 
center   0 

8 4  4
2 , 2 or (4, 4)
57  m3 ¬90 The radius is the distance from the center
m3 ¬33 to (0, 4).
m1  m2 ¬90 r   (0  4)2  [
4  (4)]2
m1  57 ¬90   16 or 4
m1 ¬33 Write the equation.
41. x2  122 ¬152 (x  h)2  (y  k)2 ¬r2
x2  144 ¬225 (x  4)2  [y  (4)]2 ¬42
x2 ¬81 (x  4)2  (y  4)2 ¬16
x ¬9 53. Since x  1 is a vertical line, the radius lies on a
42. x2  92 ¬(6  9)2 horizontal line. Count horizontally from the point
x2  92 ¬152 (1, 4) to the line x  1 to find the radius. The
x2  81 ¬225 radius is 2.
x2 ¬144 y
x ¬12
43. 72  242 ¬(x  7)2
49  576 ¬x2  14x  49
0 ¬x2  14x  576
(x  18)(x  32) ¬0
x ¬18 ¬0 or x ¬32 ¬0 x
x ¬18 x ¬32
Disregard the negative value.
So x ¬18. (x  h)2  (y  k)2 ¬r2
44. x  1
2 (68  24)
 [x  (1)]2  (y  4)2 ¬22
1 (x  1)2  (y  4)2 ¬4
x  2(44) or 22

379 Chapter 10
54. x2  y2  2.25 midpoint of B C
. The equation is y  3.
Write the equation in standard form. The intersection of x  3 and y  3 is the
(x  0)2  (y  0)2  (1.5)2 point (3, 3). This is the center we found above so
The center is at (0, 0), and the radius is 1.5. the answer checks.
y 57. y

A(0, 6) C(6, 6)

O x

B(6, 0)
O x

55. (x  4)2  (y  1)2  9


Compare each expression in the equation to the
standard form.
(x  h)2 ¬(x  4)2 (y  k)2 ¬(y  1)2 Chapter 10 Practice Test
x  h ¬x  4 y  k ¬y  1
h ¬4 k ¬1
h ¬4 k ¬1 Page 587
r2  9, so r  3. 1. Sample answer: A chord is a segment that has its
The center is at (4, 1), and the radius is 3. endpoints on a circle. A secant contains a chord
and is a segment that intersects a circle in two
y
points. A tangent intersects a circle in exactly one
point and no point of the tangent lies in the
interior of the circle.
2. Find the midpoint of the diameter using the
O x Midpoint Formula with the coordinates of the
diameter’s endpoints.
3. C ¬2r
25 ¬2r
25 ¬2r
56. Explore: You are given three points that lie on a 12.5 ¬r
circle. The radius is 12.5 units.
Plan: Graph ABC. Construct the perpendicular A
4. N , N
B, N
C, and ND are radii.
bisectors of two sides to locate the center. Find the 2
4
5. The diameter is A D, so the radius is 2 or 12.
length of the radius. Use the center and radius to CN  12
write the equation.
6. No; the diameter is the longest chord of a circle.
y 7. r  AN or 5
A (0,6) C(6,6)
C  2r
C  2(5) or 10
The circumference is 10 meters.
8.    mBNC, mBC   20
BC is a minor arc so mBC
9. 
AD is composed of adjacent arcs,  AB,  BC,
and CD.
B(6,0) x 
AD is a semicircle.
 ¬mAB   mBC   mCD 
mAD
Solve: The center is at (3, 3). Find r by using the  
180 ¬mAB  30  mAB
center and a point on the circle, (0, 6). 
150 ¬2mAB
r  (0  3)2  (6 3)2 or 18
 
75 ¬mAB
Write an equation. 2 10. B EE D
(x  3)2  (y  3)2   18   mED
mBE 
(x  3)2  (y  3)2  18 
mBE  120
Examine: Verify the location of the center by .
finding the equations of the two bisectors and 11. ADE is an inscribed angle intercepting AE
1 
solving a system of equations. mADE  mAE 
2
 bisector of  A
C: Since AC is horizontal, its  1
 2(75) or 37.5
bisector is vertical. It goes through (3, 6), the
midpoint of AC. The equation is x  3. 12. If two segments from the same exterior point are
 bisector of  B
C: Since BC is vertical, its  tangent to a circle, then they are congruent.
bisector is horizontal. It goes through (6, 3), the x  15

Chapter 10 380
13. 62 ¬x(x  5) 23. Sample answer:
36 ¬x2  5x Given: X with diameters R
0 ¬x2  5x  36 S
R and TV
 T
0 ¬(x  4)(x  9) Prove: 
RT  
VS X
x  4 ¬0 or x  9 ¬0 V
x ¬4 x ¬9 S
Discard x  9. So x ¬4. Proof:
14. 5x  6(8)
Statements Reasons
5x  48
48 1. X with diameters 1. Given
x 
5 or 9.6 S
R and T
V

15. Draw the radius to the point of tangency. Use the
2. RXT  VXS 2. Vertical  are .
Pythagorean Theorem.
3. mRXT  mVXS 3. Def. of  
62  82 ¬(x  6)2   mVS
36  64 ¬x2  12x  36 4. mRT 4. Measure of arc
0 ¬x2  12x  64 equals measure of its
0 ¬(x  4)(x  16) central angle.
x  4 ¬0 or x  16 ¬0 5. 
RT  
VS 5. Def. of  arcs
x ¬4 x ¬16
Discard x  16. So x ¬4. 24. Let x  the distance from the center to the 5-inch
16. 5(5  x) ¬4(4  7) side.
25  5x ¬16  28 x2  (2.5)2 ¬42
5x ¬19 x2  6.25 ¬16
19 x2 ¬9.75
x ¬
5 or 3.8 x ¬3.1
17. 35  1
2 [(360  x)  x]
 The height is about 4  3.1 or 7.1 in.
70  360  2x 25. A; draw the segment from C to A.
2x  290 DB  CA. Since diagonals of a rectangle are
x  145 congruent.
18. 180  (45  110)  25 DB  r
x  1
2 (45  25)

 1

2 (20) or 10 Chapter 10 Standardized Test Practice
19. 80  12 (110  x)

160  110  x
50  x Pages 588–589
20. C  d 1. A; 3y  6x  9 is the same as y  2x  3. So the
C  (50) line has slope m ¬2 and y-intercept (0, 3). The
C  157 graph that satisfies these conditions is A.
The circumference is about 157 ft. 2. C; m1 ¬m2
21. If d  50, r  25. 6x  5 ¬3x  13
(x  h)2  (y  k)2 ¬r2 6x  3x ¬13  5
[x  (2)]2  (y  5)2 ¬252 3x ¬18
(x  2)2  (y  5)2 ¬625 x ¬6
22. (x  1)2  (y  2)2  4 Use x to find the measure of 1.
Compare each expression in the equation to the m1  6x  5
standard form.  6(6)  5
(x  h)2 ¬(x  1)2 (y  k)2 ¬(y  2)2  36  5 or 31
x  h ¬x  1 y  k ¬y  2 3. A
h ¬1 k ¬2 4. B; 3(x  2) ¬2x  9
r2  4, so r  2. 3x  6 ¬2x  9
The center is at (1, 2), and the radius is 2. x ¬3
y Use x to find the measure of one side.
x  2  3  2 or 5
O Each side is 5 miles long.
x

x2 x2

x2

381 Chapter 10
5. A 13. 
ABC is a semicircle.
18
0
6. D mABC   2 or 90
 is a semicircle. 
7. C; DEA DEA is composed of 14. DF  AD or 12
adjacent arcs, 
DE and EA. DE  DF  FE
 ¬mDE   mEA 
mDEA DE  12  18 or 30
 
mDEA ¬mDFE  mEA Since AE is a tangent, CAE is a right angle and

180 ¬36  mEA AED is a right triangle. Use the Pythagorean

144 ¬mEA Theorem.
8. C; the measure of a minor arc is the measure of (AE)2  (AD)2 ¬(DE)2
its central angle. (AE)2  (12)2 ¬302
9. D (AE)2  144 ¬900
(AE)2 ¬756
10. mGDE  mGED  mDGE ¬180
AE ¬27.5
35  85  mDGE ¬180
120  mDGE ¬180 15. (BK)(KC) ¬(KF)(JK)
mDGE ¬60 8(12) ¬16(JK)
In DGE, G E
 is the shortest side because it is 96 ¬16(JK)
opposite the smallest angle. 6 ¬JK
mFGE  mGEF  mF ¬180 X
16a. V  and WY
65  55  mF ¬180 16b. Let w  width, then
120  mF ¬180 3w  2  length
mF ¬60 2(3w  2)  2w ¬164
In GFE, FG GE because mGEF mF. 6w  4  2w ¬164
So FG is the shortest side of quadrilateral DEFG. 8w ¬160
ST 30 w ¬20
8 ¬
11.  
12 The width is 20 in. and the length is 3(20)  2 or
12(ST) ¬8(30) 62 in.
12(ST) ¬240 17a. y B(1, 6)
ST ¬20
ST is 20 ft long.
12. Find a correspondence between ABC and DEF
so that their sides are proportional. (1, 2)
y
12
F O x
8 C
D E A(1, 2)
4
B 17b. The center is the midpoint of the diameter:

4
A
4 8 12
x
 2 , 2 or (1, 2).
1 
1 2  6

17c. Find r by using the Distance Formula.


4
Find the distance between the center and a point
on the circle, (1, 2).
AC  7  1  6
r  (1  1)2  (
2  2)2
DE  10  7  3
 16 or 4
AB  
(6  0)
2  (4
 1)2 17d. C  2r
 
36  9   45 C  2(4) or 8 units
EF  
(10 
8.5)2 
 (7 
10)2 17e. The center is at (1, 2), and the radius is 4.
(x  h)2  (y  k)2  r2
 (1.5)
 2
 9  11.25
 11
14  245 (x  1)2  (y  2)2  42
(x  1)2  (y  2)2  16
BC  
(6  0)
2  (4 7)2
 
45
DF  
(8.5 7)2 (10  7)2
 
(1.5) 
2 
 9  
45
2
AC AB BC

DE 
EF  DF  2.
  
So the scale factor is 2.

Chapter 10 382
Chapter 11 Areas of Polygons and Circles
Page 593 Getting Started 4. co- together, centr- center; circles with a common
1. A ¬  w 2. A ¬  w center
150 ¬  15 38 ¬  19 5. circum- around, scribe- write; to write around (a
10 ¬ 2 ¬ geometrical figure)
3. A ¬  w 4. A ¬  w 6. co- together, linear- line; together on the same line
21.16 ¬  4.6 2000 ¬  32 7. circum- around, about; ferre- to carry
4.6 ¬ 62.5 ¬ 8. Sample answers: polychromatic – multicolored,
5. A ¬  w 6. A ¬  w polymer – a chemical compound composed of a
450 ¬  25 256 ¬  20 repeating structural unit, polysyllabic – a word
18 ¬ 12.8 ¬ with more than three syllables
7. 1 1
2 a(b  c) ¬ 2  6(8  10)

¬1
2  6(18)  54


8. 1 1
11-1 Areas of Parallelograms
2 ab  2  6  8  24


9. 1 1
2 (2b  c) ¬ 2 (2  8  10)

¬1 Page 595 Geometry Activity
2 (16  10)

1. When the parallelogram is folded, the base of the
¬1
2  26  13

rectangle is 4 and the height is 5. So the area is
10. 1 1
2 d(a  c) ¬ 2  11(6  10)
 20 units2.
¬1 2. There are 2 rectangles, one on the bottom and one
2  11  16

formed by the folded triangles on top.
¬8  11  88
3. 40 units2
11. 1
(b  c) ¬1(8  10)
2 2 4. The base of the parallelogram is half of the length
¬1
2 (18) ¬9
 of the rectangle. The altitude of the parallelogram
12. 1 1
2 cd ¬ 2  10  11
 is the same width as the width of the rectangle.
¬55 5. A  w or A  bh
13. ABD is a 30°-60°-90° B
triangle. B D
, or h, is
the longer leg, A D is Page 598 Check for Understanding
the shorter leg, and 1. The area of a rectangle is the product of the
B
A  is the hypotenuse. h length and the width. The area of a parallelogram
AD ¬1 
2 AC or 6
is the product of the base and the height. For both
60˚ quadrilaterals, the measure of the length of one
h ¬6 3
A D C side is multiplied by the length of the altitude.
12
2. See students’ work.
14. In the 30°-60°-90° triangle, the hypotenuse has a
3. P  2(5)  2(9)  28 ft
length of 22, so the shorter leg has a length of 11.
Use a 30°-60°-90° triangle to find the height, with
15. In the 45°-45°-90° C x as the length of the shorter leg.
triangle, the 9 ¬2x
length of the h
9 ¬x
hypotenuse is 15. 2
AB ¬BC 2 45˚   923
height  x3  ft
A B
15 ¬h 2 15 A ¬bh
15


 ¬h
2 ¬59
 
2 3
  45
2 3
 or about 39.0 ft2
15
  2 ¬
¬h

2  2
15 
2 The perimeter of the parallelogram is 28 ft, and
  ¬h
2 the area is about 39.0 ft2.
4. P  2(13)  2(10)  46 yd
Use a 45°-45°-90° triangle to find the height, x.
Page 594 Reading Mathematics 10 ¬x 2
1. bi- 2, sector- a subdivision or region; divide into 2 10
  x
regions 2
2. poly- many, gon- closed figure; closed figure with 10 
  
2
¬x
many sides  
2 2
52 ¬x
3. equi- equal, lateral- sides; having sides of equal
length height  x  52
 yd

383 Chapter 11
A ¬bh A ¬bh
¬1352
 ¬217517  10  17
¬170 units2
¬652
 or about 91.9 yd2 8. Left Parallelogram Rectangle
The perimeter of the parallelogram is 46 yd, and A ¬bh A ¬w
the area is about 91.9 yd2.  18  15  35  18
¬270 ft2 ¬630 ft2
5. P  4(3.2)  12.8 m
Right Parallelogram
A ¬s2
A ¬bh
¬(3.2)2  18  15
¬270 ft2
¬10.24 or about 10.2 m2
The total area is 270  630  270 or 1170 ft2.
The perimeter of the square is 12.8 m, and the
area is about 10.2 m2.
Pages 598–600 Practice and Apply
6. y 9. P  2(30)  2(10)  80 in.
V ( 2, 6) X ( 6, 6) Use a 30°-60°-90° triangle to find the height, with
x as the length of the shorter leg.
10 ¬2x
5 ¬x
height  x 3  53 in.
A ¬bh
x ¬305 3
T ( 0, 0) Y ( 4, 0)
¬150 3 or about 259.8 in2.
The perimeter of the parallelogram is 80 in., and
6 0 6 the area is about 259.8 in2.
V
slope of T
2  0  2 or 3
0 6 6 10. Use the 45°-45°-90° triangle with hypotenuse 4 to
Y
slope of X
4  6  2 or 3

find the base and height of the parallelogram,
6 6 0 both equal to x.
X
slope of V
6  2  4 or 0
0 0 4  x 2
Y
 0
slope of T 4  0  4 or 0 4
  x
TVXY is a parallelogram, since opposite sides 2
have the same slope. Slopes of consecutive sides 4 2
    x
are not negative reciprocals of each other, so the 2   2
parallelogram is neither a square nor a rectangle. 2 2 ¬x
A ¬bh base  height  x  2 2m
¬4  6
P  22 2  2(4)  4 2  8 or about 13.7 m
¬24 units2
A ¬bh
7. y V ( 2, 18) ¬22 22 
T ( 10, 16) ¬8 m2
The perimeter of the parallelgram is about 13.7 m,
and the area is 8 m2.
11. Since all 4 sides have the same measure and the
angle is a right angle, the parallelogram is a square.
x P  4(5.4)  21.6 cm
X ( 3, 2)
Y ( 5, 4) A ¬s2
¬(5.4)2
¬29.16 or about 29.2 cm2
18  16 2 1 The perimeter is 21.6 cm, and the area is about
V
slope of T2  10  8 or  4
29.2 cm2.
4 (2) 2 1
Y
slope of X
5  (3)  8 or 4 12. P  2(15)  2(10)  50 in.
2  18 20
X
slope of V
3  2  5 or 4
  Use a 45°-45°-90° triangle to find the height, x.
4  16 20 10 ¬x 2
Y
slope of T 5 10  5 or 4 10  
2
    x
TVXY is a rectangle, since opposite sides have the 2 2 
same slope and the slopes of consecutive sides are 5 2 ¬x
negative reciprocals of each other. height  x  5 2 in.
Length of X  is 
Y 
[5  (3)] 2
 [4
 (2 )]2 A ¬bh
 2 17. ¬155 2
Length of T  is 
Y [10 5]2 [16 
(4)]2 ¬75 2 or about 106.1 in2
 5 17. The perimeter is 50 in., and the area is about
106.1 in2.

Chapter 11 384
13. P  2(12)  2(10)  44 m 18. A ¬bh
Use a 30°-60°-90° triangle to find the height, with 100 ¬(x  15)(x)
x as the length of the shorter leg. 100 ¬x2  15x
10 ¬2x x2  15x  100 ¬0
5 ¬x (x  20)(x  5) ¬0
height  x 3  5
3m x  20 ¬0 or x  5 ¬0
A ¬bh x ¬20 or x ¬5
Reject the negative solution.
¬125 3
h  5 units, b  20 units
¬60 3 or about 103.9 m2
19. A ¬bh
The perimeter is 44 m, and the area is about
2000 ¬(x  10)(x)
103.9 m2.
2000 ¬x2  10x
14. P  2(5.4)  2(4.2)  19.2 ft x  10x  2000 ¬0
2

A ¬bh (x  40)(x  50) ¬0


¬(5.4)(4.2) x  40 ¬0 or x  50 ¬0
¬22.68 or about 22.7 ft2 x ¬40 or x ¬50
The perimeter is 19.2 ft, and the area is about Reject the negative solution.
22.7 ft2. h  40 units, b  50 units
15. Rectangle 20. y
A ¬w
D ( 1, 5) C ( 5, 5)
 1052 
¬502  mm2
Each triangle
A ¬1

2 bh
x
 1

2 (5)(5) A ( 0, 0) B ( 4, 0)
25
¬ 2 mm
2
0 0 0
The shaded area is 502  222
5
 or about B
slope of A
4  0  4 or 0
45.7 mm2. 5 5 0
C
slope of D
5  1  4 or 0

16. Rectangular outline
5 0 5
A ¬w slope of A
D

1  0  1 or 5
 (7  4  7)(15) C
 5 0 5
¬270 cm2
slope of B 5  4  1 or 5
ABCD is a parallelogram, since opposite sides
Upper cutout
have the same slope. Slopes of consecutive sides
A ¬w
are not negative reciprocals of each other, so the
 12  3
parallelogram is neither a square nor a rectangle.
¬36 cm2
Lower cutout Base: AB lies along the x-axis and is horizontal so
A ¬w AB  4  0  4.
48 Height: Since C D and A B are horizontal
¬32 cm2 segments, the distance between them, or height,
The shaded area is 270  36  32 or 202 cm2. can be measured on any vertical segment.
Reading from the graph, the height is 5.
17. Big square
A ¬bh
A ¬s2 ¬4  5
 (9.2)2 ¬20 units2
¬84.64 ft2
Small square 21. y
A ¬s2 H ( 3, 4) G ( 5, 4)
 (3.1)2
¬9.61 ft2
Rectangle
A ¬w
 (10.8)(3.1) x
¬33.48 ft2
The shaded area is 84.64  33.48  9.61 or about
108.5 ft2. E ( 5, 3) F ( 3, 3)

3 (3) 0
F
slope of E3  (5)  8 or 0
4 
4 0
G
slope of H 
5  (3)  8 or 0

385 Chapter 11
4  (3) 7 6 2 8
G
slope of F  53  2
 P
slope of O 4  2  2 or 4
4  (3) 7 NOPQ is a parallelogram, since opposite sides
slope of EH
 3  (5)  2 have the same slope. Slopes of consecutive sides
EFGH is a parallelogram, since opposite sides
are not negative reciprocals of each other, so the
have the same slope. Slopes of consecutive sides
parallelogram is neither a square nor a rectangle.
are not negative reciprocals of each other, so the
Base: PQ is horizontal with length
parallelogram is neither a square nor a rectangle.
4  4  8.
Base: E F is horizontal with length
Height: Since P Q
 and ON are horizontal
3  (5)  8.
segments, the distance between them, or height,
Height: Since G H and E F
 are horizontal
can be measured on any vertical segment.
segments, the distance between them, or height,
Reading from the graph, the height is 8.
can be measured on any vertical segment.
A ¬bh
Reading from the graph, the height is 7.
¬8  8
A ¬bh
¬64 units2
¬8  7
¬56 units2 24. y
R ( 2, 4) S ( 8, 4)
22. y M ( 1, 6) L ( 6, 6)

x
U ( 2, 3) T ( 8, 3)

J ( 1, 4) K ( 4, 4) T 3 


 (3) 0
slope of U 8  (2)  
10 or 0
4 (4) 0 4 
4 0
K
slope of J
4  (1)  5 or 0 S
slope of R
8  (2)  10 or 0
6 6 0 3  4 7
L
slope of M
6  1  5 or 0 U
slope of R (2)  0 is undefined
2 
6  (
4) 1
0 3 4 7
M
slope of J
1  (1)  2 or 5
 T
slope of S  8  8  0 is undefined
6  ( 4) 10 RSTU is a rectangle, since the sides are not all
L
slope of K  6  4  2 or 5
  
equal but are all horizontal or vertical.
JKLM is a parallelogram, since opposite sides
Base: UT is horizontal with length
have the same slope. Slopes of consecutive sides
2  8  10.
are not negative reciprocals of each other, so the
Height: Since R S and U
T are horizontal
parallelogram is neither a square nor a rectangle.
segments, the distance between them, or height,
Base: JK is horizontal with length
can be measured on any vertical segment.
1  4  5.
Reading from the graph, the height is 7.
Height: Since J K
 and L M
 are horizontal
A ¬bh
segments, the distance between them, or height,
¬10  7
can be measured on any vertical segment.
¬70 units2
Reading from the graph, the height is 10.
A ¬bh 25. y V ( 1, 10)
¬5  10
¬50 units2 W ( 4, 8)
23. y Y ( 1, 7)

N ( 6, 2) O ( 2, 2)
X ( 2, 5)

x
5 7  2 2
slope of X
Y2  ( 1)  3 or  3


P ( 4, 6) 8  10  2 2
Q ( 4, 6) slope of W
V 4  1  3 or  3

8  5 3
6 (6) 0 slope of W
X 
42  2
P
slope of Q
4  (4)  8 or 0
10  7 3
2 
2 0
slope of V
Y 
1  ( 1)  2
O
slope of N
2  (6)  8 or 0
6  2 8
Q
slope of N (6)  2 or 4
4 

Chapter 11 386
VWXY is a rectangle, since opposite sides have 31. On the crosswalk, draw a 30°-60°-90° triangle
the same slope and the slopes of consecutive sides whose hypotenuse is 16 ft long and whose short
are negative reciprocals of each other. side lies along the left edge of the crosswalk. That
Length of Y   
X 
[2  (1)] 2 [5 7]2  13
 short side measures 8 ft, and the remaining side,
Length of X   
W [4  2]
2  [8 5]  
2 13 which represents the perpendicular distance
between the stripes, measures 8 3 or about
Thus VWXY is in fact a square.
13.9 ft.
A ¬s2 2 32. P  2(8)  2(11)  38 m
¬ 13
A  bh
¬13 units2
 8  10
26. Guest bedroom Family room  80 m2
A ¬w A ¬w The perimeter is 38 m, and the area is 80 m2.
¬20  22 ¬25  22
33. P  2(4)  2(5.5)  19 m
¬440 ft2 ¬550 ft2
A  bh
Hallway 45
¬A ¬w  20 m2
¬25  3 The perimeter is 19 m and the area is 80 m2.
¬75 ft2
34. The new perimeter is half of the original
The Bessos need 440  550  75 or 1065 ft2 of
perimeter. The new area is one half squared, or
carpet. Since there are 9 square feet per square
one fourth, the area of the original parallelogram.
yard, the family should order 1065  9 or 119 yd2
(rounded up to the nearest yd2). 35. Let the side length of one square be x. Then the
48  4x
27. The figure is composed of three 5 by 10 side length of the other square is  4 or
rectangles. 12  x.
x2  (12  x)2 ¬74
10
x  144  24x  x2 ¬74
2

2x2  24x  70 ¬0


3
5 5 x2  12x  35 ¬0
7 (x  5)(x  7) ¬0
7 x  5 or x ¬7
The two side lengths are 5 in. and 7 in.
5 5
36. Sample answer: Area is used when designing a
garden to find the total amount of materials
7 7 needed. Answers should include the following.
5 5 • Find the area of one square and multiply by the
3 number of squares in the garden.
• Knowing the area is useful when planning a
10
stone walkway or fencing in flowers or
For each rectangle, A  w  5  10 or 50 units2.
vegetables.
The total area is 3  50 or 150 units2.
37. C; the length of base A  is 
B 102 62  8, so the
28. The figure can be viewed as a large 8 by 11
area is 8  6  48 m . 2
rectangle with two rectangular cutouts measuring
1 by 2 and 3 by 3. 38. D; either A, B, or C could be true, depending on
Large rectangle Cutout 1 whether x  1 1 1
2 , x  2 , or x  2 .

A ¬w A ¬w
¬8  11 ¬1  2
¬88 units2 ¬2 units2 Page 600 Maintain Your Skills
Cutout 2 39. For the equation (x  h)2  (y  k)2  r2, the
¬A ¬w center of the circle is (h, k) and the radius is r. For
¬3  3 the equation (x  5)2  (y  2)2  49  72, the
¬9 units2 center is (5, 2) and r  7.
The shaded area is 88  2  9 or 77 units2. 40. For the equation (x  h)2  (y  k)2  r2, the
29. The triptych measures 3  5  2  12  2  5  3 center of the circle is (h, k) and the radius is r. For
or 32 inches wide by 3  12  3 or 18 inches tall. the equation (x  3)2  (y  9)2  81  0, which is
Yes, it will fit in a 45-inch by 20-inch frame. equivalent to [x  (3)]2  [y  (9)]2  92, the
30. By Exercise 29, the artwork measures 32 inches center is (3, 9) and r  9.
by 18 inches. 41. For the equation (x  h)2  (y  k)2  r2, the
¬A ¬w center of the circle is (h, k) and the radius is r. For
¬32  18 2 2
the equation x  2
3   y  9   9  0, which
 1 4
¬576 in2 2 2 2
is equivalent to x  2
3   y  9    3  ,
 1 2

the center is 2


3 , 9  and r  3 .
 1 2

387 Chapter 11
42. For the equation (x  h)2  (y  k)2  r2, the 49. Use the Pythagorean Theorem.
center of the circle is (h, k) and the radius is r. For a2  b2 ¬c2
the equation (x  2.8)2  (y  7.6)2  34.81, which 52  122 ¬2
is equivalent to [x  2.8]2  [y  (7.6)]2  5.92, 25  144 ¬2
the center is (2.8, 7.6) and r  5.9. 169 ¬2
43. Use Theorem 10.16. 13 ¬
10(10  22) ¬8(8  x) The length of plywood needed is 13 ft.
320 ¬64  8x 50. 1 1
2 (7y) ¬ 2 (7  2)

256 ¬8x ¬7
32 ¬x
51. 1 1
2 wx ¬ 2 (8)(4)

44. Use Theorem 10.15.
4  9 ¬x  x ¬16
36 ¬x2 52. 1 1
2 z(x  y) ¬ 2 (5)(4  2)

6 ¬x  15
45. Use Theorem 10.17. 53. 1 1
2 x(y  w) ¬ 2 (4)(2  8)

14  14 ¬7(7  x)
196 ¬49  7x  20
147 ¬7x
21 ¬x
46. B y B
11-2 Areas of Triangles, Trapezoids,
and Rhombi
A A

C
C
Page 601 Geometry Activity
x 1. The two smaller triangles, combined, are the
C same size as ABC.
A 2. ABC is 1 
2 the area of rectangle ACDE.
3. Since the area of rectangle ACDE is bh, for ABC
A  1

2 bh.
B
Image coordinates: A (1, 3), B (4, 6), C (5, 1)
The rotation angle is 180°. Page 605 Check for Understanding
47. 1. Sample answer:
y
F
G, H 2. Kiku is correct; she simplified the formula by
H, H

adding the terms in the parentheses before


x multiplying.
F F

3. Sometimes; two rhombi can have different


G G
corresponding diagonal lengths and have the
same area.
4. A ¬1

2 d1d2

Image coordinates: F (4, 0), G (2, 2), ¬1



2 (20)(24)
H (2, 2) ¬240 m2
The rotation angle is 90° counterclockwise. 5. area of FGHI ¬area of FGH  area of FHI
48. ¬1 1
2 bh1  2 bh2
y 

¬1 1
2 (37)(9)  2 (37)(18)

M
L 33
¬ 3
2  333
L
¬499.5 in2
L N x
6. A ¬1
2 h(b1  b2)

1
¬2(12)(24  16)
M
M
¬240 yd2
N
N

Image coordinates: L (0, 2), M (3, 3), N (4, 1)


The rotation angle is 90° counterclockwise.

Chapter 11 388
7. 
AB is horizontal with y Pages 606–608 Practice and Apply
length 5  2  7 13. A ¬1

2 bh
or 7. Point C lies above C ( 1, 3)
¬1

2 (7.3)(3.4)
B
A  a distance of
3  (3)  6 or ¬12.41 or about 12.4 cm2
6 units. x 14. A ¬1

2 bh
A ¬1
2 bh ¬1

2 (10.2)(7)
¬1

2 (7)(6) ¬35.7 ft2
B(5, 3) A(2, 3)
¬21 units2 15. A ¬1
2 h(b1  b2)

G
8. F and H J are horizontal.
¬1
2 (10)(8  11)

FG ¬5  (1) y
¬6 or 6 F ( 1, 8) G(5, 8) ¬95 km2
HJ ¬1  3 16. A ¬1
2 h(b1  b2)

¬2 or 2
¬1
2 (8.5)(8.5  14.2)

h ¬8  4 H ( 3, 4) ¬96.475 or about 96.5 yd2
¬4 or 4 J ( 1, 4)
17. A ¬1

2 d1d2
A ¬1
h(b1  b2)
2
¬1
2 (20  20)(30  30)

¬1
2 (4)(6  2)

x ¬1200 ft2
¬16 units2
P
9. L is horizontal, M
Q
 is vertical. 18. A ¬1

2 d1d2
LP ¬0  (4)
M ( 2, 4) y ¬1
2 (17  17)(12  12)

¬4 or 4 ¬408 cm2
MQ ¬2  4 L ( 4, 3) 19. area of quadrilateral ABCD
¬2 or 2  area of ADC  area of ABC
A ¬1
 P ( 0, 3)
2 d1d2 Q ( 2, 2) D C
¬1
2 (4)(2) x
¬4 units2 5m
5m
8m
B
12 m
A
10. A ¬1
2 h(b1  b2)

¬1 1
2 b1h1  2 b2h2

Substitute the known values into the formula.
¬1 1
2 (8)(5)  2 (12)(5)

250 ¬12 h(20  30)

¬50 m 2
250 ¬1

2 (50)h 20. area of quadrilateral WXYZ
250 ¬25h  area of WXY  area of WZY
10 ¬h
W
The height is 10 in. Z
11. A ¬1

2 d1d2 4 in.
6 in.
Substitute the known values into the formula. 18 in.
675 ¬1
2 (15  15)d2

X 21 in. Y
675 ¬15d2
45 ¬d2
¬1 1
2 b1h1  2 b2h2

SU ¬45 m
¬1 1
2 (21)(6)  2 (18)(4)

12. From Postulate 11.1, the area of each congruent
¬99 in2
rhombus is the same, namely 827 3 2
8  13  6 8 in .

21. In a 30°-60°-90° right triangle, the longer leg is
The width of one rhombus is 15  5  3 in. To
3 times as long as the shorter leg. Here, then,
find the other diagonal (the height), use the area
formula. b  h3, so h  b.
3
A ¬1

2 d1d2
A  bh

63
 1
8 ¬ 2 (3)d2
 
 15 15


3
51
 3  75 3 or about 129.9 mm2
8 ¬ 2 d2

17
4 ¬d2

17 1
The vertical diagonal measures  
4 or 4 4 in.

389 Chapter 11
22. 
PT and QR are horizontal. 27.  is horizontal, 
JL KM is vertical.
QR ¬5  3 y JL ¬3  (1) y
¬2 or 2 ¬4 or 4 8
PT ¬6  0 K
Q R KM ¬3  7
¬6 or 6 ¬10 or 10 4 L
h ¬7  3 J
A ¬1
2 d1d2
x
¬4 or 4 1 8 4 4 8
P T ¬2 (4)(10)
A ¬1
2 h(b1  b2)
 4 M
x ¬20 units2
¬1
2 (4)(2  6)
 8
¬16 units2
T
23. R and P Q
 are horizontal. 28.  is horizontal, 
JL KM is vertical.
RT ¬4  4 y JL ¬5  (1) y
¬8 or 8 8 ¬6 or 6
PQ ¬2  (4) T R KM ¬10  2 4
K
¬2 or 2 4 ¬12 or 12 x
h ¬6  (5) A ¬1 8 4 4 8
2 d1d2
¬11 or 11 8 4 4 8x
4 ¬1

J 4
A ¬1 2 (6)(12) L
2 h(b1  b2)

P Q ¬36 units2 8
¬1
2 (11)(8  2)
 8
M
¬55 units2
T
24. R  and PQ are horizontal. L
29. J is horizontal, K
M
 is vertical.
RT  1  6 y JL ¬10  2 y
P Q
 5 or 5 8 ¬8 or 8
PQ  6  (3) KM ¬2  6 12
 9 or 9 4 ¬4 or 4
R
h  8  2 8
T x A ¬1
2 d1d2
K
 6 or 6 8 4 4 8
¬1
 4 J
A ¬1 4 2 (8)(4) L
2 h(b1  b2)
 x
¬16 units2 M
¬1
2 (6)(5  9)
 8 4 8 12
¬42 units2
T
25. R  and PQ are horizontal. 30. A  1
2 h(b1  b2)

RT  4  (2) y Solve for h.
 2 or 2 P
PQ  1  (6)
Q 750  1
2 h(35  25)

 7  7 750 ¬1

2 (60)h
h  3  (2) x
750 ¬30h
 5 or 5
25 ¬h
A ¬1
2 h(b1  b2)
 T R The height is 25 m.
¬1 31. A  1
2 h(b1  b2)

2 (5)(2  7)

¬22.5 units2 Solve for b2.
L
26. J is horizontal, K
M
 is vertical. 188.35 ¬1
2 (8.7)(16.5  b2)

JL  12  2 y 376.7 ¬8.7(16.5  b2)
K
 10 or 10 4 376.7 ¬143.55  8.7b2
KM  2  4 J L 233.15 ¬8.7b2
 6 or 6 4 4 8 12 x 26.8 ¬b2
A ¬1

2 d1d2 4 M GK is about 26.8 ft.
32. A ¬1

2 d1d2
¬1

2 (10)(6) 8 Solve for d2.
¬30 units2 375 ¬1
12 2 (25)d2
375 ¬12.5d2
30 ¬d2
NQ  30 in.

Chapter 11 390
33. A ¬1

2 d1d2 41. A side length of 52  4 or 13 in. and a half-
Solve for d2. diagonal of 24  2 or 12 in. implies that the other
137.9 ¬1 half-diagonal measures 13  2 122 or 5 in.
2 (12.2)d2
137.9 ¬6.1d2 So d2  2  5 or 10 in. 13 13
22.6 ¬d2 A ¬1

2 d1d2 12
QS is about 22.6 m. ¬1

2 (24)(10)
24
13 13
34. A ¬1

2 bh ¬120 in2
Solve for b. 42. B
x
248 ¬1

2 b(16)
2x  3
248 ¬8b h
31 ¬b
The base measures 31 in. A
5 x C5
35. A ¬1

2 bh
Solve for h. x  10
300 ¬1
2 (30)h Let b1  x, b2  x  10, s  2x  3.
300 ¬15h P ¬b1  b2  2s
20 ¬h 52 ¬x  (x  10)  2(2x  3)
The height is 20 cm. 52 ¬6x  4
36. Each rhombus has an area of 150  2  75 ft2. 48 ¬6x
8 ¬x
A ¬1 
2 d1d2 So b1  8, b2  18, s  13.
75 ¬1
2 (12)d2 In order to find the area, we need to find h. Using
75 ¬6d2 right triangle ABC, AB  s  13, and AC  5.
12.5 ¬d2 h2  52 ¬132
Each stone walkway is 12.5 ft long. h2 ¬ 144
37. From Exercise 36, d2  12.5 ft. h ¬12
A ¬1
 d   d2 2 h(b1  b2)

2 2
s2 ¬ 21 2

¬1
2 (12)(8  18)

s ¬ 

2  2
d 2 1 d 2 2
¬156 yd2


2  
2 

2 2
¬ 12
  
12
.5 43. b ¬1

3P
¬8.7 ¬1
3 (15)
Each side measures about 8.7 ft. ¬5 in.
Drawing the height divides the triangle into two
38. A ¬1
2 h(b1  b2)

30°-60°-90° triangles each with base 1

2 b.
¬1
2 (122.81)(56  69.7)

  12b
h ¬3
¬7718.6 ft2
  12(5)
¬3
39. A ¬1
2 h(b1  b2)
 53
¬
2in.
1
¬2(199.8)(57.8  75.6) 1
A ¬ 2 bh

¬13,326.7 ft2
40. A side length of 20  4 or 5 m and a half-diagonal
¬1
2 (5) 2
5
 
3

of 8  2 or 4 m implies that the other half- 25  or about 10.8 in2
3
¬ 4

diagonal measures  52  42 or 3 m. So d2  2  3
or 6 m. 44. Because (34.0)2  (81.6)2  (88.4)2, the triangle is
5 5 a right triangle.
A ¬1

2 d1d2 4 A ¬1

2 bh
¬1

2 (8)(6) 8
5 5 ¬1

2 (34.0)(81.6)
¬24 m2
¬1387.2 m2

391 Chapter 11
45. LM ¬5   or 3 ft
2  42 50. The kite consists of two triangles, each with
JL ¬(8.5)
 
2 (4)2 or 7.5 ft b  25 in. and h  20  2 or 10 in.
JM ¬JL  LM A ¬A1  A2
¬3  7.5 ¬2A1
¬10.5 ft ¬2(1

2 bh)
A ¬1

2 bh ¬bh
¬1

2 (10.5)(4) ¬25  10
¬21 ft2 ¬250 in2
46. A rhombus is made up of two congruent triangles. 51. Comparing heights, 6  2
3  1.
Using d1 and d2 instead of b and h, its area in 52. Left triangle Right triangle
2 bh is 2 2 d1 2 d2 or
reference to A  1 1 1 P  8  8.3  6.5 P  4  4.15  3.25
1
d1d2.  22.8  11.4
2
22.8 2
11.4  1
53. 
47. False; Sample answer:  
5 40
The area for each of 3
2
The ratio is the same.
these right triangles is 54. Left triangle Right triangle
4 6
6 square units. The A ¬1
bh A ¬1

perimeter of one triangle is 12 and the perimeter 2 2 bh
of the other is 8  40 or about 14.3. ¬1

2 (8)(6) ¬1

2 (4)(3)
48. Drawing the height of an equilateral triangle ¬24 ¬6
2
6  1  1
24 4 2
divides it into two 30°-60°-90° triangles each with 55. 
base 1 
2 b.
The ratio of the areas is the square of the scale
Left triangle factor.
2
6  1  1
24 4 2
  12b
h ¬3 56. 
22.8 2
11.4  1

  12(4)
¬3
The ratio of the areas is the square of the ratio of
¬23
 the perimeters.
A ¬1

2 bh 57. BH ¬6 and HA  3
BA ¬6  2  32 
2 (4)23
¬1 

¬ 45
¬43 or about 6.9 2
area of ABCD   45 or 45 ft2
P ¬3(4)  12
area of EFGH  92  81 ft2
Right triangle 45
ratio of areas    5
  12b
h ¬3 81  9 or 5:9
58. Sample answer: Umbrellas have triangular panels
  12(5)
¬3 of fabric or nylon. In order to make the panels to
fit the umbrella frame, the area of the triangles is
¬5
2 3
 
needed. Answers should include the following.
A ¬1

2 bh • Find the area of a triangle by multiplying the
base and the height and dividing by two.
2 (5) 2 3
¬1
 5 
• Rhombi are composed of two congruent
25
4 3
¬  or about 10.8 isosceles triangles, and trapezoids are composed
P  3(5)  15 of two triangles and a rectangle.
The scale factor and ratio of perimeters is  15
 59. B; C is (0, 10), and A  1  1
2 bh  2 (10)(6) 
12 2
5
or 4. 30 units .
25
 or 2156, which 60. D; either 2x  7  0 or x  10  0, so either
4 3
The ratio of areas is    43
2
equals 4 .
5 x  72 or x  10.


49. Left rhombus Right rhombus 61. Let side b be the base and a be the other given
side. Then h  asin C.
A ¬1

2 d1d2 A ¬1

2 d1d2 area  1 
2 bh
¬1

2 (4)(6) ¬1

2 (2)(3)  1

2 absin C
¬12 ¬3
62. A  1

2 absin C
P ¬4213 P ¬413
¬1

2 (4)(7)sin 29°
¬813
¬6.79 in2
The scale factor and ratio of perimeters is
4
 
13 1
 2. 63. A ¬1

2 absin C
813 2 ¬1

2 (4)(5)sin 37°
The ratio of areas is 
12  4 , which equals 1
3 1
2 .

¬6.02 cm2

Chapter 11 392
64. A ¬1

2 absin C Page 609 Practice Quiz 1
¬1

2 (1.9)(2.3)sin 25° 1.
M y
¬0.92 ft2 8
J 4
L
x
Page 609 Maintain Your Skills
65. A ¬w 8 4 K 4 8
¬(22)(17) 4
¬374 cm2
8
66. Use a 30°-60°-90° triangle. The height of the
parallelogram  x3 , where x  12(10) or 5. K
 ¬ 0 4 4
4  (8)  4 or 1
slope of J 
A ¬bh 8 4
M
 ¬ 4
¬(15)(5 3)
slope of L 4  0  4 or 1
¬75 3 or about 129.9 in2 4 
0 4
L
slope of K ¬
0  (4)  4 or 1
67. area ¬area of large rectangle 8 4
M
 ¬ 4
¬ area of “hanging” rectangle
slope of J 4  (8)  4 or 1
¬b1h1  b2h2 Opposite sides have the same slope and slopes of
¬(21)(9)  (6)(7) consecutive sides are negative reciprocals of each
other.
¬231 ft2
JK   [4  (8)]2
 [0 4]2  42 
68. For the circle with center (h, k) and radius r, the
equation is (x  h)2  (y  k)2  r2. KL   [0  (
4)] 
2  [4 0]2  4 2
Here, (x  1)2  (y  2)2  72, or Adjacent sides have the same length. JKLM is a
(x  1)2  (y  2)2  49. square.
69. For the circle with center (h, k) and radius r, the 2. Since JKLM is a square with side length 4 2,
equation is (x  h)2  (y  k)2  r2. A  s2
2 2  (4 2)2
Here, [x  (4)]2  y  12   2  ,
 11

2
 32 units2
or (x  4)2  y  2  4.
1 12 1
P
3. N  is a horizontal segment with length
70. For the circle with center (h, k) and radius r, the 4  1  5 so b1  5. M
Q is a horizontal
equation is (x  h)2  (y  k)2  r2. segment with length 6  7  13 so b2  13.
Since N P
 and M
Q  are horizontal, the distance
Here, [x  (1.3)]2  [y  5.6]2  3.52,
between them, the height, can be measured on
or (x  1.3)2  (y  5.6)2  12.25.
any vertical segment. Reading from the graph,
71. Each semicircle has a radius of 12 (3.5)  1.75 in.

h ¬6.
total inches of trim for one flower  5( r)
A ¬1
2 h(b1  b2)

¬5(1.75 )
¬27.5 in. ¬1
2 (6)(5  13)

So she needs 10(27.5) or 275 in. to edge
¬54 units2
10 flowers.
72. 136 cos 25°, 136 sin 25°  123.3, 57.5 4. The bases of the trapezoid are vertical segments.
b1  WZ  3  (1)  4 and
73. 280 cos 52°, 280 sin 52°  172.4, 220.6
x b2  XY  7  1  6
46 ¬sin 73°
74.  Since WZ and XY are vertical segments, the
x ¬46 sin 73° distance between them, or the height of the
¬44.0 trapezoid, can be measured along any horizontal
x segment. Reading from the graph, h  5.
30 ¬sin 42°
75. 
x ¬30 sin 42° A ¬1
2 h(b1  b2)

¬20.1 ¬1
2 (5)(4  6)

76. 1 ¬25 units2
2 (6) ¬3

x
3 ¬tan 58° 5. A ¬1

2 d1d2
x ¬3 tan 58° 546 ¬1

2 (26)d2
¬4.8
546 ¬13d2
42 ¬d2
d2 is 42 yd long.

393 Chapter 11
4. Side length:
11-3 Areas of Regular Polygons and Since the perimeter
Circles is 108 meters, the 140
108
side length is 9
or 12 m. C
Page 611 Geometry Activity Apothem: C D is an 40
1. A  1
2 Pa. Since P  (number of sides)(measure of
 apothem of the
a side), each entry in the last row of the table is 1

2 regular nonagon. 20
The central angles B
times the product of the three entries above it.
6 D6
are all congruent. A
360°
The measure of each angle is  9 or 40°. C D

Number bisects ACB so mACD is 20°. AD  6. Write a
3 5 8 10 20 50 trigonometric ratio to find the length of C D.
of Sides
6 6
Measure tan 20°  
CD or CD  tan 20°  16.485 m
1.73r 1.18r 0.77r 0.62r 0.31r 0.126r
of a Side Area: A ¬1

2 Pa
Measure
of 0.5r 0.81r 0.92r 0.95r 0.99r 0.998r ¬1
2 (108)(16.485)
Apothem ¬890.2 m2
5. A 30°-60°-90°
Area 1.30r2 2.39r2 2.83r2 2.95r2 3.07r2 3.14r2 A
triangle is
2. The polygon appears to be a circle. formed by the 60
A 2.4 cm
radius of the cm
3. The areas of the polygons approach the area of 2. 4
the circle. circle, the
C 30 D
apothem, and B D B
4. The formula for the area of a circle is 3.14r2 or
half the base
r2.
of the equilateral E
triangle. AD is the
shorter leg, B D
 is the
Page 613 Check for Understanding longer leg, and A B is the
1. Sample answer: Separate a hexagon inscribed in hypotenuse.
a circle into six congruent nonoverlapping
So, AD ¬1 2  2.4  1.2

C
isosceles triangles. The area of one triangle is one- B D
half the product of one side of the hexagon and BD ¬1.2 3 and
the apothem of the hexagon. The area of the BC  2.4 3
Next, find the height of the triangle DE. Since
hexagon is 61 2 sa. The perimeter of the

mEBD  60,
hexagon is 6s, so the formula is 1 
2 Pa. DE   3 BD   3(1.2)( 3)
2. Sample answer: Another method besides  3.6
30°-60°-90° triangles is to use trigonometric shaded area ¬area of circle  area of triangle
ratios.
¬ r2  1 
2 bh
3. Side length:
¬ (2.4)2  1 2 (2.4
 3)(3.6)
Since the perimeter is 42
42 ¬10.6 cm2
yards, the side length is  
6 or
7 yd. 30˚
6. ABC is a 30°-60°-90° C
triangle. AB is the
Apothem: A 30°-60°-90°
shorter leg and A C  is 30
triangle is formed by the
7 the longer leg.
apothem and one-half of a 3.5 AC   3 AB A 60 D
side of the hexagon. The B
  3  3  3 3
shorter leg of the triangle is 1
2 (7) or 3.5. ADC is also a
The apothem is the longer leg of the triangle or 30°-60°-90° triangle in
3.5 3. which A C is the E
Area: A ¬1 
2 Pa shorter leg and A D  is the longer leg.
So AD   33 3  3  3  9.
2 (42) 3.53
¬1 

CE  2 AC because AD is an altitude of CDE
¬127.3 yd2
 6 3
shaded area ¬area of triangle  area of circle
¬1
2 CE  AD  r
 2

¬1
2 (63
 )(9)  (3)2
¬18.5 in2

Chapter 11 394
7. Small cushions Large cushion 12. Since we know that ABC C
radius  6 in. radius  10 in. is a regular triangle, it is
For cloth cover: For cloth cover: equilateral and ADC is a
r  6  3  9 in. r ¬10  3  13 in. 30°-60°-90° triangle. The 15.5 in. 30
A  r2 A ¬ r2 height, CD
, of the triangle
 (9) 2 ¬ (13)2 is the longer leg and AD is
 81 ¬169 the shorter leg.
Area of cloth to cover both sides of all cushions 15
.5
AD  2  7.75 A D B
 2(169 )  14(81 ) CD  AD 3   7.753  15.5 in.
 1472 in2 The area of triangle ABC is
To convert to square yards, divide by 1296.
2
147 1 1
2AB  CD  2(15.5)(7.753
)
1296  3.6 yd
 2

 104.0 in2
13. Apothem: The central
Pages 613–616 Practice and Apply angles of the octagon are
8. Side length: 72  8 or 9 in., and 1
2 (9)  4.5
 all congruent so
36 0 C
Central angle: 360°  8 or 45°, mACB   8 or 45°.
10
45
and 1 D
C  is an apothem of the
2 (45°)  22.5°
 4.5 9
4.5 octagon. It bisects ACB
Apothem: tan 22.5°   a

4.5 a 45 and is a perpendicular A5D B
a tan 22.5°  10.864 in. B
bisector of A. So
A ¬1
 mACD  22.5. Since the side of the octagon has
2 Pa measure 10, AD  5.
¬1
2 (72)(10.864) 5
tan 22.5°  CD
¬391.1 in2 5
CD  
9. side length  842   4 or 212 m tan 22.5°
A ¬s2  12.07
2
¬21 2 perimeter  10  8  80
¬882 m2 Area: A ¬1

2 Pa
(Note: This is easier than finding the central
¬1

2 (80)(12.07)
angle, the apothem, and A  1 
2 Pa.)
¬482.8 km2
10. ADC is a 45°-45°-90°
14. square side length  102 
triangle so AD  12.
side length  2  12  24 cm shaded area  area of circle  area of square
C  r2  s2
A  s2
 (10)2  (10 2)2
 (24) 2
45°  114.2 units 2
 576 cm2 12
(Note: This is easier than 45° 15. Circle radius: 5  2  2.5
finding the central angle A D B shaded area  area of rectangle  area of circle
1
and using A  2Pa.)  w  r2
 (10)(5)  (2.5)2
11. Side length: mACB  60  30.4 units2
so mACD  30 and ACD
16. Use 30°-60°-90° triangles. Half the base of the
is a 30°-60°-90° triangle. C equilateral triangle is 0.75 3, so the base is

AD is the shorter leg, 1.53 and the height is  3(0.75 3) or 2.25.
so CD   3 AD or shaded area ¬area of circle  area of triangle
24
24   3 AD.
 24  A D B ¬ r2  1

2 ab
AD  24  3 3
3  83 
3 3 
¬ (1.5)2  1
2 (1.53
 )(2.25)
So, AB, the side length of the regular hexagon
is 2(83)  16 3 in. ¬4.1 units2
Perimeter: P  6(16 3)  96
3 in. 17. Use 30°-60°-90° triangles. Half the base of the
equilateral triangle is 3.63, so the base is 7.23

Area: A ¬1
2Pa and the height is  3(3.63) or 10.8.
¬1
2(963
)(24) shaded area ¬area of triangle  area of circle
¬1995.3 in2 ¬1
2 bh  r
 2

¬1
2 (7.23
 )(10.8)  (3.6)2
¬26.6 units2

395 Chapter 11
18. The triangle is a 30°-60°-90° right triangle whose 23. The radius of the larger circle is x, while the
sides measure 5, 53 , and 10. Since the radius of the smaller circle is 1x. The ratio of
2
hypotenuse of the triangle is a diameter of the areas is
circle, and the length of the hypotenuse is twice  x2 x2
2  
the length of the shorter leg, the length of the x
1 1
2 x
2
diameter is 10. So the radius of the circle is 5. 2
1
shaded area ¬area of circle  area of triangle  
1
2
¬ r2  1

2 bh
 2

1 or 2:1
¬ (5)2  1
2 (5)(53
 )
24. The scale factor is 9
 3
3  1 , so the ratio of areas
¬56.9 units2
between a large cake and a mini-cake is 3 1  1.
9
2

19. Use a 30°-60°-90° triangle. So when nine mini-cakes are compared to one
Apothem: The central angle of the regular large cake, the total area is equal. Nine mini-
360°
hexagon is  6  60°. The apothem bisects the cakes are the same size as one 9-inch cake, but
central angle so it forms a 30° angle with the nine mini-cakes cost 9  $4 or $36 while the 9-inch
hypotenuse of the triangle. So the apothem is the cake is only $15. The 9-inch cake gives more cake
3  21(4.1)  2.05
longer leg and its measure is  3. for the money.
Hexagon perimeter: 6(4.1)  24.6 25. 16-inch pizza 8-inch pizza
Radius: The radius of the inscribed circle is the r8 r4
apothem of the regular hexagon. A   82 A   42
shaded area ¬area of hexagon  area of circle A  64 in.2 A  16 in.2
For 2 pizzas,
¬1
2 Pa  r
 2
A  2(16 )
¬1
2 (24.6)(2.053
 )  (2.053
)2  32 in.2
¬4.1 units2 One 16-inch pizza; the area of the 16-inch pizza is
greater than the area of two 8-inch pizzas, so you
20. circle radius  1
2 (20)  10 in.

get more pizza for the same price.
Use a 30°-60° -90° triangle. 26.
Apothem: The central angle of the regular 4 y
360°
hexagon is  6  60°. The apothem bisects the T ( 0, 0) x
central angle so it forms a 30° angle with the 8 4 4 8
hypotenuse of the triangle, the radius of the 4
circumscribed circle. So the apothem is the longer
leg and has measure  3  1
2 (10)  5
 3 in. U ( 7, 7) 8 W ( 7, 7)
Hexagon perimeter: 6(10)  60 in.
12
shaded area ¬area of circle  area of hexagon
V ( 0, 14)
¬ r2  1 
2 Pa 16
¬ (10)2  1 2 (60)(5
 3) Explore: Looking at the graph, it appears that
¬54.4 in2 quadrilateral TUVW is a square.
21. From the solution to Exercise 14, the outer Plan: Show that TUVW is a parallelogram by
shaded area is 100  200. The inner circle’s showing that TU
 W V  and T U
 W
V. A regular
1
radius is (10)  5 2 and so its area is parallelogram is a square. Find the area by using
2 the formula A  s2.
(5 2)  50 .
2
7  0 7
shaded area  100  200  50 U
Solve: slope of T
7  0  7 or 1
 
 271.2 units2 
slope of V 7  
W (14)
 7

70 7 or 1
22. Use 30°-60°-90° triangles. Half the base of the TU   
[0  (7)]2
 [0 
(7)]2  
72  72 
equilateral triangle is 4 3, so the base is 8
3 7
2
and the height is  3(4 3) or 12. Also, the radius
of the inner circle is 4. VW  
(0  7
)2  [
14  (
7)]2  
72  72  7
2
shaded area ¬area of outer circle¬ area of triangle TU  VW
¬ area of inner circle TUVW is a parallelogram.
¬ r21  1
2 bh  r2
 2
UV   (7  0)2 
[7  (14)]2
  72  72
¬ (8)2  1
2 (83
 )(12)  (4)2
 7 2
¬168.2 units2 The area of the square is s2  (UV)2  7
2
2

 98 units .
2

Examine: Another way to find the area of a


rhombus is using the formula 1 
2 d1d2.
A  1
2 (14)(14)  98. The answer is the same.


Chapter 11 396
27. y The apothem of the regular octagon is the
8 segment connecting the center (0, 0) with the
H ( 0, 4

3 )
midpoint of a side.
4
Use the Midpoint Formula to find the midpoint of
G ( 12, 0) x C
B .
1612 8 4 x x y y 0  4 
M
1
 2
2 ,

1
2   M 2 , 2   M 2 , 2 
 2
 3  3 3 7
4

8
J ( 0, 4

3 ) apothem length  
2

2
258
3  0  7  0  
2 2

A ¬1

2 Pa
58
)
¬1
2 (810 2 
J
H is vertical.  

HJ  43
  43  ¬4145
 or about 48.2 units2
 8
3 or 83 30. C  2 r
h  0  (12) rC
2
 12 or 12
34
r2  17
A ¬1

2 bh
A ¬ r2
2 83
¬1 (12)

¬ (17)2
¬483
 or about 83.1 units2 ¬289 or about 907.9 units2
28. y 31. C ¬2 r
8 C
QX r ¬
2
P R
4
17
r ¬ 17
2  2
 
N x
A ¬ r2
8 4 4 J 8 2
¬ 2 
17
4
M K 28
9
8 L ¬4 or about 227.0 units
2

32. C ¬2 r
The figure is a regular octagon centered at the C
r ¬
2
origin.
54
.8 27
.4
side length  QR   (4  0)
2  (4 5)2 r ¬2 

  17, so perimeter  8 17 A ¬ r2
2
The apothem of the regular octagon is the ¬  
27
.4
segment connecting the center (0, 0) with the
750
.76
midpoint of a side. ¬ or about 239.0 units2
R
Use the Midpoint Formula to find the midpoint of Q. 33. C ¬2 r
x x y y 04 54
M
1
2 , 
1
2   M2, 2 r ¬C
2 2 2
 M2, 9
2
 91
r ¬ .4 45
2 
 .7

(2

 0)

 

2  0
 2
9 97
 2 A ¬ r2
apothem length   2  2
¬  
45
.7
A ¬1
2 Pa 2088.49
¬  or about 664.8 units
2
97

¬1
2 (817
 ) 2   34. A ¬ r2
¬21649
 or about 81.2 units2 7850 ¬ r2
29. y B
X
r ¬ 

7850
  
5

3


14

A C C ¬2 r
5314
¬2   

x
¬10 314 or about 314.1 ft
H D
At 2 tiles per foot, and rounding up, that makes
629 tiles.
G E 35. The square tiles will touch along the inner edge of
F the border, but there will be gaps along the outer
The figure is a regular octagon centered at the edge. The tiles used to fill the gaps should be
origin. triangles. There will be 629 gaps between the
side length  BC   (3  0
)2  (3
 4)2 
10, 629 square tiles, so 629 triangular tiles will be
so perimeter  8 10 needed.

397 Chapter 11
36. A ¬ r2 45. The area of the garden equals that of a square of
¬ (1.3)2 side length 175 ft combined with a circle of radius
¬1.69 or about 5.3 cm2 1(175) or 87.5 ft.
2
37. Use radius  11 in. area  s2  r2
A  r2  (175)2  (87.5)2
 (11)2  7656.25  30,625
 121 or about 380.1 in2  54,677.8 ft2
The perimeter is the circle’s circumference plus
38. Sample answer: Multiply the total area by 40%.
two side lengths from the square.
39. Use a 30°-60°-90° triangle. For the equilateral perimeter  2 r  2s
triangle, the height is opposite the 60° angle so it  2 (87.5)  2(175)
is the longer leg of the 30°-60°-90° triangle with  175  350
hypotenuse measure of 3. So, height   3  1

2 (3)  899.8 ft
3
 23. 46. d1  d2  d3  20  40  60
For the circle, radius  1
2 (7)  3.5.
  120 or about 377.0 ft
shaded area ¬area of circle  area of triangle 47. The path is defined by an outer circle of radius
¬ r2  1

2 bh 1(60)  5  25 and an inner circle of radius
2
1(40)  20.
2 (3) 2 3
¬ (3.5)2  1 3  2
path area  area of outer circle  area of inner circle
¬12.25  94 3
   r12 r22
¬34.6 units2  (25)2  (20)2
 225 or about 706.9 ft2
40. circle diameter   122  92  15, and radius 
1(15)  7.5.
48. No; the areas of the floors will increase by the
2 squares of 1, 3, 5, and 7, or 1, 9, 25, and 49. The
shaded area  area of circle  area of rectangle ratio of the areas is the square of the scale factor.
 r2  w 4.
2 2
 (7.5)2  (9)(12) 6.3  3 or 2 : 3
49. 
 56.25  108 50. Call the perimeter of the figure on the left P1 and
 68.7 units2 the figure on the right P2.
41. r1  large radius  10, r2  small radius  5 P1  5(4.2) or 21 cm
shaded area ¬area of large circle P2  5(6.3) or 31.5 cm
¬¬area of small circles
21 2
¬ r21  2 r22 51.  .5  3
31
¬ (10)2  2 (5)2 The ratio is the same.
¬50 or about 157.1 units2 52. Using the perimeter information from Exercise 50,
42. circle radii  1.5
A1 ¬1

2 Pa
shaded area  area of square  area of circles
 s2  4 r2 ¬1

2 (21)(2.88)
 (6)2  4 (1.5)2 ¬30.24 cm2
 36  9
A2 ¬1

2 Pa
 7.7 units2
43. Flip the right half top-for- ¬1

2 (31.5)(4.32)
bottom, to make it a mirror ¬68.04 cm2
2
68.04  9   3 
image of the left half. Then 30.
53.  24 4 2
r1
there are three circles, with
radii of r1  15, r2  10, and r3  5. r2 The ratio of the areas is the square of the scale
shaded area r3 factor.
2
¬area of large circle
68.04  9   3 
30.
24 4 2
54. 
 area of medium circle  area of small circle
21 2
¬ r21  r22  r23  .5  3
31
¬ (15)2  (10)2  (5)2 The ratio of the areas is the square of the ratio of
¬150 or about 471.2 units2 the perimeters.
44. The large semicircle has radius r1  7.5, the small 55. The apothem of the larger hexagon equals the
semicircles have radii r2  2.5. radius of the circle, r  10. The apothem of the
shaded area ¬area of large semicircle smaller hexagon is 3 
  1r    r.
3
2 2
¬ area of small semicircles

2  r2 
¬2 r12  3  1
1
 2

¬1
 2 1
2 [ (7.5) ]  3  2 [ (2.5) ]
2

¬18.75 or about 58.9 units2

Chapter 11 398
2r 3
 ABCD is a parallelogram, since opposite sides
scale factor   3
r  2 have the same slope. Slopes of consecutive sides
area ratio ¬(scale factor)2 are not negative reciprocals of each other, so the
10

3 2 parallelogram is neither a square nor a rectangle.
¬  
2  A ¬bh
3 ¬(7)(3)
¬4 or a ratio of 3 to 4
¬21 units2
(The value of r is irrelevant.)
63. y
56. Sample answer: You can find the areas of regular
polygons by finding the product of the perimeter
and the apothem and then multiplying by one
half. Answers should include the following. J ( 2, 1) F ( 4, 1)
• We need to know the length of each side and the x
length of the apothem.
• One method is to divide the area of the floor by
the area of each tile. Since the floor is
hexagonal and not rectangular, tiles of different
shapes will need to be ordered to cover the floor.
H ( 2, 5) G ( 4, 5)
57. B; for the circle, A  r2 so that r  
A   


18
F

11
 0
slope of J 4  (2)  6 or 0
 32
. Square side length s  2
r 5  (5)
G  0
slope of H 4  (2)  6 or 0
 2
 (32
)  6 units. 1  (5)
J  6
58. B; since average  sum  x,
slope of H 2  (2)  0 is undefined
1  ( 5) 6
x  sum  average  90  15  6. slope of F
G  4  4  0 is undefined

FGHJ is a square, since the sides are all equal
and are all horizontal or vertical.
Page 616 Maintain Your Skills A ¬s2
59. A ¬1
2 d1d2 ¬(6)2
1 ¬36 units2
¬2(20)(26)
64. L ( 2, 5)
¬260 cm2 y M ( 1, 5)

60. area ¬area of upper triangle


¬ area of lower triangle
¬1 1
2 bh1  2 bh2

x
¬1 1
2 (16)(7)  2 (16)(6)

¬104 m2
61. Use a 30°-60°-90° triangle. Base  1(70) yd.
3 K ( 1, 3) N ( 2, 3)
A ¬bh
¬170(70) 3  (3)
3 N
slope of K   0 or 0
2  (1) 3
¬2829.0 yd2 55
M  0
slope of L 1  (2)  3 or 0
62. y 3  5
8

slope of L K
1  (2)  1 or 8
 
3  5
8
N
slope of M
2  1  1 or 8
 
A ( 3, 2) B ( 4, 2)
KLMN is a parallelogram, since opposite sides
x have the same slope. Slopes of consecutive sides
are not negative reciprocals of each other, so the
D ( 5, 1) C ( 2, 1) parallelogram is neither a square nor a rectangle.
A  bh
 (3)(8)
 24 units2
22
B
  0
slope of A 4  (3)  7 or 0
1  (1)
C
  0
slope of D 2  (5)  7 or 0
2  (1)
A
  3
slope of D 3  (5)  2
2  (1)
B
  3
slope of C 42  2

399 Chapter 11
65. y ¬1
 1 2
2 b1h  b2h  2 r
¬1 1
2 (2)(4)  2  4  2 (2)
 2

¬4  8  2
x ¬18.3 units2
y
R ( 1, 2) S ( 5, 2)

P ( 5, 7) h r
Q ( 1, 7) x
O b1 b2
7  (7)
P
  0
slope of Q 5  (1)  6 or 0
2  (2)
S
  0
slope of R 5  (1)  6 or 0 2. An irregular polygon is a polygon in which all
2  (7) sides are not congruent. If a shape can be
R  5
slope of Q 1  (1)  0 is undefined separated into semicircles or smaller circular
2  (7)
S
slope of P    5 is undefined regions, it is an irregular figure.
55 0
3. area ¬area of rectangle  area of triangle
PQRS is a rectangle, since the sides are not all
equal but are all horizontal or vertical. ¬w  1 
2 bh
A  w ¬(9.2)(3.6)  1 2 (9.2)(8  3.6)

 (6)(5) ¬53.36 or about 53.4 units2
 30 units2 4. area ¬area of rectangle  area of semicircle
66. HE ¬1 ¬w  1 2 r
 2
2 (CD  GF)

¬(32)(16)  1 2 (8)
 2
38 ¬1
2 (46  GF)

¬512  32 or about 612.5 units2
76 ¬46  GF
5. area of MNPQ ¬area of MNQ  area of QNP
30 ¬GF
¬1 1
2 bh1  2 bh2

67. WX ¬1
2 (CD  HE)

¬1 1
2 (6)(5)  2 (6)(3)

¬1
2 (46  38)

¬24 units2
¬42
6. area ¬area of rectangle¬ combined area of two
68. Use GF  30 from Exercise 66. semicircles
YZ ¬1
2 (HE  GF)
 ¬w  r2
¬1 ¬(10)(4)  (2)2
2 (38  30)

¬34 ¬40  4 or about 52.6 units2
69. h is opposite the 30° angle of a 30°-60°-90° 7. The height of the triangular portion is given by
triangle, so h  1
2 (12)  6.

(24)


2 
 143.3
2

70. h is adjacent to the 30° angle of a 30°-60°-90°


h 
41
.6
2 6 .

triangle, so h   3  15  15
3. area ¬area of rectangle  area of triangle
71. h is one leg of a 45°-45°-90° triangle, so ¬w  1

2 bh
h  1  8  4 2.
2 ¬(41.6)(24)  1
 6 )
72. h is one leg of a 45°-45°-90° triangle, so 2 (41.6)(143.3
h  1  21  2 .
21 2
 ¬1247.4 in2
2

11-4 Areas of Irregular Figures Pages 619–621 Practice and Apply


8. Notice that the triangle that is cut out on the left
is congruent to the triangle added on the right.
Page 619 Check for Understanding area ¬area of rectangle  area of triangle
1. Sample answer:  area of triangle
Area of irregular figure ¬area of triangle ¬area of rectangle
 area of rectangle ¬w
 area of semi-circle ¬(10)(5)
¬50 units2

Chapter 11 400
9. area  area of rectangle  area of semicircle 19. y
¬w  1
2 r
 2
P ( 5, 3)
¬(8)(12)  1
2 (4)
 2 N ( 0, 3)
¬96  8 or about 70.9 units2
10. area  area of rectangle  area of triangle
x
¬w1  1

2 bh M ( 4, 0) Q ( 5, 0)
¬(12)(25)  1

2 (10)(8)
¬340 units2
11. area  area of rectangle  area of triangle
¬w1  1
 area of trapezoid ¬1
2 h(b1  b2)

2 bh
¬(62)(54)  1
 ¬1
2 (3)(5  9)

2 (62)(27)
¬4185 units2 ¬21 units2
12. area  2 area of one trapezoid 20. y
¬2  1
2 h(b1  b2)
 V ( 3, 4)
U ( 2, 2) h2
¬h(b1  b2)
( 3, 2)
¬10(8  23) b2 h1
¬310 units2 x
13. area ¬area of rectangle
 combined area of two semicircles
T ( 4, 2) b1 W ( 3, 2)
¬w  r2
¬(22)(14)  (7)2
¬308  49 or about 154.1 units2
14. The semicircle has radius 18 in. The perimeter is area ¬area of triangle
48  36  48   18  132  18 or  area of trapezoid
about 188.5 in. ¬1 1
2 b2h2  2 h1(b1  b2)

15. area ¬area of rectangle  area of semicircle
¬1 1
2 (5)(2)  2 (4)(7  5)

¬w  1
2 r
 2
¬29 units2
¬(36)(48)  1
2 (18)
 2
21. y
¬1728  162 or about 2236.9 in2 I ( 2, 4)
16. area ¬area of square
 combined area of two triangles h2
¬s2  2  1

2 bh H ( 3, 1) b4 b2 x
¬s2  bh
¬(4)2  (4)(2) b1
¬24 units2 G ( 3, 1) h1 h3 b3 J ( 5, 1)
17. area ¬area of triangle  area of semicircle K ( 1, 3)
¬1 1 2
2 bh  2 r


¬1
 1
2 (6)(3)  2 (3)
2
area ¬area of trapezoid  area of upper triangle
¬9  4.5 or about 23.1 units2  area of lower triangle
¬1 1 1
2 h1(b1  b2)  2 b3h2  2 b4h3

18. area ¬area of rectangle  area of semicircle
 area of semicircle ¬1 1 1
2 (5)(2  5)  2 (3)(5)  2 (8)(2)

¬area of rectangle
¬33 units2
¬w
¬(5)(4)
¬20 units2

401 Chapter 11
22. y 34. Hexagon side length  16, and perimeter  6  16
12  96. Use a 30°-60°-90° triangle to find the
P ( 8, 7) h1 apothem ¬x 3 where x  1
2 (16)  8.
h2 Q ( 3, 7) 
8
area ¬area of circle  area of hexagon
b2 h3 b1
S ( 1, 3) 4 b3 ¬ r2  1

2 Pa
T ( 11, 1) x
¬ (16)2  1
 )
2 (96)(83
12 8 4 4 ¬256  3843
 or about 139.1 units2
4 R ( 3, 2)
35. Use a 30°-60°-90° triangle. Base  1
3 (57)  19 ft.

 where x  12(19)  9.5 ft.
Height  x3
area ¬area of left trapezoid  area of right A ¬1

2 bh
trapezoid  area of triangle
¬1
 )
2 (19)(9.53
¬1 1 1
2 h1(b1  b2)  2 h2(b1  b3)  2 b2h3

¬90.253  or about 156.3 ft2
¬1 1 1
2 (10)(4  6)  2 (4)(4  9)  2 (6)(3)
 36. The area of the rhombus is divided into four
¬67 units2 congruent right triangles with hypotenuse
23. Sample answer: (23 squares)(2500 mi2 per square)
 57,500 mi2 10
24. See students’ work.
25. Add the areas of the rectangles. 6
6  22  6  20  6  16  6  11  6  8  462
26. The actual area of the irregular region should be
smaller than the estimate. The rectangles drawn
are larger than the region. length 10 yd and one side length 6 yd. The other
27. Sample answer: Reduce the width of each side length is 10
 2 62  8. So the two
rectangle. diagonals measure 12 yd and 2(8)  16 yd.
3 x.
28. Let BC  x. Then the altitude of the triangle is  2 A ¬1

2 d1d2

ABC ¬1

2 bh   3 3
1(x) 
2
3
2 x ¬1

2 (12)(16)
area of 
area of BCDE     ¬4 or 4 : 1

s2 x2
¬96 yd2
29. Sample answer: Windsurfers use the area of the
sail to catch the wind and stay afloat on the 37. Let the shorter base  b1  x. Then b2  2x  5
water. Answers should include the following. and P  90  x  (2x  5)  2(x  3)  5x  11
5x  11 ¬90
• To find the area of the sail, separate it into
5x ¬101
shapes. Then find the area of each shape. The
sum of areas is the area of the sail. x ¬20.2
• Sample answer: Surfboards and sailboards are A ¬1
2 h (b1  b2)

also irregular figures. ¬1
2  15 (x  2x  5)

30. B; let LM  x. Then, starting at the bottom and
moving clockwise, ¬1
2 15 (20.2  40.4  5)  7.5(55.6)

P  7x  4x  4x  2x  2x  x  x  x  22x. ¬417 m2
So 22x  66 and x  LM  3. 38. The image of the point lies in quadrant IV, 6 units
area ¬area of A  area of B  area of C
away from the origin, at (x, y)  1  6,   6
1
¬s12  s22  s32 2 2
¬(12)2  (6)2  (3)2  (32, 32).
¬189 units2 39. 0.625
31. C;  16  92  4  81  85 .0
85 00

4.8

Page 621 Maintain Your Skills 20
32. area ¬area of square  area of circle 16

¬s2  r2 40
¬(14)2  (7)2 40

¬196  49 or about 42.1 units2 0
33. area ¬area of circle  area of triangle So 5
  0.63.
8
¬ r2  1

2 bh
2
¬   12  1
1

2 2 (12)(12)
¬72  72 or about 154.2 units2

Chapter 11 402
40. 0.8125 3. Use a 30°-60°-90° triangle. Half the base of the
3
161.0
00
0
 triangle is 42, so the apothem, which is also the
12.8
 radius of the circle is 42
  143
.
20 3
Triangle perimeter  3(84)  252
16
 shaded area ¬area of triangle  area of circle
40
32 ¬1
2 Pa  r
 2
 2
2 (252)143
 1   143

80 
80
 ¬17643   588
0 or about 1208.1 units2
13
  0.81.
So 
16 4. pentagon central angle  360°  5 or 72°, and
41. 0.191... 1
2(72°)  36°.
.0
479 00

4.7 half side length  21 tan 36°, and
 side length  42 tan 36°, so
4.30
4.23 perimeter  5(42 tan 36°)  210 tan 36°
  152.574.
70
47 shaded area ¬area of pentagon  area of circle

23 ¬1
2 Pa  r
 2

9
47  0.19.
So  ¬1
2 (152.574)(21)  (21)
 2

42. 0.476... ¬216.6 units2


0
211.0
00
 5. y
8.4
 8
1.60
1.47 G ( 5, 5)
 D ( 1, 3) 4
h3
130 H ( 8, 3)
b 2
126
 h1 4 x
4 8 4 h8
10 C ( 3, 2)4 b 1 ( 2 )
21  0.48.
So  J 5, 2
8
Page 621 Practice Quiz 2
1. area ¬area of left triangle  area of middle
triangle  area of upper triangle
¬1 1 1
2 b1h1  2 b2h2  2 b2h3


¬1 1 1


2 (8)(5)  2 (7)(5)  2 (7)(2)

¬44.5 units2
30
14
60
side length  2  14 ¬28
, and 11-5 Geometric Probability
 3 3
perimeter ¬6  28  563
3 
A ¬1
Pa
2 Page 625 Check for Understanding
1. Multiply the measure of the central angle of the
2 563
¬1 (14)

sector by the area of the circle and then divide the
¬3923
 or about 679.0 mm2 product by 360°.
2. side length  72  8 2. Sample answer: darts, archery, shuffleboard
62
or 9 in., and 1
2 (9)  4.5
 3. Rachel; Taimi did not multiply  360 by the area of
central angle  360°  8 the circle.
N
or 45°, and 1
2 (45°)  22.5°.
 22.5 4. A ¬  r2
360
4.5
apothem  
tan 2 ¬
80
36
2
2.5° 0 (5 )
 10.864 in. 50
¬9 or about 17.5 units2
A ¬1
2 Pa 9 P(blue) ¬
blue area
area o
f circle
¬1

2 (72)(10.864)
5
 0

¬ 9
(52)
 2

9 or about 0.22
¬391.1 in2

403 Chapter 11
5. short side of one triangle  10, N 2
15. A ¬
360 r
so area of square  4  1
 1
2 bh  4  2 (10)(10) or 200 72 80
¬ 360
(7.52)
area of circle  r2  102  100
¬23.75 or about 74.6 units2
blue area ¬area of circle  area of square yellow area
¬100  200 or about 114.2 units2 P(yellow) ¬ 
area of circle
23.75
P(blue) ¬ blue area ¬
area ofcircle (7.5 )
2

100  200
¬  ¬0.42
100
2
16. Using A  r2, we find that the smallest circle
¬1   or about 0.36 encloses yd2, the medium circle 4 yd2, and the
6. 60 out of 100 squares are shaded. large circle 9 yd2.
P(shaded)   60 3 shaded area  9  4   6 yd2
100  5 or 0.6 shaded area
P(shaded) ¬
overall area
Pages 625–627 Practice and Apply 6
¬ 2
7. 60 out of 100 squares are shaded. 9 or 3
N 2
P(shaded)   60 17. area of sector ¬
360 r
100  0.60
6
0
8. 50 out of 100 squares are shaded. ¬
360 (6 )
2
50
P(shaded)  100  0.50 ¬6
9. 54 out of 100 squares are shaded. Use a 30°-60°-90° triangle, with the length of the
P(shaded)   54 short leg  1
2 (6)  3 so that apothem  33
 .
100  0.54 1
10. A ¬N 2
r area of triangle ¬2bh
360
2 (6)33
¬1 
72 
¬ 0 (7.5 )
36
2
¬93 
¬11.25 or about 35.3 units2 area of segment  area of sector
area ofsector  area of triangle
P(blue) ¬
area of circle
5
11.2  6  9 3 or about 3.3 units2
¬
(7.52) area of segment
P(shaded) ¬area of
circle
¬0.20
6 93
11. A ¬
N ¬
360 r
 2 (6 )
2

60  1 
3
¬ 360
60
7.52 ¬6  4 or about 0.03
N 2
 18.75 or about 58.9 units2 18. area of sector ¬360 r A
5 120
P(pink) ¬18.7 ¬360 (8 )
2
(7.52)
64 16
 1 ¬ 3
 B
3 or about 0.33
12. A ¬ N r2 Use a 30°-60°-90° triangle,
360
45  4
5  45 with apothem  1
2 (8)  4.

C
¬ 360
(7.52)
 66.3 units2 area of ABC ¬1 
2 bh
1
P(purple) ¬
purple area
 ¬2(2  43
)(4)
area of circle
6 6.3
¬16 3
¬
(7.5 )
2 area of segment ¬area of sector
¬0.375 ¬ area of triangle
N 2 64
3  16
13. A ¬
360 r ¬  3 or
40 ¬about 39.3 units2
¬ 0 (7.5 )
36
2
area of segment
¬6.25 or about 19.6 units2 P(shaded)  
area of
circle
red area 64
  163
P(red) ¬ 
area o
f circle 3
 
6.25 (8 )
2
¬
(7 

.52) ¬1 3
3  4 or about 0.20
 
¬1

9 or about 0.11 N 2
19. area of sector  
360 r
N 2
14. A ¬
360 r 72

360 (7.5 )
2
55  58  60
 36 (7.52)
0  11.25
¬84.9 units2 The five central angles of the pentagon each
green area 36

P(green) ¬
area ofcircle
measure  5 or 72°. Let s  pentagon side length
84.9 and a  apothem.
¬
(7.52)
¬0.48

Chapter 11 404
72° s
7.5 sin2  2
 In Exercises 26–28, the overall area is
s  15 sin 36° (72)  49 units2. Also,
72° red area ¬  12  (  42    32)  (  72    62)
7.5 cos 2 a

a  7.5 cos 36° ¬21 units2
area of triangle ¬1
2 bh white area ¬1
2 (overall area  red area)

1
¬2sa 1
¬2(49  21)
¬1

2 (15 sin 36°)(7.5 cos 36°) ¬14 units2
¬26.7 black area ¬white area
area of segment ¬area of sector ¬14 units2
¬ ¬ area of triangle black area
26. P(black) ¬
overall
¬11.25  26.7 area
¬8.6 14
¬
49

shaded area ¬3(area of segment) 2
¬7 or about 0.29
¬25.8
whitearea
P(shaded) ¬shaded area 27. P(white) ¬
area of circle overall area
14
¬25.8 ¬49

(7.52)
2
¬0.15 ¬7 or about 0.29
red area
In Exercises 20–23, 28. P(red) ¬ 
overall area
area of region 21
probability ¬
area of ¬
circle 49

total angle in region 3
¬ 
360° . ¬7 or about 0.43
28.
20. P(red)   8° 29. The chances of landing on a black or white sector
360° ¬0.08 are the same, so they should have the same point
147.6°  97.2°
21. P(blue or green) ¬ 
360°  0.68 value.
22. P(not red or blue) ¬1  P(red or blue) 30. Of the three colors, there is the highest
28.8° 147.6° probability of landing on red, so red should have a
¬1   360° lower point value than white or black.
¬0.51 31a. No; each colored sector has a different central
23. P(not orange or green)  1  P(orange or green) angle.
18° 
97.2° 31b. No; there is not an equal chance of landing on
¬1   360°
each color.
¬0.68
32. Sample answer: Geometric probability can help
24. court area  w
you determine the chance of a dart landing on the
¬(39)(39)
bullseye or high scoring sector. Answers should
¬1521 ft2
include the following.
39 27
out-of-bound lane width ¬ 2 ¬6 ft • Find the area of the circles containing the red
out-of-bounds area ¬w  w sector. Divide the difference by the area of the
¬6  39  6  39 larger circle.
¬468 ft2 • Find the area of the center circle and divide by
out-of-bounds area the area of the largest circle on the board.
probability   court area
33. C; total area ¬area of square  area of semicircle
468
¬
1521 ¬s2  1
2 r
 2
4
¬
13 or about 0.31 ¬52  1
2 (2.5 )
 2
27
25. service box width ¬ 2  13.5 ft

¬34.8
service box area ¬1w1 shaded area ¬area of square  area of semicircle
¬(13.5)(21) ¬52  1
2 (2.5 )
 2
¬283.5 ft2
¬15.2
court area ¬2w2 shaded area
P(shaded)  
¬(39)(39) total area
15.2
¬1521 ft2 ¬
34.8
service box area
probability   ¬0.44
court area
283
.5 34. C; y  16  4  4, so 12  y  12  4  3.
¬
1521
¬0.19

405 Chapter 11
35. area  area of rectangle  area of left triangle 45. Use the Law of Cosines, with a  g, b  32,
¬ area of upper triangle c  29, and A ¬41.
¬w  1 1 a2 ¬b2  c2  2bc cos A
2 b1h1  2 b2h2

g2 ¬322  292  2(32)(29) cos 41°
¬(28)(20)  1 1
2 (16)(35)  2 (28)(15)

g2 ¬1865  1856 cos 41°
¬1050 units2 g ¬1865
 cos
1856 ° 41
36. area  area of rectangle  area of semicircle g ¬21.5
 w  1
2 r
 2
 (12)(9)  1
2 (4)
 2

 108  8 or about 82.9 ft2 Chapter 11 Study Guide and Review


37. Side length  13 (48)  16 ft. Use a 30°-60°-90°
 Pages 628–630
triangle. Height  x3  where x  12(16)  8 ft.
1. c
A ¬1
2 bh 2. e
2 (16)83
¬1 

3. a
¬643  or about 110.9 ft2 4. f
38. A ¬s2

¬(21)2 5. b
¬441 cm2 6. d
39. Use a 30°-60°-90° triangle.
1 1 7. P  2(23)  2(16)  78 ft
Half side length ¬(8), side length ¬(16), Use a 30°-60°-90° triangle to find the height, with
3
 
3
1 x as the length of the shorter leg.
perimeter ¬6  (16)  323
 in.
3 16 ¬2x
1
A ¬2Pa 8 ¬x
1
¬2(323
)(8) height ¬x3  ¬83  ft
A ¬bh
¬1283  or about 221.7 in2
¬(23)(83 )
40. mAFB  mBFC ¬180 ¬1843  or about 318.7 ft2
mAFB  72 ¬180
8. P ¬2(36)  2(22) ¬116 mm
mAFB ¬108
Use a 30°-60°-90° triangle to find the height, with
41. mCFD  mAFD ¬180 x as the length of the shorter leg.
(4a  1)  (2a  5) ¬180 22 ¬22x
6a  6 ¬180 11 ¬x ¬height
6a ¬186 A ¬bh
a ¬31 ¬(36)(11)
mCFD ¬4a  1 ¬396 mm2
¬4(31)  1 9. y
¬123
42. mCFD  mAFD ¬180
(4a  1)  (2a  5) ¬180
6a  6 ¬180 A ( 6, 1) B ( 1, 1)
6a ¬186
a ¬31 x
mAFD ¬2a  5
¬2(31)  5
¬57
43. From Exercises 40 and 42, mAFB ¬108 and
mAFD ¬57.
D ( 6, 6) C ( 1, 6)
mDFB ¬mAFB  mAFD
¬108  57
ABCD is a square.
¬165
A ¬s2
44. Use the Law of Cosines, with a ¬p, b  6.8, ¬72
c  11.1, and A ¬57. ¬49 units2
a2 ¬b2  c2  2bc cos A
p2 ¬6.82  11.12  2(6.8)(11.1)cos 57°
p2 ¬169.45  150.96 cos 57°
p ¬169.4
5 0.96
 15os c 57°
p ¬9.3

Chapter 11 406
3  1 4
10. y S
slope of R ¬ (1)   or 1
5  4
3  (1)
2
S
slope of P  ¬ (7)  2 or 1
5 
G ( 2, 2) H ( 8, 2) 13 2
R
slope of Q ¬ (3)  2 or 1
1 
x PQRS is a rectangle, since opposite sides have the
same slope and the slopes of consecutive sides are
negative reciprocals of each other.
E ( 7, 2) R
Length of S  ¬[1
  (5)]2
 [1)]
 (32
F ( 1, 2)
¬42 
R
Length of Q  ¬[1
 (3)]2
 [1 3]2  22
2  (2) PQRS is not a square since not all sides have the
G ¬ 4
slope of F 1  1 or 4
2  same length.
2  (2)
H
 ¬ 4 A ¬w
slope of E 7  1 or 4
8 
2  (2) ¬(42)(22)
E
slope of F ¬7 
1  0

6 or 0 ¬16 units2
2 2 0
H
slope of G ¬
8  2  6 or 0 13. A ¬1

2 bh
EFGH is a parallelogram, since opposite sides 336 ¬1

2 b(24)
have the same slope. Slopes of consecutive sides 336 ¬12b
are not negative reciprocals of each other, so the 28 ¬b
parallelogram is neither a square nor a rectangle. base ¬CE  28 in.
A ¬bh
14. A ¬1
2 h(b1  b2)

¬6  4
¬24 units2 75 ¬1
2 h(17  13)

11. 75 ¬15h
L(5, 5) y
5 ¬h
The height is 5 m.
15.
M (1, 1) x 20

K ( 5, 0)
36
10
J ( 1, 4)
side length  100  5  20, and 1
2 (20)  10

0  (4)
K
 ¬  4 central angle  360  5  72°, and 1
slope of J 5  (1)  4 or 1 10 2 (72°)  36°

15 4 apothem   36°  13.764
tan 
M
slope of L  ¬ (5)  4 or 1
1  A ¬1
2 Pa
1  (4)
5
M
slope of J ¬ (1)  0 undefined
1  ¬1

2 (100)(13.764)
5 0 5
L
slope of K ¬
5  (5)  0 undefined ¬688.2 in2
JKLM is a parallelogram, since opposite sides 16.
have the same slope. Slopes of consecutive sides
are not negative reciprocals of each other, so the
12
parallelogram is neither a square nor a rectangle. 18
A ¬bh
¬4  5
¬20 units2
6
12. y
half side length  12 (12)  6 mm

Q (3, 3) perimeter  10  12  120 mm
central angle  360°  10  36°, and 1
2 (36°)  18°

6
R (1, 1) apothem   tan 18°  18.466
x
A ¬1

2 Pa
¬1

2 (120)(18.466)
P ( 7, 1)
¬1108.0 mm2
S ( 5, 3)

3  (1)
4
Q
slope of P ¬ (7)  4 or 1
3 

407 Chapter 11
17. area ¬area of rectangle 5. T(5, 5)
y
¬ combined area of two semicircles
¬w  r2
¬(8)(3)  (1.5)2 S (9, 3)
¬24  2.25 or about 31.1 units2 U (3, 1)
18. For the height of the trapezoid, use a 30°-60°-90° x
triangle: x  4, so h  43 .
area ¬area of trapezoid  area of semicircle R (7, 1)
¬1 1 2
2 h(b1  b2)  2 r

¬1
 )(8  10)  1
2 (43 2 (4)
 2

¬363   8 or about 87.5 units2


1
1  2 1
19. A ¬N 2
r R
slope of U ¬ 7  3 ¬ 4 or  2

360
3 5 2 1
12
¬ 0 S
slope of T  ¬
9  5 ¬ 4 or  2

360 (6 )
2
5 1 4
¬12 slope of UT
 ¬5  3 ¬ 2 or 2
3  ( 1) 4
P(red) ¬ red area S
slope of R ¬ 9  7 ¬ 2 or 2

area of
circle
12 RSTU is a rectangle, since opposite sides have the
¬
(62) same slope and the slopes of consecutive sides are
¬1

3 or about 0.33
negative reciprocals of each other.
N 2 Length of UT ¬(5)  3 2  (5
 1)2  25
20. A ¬ r
360 Length of UR ¬(7)  31 2  ( 1)2  25
36 60
¬ 360 (6 )
2 Thus RSTU is in fact a square.
¬9.6 A ¬s2
purple or green area ¬(25)2
P(purple or green) ¬ 
area of circle ¬20 units2

9.6
¬
(62) 6.
y S ( 4, 5)
¬4 T ( 7, 5)
15 or about 0.27

Chapter 11 Practice Test


x
R ( 2, 0) U ( 5, 0)
Page 631
1. a
2. c
3. b
4. R(6, 8) 0 0 0
y U
slope of R ¬
5  2  3 or 0
5 5 0
T
slope of S  ¬
7  4  3 or 0
5 0 5
T
slope of U  ¬
75  2
S(1, 5)
5 
0 5
S
slope of R ¬42  2
U (6, 4)
RSTU is a parallelogram, since opposite sides
T (1, 1) have the same slope. Slopes of consecutive sides
x
are not negative reciprocals of each other, so the
parallelogram is neither a square nor a rectangle.
A ¬bh
14 3 3 ¬(3)(5)
T
slope of U ¬ (6)  5 or 5
1  ¬15 units2
58 3 3
S
slope of R  ¬ (6)  5 or 5
1 
84 4
R
slope of U ¬ (6)  0 is undefined
6 
51 4
S
slope of T ¬
1  (1)  0 is undefined
RSTU is a parallelogram, since opposite sides
have the same slope. Slopes of consecutive sides
are not negative reciprocals of each other, so the
parallelogram is neither a square nor a rectangle.
A ¬bh
¬(5)(4)
¬20 units2

Chapter 11 408
N 2
7. y 13. A ¬
360 r
40 60
8 ¬ 360 (6 )
2

¬10
4 S ( 9, 3) red area
P(red) ¬
area of circle
8 4 4 T ( 12, 1)
10
¬
x (6 )
2
5
¬
18 or about 0.28
R ( 3, 6)
N 2
U ( 6, 8) 14. A  
360 r
8
8
6
¬
360 (6 )
2

8 (6) 2 ¬8.6


U
 ¬  2
slope of R 63 3 or  3 orange area
13 
2 2 P(orange) ¬ 
area of circle
T 12  9  3 or  3
 ¬
slope of S   
8.
¬ 6

3  ( 6) 9 3 (62)
S
slope of R  ¬ 9  3  6 or 2

¬0.24
1  ( 8) 9 3
T
slope of U ¬ 12  6  6 or 2

15. A¬¬N 2
r
360
RSTU is a rectangle, since opposite sides have the 45  35  55
¬  (62)
360
same slope and the slopes of consecutive sides are
negative reciprocals of each other. ¬13.5
green area
U
Length of R   [6 ]  382  [6)]
 (2  13  P(green) ¬ 
area of circle
S  [9 ]  32  [3)]
 (62  313  5
13.
Length of R ¬
(62)
RSTU is not a square since not all sides have the
¬3

8 or about 0.38
same length.
A  w 16. area ¬area of trapezoid  area of parallelogram
 (13
)(313 ) ¬1
2 h(b1  b2)  b1h2

 39 units2
¬1
2 (8)(21  24)  (21)(14)

8. area ¬area of left triangle  area of right triangle
¬474 units2
¬1 1
2 b1h1  2 b2h2

17. triangle height  x3 , where x  1
2 (6)  3

¬1 1
2 (17)(6)  2 (28)(15)
 area  area of rectangles  area of triangles
¬261 m2 ¬2w  2  1

2 bh
9. area ¬area of rectangle  area of triangle ¬2(6)(5)  2  1
 )
2 (6)(33
¬w  1
2 bh ¬60  183  or about 91.2 units2
¬(39)(18)  1
2 (56  39)(18)

18. area ¬area of square
¬855 yd2 ¬ combined area of semicircles
10. area ¬1
2 h(b1b2)
 ¬s2  2  1
2 r
 2
¬1 ¬(7)  (3.5)2
2
2 (22)[32  (3  32  7)]

¬49  12.25 or about 87.5 units2
¬814 cm2
19. side length  9  6 or 1.5 in., and 1
2 (1.5)  0.75

11. octagon central angle  360°  8  45°, and
apothem  0.753 
1(45°)  22.5°
2 A ¬1
2 Pa
half side length  3 tan 22.5°, and side length 
6 tan 22.5°, so ¬1
 )
2 (9)(0.753
perimeter  8(6 tan 22.5°)  48 tan 22.5° ¬3.3753  or about 5.8 in2
 19.88 ft. 20. D; two sides are horizontal, lying on the lines
A ¬12
 Pa y  1 and y  4. The lengths of the horizontal
¬1
 sides are |3  5|  |1  7|  8. So the figure
2 (19.88)(3)
is a parallelogram, with height |4  (1)| or 5.
¬29.8 ft2
A ¬bh
12. side length  115  5  23 cm, and 1
2 (23)  11.5
 ¬(8)(5)
central angle  360°  5  72°, and 1 ¬40 units2
2 (72°)  36°

11.5
apothem   36°  15.828
tan 
A ¬1

2 Pa
¬1

2 (115)(15.828)
¬910.1 cm2

409 Chapter 11
Chapter 11 Standardized Test Practice 11. By the Exterior Angle Theorem,
mR  mS ¬mSTP
mR  90 ¬150
Pages 632–633 mR ¬60
2x  4
1. B; 3 6  ¬18
 12. Since pairs of vertical angles at C are congruent,
31 and A and E are congruent, by AA similarity
3 x  3  ¬18
 2
ABC  EDC.
x  2 ¬18 AB AC
ED ¬ EC
  
x ¬16
x ¬16  ¬1
A B 1 2
250 200
2. B; the angle of the path from the school to the
AB ¬250
200 
11
2
baseball field is a 90° angle, and the angle of
Sam’s path from the library to the baseball field ¬140
is greater than that. So the angle of Sam’s path is 13. The image of J(6,3) for the translation
obtuse. (x, y) → (x, y  5) is J
(6, 3  5) or J
(6, 2).
3. A; let p, q, and r represent parts of the given 14. Use trigonometry.
statements. 1560
a. tan 38° ¬x
p: you exercise
156 0
q: you maintain better health x ¬ 
tan 38°
r: you will live longer
¬1997 ft
Given: p → q and q → r. Use the Law of Syllogism
1560
to conclude p → r. That is, if you exercise, you will b. tan 35° ¬ y
live longer. 1560
y ¬
tan 
35°
4. C; mADE ¬180  mDEA  mEAD
¬180  40  60 ¬2228 ft
¬80 c. Use the results from (a) and (b).
Since 
DE is a transversal for 
AD and 
BE, and campground width ¬y  x
corresponding angles ADE and BEC are ¬2228  1997
congruent, A D and BE are parallel. ¬231 ft
15. y
5. D; because the front of the tent is isosceles and
the entrance is an angle bisector, the two sides of B C
the front of the tent are congruent triangles, by
SAS. Then the two bases are of equal length,
namely 3 ft, and so the distance between the x
A
stakes is 3  3 or 6 ft.
D
6. D; the gazebo is a regular hexagon. The angle
measuring x is an exterior angle of the hexagon.
Each exterior angle of a regular hexagon
360
measures  6 or 60. So x  60.
7. A; this is Theorem 10.5.
a. Since BC is horizontal, D must lie on the
8. C; A  1 1
2 Pa  2 (6  9)(7.8) ¬210.6 cm
2
x-axis. And since the slope of A is 4
B 
3 , the slope
9. The library coordinates are (4,3), and the fire C
of D 4
 must be 3. So D must be located 4 units
station coordinates are (8, 3). The post office lies down and 3 units left of C(8,4), at D(5,0).
at the midpoint of the segment from (4, 3) to
b. AD  5, and height  4.
(8, 3). So the post office coordinates are given by
x x y y A ¬bh
4  3 
M
1
2 1
, 2
 ¬ 
2 ,
8 
2 
 3
¬(5)(4)
2 2
¬(2, 0) ¬20 units2
10. Put the equation in slope-intercept form.
3x  6y ¬12
6y ¬3x  12
y ¬1
2x  2

The slope of the graph of the equation is m ¬1

2.
The slope of the perpendicular line is the negative
reciprocal of 1
2 , or 2.


Chapter 11 410
Chapter 12 Surface Area
Page 635 Getting Started 2. In a square pyramid, the faces meet at a point. In
1. True; points A, C, and D lie in plane N , so ADC a square prism the faces are perpendicular to
lies in plane N . each base.
2. False; points A, B, and C do not lie in plane K , so 3. Sample answer:
ABC does not lie in plane K.
3. Cannot be determined; neither the given
information nor the figure allow a determination 4.
of whether or not the line containing A B is
parallel to plane K.
back view
4. False; the line containing A C
 lies in plane N and
only  lies in both plane N and plane K , so the corner view
line containing A C
 does not lie in plane K. 5. The base is a hexagon, and six faces meet at a
5. The figure is a trapezoid with bases b1  19 ft point. So this solid is a hexagonal pyramid.
and b2  29 ft and height h  16 ft. The area is The base is ABCDEF. The faces are ABCDEF,
given by A  12 h(b1  b2).
 AGF, FGE, EGD, DGC, CGB, and BGA.
A  1 The edges are A F, F
E
, E
D, D C
, C
B
, BA, A
G
, FG
,
2 (16)(19  29)  384

G
E, D
G , C
G, and B
G. The vertices are A, B, C, D,
The area of the figure is 384 ft2. E, F, and G.
6. The figure is a trapezoid with bases b1  12 mm 6. The bases are squares. So this is a square prism.
and b2  35 mm and height h  13 mm. The area The bases are KJIH and MNOL. The faces are
is given by A  1
2 h(b1  b2).
 KJIH, MNOL, JNOI, JKMN, KHLM, and IHLO.
A  1 The edges are K H, K
J, J
I, IH
, J
N
, IO
, H
L, K
M
,
2 (13)(12  35)  305.5

N
M , M
L , N
O, and LO
. The vertices are H, K, J, I,
The area of the figure is 305.5 mm2.
L, M, N, and O.
7. The figure is a triangle with base b  1.9 m and
7. The bases are circles. So this is a cylinder. The
height h  1.9 m. The area is given by A  1

2 bh. bases are circles P and Q.
A  1
2 (1.9)(1.9)  1.805
 8. To get round slices of cheese, slice the cheese
Rounded to the nearest tenth, the area of the parallel to the bases. To get rectangular slices,
figure is 1.8 m2. place the cheese on the slicer so the bases are
8. A ¬r2 perpendicular to the blade.

 
2
¬ d

2
¬1
4 d
 2 Pages 640–642 Practice and Apply
9.
¬1
4 (19.0)
 2

¬283.5 cm2
9. A  r2
 (1.5)2 back view
 7.1 yd2
10. A ¬r2 corner view

 
2
¬ d
2
10.

¬1
4 d
 2
back view
¬1
4 (10.4)
 2

¬84.9 m2
corner view
11.
12-1 Three-Dimensional Figures back view

Page 639 Check for Understanding corner view


1. The Platonic solids are the five regular polyhedra. 12.
All of the faces are congruent, regular polygons.
In other polyhedra, the bases are congruent
parallel polygons, but the faces are not back view
necessarily congruent.

corner view

411 Chapter 12
13. 27. The resulting shape is a circle.
28. The resulting shape is a rectangle.
29. The resulting shape is a rectangle.
top view left view front view right view
30. The resulting shape is a square.
14. 31. intersecting three faces and parallel to base;

top view left view front view right view


15.
32. intersecting three faces and edges of base;
top view left view front view right view

16. The bases are triangles. So this is a triangular


prism. The bases are MNO, and PQR. The
faces are MNO, PQR, OMPR, ONQR, and
PQNM. The edges are M N, N
O, O
M , PQ, Q
R
, P
R,
Q
N , MP, and O
R. The vertices are M, N, O, P, Q, 33. intersecting all four faces, not parallel to any face;
and R.
17. The base is a rectangle, and four faces meet in a
point. So this solid is a rectangular pyramid. The
base is DEFG. The faces are DEFG, DHG,
GHF, FHE, and DHE. The edges are D G,
F
G , F
E, E
D, D
H
, E
H, F
H, and GH
. The vertices
are D, E, F, G, and H. 34. If the number of sides of a base of a pyramid
18. The base is a triangle, and three faces meet in a increases infinitely, the solid that results is a
point. So this solid is a triangular pyramid. The cone.
base is IJK. The faces are IJK, ILK, KLJ, 35. If the number of sides of the bases of a prism
and ILJ. The edges are IK , K
J
, IJ
, IL
, K
L
, and increases infinitely, the solid that results is a
L
J . The vertices are I, J, K, and L. cylinder.
19. The bases are circles. The solid is a cylinder. The 36. The shapes seen in an uncut diamond are
bases are circles S and T. triangles and squares or rectangles.
20. The solid is a sphere. 37. The shapes seen in the emerald-cut diamond are
21. The solid has a circle for a base and a vertex. So it rectangles, triangles, and quadrilaterals.
is a cone. The base is circle B. The vertex is A. 38. The shapes seen in the round-cut diamond are
22. 16: Yes; Euler’s formula is true. octagons, triangles, and quadrilaterals.
F  V ¬E  2 39a. 5 faces: triangular prism
5  6 ¬9  2 39b. 6 faces: cube, rectangular prism, or hexahedron
11 ¬11 39c. 6 faces: pentagonal pyramid
17: Yes; Euler’s formula is true. 39d. 7 faces: hexagonal pyramid
F  V ¬E  2
39e. 8 faces: hexagonal prism
5  5 ¬8  2
10 ¬10 40. Yes, there is a pattern. The number of sides of the
18: Yes; Euler’s formula is true. base of a prism is 2 less than the number of faces
F  V ¬E  2 in the polyhedron. The number of sides of the
4  4 ¬6  2 base of a pyramid is 1 less than the number of
8 ¬8 faces.
19–21: No; these figures are not polyhedrons, so 41. No; the number of faces is not enough information
Euler’s formula does not apply. to classify a polyhedron. A polyhedron with
23. No, not enough information is provided by the top 6 faces could be a cube, rectangular prism,
and front views to determine the shape. hexahedron, or a pentagonal pyramid. More
information is needed to classify a polyhedron.
24. Sample answer: The speaker could be shaped like
a rectangular prism, or the sides could be angled. 42.
Polyhedra
Regular
(Platonic Solids)

Cubes Tetrahedrons
right view left view
Prisms Pyramids
25. The resulting shape is a parabola.
26. The resulting shape is a triangle.

Chapter 12 412
43. Sample answer: Archaeologists use two- The figure is composed of two triangles, one
dimensional drawings to learn more about the rectangle, and one square. Find the area.
structure they are studying. Egyptologists can A ¬1 1
2 b1h1  2 b2h2  s  w
 2
compare two-dimensional drawings of the
pyramids and note similarities and any ¬1
 1
2 (3)(2)  2 (2)(5)  3  (5)(3)
2

differences. Answers should include the following. ¬32


• Viewpoint drawings and corner views are types The area is 32 units2.
of two-dimensional drawings that show three
55. y
dimensions.
• To show three dimensions in a drawing, you
L (2, 2)
need to know the views from the front, top, and
each side. M (0, 1)
44. D; a circle cannot be formed by the intersection of x
a cube and a plane.
3
45. D; xx4  1x N (0, 2)
P (4, 2)
1  1  1  1
4 3 2 1
46. There are 6 planes of symmetry. Each plane The figure is composed of two triangles and a
contains the center of the tetrahedron and one rectangle.
edge, thus bisecting the opposite edge. A ¬1 1
2 b1h1  2 b2h2  w

47. A cylinder has infinite planes of symmetry. Planes
¬1 1
2 (2)(4)  2 (2)(1)  (3)(2)

pass through the centers of the bases.
48. A sphere has infinite planes of symmetry. Planes ¬11
pass through the center of the sphere. The area is 11 units2.
56. Base and Sides: Each pair of opposite sides of a
parallelogram has the same measure. Each base
Page 642 Maintain Your Skills is 15 m and each side is 12 m.
87 Perimeter: The perimeter of a polygon is the sum
49. P(steak)  
360  0.242 of the measures of its sides. So, the perimeter of
87  102  118 the parallelogram is 2(12)  2(15) or 54 m.
50. P(not seafood)   360  0.853
102  118 Height: Use a 30-60-90 triangle to find the
51. P(either pasta or chicken)     0.611
360 height. Recall that if the measure of the leg
53  118 opposite the 30 angle is x, then the length of the
52. P(neither pasta nor steak)   
360  0.475
hypotenuse is 2x, and the length of the leg
53. y A (1, 4) opposite the 60 angle is x 3.
12  2x
16  x
D (3, 1) B (4, 1) So, the height of the parallelogram is 6 3 meters.
Area: A ¬bh
x ¬(15)(63)
¬90 3
C (1, 2) ¬155.9
The perimeter is 54 m and the area is
approximately 155.9 m2.
The figure is composed of two triangles, each with 57. Base and Sides: Each pair of opposite sides of a
height 3 and base 7. Find the area. parallelogram has the same measure. Each base
A ¬2 1

2 bh  is 25 ft and each side is 20 ft.
¬bh Perimeter: The perimeter of a polygon is the sum
¬(7)(3) of the measures of its sides. So, the perimeter of
¬21 the parallelogram is 2(25)  2(20) or 90 ft.
The area is 21 units2. Height: Use a 30-60-90 triangle to find the
height. Recall that if the measure of the leg
54. y opposite the 30 angle is x, then the length of the
hypotenuse is 2x, and the length of the leg
H (1, 1) opposite the 60 angle is x 3.
J (4, 1)
20  2x
10  x
x
G (2, 1) So, the height of the parallelogram is
10 3 feet.

F (2, 4) K (6, 4)

413 Chapter 12
Area: A ¬bh 3. 4.
¬(25)(103 )
¬250 3
¬433.0
The perimeter is 90 ft and the area is
approximately 433.0 ft2.
58. Base and Sides: Each pair of opposite sides of a
parallelogram has the same measure. Each base 5.
is 68 in. and each side is 42 in.
Perimeter: The perimeter of a polygon is the sum
of the measures of its sides. So, the perimeter of
the parallelogram is 2(68)  2(42) or 220 in. 4
Height: Use a 45-45-90 triangle to find the 7 6 7
height. Recall that if the measure of each leg is x,
then the length of the hypotenuse is x 2.
42 ¬x 2
42
  ¬x
2
212  ¬x Surface area  2(4  7)  2(4  6)  2(6  7)
So, the height of the parallelogram is  56  48  84
21 2 inches.  188
Area: A ¬bh The surface area of the rectangular prism is
¬(68)(212 ) 188 in2.
¬1428 2 6. Use the Pythagorean Theorem to find the height
¬2019.5 of the prism.
The perimeter is 220 in. and the area is 172 ¬82  h2
approximately 2019.5 in2. 289 ¬64  h2
59. A  w 60. A ¬w 225 ¬h2
 (20)(15) ¬(4)(13) 15 ¬h
 300 ¬52
The area is 300 cm2. The area is 52 ft2.
61. A  w
 (60)(72)
 4320
The area is 4320 in2.
62. A  s2
 1.72
 2.9
9
The area is approximately 2.9 m2.

15 8 17

12-2 Nets and Surface Area

Page 645 Check for Understanding


1. Sample answer:
Surface area
 8  9  9  15  9  17  1 1
2  8  15  2  8  15

 72  135  153  60  60
2. On isometric dot paper, the dots are arranged in  480
triangles, which aid in drawing three-dimensional The surface area of the right triangular prism
objects. On rectangular dot paper, the dots are is 480 ft2.
arranged in squares, which aid in drawing the
nets and orthogonal views of three-dimensional
objects.

Chapter 12 414
7. 13. 14.

15.

Surface area ¬42  4  1


2 46

¬16  48
¬64
The surface area of the square pyramid is 64 cm2. Surface area  2(32)  4(3  4)
8. C; the net only forms three of the four sides. Two  18  48
of the triangles overlap rather than being two  66
different faces. The surface area of the rectangular prism
is 66 units2.
16.
Pages 646–648 Practice and Apply
9.

10.

Surface area  2(2  5)  2(2  6)  2(5  6)


 20  24  60
 104
The surface area of the rectangular prism
is 104 units2.
17.
11.

12.

Surface area ¬42  4  1


2 45

¬16  40
¬56
The surface area of the square pyramid
is 56 units2.

415 Chapter 12
18. Surface area
1
 2  2  6  8  5  8  5  6  5  10
 48  40  30  50
 168
The surface area of the triangular prism is
168 units2.
21. Use the Pythagorean Theorem to find the height
of the triangle.
2
52 ¬2  h2
4
Surface area ¬22  4  1
2 24
 25 ¬4  h2
¬4  16 21 ¬h2
¬20 
21 ¬h
The surface area of the square pyramid
is 20 units2.
19.

1
Surface area ¬2(5  7)  4  7  2  2  4  21

¬70  28  4 21
¬116.3
The surface area of the triangular prism is
approximately 116.3 units2.
Surface area  6(4.52) 22. Use the Pythagorean Theorem to find the
 121.5 unknown edge length.
The surface area of the cube is 121.5 units2. c2 ¬a2  b2
20. Use the Pythagorean Theorem to find the c2 ¬(8  6)2  62
hypotenuse of the right triangle. c2 ¬4  36
c2 ¬a2  b2 c2 ¬40
c2 ¬62  82 c ¬40
c2 ¬36  64 c ¬210
c2 ¬100 Find the area of the trapezoid.
1
c ¬10 A ¬2h(b1  b2)
1
¬2(6)(6  8)
¬42

Chapter 12 416
Surface area
 2  13.5  5  6  2(3  5)  5  32

 27  30  30  152 
 108.2
The surface area of the prism is approximately
108.2 units2.
24.
6.99 ft 6.99 ft

2.46 ft
9.89 ft
2.46 ft 2.46 ft
6.99 ft
6.99 ft

Use the Pythagorean Theorem to find the


hypotenuse.
c2 ¬a2  b2
c2 ¬6.992  6.992
c ¬97.7202
The thickness of the sandwich is
29.5
8  13.5  8  29.5 in.   
12 ft.
Surface area ¬2  2  6.992  26.99  
12 
1 29.5

Surface area  2(6  8)  82  210
  8  2  42 29.5
 12  97.72
02 
 96  64  1610  84 
 294.6 ¬107.5
The surface area of the prism is approximately The surface area of the sandwich is
294.6 units2. approximately 107.5 ft2.
23. Use the Pythagorean Theorem to find the 25.
unknown edge length.
c2 ¬a2  b2
c2 ¬(6  3)2  32
c2 ¬9  9
c2 ¬18 26.
c ¬18
c ¬32
Find the area of the trapezoid.
1
A ¬2h(b1  b2)
1
¬2(3)(3  6) 27. 28. E F
¬13.5 H
F E H
G

B
C D A B
A D

29. N 30. Q

S R
R
M R
Q
T U T
P
N
N
Q

417 Chapter 12
31. W X Z Figure C:

U
R
T
T V R

W
Y Z Surface area  2(2  4)  2(2  5)  2(4  5)
X Z  16  20  40
32.  76
The surface area of the rectangular prism is
76 units2.
36. Figure A:
Surface area  6(2  2)
 24
The surface area of the cube is 24 units2.
Figure B:
Use the Pythagorean Theorem to find the height
33. of the triangles.
2
22 ¬h2  2
2

4 ¬h2  1
3 ¬h2
3 ¬h
1
Surface area ¬3(2  6)  2  2  2  3

34. ¬36  2 3
The surface area of the triangular prism is
(36  2 3) units2.
Figure C:
Surface area  2(4  8)  2(4  10)  2(8  10)
 64  80  160
 304
35. Figure A: The surface area of the rectangular prism is
304 units2.
37. The surface area quadruples when the
dimensions are doubled. For example, the surface
Surface area  6(1  1) area of the cube is 6(12) or 6 square units. When
6 the dimensions are doubled the surface area is
The surface area of the cube is 6 units2. 6(22) or 24 square units.
Figure B: 38. When dimensions are tripled, the surface area
Use the Pythagorean Theorem to find the height will be nine times greater than the original
of the triangles. surface area. For example, the surface area of the
2
12 ¬h2  2
1 cube is 6(12) or 6 square units. The new surface
1 area is 6(32) or 54 square units.
1 ¬h2  4
39. No; 5 and 3 are opposite faces; the sum is 8.
3
4 ¬h2 40. Sample answer: Car manufacturers want their
 
3 cars to be as fuel efficient as possible. If the car is
2 ¬h designed so the front grill and windshield have a
smaller surface area, the car meets less resistance
from the wind. Answers should include the
following.
1 
3 • A small compact car has less surface facing the
Surface area ¬3(1  3)  2  2  1  2 wind than a larger truck, so smaller sedans

3
¬9   2 tend to be more efficient than larger vehicles.
The surface area of the triangular prism is • Of the two-dimensional models studied in this
chapter, orthogonal drawings would be helpful
9  23  units2. to the designers.
41. C; only net C can be folded into a rectangular
prism.

Chapter 12 418
42. B; 16a3  54b3  2(8a3  27b3) 2. Sample answer: The bases are ACHG and BDFE.
 2[(2a)3  (3b)3] The lateral faces are ABDC, GEFH, BEGA, and
 2(2a  3b)(4a2  6ab  9b2) A
DFHC. The lateral edges are B, E
G
, F
H, and
C
D.
base D
F
Page 648 Maintain Your Skills lateral edges
43. The resulting shape is a rectangle. B
E lateral faces
44. The resulting shape is a triangle. base
C H
45. The resulting shape is a rectangle.
(20)(20) A
46. P(butterfly in the flower bed) ¬ 
(100)(200) G
¬0.02 3. Use the Pythagorean Theorem to find the
47. mFLJ  180 because it is a semicircle. By the measure of the hypotenuse of the triangular base.
Inscribed Angle Theorem, c2 ¬a2  b2
c2 ¬82  152
mFHJ ¬1

2 mFLJ c2 ¬64  225
¬1
 c2 ¬289
2 (180)
c ¬17
¬90
L  Ph
48. mLK  60 because it is 1

6 the measure of a circle  (17  15  8)(21)
(360).  (40)(21)
49. mGHL  240 because it is 4 
6 the measure of a  840
circle (360). T ¬L  2B
By the Inscribed Angle Theorem,
¬840  2  1

2 bh
mLFG ¬1

2 mGHL ¬840  (8)(15)
¬1

2 (240) ¬840  120
¬120 ¬960
50. A  bh The lateral and surface areas are 840 units2 and
 16  14 960 units2, respectively.
 224 4. L(7 9 base)  Ph
The area of the parallelogram is 224 ft2.  (2  7  2  9)(6)
 192
51. A ¬1
2 h(b1  b2)

L(6 9 base)  Ph
¬1
2  7(6  12)
  (2  6  2  9)(7)
¬63  210
The area of the trapezoid is 63 cm2. L(6 7 base)  Ph
 (2  6  2  7)(9)
52. A ¬1

2 bh  234
¬1 T  L  2B
2  6.5  4

 234  2(6  7)
¬13
 234  84
The area of the triangle is 13 yd2.
 318
53. A ¬1
2 h(b1  b2)
 The lateral areas are 192 units2 (7 9 base),
210 units2 (6 9 base), and 234 units2
¬1
2  10(13  9)

(6 7 base). The surface area is 318 units2.
¬110 5. The perimeter of the ceiling (base) is 2(20) 
The area of the trapezoid is 110 cm2. 2(15)  40  30  70 ft. Find the total surface
area to be painted.
TLB
 Ph  B
12-3 Surface Areas of Prisms  (70)(12)  (20)(15)
 840  300
Page 651 Check for Understanding  1140
The surface area to be painted is 1140 ft2.
1. In a right prism the altitude is also a lateral edge.
In an oblique prism, the lateral edges are not
perpendicular to the base.
Pages 651–654 Practice and Apply
6. L(3 4 base)  Ph
 (2  3  2  4)(12)
 168

419 Chapter 12
L(3 12 base)  Ph 14. L ¬Ph
 (2  3  2  12)(4) 156 ¬P(13)
 120 512 ¬P
L(4 12 base)  Ph The perimeter of the base must be 12 inches.
 (2  4  2  12)(3) There are three rectangles with integer values for
 96 the dimensions that have a perimeter of 12. The
The lateral areas are 168 units2 (3 4 base), dimensions of the base could be 5 1, 4 2, or
120 units2 (3 12 base), and 96 units2 3 3.
(4 12 base). 15. L ¬Ph
7. L  Ph 96 ¬P(4)
 (4  5  7)(8) 24 ¬P
 128 The perimeter of the base must be 24 meters.
The lateral area is 128 units2. There are six rectangles with integer values for
8. Use the Pythagorean Theorem to find the the dimensions that have a perimeter of 24. The
measure of the third side of the triangular base. dimensions of the base could be 1 11, 2 10,
c2 ¬a2  b2 3 9, 4 8, 5 7, or 6 6.
c2 ¬72  82 16. Use the Pythagorean Theorem to find the
c2 ¬49  64 measure of the third side of the triangular base.
c2 ¬113 c2 ¬a2  b2
c ¬113
 172 ¬a2  82
L  Ph 289 ¬a2  64
 (7  8  113)(10) 225 ¬a2
 256.3
15 ¬a
The lateral area is approximately 256.3 units2.
T ¬L  2B
9. L  Ph ¬Ph  2B
 (2  4  3  4  5)(9)
 162 ¬(8  15  17)(4)  2  1
2  8  15

The lateral area is 162 units2. ¬280
10. L  Ph The surface area of the prism is 280 units2.
 (2  7  2  4  8  2)(9) 17. T  L  2B
 342  Ph  2B
The lateral area is 342 cm2.  (3  3  3  3)(8)  2(3  3)
11. Find the lateral area of the hollowed-out prism,  114
including the inner faces. The surface area of the prism is 114 units2.
L  (outer perimeter)h  (inner perimeter)h 18. T  L  2B
 (4  4)(8)  (4  1)(8)  Ph  2B
 160  (2  4  2  11)(7.5)  2(4  11)
The lateral area is 160 units2 (square base).  313
L  2(42  12)  2(4  8)  4(1  8) The surface area of the prism is 313 units2.
 2(15)  2(32)  4(8)
19. Use the Pythagorean Theorem to find the
 30  64  32
measure of the third side of the triangular base.
 126
c2 ¬a2  b2
The lateral area is 126 units2 (rectangular base).
152 ¬a2  92
12. The surface area of a cube is given by A  6s2, 225 ¬a2  81
where s is the length of a lateral edge. 144 ¬a2
864  6s2
12 ¬a
144  s2
T ¬L  2B
812  s
¬Ph  2B
The length of the lateral edge is 12 in.
13. Use the Pythagorean Theorem to find the ¬(9  12  15)(11.5)  2  1
2  9  12

measure of the hypotenuse of the triangular base. ¬522
c2 ¬a2  b2 The surface area of the prism is 522 units2.
c2 ¬52  122
c2 ¬169
c ¬13
So, P  5  12  13  30.
T ¬L  2B
T ¬Ph  2B
540 ¬30(h)  2  12  5  12

540 ¬30h  60
480 ¬30h
16 ¬h
The height is 16 cm.

Chapter 12 420
20. Use trigonometry, the Pythagorean Theorem, and 23. L  Ph
the figure to find the measures of the missing  (15  15  15  15)(10)
sides of the bases.  600
60
0
400  1.5 gallons needed for 1 coat

60
c So, 3 gallons are needed for 2 coats.
4
24. T  L  B
 Ph  B
9 h  (15  15  15  15)(10)  152
 825
For two coats, the surface area is 1650 ft2.
1650
400  4.125

Since only whole gallons may be purchased,
tan60 ¬h
4
5 gallons must be purchased to paint the walls
and ceiling.
4tan60 ¬h
5 $16  $80, so it will cost $80 to paint the
4 3 ¬h
walls and ceiling.
c2 ¬a2  b2
c2 ¬42  (43)2 25. Estimate the surface area of the Corn Palace
c2 ¬16  48 using a rectangular prism.
c2 ¬64 L  Ph
c ¬8  (2  310  2  185)(45)
Find the surface area.  44,550
T ¬Ph  2B The area to be covered is estimated to be
44,550 ft2.
)(11)  2  1
¬(5  8  9  43  )(5  9)
2 (43 26. L ¬Ph
¬415.2 ¬(2  310  2  185)(45)
The surface area is approximately 415.2 units2. ¬44,550
21. Use trigonometry, the Pythagorean Theorem, and 44,5
50
15 ¬2970

the figure to find the measures of the missing
It takes 2970 bushels of grain to cover the Corn
sides of the bases.
Palace.
7
27. The actual amount needed will be higher because
the building is not a perfect rectangular prism.
28. Use the Pythagorean Theorem to find the missing
c measure.
h h c2 ¬a2  b2
c2 ¬22  (7  6)2
c2 ¬4  1
60 c ¬5
Find the area of the trapezoids.
4
A ¬1
2 h(b1  b2)

11
¬1
2 (2)(6  7)

tan 60 ¬h
4
¬13
4tan60 ¬h
Surface area  2(13)  62  6  5
43 ¬h
 26  36  65 
c2 ¬a2  b2
 75.4
c2 ¬42  (43)2
The surface area of the glass is approximately
c ¬16  48
2
75.4 ft2.
c2 ¬64
c ¬8 29. Use the Pythagorean Theorem to find the
Find the surface area. measures of the third sides of the triangular
T ¬Ph  2B bases.
Prism A:
¬(7  8  11  43)(10)  2  1
 )(7  11)
2 (43 c2 ¬a2  b2
¬454.0 c2 ¬32  42
The surface area is approximately 454.0 units2. c2 ¬9  16
22. L  Ph c2 ¬25
 (15  15  15  15)(10) c ¬5
 600
No, the walls are 600 ft2; 1.5 gallons will only be
enough for 1 coat.

421 Chapter 12
Prism B: So, the ratios of the surface areas of the prisms
c2 ¬a2  b2 are A : B  1 : 4, B : C  30 : 11, and
c2 ¬62  82 A : C  15 : 22.
c2 ¬36  64 33. A and B, because the heights of A and B are in
c2 ¬100 the same ratio as perimeters of bases.
c ¬10 34. Surface area of TV  Ph  2B
Prism C:  (2  20  2  30)(84)
c2 ¬a2  b2  2(20  30)
52 ¬a2  32  8400  1200
25 ¬a2  9  9600 cm2
16 ¬a2 Surface area of VCR  Ph  2B
4 ¬a  (100)(76)  2(600)
The base of Prism A  the base of Prism C  7600  1200
because of the SSS Postulate.  8800 cm2
The sides of the bases of Prism B are proportional Surface area of CD  Ph  2B
to the sides of the bases of Prisms A and C so  (100)(60)  2(600)
base of A  base of B and base of C  base of B.  6000  1200
30. Using the side lengths calculated in Exercise 29,  7200 cm2
the perimeters of the bases are as follows. Surface area of video game system
Prism A: P  3  4  5  12  Ph  2B
Prism B: P  6  8  10  24  (100)(39)  2(600)
Prism C: P  3  4  5  12  3900  1200
So, the ratios of the perimeters of the bases are  5100 cm2
A : B  1 : 2, B : C  2 : 1, and A : C  1 : 1. Surface area of DVD  Ph  2B
31. Prism A:  (100)(35)  2(600)
B ¬1  3500  1200
2 bh
 4700 cm2
¬1

2 (3)(4) 35. No, the surface area of the finished product will
¬6 be the sum of the lateral areas of each prism plus
Prism B: the area of the bases of the TV and DVD prisms.
B ¬1
2 bh It will also include the area of the overhang
¬1

2 (6)(8)
between each prism, but not the area of the
overlapping prisms.
¬24
Prism C: 36. Area of ends  10(20  30)
Use the Pythagorean Theorem to find the  6000
measure of the third side of the triangular base. Area of top  (30)(84)
c2 ¬a2  b2  2520
52 ¬a2  32 Area of bottom  (30)(35)
25 ¬a2  9  1050
16 ¬a2 Area of sides  2(20)(84  76  60  39  35)
4 ¬a  11,760
Area of overhangs  30[(84  76)  (76  60)
B ¬1

2 bh  (60  39)  (39  35)]
¬1

2 (3)(4)
 1470
¬6 Total surface area
So, the ratios of the areas of the bases of the prisms  6000  2520  1050  11,760  1470
are A : B  1 : 4, B : C  4 : 1, and A : C  1 : 1.  22,800
The total surface area of the finished model is
32. Using the values of the perimeters calculated in
22,800 cm2.
Exercise 30, the surface areas are as follows.
Prism A: 37. L ¬Ph  144
T ¬Ph  2B  ¬3w
h ¬2w
¬(12)(6.5)  2  1
2 34

Find the perimeter in terms of h.
¬90 P ¬2  2w
Prism B: ¬2(3w)  2w
T ¬Ph  2B ¬8w
¬(24)(13)  2  1 ¬8h
2 68 2

¬360 ¬4h
Prism C:
T ¬Ph  2B
¬(12)(10)  2  1
2 34

¬132

Chapter 12 422
Find h.
144 ¬Ph
144 ¬(4h)h 8
144 ¬4h2 10 8
36 ¬h2 6
6 ¬h
So, P  (4)(6)  24, w  6
2  3, and   (3)(3)  9.
 12 12
T  Ph  2B
 (24)(6)  2(3)(9)
6
 144  54 10 8
 198 8
The surface area is 198 cm2.
38. Sample answer: Brick masons use the
measurements of the structure and the T  Ph  2B
measurements of the bricks to find the number of
bricks that will be needed. Answers should T  (6  8  10)(12)  2  1
2 68

include the following. T  288  48
• The lateral area is important because the sides T  336
of the brick will show. Also, depending on the The surface area of the triangular prism is
project, only the lateral area of the structure 336 units2.
may be covered with brick. 45.
• It is important to overestimate the number of
bricks ordered in case some are damaged or the
calculations were inaccurate.
39. B; 121.5  6s2 where s is the length of each edge.
121.5
s2 ¬ 
6
s ¬
121.5
 
6
s ¬4.5 m
a 
2 16 (a  4)(a  4) a 
4
4a  16 
40. D;   4 ,a
4
 
4(a  4)
41. L  Ph
L  (2  16  2  20)(18)
L  1296 cm2
T  L  2B
T  1296  2(20)(15) T  Ph  2B
T  1896 cm2 T  (2  3  2  4)(6)  2(3)(4)
42. L  Ph T  84  24
L  (1  4  4.6)(3) T  108
L  28.8 cm2 The surface area of the rectangular prism is
T  L  2B 108 units2.
T  28.8  2  1 46.
2  0.8  4.6

T  32.48 cm 2

43. See students’ work.

Page 654 Maintain Your Skills


44. Use the Pythagorean Theorem to find the
measure of the third side of the triangular base.
c2 ¬a2  b2
c2 ¬62  82
T  Ph  2B
c2 ¬36  64
T  (2  4  2  5)(3)  2(4)(5)
c2 ¬100
T  54  40
c ¬10
T  94
The surface area of the rectangular prism is
94 units2.
47.

back view

423 Chapter 12
53. To find this ratio, convert the height of the house
to inches from feet, then divide the height of the
drawing by the height of the house.
height of drawing in inches 5.5 5.5
height of house in inches   1
33  12  396  72
   
1
The scale factor of the drawing is 
72 .
corner view
48. 54. Use a calculator.
A  r2
 (40)2
back view  5026.55 cm2
55. Use a calculator.
A ¬r2
2
¬d
2

2
¬2 
50

¬(25)2
corner view
¬1963.50 in2
49. Since the radius of Q is 24, AQ  QC  24.
56. Use a calculator. 57. Use a calculator.
QB  BC ¬QC
A  r2 A ¬r2
QB ¬QC  BC
 (3.5)2 ¬(82)2
QB ¬24  5
 38.48 ft2 ¬21,124.07 mm2
QB ¬19
AB  AQ  QB
AB  24  19
AB  43
12-4 Surface Areas of Cylinders
50. Since the radius of Q is 24, AC  48.
Since the radius of R is 16, RD  BR  16.
AD  AC  CR  RD Page 657 Check for Understanding
AD  48  CR  16 1. Multiply the circumference of the base by the
AD  64  CR height and add the area of each base.
Find CR.
2. Sample answer:
BC  CR ¬BR
CR ¬BR  BC
CR ¬16  5
CR ¬11
So, AD  64  11  75.
51. Since the radius of Q is 24, QC  24.
Since the radius of R is 16, BR  16.
QB  BC ¬QC 3. Jamie; since the cylinder has one base, the surface
QB ¬QC  BC area will be the sum of the lateral area and one
QB ¬24  5 base.
QB ¬19 4. Use a calculator.
BC  CR ¬BR T  2rh  2r2
CR ¬BR  BC  2(4)(6)  2(4)2
CR ¬16  5
 251.3
CR ¬11
The surface area is approximately 251.3 ft2.
QR  QB  BC  CR
QR  19  5  11 5. Use a calculator.
QR  35 T ¬2rh  2r2
2
52. Let x be the distance climbed by the airplane as it ¬2d
2h  22
d

flies (horizontally) 50 miles. A right triangle is


¬dh  1
2 d
 2
formed by x (leg opposite), the 50-mile horizontal
distance (leg adjacent), and the path of the ¬(22)(11)  1
2 (22)
 2

airplane (hypotenuse). Find x. ¬1520.5


leg opposite
tan3.5 ¬ 
leg adjacent The surface area is approximately 1520.5 m2.
x
tan3.5 ¬
50
50tan3.5 ¬x
Use a calculator to find x.
Keystrokes: 50 TAN 3.5 ENTER
x  3.1 mi
The total height above sea level is approximately
3  3.1  6.1 mi.

Chapter 12 424
6. Use the formula for surface area to write and 12. Use a calculator.
solve an equation for the radius. T  2rh  2r2
T ¬2rh  2r2  2(14)(14)  2(14)2
96 ¬2r(8)  2r2  2463.0
96 ¬16r  2r2 The surface area is approximately 2463.0 mm2.
48 ¬8r  r2 13. Use a calculator.
0 ¬r2  8r  48 T  2rh  2r2
0 ¬(r  12)(r  4)  2(4)(6)  2(4)2
r ¬4 or 12  251.3
Since the radius of a circle cannot have a negative The surface area is approximately 251.3 ft2.
value, 12 is eliminated. So, the radius of the 14. Use a calculator.
base is 4 cm. T ¬2rh  2r2
7. Use the formula for surface area to write and 2
¬2d
2 h  2 2 
 d
solve an equation for the radius.
T ¬2rh  2r2 ¬dh  1
2 d
 2
140 ¬2r(9)  2r2
140 ¬18r  2r2 ¬(8.2)(7.2)  1
2 (8.2)
 2

70 ¬9r  r2 ¬291.1
0 ¬r2  9r  70 The surface area is approximately 291.1 yd2.
0 ¬(r  14)(r  5) 15. Use a calculator.
r ¬5 or 14 T  2rh  2r2
Since the radius of a circle cannot have a negative  2(0.9)(4.4)  2(0.9)2
value, 14 is eliminated. So, the radius of the  30.0
base is 5 ft. The surface area is approximately 30.0 cm2.
8. Find the area of one label. 16. Use a calculator.
L ¬2rh T ¬2rh  2r2
¬2d
2 h
2
 ¬2d
2h  22
d

¬dh ¬dh  1
2 d
 2
¬(2.5)(4)
¬10 ¬(9.6)(3.4)  1
2 (9.6)
 2

Find the area of 3258 labels. Use a calculator. ¬247.3


3258L  3258(10) The surface area is approximately 247.3 m2.
 102,353.1
17. Use the formula for surface area to write and
The total area is approximately 102,353.1 in2.
solve an equation for the radius.
T ¬2rh  2r2
48 ¬2r(5)  2r2
Pages 657–658 Practice and Apply 48 ¬10r  2r2
9. Use a calculator. 24 ¬5r  r2
T  2rh  2r2 0 ¬r2  5r  24
 2(13)(15.8)  2(13)2 0 ¬(r  8)(r  3)
 2352.4 r ¬3 or 8
The surface area is approximately 2352.4 m2. Since the radius of a circle cannot have a negative
10. Use a calculator. value, 8 is eliminated. So, the radius of the base
T ¬2rh  2r2 is 3 cm.
2
¬2d
2h  22
d 18. Use the formula for surface area to write and
solve an equation for the radius.
¬dh  1
2 d
 2 T ¬2rh  2r2
¬(13.6)(1.9)  1 340 ¬2r(7)  2r2
2 (13.6)
 2
340 ¬14r  2r2
¬371.7 170 ¬7r  r2
The surface area is approximately 371.7 ft2. 0 ¬r2  7r  170
11. Use a calculator. 0 ¬(r  17)(r  10)
T ¬2rh  2r2 r ¬10 or 17
2
¬2d
2h  22
d Since the radius of a circle cannot have a negative
value, 17 is eliminated. So, the radius of the
¬dh  1
2 d
 2 base is 10 in.
¬(14.2)(4.5)  1
2 (14.2)
 2

¬517.5
The surface area is approximately 517.5 in2.

425 Chapter 12
19. Use the formula for surface area to write and 24. L ¬2rh
solve an equation for the radius. ¬2d
2h
T ¬2rh  2r2
320 ¬2r(12)  2r2 ¬dh
320 ¬24r  2r2 ¬(5)(13)
160 ¬12r  r2 ¬204.2
0 ¬r2  12r  160 The lateral area of the silo is approximately
0 ¬(r  20)(r  8) 204.2 m2.
r ¬8 or 20 25. From Exercise 24, L  65. Use the formula for
Since the radius of a circle cannot have a negative lateral area to write and solve an equation for
value, 20 is eliminated. So, the radius of the the radius.
base is 8 m. L ¬2rh
20. Use the formula for surface area to write and 65 ¬2r(26)
solve an equation for the radius. 1.25 ¬r
T ¬2rh  2r2 The radius of the silo is 1.25 m.
425.1 ¬2r(6.8)  2r2 26. x
425.1 ¬13.6r  2r2
0 ¬2r2  13.6r  425.1
Use the quadratic formula to find r.
a ¬2 x–
2
b ¬13.6
c ¬425.1 Let the diameter of the circle be x. A regular
b ac
b 4
2
r ¬ 2a
hexagon can be separated into 6 congruent
13.6 (13.6
) 425.1
2  4(2)() nonoverlapping equilateral triangles. The sides of
¬  2(2) each triangle are 2x. The perimeter of the hexagon
¬5.5 or 12.3 is 3x. The lateral area of the hexagonal pencil is
Since the radius of a circle cannot have a negative 33x. The radius of the circle is also 2x. The
value, 12.3 is eliminated. So, the radius of the
circumference of the circle is 22x or x. The
base is approximately 5.5 ft.
lateral area is approximately 34.6x square inches.
21. Since L  2rh, the lateral areas will be in the
The cylindrical pencil has the greater surface
ratio 3 : 2 : 1.
area.
L1 ¬25
2 (9)

27. Sample answer: Extreme sports participants use
¬141.4 a semicylinder for a ramp. Answers should
L2 ¬25
2 (6)
 include the following.
• To find the lateral area of a semicylinder like
¬94.2
the half-pipe, multiply the height by the
L3 ¬25
2 (3)
 circumference of the base and then divide by 2.
¬47.1 • A half-pipe ramp is half of a cylinder if the
The lateral areas are approximately 141.4 in2, ramp is an equal distance from the axis of the
94.2 in2, and 47.1 in2. cylinder.
22. To find the amount of aluminum foil needed to 28. B; T ¬2rh  2r2
2
cover the inside of the reflector, divide the ¬2d
2h  22
d
formula for surface area of a cylinder by 2.
2rh  2r 2 ¬dh  1
2 d
 2
T ¬ 2
¬rh  r2 ¬(8.2)(13.4)  1
2 (8.2)
 2

¬d
2
¬450.8 cm2
2h  2
d
29. C; let x be the number of adult tickets sold. Then
¬1 1 2
2 dh  4 d
 200  x is the number of student tickets sold.
   
2
¬1 1 1 1 total sales ¬adult sales  student sales
2  5 2 (18)  4  5 2

500 ¬5x  2(200  x)
¬179.3 500 ¬5x  400  2x
Approximately 179.3 in2 of aluminum foil is 100 ¬3x
needed. 33.3 ¬x
23. T  L  2B Since the total sales were more than $500, the
 2rh  2r2 minimum number of adult tickets sold was 34.
Triple the height. 30. The locus of points 5 units from a given line is a
T  2r(3h)  2r2 cylinder with a radius of 5 units.
 3(2rh)  2r2
 3L  2B
The lateral area is tripled. The surface area is 5
increased, but not tripled.

Chapter 12 426
31. The locus of points equidistant from two opposite 39. Use the Pythagorean Theorem to find x.
vertices of a face of a cube is a plane c2 ¬a2  b2
perpendicular to the line containing the opposite (5  x)2 ¬52  122
vertices of the face of the cube. 25  10x  x2 ¬25  144
1 unit x2  10x  144 ¬0
(x  18)(x  8) ¬0
8 or 18 ¬x
Since the length of a segment cannot be negative,
18 is eliminated. So, x  8.
40. Use the Law of Cosines since the measures of two
sides and the included angle are known.
Page 659 Maintain Your Skills a2 ¬b2  c2  2bccosA
32. L(8 15 base)  Ph a2 ¬6.32  7.12  2(6.3)(7.1)cos54
 (2  8  2  15)(6) a2 ¬90.1  89.46cos54
 276 a ¬90.1  89.46cos54
L(6 15 base)  (2  6  2  15)(8) a ¬6.1
 336 Use the Law of Sines to find mB and mC.
L(8 6 base)  (2  8  2  6)(15) sinB sinA
b ¬ a
   
 420 b
The lateral areas are 276 units2 (8 15 base), sinB ¬asinA
336 units2 (6 15 base), and 420 units2 B ¬sin1ab
sinA
(8 6 base).
B ¬sin1 25 sin54
6.3
6.1
33. Use the Pythagorean Theorem to find the
measure of the third side of the triangular base. B ¬56.3
c2 ¬a2  b2 C
sin sinA
c ¬a

c2 ¬52  122
c2 ¬25  144 C ¬sin1acsinA
c2 ¬169 C ¬sin1
6.125 sin54
7.1

c ¬13
L  Ph C ¬69.7
 (5  12  13)(10) So, mB  56.3, mC  69.7, and a  6.1.
 300 41. Use the Angle Sum Theorem to find mA.
The lateral area is 300 units2. mA  mB  mC ¬180
mA  47  69 ¬180
34. L(8 18 base)  Ph
mA ¬64
 (2  8  2  18)(6)
Use the Law of Sines to find b and c.
 312
sinA sinB
a ¬b
L(6 18 base)  (2  6  2  18)(8) 
 384 asi
b ¬ nB
sinA
L(8 6 base)  (2  8  2  6)(18)
15sin47
 504 b ¬ 
sin64
The lateral areas are 312 units2 (8 18 base), b ¬12.2
384 units2 (6 18 base), and 504 units2 sinA sinC
a ¬c

(8 6 base).
asinC
35. 36. c ¬sinA
15sin 69
c ¬ 
sin64
c ¬15.6
So, mA  64, b  12.2, and c  15.6.
42. A ¬1

2 bh
37. According to Theorem 10.11 on page 554, two
¬1

2 (20)(17)
segments that originate from the same exterior
point and are tangent to a circle are congruent. ¬170
So, x  27. The area of the triangle is 170 in2.
38. Use the Pythagorean Theorem to find x. 43. A ¬1
2 h(b1  b2)

c2 ¬a2  b2 ¬1
2 (6)(7  11)

(4  6)2 ¬x2  62
100 ¬x2  36 ¬54
The area of the trapezoid is 54 cm2.
64 ¬x2
8 ¬x 44. A ¬1

2 bh
¬1

2 (38)(13)
¬247
The area of the triangle is 247 mm2.

427 Chapter 12
Page 659 Practice Quiz 1 2. A regular pyramid is only a regular polyhedron if
1. 2. all of the faces including the base are congruent
regular polygons. Since the faces of a pyramid are
triangles, the only regular pyramid that is also a
regular polyhedron is a tetrahedron.
3. The slant height  is the hypotenuse of a right
corner view triangle with legs that are the altitude and a
3. Use the Pythagorean Theorem to find the segment with a length that is one-half the side
measure of the third side of the triangular base. measure of the base. Use the Pythagorean
c2 ¬a2  b2 Theorem to find the slant height of the regular
82 ¬a2  62 pyramid.
64 ¬a2  36 c2 ¬a2  b2
28 ¬a2 2 ¬22  72
28
 ¬a  ¬53
27  ¬a Find the surface area.
L  Ph T ¬1
2 P  B

 (6  8  27 )(12)
 231.5 ¬1
2 (4  4  4  4)53
   42
The lateral area of the prism is approximately ¬74.2
231.5 m2. The surface area of the regular pyramid is
4. From Question 3, L  (6  8  2 7)(12) approximately 74.2 ft2.
T ¬L  2B 4. The altitude, slant height, and apothem form a
¬Ph  2B right triangle. Use the Pythagorean Theorem to
)(12)  2  1
¬(6  8  27 2  6  27
  find the length of the apothem. Let x represent
the length of the apothem.
¬263.2 c2 ¬a2  b2
The surface area of the prism is approximately (32)2 ¬x2  32
263.2 m2. 18 ¬x2  9
5. Use the formula for surface area to write and 9 ¬x2
solve an equation for the radius. 3 ¬x
T ¬2rh  2r2 The side measure of the base is 2x  2(3)  6.
560 ¬2r(11)  2r2 Find the surface area.
280
 ¬11r  r T ¬1
2
2 P  B

280
0 ¬r2  11r   ¬1
2 (6  6  6  6)(32
)  62

Use the quadratic formula to find r. ¬86.9
a ¬1 The surface area of the regular pyramid is
b ¬11 approximately 86.9 cm2.
280
c ¬  5. The slant height  is the hypotenuse of a right
b ac
b 4
2
r ¬ 2a
triangle with legs that are a lateral edge and a
segment with a length that is one-half the side
11
¬ 
2(1)
11
 2

4(1)
280
measure of the base. Use the Pythagorean
Theorem to find the measure of the slant height.
¬5.4 or 16.4 c2 ¬a2  b2
2
Since the radius of a circle cannot have a negative 132 ¬2  2 
10
value, 16.4 is eliminated. So, the radius of the
169 ¬2  25
base is approximately 5.4 ft.
144 ¬2
12 ¬
Find the surface area.
12-5 Surface Areas of Pyramids T ¬1
2 P  B


¬1
2 (10  10  10  10)(12)  10
 2

Page 663 Check for Understanding ¬340


1. Sample answer: The surface area of the regular pyramid is
340 cm2.
6. To find the amount of paper used for one pyramid,
find the lateral area of the pyramid.
L ¬1

2 P
square base rectangular base ¬1
2 (2  2  2  2)(4)

(regular) (not regular)
¬16

Chapter 12 428
Find the total amount of paper used given that 10. Use the Pythagorean Theorem to find the length
there are 6 pyramids per star. of the apothem. Let x represent the length of the
6L  6(16) apothem.
 96 c2 ¬a2  b2
The amount of paper used is 16 in2 per pyramid 92 ¬x2  62
and 96 in2 per star. 81 ¬x2  36
45 ¬x2
35  ¬x
Pages 663–665 Practice and Apply The side measure of the base is 2x  2(35) 
65 . Find the surface area.
7. T ¬1
2 P  B

T ¬1
2 P  B

¬1
2 (7  7  7  7)(5)  7
 2
¬1
2 (4  65
 )(9)  (65
)2
¬119
The surface area of the regular pyramid is ¬421.5
119 cm2. The surface area of the regular pyramid is
approximately 421.5 cm2.
8. Find the measure of the apothem of the base, x.
36
0 11. Use the Pythagorean Theorem to find the slant
The central angle of the hexagon is 6  60. So, height .
the angle formed by a radius and the apothem is c2 ¬a2  b2
60
2  30.
  2
82 ¬6
2  
 2

64 ¬9  2
55 ¬2
55 ¬
Find the measure of the apothem of the base, x.
36
0
The central angle of the pentagon is 5  72.
30 x
So, the angle formed by a radius and the apothem
72
is 2  36.


2.25 in.
2.25
tan30 ¬ x

2.25
x ¬
tan30
Find the surface area.
36 x
T ¬1 1
2 P  2 Px


¬1
2 P(  x)

3 yd
2 (6  4.5)6  tan30 
¬1
 2.2
5
tan36 ¬3x
¬133.6
3
The surface area of the regular pyramid is x ¬
tan36
approximately 133.6 in2. Find the surface area.
9. Use the Pythagorean Theorem to find the height T ¬1 1
2 P  2 Px

of the triangular base.
c2 ¬a2  b2 ¬1
2 P(  x)

2
82 ¬a2  8
2 2 (5  6)55
¬1 36 
 3
 tan
64 ¬a2 16 ¬173.2
48 ¬a2 The surface area of the regular pyramid is
43 ¬a approximately 173.2 yd2.
Find the surface area. 12. Use the Pythagorean Theorem to find the slant
T ¬1 height .
2 P  B

c2 ¬a2  b2
¬1
 1
2 (3  8)(10)  2 (8)(43
) 2
6.42 ¬2  
3.2
 2
¬147.7
The surface area of the regular pyramid is 40.96 ¬2.56  2
approximately 147.7 ft2. 38.4 ¬2
38.4
 ¬
Find the measure of the apothem of the base, x.
36
0
The central angle of the hexagon is 6  60. So,
the angle formed by a radius and the apothem is
60
2  30.
 

429 Chapter 12
Find the altitude of the triangular base.
c2 ¬a2  b2
122 ¬a2  62
144 ¬a2  36
108 ¬a2
63  ¬a
30 x Find the surface area.
T ¬1
2 P  B

1.6 m ¬1
 )  1
2 (3  12)(27
 )
2 (12)(63
1.6
tan30 ¬x

¬157.6
1.
6
x ¬
tan30
The surface area of the regular pyramid is
approximately 157.6 cm2.
x ¬1.63
Find the surface area. 15. Use the Pythagorean Theorem to find the slant
height .
T ¬1 1
2 P  2 Px

c2 ¬a2  b2
¬1
2
2 P(  x)
 42 ¬2  4
2

¬1
2 (6  3.2)(38.4
   1.63
) 16 ¬2  4
¬86.1 12 ¬2
The surface area of the regular pyramid is 23  ¬
approximately 86.1 m2.  is also the altitude of the triangular base. Find
the surface area.
13. Use the Pythagorean Theorem to find the
apothem, x. T ¬1
2 P  B

c2 ¬a2  b2 ¬1 )  1
2 (3  4)(23 )
 
2 (4)(23
132 ¬122  x2
169 ¬144  x2 ¬27.7
25 ¬x2 The surface area of the regular pyramid is
5 ¬x approximately 27.7 ft2.
Find the side measure of the base, y. The central 16. Use the Pythagorean Theorem to find the
36
0 apothem, x.
angle of the pentagon is 5  72. So, the angle c2 ¬a2  b2
formed by the radius and the apothem is
72 202 ¬52  x2
2  36.
  400 ¬25  x2
375 ¬x2
 ¬x
515
The side measure of the base is twice the
apothem: 2x  1015 .
Find the lateral area of the roof.
36 5 in.
L ¬1

2 P
¬1
2 (4  1015
 )(20)
y
¬1549.2
2 The area of the roof is approximately 1549.2 ft2.
y
2
tan36 ¬ 
5
17. Find the surface area of the first bottle.
10 tan36 ¬y T ¬1
2 P  B

Find the surface area. ¬1
2 (4  3)(4)  3
 2

T ¬1 1
2 P  2 Px
 ¬33
¬1 Find the dimensions of the base of the second
2 P(  x)

bottle. Let the side measure be x.
1
¬2(5  10 tan36)(13  5) T ¬1
2 P  B

¬326.9
T ¬1
2 (4x)(6)  x
 2
The surface area of the regular pyramid is
approximately 326.9 in2. 33 ¬12x  x2
14. Use the Pythagorean Theorem to find the slant 0 ¬x2  12x  33
height . Use the quadratic formula to find x.
c2 ¬a2  b2 a ¬1
2 b ¬12
82 ¬2  2 
12

c ¬33
64 ¬  36
2

28 ¬2
27 ¬

Chapter 12 430
b b
x ¬  ac
2 4 1 P
L ¬\ 
2a 2
12 12
¬   33)
4(1)(
2
¬1 107
.25
2 (4  214.5) cos53
 
2(1)
¬76,452.5
¬2.3 or 14.3
The lateral area is approximately 76,452.5 m2.
The length of the side cannot be negative, so
14.3 is eliminated. The base of the second bottle 21. Find the height of the pyramid using the
is approximately 2.3 inches on each side. Pythagorean Theorem. The apothem of the
pyramid is half the length of its base, which is
18. Find the side measure of the base. 12
 360,0
00  600 12 ft:  2  6.
So, P  4(600)  2400 and the apothem is c2 ¬a2  b2
600 102 ¬a2  62
2  300.

100 ¬a2  36
Use the Pythagorean Theorem to find the slant
64 ¬a2
height .
8 ¬a
c2 ¬a2  b2
The height of the solid is 12  8  20 ft.
2 ¬3002  3212
2 ¬193,041 22. Add the lateral areas of the pyramid and the
 ¬193,0
41  cube.
Find the lateral area. L ¬1
2 P  Ph

L ¬1
 ¬1
2 P 2 (4  12)(10)  (4  12)(12)

¬1
 41
2 (2400)193,0 ¬816
The lateral area of the solid is 816 ft2.
¬527,237.2
The lateral area of the pyramid is approximately 23. The surface area of the solid is equal to the
527,237.2 ft2. lateral areas of the pyramid and the cube plus the
64
6 area of the cube’s base.
19. The apothem is half the length of the edge:  2 
T ¬1
2 P  Ph  B

323. Use the Pythagorean Theorem to find the
slant height . ¬1
2 (4  12)(10)  (4  12)(12)  12
 2

c2 ¬a2  b2 ¬960
2 ¬3232  3502 The surface area of the solid is 960 ft2.
2 ¬226,829
24. Each lateral face of the frustum is a trapezoid.
 ¬226,8
29 
Find the area of one face.
Find the lateral area to find the area of the glass.
A ¬1
2 h(b1  b2)

L ¬1

2 P
¬1
2 (b1  b2)

¬1
2 (4  646)226,8
 29 
¬615,335.3 ¬1
2 (3)(2  4)

The area of the glass is approximately ¬9
615,335.3 ft2. The lateral area is 4A  4(9)  36 yd2.
20. The apothem is half the length of the side of the 25. Find the surface area of the truncated cube. The
214.5 area of the three intact faces is 3 in2. The
base:  2  107.25.

truncation cuts three faces in half, leaving three
triangles with a total area of 1.5 in2. The final
face is an equilateral triangle. Use the
 Pythagorean Theorem to find the side measure of
the triangle.
53 c2 ¬a2  b2
c2 ¬12  12
c2 ¬2
c ¬2 
Find the altitude of the triangle.
107.25
c2 ¬a2  b2
2

Find the slant height. )2 ¬a2  
(2 2 
2
107
.25
cos53 ¬  2 ¬a2  1

2
107
.25 1.5 ¬a2
 ¬
cos53
Find the lateral area. 1.5
 ¬a
Find the area of the triangle.
A ¬1

2 bh
¬1
 )1.5
2 (2 

3
¬2

431 Chapter 12

3 A ¬bh
The surface area is 3  1.5   2  5.37 in . The
2

153

surface area of the original cube is 6 square ¬(22) 2 

inches. The surface area of the truncated cube is
¬285.8
approximately 5.37 square inches. Truncating the
The perimeter is 74 ft and the area is
corner of the cube reduces the surface area by
approximately 285.8 ft2.
0.63 square inch.
34. P  24  2(32)  10  2(5)  6  (24  10  6)
26. Sample answer: Pyramids are used as an
 122
alternative to rectangular prisms for the shapes
A  (32)(24)  (5)(24 10  6)
of buildings. Answers should include the
 808
following.
The perimeter is 122 m and the area is 808 m2.
• We need to know the dimensions of the base
and slant height to find the lateral area and 35. P  17  12  (22  17)  9  22
surface area of a pyramid.  (12  9  3  6)  3(6)  3
• Sample answer: The roof of a gazebo is often a  98
hexagonal pyramid. A  (22)(12  9)  (12)(22  17)  62
 366
27. D; T ¬1
2 P  B
 The perimeter is 98 m and the area is 366 m2.
2
2 (20)(10)   4 
¬1
 2
 0 36. M
F  is reflected in line b, but F
M
 lies on line b. So,
¬125 cm2 the reflected image of FM is F
M.
1 37. The reflected image of J K
 in line a is G
F.
28. A; x ⊗ y ¬
xy 38. The reflected image of L in point M is point H.
1 ⊗ 3 ¬  1
39. The reflected image of G M in line a is J
M
.
2 4
1  3
2 4
1 40. False; each pair of opposite sides must be
¬  congruent.
2  3
4 4
1 41. True; each pair of opposite sides is congruent.
¬ 
1 
4
42. c2 ¬a2  b2
¬4 122 ¬82  b2
144 ¬64  b2
80 ¬b2
Page 665 Maintain Your Skills 45  ¬b
8.9 ¬b
29. T  2rh  2r2
The length is approximately 8.9 in.
 2(7)(15)  2(7)2
 967.6 43. c2 ¬a2  b2
The surface area is approximately 967.6 m2. c2 ¬142  162
c2 ¬196  256
30. T ¬2rh  2r2
2 c2 ¬452
¬2d
2h  22
d
c ¬2113

¬dh  1 c ¬21.3
2 d
 2
The length is approximately 21.3 m.
¬(22)(14)  1
2 (22)
 2
44. c2 ¬a2  b2
¬1727.9 112 ¬a2  62
The surface area is approximately 1727.9 cm2. 121 ¬a2  36
31. T  2rh  2r2 85 ¬a2
 2(9)(23)  2(9)2 85 ¬a
 1809.6 9.2 ¬a
The surface area is approximately 1809.6 yd2. The length is approximately 9.2 km.
32. T  L  2B
 Ph  2B
 (2  6  2  2.5)(14)  2(6)(2.5)
 268
12-6 Surface Areas of Cones
The surface area of the box is 268 in2.
33. P  2(22)  2(15) Page 668 Check for Understanding
 44  30 1. Sample answer:
 74 vertex
Find the height.
h ¬sin60 center of base
15
h ¬15sin60
153
h ¬ 
2

Chapter 12 432
2. The formula for the lateral area is derived from 9. Use the Pythagorean Theorem to find the slant
the area of a sector of a circle. If the vertex of the height.
cone is not the center of this circle, the formula is c2 ¬a2  b2
not valid. 2 ¬92  92
3. T  r  r2 2 ¬162
 (10)(17)  (10)2  ¬92 
 848.2 T  r  r2
The surface area is approximately 848.2 cm2.  (9)(92 )  (9)2
4. Use the Pythagorean Theorem to find the slant  614.3
height. The surface area is approximately 614.3 in2.
c2 ¬a2  b2 10. Use the Pythagorean Theorem to find the radius.
2 ¬122  102 c2 ¬a2  b2
2 ¬144  100 172 ¬r2  152
2 ¬244 289 ¬r2  225
 ¬261 64 ¬r2
T  r  r2 8 ¬r
 (10)(261)  (10)2 T  r  r2
 804.9  (8)(17)  (8)2
The surface area is approximately 804.9 ft2.  628.3
5. Use the Pythagorean Theorem to find the slant The surface area is approximately 628.3 ft2.
height. 11. Use the Pythagorean Theorem to find the radius.
c2 ¬a2  b2 c2 ¬a2  b2
2 ¬82  82 122 ¬r2  7.52
2 ¬128 144 ¬r2  56.25
 ¬82  87.75 ¬r2
T  r  r2 87.75
 ¬r
 (8)(82)  (8)2 T  r  r2
 485.4  (87.75
)(12)  (87.75 )2
The surface area is approximately 485.4 in2.  628.8
6. Use the Pythagorean Theorem to find the slant The surface area is approximately 628.8 m2.
height. 12. Use the Pythagorean Theorem to find the slant
c2 ¬a2  b2 height.
2 c2 ¬a2  b2
2 ¬552  2 
8.5

2 ¬2.62  6.42
 ¬3043.
0625 2 ¬47.72
L ¬r  ¬47.72

¬ T  r  r2
2 3043.
8.5
 0625 
 (2.6)47.72
  (2.6)2
¬736.5
 77.7
The lateral area is approximately 736.5 ft2.
The surface area is approximately 77.7 yd2.
13. Use the Pythagorean Theorem to find the radius.
c2 ¬a2  b2
Pages 668–670 Practice and Apply 182 ¬r2  162
7. Use the Pythagorean Theorem to find the slant 324 ¬r2  256
height. 68 ¬r2
c2 ¬a2  b2 217 ¬r
2 ¬52  122 T  r  r2
2 ¬169  (217 )(18)  (217 )2
 ¬13  679.9
T  r  r2 The surface area is approximately 679.9 in2.
 (5)(13)  (5)2
14. Use the Pythagorean Theorem to find the radius.
 282.7
c2 ¬a2  b2
The surface area is approximately 282.7 cm2.
19.12 ¬r2  8.72
8. Use the Pythagorean Theorem to find the radius. 364.81 ¬r2  75.69
c2 ¬a2  b2 289.12 ¬r2
102 ¬r2  82 289.1
2  ¬r
100 ¬r2  64 T  r  r2
36 ¬r2  (289.1
2 )(19.1)  (289.1
2 )2
6 ¬r  1928.6
T  r  r2 The surface area is approximately 1928.6 m2.
 (6)(10)  (6)2
 301.6
The surface area is approximately 301.6 ft2.

433 Chapter 12
15. Solve the surface area equation for the slant T  r  r2  2rh
height.  r(  r  2h)
T ¬r  r2  (6)[213  6  2(6)]
T  r2 ¬r  475.2
T  r2 The surface area is approximately 475.2 in2.

r ¬
1020   (14.5)2 20. T  r  r2  2rh
 ¬
(14.5)  r(  r  2h)
7.9 ¬  (3)[5  3  2(5)]
The slant height is approximately 7.9 m.  169.6
16. Solve the surface area equation for the slant The surface area is approximately 169.6 ft2.
height. 21. Use the Pythagorean Theorem to find the slant
T ¬r  r2 height of the cone.
T  r2 ¬r c2 ¬a2  b2
T 
 r2
¬ 2 ¬142  6.22
r  ¬234.4
4 
293.2
 ¬ (6.1) 2
(6.1) T  r  2rh  r2
9.2 ¬  r(  2h  r)
The slant height is approximately 9.2 ft.  (6.2)[234.4
4   2(28)  6.2]
 1509.8
17. Use the surface area equation to solve for the
The surface area is approximately 1509.8 m2.
radius.
T ¬r  r2 22. T ¬r
359 ¬r(15)  r2 ¬d
2
359
0 ¬r2  15r   ¬1
2 (42)(47.9)

Use the Quadratic Formula to solve for r. ¬3160.1
a ¬1
The area of the canvas used is approximately
b ¬15
3160.1 ft2.
359
c ¬ 23. Find the radius.
b ac
b 4
2
C ¬2r
r ¬ 2a
C ¬r
¬ 

15 152  4(1) 
359
 2
22
2 ¬r
2(1)  
¬5.6 or 20.6 11
 ¬r
Since the radius of a circle cannot be negative,
Use the Pythagorean Theorem to find the slant
20.6 is eliminated. So, the radius of the cone is
height.
approximately 5.6 ft.
c2 ¬a2  b2
18. Use the surface area equation to solve for the 2
radius. 2 ¬  18
11 2

T ¬r  r2
523 ¬r(12.1)  r2 
 ¬ 

12
1
2  324

523 Find the lateral area of all eight hats.


0 ¬r2  12.1r   8L  8r
Use the Quadratic Formula to solve for r.
a ¬1 ¬8 
11 12

1

2  324

b ¬12.1 ¬1613.7
523
c ¬ She will use approximately 1613.7 in2 of material.

24. Use the Pythagorean Theorem to find the slant
b ac
b 4
2
r ¬ 2a height.
c2 ¬a2  b2
12.1 12.1
 24(1)
 
52
3
2
¬ 
2(1) 2 ¬242  
4
5
 2
¬8.2 or 20.3  ¬1082.
25 
Since the radius of a circle cannot be negative, L ¬r
20.3 is eliminated. So, the radius of the cone is
¬
4
 25
5 1082.
approximately 8.2 m. 2
19. Use the Pythagorean Theorem to find the slant ¬2325.4
height of the cone. The lateral area is approximately 2325.4 ft2.
c2 ¬a2  b2
2 ¬42  62
2 ¬16  36
 ¬213


Chapter 12 434
25. Use the equation for surface area to solve for the 29. Using the store feature on the calculator is the
diameter. most accurate technique to find the lateral area.
T ¬r  r2 Rounding the slant height to either the tenths
2
T ¬d
2    2 
 d place or hundredths place changes the value of
the slant height, which affects the final
T ¬1 1 2
2 d  4 d
 computation of the lateral area.
500 ¬1 1 2
2 d(20)  4 d
 30. Never; the pyramid could be inscribed in the cone.
2000 31. Sometimes; only when the heights have the same
 ¬40d  d
  2
proportion.
2000
0 ¬d2  40d   
 32. As the altitude approaches zero, the slant height
Use the Quadratic Formula to solve for d. of the cone approaches the radius of the base. The
a1 lateral area approaches the area of the base. The
b  40 surface area approaches twice the area of the
2000 base.
c  

b bac
 4
2 33. Sample answer: Tepees are conical shaped
d ¬ 
2a structures. Lateral area is used because the

¬ 

40 402  4(1)
2000 ground may not always be covered in circular
canvas. Answers should include the following.
2(1)
• We need to know the circumference of the base
¬12 or 52
or the radius of the base and the slant height of
Since the diameter of a circle cannot be negative,
the cone.
52 is eliminated. So, the diameter of light on
• The open top reduces the lateral area of canvas
stage is approximately 12 ft.
needed to cover the sides. To find the actual
26. Use the Pythagorean Theorem to find the slant lateral area, subtract the lateral area of the
height. conical opening from the lateral area of the
c2 ¬a2  b2 structure.
2 ¬72  42
34. B; L ¬r
 ¬65

91.5 ¬r(15)
 ¬8.062257748
6.1 ft ¬r
Use the store feature of the calculator to save .
L  r 35. D; let the odd integers be x  4, x  2, and x.
 (4)(8.062257748) 3(x  4) ¬3  2x
 101.3133 3x  12 ¬3  2x
The lateral area of the cone is approximately x ¬15
101.3133 in2.
27. Use the Pythagorean Theorem to find the slant
Page 670 Maintain Your Skills
height.
36. Use the Pythagorean Theorem to find the slant
c2 ¬a2  b2
height. The apothem is half the length of the
2 ¬72  42
base’s side.
 ¬65

c2 ¬a2  b2
 ¬8.1 2
L  r 2  2   853
14
9 2

 (4)(8.1)  ¬733,1
59.25

 101.7876
L ¬1
P
2
The slant height and lateral area of the cone are
approximately 8.1 in. and 101.7876 in2, ¬1
2 (4  149)733,1
 59.25

respectively. ¬255,161.7
28. Use the Pythagorean Theorem to find the slant The lateral area is approximately 255,161.7 ft2.
height. 37. T ¬2rh  2r2
c2 ¬a2  b2 563 ¬2r(9.5)  2r2
2 ¬72  42 56
3
 ¬65
 0 ¬r2  9.5r  
2
 ¬8.06 Use the quadratic formula to solve for r.
L  r a1
 (4)(8.06) b  9.5
 101.2849 56
3
c  2
The slant height and lateral area of the cone are b b
 
2  4ac
approximately 8.06 in. and 101.2849 in2, r ¬ 2a
respectively.
¬ 

9.5 9.52  4(1)
2 
56
3

2(1)
¬5.8 or 15.3

435 Chapter 12
Since the radius of a circle cannot be negative, 44. Since the radius M K is perpendicular to the chord
15.3 is eliminated. So, the radius is G
F, it bisects the chord. So, LG is equal to FL,
approximately 5.8 ft. or 24.
38. T ¬2rh  2r2 45. Since the radius M P is perpendicular to the chord
185 ¬2r(11)  2r2 J
H , it bisects the chord and its arc. So, mPJ is
18
5
0 ¬r2  11r  
2
equal to mHP , or 45.
Use the Quadratic Formula to solve for r. 46. Since the radius M P is perpendicular to the chord
a1 J
H , it bisects the chord and its arc. So, mHJ is
b  11 twice mHP , or 90.
18
5 47. Let x represent the geometric mean.
c  
2
b b 
2  4ac 7
x ¬
x
63
r ¬ 
2a
x2 ¬441
¬ 

11 112  4(1)
2 
18
5
x ¬21
2(1) 48. Let x represent the geometric mean.
¬2.2 or 13.2 8 x
Since the radius of a circle cannot be negative, x ¬
18
13.2 is eliminated. So, the radius is x2 ¬144
approximately 2.2 m. x ¬12
39. T ¬2rh  2r2 49. Let x represent the geometric mean.
470 ¬2r(6.5)  2r2 16 x
x ¬

4
4
47
0
0 ¬r2  6.5r  
2 x2 ¬704
Use the Quadratic Formula to solve for r. x ¬811  26.5
a1 50. C  2r 51. C ¬d
b  6.5  2(6) ¬(8)
47
c   0  37.7 ¬25.1
2
b b 
2  4ac 52. C  d 53. C ¬2r
r ¬ 
2a ¬(18) ¬2(8.2)
6.5
¬ 
6.5

2

4(1)
2 
47
0  56.5
54. C  d
¬51.5
55. C ¬2r
2(1)
 (19.8) ¬2(4.1)
¬6.0 or 12.5
 62.2 ¬25.8
Since the radius of a circle cannot be negative,
12.5 is eliminated. So, the radius is
approximately 6.0 yd.
40. T ¬2rh  2r2 Page 670 Practice Quiz 2
951 ¬2r(14)  2r2 1. Use the Pythagorean Theorem to find the slant
95
1 height. The apothem is equal to half the base’s
0 ¬r2  14r  
2 edge.
Use the Quadratic Formula to solve for r. c2 ¬a2  b2
a1 2 ¬62  102
b  14  ¬234

95
1
c   T ¬1
2 P  B
2 
b b 
2  4ac
r ¬ 
2a ¬1
2 (4  12)(234
 )  122

¬ 

14 142  4(1)
2 
95
1 ¬423.9
The surface area is approximately 423.9 cm2.
2(1)
2. Find the apothem of the base, x. The central angle
¬7.2 or 21.2 36
0
Since the radius of a circle cannot be negative, of the hexagon is 6  60. So, the angle formed
60
by the radius and the apothem is  2  30.
21.2 is eliminated. So, the radius is 
approximately 7.2 cm.
41. Since the radius M K is perpendicular to the chord
G
F, it bisects the chord. So, FG is twice FL, or 48.
42. Since the radius M P is perpendicular to the chord
J
H , it bisects the chord. So, NJ is half of HJ,
or 24. 30 x
43. Since the radius M P is perpendicular to the chord
J
H , it bisects the chord. So, HN is half of HJ, 2 in.
or 24.

Chapter 12 436
tan30 ¬2x 3. Use the Pythagorean Theorem to find AB.
2 AB2 ¬AC2  BC2
x ¬tan 30 AB2 ¬92  122
x ¬23 AB2 ¬81  144
Find the surface area. AB2 ¬225
T ¬1 1 AB ¬15
2 P  2 Px

4. Use the Pythagorean Theorem to find AC.
¬1
2 P(  x)
 AB2 ¬AC2  BC2
¬1 152 ¬AC2  102
2 (6  4)(11  23
)

225 ¬AC2  100
¬173.6 125 ¬AC2
The surface area is approximately 173.6 in2. 11.2 ¬AC
3. Use the Pythagorean Theorem to find the slant 5. If Q is a point on C, then AQ is equal to the
height. radius of the sphere, and thus, AB. So, AQ  18.
c2 ¬a2  b2
6. T  4r2
2 ¬32  122
 4(6.8)2
 ¬317

 581.1
T  r  r2
The surface area is approximately 581.1 in2.
 (3)(317)  (3)2
 144.9 7. Find the radius.
The surface area is approximately 144.9 ft2. C ¬2r
8 ¬2r
4. Use the Pythagorean Theorem to find the slant
4 ¬r
height.
c2 ¬a2  b2 T ¬1
2 (4r )  r
 2 2

2 ¬62  22 ¬2(4)2  (4)2


 ¬210
 ¬150.8
T  r  r2 The surface area is approximately 150.8 cm2.
 (6)(210)  (6)2 8. Find the radius.
 232.3 A ¬r2
The surface area is approximately 232.3 m2. 18.1 ¬r2
5. Use the equation for the lateral area to solve for
the slant height.

18
 .1
 ¬r
L ¬r T ¬4r2
 
2
 ¬L
r
¬4
18
.1

 ¬123 ¬72.4
(1
0) The surface area is approximately 72.4 m2.
 ¬3.9
9. T  4r2
The slant height is approximately 3.9 in.
 4(4.75)2
 283.5
The surface area is approximately 283.5 in2.
12-7 Surface Areas of Spheres
Pages 674–676 Practice and Apply
Page 672 Geometry Activity: Surface Area 10. Use the Pythagorean Theorem to find PR.
of a Sphere PR2 ¬PT 2  RT 2
1. 1 PR2 ¬42  32
4
2. r2 PR2 ¬16  9
PR2 ¬25
3. The surface area of a sphere is 4 times the area of
PR ¬5
the great circle.
11. Use the Pythagorean Theorem to find PR.
PR2 ¬PT 2  RT 2
PR2 ¬32  82
Page 674 Check for Understanding
PR2 ¬9  64
1. Sample answer: PR2 ¬73
PR ¬8.5
12. Use the Pythagorean Theorem to find PT.
PR2 ¬PT 2  RT 2
132 ¬PT 2  122
169 ¬PT 2  144
2. Tim; the surface area of a hemisphere is half of 25 ¬PT 2
the surface area of the sphere plus the area of the 5 ¬PT
great circle.

437 Chapter 12
13. Use the Pythagorean Theorem to find PT. ¬290.3
PR2 ¬PT 2  RT 2 The surface area is approximately 290.3 ft2.
172 ¬PT 2  152 23. Find the radius.
289 ¬PT 2  225 A ¬r2
64 ¬PT 2 814.3 ¬r2
8 ¬PT
14. If X is a point on T, then PX is equal to the

814.3
  ¬r
radius of the sphere, and thus, PR. So, PX  9.4. T ¬4r2
 
2
814.3
15. If Y is a point on T, then PY is equal to the ¬4 
radius of the sphere, and thus, PR. So, PY  12.8. ¬3257.2
16. Use the Pythagorean Theorem to find the radius The surface area is 3257.2 m2.
of the charcoal rack. 24. Find the radius.
c2 ¬a2  b2 A ¬r2
112 ¬r2  52 227.0 ¬r2
121 ¬r2  25
96 ¬r2 
227.0
  ¬r
Find the difference in the areas. T ¬1
2 (4r )  r
 2 2
(11)2  r2  121  96 ¬3r 2
 25
 
2
 78.5 ¬3
227.0

The difference in the areas is 25  78.5 in2. ¬681.0


17. T  4r2 The surface area is 681.0 km2.
 4(25)2 25. True; a great circle is formed by the intersection
 7854.0 of a plane with a sphere such that the plane
The surface area is approximately 7854.0 in2. contains the center of the sphere, so they have the
18. T  4r2 same center and radii.
 4(14.5)2 26. False; two great circles will intersect at two
 2642.1 points.
The surface area is approximately 2642.1 cm2.
19. T ¬4r2
2
¬4d
2
¬d2
¬(450)2
27. True; two spheres can intersect in a point or a
¬636,172.5
circle, regardless of the lengths of their radii. So
The surface area is approximately 636,172.5 m2.
the statement is true.
20. T ¬4r2
2
¬4d
2

¬d2
¬(3.4)2
¬36.3
The surface area is approximately 36.3 ft2. 28. True; a chord that contains the center of a sphere
21. Find the radius. is a diameter of a sphere. The diameter is the
C ¬2r width of a sphere, and thus, is the longest chord.
40.8 ¬2r 29. True; when two spheres are tangent they
20
.4
 ¬r
 intersect in one point.
30. Pole to pole: T ¬4r2
T ¬1
2 (4r )  r
 2 2
2
¬4d
2
¬3r2
2 ¬d2
¬3 
20
.4
¬(7899.83)2
¬397.4 ¬196,058,359.3 mi2
The surface area is approximately 397.4 in2. Equator: T  (7926.41)2
22. Find the radius.  197,379,906.2 mi2
C ¬2r
30.2 ¬2r
15
.1
 ¬r
T ¬4r2
2
¬4 
15
.1

Chapter 12 438
31. Find the mean diameter. 40. The distance between opposite corners of the cube
7899.83  7926.41  7913.12
 is equal to the diameter of the sphere. Use the
2 Pythagorean Theorem to find the diagonal of a
The diameter of the atmosphere is 200 miles cube face if the side is x.
longer than the mean value: 8113.12 mi. c2 ¬a2  b2
T ¬4r2 c2 ¬x2  x2
2
¬4d
2
 c ¬x2 
Now, find the opposite corner distance.
¬d2
c2 ¬a2  b2
¬(8113.12)2
c2 ¬x2  (x2 )2
¬206,788,161.4 mi2
c ¬3x
2 2
32. Find the mean diameter.
c ¬x3 
7899.83  7926.41  7913.12
 x
3
2 The radius is half this, so r   2 , where x is the
T  4r2, so 0.75T  0.75(4r2)  0.75d2 is the length of each side of the cube.
surface area of the water. 41. None; every line (great circle) that passes through
0.75(7913.12)2  147,538,933.4 mi2 X will also intersect g. All great circles intersect.
1 (4r2)  r2
33. T ¬  42. Sample answer: Sports equipment manufacturers
2
2 2
use the surface area of spheres to determine the
¬2d
2    2 
 d amount of material to cover the balls for different
sports. Answers should include the following.
¬1
 2 1 2
2 d  4 d • The surface area of a sphere is four times the
¬3
4 (13)
 2 area of the great circle of the sphere.
• Racquetball and basketball are other sports
¬398.2
that use balls.
The surface area is approximately 398.2 ft2.
43. A; the distance between opposite corners of the
34. T  4r2, so if the radius is twice as large,
rectangular solid is equal to the diameter of the
4(2r)2  16r2, and the ratio is
sphere.
16r2
  4, or 4 : 1. Use the Pythagorean Theorem to find the
4r2
35. Let T2 ¬1  diagonal of the 4 ⊗ 5 face.
2 T1. Then
1 c2 ¬a2  b2
4r2 ¬2(4r21)
2
c2 ¬42  52
r22 ¬1
 2
2 r1
c ¬41
r1 Now, find the opposite corner distance.
r2 ¬ c2 ¬a2  b2
2 
 
2 c2 ¬72  (41)2
r2 ¬2 r1 c ¬90
2
 
2 c ¬310 
The ratio is 2 : 1.
The radius is half this, so
36. T  4r2, so if the radius is three times as large,
3
10
4(3r)2  36r2, and the ratio is r 2  4.74 in.
36r2 44. C; x
2  7  2 ¬x  1
  9, or 9 : 1.
4r2
x2  7 ¬x  1
37. T ¬4r2
2 x2  7 ¬(x  1)2
¬4d
2
 x2  7 ¬x2  2x  1
¬d2 6 ¬2x
12 mi : d2  (12)2 3 ¬x
 144
 452.4
20 mi : d2  (20)2 Page 676 Maintain Your Skills
 400 45. Use the Pythagorean Theorem to find the radius
 1256.6 of the base.
The surface area can range from 452.4 to c2 ¬a2  b2
1256.6 mi2. 192 ¬r2  132
38. T ¬4r2 192 ¬r2
2
¬4d
2
83  ¬r
T  r  r2
¬d2
 (83 )(19)  (83
)2
¬(7)2
 1430.3
¬153.9
The surface area is approximately 1430.3 in2.
The surface area is approximately 153.9 mi2.
39. The side length of the cube is equal to the
diameter of the sphere, so the radius of the sphere
is half the side of the cube.

439 Chapter 12
46. Use the Pythagorean Theorem to find the slant 53. Find the radius squared.
height. (x  h)2  (y  k)2 ¬r2
c2 ¬a2  b2 [3  (2)]2  (2  7)2 ¬r2
2 ¬72  102 52  (5)2 ¬r2
 ¬149
 25  25 ¬r2
T  r  r2 50 ¬r2
 (7)149
  (7)2 The equation of the circle is
 422.4 (x  2)2  (y  7)2  50.
The surface area is approximately 422.4 m2. 54. Find the center.
47. T  r  r2 6 
h 2
 (4.2)(15.1)  (4.2)2 2 4
8  5
 254.7 k 2
  3

2
The surface area is approximately 254.7 cm2. Find the radius squared.
2
48. Use the Pythagorean Theorem to find the slant (2  4)2 
5  3
2  ¬r
 2
height. 2
c2 ¬a2  b2 (2)2  2  ¬r
13 2
2
2 ¬7.42  2 
11
.2 18
5
4 ¬r
 2

 ¬86.12
 The equation of the circle is
T ¬r  r2 (x  4)2  y  3
2
2  4 .
 18
 5
2
¬d
2    2 
 d

¬1 1 2
2 d  4 d


¬1   1
Page 677 Geometry Activity: Locus and Spheres
2 (11.2)86.12 4 (11.2)
  2
1.
¬261.8
5 15 5
The surface area is approximately 261.8 ft2.
49. T ¬1
2 P  B

The locus of all points in space at a specific
¬1
2 (4  19)(16)  19
 2
distance from a given point is a sphere. Thus, for
¬969 this problem, the locus of points is two spheres
The surface area is 969 yd2. each with a radius of 5 units with centers that are
50. The apothem is half the base, or 6 feet. endpoints of the given line segment.
Use the Pythagorean Theorem to find the slant 2. Yes, the radii are congruent.
height. 3. Each sphere has a radius of 5 units and a
c2 ¬a2  b2 diameter of 10 units.
2 ¬62  (13)2
4. The segment is 25 units long and the radii of the
 ¬205

spheres are 5 units. So, the spheres are 15 units
T ¬1
2 P  B
 apart on the given segment.
¬1
2 (4  12)205
   122 5. The spheres intersect at a plane. The intersection
of a plane and a sphere is a circle or a point. So,
¬487.6
the intersection is a circle.
The surface area is approximately 487.6 ft2.
6. A circle is a locus of points on a plane.
51. T ¬1
2 P  B

7. The intersection is the set of all points
¬1
2 (4  11)(24)  11
 2 equidistant from the midpoint of the given line
segment in the plane containing the
¬649
perpendicular bisector of the given line segment.
The surface area is 649 cm2.
8. The particles from an explosion disperse in a
52. The diameter of the fabric required is 9  2(3) 
spherical pattern. Since the explosion is at ground
15 in.
level, the locus of points describing the dispersion
A ¬r2
2 of particles is a hemisphere with a radius of
¬d
2 300 ft.
¬1
4 (15)
 2

¬176.7
The area of fabric needed is approximately Chapter 12 Study Guide and Review
176.7 in2.
Page 678 Vocabulary and Concept Check
1. d 2. i 3. b 4. h
5. a 6. j 7. e
8. g 9. c 10. f

Chapter 12 440
Pages 678–682 Lesson-by-Lesson Review T ¬1
2 P  B

11. The solid is a cylinder. ¬1
2 (4  10)(12)  10
 2
Bases: F and G
There are no faces, edges, or vertices. ¬340
12. The solid is a rectangular prism. The surface area is 340 units2.
Bases: rectangle WXYZ and rectangle STUV 16.
Faces: rectangles WXYZ, STUV, WXTS, XTUY,
YUVZ, and WZVS
Edges: WX, X
Y
, Y
Z, Z
W, S
T
, T
U
, U
V, V
S
, W
S
, X
T
,
U
Y , and Z
V
Vertices: S, T, U, V, W, X, Y, and Z
13. The solid is a triangular prism.
Base: BCD
Faces: ABC, ABD, ACD, and BCD
Edges: AB
, B
C, A
C, A
D
, B
D, and C
D
Vertices: A, B, C, and D
14. Use the Pythagorean Theorem to find the
hypotenuse of the triangular base. T  6s2
c2 ¬a2  b2  6(4)2
c2 ¬32  42  96
c2 ¬25 The surface area is 96 units2.
c ¬5 17. Use the Pythagorean Theorem to find the
measure of the third side of the triangular base.
c2 ¬a2  b2
62 ¬42  b2
36 ¬16  b2
20 ¬b2
25  ¬b

T ¬Ph  2B
¬(3  4  5)(6)  2  1
2 34

¬84
The surface area is 84 units2.
15. Use the Pythagorean Theorem to find the slant
height.
c2 ¬a2  b2
2
132 ¬2  2 
10

169 ¬2  25
144 ¬2
12 ¬ T ¬Ph  2B
)(8)  2  1
¬(4  6  25 2  4  25
 
¬133.7
12 The surface area is approximately 133.7 units2.
18.

13
10

T  Ph  2B
 (2  2  2  4)(5)  2(2)(4)
 76
The surface area is approximately 76 units2.

441 Chapter 12
19. Use the Pythagorean Theorem to find the 25. T  2rh  2r2
measure of the fourth side of the trapezoidal base.  2(4)(58)  2(4)2
c2 ¬a2  b2  1558.2
c2 ¬42  (8  5)2 The surface area is approximately 1558.2 mm2.
c2 ¬25 26. T ¬2rh ¬2r2
c ¬5 2
¬2d
2 h  2 2 
 d

¬dh  1
2 d
 2

¬(4)(8)  1
2 (4)
 2

¬125.7
The surface area is approximately 125.7 km2.
27. T ¬1
2 P  B


¬1
2 (4  8)(15)  8
 2

¬304
The surface area is 304 units2.
28. Use the Pythagorean Theorem to find the slant
height.
c2 ¬a2  b2
2
132 ¬2  2 
10

169 ¬2  25
144 ¬2
12 ¬
The central angle of the pentagon measures
36
 0
T ¬Ph  2B 5 or 72. So, the angle formed by a radius and
72
the apothem is  
2 or 36.
¬(4  5  5  8)(8)  2  1
2 (4)(5  8)

Find the apothem, x.
¬228
The surface area is 228 units2.
20. Use the Pythagorean Theorem to find the
hypotenuse of the triangular base.
c2 ¬a2  b2
c2 ¬152  202
c2 ¬625 36 x
c ¬25
L  Ph
 (15  20  25)(18) 5
 1080 tan36 ¬5x
The lateral area is 1080 units2. 5
x ¬
21. L  Ph tan36
 (3  5  6  10)(3) T ¬1 1
2 P  2 Px

 72
¬1
2 P(  x)

The lateral area is 72 units2.
2 (5  10)12  tan36 
¬1
22. L  Ph  5
 (3  8  7  5)(4) ¬472.0
 92 The surface area is approximately 472.0 units2.
The lateral area is 92 units2.
29. Use the Pythagorean Theorem to find the height
23. T ¬2rh  2r2 of the triangular base.
2
¬2d
2h  22
d c2 ¬a2  b2
2
¬dh  1 52 ¬h2  5
2

2 d
 2
25 ¬h2  6.25
¬(4)(12)  1
2 (4)
 2
18.75
 ¬h
¬175.9
T ¬1 1
2 P  2 bh

The surface area is approximately 175.9 in2.
24. T  2rh  2r2 ¬1
 1
2 (3  5)(3)  2 (5)18.75

 2(6)(8)  2(6)2 ¬33.3
 527.8 The surface area is approximately 33.3 units2.
The surface area is approximately 527.8 ft2.

Chapter 12 442
30. T  r  r2 37. T ¬1
2 (4r )  r
 2 2

 (5)(18)  (5)2 ¬3r2


 361.3 ¬3(16)2
The surface area is approximately 361.3 mm2. ¬2412.7
31. Use the Pythagorean Theorem to find the radius The surface area is approximately 2412.7 ft2.
of the base. 38. T ¬4r2
c2 ¬a2  b2 2
52 ¬42  r2 ¬4d
2

25 ¬16  r2 ¬d2
9 ¬r2 ¬(5)2
3 ¬r ¬78.5
T  r  r2 The surface area is approximately 78.5 m2.
 (3)(5)  (3)2 39. Find the radius squared.
 75.4 A ¬r2
The surface area is approximately 75.4 yd2. 220 ¬r2
32. Use the Pythagorean Theorem to find the slant 220
 ¬r
  2
height.
c2 ¬a2  b2 T ¬4r2
2 ¬32  72 ¬4 
220

2 ¬9  49
 ¬58 ¬880
T  r  r2 The surface area is 880 ft2.
 (3)58
  (3)2 40. Find the radius squared.
 100.1 A ¬r2
The surface area is approximately 100.1 in2. 30 ¬r2
33. T ¬4r2 30
 ¬r
 2
2
¬4d
2

T ¬1
2 (4r )  r
 2 2

¬d2
¬3r2
¬(18.2)2
¬1040.6 ¬3
30

The surface area is approximately 1040.6 ft2. ¬90


34. T ¬1
2 (4r )  r
 2 2 The surface area is 90 cm2.
¬3r2
¬3(3.9)2
¬143.4 Chapter 12 Practice Test
The surface area is approximately 143.4 cm2.
35. Find the radius squared. Page 683
A ¬r2 1. c 2. a 3. b
121 ¬r2
4. The solid is a rectangular prism.
121
 ¬r
  2
Bases: rectangles PQRS and TUVW
T ¬1 Faces: rectangles PQRS, TUVW, SPUT, QRWV,
2 (4r )  r
 2 2
STWR, and PUVQ
¬3r2 Edges: PS
, Q
R, V
W, U
T, P
U, S
T
, R
W
, QV
, P
Q, S
R
,
¬3 
121
 V
U , and T
W
Vertices: P, Q, R, S, T, U, V, and W
¬363
The surface area is 363 mm2. 5. The solid is a sphere.
36. Find the radius squared. 6. The solid is a cone.
A ¬r2 Base: F
218 ¬r2 Vertex: H
218
 ¬r
2

T ¬4r2
¬4
218

¬872
The surface area is 872 in2.

443 Chapter 12
7. 11. L  Ph
 (5  3  4  7  4)(8)
 184
The lateral area is 184 units2.
12. T  2rh  2r2
 2(8)(22)  2(8)2
 1508.0
The surface area is approximately 1508.0 ft2.
13. T  2rh  2r2
 2(3)(2)  2(3)2
 94.2
The surface area is approximately 94.2 mm2.
T  Ph  2B 14. T  2rh  2r2
 (2  4  2  6)(3)  2(4)(6)  2(78)(100)  2(78)2
 108  87,235.7
The surface area is 108 units2. The surface area is approximately 87,235.7 m2.
8. Use the Pythagorean Theorem to find the slant 15. Use the Pythagorean Theorem to find the slant
height. height of the tetrahedron.
c2 ¬a2  b2 c2 ¬a2  b2
2
)2 ¬2  4
2
2
(226  62 ¬2  6
2

104 ¬2  4 36 ¬2  9
100 ¬2 27 ¬2
10 ¬ 33 ¬
36
0
The central angle of the triangular base is 3 or
120. So, the angle formed by a radius and the
12
0
apothem is  2 or 60. Find the apothem, x.
2 26
4

60 x

3
T ¬1
2 P  B
 tan60 ¬3x
¬1
2 (4  4)(10)  4
 2
x ¬ 3
tan
60
¬96 x ¬3 
The surface area is 96 units2. Use the Pythagorean Theorem with the apothem
9. L(3 5 base)  Ph and slant height to find the height of the
 (2  3  2  5)(6) tetrahedron.
 96 c2 ¬a2  b2
L(3 6 base)  (2  3  2  6)(5) (33)2 ¬(3 )2  b2
 90 27 ¬3  b2
L(6 5 base)  (2  6  2  5)(3) 24 ¬b2
 66 4.9 ¬b
The lateral areas are 96 units2 (3 5 base), The height is approximately 4.9  8  12.9 units.
90 units2 (3 6 base), and 66 units2 16. Use the Pythagorean Theorem to find the slant
(6 5 base). height of the tetrahedron.
10. Use the Pythagorean Theorem to find the c2 ¬a2  b2
2
hypotenuse. 62 ¬2  6
2

c2 ¬a2  b2
36 ¬2  9
c2 ¬152  202
27 ¬2
c2 ¬625
33 ¬
c ¬25
L ¬Lprism  Ltetrahedron
L  Ph
 (15  20  25)(8) ¬Ph  1

2 P
 480
The lateral area is 480 units2. ¬(3  6)(8)  1
2 (3  6)(33
 )
¬190.8
The lateral area is approximately 190.8 units2.

Chapter 12 444
17. Use the Pythagorean Theorem to find the slant 24. The area of the plastic is equal to the area of the
height of the tetrahedron. The slant height is the prism minus the area of the 12 ft by 25 ft
same as the altitude of the base. rectangular face.
c2 ¬a2  b2 Use the Pythagorean Theorem to find the height
2
62 ¬2  6
2
 of the triangular part of the base.
c2 ¬a2  b2
36 ¬2  9 2
27 ¬2 102 ¬a2  2 
12
33 ¬ 100 ¬a2  36
T ¬Ph  1 64 ¬a2
2 P  B

8 ¬a
¬(3  6)(8)  1
 )  1
2 (3  6)(33 2  6  33
  T ¬Ph  2B  (12)(25)
¬206.4 ¬(2  10  2  8  12)(25)  2
(8)(12)
The surface area is approximately 206.4 units2.
2  12  8  300
 1

18. Use the Pythagorean Theorem to find the slant
height. ¬1188
c2 ¬a2  b2 The amount of plastic needed is 1188 ft2.
2 ¬242  72 25. D; T ¬6s2
2 ¬625 150 ¬6s2
 ¬25 25 ¬s2
T  r  r2 5 ¬s
 (7)(25)  (7)2 The length of each edge is 5 cm.
 703.7
The surface area is approximately 703.7 units2.
19. Use the Pythagorean Theorem to find the radius.
c2 ¬a2  b2 Chapter 12 Standardized Test Practice
42 ¬32  r2
16 ¬9  r2 Pages 684–685
7 ¬r2 1. D; 3x  16 ¬2x  9
7  ¬r x ¬25
T  r  r2 [3(25)  16] ¬59
 (7 )(4)  (7)2 2. D; the change in x is 6  0  6 units.
 55.2 D; The change in y is 9  1  8 units.
The surface area is approximately 55.2 units2. So, the other endpoint has coordinates
20. T  r  r2 (10  6, 6  8)  (4, 2) or
 (7)(12)  (7)2 (10  6, 6  8)  (16, 14).
 417.8 3. C
The surface area is approximately 417.8 units2.
4. B; the relative length of each side corresponds to
21. T  4r2 the relative measure of its opposite angle.
 4(15)2 mC  180  70  48  62
 2827.4 C
A  is longest, A
B
 is in the middle, and B
C is
The surface area is approximately 2827.4 in2. shortest.
22. T ¬4r2 5. B; use the Pythagorean Theorem to find the
2
¬4d
2
length of AB
.
AC2 ¬BC2  AB2
¬d2
122 ¬52  AB2
¬(14)2
144 ¬25  AB2
¬615.8
119 ¬AB2
The surface area is approximately 615.8 m2.
10.9 ¬AB
23. Find the radius squared. The length of AB is approximately 10.9 in.
A ¬r2 N 2
116 ¬r2 6. B; A ¬
360 r
2
116 ¬  2 
N d
 ¬r
2
360
T ¬4r2 N
¬ 40 d
14
2

¬4
116 360 
¬ 120
1440 (18)
2

¬464 ¬54
The surface area is 464 ft2. 7. C; for example, a tetrahedron is a Platonic Solid
but it is not a prism.

445 Chapter 12
8. B; use the Pythagorean Theorem to find the 14a. Sample answer:
height of the triangular bases.
c2 ¬a2  b2
2
22 ¬a2  2
2

4 ¬a2  1
3 ¬a2
3 ¬a
T ¬Ph  2B 15 ft
¬(3  2)(4)  2  1
2  2  3
 
¬27
The surface area is approximately 27 cm2.
9. A; T ¬4r2 18 in.
2
¬4d
2

¬d2 14b. Change the diameter from inches to feet. There
¬(4)2 1
are 12 inches in 1 foot, so 18 inches  8
12 or
¬50 1.5 feet.
The surface area is approximately 50 ft2. L ¬2rh
10. Let y be the height of the opposite side of the ¬2d
2 h

right triangle.
y ¬dh
tan58 ¬
47 ¬(1.5)(15)
47tan58 ¬y ¬71
75 ¬y The lateral area is approximately 71 ft2.
So, the height of the tree is approximately 14c. T ¬2rh  2r2
75  5  80 ft. 2
¬2d
2h  22
d
11. A ¬1

2 bh
¬dh  1
2 d
 2
¬1
2 (12x  2x)(8x  2x)

¬(1.5)(15)  1
2 (1.5)
 2
1
¬2(10x)(6x) ¬74
¬30x2 The surface area is approximately 74 ft2.
The area is 30x2 units2. 15a. T ¬r  r2
12. area of deck 2
¬d
2    2 
 d
 area of rectangle  area of semicircle
 w  1 ¬1 1 2
2 d  4 d

2 r
 2
26  2
¬1
 1
2 (8)(7)  4 (8)
2
2  2 
 (26)(16)  1
   6

 259 ¬138
The area of the deck is approximately 259 ft2. The surface area is approximately 138 in2.
13. Find the apothem, x. 15b. T ¬2rh  2r2
2
The central angle of the pentagonal base is ¬2d
2h  22
d
36
 0
5 or 72. So, the angle formed by a radius and ¬dh  1
2 d
72  2
the apothem is  
2 or 36.
¬(8)(22)  1
2 (8)
 2

¬653
The surface area is approximately 653 in2.
15c. T ¬r  2rh  r2
2
¬d
2  22h  2
d d

36 x ¬1 1 2


2 d  dh  4 d


¬1 1
2 (8)(7)  (8)(22)  4 (8)
 2

4.5 cm
¬691
4.5
tan36 ¬ x The surface area is approximately 691 in2.
4.5
x ¬ tan36
T ¬1
2
P  1Px
2
¬1(5  9)(15)  1(5  9) 36 
4.5
tan
2 2
¬476.9
The surface area is approximately 476.9 cm2.

Chapter 12 446
Chapter 13 Volume
Page 687 Getting Started 8.
1. a2  122  132
Solve for a.
a2  144 ¬169
a2 ¬25
a ¬25

30
a ¬5
2
2. 43  b2  82
7
Solve for b. 3.5
48  b2 ¬64 Apothem: A 30°-60°-90° triangle is formed by the
b2 ¬16 apothem and one-half of a side of the hexagon.
b ¬16
The shorter leg of the triangle is 1

2 (7) or 3.5.
b ¬4
2 The apothem is the longer leg of the triangle or
3. a2  a2  32  3.53.
Solve for a. perimeter  7  6  42
2a2 ¬18
a2 ¬9 Area: A  1

2 Pa
a ¬  1
2 (42)3.53

9 
a ¬3
 127.3
4. b2  3b2  192
Solve for b. The area is approximately 127.3 ft2.
4b2 ¬192 9.
b2 ¬48
b ¬ 48
b ¬4 3 C
5. 256  72  c2
Solve for c. 45
256  49 ¬c2
305 ¬c2 13.4
c ¬ 305
6. 144  122  c2 A D B
Solve for c. Apothem: The central angles of the octagon are
360
144  144 ¬c2 all congruent, so mACB   D
8 or 45. C is an
288 ¬c2 apothem of the octagon. It bisects ACB and is a
 288 ¬c B
perpendicular bisector of A . So mACD  22.5.
c ¬122  Since the side of the octagon has measure 13.4,
7. AD  6.7.
6.
7
tan22.5° ¬
CD
6.7
CD ¬ 
tan 22.5°
¬16.175

30
perimeter  (13.4)(8)  107.2
Area: A ¬1

2 Pa
7.2 ¬1

2 (107.2)(16.175)
3.6
Apothem: A 30°-60°-90° triangle is formed by the ¬867.0
apothem and one-half of a side of the hexagon. The area is approximately 867.0 mm2.
The shorter leg of the triangle is 1
(7.2) or 3.6.
2
The apothem is the longer leg of the triangle or
3.63.
perimeter  (7.2)(6)  43.2
Area: A  1
Pa
2
(43.2)3.63
 1 
2
 134.7
The area is approximately 134.7 cm2.

447 Chapter 13
10. 17. Let A be (x1, y1) in the Midpoint Formula.

 1 2 x
W(10, 10)  W 2 , 2
1  y
2 
Write two equations to find the coordinates of B.
C 1x 1  y
10  2 10 ¬2
2 2
45 20  1  x2 20 ¬1  y2

10 19  x2 21 ¬y2
The coordinates of B are (19, 21).
A D B 18. Let B be (x2, y2) in the Midpoint Formula.
Apothem: The central angles of the octagon are
all congruent, so mACB   360
D
8 or 45. C is an W(0, 0)  W 
1
 x 2(2)
, 
1

y 2
2 
apothem of the octagon. It bisects ACB and is a Write two equations to find the coordinates of A.
x  (2) y 2
B
perpendicular bisector of A . So mACB  22.5. 0
1
 0 ¬
1

2 2
Since the side of the octagon has measure 10,
0  x1  2 0 ¬y1  2
AD  5.
5 2  x1 2 ¬y1
tan22.5° ¬
CD The coordinates of A are (2, 2).
5
CD ¬
tan22.5°
¬12.07
perimeter  (10)(8)  80 13-1 Volumes of Prisms and
Area: A ¬1
2 Pa
Cylinders
1
¬2(80)(12.07)
¬482.8 Page 688 Geometry Activity: Volume of a
The area is approximately 482.8 square in2. Rectangular Prism
1. 12 cubes in top layer 12 cubes in bottom layer
11. (5b)2  (5b)(5b)
 24 cubes
 (5)(5)(b)(b)
 25b2 2. The prism is 4 cubes long, 3 cubes wide, and
2 2 cubes high. 4  3  2  24.
12. n
4    4  4 
 n n
3. They are the same.
n 
n

44 4. See students’ work.
n

2
5. V  wh
1
6
x 2
13. 3 3
x  3
x
4y    4y  4y 
3x 3x Page 691 Check for Understanding

4y  4y 1. Sample answers: cans, roll of paper towels, and
3  3 
xx

44yy chalk; boxes, crystals, and buildings
9x

2 2. Julia; Che did not multiply 33 correctly.
16y2
3. V  Bh
4y 2 4y 4y
14.     1

2 (8)(12)(6)
7 7 7
4y  4y  288
  The volume of the prism is 288 cm3.
77
44yy 4. The diameter of the base, the diagonal, and the
 
77 lateral edge form a right triangle. Use the
16y2 Pythagorean Theorem to find the height.
 
49 a2  b2 ¬c2
15. Let A be (x1, y1) and B be (x2, y2). The coordinates h2  82 ¬172
of the midpoint are h2  64 ¬289
x x y y 0  (5) 1  4 h2 ¬225

1

2
2
,
1
2  
 2
 2, 2
h ¬15
 5
 3
2 , 2  or (2.5, 1.5) Now find the volume.
16. Let A be (x1, y1) and B be (x2, y2). The coordinates V  r2h
of the midpoint are  (42)(15)
x x y y 5  (3) 0  6  754.0

1

2
2
,
1
2 
 2
 2, 2 The volume is approximately 754.0 in3.
 (1, 3) 5. V  r2h
 (7.52)(18)
 3180.9
The volume is approximately 3180.9 mm3.

Chapter 13 448
6. Use the formula for the volume of a rectangular 13. Find the volume of the oblique prism using the
prism. formula for a rectangular prism.
V  Bh V  Bh
 (12)(12)(14)  (2.5)(3.5)(3.2)
 2016  28
The volume of the prism is 2016 ft3. The volume of the oblique prism is 28 ft3.
14. Find the volume of the oblique prism using the
formula for a rectangular prism.
Pages 692–694 Practice and Apply V  Bh
7. The diameter of the base, the diagonal, and the  (55)(35)(30)
lateral edge form a right triangle. Use the  57,750
Pythagorean Theorem to find the height. The volume of the oblique prism is 57,750 m3.
a2  b2 ¬c2 15. Find the volume of the oblique cylinder using the
h2  92 ¬152 formula for a right cylinder.
h2  81 ¬225 V  r2h
h2 ¬144  (13.22)(27.6)
h ¬12  15,108.0
Now find the volume. The volume is approximately 15,108.0 mm3.
V  r2h 16. Find the volume of the oblique cylinder using the
 (4.52)(12) formula for a right cylinder.
 763.4 V  r2h
The volume is approximately 763.4 cm3.  (2.62)(7.8)
8. V  Bh  165.6
 (18.7)(12.2)(3.6) The volume is approximately 165.6 yd3.
 821.3 17. V ¬ r2h
The volume is approximately 821.3 in3. Solve for r.
9. Use the Pythagorean Theorem to find the height 615.8 ¬ r2(4)
of the prism’s triangular base. 49 ¬r2
Let b  1
2 (base of triangle)  3 cm
 7 ¬r
a2  b2 ¬c2 The diameter is about 2(7) or 14 m.
a2  32 ¬82 18. V ¬Bh
a2  9 ¬64 1152 ¬64h
a2 ¬55 18 ¬h
a ¬ 55 The lateral edge length is 18 in.
Now find the volume of the prism. 19. The solid is a rectangular prism with   4,
V  Bh w  3, h  2.
 1 V  Bh
2 (6)(55
)(12)

 (4)(3)(2)
 267.0
 24
The volume is approximately 267.0 cm3. The volume is 24 units3.
10. The radius of the base is 1

2 (18) or 9. 20. The solid is a triangular prism. Its height is h  5
V  r2h
and its base has area B  1
2 (1)(1).
 (92)(12.4)
V  Bh
 3155.4
The volume is approximately 3155.4 m3.  1
2 (1)(1)(5)
11. V  Bh  2.5
 (15)(10)(5) The volume is 2.5 units3.
 750 21. The solid is a cylinder with r  2.1, h  3.5.
The volume of the prism is 750 in3. V  r2h
12. Find the area, B, of the base using the formula for  (2.12)(3.5)
the area of a trapezoid.  48.5
The volume is approximately 48.5 mm3.
B  1
2 h(b1  b2)


 1
2 (4)(6  10)


 32
Now find the volume of the prism.
V  Bh
 (32)(18)
 576
The volume of the prism is 576 in3.

449 Chapter 13
22. Treat the solid as a large rectangular prism with 26. The core radius is 1

2 (35) or 17.5. The core height
a smaller one attached underneath. is 586  40 or 626.
22 cm V  r2h
B1  (17.52)(626)
16 cm  602,282.6
The volume is approximately 602,282.6 ft3.
h1  16 cm
8 cm 27. The volume in gallons equals the volume r2h in
cubic feet multiplied by a conversion factor of
7 1
 3
2 gal/ft .
h2  6 cm V  r2h71
2

B2 8 cm
200,000  r2h(7.5)
volume  volume of large rectangular prism  Solve for r.
volume of smaller rectangular prism 26,667 ¬ r2h
 B1h1  B2h2 26,667 ¬ r2(23)
r2 ¬369.1
 (22)(8)(16)  (8)(6)(6)
r ¬19.2
 3104 The radius is approximately 19.2 ft.
The volume of the solid is 3104 cm3.
28. Consider the water to be added as a rectangular
23. volume  volume of rectangular prism  prism, with a height of 3  0.3 or 2.7.
volume of cylinder V  Bh
 Bh  r2h  (50)(25)(2.7)
 (6)(6)(6)  (1.52)(6)  3375 m3
 173.6 Since 1 cubic meter equals 1000 liters, the volume
The volume is approximately 173.6 ft3. of water to be added is 3375(1000) or 3,375,000 L.
24. The solid covers 360°  120°  240° of the 360° in 29. Begin by finding the area of the hexagon.
24
0
a cylinder. So the solid is 
360 of a cylinder.
24
0
volume  
360  volume of cylinder
 2
3 r h
 2
20
 2
3 (4 )(10)
 2
10
 335.1
30
The volume is approximately 335.1 ft3.
25. The holder can be modeled as a cylindrical solid
with a smaller cylindrical solid cut out of it. The Apothem: A 30°-60°-90° triangle is formed by the
smaller solid has the same radius as a can, and apothem and one-half of a side of the hexagon.
its height is 11.5  1  10.5. The larger solid has The hypotenuse is 1 
2 (20) or 10. So the shorter leg
a radius of 1
2 (8.5) or 4.25.
is 1
(10)  5 and the longer leg is 53
2 . This is the
length of the apothem. One side of the hexagon
r2 6.5 cm measures 10 mm, so the perimeter is 10  6 or 60.
volume of brass block
 volume of rectangular prism 
volume of hexagonal prism
 B1h1  B2h2
h1  11.5 cm h2  10.5 cm
 wh1  1

2 Pah2
 (50)(40)(60)  1
2 (60)(53
 )(60)
 104,411.5
r1 The volume is approximately 104,411.5 mm3.
8.5 cm
30. Set the density equal to mass
volume, then use
volume  volume of holder and “cut out”  8.0 g/cm3 as the density. For the volume, use
volume of “cut out” 104,411.5 mm3  104.4115 cm3 from Exercise 29.
 r12h1  r22h2
d ¬m

V
 (4.25)2(11.5)  (3.25)2(10.5) m
 304.1 8.0 ¬
104.4115
The volume is approximately 304.1 cm3. m ¬(8.0)(104.4115)
¬835.3
The mass is approximately 835.3 g.

Chapter 13 450
31. Start by finding the area of the base. 38. T  4 r2
 4 (8.5)2
 907.9
C The surface area is approximately 907.9 in2.
39. T  r  r2
72
4  (6)(11)  (62)
 320.4
A D B
The surface area is approximately 320.4 m2.
Apothem: The central angles of the pentagon are 40. radius  1
2 (16)  8


all congruent, so mACB   36


0
D
 is an T  r  r2
5 or 72. C
 (8)(13.5)  (82)
apothem of the pentagon. It bisects ACB and is
 540.4
B
a perpendicular bisector of A. So, mACD  36.
20 The surface area is approximately 540.4 cm2.
Since the perimeter is 20, AB  5  4 and

1 41. Use the Pythagorean Theorem to find the slant
AD  2(4)  2. height.
2
tan 36° ¬
CD
2
CD ¬
tan 36°
 h  12
¬2.7528
Area: A  1

2 Pa
 1

2 (20)(2.7528) r5
Now find the volume of the prism. 2  52  122
V ¬Bh 2  169
  13
¬1
2 (20)(2.7528)(5) Now find the surface area of the cone.
¬137.6
T  r  r2
The volume is approximately 137.6 ft3.  (5)(13)  (52)
32. Sample answer: Cartoonists use mathematical  282.7
concepts or terms in comics because of the The surface area is approximately 282.7 in2.
difficulty that many people have had with math. 42. radius  1
2 (14)  7

Answers should include the following.
Use the Pythagorean Theorem to find the slant
• A volume means a book. height.
• In mathematics, volume refers to the amount of
space that a figure encloses.
33. A; use the formula for the volume of a rectangular
prism.
V ¬Bh  h  24
16,320 ¬(85)w(8)
16,320 ¬680w
w ¬24 ft
34. B; r2h  2 rh  rh(r)  rh(2) r7
 rh(r  2) 2  72  242
2  625
  25
Page 694 Maintain Your Skills Now find the surface area of the cone.
35. radius  1
2 (12)  6
 T  r  r2
T  4 r2  (7)(25)  (72)
 4 (62)  703.7
 452.4 The surface area is approximately 703.7 in2.
The surface area is approximately 452.4 ft2. 43. Treat the floor plan as a large rectangle of width
36. T  4 r2 12  9  21 and length 13  8  7  28, plus a
 4 (412) 9  9 square.
 21,124.1 total area  area of large rectangle  area of square
The surface area is approximately 21,124.1 cm2.  w  s2
37. radius  1  (28)(21)  (9)2
2 (18)  9

 669
T  4 r 2
shaded area  1w1  2w2  s2
 4 (92)
 1017.9  (9)(13)  (12)(7)  (9)2
The surface area is approximately 1017.9 m2.  282

451 Chapter 13
shaded area
P(shaded)  total area 48. Use Theorem 10.17, about a tangent segment and
a secant segment.
28
2

669 x2  12(12  9.5)
 0.42 x2  258
The probability is about 0.42, or 42%. x   258 or  258
44. Since x is a length, it must be positive. So discard
 258.
x  16.1
49. Drawing the height divides the triangle into two
30°-60°-90° triangles each with base 1 
2 (7) or 3.5.
The height is then 3.5 3. For the whole
30 equilateral triangle,
A  1

2 bh
26 13
 1
2 (7)(3.53
)
15
6
The length of one side is 6 or 26.  21.22
Apothem: A 30°-60°-90° triangle is formed by the The area is approximately 21.22 in2.
apothem and one-half of a side of the hexagon. 50.
The shorter leg of the triangle is 1

2 (26) or 13. The
apothem is the longer leg of the triangle or 133 .
Area: A  1

2 Pa
 1
2 (156)(133
 )
30
 1756.3
The area is approximately 1756.3 in2. 12 6

45. perimeter  (6.2)(8)  49.6 Apothem: A 30°-60°-90° triangle is formed by the


apothem and one-half of a side of the hexagon. The
A  1

2 Pa shorter leg of the triangle is 21(12) or 6. The
 1

2 (49.6)(7.5) apothem is the longer leg of the triangle or 6 3.
 186 perimeter  12  6  72
The area of the octagon is 186 m2. Area: A  1
2 Pa
46. Use Theorem 10.17 about a tangent segment and
 1
2 (72)(63
 )
a secant segment.
132 ¬x(x  8)  374.12
169 ¬x2  8x The area is approximately 374.12 cm2.
0 ¬x2  8x  169 51.
Use the Quadratic Formula.
b  b  42
ac
x 
2a C
8   69)
82  4(1)(1
  2(1) 72
6
740
8  
  or  740
8  
2 2
Since x is a length, it must be positive. So discard
A D B
740
8  
 .
2 Apothem: The central angles of the pentagon are
x  9.6 all congruent, so mACB   36
0
D
 is an
5 or 72. C
47. Use Theorem 10.16 about 2 secant segments. apothem of the pentagon. It bisects ACB and is
8(8  6) ¬x(x  4) B
a perpendicular bisector of A. So, mACD  36.
112 ¬x2  4x Since the side of the pentagon has measure 6,
0 ¬x2  4x  112 AD  3.
Use the Quadratic Formula. 3
tan 36° ¬
CD
b  b  42
ac 3
x 2
a CD ¬
tan36°
4  4
 12)
 4(1)(1
2  ¬4.129
  2(1) perimeter  5  6  30
429
4    or  429
4    Area: A  1

 2 2 2 Pa
Since x is a length, it must be positive. So discard  1

2 (30)(4.129)
429
4  
 .  61.94
2
x  8.8 The area is approximately 61.94 m2.

Chapter 13 452
52. 3. The heights are the same.
4. V  1

3 Bh

C
Pages 698–699 Check for Understanding
45 1. Consider first the base of the cone or pyramid.
When a circle with area r2 is doubled in size, the
50 new area is (2r)2  4( r2), or 4 times the original
area. Similarly, when a square with area s2 is
A D B doubled in size, the new area is (2s)2  4s2, or 4
Apothem: The central angles of the octagon are times the original area.
360
all congruent, so mACB   D
8 or 45. C is an When, for a cone or pyramid, the height is also
apothem of the octagon. It bisects ACB and is a doubled, the new volume is
B
perpendicular bisector of A . So, mACD  22.5. V  1

3 (4B)(2h)
Since the side of the octagon has measure 50,
AD  25.  81
3 Bh

25 So in each case the volume is 8 times the original
tan 22.5° ¬
CD

volume.
25
CD ¬ 
tan 22.5° 2. The volume of a pyramid is one-third the volume
¬60.35534 of a prism of the same height as the pyramid and
perimeter  50  8  400 with bases congruent to the bases of the pyramid.
3. Sample answer:
Area: A  1

2 Pa
 1

2 (400)(60.35534)
 12,071.07
The area is approximately 12,071.07 ft2. 16
9
3 4
Page 695 Spread Sheet Investigation: Prisms
Use the formula A  2(  w)h  2w in column E.
Use the formula V  wh in column F. V  1
3 (3 )(16)
 2 V  1
3 (4 )(9)
 2

Prism Length Width Height Surface Area Volume V  48 V  48


1 1 02 03 0022 0006 4. V  1

3 Bh
2 2 04 06 0088 0048  1

3 (16)(10)(12)
3 3 06 09 0198 0162  640
4 4 08 12 0352 0384 The volume of the pyramid is 640 in3.
5 8 16 24 1408 3072 5. Use the 30°-60°-90° triangle to find the radius of
the base. The height of the cone is the longer leg
1. For each pair of prisms, the dimensions of the and the radius of the cone is the shorter leg.
second prism are two times the dimensions of the 1
first prism. r    12  43

3
2. The surface areas of prisms 2, 4, and 5 are four V  1
3 r h
 2
times the respective surface areas of prisms 1, 2,
and 4.  1
3 (43
 )2(12)
3. The volumes of prisms 2, 4, and 5 are eight times  603.2
the respective volumes of prisms 1, 2, and 4. The volume is approximately 603.2 mm3.
4. When the dimensions are doubled, the surface
6. V  1

3 Bh
area is multiplied by 4, or 22, and the volume is
multiplied by 8, or 23.  1
3 r h
 2

 1
3 (8 )(20)
 2

 1340.4
13-2 Volumes of Pyramids and Cones The volume is approximately 1340.4 ft3.
7. Use the formula for the volume of a cone.
Page 696 Geometry Activity: Investigating the V  1

3 Bh
Volume of a Pyramid  1

3 (38,000)(77)
1. It took 3 pyramids of rice.
 975,333.3
2. The areas of the bases are the same.
The volume is approximately 975,333.3 ft3.

453 Chapter 13
Pages 699–701 Practice and Apply a2  b2  c2
8. Find the area of the base first. 7.52  h2  (103)2
56.25  h ¬300
2

h2 ¬243.75
h ¬15.612
For the pyramid,
C
V  1

3 Bh
72
6  1

3 (20)(15)(15.612)
 1561.2
The volume of the pyramid is approximately
A D B 1561.2 ft3.
Apothem: The central angles of the pentagon are 10. Use the Pythagorean Theorem with h  pyramid
36
0
all congruent, so mACB   D
5 or 72. C is an height.
apothem of the pentagon. It bisects ACB and is a2  b2  c2
B
a perpendicular bisector of A. So, mACD  36. 92  h2  152
Since the side of the pentagon has measure 6, 81  h2 ¬225
AD  3. h2 ¬144
3 h ¬12
tan36° ¬
CD For the pyramid,
3
CD ¬tan V  1

36° 3 Bh
¬4.13  1

3 (24)(18)(12)
perimeter  5(6)  30 cm
 1728
Base area: A ¬1

2 Pa The volume of the pyramid is 1728 in3.
¬1
 11. Use the Pythagorean Theorem with h  cone
2 (30)(4.13)
height.
¬61.95
a2  b2  c2
Now find the volume of the pyramid. 182  h2  302
V ¬1

3 Bh 324  h2 ¬900
h2 ¬576
¬1

3 (61.95)(10) h ¬24
¬206.5 For the cone,
The volume of the pyramid is approximately
V  1
3 r h
 2
206.5 cm3.
9. Find the height of the pyramid by first finding the  1
3 (18 )(24)
 2

height of one of the 20-20-20 triangular faces (a  8143.0


pyramid slant height). The volume is approximately 8143.0 mm3.
12. Use the 45°-45°-90° triangle to find the radius of
the base: r  1  10  5
2. In the same way, find

2
30 the height: h  1  10  52
. For the pyramid,
20 20 
2
V  1
3 r h
 2

 1
3 (52
 )2(52
)
 370.2
The volume is approximately 370.2 in3
10 10
13. r
The height of this triangle is 103
.
Now use the Pythagorean Theorem with
h  pyramid height.
h
The distance from the center of the base of the
pyramid to one of the 20-in. edges is 1

2 (15) or 7.5. A

30

36
10 3 h

7.5

Chapter 13 454
Use trigonometry to find the radius of the base. 15. Use the Pythagorean Theorem with d  diameter
1(36°)  18° of base.
2 a2  b2  c2
opposite
sin A ¬ 
hypotenuse d2  52  132
r d2  25 ¬169
sin18° ¬ 
30 d2 ¬144
r ¬30sin18° d ¬12
r ¬9.2705 For the cone, the radius is 21(12) or 6. The volume is
Similarly, find the height.
adjacent V  1

3 Bh
cosA ¬ 
hypotenuse
h
 1
3 r h
 2
cos 18°  
30
 1
3 (6 )(5)
 2
h ¬30 cos 18°
h ¬28.5317  188.5
Now find the volume. The volume is approximately 188.5 cm3.
16. The radius of the base is r  1
2 (24)  12.

V  1
3 r h
 2
Use the 30°-60°-90° triangle to find the height,
 1
3 (9.2705) (28.5317)
 2
which is the shorter leg: h  1
2 (15)  7.5.

 2567.8 For the cone,
The volume of the cone is approximately
V  1
3 r h
 2
2567.8 m3.
14. Calculate the area of the base, B, first.  1
3 (12 )(7.5)
 2

 1131.0
The volume is approximately 1131.0 ft3.
17. Find the height of the pyramid portion by first
finding the height of one of the triangular sides
(the pyramid slant height). Use the Pythagorean
15 15 Theorem.
h1

8 10 10
4
h1
1(8)  4. Now use the Pythagorean Theorem to
2
find the height of the triangular base.
a2  b2  c2
12
42  h12  152 6
16  h12 ¬225 a2   b2 c2
h12 ¬209 62  h12  102
h1 ¬209
 36  h12 ¬100
Now calculate the area of the base. h12 ¬64
B  1

2 bh1
h1 ¬8
Now find the height of the pyramid, again using
 1
2 (8)(209
 )
the Pythagorean Theorem.
 4209

Now find the height of the pyramid.
a2  b2  c2
152  h22  172 8 8
225  h22 ¬289 h2
h22 ¬64
h2 ¬8
For the pyramid, 12
6
V  1

3 Bh2
 1
3 (4209
 )(8)
 154.2
The volume is approximately 154.2 m3.

455 Chapter 13
a2  b2  c2 a2  b2 ¬c2
62  h22  82 (24  16)2  h2 ¬102
36  h22 ¬64 64  h2 ¬100
h22 ¬28 h2 ¬36
h ¬6 cm
h2 ¬27 
Let x equal the height of the “missing” cone. By
volume of solid  volume of cube 
using similar triangles,
volume of pyramid 16 x
24 ¬ x  6

 s3  1

3 Bh2
2 ¬x
 123  1
3 (12)(12)(27
 ) 3 x6
 1982.0 2(x  6) ¬3x
2x  12 ¬3x
The volume is approximately 1982.0 mm3.
x ¬12 cm
18. Calculate the area of the hexagon first. The
center-to-vertex distance, s, which in a regular volume of frustum ¬volume of large cone 
hexagon equals the side length, is found using the volume of “missing” cone
Pythagorean Theorem. ¬1 1 2
3 r (x  h)  3 r x
 2

¬1
 2 1
3 (24 )(12  6)  3 (16 )(12)
2

¬7640.4 cm3
10 20. r  1
2 (103)  51.5 km, h  4.17 km

9.3
V  1
3 r h
 2

 1
3 (51.5 )(4.17)
 2

s  11,581.9
a  b ¬c2
2 2 The volume is approximately 11,581.9 km3.
s2  9.32 ¬102 21. m
3.776 k
s2  86.49 ¬100 9
s2 ¬13.51 Use trigonometry to find the radius of the base.
s ¬13.51
 3.7
76
Apothem: tan 9° ¬ r
3.7
76
r ¬
tan 9°
¬23.840726
V  1
3 r h
 2

¬1
3 (23.840726 )(3.776)
 2

30  2247.5
The volume is approximately 2247.5 km3.
22.
1
2 13.51
410
  1213.51
The apothem is 3 . 33
perimeter  613.51
 r
Use trigonometry to find the radius of the base.
Area: A  1

2 Pa 410
tan 33° ¬r
 1  

3
2 (613.51
 )  2 13.51  r ¬ 41
0
tan 33°
 35.1 ¬631.344635
volume of solid ¬2  volume of one pyramid
V  1
3 r h
 2
¬2  1

3 Bh  1
3 (631.344635 )(410)
 2

¬2  1

3 (35.1)(9.3)  171,137,610.4
¬217.6 ft3 The volume is approximately 171,137,610 m3.
19. Use the Pythagorean Theorem to find the height
of the frustum.
16 mm

10

24 mm 24  16 mm

Chapter 13 456
23. r  1
2 (22.3)  11.15
 30. Side view of tower with “missing portion”:
V  1
3 r h
 2

 1
3 (11.15 )(1.22)
 2

 158.8
The volume is approximately 158.8 km3.
24. 2

3 ; The volume of each pyramid that makes up the
h
solid on the left is 1

3 of the volume of the prism, so
the total volume of the solid on the left is 1 1
3  3

or 2 
3 of the volume of the prism. 8
25. Use the formula for the volume of a pyramid.
V  1

3 Bh
 1
 2
3 (755 )(481)
35
 91,394,008.3
The original volume of the pyramid is
approximately 91,394,008.3 ft3.
26. Use the formula for the volume of a pyramid.
15
V  1

3 Bh By similar triangles,
 1
 2
3 (755 )(449) 8
15 
h
h  35
 85,313,741.7 8h  280 ¬15h
The present day volume of the pyramid is 280 ¬7h
approximately 85,313,741.7 ft3. h ¬40 ft
volume of frustum ¬volume of large pyramid 
27. 91,394,008.33  85,313,741.67  6,080,266.7
volume of “missing” portion
Approximately 6,080,266.7 cubic feet have been
lost. ¬1 1
3 B1h1  3 B2h2

28. The volume of the cone is 1

3 of the volume of the ¬1 1 2
3 (15 )(35  40)  3 (8 )(40)
 2

cylinder, so
¬4771.7
probability  1  1

3
2
The volume of the frustum is approximately
3
 4771.7 ft3.
29. Find the height of the pyramid using the 31. Calculate the slant height, using the fact that
Pythagorean Theorem. each face is an equilateral triangle.
A

12 30 12
8 8
h1 

6 6
B C
5 5 The slant height  equals 63  because it is the
a2  b2  c2 measure of the longer leg of the 30°-60°-90° right
52  h12  82 triangle. Now find the apothem of the base.
25  h12 ¬64
h12 ¬39
h1 ¬ 39 12 12

For the cone, radius  1


2 (10) or 5.
volume of solid ¬volume of oblique pyramid  a
volume of cone 30
6
¬1 1 2
3 Bh1  3 r h2
 6
1
The apothem a equals   6  23
. Now find
¬1
 2 )  13 (52)(12)
3 (10 )(39
3

the height of the solid, using the Pythagorean
¬522.3
Theorem.
The volume of the solid is approximately 522.3
units3.

457 Chapter 13
36. V  r2h
 (82)(17)
 3418.1
6 3 The volume is approximately 3418.1 m3.
h
37. Use the Pythagorean Theorem to find the height
of the triangle. One leg of the right triangle is 21(10)
or 5 ft.
2 3
a2  b2 ¬c2
h  (23
2 )2 ¬(63)2
h  12 ¬108
2 13 13
h2 ¬96 h
h ¬4 6
Find the area of the base, B.
B  1

2 Pa 10 5
 1 a2  b2 ¬c2
2 (36)(23
)

h2  52 ¬132
 363  h2  25 ¬169
For the pyramid, h2 ¬144
V ¬1
 h ¬12
3 Bh
For the prism, taking the triangles as bases,
¬1
3 (363
 )(46) V ¬Bh
¬203.6 ¬1

2 (10)(12)(19)
The volume is approximately 203.6 in3. ¬1140
32. Sample answer: Architects use geometry to design The volume of the prism is 1140 ft3.
buildings that meet the needs of their clients. 38. Use the circumference to find the radius.
Answers should include the following. C ¬2 r
• The surface area at the top of a pyramid is 86 ¬2 r
much smaller than the surface area of the base. 43
There is less office space at the top, than on the
  ¬r
first floor. Now find the area.
• The silhouette of a pyramid-shaped building is T  4 r2
2
4
3
smaller than the silhouette of a rectangular  4 

prism with the same height. If the light
 2354.2
conditions are the same, the shadow cast by
the pyramid is smaller than the shadow cast The surface area is approximately 2354.2 cm2.
by the rectangular prism. 39. T  4 r2
33. B; V  1   4(64.5)
3 Bh  258
 1
 2
3b h The surface area is 258 yd2.
 1
 2
3 (2h) h
40. For the “missing” triangular corner,
4h 3 base  335  190  145
3 height  325  220  105
34. A; x3  9x  x(x2  9) area of field ¬area of large rectangle 
 x(x2  9) or x(x2  9) area of “missing” triangle
 x(x  3)(x  3) or x(x2  9) ¬w  1
2 bh
x(x2  9) is not one of the choices given. ¬(325)(335)  1

So the factors that could represent length, width, 2 (145)(105)
¬101,262.5
and height are x, x  3, and x  3.
The total area is 101,262.5 ft2.
41. 4 r2  4 (3.42)
 145.27
Page 701 Maintain Your Skills
35. V  Bh 42. 4
 3 4
3 r  3 (7 )
3

 (14)(12)(6)  1436.76
 1008 43. 4 r2  4 (122)
The volume of the prism is 1008 in3.  1809.56

Chapter 13 458
Page 701 Practice Quiz 1 5. Use the Pythagorean Theorem to find the height
1. Use the formula for the volume of a cylinder. The of the triangular base.
radius is r  1

2 (4) or 2.
V  r2h
 (22)(10)
 125.7
The volume is approximately 125.7 in3. 11 11
2. Use the formula for the volume of a cylinder.
The radius is r  1 
2 (12) or 6.
V  r2h
 (62)(15)
 1696.5 6
The volume is approximately 1696.5 m3. 3
3. First find the area of the hexagonal base. a2  b2 ¬c2
32  h2 ¬112
9  h2 ¬121
h2 ¬112
h ¬4 7
Now find the area of the base, B.
30 B ¬1

2 bh
6 ¬1
2 (6)(47
 )
3
Apothem: A 30°-60°-90° triangle is formed by the ¬127

apothem and one-half of a side of the hexagon. For the pyramid,
The shorter leg of the triangle is 1

2 (6) or 3. The
apothem is the longer leg of the triangle or 33 . V ¬1

3 Bh
perimeter  6  6  36 ¬1
3 (127
)(4)

The area of the hexagonal base is
¬42.3
A  1

2 Pa The volume is approximately 42.3 in3.
 1
2 (36)(33
 )
 543 
For the prism, 13-3 Volumes of Spheres
V  Bh
 (54 3)(10)
 935.3 Page 704 Check for Understanding
The volume is approximately 935.3 cm3. 1. The volume of a sphere was generated by adding
the volumes of an infinite number of small
4. r pyramids. Each pyramid has its base on the
surface of the sphere and its height from the base
to the center of the sphere.
2. Kenji; Winona divided the 12 by 3 before raising
h the result to the third power. Thus the order of
20
30 operations was not followed correctly.
3. V ¬4
3 r
 3

¬4
3 (13 )
 3

¬9202.8
Use a 30°-60°-90° triangle to find the radius of
The volume is approximately 9202.8 in3.
the base and the height.
4. The radius is 1
2 (12.5)  6.25.

r  1
2 (20) or 10 ft, and h  103
  ft.
4
For the cone, V ¬3 r3

V ¬1 ¬4
3 (6.25 )
 3
3 r h
 2

¬1 ¬1022.7
3 (10 )(103
)
 2
The volume is approximately 1022.7 cm3.
¬1813.8
The volume is approximately 1813.8 ft3.

459 Chapter 13
5. V ¬4
3 r
 3 13. Use the circumference to find the radius.
¬4 C ¬2 r
3 (4 )
 3
24 ¬2 r
¬268.1 12
¬r

The volume is approximately 268.1 in3.
6. Use the circumference to find the radius. Now find the volume.
C ¬2 r V ¬4
3 r
 3
18 ¬2 r 3
¬4 12
3
    
9 ¬r
π ¬233.4
V ¬4 3 r
 3 The volume is approximately 233.4 in3.
3
14. V ¬4
3 r
 3
¬4 9
3 π  
¬98.5 ¬4
3 (35.8 )
 3

The volume is approximately 98.5 cm3. ¬192,193.1


7. The radius is 12 (8.4)  4.2.
 The volume is approximately 192,193.1 mm3.
1 4

V ¬2 3 r3
 15. V ¬1 
 4 3
2 3 r 
¬2 ¬2
3 (3.2 )
 3
3 (4.2 )
 3

¬155.2 ¬68.6
The volume is approximately 155.2 m3. The volume is approximately 68.6 m3.
8. For the sphere, 16. radius  1 2 (28)  14

V ¬4
3 r
 3 1
 4

V ¬2 3 r3
¬4
3 (5 )
 3
¬2
3 (14 )
 3

50
0
¬ 3 ¬5747.0
For the cone, The volume is approximately 5747.0 ft3.
V ¬1 17. V ¬4
3 r
 3
3 r h
 2

¬1 ¬4
3 (12 )
 3
3 (5 )(20)
 2

¬50
0 ¬7238.2
3 The volume is approximately 7238.2 in3.
The two volumes are equal.
18. Find the radius.
Pages 704–706 Practice and Apply C ¬2 r
48 ¬2 r
9. V ¬4
3 r
 3
24
 ¬r

¬4
3 (7.62 )
 3
Now find the volume.
¬1853.3
The volume is approximately 1853.3 m3. V ¬4
3 r
 3
3
10. radius  12 (33)  16.5
 ¬4 24
3
    
4
V ¬3 r3 ¬1867.6
The volume is approximately 1867.6 cm3.
¬4
3 (16.5 )
 3
19. Use the formula for the volume of a sphere.
¬18,816.6 radius  1
2 (3476)  1738

The volume is approximately 18,816.6 in3.
V ¬4
3 r
 3
11. radius  12 (18.4)  9.2

4 ¬4
3 (1738 )
 3
V ¬3 r3

¬21,990,642,871
¬4
3 (9.2 )
 3
The volume is approximately 21,990,642,871 km3.
¬3261.8 20. For the golf ball, r  1
2 (4.3)  2.15.

The volume is approximately 3261.8 ft3.
V ¬4
3 r
 3
12. V ¬4
3 r
 3

 ¬4
3 (2.15 )
 3
 
3
¬4 
3
3
2
¬41.63
¬2.7
The volume is approximately 2.7 cm3.

Chapter 13 460
For the tennis ball, r  1
2 (6.9)  3.45.
 28. For the cylinder, r  1
2 (2.5)  1.25.

V ¬4 V  r2h
3 r
 3
 (1.252)(7.5)
¬4
3 (3.45 )
 3
 36.82
¬172.01 For the three balls, r  12 (2.5)  1.25.

The difference is about 172.01  41.63 or
130.4 cm3.

V ¬3 4
3 r
 3 
¬4 (1.253)
21. For the cone, r  1
2 (4)  2.

¬24.54
V ¬1
3 r h
 2 volume of empty space  36.82  24.54
 12.3 in3
¬1
3 (2 )(10)
 2
29. For the cube,
¬41.9 cm3 V ¬s3
For the ice cream, r  1
2 (4)  2.
 216 ¬s3
6 ¬s
V ¬4
3 r
 3

¬4
3 (2 )
 3

¬33.5 cm3
Since the volume of the ice cream is less than the
volume of the cone, the cone will not overflow. r 3
33
volume of ice cream   .5 3 2
22. 
volume of cone 41.9
3 3
 0.80
The cone will be about 80% filled.
23. V ¬4
3 r
 3

¬4
3 (17 )
 3

¬20,579.5
The volume is approximately 20,579.5 mm3. The radius of the sphere appears as r in the
figure. It is found by two applications of the
24. total volume  “just right” percentage
Pythagorean Theorem. First, for the horizontal
 20,579.5  0.59
right triangle:
 12,141.9
a2  b2 ¬c2
The “just right” volume is approximately
32  32 ¬c2
12,141.9 mm3.
18 ¬c2
25. T  4 r2 18
 ¬c
 4 (172) 32 ¬c
 3631.68
Now for the vertical right triangle:
total area  “wish for more” percentage
 3631.7  0.32 a2  b2 ¬c2
 1162.1 (32)2  32 ¬r2
The “wish for more” surface area is approximately 27 ¬r2
1162.1 mm2. 27 ¬r
26. A  r2 33  ¬r
 (172) For the sphere,
 907.9 mm2 V ¬4
3 r
 3
total area  “wish for less” percentage
¬4
3 (33
 )3
 907.9  0.09
 81.7 ¬587.7
The area of the “wish for less” sector is The volume of the sphere is approximately
approximately 81.7 mm2. 587.7 in3.
27. For the sphere, 30. Find the radius.
T ¬4 r2
V ¬4
3 r
 3
784 ¬4 r2
¬4
3 (6 )
 3 196 ¬r2
14 ¬r
¬288 cm3
Now find the volume.
For the cylinder,
V  r2h V ¬4
3 r
 3
 (62)(12)
¬4
3 (14 )
 3
 432
volume of sphere 288 ¬11,494.0
   2
432  3
 The surface area is approximately 11,494.0 in3.
volume of cylinder

461 Chapter 13
31. total area  area of curved half-sphere  40. Use the formula for the volume of a rectangular
area of flat bottom prism.
V ¬Bh
T  1
2 (4 r )  r
 2 2
25.9 ¬(2.4)(5.0)
Find the radius. 25.9 ¬12
18.75 ¬3 r2 2.2 ¬
6.25 ¬r2 The depth is approximately 2.2 ft.
2.5 ¬r 41. (x  h)2  (y  k)2 ¬r2
Now find the volume. (x  2)2  [y  (1)]2 ¬82
(x  2)2  (y  1)2 ¬64
V ¬1
 4 3
2 3 r  42. (x  h)2  (y  k)2 ¬r2
¬2
3 (2.5 )
 3
[x  (4)]2  [y  (3)]2 ¬(19)
2

¬32.7 (x  4)2  (y  3)2 ¬19


The volume is approximately 32.7 m3. 43. Find the center of the circle.
x x y y
32. radius  1
2 (142)  71
 M
1
   5  (1)
4  6
2, 2  2, 2
2 1 2

volume  volume of cylinder   (2, 1)
volume of hemisphere
Find the radius.
 r2h  1 
 4 3
2 3 r  1  diameter  1 
2 2 [1 
5]2  2
[6  (4)]
 (712)(71)  2
3 (71 )
 3
 34
 1,874,017.6
(x  h)2  (y  k)2 ¬r2
The volume is approximately 1,874,017.6 ft3. 2
(x  2)2  (y  1)2 ¬(34
)
33. radius  1
2 (4)  2
 (x  2)  (y  1) ¬34
2 2

volume  volume of cylinder  44. (2a)2  (2a)(2a)


combined volume of 2 hemispheres 22aa
 r2h  4  4a2
3 r
 3
45. (3x)  (3x)(3x)(3x)
3
 (22)(12)  4
3 (2 )
 3
333xxx
 184  27x3
The volume is approximately 184 mm3. 2

34. Sample answer: If a student knows the


5a
46.      
5a 5a
b  b b
5a 
5a
circumference of a sphere, then the volume can be bb
found. Answers should include the following. 5  5 
aa
 bb
• One needs to know the radius of the Earth.
a
25 2
• The radius of Earth is about 6366.2 km and the  2
b
volume is about 1.1  1012 km3. 3
35. See student’s work.
2
 
47. 5  5k
k 2
   
  2k
 
5
2k
5

2k  2k  2k
36. A; the ratio of the volumes is the cube of the
3
 5  5 5
ratio of the radii, namely 3  
5  0.216 or 21.6%.

 2  2  2 

555
kkk
37. A; 1
 2 2 1
2 (4 r )  r h  2 (4 r )  2 r  r h  2 r
2 2 2 2
8k 3
12
 r2(2  h  2) 5
 r2(4  h)

13-4 Congruent and Similar Solids


Page 706 Maintain Your Skills
38. V ¬1
3 r h
 2
Page 709 Spreadsheet Investigation: Explore
¬1
3 (6 )(9.5)
 2
Similar Solids
¬358.1 1. If a number in row 6 is a, then row 7 contains a2,
The volume is approximately 358.1 m3. and row 8 contains a3.
39. radius  12 (15)  7.5
 2. The ratio of the surface areas is a2 : b2.
1 3. The ratio of the volumes is a3 : b3.
V ¬3 r h
2

¬1
3 (7.5 )(7)
 2

¬412.3
The volume is approximately 412.3 m3.

Chapter 13 462
Page 710 Check for Understanding Pages 711–713 Practice and Apply
1. Sample answer: 7 in. 11. The bases have different shapes, so the two
5 in.
7 in. pyramids are neither congruent nor similar.
5 in.
12. The spheres are identical in shape but not in size
(unless a  b), so they are considered similar.
13. Use the Pythagorean Theorem to find the height
7 in. of the second cylinder.
6 in. a2  b2 ¬c2
12 in. 8 in.
122  h2 ¬202
144  h2 ¬400
h2 ¬256
2. If two solids are similar with a scale factor of
h ¬16 in.
a : b, then the surface areas have a ratio of a2 : b2
Since the two cylinders have the same height and
and the volumes have a ratio of a3 : b3.
diameter (2  6  12), they are congruent.
3. The two cones are of exactly the same shape and base edge of larger pyramid 12 
6
size, so they are congruent. 14.    
base edge of smaller pyramid 43
height of larger cylinder
4.    30
  3 2
height of smaller cylinder 20
height of larger pyramid
  
36 
6
 3

2 height of smaller pyramid 122 
diameter of larger cylinder 22
.5  33
   15 Since the ratios are not the same, the pyramids
diameter of smaller cylinder
 3

2
are neither congruent nor similar.
length of smaller prism10
15.     2
The two solids are similar. Since the scale factor length of larger prism 15  3
is not 1, they are not congruent.
width of smaller prism 1
height of larger pyramid 24   3
5.    18
 width of larger prism
height of smaller pyramid
Since the ratios are not the same, the pyramids
 4

3 are neither congruent nor similar.
The scale factor is 4 : 3. 16. The cubes are identical in shape but not in size,
surface area of larger pyramid 2
6.   a2 so they are similar (but not congruent).
surface area of smaller pyramid b
2 17. 26  5
1  130 m high

 42
3
49  5
1  245 m wide

16
 9

93  5
1  465 m long

The ratio of the surface areas is 16 : 9.
volume of larger pyramid 3 18. Always; spheres have only one measure to
7.   a3 compare.
volume of smaller pyramid b
43
 3
 19. Always; congruent solids have equal dimensions.
3
64 20. Sometimes; if the solids have a scale factor of 1,

27

the volumes will be equal.
The ratio of the volumes is 64 : 27. 21. Never; different solids cannot be similar.
diameter of smaller ball  2
8.  22. Never; different solids cannot be similar.
diameter of larger ball 16
23. Sometimes; solids that are not similar can have
 1

8 the same surface area.
The scale factor is 1 : 8. 1
2
surface area of smaller ball  a
24. 15  
1000  0.015
9. 
surface area of larger ball b2 The Micro-Car door handle is 0.015 cm long.
2
 12 00 2
 
10 1,000,000
8 25. x   1  x  1
1

64  1,000,000x
The ratio of the surface areas is 1 : 64. The full-sized car’s surface area is 1,000,000x cm2.
volume of smaller ball 3
10.   a3 26. 15  1 15
18  18

volume of larger ball b
 5
3
 13
 
8 6
1 The miniature door handle would be 5

6 or about
512 0.83 cm long.
The ratio of the volumes is 1 : 512.

463 Chapter 13
perimeter of smaller prism volume of smaller ball
29 3
27.  35.    3
perimeter of larger prism volume of larger ball 30
24,389
height of smaller prism’s base
  
27,0
00
height of larger prism’s base
The ratio of the volumes is 24,389 : 27,000.
4
 10
 36. 32 ft  32  12  384 in.
 2
5
length of gigantic ear of corn
  
384
length of real ear of corn 10
The ratio of the perimeters of the bases is 2 : 5. 19
2
5
surface area of smaller prism 2
28.   a2 The scale factor is 192 : 5.
surface area of larger prism b
2 2 37. Let V  volume of normal kernel.
 2
5 231 a3
 4 V ¬b
3
25 3
¬ 2
19
The ratio of the surface areas is 4 : 25. 53
7,077,888
volume of smaller prism 3 ¬ 12
29.   a3 5
volume of larger prism b
3 28,875 ¬7,077,888V
 23 V ¬0.004
5
8
 125 The volume of a normal kernel is approximately
The ratio of the volumes is 8 : 125. 0.004 in3.
5
30. Let V  volume of larger prism. 38. scale factor  
10
48 8
V ¬
  1

125 2
6000 ¬8V volume of smaller cone
  a
3

V ¬750 volume of original cone b3


3
The volume of the larger prism is 750 in3.  13
2
31. scale factor  5

6  1
8
Let V  volume of larger cone.
Since the smaller cone has 1
8 the volume of the
125 a3
V ¬b 3 original cone, the frustum has 1  1
 7
8  8 the
125 53
V ¬ volume of the original cone.
63
125 12 5 volume of frustum 7 7

V
 ¬
216   
1  8 or 7:8
8
volume of original cone
27,000 ¬125V volume of frustum 7
V ¬216   8
 7

1 or 7:1
volume of smaller cone 1
8
Since V ¬1
3 r h,
 2 39. Let r  the radius of the upper circle. Since the
5 1
216 ¬1 scale factor  
10 or 2,
3 (6 )h
 2

radius of upper circle 1


18 ¬h   2 5 cm
radius of lower circle
The height of the larger cone is 18 cm.
r 1
32. 5 ft  5  12  60 in. radius of lower circle  2
 10 cm

diameter of normal pie 8


   60 radius of lower circle  2r r
diameter of large pie
2 The lateral area of the
 15

smaller cone is
volume of normal pie a3
   L  r(5)  5 r.
volume of large pie b3 2r
3 The lateral area of the larger
 23
15 cone is L  (2r)(10)  20 r.
8
3375 lateral area of the frustum
The ratio of the volumes is 8 : 3375.  lateral area of the larger cone 
diameter of smaller ball 29 lateral area of the smaller cone
33.    
diameter of larger ball 30  20 r  5 r
The scale factor is 29 : 30.  15 r
surface area of smaller ball 2 lateral area of frustum r
34.   a2    15 3
surface area of larger ball b lateral area of original cone 20 r  4 or 3 : 4
2
29
 lateral area of frustum
   r
15 3
302
lateral area of smaller cone 5 r  1 or 3 : 1
84
1

900 40. Yes, both cones have congruent radii. If the
The ratio of the surface areas is 841 : 900. heights are the same measure, the cones are
congruent.

Chapter 13 464
41. The volume of the cone on top is equal to the sum 49. r
of the volumes of the cones inside the cylinder.
Justification: Call h the height of both solids. The
volume of the cone on top is 1
3 r h. If the height
 2
A
of one cone inside the cylinder is c, then the 13
h
height of the other one is h  c. Therefore, the
sum of the volume of the two cones is: 1 3 r c 
 2
1 1 1
 r2(h  c) or  r2(c  h  c) or  r2h.
3 3 3 46
42. Sample answer: Scale factors relate the actual
object to the miniatures. Answers should include
the following.
Use trigonometry to find the radius of the base.
• The scale factors that are commonly used are
1 : 24, 1 : 32, 1 : 43, and 1 : 64. 1(46°) ¬23°
2
opposite
• The actual object is 108 in. long. sinA ¬ 
hypotenuse
smallarea 2
a
large area ¬ b2
43. C;  r
sin23° ¬ 
13
¬4
9
 r ¬13sin23°
2
¬22 ¬5.0795
3
Similarly, find the height.
scale factor  2

3 adjacent
3 cosA ¬
hypote
nuse
  a
sm all vo lume
large volume b3 h
3 cos23° ¬
1
 23 3
3
8 h ¬13cos23°
 or 8 : 27
27 ¬11.967
yz xyz
44. D; xy  xy  y
z  Now find the volume.
y2z
V ¬1
3 r h
 2
 4

5 or 0.8
¬1
3 (5.0795) (11.967)
 2

¬323.3
Page 713 Maintain Your Skills The volume is approximately 323.3 in3.
45. radius  12 (8)  4
 50. V  1

3 Bh
4
V  3 r3
 1

3 (11)(7)(15)
 4
3 (4 )
 3
 385
 268.1 The volume of the pyramid is 385 m3.
The volume is approximately 268.1 ft3. 51. Use trigonometry to find the radius of the base.
opposite
46. V  4
3 r
 3 tanA ¬
adjacent
 4
3 (9.5 )
 3 21
tan62° ¬
r
 3591.4 r ¬ 21

tan 62°
The volume is approximately 3591.4 m3.
¬11.166
47. V  4
3 r
 3
Now find the volume.
 4
3 (15.1 )
 3
V ¬1
3 r h
 2
 14,421.8 ¬1
3 (11.166) (21)
 2
The volume is approximately 14,421.8 cm3.
¬2741.8
48. radius  12 (23)  11.5
 The volume is approximately 2741.8 ft3.
4
V  3 r3 52. Start with the formula for the surface area of a
cylinder.
 4
3 (11.5 )
 3
T ¬2 rh  2 r2
 6370.6 430 ¬2 r(7.4)  2 r2
The volume is approximately 6370.6 in3. Solve for r.
2 r2  14.8 r  430  0

465 Chapter 13
Use the Quadratic Formula. 57. y  3x  5
b  b
 
2  4ac Use x  4, y  17.
r ¬ 
2a 17  3(4)  5
14.8  (14.8
 430)
)2  4(2 )( 17  12  5
¬  2(2 ) 17  17
¬5.4 or 12.8 Yes, the ordered pair is on the graph.
Since the radius cannot be negative, discard 58. y  4x  1
12.8. Use x  2, y  9.
The radius is approximately 5.4 cm. 9  4(2)  1
53. Start with the formula for the surface area of a 981
cylinder. 99
T ¬2 rh  2 r2 Yes, the ordered pair is on the graph.
224.7 ¬2 r(10)  2 r2 59. y  7x  4
Solve for r. Use x  1, y  3.
2 r2  20 r  224.7  0 3  7(1)  4
Use the Quadratic Formula. 3  7  4
b  b  
2  4ac 3 11
r ¬ 2a

No, the ordered pair is not on the graph.
20  
(20 )2 24.7)
 4(2 )(2
¬  2(2 )
¬2.8 or 12.8 Page 713 Practice Quiz 2
Since the radius cannot be negative, discard
1. V ¬4
3 r
 3
12.8.
The radius is approximately 2.8 yd. ¬4
3 (25.3)
 3

54. Use the Pythagorean Theorem to find the height ¬67,834.4


of the triangle. The volume is approximately 67,834.4 ft3.
2. radius  12 (36.8)  18.4

4
V  3 r3
 4
3 (18.4)
 3
100 100
 26,094.1
The volume is approximately 26,094.1 cm3.
base side of left pyramid
3.   7 
5
base side of right pyramid
The scale factor is 7 : 5.
70
surface area of left pyramid 2
35 4.   a2
surface area of right pyramid b
a2  b2 ¬c2 7 2
 2
352  h2 ¬1002 5
49
1225  h2 ¬10,000 25

h2 ¬8775 The ratio of the surface areas is 49 : 25.
h ¬15 39 volume of left pyramid 3
5.   a3
For the triangle, volume of right pyramid b
3
A ¬1

2 bh
 73
5
34 3
¬1
2 (70)(1539
 )  125
¬3279 The ratio of the volumes is 343 : 125.
The area is approximately 3279 yd2.
55. Use the formula for the area of a rectangle.
A  w
 (12)(3)
13-5 Coordinates in Space
 36
The area of the rowboat is 36 ft2. Page 717 Check for Understanding
56. 3279 yd2  3279  9  29,511 ft2 1. The coordinate plane has 4 regions or quadrants
area of ro
probability ¬  wboat with 4 possible combinations of signs for the
shaded area ordered pairs. Three-dimensional space is the
36
¬
29,5
11 intersection of 3 planes that create 8 regions with
8 possible combinations of signs for the ordered
¬0.0012
triples.

Chapter 13 466
2. Sample answer: Use the point at (2, 3, 4); 8. First, write a vertex matrix.
A(2, 3, 4), B(2, 0, 4), C(0, 0, 4), D(0, 3, 4), E(2, 3, 0), M N P Q R S T V

 
F(2, 0, 0), G(0, 0, 0), and H(0, 3, 0). x 0 3 3 0 0 0 3 3
z y 0 0 4 4 0 4 4 0
D (0, 3, 4) z 0 0 0 0 2 2 2 2
C (0, 0, 4)
Next, multiply each element by the scale factor, 2.
B (2, 0, 4) M N P Q R S T V
A (2, 3, 4)

 
0 3 3 0 0 0 3 3
G (0, 0, 0) H (0, 3, 0) 2 0 0 4 4 0 4 4 0
O y 0 0 0 0 2 2 2 2
F (2, 0, 0)
M N P Q R S T V
x
E (2, 3, 0)

 
0 6 6 0 0 0 6 6
 0 0 8 8 0 8 8 0
3. A dilation of a rectangular prism will provide a
0 0 0 0 4 4 4 4
similar figure, but not a congruent one unless
r  1 or r  1. The coordinates of the vertices of the dilated
4. z image are M (0, 0, 0), N (6, 0, 0), P (6, 8, 0),
C (0, 0, 5) D (0, 1, 5) Q (0, 8, 0), R (0, 0, 4), S (0, 8, 4), T (6, 8, 4), and
V (6, 0, 4).
B (2, 0, 5)
z V
A (2, 1, 5) T
G (0, 0, 0) N
4 P
O H (0, 1, 0) y R 4 S
F (2, 0, 0) M
E (2, 1, 0)
x O 4 Q y
x
5. z
9. Write the coordinates of each corner. Then use the
Q ( 1, 0, 2) translation equation (x, y, z) → (x  48, y, z  16)
P ( 1, 4, 2) to find the coordinates of each vertex of the
R (0, 0, 2) rectangular prism that represents the storage
S (0, 4, 2)
U ( 1, 0, 0) container.
V (0, 0, 0) T ( 1, 4, 0)
O W (0, 4, 0) y Coordinates of the Translated coordinates,
x
vertices, (x, y, z) (x  48, y, z  16)
Preimage Image
6. DE  
(x2 x 2 
1)  (y 2  y 
1)  (z
2
2  z
1)
2
(12, 8, 8) (36, 8, 24)
 
(1  0
)2  (5
 0)2
 (7 
 0)2 (12, 0, 8) (36, 0, 24)
 75
 or 53
 (0, 0, 8) (48, 0, 24)
x x y y z z (0, 8, 8) (48, 8, 24)
M 
1

2, 2, 2
2 1 2 1 2
 (12, 8, 0) (36, 8, 16)
 2, 2, 2
0  1 0  5 0  7
(12, 0, 0) (36, 0, 16)
 1
2, 2, 2
 5 7 (0, 0, 0) (48, 0, 16)
(0, 8, 0) (48, 8, 16)
7. GH  
(x2  
x1)2 
(y2 
y1)2 
(z2 
z1)2
 
[5  (
3)]2 
 [3 
 (4)]2 
(5 
6)2
 186
 Pages 717–719 Practice and Apply
x x y y z z
M 
1

2, 2, 2
2 1 2 1 2
 10. z
F ( 2, 0, 2)
 2, 2, 2
3  5 4  3 6  5
H ( 2, 0, 0) C ( 2, 2, 2)
E (0, 0, 2) D (0, 2, 2)
 1, 7
2, 2
 1 G ( 2, 2, 0)
I (0, 0, 0)
O J (0, 2, 0) y
x

467 Chapter 13
11. z 16. KL  (x
 2
x1)2 
(y2 
y1)2 
(z2 
z1)2
S (0, 4, 1) T (0, 0, 1)  
(2 
2)2 
(2 
2)2 
(0  
0)2
W (0, 4, 0) O
32
 or 42

A (0, 0, 0) y
x x y y z z
R (3, 4, 1)
U (3, 0, 1) M 
1

2, 2, 2
2 1 2 1 2

2  (2) 2  (2) 0  0
 2, 2, 2
V (3, 4, 0) B (3, 0, 0)
x
 (0, 0, 0)
12. z
17. PQ  
(x2  
x1)2 
(y2 
y1)2 
(z2 
z1)2

O D (0, 6, 0)   
[3  (2)]2
 [2 
 (5)]2  (
1  
8)2
A (0, 0, 0) y
 115

E (0, 0, 3) x x y y z z
B (4, 0, 0) C (4, 6, 0) M 
1

2, 2, 2
2 1 2 1 2

G (0, 6, 3) 5  
 2  3  (1)
2
(2) 8

2 , 2 ,
x  2, 2, 2
1 7 7

F (4, 0, 3) P (4, 6, 3) 18. FG  


(x2  
x1)2 
(y2 
y1)2 
(z2 
z1)2

 5
13. 2 2
z  0  3  (3  0)  0  4 2
5

M (0, 0, 0) O  10

x x y y z z
N (0, 1, 0) y M 
1

2, 2, 2
2 1 2 1 2

I (0, 0, 3)
L (4, 0, 0) 
 
3
03
5
2 , 2, 
2
0 4
5 0

J (0, 1, 3)

 3 3 2
10, 2, 5 
K (4, 1, 0)
x
19. GH  (x
 2
x1)2 
(y2 
y1)2 
(z2 
z1)2
H (4, 0, 3)
1  1)  2  (1)  (2  6)
(
G (4, 1, 3) 2 2
 5 5
2

14. z
P ( 2, 4, 0)  17

Q ( 2, 0, 0) x x y y z z
S (0, 4, 0) R (0, 0, 0) M 
1

2, 2, 2
2 1 2 1 2

 
O y 1  
1  1
6 2 25 
 5
2 , 
2 , 2
K ( 2, 4, 4)
L ( 2, 0, 4)  3
5 ,  10 , 4
 7
N (0, 4, 4) M (0, 0, 4)
20. ST  (x
 2
x1)2 
(y2 
y1)2 
(z2 
z1)2
x
 (  6
)  
(5  4   4
2 2 2
43 3 )  (2
 )
2
15. z  31

x x y y z z
R ( 1, 3, 0)
Q ( 1, 0, 0)
M 
1

2, 2, 2
2 1 2 1 2

O
S (0, 3, 0) y 
63
   
2
43,  5 42
4    2
2 , 2


P (0, 0, 0)
 5
, 92, 
 53 2

2

W ( 1, 3, 6) 21. BC  
(x2  
(y2 
y1)2 
x1)2 (z2 
z1)2
 
(2
3  
 3)2 
 (4  2
)2  (4 
2 )2
22
x V ( 1, 0, 6)  39

x x y y z z
T (0, 3, 6) U (0, 0, 6) M 
1

2, 2, 2
2 1 2 1 2


3
 
2
 
23
, 2   42
4 22
2, 2


 

3

2 , 3, 32

Chapter 13 468
22. distance The coordinates of the dilated image are
 (x
x G (8, 6, 4), H (8, 0, 4), J (0, 0, 4), K (0, 6, 4),
2
1) 
(y2 
y1)2 
(z2 
2 z1)2
L (8, 6, 0), M (0, 6, 0), N (0, 0, 0), P (8, 0, 0).
 (240
  50)240
 (100)
 12 
(2.5 
2)2
z
 292.7
The distance is approximately 292.7 miles. K J 4
23. First, write a vertex matrix. 4
E M 4 N
A B C D H G F
O 4 y

 
x 1 0 0 1 1 0 0 1 G
y 0 0 1 1 0 0 1 1 H
4 4
z 1 1 1 1 1 1 1 1
L P
Next, multiply each element by the scale factor, 3.
A B C D E H G F x

 
1 0 0 1 1 0 0 1
3 0 0 1 1 0 0 1 1 25. Write the coordinates of each corner. Then use the
1 1 1 1 1 1 1 1 translation equation (x, y, z) → (x  2, y  5,
z  5) to find the coordinates of each vertex of the
A B C D E H G F rectangular prism.

 
3 0 0 3 3 0 0 3
Coordinates of the Translated coordinates,
 0 0 3 3 0 0 3 3
vertices, (x, y, z) (x  2, y  5, z  5)
3 3 3 3 3 3 3 3
Preimage Image
The coordinates of the dilated image are
P(2, 3, 3) P (0, 2, 2)
A (3, 0, 3), B (0, 0, 3), C (0, 3, 3), D (3, 3, 3),
E (3, 0, 3), H (0, 0, 3), G (0, 3, 3), and F (3, 3, 3). Q(2, 0, 3) Q (0, 5, 2)
z R(0, 0, 3) R (2, 5, 2)
H S(0, 3, 3) S (2, 2, 2)
G
T(2, 0, 0) T (0, 5, 5)
E U(2, 3, 0) U (0, 2, 5)
F
V(0, 3, 0) V (2, 2, 5)
O y
W(0, 0, 0) W (2, 5, 5)
26. Write the coordinates of each corner. Then use the
B C translation equation (x, y, z) → (x  2, y  1,
x
z  1) to find the coordinates of each vertex of the
A rectangular prism.
D
Coordinates of the Translated coordinates,
24. First, write a vertex matrix. vertices, (x, y, z) (x  2, y  1, z  1)
G H J K L M N P Preimage Image

 
x 4 4 0 0 4 0 0 4 A(2, 0, 1) A (0, 1, 0)
y 3 0 0 3 3 3 0 0
z 2 2 2 2 0 0 0 0 B(2, 0, 0) B (0, 1, 1)

Next, multiply each element by the scale factor, 2. C(2, 1, 0) C (0, 2, 1)

G H J K L M N P
D(2, 1, 1) D (0, 2, 0)

 
4 4 0 0 4 0 0 4 E(0, 0, 1) E (2, 1, 0)
2 3 0 0 3 3 3 0 0 F(0, 1, 1) F (2, 2, 0)
2 2 2 2 0 0 0 0
G(0, 1, 0) G (2, 2, 1)
G H J K L M N P
H(0, 0, 0) H (2, 1, 1)

 
8 8 0 0 8 0 0 8
 6 0 0 6 6 6 0 0
4 4 4 4 0 0 0 0

469 Chapter 13
27. Write the coordinates of each corner. Then use the 29. Write a vertex matrix and multiply it by the scale
translation equation (x, y, z) → (x  1, y  2, factor, 2.
z  2) to find the coordinates of each vertex of the A B C D E F G H A B C D E F G H

   
cube. 3 3 0 0 3 3 0 0 6 6 0 0 6 6 0 0
Coordinates of the Translated coordinates, 2 3 0 0 3 3 0 0 3  6 0 0 6 6 0 0 6
vertices, (x, y, z) (x  1, y  2, z  2) 3 3 3 3 0 0 0 0 6 6 6 6 0 0 0 0
Preimage Image The coordinates of the dilated image are
A(3, 3, 3) A (4, 5, 1) A (6, 6, 6), B (6, 0, 6), C (0, 0, 6), D (0, 6, 6),
E (6, 6, 0), F (6, 0, 0), G (0, 0, 0), and H (0, 6, 0).
B(3, 0, 3) B (4, 2, 1)
z
C(0, 0, 3) C (1, 2, 1) C D
D(0, 3, 3) D (1, 5, 1)
E(3, 3, 0) E (4, 5, 2)
C
F(3, 0, 0) F (4, 2, 2) D
G(0, 0, 0) G (1, 2, 2) B A
H(0, 3, 0) H (1, 5, 2) B A H
G
G O H y
z

C F
D E

A
B C D x
F E
G
O H y
side length of dilated cube  6 units
B A V  s3
F E H  63
G
x  216 units3
30. Write a vertex matrix and multiply it by the scale
F E
factor 1

3.
28. Write the coordinates of each corner. Then use the A B C D E F G H A B C D E F G H
translation equation (x, y, z) → (x  2, y  3,

   
3 3 0 0 3 3 0 0 1 1 0 0 1 1 0 0
z  2) to find the coordinates of each vertex of the 1 3 0 0 3 3 0 0 3  1 0 0 1 1 0 0 1
3
cube. 3 3 3 3 0 0 0 0 1 1 1 1 0 0 0 0
Coordinates of the Translated coordinates, The coordinates of the dilated image are
vertices, (x, y, z) (x  2, y  3, z  2) A (1, 1, 1), B (1, 0, 1), C (0, 0, 1), D (0, 1, 1),
Preimage Image E (1, 1, 0), F (1, 0, 0), G (0, 0, 0), and H (0, 1, 0).
A(3, 3, 3) A (1, 0, 5)
z
B(3, 0, 3) B (1, 3, 5)
C(0, 0, 3) C (2, 3, 5) C
D
D(0, 3, 3) D (2, 0, 5)
E(3, 3, 0) E (1, 0, 2) D
F(3, 0, 0) F (1, 3, 2) B
C
G(0, 0, 0) G (2, 3, 2) B A A

H(0, 3, 0) H (2, 0, 2) G H H y
G
F E
C z
D
F E
A
B
G H x
C D
E
The scale factor is 1

3 , so the ratio of the volumes
F
B A for these two cubes is
G H volume of new cube 3
  1 1
2
7.
O y
volume of original cube 33

F E
x

Chapter 13 470
31. first balloon location: (12, 12, 0.4) 36. The cube extends from x  2  4  2 to
second balloon location: (4, 10, 0.3) x  2  4  6, from y  4  4  0 to y  4  4  8,
and from z  6  4  2 to z  6  4  10. So the
distance  (x
 2
x1)2 
(y2 
y1)2 
(z2 
z1)2
coordinates of the vertices are A(2, 0, 2),
 [4  
(12)] 2  [
10 (12
)]2  [
0.3  
0.4]2 B(6, 0, 2), C(6, 8, 2), D(2, 8, 2), E(2, 8, 10),
 8.2 F(6, 8, 10), G(6, 0, 10), and H(2, 0, 10).
The distance between the balloons is z
approximately 8.2 miles. H E
x1  x2 y1  y2 z1  z2
32. M   2, 2, 2 
2x 4y 7z

(5, 1, 2)  22 , 22 , 22  G F
2x 4y 7z A
5 ¬22 1 ¬22 2 ¬22 2 D
10 ¬2  x2 2 ¬4  y2 4 ¬7  z3
O 2 y
8 ¬x2 2 ¬y2 3 ¬z3
B C
Point B has coordinates (8, 2, 3). 4
33. The center of the sphere is the midpoint of the x
diameter. 37. Sample answer: Three-dimensional graphing is
x x y y z z
M 
1

2, 2, 2
2 1 2 1 2
 used in computer animation to render images and
8  x 10  y 2  z
allow them to move realistically. Answers should
(4, 2, 6)  2, 2, 2
2 2 2
include the following.
8x 10  y 2  z • Ordered triples are a method of locating and
4  22 2 ¬22 6 ¬22 naming points in space. An ordered triple is
8  8  x2 4 ¬10  y2 12 ¬2  z2 unique to one point.
0  x2 14 ¬y2 14 ¬z2 • Applying transformations to points in space
The other endpoint has coordinates (0, 14, 14). would allow an animator to create realistic
movement in animation.
34. Use the midpoint formula with the endpoints of
x x y y z z
the diameter.
x1  x2 y1  y2 z1  z2
38. C; 
M 
1
2, 2, 2
2 1 2 1 2


M ¬  2, 2, 2  5  x 4  y 2  z
(4, 5, 3)  2, 2, 2 2 2 2

center  12 
 , 10 2
14   , 12 2
8   2
 5x 4  y 2  z
2 4  22 5 ¬22 3  22
¬(1, 1, 7)
The radius is the distance from the center (1, 1, 7) 8  5  x2 10  4  y2 6  2  z2
to (14, 8, 2). Let the center be the point 3  x2 6 ¬y2 8  z2
(xc, yc, zc). The other endpoint has coordinates (3, 6, 8).
radius  
(x2  
xc)2 
(y2 
yc)2 
(z2 
zc)2 39. B;  x  1 ¬x  1
x  1 ¬(x  1)2
 
(14 
1)2 
(8 
1)2 
(2  7
)2 x  1 ¬x2  2x  1
 
275 or 511 0 ¬x2  3x
35. The prism has moved down 5 units, right 3 units, 0 ¬x (x  3)
and forward 2 units. x  0 or x  3
(x, y, z) → (x  2, y  3, z  5) Check if each value satisfies the original
equation.
 0  1 ¬0  1  3  1 ¬3  1
1 ¬1 4 ¬2

1 ¬1 2 ¬2
So the solution is x  3.
40. The locus of points in space that satisfy the graph
of x  y  5 is a plane perpendicular to the
xy-plane whose intersection with the xy-plane is
the graph of y  x  5.
41. The locus of points in space that satisfy the graph
of x  z  4 is a plane perpendicular to the
xz-plane whose intersection with the xz-plane is
the graph of z  x  4.

471 Chapter 13
Page 719 Maintain Your Skills Pages 720–722 Lesson-by-Lesson Review
width of smaller prism 9 11. V  Bh
42.   
width of larger prism 18  (18)(7)(4)
 1
  504
2
height of smaller prism
The volume of the prism is 504 in3.
7
   13 12. V  r2h
height of larger prism
 (32)(11)
Since the ratios are not the same, the prisms are
 311.0
neither similar nor congruent.
The volume is approximately 311.0 m3.
2 
diameter of smaller cylinder 5
43.    15 13. The 15-ft diagonal forms a right triangle with the
diameter of larger cylinder
height and width. Use the Pythagorean Theorem
 2

3 to find the width.
height of smaller cylinder 12
    a2  b2 ¬c2
height of larger cylinder 18
2 w2  32 ¬152
 3 w2  9 ¬225
The two cylinders are similar. Since the scale w2 ¬216
factor is not 1, they are not congruent. w ¬66  ft
44. V  4
3 r
 3 Now find the volume.
 4 V  Bh
3 (10 )
 3
 (17)(66 )(3)
 4188.8  749.5
The volume is approximately 4188.8 cm3.
The volume is approximately 749.5 ft3.
45. radius  12 (13)  6.5 yd

14. First find the area of the hexagonal base.
4
V  3 r3

 4
3 (6.5 )
 3

 1150.3
The volume is approximately 1150.3 yd3.
46. V  4
3 r
 3
30
 4
3 (17.2 )
 3

 21,314.4 3
1.5
The volume is approximately 21,314.4 m3.
Apothem: A 30°-60°-90° triangle is formed by the
47. radius  1
2 (29)  14.5 ft

apothem and one-half of a side of the hexagon.
V  4
3 r
 3 The shorter leg of the triangle is 1

2 (3) or 1.5.
 4
3 (14.5 )
 3 The apothem is the longer leg of the triangle or
 12,770.1 1.53.
The volume is approximately 12,770.1 ft3. perimeter  3  6  18
Area: A ¬1
2 Pa
1
¬2(18)(1.53
)
Chapter 13 Study Guide and Review ¬13.53 
Now find the volume of the pyramid.
Page 720 Vocabulary and Concept Check V ¬1

1. pyramid 3 Bh

2. Congruent ¬1
 )(14)
3 (13.53
3. an ordered triple ¬109.1
4. cylinder The volume is approximately 109.1 cm3.
5. similar 15. radius  1
2 (15)  7.5

6. prism Use the Pythagorean Theorem to find the height.
7. the Distance Formula in Space a2  b2 ¬c2
7.52  h2  262
8. sphere
56.25  h2 ¬676
9. Cavalieri’s Principle h2 ¬619.75
10. cone h ¬619.7
5 

Chapter 13 472
For the cone, Now solve for .
V  1 232 ¬22  56
3 r h
 2
176 ¬22
¬1
3 (7.5 )(619.7
 2 5 ) 8 ¬
¬1466.4 For the right solid, the surface area is
The volume is approximately 1466.4 ft3. T ¬Ph  2B
232 ¬2(8  7)(h)  2(8  7)
16. V ¬1

3 Bh Now solve for h.
¬1 232 ¬30h  112
3 (17)(5)(13)
120 ¬h
¬368.3
4 ¬h
The volume is approximately 368.3 m3.
The solids have the same dimensions, so they are
17. V ¬4
3 r
 3 congruent.
¬4
3 (2 )
 3 23. Two spheres with different radii are similar,
though not congruent.
¬33.5
24. AB   (x2  
x1)2  (y2 
y1)2 (z2 z1)2
The volume is approximately 33.5 ft3.
18. radius  1
2 (4)  2
  
[3  (
5)]2 
 [8
 (
8)]2 
[4  
(2)]2
V ¬4  100
  10
3 r
 3
x x y y z z
¬4
3 (2 )
 3 M ¬ 
1

2, 2, 2
2 1 2 1 2

3 8  (8) 
¬33.5 5 
¬  2
 , 2, 22 4

The volume is approximately 33.5 ft3. ¬(1, 8, 1)
19. Find the radius.
25. CD  
(x2  
x1)2 
(y2 
y1)2 
(z2 
z1)2
C ¬2 r
65 ¬2 r ¬
[9 
(9)]2
 [9
 2]2
 [7 
4]2
65
 ¬r ¬58


2 x x y y z z
Now find the volume. M 
1

2, 2, 2
2 1 2 1 2

V ¬4
3 r
 3 9  (9) 
¬  , 2 9 47

3 2 2, 2
3  2 
¬4  65
 ¬(9, 5.5, 5.5)
¬4637.6 26. EO ¬(x
 2  
x1)2 
(y2 
y1)2 
(z2 
z1)2
The volume is approximately 4637.6 mm3. ¬
(4 
0)2 
(5  0
)2  (5
 0)2
20. Find the radius. ¬66

T ¬4 r2 x x y y z z
126 ¬4 r2 M ¬ 
1

2, 2, 2
2 1 2 1 2

4 
, 5 0 50
63
 ¬r2
2 ¬ 2
 0 
2, 2

63
2 ¬r
  ¬(2, 2.5, 2.5)
Now find the volume. 27. FG  
(x2  
x1)2 
(y2 
y1)2 
(z2 
z1)2
V ¬4  (2
   5
2 )2  (
2  
37 )
37 2  (
12  6
)2
3 r
 3
 422

 
3
3 
¬4
 63

2 x x y y z z
¬133.0
M ¬ 
1

2, 2, 2
2 1 2 1 2

 6  (12)
The volume is approximately 133.0 cm3. 
¬   
52
2
 
22  
, 37 2
37
, 2 
21. Find the radius.
¬(1.52
, 37
, 3)
A ¬ r2
25 ¬ r2
25 ¬r2
5 ¬r Chapter 13 Practice Test
Now find the volume.
V ¬4
3 r
 3 Page 723
¬4 1. b
3 (5 )
 3
2. c
¬523.6
The volume is approximately 523.6 units3. 3. a
22. For the left solid, the surface area is
T ¬Ph  2B
232  2(  7)(4)  2(  7)

473 Chapter 13
4. The diameter of the base, the diagonal, and the Find the height using the Pythagorean Theorem
lateral edge form a right triangle. Find the and the fact that for a regular hexagon the
diameter using the Pythagorean Theorem. distance from the center to a vertex is the same
a2  b2 ¬c2 as the side length.
82  d2 ¬102
64  d2 ¬100
d2 ¬36
d ¬6 yd
radius  12 (6)  3

13
Now find the volume, using the formula for a h
cylinder.
V ¬ r2h
¬ (32)(8)
¬226.2
The volume is approximately 226.2 yd3. 5
5. Use the formula for a rectangular prism. a2 ¬c2
b2
V ¬Bh 52 h ¬132
2

¬(6)(14)(10) 25  h2 ¬169
¬840 h2 ¬144
The volume of the prism is 840 mm3. h ¬12 m
6. Find the width using the Pythagorean Theorem. Now find the volume.
a2  b2 ¬c2 V ¬1 
3 Bh
72  w2  (74 )2
49  w2 ¬74 ¬31 (37.53
)(12)
w2 ¬25 ¬259.8
w ¬5 The volume is approximately 259.8 m3.
Now find the volume, using the formula for a
9. Use the formula for a cone.
rectangular prism.
V ¬Bh radius  1
2 (8.2)  4.1

¬(7)(5)(2) V ¬1
3 r h
 2
¬70
¬1
3 (4.1 )(6.8)
 2
The volume of the prism is 70 km3.
7. Use the formula for a pyramid. ¬119.7
The volume is approximately 119.7 cm3.
V ¬1
3 Bh 10. Use the formula for an oblique cone.
¬1

3 (5)(5)(3) First find the radius.
¬25 C ¬2 r
The volume of the pyramid is 25 ft3. 22 ¬2 r
11 ¬r
8. First find the area of the hexagonal base.
Now find the volume.
V ¬1 
3 Bh
¬31 r2h
¬1
3 (11 )(9)
 2

30
¬1140.4
The volume is approximately 1140.4 in3.
2.5
11. The length, the width, and the diagonal form a
5
right triangle. Use the Pythagorean Theorem to
Apothem: A 30°-60°-90° triangle is formed by the
find the width w.
apothem and one-half of a side of the hexagon.
a2  b2  c2
The shorter leg of the triangle is 1

2 (5) or 2.5. 782  w2  110.32
The apothem is the longer leg of the triangle or 6084  w2 ¬12,166.09
2.53. w2 ¬6082.09
perimeter  5  6  30 w ¬77.98776 ft
Area: A ¬1

2 Pa
¬1 )
2 (30)(2.53
¬37.53


Chapter 13 474
Consider the water as a rectangular prism. Now surface area of larger cylinder 2
find the volume in cubic feet, multiplied by a 17.  ¬a 
b2
surface area of smaller cylinder
conversion factor of 71

2 gallons per cubic foot.
2
¬
15
10
V ¬Bh71
2
 2
¬3
2

¬(78)(77.98776)(17)(7.5)
2
¬775,588 ¬3
22
The volume of water is approximately
775,588 gal. ¬9
4

The ratio of the surface areas is 9 : 4.
12. V ¬4
3 r
 3
3
volume of larger cylinder
a
¬4 volume of smaller cylinder ¬ b3
18. 
3 (3 )
 3
3
¬113.1 ¬
10 
15

The volume is approximately 113.1 cm3. 3
¬3
2

13. Find the radius.
C ¬2 r ¬3
3
23
34 ¬2 r 2
7
17 ¬ 8
¬r

The ratio of the volumes is 27 : 8.
Now find the volume.
19. d ¬
(x2  
x1)2 
(y2 
y1)2 
(z2 
z1)2
V ¬4
3 r
 3
3 ¬
(0  0
)2  (
3  0
)2  (5
 0)2
3  
¬4 17
 ¬34

x x y y z z
¬663.7
The volume is approximately 663.7 ft3.

M ¬ 
1
2, 2, 2
2 1 2 1 2

0 0  (3) 0  5
14. Find the radius. ¬ 2 , 2, 2
0 
T ¬4 r2
¬(0, 1.5, 2.5)
184 ¬4 r2
46 20. d ¬
(x2  
x1)2 
(y2 
y1)2 
(z2 
z1)2
  ¬r
2

r ¬3.8265 ¬
(1 
0)2 
(10 
0)2 
(5 
0)2
Now find the volume. ¬126
 or 314
x x y y z z
V ¬4
3 r
 3 
M ¬ 
1
2, 2, 2
2 1 2 1 2

¬4 0  (1) 
¬ 10 0  (5)
2 , 2
0 
3 (3.8265 )
 3
,
2
¬234.7 ¬(0.5, 5, 2.5)
The volume is approximately 234.7 in3.
21. d ¬
(x2  
x1)2 
(y2 
y1)2 
(z2 
z1)2
15. Find the radius.
A ¬ r2 ¬
(9  0
)2  (5
 0)2
 (
7  0
)2
157 ¬ r2 ¬155

x x y y z z
157

 ¬r
2

M ¬ 
1
2, 2, 2
2 1 2 1 2

r ¬7.06928 9 0  5 0  (7)
Now find the volume. 
0 
¬  2 , 2, 2 
V ¬4 ¬(4.5, 2.5, 3.5)
3 r
 3

¬4 22. d ¬


(x2  
x1)2 
(y2 
y1)2 
(z2 
z1)2
3 (7.06928 )
 3

¬1479.8 ¬
[3 
(2)]2
 [
5  2
]2  [
4  2
]2
The volume is approximately 1479.8 mm3. ¬86

x x y y z z
radius of larger cylinder
16. 
radius of smaller cylinder 
M ¬ 
1
2, 2, 2
2 1 2 1 2

2  (3) 2  (5) 2  (4)
height of larger cylinder
 
height of smaller cylinder

¬  , , 
2 2 2 
15 ¬(2.5, 1.5, 1)

10 23. d ¬
(x2  
x1)2 
(y2 
y1)2 
(z2 
z1)2
 3

2 ¬
(9 
9)2 
(7 
3)2 
(6  
4)2
The ratio of the radii is 3 : 2.
¬428
 or 2107

x x y y z z

M ¬ 
1
2, 2, 2
2 1 2 1 2

9 9 37 46
¬ 2 , 2, 2
¬(0, 2, 5)

475 Chapter 13
24. d ¬
(x2  
x1)2 
(y2 
y1)2 
(z2 
z1)2 12z  27z  6z  
5 2 z
12 z
27 5 6z 2
9.  3z  3z  3z
3z  
¬
(3 
8)2  2
[5  (6)]  (10
 1)2
 4z4  9z  2
¬323

10. Sierra: p → q
x x y y z z
M ¬ 
1

2, 2, 2
2 1 2 1 2
 Carlos: q → p
Carlos formed the contrapositive.
8  (3) 

¬  2
,
6 
2
 , 1 2
5  10
 11. If the measures of the corresponding sides are the
¬(2.5, 0.5, 5.5) same, the triangles are congruent.
25. C; V ¬Bh 12. From Theorem 8.2, the sum of eight exterior
360 ¬(15)w(2) angles is 360, so that one exterior angle
36
0
360 ¬30w measures  8  45.
12 ¬w

B 45 C
Chapter 13 Standardized Test Practice x
x
Pages 724–725 A
1. A; ACD and ACB together form the right
angle, BCD.
2. B; 5x° ¬x°  90°
4x° ¬90°
x ¬22.5
and since x  mDEF  90,
22.5  mDEF ¬90 Because the octagon is regular, AB  CB, and
mDEF ¬67.5 ABC is an isosceles triangle, so mA  mC.
3. C; the third side length must be greater than By the Exterior Angle Theorem,
21  13  8 and less than 21  13  33. mA  mC  45
4. C; from the statement QRS is similar to TUV, x  x ¬45
we know that Q and T are corresponding 2x ¬45
angles. x ¬22.5
5. B; volume of drilled block  volume of prism  13. Point A lies on the x-axis, b units to the left of
volume of cylinder D(0, c) just as B is b units to the right of C. The
coordinates of A are (b, 0).
 Bh  r2h
 (11)(5)(8)  (22)(8) 14. V ¬1
3 r h
 2
 339.5 cm3 ¬1
3 (10 )(18)
 2
6. B; C ¬2 r
¬600 cm3
25 ¬2 r
25 15a. The surface area of the small can is 54 in2 and
 ¬r
2 the surface area of the large can is 90 in2.
V ¬4
3 r
 3 When the height is doubled, the lateral area of
3 the cylinder is doubled, but the area of the bases
¬3 
4 
25
2 remains the same. The surface area increases by
¬264 in3 a factor of 12

3 times.
height of larger cylinder 16 15b. The volume of the small can is 54 in3 and the
height of smaller cylinder  12
7. C;  
volume of the larger can is 108 in3. The volume
 4
 increases by a factor of 2.
3
r 16. volume of tank  volume of cone 

4.5 ¬4

3 volume of cylinder 
3r ¬18 volume of hemisphere
3 r h1  r h2  2  3 r 
¬1
r ¬6  2 2 1 4 3
V ¬ r2h
¬1
 2 2 2
3 (5 )(15)  (5 )(45)  3 (5 )
3
¬ (62)(16)
¬1809.6 cm3 ¬4188.8
8. D; r ¬(x2  
x1)2 
(y2 
y1)2 
(z2 
z1)2 The volume is approximately 4188.8 m3.

¬(9  3)
2  (
2  1
)2  (
2  4
)2
¬9

Chapter 13 476
Prerequisite Skills
Pages 728–729 Graphing Ordered Pairs 16–31. Graph for Ex. 16–31:
1. The x-coordinate is 2. y T
The y-coordinate is 3. D
The ordered pair is (2, 3). M E
2. The x-coordinate is 1. A G
The y-coordinate is 1. C
N
The ordered pair is (1, 1). O x
H S
3. The x-coordinate is 2. B
The y-coordinate is 2. R F I
The ordered pair is (2, 2). L
4. The x-coordinate is 3. P
The y-coordinate is 3. 16. Start at the origin. Move 1 unit left, since the
The ordered pair is (3, 3). x-coordinate is 1. Then move 3 units up, since
5. The x-coordinate is 3. the y-coordinate is 3. Draw a dot, and label it M.
The y-coordinate is 1. Point M(1, 3) is in Quadrant II.
The ordered pair is (3, 1). 17. Start at the origin. Move 2 units right, since the
6. The point lies on the y-axis, so its x-coordinate x-coordinate is 2. Since the y-coordinate is 0, the
is 0. The y-coordinate is 3. The ordered pair is point lies on the x-axis. Draw a dot, and label it S.
(0, 3). Because it is on one of the axes, point S(2, 0) is
not in any quadrant.
7. The x-coordinate is 4.
The y-coordinate is 1. 18. Start at the origin. Move 3 units left, since the
The ordered pair is (4, 1). x-coordinate is 3. Then move 2 units down, since
the y-coordinate is 2. Draw a dot, and label it R.
8. The x-coordinate is 3.
Point R(3, 2) is in Quadrant III.
The y-coordinate is 2.
The ordered pair is (3, 2). 19. Start at the origin. Move 1 unit right, since the
x-coordinate is 1. Then move 4 units down, since
9. The x-coordinate is 1.
the y-coordinate is 4. Draw a dot, and label it P.
The y-coordinate is 1.
Point P(1, 4) is in Quadrant IV.
The ordered pair is (1, 1).
20. Start at the origin. Move 5 units right, since the
10. The x-coordinate is 1.
x-coordinate is 5. Then move 1 unit down, since
The y-coordinate is 4.
the y-coordinate is 1. Draw a dot, and label it B.
The ordered pair is (1, 4).
Point B(5, 1) is in Quadrant IV.
11. The x-coordinate is 3. The point lies on the x-axis,
21. Start at the origin. Move 3 units right, since the
so its y-coordinate is 0. The ordered pair is (3, 0).
x-coordinate is 3. Then move 4 units up, since the
12. The x-coordinate is 2. y-coordinate is 4. Draw a dot, and label it D. Point
The y-coordinate is 4. D(3, 4) is in Quadrant I.
The ordered pair is (2, 4).
22. Start at the origin. Move 2 units right, since the
13. The x-coordinate is 2. x-coordinate is 2. Then move 5 units up, since the
The y-coordinate is 4. y-coordinate is 5. Draw a dot, and label it T. Point
The ordered pair is (2, 4). T(2, 5) is in Quadrant I.
14. The x-coordinate is 3. 23. Start at the origin. Move 4 units left, since the
The y-coordinate is 3. x-coordinate is 4. Then move 3 units down, since
The ordered pair is (3, 3). the y-coordinate is 3. Draw a dot, and label it L.
15. The x-coordinate is 4. Point L(4, 3) is in Quadrant III.
The y-coordinate is 2. 24. Start at the origin. Move 2 units left, since the
The ordered pair is (4, 2). x-coordinate is 2. Then move 2 units up, since
the y-coordinate is 2. Draw a dot, and label it A.
Point A(2, 2) is in Quadrant II.
25. Start at the origin. Move 4 units right, since the
x-coordinate is 4. Then move 1 unit up, since the
y-coordinate is 1. Draw a dot, and label it N. Point
N(4, 1) is in Quadrant I.
26. Start at the origin. Move 3 units left, since the
x-coordinate is 3. Then move 1 unit down, since
the y-coordinate is 1. Draw a dot, and label it H.
Point H(3, 1) is in Quadrant III.

477 Prerequisite Skills


27. Start at the origin. Since the x-coordinate is 0, the 35. Graph the ordered pairs on a coordinate plane.
point lies on the y-axis. Move 2 units down, since Connect each pair of consecutive points. The
the y-coordinate is 2. Draw a dot, and label it F. polygon is a rectangle.
Because it is on one of the axes, point F(0, 2) is y
not in any quadrant.
28. Start at the origin. Move 3 units left, since the
x-coordinate is 3. Then move 1 unit up, since the R( 2, 1) S(4, 1)
y-coordinate is 1. Draw a dot, and label it C. Point
C(3, 1) is in Quadrant II. O x
29. Start at the origin. Move 1 unit right, since the P ( 2, 1) Q (4, 1)
x-coordinate is 1. Then move 3 units up, since the
y-coordinate is 3. Draw a dot, and label it E. Point
E(1, 3) is in Quadrant I.
30. Start at the origin. Move 3 units right, since the 36. Make a table.
x-coordinate is 3. Then move 2 units up, since the Choose four values for x.
y-coordinate is 2. Draw a dot, and label it G. Point Evaluate each value of x for 2x.
G(3, 2) is in Quadrant I. x 2x y (x, y)
31. Start at the origin. Move 3 units right, since the
x-coordinate is 3. Then move 2 units down, since 1 2(1) 2 (1, 2)
the y-coordinate is 2. Draw a dot, and label it I. 0 2(0) 0 (0, 0)
Point I(3, 2) is in Quadrant IV.
32. Graph the ordered pairs on a coordinate plane. 1 2(1) 2 (1, 2)
Connect each pair of consecutive points. The 2 2(2) 4 (2, 4)
polygon is a square.
y
y
(2, 4)
W ( 3, 3) Y ( 1, 3)
(1, 2)

O (0, 0) x
O x
X ( 3, 1) Z ( 1, 1) ( 1, 2)

37. Make a table.


33. Graph the ordered pairs on a coordinate plane. Choose four values for x.
Connect each pair of consecutive points. The Evaluate each value of x for 1  x.
polygon is a square.
y M ( 1, 5) x 1x y (x, y)
0 10 1 (0, 1)
L( 2, 2) 1 11 2 (1, 2)
J ( 4, 2)
2 12 3 (2, 3)
O x 3 13 4 (3, 4)
K ( 1, 1)
y
(3, 4)
34. Graph the ordered pairs on a coordinate plane.
(2, 3)
Connect each pair of consecutive points. The (0, 1) (1, 2)
polygon is a triangle.
y F(2, 4) O x

G( 3, 2)

O x

H( 1, 3)

Prerequisite Skills 478


38. Make a table. 7. Since the liquid in a cup is a small amount, the
Choose four values for x. milliliter is the appropriate unit of measure.
Evaluate each value of x for 3x  1. 8. Since the water in a bath tub is a large amount,
the liter is the appropriate unit of measure.
x 3x  1 y (x, y)
9. There are 12 inches in a foot.
1 3(1)  1 4 (1, 4) 120 in.  12  10 ft
0 3(0)  1 1 (0, 1) 10. There are 3 feet in a yard.
18 ft  3  6 yd
1 3(1)  1 2 (1, 2)
11. There are 1000 meters in a kilometer.
2 3(2)  1 5 (2, 5) 10 km  1000  10,000 m
12. There are 10 millimeters in a centimeter.
y 210 mm  10  21 cm
(2, 5)
13. There are 100 centimeters in a meter.
First change millimeters to centimeters.
(1, 2) 180 mm¬ ? cm
180 mm  10¬ 18 cm
O x Then change centimeters to meters.
(0, 1) 18 cm¬ ? m
18 cm  100¬ 0.18 m
( 1, 4) 14. There are 1000 meters in a kilometer.
3100 m  1000  3.1 km
39. Make a table. 15. There are 3 feet in a yard.
Choose four values for x. First change inches to feet.
Evaluate each value of x for 2  x. 90 in.¬ ? ft
90 in.  12¬ 7.5 ft
x 2x y (x, y)
Then change feet to yards.
1 2  (1) 3 (1, 3) 7.5 ft¬ ? yd
7.5 ft  3¬ 2.5 yd
0 20 2 (0, 2)
16. There are 5280 feet in one mile.
1 21 1 (1, 1) First change yards to feet.
5280 yd¬ ? ft
2 22 0 (2, 0)
5280 yd  3¬ 15,840 ft
y Then change feet to miles.
( 1, 3) 15,840 ft¬ ? mi
15,840 ft  5,280¬ 3 mi
(0, 2) (1, 1) 17. There are 3 feet in a yard.
8 yd  3  24 ft
O (2, 0) x
18. There are 1000 meters in a kilometer.
0.62 km  1000  620 m
19. There are 1000 milliliters in a liter.
370 mL  1000  0.370 L
20. There are 1000 milliliters in a liter.
12 L  1000  12,000 mL
21. There are 8 fluid ounces in a cup.
Pages 730–731 Changing Units of Measure
32 fl oz  8  4 c
within Systems
22. There are 2 pints in one quart.
1. Since a tennis ball has a small radius, the
First change quarts to pints.
centimeter is the appropriate unit of measure.
5 qt¬ ? pt
2. Since a notebook has a small length, the 5 qt  2¬ 10 pt
centimeter is the appropriate unit of measure. Then change pints to cups.
3. Since a textbook has a fairly heavy mass, the 10 pt¬ ? c
kilogram is the appropriate unit of measure. 10 pt  2¬ 20 c
4. Since a beach ball has a light mass, the gram is 23. There are 2 pints in a quart.
the appropriate unit of measure. 10 pt  2  5 qt
5. Since a football field has a long width, the meter 24. There are 4 quarts in one gallon.
is the appropriate unit of measure. First change cups to pints.
6. Since a penny has a very small thickness, the 48 c¬ ? pt
millimeter is the appropriate unit of measure. 48 c  2¬ 24 pt

479 Prerequisite Skills


Then change pints to quarts. 6. P  4s
24 pt¬ ? qt  4(5.3)
24  2¬ 12 qt  21.2
Then change quarts to gallons. A  s2
12 qt¬ ? gal  (5.3)2
12 qt  4¬ 3 gal  28.09
25. There are 4 quarts in a gallon. The perimeter is 21.2 meters, and the area is
4 gal  4  16 qt 28.09 square meters.
26. There are 1000 milligrams in a gram. 7. P  2(  w)
36 mg  1000  0.036 g  2(7  11)
27. There are 16 ounces in a pound.  36
13 lb  16  208 oz A  w
 7  11
28. There are 1000 grams in a kilogram.
 77
130 g  1000  0.130 kg
The perimeter is 36 meters, and the area is
29. There are 1000 grams in a kilogram. 77 square meters.
9.05 kg  1000  9050 g
8. P  4s
 4(4.5)
 18
Pages 732–733 Perimeter and Area of A  s2
Rectangles and Squares  (4.5)2
1. P ¬4s  20.25
¬4(11) The perimeter is 18 inches, and the area is
¬44 20.25 square inches.
A ¬s2 9. P  2(  w)
¬112  2(2.4  1.6)
¬121 8
The perimeter is 44 inches, and the area is A  w
121 square inches.  (2.4)(1.6)
2. P  2(  w)  3.84
 2(7.5  3) The perimeter is 8 meters, and the area is
 21 3.84 square meters.
A  w 10. P  4s
 7.5  3  4(6.5)
 22.5  26
The perimeter is 21 kilometers, and the area is A  s2
22.5 square kilometers.  (6.5)2
3. P  4s  42.25
 4(3.5) The perimeter is 26 yards, and the area is
 14 42.25 square yards.
A  s2 11. P  4s
 (3.5)2  4(12)
 12.25  48
The perimeter is 14 yards, and the area is A  s2
12.25 square yards.  122
4. P  2(  w)  144
 2(4  2.5) The perimeter is 48 feet, and the area is
 13 144 square feet.
A  w 12. P  2(  w)
 4  2.5  2(4.2  15.7)
 10  39.8
The perimeter is 13 feet, and the area is A  w
10 square feet.  (4.2)(15.7)
5. P  2(  w)  65.94
 2(5.7  1.8) The perimeter is 39.8 inches, and the area is
 15 65.94 square inches.
A  w 13. P  4s
 5.7  1.8  4(18)
 10.26  72
The perimeter is 15 centimeters, and the area is
10.26 square centimeters.

Prerequisite Skills 480


A  s2 21. 4  6  4  6
 182  4  (6)
 324  2
The perimeter is 72 centimeters, and the area is 22. 7  1  7  1
324 square centimeters. 8
14. P  2(  w) 23. 1  2  1  2
 2(5.3  7) 3
 24.6 24. 2  5  2  5
A  w  2  (5)
 5.3  7  3
 37.1
25. 5  2  |3|
The perimeter is 24.6 feet, and the area is
3
37.1 square feet.
26. 6  4  |10|
15. P  2(  w)
 10
 2(360  121)
 962 27. 3  7  3  (7)
Jansen needs 962 feet of fence.  4
4
16. Find the area of the room and compare it to
105 square feet. 28. 3  3  3  (3)
A  w  6
 11  10 6
 110 29. 36  9  4
Leonardo’s bedroom is 110 square feet. Since 30. 3(7)  21
110  105, the remnant cannot be used to cover 31. 6(4)  24
his bedroom floor. 32. 25  5  5
33. 6(3)  18
34. 7(8)  56
Pages 734–735 Operations with Integers
35. 40  (5)  8
1. 3  3
36. 11(3)  33
2. 4  4
37. 44  (4)  11
3. 0  0
38. 63  (7)  9
4. 5  5
39. 6(5)  30
5. 4  5  4  (5)
 9 40. 7(12)  84
6. 347 41. 10(4)  40
7. 9  5  9  (5) 42. 80  (16)  5
4 43. 72  9  8
8. 2  5  2  (5) 44. 39  3  13
 7
9. 3  5  3  (5)
 2 Page 736 Evaluating Algebraic Expressions
10. 6  11  5 1. 2a  c  2(2)  (1)
11. 4  (4)  8  4  (1)
3
12. 5  9  5  (9)
 4 bd (3)(4)
2. 
2c ¬ 2(1)

13. 3  1  2 1
2
¬
2
14. 4  (2)  6
¬6
15. 2  (8)  2  8 2(4)  2
2d  a
 10 3.  b
 ¬ 3
16. 7  (3)  4 82
¬3
17. 4  (2)  4  2 6
¬
3
 2
18. 3  (3)  3  3 ¬¬2
6 4. 3d  c  3(4)  (1)
19. 3  (4)  1  12  (1)
 12  1
20. 3  (9)  3  9
 13
6

481 Prerequisite Skills


3b 3(3)
5.   ¬
5a  5(2)  3. d  7 ¬8
c (1)
9 d  7  7 ¬8  7
¬ 10  (1)
 
d ¬15
9
¬9 4. 8a ¬6
5
¬1
 a ¬5(6)
5 8
6. 5bc  5(3)(1) 8 5 8
 15(1) a ¬ 15
4
 15 p
5.  2 ¬6
1
7. 2cd  3ab  2(1)(4)  3(2)(3)
12 2  ¬12(6)
p
 2(4)  6(3)
1
 8  (18)
p ¬72
 26 x
1  2(4) 6. 4 ¬8
c  2d
8.  a ¬2 x
1  8 4  4 ¬4  8
¬ 2

x ¬32
9
¬2 12
f ¬18
7. 5
¬9

12  5 f ¬
2 5 12 5
   12 (18)
9. 24  x  4  24  2  4
15
 24  2 f ¬2
 24  2 8.
y
7 ¬11
 26
77 ¬7(11)
y
10. 13  8  y  13  8  (3)
 13  5 y ¬77
 13  5
9. 6y ¬3
 18 7
11. 5  z  11  5  1  11 y ¬7(3)
7 6
6 7 6
 4  11
 4  11 y ¬7
2

 15 10. c  14 ¬11
12. 2y  15  7  2(3)  15  7 c  14  14 ¬11  14
 6  15  7 c ¬3
 21  7 11. t  14 ¬29
 21  7 t  14  14 ¬29  14
 28 t ¬15
13. y  7  3  7 12. p  21 ¬52
37 p  21  21 ¬52  21
 4 p ¬73
14. 11  7  x  11  7  2 13. b  2 ¬5
 11  7  2 b  2  2 ¬5  2
42 b ¬7
6
14. q  10 ¬22
15. x  2z  2  2(1) q  10  10 ¬22  10
 2  2 q ¬12
22
15. 12q ¬84
0 12q 84
 ¬
16. z  y  6  1  (3)  6 12 1
2
 1  3  6 q ¬7
 4  6 16. 5s ¬30
46
5
s 30
5 ¬ 5

 10
s ¬6
17. 5c  7 ¬8c  4
Pages 737–738 Solving Linear Equations 5c  7  7 ¬8c  4  7
1. r  11 ¬3 5c ¬8c  3
r  11  11 ¬3  11 5c  8c ¬8c  3  8c
r ¬8 3c ¬3
3
c 3
3 ¬ 3
2. n  7 ¬13 
n  7  7 ¬13  7 c ¬1
n ¬6

Prerequisite Skills 482


18. 2  6 ¬6  10 27. 2x  4 ¬2
9 3
2  6  6 ¬6  10  6
2
x  4  4 ¬2  4
2 ¬6  16 9 3
2  6 ¬6  16  6 2 1
x ¬ 4
9 3
4 ¬16
92x ¬9
2 3 
4 16 1
4
4 ¬ 4
  2 9
 ¬4 x ¬21
m
19. 0  15 ¬21
1 28. 9  4g ¬15
m 9  4g  9 ¬15  9
0  15  15 ¬21  15
1 4g ¬24
m
0 ¬6
1 4g 2
 ¬ 4
4 4
m
10  0 ¬10  6
1 g ¬6
m ¬60 29. 4  p ¬2
m 4  p  4 ¬2  4
20. 8  7 ¬5
m p ¬2
 8  7  7 ¬5  7
 
p ¬2
m
8 ¬2 30. 21  b ¬11
21  b  21 ¬11  21
8m
8 ¬8(2) b ¬10
m ¬16 b ¬10
21. 8t  1 ¬3t  19 31. 2(n  7) ¬15
8t  1  1 ¬3t  19  1 2n  14 ¬15
8t ¬3t  20 2n  14  14 ¬15  14
8t  3t ¬3t  20  3t 2n ¬29
5t ¬20 2
n 29
2 ¬ 2
 
20
5t ¬
5 5 n ¬29

2
t ¬4
32. 5(m  1) ¬25
22. 9n  4 ¬5n  18 5m  5 ¬25
9n  4  4 ¬5n  18  4 5m  5  5 ¬25  5
9n ¬5n  14 5m ¬20
9n  5n ¬5n  14  5n 5m 20
5 ¬ 5
   
4n ¬14
4n 14 m ¬4
4 ¬ 4
  
33. 8a  11 ¬37
7 8a  11  11 ¬37  11
n ¬2
8a ¬48
23. 5c  24 ¬4
8
a 48
8 ¬ 8
5c  24  24 ¬4  24   
5c ¬20 a ¬6
5 c 2
0
5 ¬ 5
 
34. 7q  2 ¬5
4
c ¬4
7q  2  2 ¬5  2
24. 3n  7 ¬28 4
3n  7  7 ¬28  7 7q ¬3
4
3n ¬21
3n 21 q ¬43
4 7
3 ¬ 3
   7 4 7
n ¬7 q ¬ 12

7
25. 2y  17 ¬13
35. 2(5  n) ¬8
2y  17  17 ¬13  17
10  2n ¬8
2y ¬30
2 y 3
0 10  2n  10 ¬8  10
2 ¬ 2
  2n ¬2
y ¬15 2
n 
2
2 ¬ 2
 
t
26. 3  2 ¬3
1 n ¬1
t 3(d  7) ¬6
3  2  2 ¬3  2
1
36.
t 3d  21 ¬6
13 ¬5 3d  21  21 ¬6  21
13 3d ¬15
3  ¬13(5)
t
1 3
d 15
3 ¬ 3
 
t ¬65
d ¬5

483 Prerequisite Skills


Pages 739–740 Solving Inequalities in One 11. 3z  8 ¬2
Variable 3z  8  8 ¬2  8
1. x  7 ¬6 3z ¬6
3z 6
3 ¬ 3
x  7  7 ¬6  7  
x ¬13 z ¬2
The solution set is {xx 13}. The solution set is {zz 2}.
2. 4c  23
¬13 12. a  7 ¬5
4c  23  23
¬13  23 a  7  7 ¬5  7
4c
¬36 a ¬12
4
c 36 The solution set is {aa 12}.
4
¬ 4

c
¬9 13. m  21 ¬8
The solution set is {cc
9}. m  21  21 ¬8  21
3.
p
5 ¬14 m ¬29
The solution set is {mm 29}.
55
¬5(14)
p
14. x  6 ¬3
p
¬70 x  6  6 ¬3  6
The solution set is {pp
70}. x ¬9
The solution set is {xx 9}.
4. a8 ¬5

15. 3b
¬48
8a 3
8  ¬8(5)
 b 48
3 ¬ 3
  

a ¬40 b ¬16
The solution set is {aa  40}. The solution set is {bb 16}.
5. t ¬7 16. 4y ¬20
6 4y 20
 ¬
66t ¬6(7) 4 4
y ¬5
t ¬42 The solution set is {yy 5}.
The solution set is {tt  42}. 17. 12k ¬36
a
1
¬8
 3
6. 1
12k 6
12 ¬ 12
 
11a
11 
¬11(8)
k ¬3
The solution set is {kk 3}.
a
¬88
18. 4h ¬36
The solution set is {aa
88}. 4
h 36
4 ¬ 4
  
7. d  8
¬12
d  8  8
¬12  8 h ¬9
d
¬4 The solution set is {hh 9}.
The solution set is {dd
4}. 19. 2b  6
¬2
5
8. m  14 ¬10 2b  6  6
¬2  6
m  14  14 ¬10  14 5
m ¬4 2b
¬4
5
The solution set is {mm  4}.
52b
¬5(4)
9. 2z  9 ¬7z  1 2 5 2
2z  9  9 ¬7z  1  9 b
¬10
2z ¬7z  10 The solution set is {bb
10}.
2z  7z ¬7z  10  7z 20. 8t  1 ¬5
3
5z ¬10
5
z 10 8t  1  1 ¬5  1
5 ¬ 5
   3
z ¬2 8t ¬6
3
The solution set is {zz  2}.
t ¬3(6)
3 8
10. 6t  10 ¬4t 8 3 8
6t  10  10 ¬4t  10 t ¬9
4
6t ¬4t  10
6t  4t ¬4t  10  4t The solution set is tt  9
4 .

2t ¬10
2t ¬
10
2 2
t ¬5
The solution set is {tt 5}.

Prerequisite Skills 484


21. 7q  3 ¬4q  25 2. 2x  5y  10
7q  3  3 ¬4q  25  3 To find the x-intercepts, let y  0
7q ¬4q  22 2x  5y ¬10
7q  4q ¬4q  22  4q 2x  5(0)  10
11q ¬22 2x ¬10
11q 22
 ¬
x ¬5
11 11 To find the y-intercept, let x  0
q ¬2 2x  5y ¬10
The solution set is {qq 2}. 2(0)  5y ¬10
22. 3n  8 ¬2n  7 5y ¬10
3n  8  8 ¬2n  7  8 y ¬2
3n ¬2n  15 Put a point on the x-axis at 5 and a point on the
3n  2n ¬2n  15  2n y-axis at 2. Draw a line through the two points.
5n ¬15
y
5
n 15
5 ¬ 5
  
n ¬3 2x 5y 10
The solution set is {nn 3}.
(0, 2)
23. 3w  1
¬8
x
3w  1  1
¬8  1
O (5, 0)
3w
¬7
3
w 7
3 ¬ 3


w ¬7
3

3. 3x  y  3
The solution set is ww 7
3 .
 To find the x-intercept, let y  0.
4 3x  y ¬3
5 k  17 ¬11
24. 
3x  0 ¬3
4
5 k  17  17 ¬11 17
 3x ¬3
4 x ¬1
5 k ¬28

To find the y-intercept, let x  0.
4  5 k ¬ 4 (28)
5
 4 5 3x  y ¬3
3(0)  y ¬3
k ¬35
y ¬3
The solution set is {kk 35}.
y ¬3
Put a point on the x-axis at 1 and a point on the
y-axis at 3. Draw a line through the two points.
Page 741 Graphing Using Intercepts and Slope
y
1. 2x  3y  6
To find the x-intercept, let y  0.
2x  3y ¬6
2x  3(0) ¬6 O (1, 0) x
2x ¬6
x ¬3 (0, 3)
To find the y-intercept, let x  0. 3x y 3
2x  3y ¬6
2(0)  3y ¬6
3y ¬6 4. x  2y  2
y ¬2 To find the x-intercept, let y  0.
Put a point on the x-axis at 3 and a point on the x  2y ¬2
y-axis at 2. Draw a line through the two points. x  2(0) ¬2
y x ¬2
x ¬2
2x 3y 6 To find the y-intercept, let x  0.
x  2y  2
(0, 2) 0  2y  2
( 3, 0)
2y  2
O x y1

485 Prerequisite Skills


Put a point on the x-axis at 2 and a point on the 7. y  x  2
y-axis at 1. Draw a line through the two points. The y-intercept is 2. So, plot a point at (0, 2). The
y
slope is 1. From (0, 2), move down 1 unit and
right 1 unit. Plot a point. Draw a line connecting
x 2y 2 the points.
y
( 2, 0) (0, 1) x
O
(0, 2) (1, 1)

O x

5. 3x  4y  12
y x 2
To find the x-intercept, let y  0.
3x  4y ¬12
3x  4(0) ¬12 8. y  x  2
3x ¬12 The y-intercept is 2. So, plot a point at (0, 2).
x ¬4 The slope is 1. From (0, 2), move up 1 unit and
To find the y-intercept, let x  0. right 1 unit. Plot a point. Draw a line connecting
3x  4y ¬12 the points.
3(0)  4y ¬12 y
4y ¬12
y ¬3 y x 2
Put a point on the x-axis at 4 and a point on the
y-axis at 3. Draw a line through the two points.
y O x
(0, 2) (1, 1)
3x 4y 12
(0, 3)

x 9. y  x  1
O (4, 0) The y-intercept is 1. So, plot a point at (0, 1). The
slope is 1. From (0, 1), move up 1 unit and right
1 unit. Plot a point. Draw a line connecting the
points.
6. 4y  x  4 y
To find the x-intercept, let y  0. y x 1
4y  x ¬4
4(0)  x ¬4
x ¬4
To find the y-intercept, let x  0. (0, 1) (1, 2)
4y  x ¬4
4y  0 ¬4 O x
4y ¬4
y ¬1
Put a point on the x-axis at 4 and a point on the 10. y  3x  1
y-axis at 1. Draw a line through the two points. The y-intercept is 1. So, plot a point at (0, 1).
y The slope is 3. From (0, 1), move up 3 units and
right 1 unit. Plot a point. Draw a line connecting
the points.
y
(0, 1) 4y x 4
x
O (4, 0)
y 3x 1
(1, 2)

O x
(0, 1)

Prerequisite Skills 486


11. y  2x  3 15. y  2x  2
The y-intercept is 3. So, plot a point at (0, 3). The To find the x-intercept, let y  0.
slope is 2. From (0, 3), move down 2 units and y ¬2x  2
right 1 unit. Plot a point. Draw a line connecting 0 ¬2x  2
the points. 2x ¬2
y
x ¬1
To find the y-intercept, let x  0.
y 2x 3
(0, 3) y  2x  2
y  2(0)  2
(1, 1) y02
O
(2, 1)
x y  2
Put a point on the x-axis at 1 and a point on the
y-axis at 2. Draw a line through the two points.
y

12. y  3x  1
The y-intercept is 1. So, plot a point at (0, 1). y 2x 2
The slope is 3. From (0, 1), move down 3 units
and right 1 unit. Plot a point. Draw a line O (1, 0) x
connecting the points.
(0, 2)
y

O x 16. 6x  y  2
(0, 1) y 3x 1 To find the x-intercept, let y  0.
6x  y ¬2
6x  0 ¬2
(1, 4) 6x ¬2
x ¬1
3

To find the y-intercept, let x  0.
13. y  2
3x  3
 6x  y ¬2
The y-intercept is 3. So, plot a point at (0, 3). 6(0)  y ¬2
The slope is 2
3 . From (0, 3), move up 2 units and
 y ¬2
right 3 units. Plot a point. Draw a line connecting Put a point on the x-axis at 1 
3 and a point on
the points. the y-axis at 2. Draw a line through the two
y points.
y
2–
y 3
x 3
(0, 2)

O x –1, 0
3
(3, 1) O x
6x y 2
(0, 3)

14. y  1
2x  1

The y-intercept is 1. So, plot a point at (0, 1). 17. 2y  x  2
To find the x-intercept, let y  0.
The slope is 1
2 . From (0, 1), move up 1 unit and

2y  x ¬2
right 2 units. Plot a point. Draw a line connecting 2(0)  x ¬2
the points. x ¬2
y x ¬2
y 1–
x 1 To find the y-intercept, let x  0.
2
2y  x ¬2
2y  0 ¬2
(2, 0)
2y ¬2
O x
y ¬1
(0, 1)

487 Prerequisite Skills


Put a point on the x-axis at 2 and a point on the 20. 4x  y  4
y-axis at 1. Draw a line through the two points. To find the x-intercept, let y  0.
y 4x  y ¬4
4x  0 ¬4
4x ¬4
(2, 0) x ¬1
O x To find the y-intercept, let x  0.
(0, 1) 4x  y ¬4
4(0)  y ¬4
2y x 2
y ¬4
Put a point on the x-axis at 1 and a point on the
y-axis at 4. Draw a line through the two points.
18. 3x  4y  12 y
To find the x-intercept, let y  0.
3x  4y ¬12
3x  4(0) ¬12 (0, 4)
3x ¬12 4x y 4
x ¬4
To find the y-intercept, let x  0.
3x  4y ¬12 O (1, 0) x
3(0)  4y ¬12
4y ¬12
y ¬3 21. y  2x  3

2
Put a point on the x-axis at 4 and a point on the
2 . So, plot a point at 0,  2 .
The y-intercept is 3
 3
y-axis at 3. Draw a line through the two points.
y The slope is 2. From 0, 3
2 , move up 2 units and

right 1 unit. Plot a point. Draw a line connecting
the points.
( 4, 0)
O x y
3–
y 2x 2
(0, 3)

3x 4y 12 1,–2 1
O x
0, –3
19. 4x  3y  6 2

To find the x-intercept, let y  0.


4x  3y ¬6
4x  3(0) ¬6
4x ¬6
x ¬3

2 Pages 742–743 Solving Systems of Linear
To find the y-intercept, let x  0. Equations
4x  3y ¬6
1.
4(0)  3y ¬6 y
3y ¬6 y  x  2
y ¬2
Put a point on the x-axis at 3
2 and a point on the (2, 0)
y-axis at 2. Draw a line through the two points. 1 x
y 2x 1
y

O –,3 0 x The graphs appear to intersect at (2, 0).


2
4x 3y 6 Check this estimate by replacing x with 2 and y
(0, 2)
with 0 in each equation.
1
Check: y ¬x  2 y ¬2x  1
1
0 ¬2  2 0 ¬2(2)  1
0 ¬0 ✓ 0 ¬0 ✓
The system has one solution at (2, 0).

Prerequisite Skills 488


2. y 6. y

(2, 3)
yx1
x
(0, 1)
x
y  3x  3 y  3x  1
3y  x = 3

The graphs appear to intersect at (2, 3).


Check this estimate by replacing x with 2 and y The graphs appear to intersect at (0, 1).
with 3 in each equation. Check this estimate by replacing x with 0 and y
Check: y ¬3x  3 y ¬x  1 with 1 in each equation.
3 ¬3(2)  3 3 ¬2  1 Check: 3y  x ¬3 y  3x ¬1
3 ¬3 ✓ 3 ¬3 ✓ 3(1)  0 ¬3 1  3(0) ¬1
The system has one solution at (2, 3). 3 ¬3 ✓ 1 ¬1 ✓
3. The system has one solution at (0, 1).
y
7. Since x  2y  8, then x  8  2y.
Substitute 8  2y for x in the first equation.
y  2x 1 5x  3y ¬12
5(8  2y)  3y ¬12
x 40  10y  3y ¬12
2y  4x 1 40  13y ¬12
40  13y  40 ¬12  40
13y ¬52
13y 52
 ¬
13 13
The graphs of the equations are parallel lines. y ¬4
Since they do not intersect, there are no solutions Use x  8  2y to find the value of x.
of this system of equations. x ¬8  2y
4. y x ¬8  2(4)
6x 12y  6 x ¬8  8
x ¬0
The solution is (0, 4).
8. Since x  4y  22, then x  4y  22.
x
Substitute 4y  22 for x in the second equation.
2x  4y  2
2x  5y ¬21
2(4y  22)  5y ¬21
8y  44  5y ¬21
13y  44 ¬21
The graphs of the equations are the same line. 13y  44  44 ¬21  44
There are infinitely many solutions. 13y ¬65
5. 13y 6
5
y  ¬
13 13
y ¬5
4x  3y  12 Use x  4y  22 to find the value of x.
(3, 0) x  4y  22
x x  4(5)  22
3x  y  9 x  20  22
x2
The solution is (2, 5).
9. Since y  5x  3, then y  5x  3.
The graphs appear to intersect at (3, 0). Substitute 5x  3 for y in the second equation.
Check this estimate by replacing x with 3 and y 3y  2x ¬8
with 0 in each equation. 3(5x  3)  2x ¬8
Check: 4x  3y ¬12 3x  y ¬9 15x  9  2x ¬8
4(3)  3(0) ¬12 3(3)  0 ¬9 17x  9 ¬8
12 ¬12 ✓ 9 ¬9 ✓ 17x  9  9 ¬8  9
The system has one solution at (3, 0). 17x ¬17
1
7x 17
17 ¬ 17
  
x ¬1

489 Prerequisite Skills


Use y  5x  3 to find the value of y. Now substitute 3 for y in either equation to find
y  5x  3 the value of x.
y  5(1)  3 3x  2y ¬2
y53 3x  2(3) ¬2
y2 3x  6 ¬2
The solution is (1, 2). 3x  6  6 ¬2  6
10. Since y  2x  2, then y  2x  2. 3x ¬4
Substitute 2x  2 for y in the second equation. 3x 4
3 ¬ 3
 
7y  4x ¬23
x ¬4
7(2x  2)  4x ¬23 3
14x  14  4x ¬23 The solution is (4

3 , 3).
18x  14 ¬23
14. Add the equations to eliminate y.
18x  14  14 ¬23  14
3x  4y ¬1
18x ¬9
9x  4y ¬ 13
18
x 9 
18 ¬ 18

6x ¬ 12
x ¬1
  6x
 ¬ 12

2 6 6
Use y  2x  2 to find the value of y. x ¬2
y  2x  2 Now substitute 2 for x in either equation to find
y  21
2  2
 the value of y.
y12 3y  4y ¬1
y3 3(2)  4y ¬1
6  4y ¬1
The solution is 1
2 , 3.

6  4y  6 ¬1  6
11. Since x  2y  5, then x  2y  5. 4y ¬5
Substitute 2y  5 for x in the first equation. 4y
 ¬5
2x  3y ¬8 4 4
2(2y  5)  3y ¬8 y ¬5

4
4y  10  3y ¬8
The solution is 2, 5
4 .

y  10 ¬8
y  10  10 ¬8  10 15. Multiply the second equation by 2. Then add the
y ¬2 equations to eliminate x.
Use x  2y  5 to find the value of x. 4x  5y ¬11
x  2y  5 4x  6y ¬22

x  2(2)  5 11y ¬11
x45 11y 11
 ¬
x  1 11 11
The solution is (1, 2). y ¬1
12. Since 3x  y  10, then y  3x  10. Now substitute 1 for y in either equation to find
Substitute 3x  10 for y in the first equation. the value of x.
4x  2y ¬5 2x  3y ¬11
4x  2(3x  10) ¬5 2x  3(1) ¬11
4x  6x  20 ¬5 2x  3 ¬11
10x  20 ¬5 2x  3  3 ¬11  3
10x  20  20 ¬5  20 2x ¬8
2x 8
2 ¬ 2
10x ¬25 
10
x 25 x ¬4
10 ¬ 10
  
5 The solution is (4, 1).
x ¬2
16. Multiply the second equation by 3. Then add the
Use y  3x  10 to find the value of y.
equations to eliminate x.
y  3x  10
6x  25y ¬21
y  35
2   10
  6x  27y ¬21

y  5

2 22y ¬22
22y
The solution is 5
2 ,  2 .
 5 22
 ¬
22 22
13. Add the equations to eliminate x. y ¬1
3x  2y ¬7
3x  2y ¬2

3y ¬9
3y
 ¬9
3 3
y ¬3

Prerequisite Skills 490


Now substitute 1 for y in either equation to find Use y  4x  11 to find the value of y.
the value of x. y  4x  11
6x  5y ¬1 y  4(3)  11
6x  5(1) ¬1 y  12  11
6x  5 ¬1 y1
6x  5  5 ¬1  5 The solution is (3, 1).
6x ¬6 20. Solve by elimination.
6x 6
6 ¬ 6
 Multiply the first equation by 5 and the second
x ¬1 equation by 2. Then add the equations.
The solution is (1, 1). 20x  30y ¬ 15
20x  30y ¬8
17. Multiply the first equation by 3 and the second 
equation by 2. Then add the equations to 0 ¬ 7
eliminate y. Since this is false, there is no solution.
9x  6y ¬24 21. Solve by graphing.
10x  6y ¬32
 y
x ¬38
Now substitute 8 for x in either equation to find (4, 3)
the value of y.
5x  5y  5
3x  2y ¬8
3(8)  2y ¬8
24  2y ¬8 x
24  2y  24 ¬8  24 3x  2y  6
2y ¬16
2y 1
6
 ¬
2 2 The graphs appear to intersect at (4, 3).
y ¬8 Check this estimate by replacing x with 4 and y
The solution is (8, 8). with 3 in each equation.
18. Multiply the first equation by 3 and the second Check: 3x  2y ¬6 5x  5y ¬5
equation by 4. Then add the equations to 3(4)  2(3) ¬6 5(4)  5(3) ¬5
eliminate x. 12  6 ¬6 20  15 ¬5
12x  21y ¬51 6 ¬6 ✓ 5 ¬5 ✓
12x  28y ¬12 The system has one solution at (4, 3).

13y ¬39 22. Solve by elimination or substitution.
13y 39 Since 3y  x  3, then x  3  3y.
 ¬
13 1
3 Substitute 3  3y for x in the second equation.
y ¬3
2y  5x ¬15
Now substitute 3 for y in either equation to find 2y  5(3  3y) ¬15
the value of x. 2y  15  15y ¬15
4x  7y ¬17 15  17y ¬15
4x  7(3) ¬17 15  17y  15 ¬15  15
4x  21 ¬17 17y ¬0
4x  21  21 ¬17  21 17y 0
4x ¬4  ¬
17 17
4 x 4 y ¬0
4 ¬ 4

Use x  3  3y to find the value of x.
x ¬1
x  3  3y
The solution is (1, 3).
x  3  3(0)
19. Solve by elimination or substitution. x30
Since 4x  y  11, then y  4x  11. x3
Substitute 4x  11 for y in the second equation. The solution is (3, 0).
2x  3y ¬3
23. Solve by elimination.
2x  3(4x  11) ¬3
Multiply the second equation by 2 and add the
2x  12x  33 ¬3
equations.
10x  33 ¬3
4x  7y ¬ 8
10x  33  33 ¬3  33
4x  10y ¬2
10x ¬30 
1
0x 3
0 3y ¬ 6
10 ¬ 10
 
3y
 ¬6
x ¬3 3 3
y ¬2

491 Prerequisite Skills


Substitute 2 for y in either equation to find the 7. 98x
  ¬
3y6 2  7 
7  x3
 y6
value of x.
¬2
  7
2  x3  
y6
4x  7y ¬8
4x  7(2) ¬8 ¬2
  7  x  x  y3
4x  14 ¬8 ¬7xy32x

4x  14  14 ¬8  14 8. 56a
 2b
4c5 ¬
2  2 
2  7 
a2  b
4  c5
4x ¬22
4x 22 ¬2
2  14
  
a2  b
4  
c5
4 ¬ 4
  
¬2  14
  a  b2  c2  c
1 1
x ¬2 ¬2ab2c214c

The solution is  
11
2 ,2 9. 
81 8
49 ¬
 
1
4
9
24. Solve by elimination or substitution.
¬9

Since x  3y  6, then x  6  3y. 7


Substitute 6  3y for x in the second equation. 121 1
21
10. 16 ¬
1

4x  2y ¬32 6
11
4(6  3y)  2y ¬32 ¬
4
24  12y  2y ¬32
 
63 63
8 ¬
11.  
24  14y ¬32 
8
24  14y  24 ¬32  24 3
  3 
7
14y ¬56 ¬
2
  2 
2
14y 5
6
 ¬ 37
¬ 
14 14
y ¬4 22
37 2
Use x  6  3y to find the value of x. ¬   
2 2  2
x  6  3y 3 
14
x  6  3(4) ¬ 4
x  6  12
x  6 12. 
28
8 
147 ¬
288

1
47
The solution is (6, 4).
144  2

 49  3
122
Pages 744–745 Square Roots and Simplifying ¬
73
Radicals 122 3
¬  
1. 32
 ¬
2  2 
2  2 
2 73  3
¬
42  2
 12 6 4
6
¬7 
3 or 7
¬42
 p 2 
1
0p3 10p
2. 75
 ¬
5  5 
3 13.  ¬

27 9 3
¬5
2  3
 p10p
¬
¬53
 33
3. 50
  10
 ¬ 
50  10 p10p 33
¬¬  
3
3 33
¬
2  5 
5  2 
5
p30p

¬
22  5
2  5
 ¬9
¬2  5  5
 
108 
36  3
14.   ¬
¬105
 2q6 q
6
2
4. 12
  20
 ¬ 
12  20 ¬ 

6 3
3 q 2

¬
2  2 
3  2 
25
  
6 32 
¬
42  15
 ¬ 
3 q 2
 
2
¬415
 6  or 
6 3 
6
¬  
5. 6
  6
 ¬
66 3 q   2
3 q 
¬36
 or 6 4 4 5  2 
3
15.  ¬  
5  2
3 5  23
 5  2 3

6. 16
  25
 ¬ 
16  25
45  23
¬
4  4 
55 ¬  2
52  23

¬4 2  5
2
¬4  5 or 20 83
20   
¬
25  12
20 83
¬
13

Prerequisite Skills 492


  5  26
 5. (fg8)(15f 2g)  (1)(15)(f  f 2)(g8  g)
16.  ¬  
7 3 7 3
5  26
 5  26
 5  26  15(f 12)(g81)
735  26   15f 3g9
¬  2 6. (6j k )(j k)  (6)(1)(j4  j2)(k4  k)
4 4 2
52  26
 6(j42)(k41)
35 1418
3    6j6k5
¬
25  24
7. (2ab )(4a b2)  (2)(4)(a  a2)(b3  b2)
3 2
35
¬    14
3 92
 8(a12)(b32)
1
 8a3b5
¬353
  422

8. 8
5 x y(4x y ) ¬ 5 (4)(x  x )(y  y )
 3 3 2 8 3 3 2
3 3
17.  ¬
48
 163 32
¬  33)(y12)
3 5 (x
¬
43 32
¬  6 3
3 5 x y
3 
¬   9. 2q2(q2  3)  2q2(q2)  2q2(3)
43  3
3 or 
3 
3  2q4  6q2
¬
43 4 10. 5p(p  18)  5p(p)  5p(18)


2 4 
46  5p2  90p
18.  ¬

125 25  5 11. 15c(3c2  2c  5)  15c(3c2)  15c(2c)  15c(5)
26  45c3  30c2  75c
¬
55
12. 8x(4x2  x  11)  8x(4x2)  8x(x)  8x(11)
26 5
¬     32x3  8x2  88x
5 
5 5
2
30 13. 4m2(2m2  7m  5)
¬
25 ¬4m2(2m2)  4m2(7m)  4m2(5)
  2  2
 ¬8m4  28m3  20m2
19.  ¬  
3 5 3 5
2  2
 2  2
 2  2
 14. 8y2(5y3  2y  1)  8y2(5y3)  8y2(2y)  8y2(1)
352  2   40y5  16y3  8y2
¬ 
  ¬32 (m ) (n )
2 2
2  
2 22 15. 3
 3 2
2 m n
3 2 2 2

65
 
310

¬
42
¬9  6 4
4m n
6 
5  310
 16. (2c d )  (2)2(c3)2(d2)2
3 2 2
¬  4c6d4
2
3 3 2  1
 3 17. (5wx )  (5)3(w)3(x5)3
5 3
20.  ¬  
2  13
 2  13
 2  
13  125w3x15
32  13
 18. (6a b)  63(a5)3b3
5 3

¬   216a15b3
(2)2  13
2
19. (k ) (13k2)2  (k2)3(3)(132)(k2)2
2 3
313
6     k63(169)k4
¬
4  13
 169k6  k4  3
32  13
  169k103
¬ 
9
20. (5w x ) (2w5)2  (5)2(w3)2(x2)2(2)2(w5)2
3 2 2

2 
13  25w6x4(4)w10
¬ 
3  (25)(4)w6  w10  x4
 100w16x4
21. (7y z )(4y )  (7)y3z2(4)4(y2)4
3 2 2 4

Pages 746–747 Multiplying Polynomials  (7)y3z2(256)y8


1. (3q2)(q5)  (3)(1)(q2  q5)  (7)(256)y3  y8  z2
 (3)(1)(q25)  1792 y11z2
 3q7
  
2 2
22. 1 3 3 1
2 p q (4pq ) ¬ 2 (p ) (q ) (4) (p) (q )
 2 2 2 2 2 2 3 3 3 3
2. (5m)(4m3)  (5)(4)(m  m3)
 (5)(4)(m13) ¬1
 4 4 3 9
4 p q (64) p q
 20m4
¬1
4  64  p  p  q  q
 4 3 4 9

 
3. 9  
9
2 c (8c ) ¬ 2 (8)(c  c )
 5 5
¬16p7q13
 
¬ 9
2 (8)(c
15) 23. (m  1)(m  4)
 (m)(m)  (m)(4)  (1)(m)  (1)(4)
¬36c6
 m2  4m  m  4
4. (n6)(10n2)  (1)(10)(n6  n2)  m2  5m  4
 10(n62)
 10n8

493 Prerequisite Skills


24. (s  7)(s  2) Pages 748–749 Dividing Polynomials
 (s)(s)  (s)(2)  (7)(s)  (7)(2)
  
ac 2 2 2 2
a c
 s2  2s  7s  14 2a ¬ a
1.  2
 s2  9s  14
¬(a )c2
2
21
25. (x  3)(x  4)  (x)(x)  (x)(4)  (3)(x)  (3)(4)
2
 x2  4x  3x  12 ac
¬2
 x2  x  12
5q5r3
 q  
5 3
26. (a  3)(a  6)  (a)(a)  (a)( 6)  3(a)  (3)(6) 2.   ¬5  r
q2r2 q2 r2
 a2  6a  3a  18 ¬5(q52)(r32)
 a2  3a  18
¬5q3r
27. (5d  3)(d  4)
b2d
  
5 2 5
 (5d)(d)  (5d)(4)  (3)(d)  (3)(4) 3.   ¬1 b d
8b2d3 8 b 2 d3
 5d2  20d  3d  12
 5d2  17d  12 ¬1
8 (b
 2(2))(d53)
b4d2
28. (q  2)(3q  5)  (q)(3q)  (q)(5)  (2)(3q)  (2)(5) ¬1
 4 2
8 b d or 8
 
 3q2  5q  6q  10
 3q2  11q  10
5p3x
4.   ¬5 
2p7
p3
2 p 7 x  
29. (2q  3)(5q  2) ¬5 3(7))x
 (2q)(5q)  (2q)(2)  (3)(5q)  (3)(2) 2 (p
 10q2  4q  15q  6 ¬5 4
2p x
 10q2  19q  6
   
3
3r  2
st 4 r3 s2 4
30. (2a  3)(2a  5) 5. 
2r2st3
¬3  
2 r2
s t
t 3
 (2a)(2a)  (2a)(5)  (3)(2a)  (3)(5) ¬3 32)(s21)(t4(3))
2 (r
 4a2  10a  6a  15
 4a2  16a  15 ¬3 5 1 7
2r s t
31. (d  1)(d  1)  (d)2  (1)2 3st7
¬
2r5
 d2  1
3x3y1z5
  y  
3 1 5
32. (4a  3)(4a  3)  (4a)2  (3)2 6.   ¬3 x y z
xyz2 x z2
 16a2  9
¬3(x31)(y11)(z52)
33. (s  5)  (s)  2(s)(5)  (5)2
2 2

 s2  10s  25 ¬3x2y2z3
3x z 2 3
34. (3f  g)  (3f )2  2(3f )(g)  (g)2
2
¬ 
2
 9f 2  6fg  g2 y

 
w 4 3 (w) 4 3
35. (2r  5)  (2r)2  2(2r)(5)  (5)2
2
7. 6 ¬ 3
6
 4r2  20r  25 w12
¬
   
2 2 216
36. t  8

3 ¬(t)2  2(t) 8 8
3  3

1
¬t2   6 64
3 t 9
 
8. 3q2 3
5 ¬  (3)3(q2)3
53
27q6
37. (x  4)(x2  5x  2) ¬ 
125
 x(x2  5x  2)  4(x2  5x  2)
2y

2 2 (2)2(y2)2
 x3  5x2  2x  4x2  20x  8 9. 7 ¬
72
 x3  x2  22x  8 4y4
¬49
38. (x 2)(x2  3x  7)
 
2 4 4 2)4
 x(x2  3x  7)  2(x2  3x  7) 5m
  ¬ 5 (m 
10. 3 34
 x3  3x2  7x  2x2  6x  14
625m8
 x3  x2  13x  14 ¬8 1
39. (3b  2)(3b2  b  1) 4z2  16z  36
¬
2
4z z
16 3 6
4z  4z  4z
11.   
 3b(3b2  b  1)  2(3b2  b  1) 4z
9
 9b3  3b2  3b  6b2  2b  2 ¬z  4  z
 9b3  3b2  b  2 5d2  8 d  20
12. (5d2  8d  20)  10d ¬
40. (2j  7)(j2  2j  4) 10d
5d2 8d 20
 2j(j2  2j  4)  7(j2  2j  4) ¬ 10d  10d  10d
   
 2j3  4j2  8j  7j2  14j  28
¬d 4 2
2  5  d

 2j3  3j2  6j  28
p3  12p2  3p  8
13. (p3  12p2  3p  8)  4p ¬ 
4p
p3 12p2 3p 8
¬       
4p
4p 4p 4p
p2
¬  3p  3 4  p
 2
4

Prerequisite Skills 494


b  4b  10
3 2
14. (b3  4b2  10)  2b ¬ 2b
24. d2  4d  12
b3 10 4b2 d  2
d
3
d
22
d
44
2
¬
2b  2b  2b
   3  2d2
().d
2 2
¬b2  2b  5
 4d  4d
b
() 4d2  8d
a3  6a2  4a  3 ¬a
3 6a2 4a 
15.  2 2 
 2 
 3
2  2 12d  24
a a aa a
() 12d  24
¬a  6  a  32
4 
a 0
8x2y  10xy2  6x3 8x2y 10xy2 6x3
16.  ¬     Therefore, (d3  2d2  4d  24)  (d  2)
2x2 2x2 2x2 2x2
 d2  4d  12.
5y2
¬4y    3x 25. 2j2  4j  13
x
s2  2s 8 (s  4)(s  2) j  2
2

j3j
02j
56
2
17.  s4
 ¬
s4

() 2j3  4j2
(s  4)(s  2) 
¬ 
(s  4) 4j2  5j
¬(s  2) () 4j2  8j

18. (r2 r  9r
 9r  20)  (r  5) ¬  202 13j  26
(r  5) () 13j  26
(r  5)(r
¬   4) 
(r  5) 0
(r  5)(r  4) Therefore, (2j3  5j  26)  (j  2)  2j2  4j  13.
¬ 
(r  5)
¬r  4 26. 2x2  11x  44
t2  7t  12 x x3
4
2 x
32
x
0
7
16

19. (t2  7t  12)  (t  3) ¬
(t 
3) 3  8x2
().2x
(t  3)(t  4) 
¬ (t 3) 11x2  0x
(t  3)(t  4) () 11x2  44x
¬ 
(t  3)

44x  176
¬t  4 () 44x  176
c  3c 54
2 
20. (c2  3c  54)  (c  9) ¬ (c  9) 0
(c  9)(c  6) 2x  3x
3  176
2
¬ Therefore,   2x2  11x  44.
(c 9) x4
(c  9)(c  6) 27. x2
¬ (c 9)
¬c  6 x  4
x2x
63
(). x 2  4x
2q2  9q  5 
21. (2q2  9q  5)  (q  5) ¬ 
(q  5) 2x  3
(q  5)(2q  1) () 2x  8
¬  
(q  5) 11
(q  5)(2q  1) Therefore, (x2  6x  3)  (x  4)  x  2   11
¬ 
(q  5) x  4.
28. h2  4h  2
¬2q  1
h  2
h
3
h
22
h61
3z2  2z 5 (z  1)(3z  5) 3  2h2
22.  
z1 ¬ 
(z  1)
().h
2
(z  1)(3z  5) 4h  6h
¬ 
(z  1) () 4h2  8h

¬3z  5 2h  1
23. m2  4m  1 () 2h  4

m  1
m
3
m
32
m
51 5
().m 3  m2
2 h3  2h2  6h  1  h2  4h  2  5
Therefore, 
4m  5m h2 h2
() 4m2  4m

m  1
() m  1 Pages 750–751 Factoring to Solve Equations

0 1. Factor u2  12u.
(m3  3m2  5m  1) u2  u  u, 12u 2  2  3  u
Therefore,  m1  m2  4m  1. GCF: u
u2  12u  u  u  u  12
 u(u  12)

495 Prerequisite Skills


2. Factor w2  4w. 10. x2  14x  48
w2  w  w, 4w  2  2  w In this equation, b is 14 and c is 48. Find two
GCF: w numbers with a product of 48 and with
w2  4w  w  w  w  4 a sum of 14.
 w(w  4) Factors of 48 Sum of Factors
3. Factor 7j  28j.
2
1, 48 49
7j2  7  j  j, 28j  2  2  7  j
2, 24 26
GCF: 7  j or 7j
3, 16 19
7j2  28j  7j  j  7j  4
 7j(j 4) 4, 12 16
6, 8 14
4. Factor 2g2  24g.
2g2  2  g  g, 24g  2  2  2  3  g The correct factors are 6 and 8; m  6, n  8.
GCF: 2  g or 2g x2  14x  48  (x  m)(x  n)
2g2  24g  2g  g  2g  12  (x  6)(x  8)
 2g(g  12) 11. m2  6m  7
5. Factor 6x2  2x. In this equation, b is 6 and c is 7. Find two
6x2  2  3  x  x, 2x  2  x numbers with a product of 7 and with a sum of 6.
GCF: 2  x or 2x Factors of 7 Sum of Factors
6x2  2x  2x  3x  2x  1
 2x(3x  1) 1, 7 6
6. Factor 5t2  30t. 1, 7 6
5t2  5  t  t, 30t  2  3  5  t
The correct factors are 1 and 7; k 1, n  7.
GCF: 5  t or 5t
m2  6m  7  (m  k)(m  n)
5t2  30t  5t  t  5t  6
 (m  1)(m  7)
 5t(t  6)
12. b2  2b  24
7. z  10z  21
2
In this equation, b is 2 and c is 24. Find two
In this equation, b is 10 and c is 21. Find two
numbers with a product of 24 and with
numbers with a product of 21 and with
a sum of 2.
a sum of 10.
Factors of 24 Sum of Factors
Factors of 21 Sum of Factors
1, 21 22 1, 24 23
3, 7 10 1, 24 23
The correct factors are 7 and 3; m  7, n  3. 2, 12 10
z2  10z  21  (z  m)(z  n) 2, 12 10
 (z  7)(z  3)
3, 8 5
8. n2  8n  15
In this equation, b is 8 and c is 15. Find two 3, 8 15
numbers with a product of 15 and with 4, 6 2
a sum of 8.
4, 6 12
Factors of 15 Sum of Factors
1, 15 16 The correct factors are 4 and 6; m 4, n  6.
b2  2b  24  (b  m)(b  n)
3, 5 18
 (b  4)(b  6)
The correct factors are 3 and 5; m  3, p  5. 13. q2  9q  18
n2  8n  15  (n  m)(n  p) In this equation, b is 9 and c is 18. This means
 (n  3)(n  5) that m  n is negative and mn is positive. So, m
9. h2  8h  12 and n must both be negative.
In this equation, b is 8 and c is 12. Find two
numbers with a product of 12 and with Factors of 18 Sum of Factors
a sum of 8. 1, 18 19
Factors of 12 Sum of Factors 2, 9 11
1, 12 13 3, 6 19
2, 6 8
The correct factors are 3 and 6; m  3,
3, 4 7
n  6.
The correct factors are 2 and 6; m  2, n  6. q2  9q  18  (q  m)(q  n)
h2  8h  12  (h  m)(h  n)  (q  3)(q  6)
 (h  2)(h  6)

Prerequisite Skills 496


14. p2  5p  6 18. y2  3y  88
In this equation, b is 5 and c is 6. This means In this equation, b is 3 and c is 88. Find two
that m  n is negative and mn is positive. So, m numbers with a product of 88 and with
and n must both be negative. a sum of 3.
Factors of 6 Sum of Factors Factors of 88 Sum of Factors
1, 6 7 1, 88 87
2, 3 5 1, 88 87
The correct factors are 2 and 3; m  2, 2, 44 42
n   3. 2, 44 42
p2  5p  6  (p  m)(p  n)
 (p  2)(p  3) 4, 22 18
15. a2  3a  4 4, 22 18
In this equation, b is 3 and c is 4. Find two 8, 11 3
numbers with a product of 4 and with
a sum of 3. 8, 11 13
Factors of 4 Sum of Factors The correct factors are 11 and 8; m  11,
n  8.
1, 4 3
y2  3y  88  (y  m)(y  n)
1, 4 3  (y  11)(y  8)
2, 2 0 19. 3z2  4z  4
The correct factors are 4 and 1; m  4, n  1. In this equation, a is 3, b is 4, and c is 4. Find
a2  3a  4  (a  m)(a  n) two numbers with a product of 12 and
 (a  4)(a  1) with a sum of 4.
16. k2  4k  32 Factors of 12 Sum of Factors
In this equation, b is 4 and c is 32. Find two
1, 12 11
numbers with a product of 32 and with
a sum of 4. 1, 12 11
Factors of 32 Sum of Factors 2, 6 4
1, 32 31 2, 6 14
1, 32 31 3, 4 11
2, 16 14 3, 4 11
2, 16 14 The correct factors are 2 and 6; m  2, n  6.
3z2  4z  4  3z2  mz  nz  4
4, 8 4
 3z2  (2)z  6z  4
4, 8 14  (3z2  2z)  (6z  4)
 z(3z  2)  2(3z  2)
The correct factors are 8 and 4; m  8, n  4.
 (3z  2)(z  2)
k2  4k  32  (k  m)(k  n)
 (k  8)(k  4) 20. 2y2  9y  5
In this equation, a is 2, b is 9, and c is 5. Find
17. n2  7n  44
two numbers with a product of 10 and
In this equation, b is 7 and c is 44. Find two
with a sum of 9.
numbers with a product of 44 and with
a sum of 7. Factors of 10 Sum of Factors
Factors of 44 Sum of Factors 1, 10 9
1, 44 43 1, 10 9
1, 44 43 2, 5 3
2, 22 20 2, 5 3
2, 22 20 The correct factors are 1 and 10; m  1,
4, 11 7 n  10.
2y2  9y  5  2y2  my  ny  5
4, 11 17  2y2  (1)y  10y  5
The correct factors are 11 and 4; m  11,  (2y2  y)  (10y  5)
p  4.  y(2y  1)  5(2y  1)
n2  7n  44  (n  m)(n  p)  (2y  1)(y  5)
 (n  11)(n  4)

497 Prerequisite Skills


21. 5x2  7x  2 The correct factors are 1 and 16; m 1, n  16.
In this equation, a is 5, b is 7, and c is 2. Find 8a2  15a  2  8a2  ma  na  2
two numbers with a product of 10 and with  8a2  (1)a  16a  2
a sum of 7.  (8a2  a)  (16a  2)
Factors of 10 Sum of Factors  a(8a  1)  2(8a  1)
 (8a  1)(a  2)
1, 10 11

2
2, 5 17 25. w2  9
 2 3
4 ¬w  2
The correct factors are 2 and 5; m  2, n  5. ¬w  3
2 w  2 
 3
5x2  7x  2  5x2  mx  nx  2
 5x2  2x  5x  2 26. c2  64  c2  82
 (5x2  2x)  (5x  2)  (c  8)(c  8)
 x(5x  2)  1(5x  2) 27. r2  14r  49  r2  2(1r)(7)  72
 (5x  2)(x  1)  (r  7)2
22. 3s  11s  4
2
28. b  18b  81  b2  2(1b)(9)  92
2

In this equation a is 3, b is 11, and c is 4. Find  (b  9)2


two numbers with a product of 12 and with 29. j  12j  36  j2  2(1j)(6)  62
2
a sum of 11.  ( j  6)2
Factors of 12 Sum of Factors 30. 4t  25  (2t)2  52
2

1, 12 11  (2t  5)(2t  5)


31. 10r2  35r  0
1, 12 11 5r(2r  7)  0
2, 6 4 5r  0 or 2r  7  0
r0 r  7

2, 6 14 2
32. 3x2  15x  0
3, 4 11 3x(x  5)  0
3, 4 11 3x  0 or x  5  0
x0 x  5
The correct factors are 1 and 12; m  1, n  12.
33. k2  13k  36  0
3s2  11s  4  3s2  ms  ns  4
(k  4)(k  9)  0
 3s2  (1)s  12s  4
k  4  0 or k  9  0
 (3s2  s)  (12s  4)
k  4 k  9
 s(3s  1)  4(3s  1)
 (3s  1)(s  4) 34. w2  8w  12  0
(w  2)(w  6)  0
23. 6r2  5r  1
w  2  0 or w  6  0
In this equation, a is 6, b is 5, and c is 1. Find
w2 w6
two numbers with a product of 6 and with a sum
of 5. Note that m and n are both negative. 35. c2  5c  14  0
(c  2)(c  7)  0
Factors of 6 Sum of Factors c  2  0 or c  7  0
1, 6 7 c  2 c7
2, 3 5 36. z2  z  42  0
(z  6)(z  7)  0
The correct factors are 2 and 3; m  2, n  3. z  6  0 or z  7  0
6r2  5r  1  6r2  mr  nr  1 z  6 z7
 6r2  (2)r  (3)r  1 37. 2y2  5y  12  0
 (6r2  2r)  (3r  1) (2y  3)(y  4)  0
 2r(3r  1)  1(3r  1) 2y  3  0 or y  4  0
 (2r  1)(3r  1)
y  3
2 y4
24. 8a2  15a  2
38. 3b  4b  15  0
2
In this equation, a is 8, b is 15, and c is 2. Find
(3b  5)(b  3)  0
two numbers with a product of 16 and with
3b  5  0 or b  3  0
a sum of 15.
b  53
 b3
Factors of 16 Sum of Factors
39. t  12t  36 ¬0
2

1, 16 15 (t  6)2 ¬0


t  6 ¬0
1, 16 15
t ¬6
2, 8 6
2, 8 1 6
4, 4 10

Prerequisite Skills 498


25
40. u2  5u  4 ¬0



8 0 1 3
 
2
u  5
 ¬0 4. C  B  2 2  2 8
2
10 6 7 6
u  5
2 ¬0



u ¬5
 8  (1) 0  (3)
2  2  2 28
41. q  8q  16 ¬0
2
10  7 66
(q  4)2 ¬0


q  4 ¬0 9 3
q ¬4  4 6
42. a2  6a  9 ¬0 17 0
(a  3)2 ¬0


10 9
a  3 ¬0
5. 3A  3 4 3
a ¬3
1 11


3(10) 3(9)
Pages 752–753 Operations with Matrices  3(4) 3(3)
3(1) 3(11)


10 9 1 3


1. A  B  4 3  2 8 30 27
1 11 7 6  12 9
3 33
10  (1) 9  (3)

42
1  7
3  8
11  6 6. 5B  5

1 3
2 8

7 6


9 12

5(1) 5(3)
 6 5  5(2) 5(8)
6 17 5(7) 5(6)


5 15


1 3 8 0
2. B  C  2 8  2 2  10 40
7 6 10 6 35 30


8 0


1  8 3  0 7. 4C  4 2 2
 2  (2) 8  2 10 6
7  (10) 6  6


4(8) 4(0)


7 3  4(2) 4(2)
 0 10 4(10) 4(6)
3 12


32 0


10 9 8 0  8 8
3. A  C  4 3  2 2 40 24
1 11 10 6


8 0
8. 1 1
2C  2
 2 2


10  8 9  0
10 6
 4  (2) 3  2
1  (10) 11  6


1(8) 1

2 (0)
2
1 1
(2) 2(2)


2 9  2
 6 5 (10) 1
1 
2 2 (6)
9 5


4 0
 1 1
5 3

499 Prerequisite Skills



10 9 8 0 10 9 1 3
9. 2A  C  2 4 3  2 2 12. 3A  3B  3 4 3  3 2 8
1 11 10 6 1 11 7 6


2(10) 2(9) 8 0 3(10) 3(9) 3(1) 3(3)
 2(4) 2(3)  2 2  3(4) 3(3)  3(2) 3(8)
2(1) 2(11) 10 6 3(1) 3(11) 3(7) 3(6)


20 18


8 0 30 27 3 9
 8 6  2 2  12 9  6 24
2 22 10 6 3 33 21 18


20  8 18  0


30  (3) 27  (9)
 8  (2) 6  2  12  6 9  24
2  (10) 22  6 3  21 33  18


28 18


33 18
 6 4  6 33
12 28 24 15


10 9 8 0
10. A  5C  4 3  5 2 2 13. X  Z  102 84  74 80
1 11 10 6
 4 8  (8)
 10 2(7) 4  0

10 9 5(8) 5(0)
 4 3  5(2) 5(2)
1 11 5(10) 5(6)  63 16
4
10 9


40 0
 4 3 
1 11
10 10 14. Y  Z  16 0
5

4 8

7 0
50 30
 6 1 4 0  (8)


10  40 9  0  (7) 5  0
 4  (10) 3  10
1  (50) 11  30  13 8
5


30 9
 14 13 15. X  Y  102 84  16 0
5
49 19
 2 10(1) 8  0
 6 4  (5)


8 0 1 3
11. 1
C  B  1 2 2  2 8
2 2
10 6 7 6  34 89
16

1(8) 1(0) 16. 3Y  3
0
2 2 5

1 3
1(2) 1(2)
 2 2  2 8
1(10) 1(6)
2 2 7 6  3(1)
3(6)
3(0)
3(5)
3 0


4 0 1 3 
18 15
 1 1  2 8
5 3 7 6
17. 6X  6 102 84

4  (1) 0  (3) 6(2) 6(8)
 1  2 18  6(10) 6(4)
5  7 36
 12
60
48
24

3 3
 1 9
2 9

Prerequisite Skills 500


2 8 4 8
18. 1 1
2 X  Z  2 10
 4

7
0 22. XZ  102 84 74 80
2(4)  (8)(7) 2(8)  (8)(0)

10(8)  4(0)

1(2) 1

2 (8) 10(4)  4(7)

2
1(10) 1(4)
 74 80 16
2 2
 64
12 80
 15 42  74 8
0 2 (XZ)  2 10
23. 1
 1
2 8 4 8
4 7 0
 4 4  (8)
 5 1(7) 2  0  1
2(4)  (8)(7) 2(8)  (8)(0)
2 10(4)  4(7) 10(8)  4(0)
64 16
25 122 2 12 80
 1
 

74 80  2 16 50 1(64)  1(16)



19. 5Z  2Y  5 2 2

1(12)  1(80)

5(7) 5(0) 2(6) 2(5)
5(4) 5(8) 2(1) 2(0) 2 2

8

20 40 2  326 40
0 12 10
0

35


20  (2) 40  0 24. XY  2Z  102 84  16 0

2
4 8

35  12 0  (10)
5 7 0

 (8)(6) 2(0)  (8)(5)



22 40
47 10
 2(1)
10(1)  4(6) 10(0)  (4)(5)
20. XY 
2 8

1 0
10 4 6 5
 2(7)
2(4) 2(8)
2(0)
2(1)  (8)(6) 2(0)  (8)(5)

10(1)  4(6) 10(0)  (4)(5)
 50
14 20 14
40

8 16
0
50 40 50  8 40  (16)

14 20

14  (14) 20  0
42 24

16 50 74 80
21. YZ  0 20
(1)(4)  0(7) (1)(8)  0(0)

6(4)  (5)(7) 6(8)  (5)(0)
4

59 48
8

501 Prerequisite Skills


Extra Practice
Page 754 Lesson 1-1 8. 32
1. There are 8 planes: plane ABD, plane GLJ, plane
AFL, plane FEK, plane EDJ, plane DCI, plane 17 3m
CBH, and plane ABH.
2. B, O, and C or D, M, and J A B C
3. planes AFG, ABG, and GLK AC ¬AB  BC
4. 
DE 32 ¬17  3m
5. Sample answer: planes ABD and GHJ 15 ¬3m
5 ¬m
6. plane FEK
BC ¬3m
7. A line; two planes intersect in a line, not a point. BC ¬3(5)
8. BC ¬15
9. 42
X Y Z a
9a 12a
A B C

9. A B C
Z AC ¬AB  BC
X 42 ¬9a  12a
A B b 42 ¬21a
42 21
a
21 ¬ 21
 
2 ¬a
m BC ¬12a
BC ¬12(2)
Page 754 Lesson 1-2 BC ¬24
1. The measurement is precise to within 1
2 in. 10. 7b  13
So, a measurement of 42 in. could be 41 1

2 25 3b
to 42 1

2 in.
2. The measurement is precise to within 0.5 mm. So,
a measurement of 86 mm could be 85.5 to A B C
86.5 mm. AC ¬AB  BC
7b  13 ¬25  3b
3. The measurement is precise to within 0.5 cm. So,
7b  13  13 ¬25  3b  13
a measurement of 251 cm could be 250.5 to
7b ¬12  3b
251.5 cm.
7b  3b ¬12  3b  3b
4. The measurement is precise to within 0.05 in. So, 4b ¬12
a measurement of 33.5 in. could be 33.45 to 4b 12
4 ¬ 4
 
33.55 in.
b ¬3
5. The measurement is precise to within 1
8 ft. BC  3b
So, a measurement of 5 1
 1 3
4 ft could be 5 8 to 5 8 ft. BC  3(3)
6. The measurement is precise to within 0.5 m. So, a BC  9
measurement of 89 m could be 88.5 to 89.5 m. 11. 54
7.
4x 5x 5n  5 2n
16
A B C A B C
AB  4x ¬16 AC ¬AB  BC
4x 16 54 ¬5n  5  2n
4 ¬ 4
  
54 ¬7n  5
x ¬4 54  5 ¬7n  5  5
BC  5x 49 ¬7n
BC  5(4) 49 7n
7 ¬ 7
  
BC  20
7 ¬n
BC ¬2n
BC ¬2(7)
BC ¬14

Extra Practice 502


12. 65 4. y
6 O
6c  8 3c  1

2
A B C 6 2 2 6 x
AC ¬AB  BC 2
65 ¬6c  8  3c  1
6
65 ¬9c  7
65  7 ¬9c  7  7 M X
72 ¬9c (MO)2 ¬(MX)2  (OX)2
2
7 9 c
9 ¬ 9 (MO)2 ¬(8)2  (15)2
8 ¬c (MO)2 ¬289
BC ¬3c  1 MO ¬17
BC ¬3(8)  1 5. y
BC ¬25
6
Page 754 Lesson 1-3
T 2
1. y O
B
2 2 6 x

6

X A x
X R
(TR)2 ¬(TX)2  (RX)2
(TR)2 ¬(16)2  (12)2
(TR)2 ¬400
(AB)2 ¬(AX)2  (BX)2 TR ¬20
(AB)2 ¬(3)2  (4)2 6. y
(AB)2 ¬25 N
AB ¬5 6
2. 2
y N
O
6 2 6 x

4
X F

C (FN)2 ¬(FX)2  (NX)2


X (FN)2 ¬(10)2  (12)2
(FN)2 ¬244
O x FN ¬ 244
(CN)2 ¬(CX)2  (NX)2 FN ¬15.6
(CN)2 ¬(6)2  (8)2 d ¬(x

7. 2
x1)2 
(y2 
y1)2
(CN)2 ¬100
CN ¬10 DM ¬
(8  0)
2  (
7  0)2
3. y
DM ¬8 2
2  (7)

DM ¬ 
64  49
6
DM ¬113
2
Z DM ¬10.6
6 O2 x 8. d ¬
(x2  
x1)2 
(y2 
y1)2
X 4
Y XY ¬ 
[1  (1)]2
 (1
 1)2
XY ¬22  (
2)2
8
XY ¬44
(XZ)2 ¬(XY)2  (ZY)2 XY ¬8
(XZ)2 ¬(12)2  (5)2 XY ¬2.8
(XZ)2 ¬169
XZ ¬13

503 Extra Practice


d ¬
(x2  
x1)2 
(y2 
9. y1)2 19. B(5, 6)  
1
 x 2 0
, 
1

y 0
2 
ZA ¬
[3 
(4)]2
 (7
 0)2 x1  0 y 0
5 ¬  6 ¬
1

ZA ¬12  72 2 2
10 ¬x1  0 12 ¬y1  0
ZA ¬1  49
10 ¬x1 12 ¬y1
ZA ¬50
The coordinates of A are (10, 12).
ZA ¬7.1
 (7) y  (4)
10. d ¬
(x2  
x1)2 
(y2 
y1)2 20. B(3, 5)  
1
x
, 
1

2 2 
KD ¬
(3 
(3 
6)2 6)2 x1  (7) y1  (4)
3 ¬ 5 ¬ 
2 2
KD ¬(9)2 
 (9)2
6 ¬x1  (7) 10 ¬y1  (4)
KD ¬ 
81  81
13 ¬x1 14 ¬y1
KD ¬162
The coordinates of A are (13, 14).
KD ¬12.7
11. d ¬
(x2  
x1)2 
(y2 
y1)2
21. B(10, 2)  
1
 x 2 8
, 
1

y  (4)
2 
x1  8 y1  (4)
TN ¬
[0  (
1)]2 
 (2 
3)2 10 ¬  2 ¬ 
2 2
TN ¬1 2
2  (1) 20 ¬x1  8 4 ¬y1  (4)
TN ¬11 28 ¬x1 8 ¬y1
TN ¬2 The coordinates of A are (28, 8).
TN ¬1.4 22. B(3, 5)  
1
 x 2 6
, 
1

y 8
2 
12. d ¬
(x2  x1)2  (y2  y1)2 x1  6 y 8
3 ¬  5 ¬
1

SE ¬(6  7)2  (7  2)2 2 2
6 ¬x1  6 10 ¬y1  8
SE ¬(13)
2  (5)2
12 ¬x1 2 ¬y1
SE ¬169 
25
The coordinates of A are (12, 2).
SE ¬194
SE ¬13.9 23. B(3, 4)  
1
 x 2 6
, 
1

y  (8)
2 
x1  x2 y1  y2 x1  6 y1  (8)
13.  ,  ¬ 2, 2
2 2  
0  (2) 0  (8)
 3 
2
4 ¬
2

2 8 6  x1  6 8 ¬y1  (8)
¬2 , 2
0  x1 0 ¬y1
¬(1, 4)
x1  x2 y1  y2 The coordinates of A are (0, 0).
14.  2 2 
4  2 3  2
,  ¬2, 2
24. B(0, 5)  
1
x (2) y  (4)
, 
1
 
2 1
¬2 , 2 x1  (2)
2 2
y1  (4)
0  5 ¬ 
¬(1, 0.5) 2 2
x1  x2 y1  y2 0  x1  (2) 10 ¬y1  (4)
15.  2 2 
4  5 5  4
,  ¬2, 2
2  x1 14 ¬y1

¬1
2, 2 
  1
The coordinates of A are (2, 14).
¬(0.5, 0.5)
x1  x2 y1  y2 Page 755 Lesson 1-4
16.  2 2 
10  8 5  4
,  ¬2, 2
1. B

¬2 , 2
2 9
2. E
¬(1, 4.5) 3. G
x1  x2 y1  y2
17.  2 2 
2.8  1.2 3.4  5.6
,  ¬2, 2
4.
5.
I

IA, 
IE
¬4
2, 2 
 2.2
 
6. ED, 
EF
¬(2, 1.1) 7. 
GC, 
GH
x1  x2 y1  y2
18.   
6.2  3.4 7  (4.8)
,  ¬2, 2
2 2
8. , HF
HA 
2 9. DCG
¬2 , 2 
.8 2.2

10. 4
¬(1.4, 1.1)
11. BCG
12. 120°; 120  90 and 120  180 so ABC is obtuse.
13. 90°; CGF is a right angle.
14. 60°; 60  90 so HIF is acute.

Extra Practice 504


Page 755 Lesson 1-5 Page 755 Lesson 1-6
1. Sample answer: BGC and FGE are vertical 1. There are 4 sides, so the polygon is a
angles. They each have measures less than 90, so quadrilateral. No line containing any of the sides
they are acute. will pass through the interior of the quadrilateral,
2. Sample answer: BGF and CGE are vertical so it is convex. The sides are congruent, and the
angles. They each have measures greater than 90, angles are congruent, so it is regular. Its
so they are obtuse. perimeter is 22.5 m  22.5 m  22.5 m  22.5 m,
3. Sample answer: BED is a right angle, so or 90 m.
BEC and CED are adjacent angles that are 2. There are 6 sides, so the polygon is a hexagon.
complementary because mBEC  mCED  There is a side such that a line containing that
mBED. side will pass through the interior of the hexagon,
4. Sample answer: CEF and CED are adjacent so it is concave. The sides are not congruent, so it
angles that are supplementary because they form is irregular. Its perimeter is 25 cm  25 cm 
a linear pair. 25 cm  25 cm  28 cm  28 cm, or 156 cm.
5. Sample answer: ABE is a right angle and CBE 3. There are 16 sides, so the polygon is a 16-gon.
is adjacent to and forms a linear pair with ABE, There is a side such that a line containing that
so CBE is also a right angle. side will pass through the interior of the polygon,
so it is concave. The sides are not congruent, so it
6. BGC and FGE are vertical angles.
is irregular. Its perimeter is 24  12  12  48 
mBGC ¬mFGE
12  12  24  12  12  12  12  24  12 
4x  5 ¬6x  15
12  12  12 or 264 in.
5 ¬2x  15
20 ¬2x 4. d ¬
(x2  
x1)2 
(y2 
y1)2
10 ¬x XY ¬
(2 
3)2 
(1  
3)2
BGF and FGE form a linear pair, so ¬(5)
 2 
 (2)2
mBGF  mFGE ¬180.
¬29
mBGF  mFGE ¬180
mBGF  6x  15 ¬180 YZ ¬ [1  (
2)]2 
 (3  1)2
mBGF  6(10)  15 ¬180 ¬3 2 
 (4) 2

mBGF  45 ¬180 ¬ 25 or 5


mBGF ¬135 XZ ¬ (1  3
)2  (3  3
)2
C
7. A C D if BCD is a right angle, so ¬(2)
 2 
 (6)2
mBCD  90.
¬ 40
mBCD ¬mBCG  mGCE  mECD
90 ¬5a  5  3a  4  4a  7 The perimeter is XY  YZ  XZ  29
5
90 ¬12a  6 40  16.7 units.
96 ¬12a d ¬
(x2 x
1)  
(y2  
5. 2 y1)2
8 ¬a
PE ¬
[5 
(2)]2
 (0
 3)2
8. A and B form a linear pair, so mA  mB 
180. We are given that mA  mB  9. ¬(3)
 2 
 (3)2
mA  mB ¬180 ¬18
mB  9  mB ¬180 EN ¬ [2  (5)]2 (4  0
)2
2(mB) ¬189 ¬3 2 
 (4) 2
mB ¬94.5
¬ 25 or 5
mA ¬mB  9
¬94.5  9 NT ¬ 
[2  (2)] 2 
 [1 (4)]2
¬85.5 ¬4 2  32

9. Let x represent the measure of the angle. Then its ¬ 25 or 5


complement has measure x  17. TA ¬ (2  2)2  [2 (1)]2
x  x  17 ¬90 ¬0 2  32
2x  17 ¬90 ¬ 9 or 3
2x ¬73
PA ¬ 
[2  (2)] 2 
 (2 3)2
x ¬36.5
x  17 ¬53.5 ¬4  2
2  (1)

The angle has measure 36.5, and its complement ¬ 17


has measure 53.5. The perimeter is PE  EN  NT  TA  PA 
18  5  5  3   17  21.4 units.

505 Extra Practice


d ¬
(x2 x  is an angle bisector of TSU.
1)  
(y2  
6. 2 y1)2 4. Given: SR
HE ¬
(3 
0)2  2
(2  4) T
¬(3)
 2 
 (2)2
R
¬13

EX ¬ [3  (3)]2 (2  2 )2 S
U
¬02 
 (4) 2
TSR and RSU have equal measures, by
¬16 or 4 definition of bisector.
XG ¬ [0  ( 3)]2 
 [5  ( 2)]2 Conjecture: TSR  RSU
¬32 
 (3) 2
5. EF and FG are side lengths of equilateral triangle
¬18 EFG. By definition, the three side lengths of any
GO ¬ (5  0 )2  [2  (5)]2 equilateral triangle are equal. The conjecture is
true.
¬52  3 2
6. Some rational numbers are not whole numbers.
¬34
For example, r  0.5 is a rational number but not
ON ¬ (5  5 )2  [2 (2)]2 a whole number. So the conjecture is false.
¬02  42  7. Every whole number n is a rational number of the
¬16 or 4 form n
1. The conjecture is true.
HN ¬ (5  0 )2  (2 4)2 8. Two angles can be supplementary without being
¬52 
 (2) 2 a linear pair. The angles shown have measures
¬29 that sum to 180, but they are not adjacent with
The perimeter is HE  EX  XG  GO  ON  opposite rays as their noncommon sides.
HN   13  4   18  34  4  29

 27.1 units. 70

Page 756 Lesson 2-1 110


1. Given: Lines j and k are parallel. The conjecture is false.
j
Page 756 Lesson 2-2
k
1. (3)2  9 and a robin is a fish.
p and q is false, because p is true and q is false.
Since parallel lines by definition never intersect, 2. (3)2  9 or a robin is a fish.
lines j and k cannot intersect. p or q is true because p is true. It does not matter
Conjecture: Lines j and k do not intersect. that q is false.
2. Given: points A(1, 7), B(4, 7), C(4, 3), 3. (3)2  9 and an acute angle measures less than
D(1, 3) 90°. p and r is true, because p is true and r is
y true.
4. (3)2  9 or an acute angle measures less than
O x 90°. p or r is true, because p is true and r is true.
5. (3)2  9 or a robin is a fish.
D ( 1, 3) C (4, 3)
p or q is false, because p is false and q is false.
6. (3)2  9 or an acute measures 90° or more. p or
r is true, because p is true. It does not matter
that r is false.
A ( 1, 7) B (4, 7) 7. A robin is a fish and an acute angle measures less
Line segments AB
 and C
D are horizontal, while than 90°.
C
line segments B  and A
D
 are vertical. q  r is false because q is false and r is true.
Conjecture: ABCD is a rectangle 8. (3)2  9 and a robin is a fish, or an acute angle
3. Given: AB
 bisects C
D
 at K. measures less than 90°.
(p  q) r true because r is true. It does not
B matter whether p  q is true or false.
9. (3)2  9 or an acute angle measures 90° or more.
p r is false, because p is false and r
C D is false.
K

A
D
K is the midpoint of C, by definition of bisect.
Conjecture: CK  KD

Extra Practice 506


10. p q q p  q Converse: If a figure is a polygon, then it is a
triangle. The converse is false. A pentagon is a
T T F T polygon but is not a triangle.
T F T T Inverse: If a figure is not a triangle, then it is not
F T F F a polygon. The inverse is false. A hexagon is not a
triangle, but it is a polygon.
F F T T The contrapositive is the negation of the
11. hypothesis and conclusion of the converse.
p q p q p  q Contrapositive: If a figure is not a polygon, then
T T F F F it is not a triangle. The contrapositive is true.
T F F T T 10. Conditional: If two angles are congruent angles,
then they have the same measure. The conditional
F T T F T statement is true.
F F T T T Write the converse by switching the hypothesis
and conclusion of the conditional.
Converse: If two angles have the same measure,
Page 756 Lesson 2-3 then they are congruent angles. The converse is
1. If no sides of a triangle are equal, then it is a scalene triangle. true.
hypothesis conclusion Inverse: If two angles are not congruent angles,
then they do not have the same measure. The
Hypothesis: no sides of a triangle are equal
inverse is true.
Conclusion: it is a scalene triangle
The contrapositive is the negation of the
2. If it rains today, you will be wearing your raincoat. hypothesis and conclusion of the converse.
hypothesis conclusion Contrapositive: If two angles do not have the
Hypothesis: it rains today same measure, then they are not congruent
Conclusion: you will be wearing your raincoat angles. The contrapositive is true.
3. 11. Conditional: If three points lie on the same line,
If 6 x 11, then x 5.
then they are collinear. The conditional statement
hypothesis conclusion is true.
Hypothesis: 6  x  11 Write the converse by switching the hypothesis
Conclusion: x  5 and conclusion of the conditional.
4. If you are in college, you are at least 18 years old.
Converse: If three points are collinear, then they
lie on the same line. The converse is true.
hypothesis conclusion Inverse: If three points do not lie on the same
Hypothesis: you are in college line, then they are not collinear. The inverse is true.
Conclusion: you are at least 18 years old The contrapositive is the negation of the
5. The sum of the measures of two supplementary hypothesis and conclusion of the converse.
angles is 180. Contrapositive: If three points are not collinear,
Hypothesis: two angles are supplementary then they do not lie on the same line. The
Conclusion: the sum of their measures is 180 contrapositive is true.
If two angles are supplementary, then the sum of 12. Conditional: If  PQ is a perpendicular bisector of
their measures is 180. M
L , then a right angle is formed. The conditional
6. A triangle with two congruent sides is an statement is true.
isosceles triangle. Write the converse by switching the hypothesis
Hypothesis: a triangle has two congruent sides and conclusion of the conditional.
Converse: If a right angle is formed by PQ  and
Conclusion: it is an isosceles triangle
M
L , then PQ is a perpendicular bisector of L
M.
If a triangle has two congruent sides, then it is an
isosceles triangle. The converse is false. 
PQ may not pass through
7. Two lines that do not intersect are parallel lines. the midpoint of L M
.
Hypothesis: two lines do not intersect Inverse: If 
PQ is not a perpendicular bisector of
Conclusion: they are parallel M
L , then a right angle is not formed. The inverse
 could be perpendicular to L
is false. PQ M without
If two lines do not intersect, then they are
parallel. bisecting LM.
The contrapositive is the negation of the
8. A Saint Bernard is a dog.
hypothesis and conclusion of the converse.
Hypothesis: an animal is a Saint Bernard
Contrapositive: If a right angle is not formed by
Conclusion: it is a dog  and L is not a perpendicular
PQ M
, then PQ
If an animal is a Saint Bernard, then it is a dog.
bisector of LM
. The contrapositive is true.
9. Write the conditional in if-then form.
Conditional: If a figure is a triangle, then it is
a polygon. The conditional statement is true.
Write the converse by switching the hypothesis
and conclusion of the conditional.

507 Extra Practice


5x  1
Page 757 Lesson 2-4 4. Given:  
8 3
1. (1) If it rains, then the field will be muddy. Prove: x  5
(2) If the field is muddy, then the game will be Proof:
cancelled. Statements Reasons
Let p, q, and r represent the parts of the
5x  1
statement. a.  8  3 a. ? Given
p: it rains
q: the field will be muddy b. ? 5x 
8  
8
1
 8(3) b. Mult. Prop.
r: the game will be cancelled
c. 5x  1  24
Statement (1): p → q c. ? Substitution
Statement (2): q → r d. 5x  25 d. ? Add. Prop.
Since the given statements are true, use the Law
of Syllogism to conclude p → r. That is, if it e. ? x5 e. Div. Prop.
rains, then the game will be cancelled.
Page 758 Lesson 2-7
2. There is no valid conclusion. Although both
statements may be true, the conclusion of one 1. Addition Property
statement is not used as the hypothesis of the 2. Symmetric Property
other. 3. Transitive Property
3. p: it is snowing outside 4. Substitution Property
q: you will wear your winter coat. 5. Reflexive Property
Statement (3) is a valid conclusion by the Law of 6. Subtraction Property
Detachment. 7. Addition Property
4. Statement (1) is true, but statement (3) does not 8. Transitive Property
follow from statement (2). Not all pairs of acute 9. Given:  A
AB F, D
angles are complementary. F
AE D,
Statement (3) is invalid. D
E C D
 E C
Prove: ABCD
Page 757 Lesson 2-5 F B
1. Sometimes; 
RS and 
PS could intersect to form a
45° angle. A
2. Sometimes; if the points are collinear, then they Proof:
lie on one line.
Statements Reasons
3. Sometimes; the line perpendicular to BC could lie
in plane K, but it does not have to. B
1. AAF
, A
F
E

D 1. Given
4. The fact that 
DF lies in plane R is an instance of B
2. AED
 2. Transitive Property
Postulate 2.5, which says that if two points lie in D
3. E C
D
 3. Given
a plane, then the entire line containing those B
4. AC
D
 4. Transitive Property
points lies in that plane.
5. The fact that E and C are collinear is an instance 10. Given: AC  DF, AB  DE B C
A
of Postulate 2.1, which says that through any two Prove: BC  EF
points, there is exactly one line.
E D
6. The fact that D, F, and E are coplanar is an
instance of Postulate 2.2, which says that through Proof: F
any three points not on the same line, there is
exactly one plane. Statements Reasons
7. The fact that E and F are collinear is an instance 1. AC  AB  BC and 1. Segment Addition
of Postulate 2.1, which says that through any two DF  DE  EF Postulate
points, there is exactly one line. 2. AC  DF 2. Given
3. AB  BC  DE  EF 3. Substitution
Page 757 Lesson 2-6 4. AB  DE 4. Given
1. x  5 ¬6 5. BC  EF 5. Subtraction Property
Add 5 to both sides of the equation.
x ¬11
The property that justifies the statement “If x  5 Page 758 Lesson 2-8
 6, then x  11” is the Addition Property. 1. 9 and 10 form a linear pair, so by the
2. Transitive Property Supplement Theorem,
m9  m10 ¬180
3. Symmetric Property
141  x  25  x ¬180
166  2x ¬180
2x ¬14
x ¬7

Extra Practice 508


m9  141  x 2. By the Corresponding Angles Postulate,
 141  7  148 13  5.
m10  25  x m13  m5
 25  7  32 m13  72
2. 13 forms a linear pair with the right angle, so 3. By the Vertical Angles Theorem, 4  1.
by the Supplement Theorem, m1  102 (Exercise 1)
m13  90 ¬180 m4  m1
m13 ¬90 m4  102
m13 ¬90 and m13 ¬3x  30, so 4. 9 and 10 form a linear pair. By the
3x  30 ¬90 Supplement Theorem,
3x ¬60 m9  m10 ¬180
x ¬20 102  m10 ¬180
m11  x  40 m10 ¬78
 20  40  60 5. 5 and 7 form a linear pair. By the Supplement
m12  x  10 Theorem,
 20  10  30 m5  m7 ¬180
3. By the Angle Addition Postulate, 72  m7 ¬180
m14  m15  m16 ¬180 m7 ¬108
x  25  4x  50  x  45 ¬180 6. By the Vertical Angles Theorem, 16  13.
6x  120 ¬180 m13  72 (Exercise 2)
6x ¬60 m16  m13
x ¬10 m16  72
m14  x  25 7. By the Corresponding Angles Postulate, the
 10  25  35 angles labeled (8x  5)° and 75° are congruent,
m15  4x  50 and so their measures are equal.
 4(10)  50  90 8x  5 ¬75
m16  x  45 8x ¬80
 10  45  55 x ¬10
4. Sometimes; if two complementary angles each The angle labeled a° and the angle labeled 75° are
measure 45°, they are congruent. Otherwise they corresponding angles, so they are congruent. So
are not. a  75.
5. Never; the sum of the measures of two angles that
form a linear pair is 180, not 90.
6. Sometimes; if two congruent angles each measure (8x  5)
90, then they are supplementary, but otherwise
they are not.
7. Always; perpendicular lines are, by definition, 75
lines that intersect at right angles.
8. Always; the measure of one right angle is 90,
so the sum of the measures of two right angles a (9y  3)
is 180.
9. Sometimes; if the intersecting lines are
perpendicular they form four right angles.
Otherwise they do not.
The angle labeled a° and the angle labeled
Page 758 Lesson 3-1 (9y  3)° are consecutive interior angles. By the
Consecutive Interior Angles Theorem, the two
1. 
LP
angles are supplementary.
2. Planes ABM, OCN, ABC, LMN, and AEP a  9y  3 ¬180
3. M
B , A
L
, E
P, O
P
, P
L, L
M
, and M
N
 75  9y  3 ¬180
4. consecutive interior 9y  72 ¬180
5. corresponding 9y ¬108
6. alternate interior y ¬12
7. alternate exterior 8. By the Corresponding Angles Postulate, the
angles labeled 55° and (4x  7)° are congruent,
and so their measures are equal.
Page 759 Lesson 3-2 4x  7 ¬55
1. By the Corresponding Angles Postulate, 1  9. 4x ¬48
m1  m9 x ¬12
m1  102

509 Extra Practice


a b 9. Find the slopes of 
RS and 
TU.
y  ¬
slope of RS 2  11
(4x  7) 25

60 ¬ 9

3 or 3
 1 0
55 slope of TU ¬ 2  ( 1)

By the Angle Addition Postulate, ¬1



3
a  y  b  180. The slopes are not the same, so 
RS and 
TU are
By the Alternate Interior Angles Theorem, not parallel. The product of the slopes is
a  55 and b  60.
By substitution 
(3) 13 or 1. So, 
AB and 
CD are neither parallel
nor perpendicular.
55  y  60 ¬180
10. Find the slopes of  RS and 
TU.
y  115 ¬180
 7 
4
y ¬65 slope of RS ¬ 
3  (1)
3 3
¬2 or  2
Page 759 Lesson 3-3 1  (
1. Use the ris
e slope of 
TU ¬ 
85
1)
run method.
From R(1, 3) to S(4, 1), go down 4 units and ¬2

3
right 3 units.
ris
e 4 4
  
The product of the slopes is 3
 2
2 3 or 1. So,
run  3 or  3
  
RS is perpendicular to 
TU.
ris
e
2. Use the 
run method. 11. Find the slopes of 
RS and 
TU.
From T(5, 0) to U(1, 1), go up 1 unit and right  1  5
slope of RS ¬ 4  (2)
6 units.
4
ris
e 1 ¬
2 or 2
run  6

5 
 ¬ 3
3. Use the slope formula. slope of TU 13
Let W(4, 4) be (x1, y1) and V(4, 1) be (x2, y2). ¬2
(y2  y1) 2 or 1
m ¬   and 
The slopes are not the same, so RS TU are
(x2  x1)
1  4 not parallel. The product of the slopes is 2(1) or
¬
4  (4) 2. So, 
RS and 
TU are neither parallel nor
5 perpendicular.
or 0, which is undefined
4. Use the slope formula. Page 759 Lesson 3-4
Let W(4, 4) be (x1, y1) and R(1, 3) be (x2, y2). 1. y  mx  b
(y  y ) y  1x  (5)
m ¬
2
1
(x2  x1) yx5
3 4 1 The slope-intercept form of the equation of the
¬
1  ( 4) or  5 line is y  x  5.
5. The slope of 
TU is 1 
6 (Exercise 2). A line parallel 2. y  mx  b

to TU will also have a slope of 1 
6. y  1 1
2x  2

 1
6. The slope of WR is 5 (Exercise 4). The product
The slope-intercept form of the equation of the
of the slopes of two perpendicular lines is 1.
line is y  1 1
2x  2.

 
Since 15 (5)  1, the slope of a line

3. y  mx  b
perpendicular to 
WR is 5.
7. 
WV is vertical (with undefined slope). A line  
y  3x  1

4
perpendicular to 
WV must be horizontal and thus
y  3x  1

4
have a slope of 0.
The slope-intercept form of the equation of the
8. Find the slopes of 
RS and 
TU.
6  line is y  3x  1

4.
slope of 
RS ¬ 53
5
4. y  y1 ¬m(x  x1)
¬1
2
 y  4 ¬3[x  (2)]
 ¬ 3 
0 y  4 ¬3(x  2)
slope of TU 4  (2) The point-slope form of the equation of the line is
¬3
 1
6 or 2
y  4  3(x  2).
The slopes are the same, so   are
RS and TU 5. y  y1 ¬m(x  x1)
parallel. y  3 ¬4(x  0)
y  3 ¬4x
The point-slope form of the equation of the line is
y  3  4x.

Extra Practice 510


6. y  y1 ¬m(x  x1) slope is 1
3 that contains (2, 2).

y  (7) ¬2 y  y1 ¬m(x  x1)
3 (x  5)


y  7 ¬2 y  (2) ¬1


3 [x  (2)]

3 (x  5)

The point-slope form of the equation of the line is y  2 ¬1 2
3x  3

y  7  2
3 (x  5).
 y ¬1 8
3x  3

7. Find the slope of p by using (0, 1) and (1, 1). The slope-intercept form of the equation of the
1
1 
m ¬ line is y  1 8
3x  3.
10 
2 14. From Exercise 7, the slope of p is 2, so the slope
¬1 or 2
Notice that the y-intercept is 1. of a line perpendicular to p is 1

2 . Notice that the
The slope-intercept form of the equation of p is y-intercept is 0.
y  2x  1. The slope-intercept form of the equation of the
8. Find the slope of q by using (0, 3) and (3, 0). line is y  1

2 x.
0  (3)
m ¬ 
30 Page 760 Lesson 3-5
¬3

3 or 1
1. 9 and 16 are alternate exterior angles for lines
c and d cut by transversal n. If alternate exterior
Notice that the y-intercept is 3.
angles are congruent, the two lines are parallel.
The slope-intercept form of the equation of q is
Since 9  16, c
d.
y  x  3.
9. Find the slope of r by using (0, 2) and (3, 0). 2. 10 and 16 are not corresponding, alternate
0  (2) exterior, or alternate interior angles. No lines can
m ¬ 
30 be shown to be parallel.
¬2
3
 3. 12 and 13 are alternate interior angles for
Notice that the y-intercept is 2. lines c and d cut by transversal n. If alternate
The slope-intercept form of the equation of r is interior angles are congruent, the two lines are
y  2 parallel. Since 12  13, c
d.
3 x  2.

10. Find the slope of s by using (0, 0) and (3, 1). 4. 12 and 14 are consecutive interior angles for
0
1  lines c and d cut by transversal n. If consecutive
m ¬
30 interior angles are supplementary, the two lines
1 1 are parallel. Since m12  m14  180, c
d.
¬3 or 3
5. Explore: From the figure, we know that the
Notice that the y-intercept is 0.
angles marked 135° and (2x  15)° are
The slope-intercept form of the equation of s is
consecutive interior angles for lines r and s.
y  1 
3 x. Plan: For line r to be parallel to line s,
11. From Exercise 8, the slope of q is 1, so the slope of consecutive interior angles must be
a line parallel to q is also 1. Write an equation in supplementary.
point-slope form of the line whose slope is 1 that Solve: 135  2x  15 ¬180
contains (2, 5). 2x  150 ¬180
y  y1 ¬m(x  x1) 2x ¬30
y  (5) ¬1(x  2) x ¬15
y  5 ¬x  2 Examine: Verify that the two angles are
y ¬x  7 supplementary by substituting the value found
The slope-intercept form of the equation of the for x.
line is y  x  7. 135  2x  15 ¬135  2(15)  15
12. From Exercise 9, the slope of r is 2 
3 , so the slope ¬180
3
of a line perpendicular to r is 2. Write an So r
s.
equation in point-slope form of the line whose 6. Explore: From the figure, we know that the
slope is 3
 angles marked (2x  35)° and (3x  5)° are
2 that contains (0, 1).
alternate exterior angles for lines r and s.
y  y1 ¬m(x  x1)
Plan: For line r to be parallel to line s, alternate
y  1 ¬3
2 (x  0)
 exterior angles must be congruent, which means
their measures must be equal.
y 1 ¬3

2x Solve: 2x  35 ¬3x  5
y ¬3
2x  1
 2x  40 ¬3x
40 ¬x
The slope-intercept form of the equation of the
Examine: Verify that the two angles are
line is y  3
2 x  1.

congruent by substituting the value found for x.
13. From Exercise 10, the slope of s is 1

3 , so the 2x  35 ¬3x  5
slope of a line parallel to s is also 1

3 . Write an
2(40)  35 ¬3(40)  5
equation in point-slope form of the line whose 115 ¬115
So r
s.

511 Extra Practice


7. Explore: From the figure, we know that the y
angles marked (2x  92)° and (66  11x)° are
corresponding angles for lines r and s. p m
Plan: For line r to be parallel to line s,
corresponding angles must be congruent, which 
means their measures must be equal. x
Solve: 2x  92 ¬66  11x (0, 2)
13x  92 ¬66
13x ¬26
x ¬2
Examine: Verify that the two angles are y  y1 ¬m(x  x1)
congruent by substituting the value found for x. y  (2) ¬3
2 (x  0)

2x  92 ¬66  11x
2(2)  92 ¬66  11(2) y  2 ¬3

2x
88 ¬88 y ¬3
2x  2

So r
s. Next, use a system of equations to determine the
point of intersection of lines m and p.
Page 760 Lesson 3-6
m: y ¬2 1
3x  2

1. Since the distance from a line to a point not on
the line is the length of the segment p: y ¬3
2x  2

perpendicular to the line from the point, draw
Substitute 2 1
3 x  2 for y in the second equation.

T
P  so that P  
T RS.
P Q 2x  1 ¬3x  2
3 2 2
x  x ¬1  2
2 3
3 2 2
13 5

6 x ¬ 2
15
R T S x ¬13
15
2. Since the distance from a line to a point not on Substitute  13 for x in the first equation.
the line is the length of the segment
perpendicular to the line from the point, extend y ¬2  15
3  13  2
   1
L
K  and draw J M so that J
M .
  KL 7
y ¬ 26
J
The point of intersection is  15
7
13 ,  26 .
 
Then, use the Distance Formula to determine the
15
distance between (0, 2) and  7
13 ,  26 .
  
d ¬(x2  
x1) 
2 (y2 y1)2

2 6  (
2 2
¬  0  
15 7
2)
13 
K L M
B
F E
. Therefore

3. In octagon ABCDEFGH, A 225 15 15 
52
¬ 52   or
 
E
B F E
. So B
E
 represents the distance from B 525
 2
. 15 or about
52
to FE The distance between the lines is  52
A B 2.08 units.
5. Solve a system of equations to find the endpoints
H C
of a segment that is perpendicular to : y  2x  4
and m: y  2x  5 which is the same as
G D y  2x  5. The slope of lines  and m is 2.
First, write an equation of a line p perpendicular
F E to  and m. The slope of p is the opposite
4. Solve a system of equations to find the endpoints
reciprocal of 2, or 1
2 . Use the y-intercept of line
of a segment that is perpendicular to
, (0, 4), as one of the endpoints of the
: y  2 2 1
3 x 2 and m: y  3 x  2 . The slope of lines

perpendicular segment.
2
 and m is 3.
y
First, write an equation of a line p perpendicular (0, 4)
to  and m. The slope of p is the opposite reciprocal p
of 32, or 23. Use the y-intercept of line , (0, 2), as
one of the endpoints of the perpendicular segment.
x


Extra Practice 512


y  y1 ¬m(x  x1) Substitute 1 3
4 x  2 for y in the second equation.


y  4 ¬1 1 3


4 x  2 ¬4x  1

2 (x  0)


y  4 ¬1
 1
 3
4 x  4x ¬ 2  1
2x
17 5
y ¬1 4 x ¬ 2
2x  4

10
Next, use a system of equations to determine the x ¬
17
point of intersection of lines m and p. 10
m: y ¬2x  5 Substitute 
17 for x in the second equation.
p: y ¬12x  4
 y  4 
10
17  1
 
Substitute 2x  5 for y in the second equation. 23
y  
17
2x  5 ¬1
2x  4


2x  1
10
The point of intersection is  
23
17 ,  17 .
   
2 x ¬5  4

Then, use the Distance Formula to determine the
 
5
x ¬9 10 23
2 distance between (0, 1) and 
17 ,  17 .
  
18
x ¬5 d ¬(x2  
x1) 
2 (y2  y1)2

18
Substitute  
 17 213
  0  
7  1
2 2
5 for x in the first equation. ¬ 10

 
1 8
y  2 5  5

y 11

5
¬ 
100 10
17   or
10
 
17

17
1
 7
10 or about
17
The distance between the lines is 
The point of intersection is  18  
11

5 , 5 . 2.43 units.
17
Then, use the Distance Formula to determine the
7. 1. Graph line  and point P. Place the compass
distance between (0, 4) and  
18 1
1
5 , 5 .  point at point P. Make the setting wide enough so
d ¬(x2  
x1)2 
(y2 y1)2 that when an arc is drawn, it intersects  in two
places. Label these points of intersection A and B.
 5 
0 
151 
4
2 2
18
¬  2. Put the compass at point A and draw an arc
above line .

81
¬ 
9 9
5   or 5
  
5
5

3. Using the same compass setting, put the
9 
5 compass at point B and draw an arc to intersect
The distance between the lines is  5 or about the one drawn in step 2. Label the point of
4.02 units.
intersection Q.
6. Solve a system of equations to find the endpoints
4. Draw 
PQ. 
PQ  . Label point R at the
of a segment that is perpendicular to  and m,
intersection of 
PQ and . The segment constructed
where : x  4y  6 is the same as y  1 3
4x  2

from point P(2, 1) perpendicular to the line ,
and m: x  4y  4 is the same as y  1
The slope of lines  and m is 1 .
4 x  1.

appears to intersect line  at R 3
 5 
2 , 2 . Use the 
4 Distance Formula to find the distance between
First, write an equation of a line p perpendicular
point P and line .
to  and m. The slope of p is the opposite
reciprocal of 1
4 , or 4. Use the y-intercept of line y
m, (0, 1), as one of the endpoints of the Q B
perpendicular segment.
R
y

p A O x
m P
x


d ¬
(x2  
x1)2 
(y2 
y1)2

2 
2 2
¬  2  5  (1)
3 2
y  y1  m(x  x1)
y  1 ¬4(x  0)
4
¬ 
9 7 7
2   or 2
 
2
2

7
2
y  1 ¬4x The distance between P and  is  2 units.
y ¬4x  1 8. 1. Graph line  and point P. Place the compass
Next, use a system of equations to determine the point at point P. Make the setting wide enough so
point of intersection of lines  and p. that when an arc is drawn, it intersects  in two
: y  1 3 places. Label these points of intersection A and B.
4 x 2

p: y  4x  1

513 Extra Practice


2. Put the compass at point A and draw an arc 9. TAC is isosceles with T A
A C
, so TA  AC.
above line . Therefore,
3. Using the same compass setting, put the 3b  1 ¬4b  11
compass at point B and draw an arc to intersect 1  11 ¬3b  4b
the one drawn in step 2. Label the point of 12 ¬b
intersection Q. Next, substitute to find the length of each side.
4. Draw 
PQ. 
PQ  . Label point R at the TA  3b  1 AC  4b  11
 and . The segment constructed
intersection of PQ  3(12)  1  4(12)  11
from point P(2.5, 3) perpendicular to the line ,  37  37
appears to intersect line  at R(1.4, 3.8). Use the TC  6b  2
Distance Formula to find the distance between  6(12)  2
point P and line .  70
For TAC, b  12, TA  AC  37, and TC  70.
y
A Q Page 761 Lesson 4-2
P R 1–4. Find m2 first because the measures of two
angles of the triangle are known.
B m2  50  70  180
m2  120 ¬180
O x m2 ¬60
 1 and 2 are congruent vertical angles. So
m1  60. 1 and 4 form a linear pair.
d ¬
(x2  
x1)2 
(y2 
y1)2 m1  m4 ¬180
60  m4 ¬180
¬[1.4
  (2
.5)]2 
[3.8 3]2 m4 ¬120
¬1.85
 or about 1.4 Use the Angle Sum Theorem to find m3.
The distance between P and  is approximately m3  65  m1 ¬180
1.4 units. m3  65  60 ¬180
m3  125 ¬180
Page 760 Lesson 4-1 m3 ¬55
1. All three angles are congruent, so the triangle is Therefore, m1  60, m2  60, m3  55,
equiangular. and m4  120.
2. The triangle has one right angle. So the triangle 5. Use the Angle Sum Theorem.
is a right triangle. m5  32  54 ¬180
3. The triangle has one obtuse angle ( 90°). So the m5  86 ¬180
triangle is obtuse. m5 ¬94
4. Right triangles have one right angle. So, DAB, 6. 5 and 6 form a linear pair. From Exercise 5,
ABC, BCD, and ADC are right triangles. m5  94.
m5  m6 ¬180
5. Obtuse triangles have one obtuse angle. ABE
94  m6 ¬180
and CDE are obtuse.
m6 ¬86
6. Acute triangles have all acute angles. BEC and
7. 5 and 7 are vertical angles. So 5  7.
AED are acute.
m7  94
7. Isosceles triangles have at least two sides
8. 6 and 8 are vertical angles. So 6  8.
congruent. So, ABE, CDE, BEC, and AED
m8  86
are isosceles.
9. Use the Angle Sum Theorem.
8. Since MNO is equilateral, each side has the
m9  m8  42 ¬180
same length. So MN  NO. Therefore,
m9  86  42 ¬180
5a ¬4a  6
m9  128 ¬180
a ¬6
m9 ¬52
Next, substitute to find the length of each side.
MN  5a NO  4a  6 10. Use the Angle Sum Theorem.
 5(6)  4(6)  6 m10  m7  62 ¬180
 30  30 m10  94  62 ¬180
MO  7a  12 m10  156 ¬180
 7(6)  12 m10 ¬24
 30
For MNO, a  6 and the measure of each side is Page 761 Lesson 4-3
30. 1. ABC  FDE
2. JKH  JIH
3. RTS  UVW
4. LMN  NOP

Extra Practice 514


5. Given: ANG  NGA G 4. SWG  IWN 4. Given
NGA  GAN
Prove: AGN is equilateral 5. SWG  IWN 5. SAS
and equiangular.
Proof: A N 4. Given: ANM ¬ANI

DI ¬O
M
Statements Reasons D
N  ¬NO

1. ANG  NGA 1. Given Prove: DIN  OMN
I M
N
2. AN
G
, A  N 2. CPCTC A
3. NGA  GAN 3. Given
D O
G
4. N GA
, N  G 4. CPCTC
N
5. AN G
G A 5. Transitive Property
N
6. AGN is equilateral. 6. Definition of
equilateral triangle Proof:
7. A  N  G 7. Transitive Property Statements Reasons
8. AGN is 8. Definition of 1. ANM  ANI 1. Given
equiangular. equiangular triangle
2. IN
M
N
 2. CPCTC
I  O
3. D M
 3. Given
Page 761 Lesson 4-4
1. Use the Distance Formula to show that the D
4. NN
O
 4. Given
corresponding sides are congruent. 5. DIN  OMN 5. SSS
RS ¬ [6  (4)]2 [2 4]2
¬4  4 or  8 Page 762 Lesson 4-5
JK ¬ [6  4]2  [2  ( 4)]2 1. Given: TEN is isosceles E
¬4  4 or  8 N
with base T.
ST ¬ [4  (2)]2 [4 2]2 1  4, T  N
Prove: TEC  NEA 1 2 3 4
¬4  4 or  8 T C A N
KL ¬ [4  2]2  [4  (2)]2
Proof: If TEN is isosceles with base T
N
, then
¬4  4 or  8
E
TN E. Since 1  4 and T  N are
RT ¬ [6  (2)]2 [2 2]2 given, then TEC  NEA by AAS.
¬16 or 4 2. Given: S  W, S V
JL ¬ [6  2]2  [2  (2)]2 Y
S Y W

¬16 or 4 Prove: STW V
RS  JK, ST  KL, and RT  JL. By definition of Y
congruent segments, all corresponding segments
are congruent. Therefore, RST  JKL by SSS. T W
Proof: S  W and SY
YW are given, and
2. Use the Distance Formula to show that
SYT  WYV since vertical angles are
corresponding sides are not congruent.
congruent. Then SYT  WYV by ASA and
RS ¬ [6  (4)]2  [3 7]2 T
SW V by CPCTC.
¬4  16 or  20 3. Given: 1  2, P L
JK ¬ (2  5)
2  (3  7)2 3  4 1 2
¬9  16 or 5 Prove: 
PT  
LX R
Since RS  JK, RST   JKL. 4 3
3. Given: GWN is W T X
equilateral. Proof:
  W
WS I
S I 1 2 3 4 TX TX
SWG  IWN
Given Given Reflexive
Prove: SWG  IWN
Prop.
Proof: G N
Statements Reasons PXT LTX
1. GWN is 1. Given AAS
equilateral.
G
2. WW
N
 2. Definition of PT LX
equilateral triangle CPCTC
S
3. WW
I 3. Given

515 Extra Practice


4. Given: 
FP

ML, F L
• Since Z is on the x-axis, its y-coordinate is 0. Its

MP
FL  1 x-coordinate is 4b because the base of the
3 triangle is 4b units long.
Prove: 
MP  
FL 4
2 • Because XYZ is equilateral and thus
P M Y
X Z Y, the x-coordinate of Y is halfway
between X and Z at 2b. We cannot determine
Proof: the y-coordinate in terms of b, so call it c.
FP || ML FL || MP y
Y(2b, c)
Given Given

3 4 1 2 PL PL Z(4b, 0)
Alt. Int. Alt. Int. Reflex. X(0, 0) x
Th. Th. Prop.
3. • Use the origin as vertex S of the triangle.
FLP MPL • Place the base of the triangle, SR
, along the
ASA positive x-axis.
• Position the triangle in the first quadrant.
MP FL • Since R is on the x-axis, its y-coordinate is 0. Its
CPCTC x-coordinate is (3  a) because the base of the
triangle is (3  a) units long.
• Because R T
 is vertical, the x-coordinate of T is
Page 762 Lesson 4-6
(3  a). And because RT  3  a, the
1. DAB is opposite BD and DBA is opposite AD
,
y-coordinate of T is (3  a).
so DAB  DBA. y
2. FBG is opposite FG and FGB is opposite B
F, T(3 a, 3 a)
so FBG  FGB.
3. BEF is opposite BG and BGF is opposite B
E, R(3 a, 0)
so BEF  BGF. S(0, 0) x
4. By the converse of the Isosceles Triangle
Theorem, the sides opposite congruent angles are 4. • Use the origin as vertex D of the triangle.
B
congruent. So, FF E
. • Place the base of the triangle along the positive
5. By the converse of the Isosceles Triangle x-axis.
Theorem, the sides opposite congruent angles are • Position the triangle in the first quadrant.
A
congruent. So, B B C
. • Since E is on the x-axis, its y-coordinate is 0. Its
6. By the converse of the Isosceles Triangle x-coordinate is 1

4 b because the base of the
Theorem, the sides opposite congruent angles are triangle is 1
b units long.
4
D
congruent. So, B C D
. • Because CDE is equilateral and thus  C
D C E,
the x-coordinate of C is halfway between D and
Page 762 Lesson 4-7 E at 1

8 b. We cannot determine the y-coordinate
1. • Use the origin as vertex B of the triangle.
in terms of b, so call it c.
• Place the base of the triangle along the positive
y
x-axis. C(1–8b, c)
• Position the triangle in the first quadrant.
• Since C is on the x-axis, its y-coordinate is 0. Its E(1–4b, 0)
x-coordinate is r because the base of the D(0, 0) x
triangle is r units long.
• Since ABC is isosceles, the x-coordinate of A 5. Vertex A is on the y-axis, at an unspecified
is halfway between 0 and r or 2r. We cannot distance from the origin. Its coordinates are (0, b).
determine the y-coordinate in terms of r, so call Since B is on the x-axis, the y-coordinate of B is 0.
it b. Because ABC is isosceles, the x-coordinate of A
y is halfway between the x-coordinate of B and C.
A(r–2, b) So the x-coordinate of B is a. The coordinates of
B are (0, a).
C(r, 0) 6. Side FE
 can be seen to be vertical, so the
B(0, 0) x x-coordinate of F is the same as the x-coordinate
of E, namely b. F is the same distance from E as
2. • Use the origin as vertex X of the triangle.
E is from the origin, namely b units. So the
Z
• Place the base of the triangle, X , along the y-coordinate of F is b. The coordinates of F are
positive x-axis. (b, b).
• Position the triangle in the first quadrant.

Extra Practice 516


7. Vertex I is on the y-axis, at an unspecified 8. Always; unlike altitudes, the angle bisectors of a
distance from the origin; its coordinates are (0, b). triangle always run through the interior of the
Since G is on the x-axis, the y-coordinate of G is 0. triangle and intersect inside the triangle.
Because GHI is isosceles, the x-coordinate of I is
halfway between the x-coordinates of G and H. So, Page 763 Lesson 5-2
the x-coordinate of G is a  2. The coordinates of 1. The side opposite TPS is longer than the side
G are (a  2, 0). opposite TSP, so mTPS  mTSP.
2. The side opposite PRZ is longer than the side
Page 763 Lesson 5-1 opposite ZPR (27.5  12  24.5), so
1. Assume that S is the circumcenter. Then AP is mPRZ  mZPR.
the perpendicular bisector of CB. So CP  PB. 3. The side opposite SPZ is shorter than the side
7x  1 ¬6x  3 opposite SZP (17.2  34), so mSPZ  mSZP.
x ¬4
4. The side opposite SPR is the same length as the
CP ¬7x  1
side opposite SRP (15  19  34), so
CP ¬7 4  1  27
mSPR  mSRP.
2. Find a. Because S is the incenter, 
CT bisects
5. Given: FH  FG
ACB.
Prove: m1  m2
mACT ¬1 
2 (mACB) E H
15a  8 ¬1
2 (74) 1 2
G
15a  8 ¬37
15a ¬45
a ¬3 F
Find mACT. Proof:
mACT  15a  8 Statements Reasons
mACT  15(3)  8 1. FH  FG 1. Given
mACT  37
2. mFGH  m2 2. If one side of a triangle
3. Find b. Because Z is the centroid, O is the is longer than another,
R
midpoint of T . the angle opposite the
TO ¬OR longer side is greater
7b  5 ¬13b  10 than the angle opposite
5  10 ¬7b  13b the shorter side.
15 ¬6b
3. m1  mFGH 3. Exterior Angle
2.5 ¬b
Inequality Theorem
Find TR. 4. m1  m2 4. Transitive Property of
TR  TO  OR Inequality
TR  7b  5  13b  10
TR  20b  5  bisects SRT.
6. Given: RQ R
TR  20(2.5)  5
TR  45 Prove: mSQR  mSRQ
4. Find n. Because Z is the centroid,
ZR ¬2

3 XR S Q T
10n  4 ¬2
3 (19n  14)

Proof:
10n  4 ¬38
3n   28
3
 Statements Reasons
3 8 2
0n  3n ¬4  38  bisects SRT.
1. RQ 1. Given
8 4
0 2. SRQ  QRT 2. Definition of angle
3 n ¬ 3
 
bisector
n ¬5
3. mSRQ  mQRT 3. Definition of congruent
Find ZR. angles
ZR  10n  4 4. mSQR  mQRT 4. Exterior Angle
ZR  10(5)  4 Inequality Thorem
ZR  54
5. mSQR  mSRQ 5. Substitution
5. Sometimes; they are the same for an equilateral
triangle but not for all triangles.
6. Sometimes; the three altitudes intersect inside Page 763 Lesson 5-3
the triangle when the triangle is acute, but not 1. ABC  XYZ
when it is obtuse. 2. An angle bisector of an equilateral triangle is not
7. Always; the symmetry of an equilateral triangle a median.
guarantees this.  does not bisect ARC.
3. RS

517 Extra Practice


4. Given: AOY  AOX 5. Check each inequality.
? ? ?

XOY O
 15  20¬ 30 15  30¬ 20 20  30¬ 15
 is not the angle bisector of XAY.
Prove: AO 35¬ 30 ✓ 45¬ 20 ✓ 50¬ 15 ✓
X All of the inequalities are true, so 15, 20, and 30
can be the lengths of the sides of a triangle.
?
O 6. 1  3  5
A 4
¬5
Because the sum of the two smaller measures is
Y not greater than the largest measure, the measures
cannot be the lengths of the sides of a triangle.
Indirect Proof: ?
Step 1: Assume  AO is the angle bisector of XAY. 7. 2.5  3.5  6.5
 is the angle bisector of XAY, then
Step 2: If AO 6
¬6.5
XAO  YAO. AOY  AOX is given Because the sum of the two smaller measures is
not greater than the largest measure, the measures
as true, and AO
A O by the Reflexive Property of
cannot be the lengths of the sides of a triangle.
Congruence. Then XAO  YAO by ASA, and
O
XY O by CPCTC. 8. Check each inequality.
? ? ?
Step 3: This conclusion contradicts the given fact 0.3  0.4¬ 0.5 0.3  0.5¬ 0.4 0.4  0.5¬ 0.3
O
X Y . Thus, 
O AO is not the angle 0.7¬ 0.5 ✓ 0.8¬ 0.4 ✓ 0.9¬ 0.3 ✓
bisector of XAY. All of the inequalities are true, so 0.3, 0.4, and 0.5
5. Given: RUN can be the lengths of the sides of a triangle.
Prove: There can be no more than one right 9. Let the measure of the third side be n, and solve
angle in RUN. each inequality to determine the range of values
Indirect Proof: for n.
Step 1: Assume RUN has two right angles. 6  10  n 6  n  10 n  10  6
Step 2: By the Angle Sum Theorem, mR  16  n or n  16 n4 n  4
mU  mN  180. If you substitute 90 Graph the inequalities on the same number line.
for two of the angle measures, since the n  16
triangle has two right angles, then 90  16
90  mN  180 or 180  mN  180.
Step 3: This conclusion means that mN  0. n4
This is not possible if RUN is a triangle. 4
Thus, there can be no more than one
right angle in RUN. n  4
4
Page 764 Lesson 5-4
intersection
?
1. 2  2 ¬6 4 16
4
¬6
Because the sum of the two smaller measures is The range of values that fits all three is
not greater than the largest measure, the measures 4  n  16.
cannot be the lengths of the sides of a triangle. 10. Let the measure of the third side be n, and solve
2. Check each inequality. each inequality to determine the range of values
? ? ? for n.
2  3 ¬4 2  4¬¬3 3  4¬¬2
5 ¬4 ✓ 6¬¬3 ✓ 7¬¬2 ✓ 25n 2n5 n52
7  n or n  7 n3 n  3
All of the inequalities are true, so 2, 3, and 4 can
Graph the inequalities on the same number line.
be the lengths of the sides of a triangle.
n7
3. Check each inequality. 7
? ? ?
6  8¬ 10 6  10¬ 8 8  10¬ 6
14¬ 10 ✓ 16¬ 8 ✓ 18¬ 6 ✓ n3
All of the inequalities are true, so 6, 8, and 10 can 3
be the lengths of the sides of a triangle.
? n  3
4. 1  1  2
2
¬2 3
Because the sum of the two smaller measures is
intersection
not greater than the largest measure, the measures
3 7
cannot be the lengths of the sides of a triangle.
The range of values that fits all three is
3  n  7.

Extra Practice 518


11. Let the measure of the third side be n, and solve 14. Let the measure of the third side be n, and solve
each inequality to determine the range of values each inequality to determine the range of values
for n. for n.
12  20  n 12  n  20 n  20  12 32  34  n 32  n  34 n  34  32
32  n or n  32 n8 n  8 66  n or n  66 n2 n  2
Graph the inequalities on the same number line. Graph the inequalities on the same number line.
n  32 n  66
32 66

n8 n2
8 2
n  2
n  8 2
8
intersection
intersection 2 66
8 32
The range of values that fits all three is
The range of values that fits all three is 2  n  66.
8  n  32. 15. Let the measure of the third side be n, and solve
12. Let the measure of the third side be n, and solve each inequality to determine the range of values
each inequality to determine the range of values for n.
for n. 2  29  n 2  n  29 n  29  2
88n 8n8 n88 31  n or n  31 n  27 n  27
16  n or n  16 n0 n0 Graph the inequalities on the same number line.
Graph the inequalities on the same number line. n  31
31
n  16
16
n  27
27
n0
0 n  27
27
intersection
intersection
0 16 27 31
The range of values that fits all the inequalities is The range of values that fits all three is
0  n  16. 27  n  31.
13. Let the measure of the third side be n, and solve 16. Let the measure of the third side be n, and solve
each inequality to determine the range of values each inequality to determine the range of values
for n. for n.
18  36  n 18  n  36 n  36  18 25  80  n 25  n  80 n  80  25
54  n or n  54 n  18 n  18 105  n or n  105 n  55 n  55
Graph the inequalities on the same number line. Graph the inequalities on the same number line.
n  54 n  105
54 105

n  18 n  55
18 55

n  18 n  55
18
55
intersection
18 54 intersection
55 105
The range of values that fits all three is
The range of values that fits all three is
18  n  54.
55  n  105.

519 Extra Practice


17. Given: RS  RT U V 6. B
Prove: UV  VS  UT
6.56
15 C
R
25
T S
A 55 2x  7
Proof:
15
Statements Reasons D

1. RS  RT 1. Given B
A A D, A
C
A C, and mCAD  mCAB. By
the SAS Inequality, CD  CB.
2. UV  VS  US 2. Triangle Inequality
CD  CB
Theorem
2x  7  6.56
3. US  UR  RS 3. Segment Addition 2x  13.56
Postulate x  6.78
4. UV  VS  UR  4. Substitution Also, by the Triangle Inequality Theorem,
RS AB  AD  BD.
5. UV  VS  UR  5. Substitution AB  AD  BD
RT 15  15  6.56  2x  7
6. UR  RT  UT 6. Triangle Inequality 30  2x  0.44
Theorem 30.44  2x
15.22  x
7. UV  VS  UT 7. Transitive Property
The two inequalities can be written as the
of Inequality
compound inequality 6.78  x  15.22.
18. Given: quadrilateral ABCD 7. 3x  1
A B A B
Prove: AD  CD  AB  BC
45
3x  1
3x  1
3x  1
D C
Proof:
D C
5x  21
Statements Reasons
Because ABD is equilateral, mABD  60 and
1. quadrilateral ABCD 1. Given so mABD  mDBC. Since A BC B and D
B
C
2. Draw A. 2. Through any two B
D , the SAS inequality allows us to conclude that
points there is AD  DC.
exactly one line. AD  DC
3. AD  CD  AC 3. Triangle Inequality 3x  1  5x  21
AB  AC  BC Theorem 1  21  3x  5x
4. AC  BC  AB 4. Subtraction 20  2x
Property of 10  x
Inequality Also, the measure of any side is always greater
5. AD  CD  BC  than 0.
5. Transitive Property
AB 5x  21  0
of Inequality
5x  21
6. AD  CD  AB  6. Addition Property x  4.2
BC of Inequality The two inequalities can be written as the
compound inequality 4.2  x  10.

Page 764 Lesson 5-5 Page 764 Lesson 6-1


1. XZ  32 and OZ  30, so XZ  OZ. 1. Write the given information, using x for the
2. In ZIO and ZUX, Z I  U
X
, IO
UZ
, and actual house width in the substitution.
XZ  OZ. The SSS inequality allows us to model height model width

actual 
height
 actual 
width
conclude that mZIO  mZUX. 1 1 6
3. AEZ is an isosceles triangle with AE
A Z
. By
63  x
the Isosceles Triangle Theorem, the angles 1 x  (63)(16)
opposite congruent sides are congruent. So x  1008
mAEZ  mAZE. The actual width of the house is 1008 in., which
4. IO  18 and AE  30, so IO  AE. equals 1008 12 or 84 ft.
5. In AZE and IZO, A Z
Z O, Z
EZ
I, and
AE  IO. The SSS inequality allows us to
conclude that mAZE  mIZO.

Extra Practice 520


2. Write the given information, using x for the Page 765 Lesson 6-2
smaller length. 1. By the Angle Sum Theorem, mA  21.8  38.2 
x¬ 2 180, which means that mA  180  21.8  38.2
64  x 3
x 3¬ (64  x)(2) or 120. So mA  mX, and therefore A  X.
3x¬ 128  2x Similarly, mY  120  38.2  180 and mY 
5x¬ 128 180  120  38.2 or 21.8. So mB  mY, and
x¬ 25.6 therefore B  Y. Since mC  mZ,
The two lengths are 25.6 in. and 64  25.6 or C  Z. Thus, all corresponding angles are
38.4 in. congruent.
x 4 Now determine whether corresponding sides are
1
26   3
3. 
proportional.
(x  4)(3)  26(1) AB 12.5 BC 17
.5
XY  5 or 2.5 YZ  7 or 2.5
     
3x  12  26
AC 7.5
XZ  3 or 2.5
3x  38    
38
x   3 The ratios of the measures of the corresponding
3x  1 sides are equal, and the corresponding angles are
5
14  7
4.  
congruent, so ABC  XYZ.
(3x  1)(7)  14(5)
2. S  W. T  X. U  Y. R  V.
21x  7  70
Thus, all of the corresponding angles are congruent.
21x  63
Now determine whether corresponding sides are
x3
proportional.
x 
 3 x1
5. 4 ¬ 5 R

VW
S
 
4 or 1.5 ST

W
6
X  4 or 1.5
8
(x  3)(5)¬ 4(x  1) 3
5x  15¬ 4x  4 TU RU
XY   VY  
 4 or 1.5   10 or 1.5
8 2
0
5x  4x¬ 15  4 3 3
x¬ 19 The ratios of the measures of the corresponding
2x  2 sides are equal, and the corresponding angles are
1
2x  1  3
6.  
congruent, so polygon RSTU  polygon VWXY.
(2x  2)(3)  (2x  1)(1) 3. The triangle remains the same shape and size,
6x  6  2x 1 but each vertex is shifted to the left 3 units and
6x  2x  6  1 down 3 units. R(3, 6) becomes R (0, 3), S(1, 2)
4x  7 becomes S (2, 1), and T(3, 1) becomes
x  7
4
 T (0, 4). The new triangle is both congruent and
7. Rewrite 9 : 6 : 5 as 9x : 6x : 5x and use those similar to the original.
measures for the sides of the triangle. Write an 4. The triangle remains the same shape, but the
equation to represent the perimeter of the length of each side is one half the length of the
triangle as the sum of the measures of its sides. corresponding sides of the original triangle.
9x  6x  5x¬ 100 The new triangle is similar to the original.
20x¬ 100
x¬ 5 Page 765 Lesson 6-3
Use this value of x to find the measures of the 1. mN  mX, so N  X. If the measures of the
sides of the triangle. corresponding sides that include the angles are
9x  9(5)  45 proportional, then the triangles are similar.
6x  6(5)  30 YX 12.5 XZ 1
5
LN  5 or 2.5 and NM  6 or 2.5
   
5x  5(5)  25
YX X
Z
LN  NM . So, by SAS Similarity,
By substitution, 
The side lengths are 45 inches, 30 inches, and  
25 inches. LNM  YXZ.
8. Rewrite 13 : 16 : 21 as 13x : 16x : 21x and use 2. mC  mR so C  R. By the Angle Sum
those measures for the angles of the triangle. Theorem, mB  70  22  180, which means
Write an equation for the sum of the angles, using that mB  180  70  22 or 88. Therefore
the Angle Sum Theorem. mB  mS, and so B  S. By AA Similarity,
13x  16x  21x¬ 180 ABC  TSR.
50x¬ 180
x¬ 3.6
Use this value of x to find the measures of the
angles of the triangle.
13x  13(3.6)  46.8
16x  16(3.6)  57.6
21x  21(3.6)  75.6
The angle measures are 46.8, 57.6, and 75.6.

521 Extra Practice


25 8x  2
3. Since 
RT

SQ, TRV  QSV and RTV     
15 ¬ 3x  6
SQV. By AA Similarity, RTV  SQV. By the 8x 
5¬   2
definition of similar polygons, 3 3x  6
RV TV 5(3x  6)¬ 3(8x  2)
SV  QV .
  
15x  30¬ 24x  6
By the Segment Addition Postulate, RV  15x  24x¬ 30  6
RS  SV and TV  TQ  QV. 9x¬ 36
RS  SV TQ  QV x¬ 4

SV  QV Find AD and DR by substitution.
15  7
 x9 12  24
7x  9 ¬ 24 AD  3x  6
7x 
 24 3
AD  3(4)  6
7x  9 ¬ 2 AD  18
(7x  24)(2)¬ (7x  9)(3) DR  8x  2
14x  48¬ 21x  27 DR  8(4)  2
14x  21x¬ 48  27 DR  30
7x¬ 21 QR
UD, so U  Q and D  R since they
x¬ 3 are pairs of corresponding angles. AQR  AUD
Use the definition of similar polygons and by AA similarity.
substitution again. QR AR
By the definition of similar polygons, 
UD 
AD,
RT TV

SQ Q V and by the Segment Addition Postulate,
R
 T
 TQ 
 QV AR  AD  DR. Use substitution.
SV QV QR AD  DR
RT 12  24 
U

18  
 D AD
24 QR 18  30
RT 3  
18  2
 15 18
QR 48
2(RT)  (18)(3) 
15 18

2(RT)  54 QR 8
   3
RT  27 15
Finally, find SV by substitution. QR  158 3

SV  7x  9 QR  40
SV  7(3)  9 So x  4, AD  18, DR  30, and QR  40.
SV  30
So RTV  SQV, x  3, RT  27, and SV  30. 3. By the Converse of the Triangle Proportionality
Theorem, x should be chosen so that

OP
4. Since LM , LMN  OPN and MLN 
XL YM
PON. By AA Similarity, MNL  PNO. Use 
LD 
MD.
the definition of similar polygons. Substitute known values and solve for x.
M
N LN  3   5
PN ¬ ON 9 x 3
5x 
 2 7 3(x  3)  9(5)
x 5 ¬ 5
3x  9  45
(5x  2)(5)¬ (x  5)(7)
3x  36
25x  10¬ 7x  35
25x  7x¬ 10  35 x  12
18x¬ 45 4. By the Converse of the Triangle Proportionality
x¬ 2.5 Theorem, x should be chosen to that
XL YM
Find PN and MN by substitution. 
LD M D.
PN  x  5 MN  5x  2 Substitute known values and solve for x.
PN  2.5  5 MN  5(2.5)  2  4 3
PN  7.5 MN  10.5 3x  1 ¬ x  7
So MNL  PNO, x  2.5, PN  7.5, and 4(x  7)¬ (3x  1)(3)
MN  10.5. 4x  28¬ 9x  3
4x  9x¬ 28  3
5x¬ 25
Page 765 Lesson 6-4
x¬ 5
1. From the Triangle Proportionality Theorem,
5. By the Converse of the Triangle Proportionality
IJ LK

HI LH. Substitute the known measures. Theorem, x should be chosen to that
IJ 8 XL YM
8 
2 21 
LD 
MD.
IJ  28 8
21
Substitute known values and solve for x.

IJ  3 or 10 2
3 2 
3
2. From the Triangle Proportionality Theorem,
QU DR
AU  AD .
  
Substitute the known measures.

Extra Practice 522


 5 3 2.
5x  1 ¬ 5x  6
5(5x  6)¬ (5x  1)(3)
25x  30¬ 15x  3
25x  15x¬ 30  3
10x¬ 33
x¬ 3.3
There are 1 large shaded square, 3 small shaded
Page 766 Lesson 6-5 squares, and 9 very small shaded squares, or
AC BC 13 shaded squares in all.
1. By the definition of similar polygons, 
DE  
BE .
Substitute known values. 3. Use a calculator. The sequence of values is
AC 15 approximately 6, 1.565, 1.118, 1.028, 1.007,
5  6 1.002, .... The values converge to 1.
AC  5 6 
15 4. Use a calculator. The sequence of values is
AC  12.5 approximately 0.4, 1.741, 11.175, 5345856.098,
The perimeter of ABC is 15  17.5  12.5 or and then comes an overflow error. The values
45 units. approach positive infinity.
2. Let x represent the perimeter of RST. Use the 5. Use a calculator. The sequence of values is
Proportional Perimeter Theorem. approximately 0.5, 0.125, 0.002, .... The values
XZ perimeter of XYZ converge to 0.

RT  
perimeter of RST 6. Use a calculator. The sequence of values is
8  10, 59049, and then comes an overflow error. The

12  2x2

values approach positive infinity.
  x2
2 2 
3
2x  3(22) Page 766 Lesson 7-1
2x  66 1. Let x represent the geometric mean.
x  33 8 x
x  
12
The perimeter of RST is 33 units.
x2  96
3. Let x represent the perimeter of LMN. The
x  96

perimeter of NXY is 14  11  9 or 34. Use the
x  46
Proportional Perimeter Theorem.
The geometric mean is 46  or approximately 9.8.
YN perimeter of NXY
LN  
 2. Let x represent the geometric mean.
perimeter of LMN
15
  x
9 
 3x4
 x ¬ 20
27
x2¬ 300
1  
34
3 x x¬ 300
1x  3(34) x¬ 103 
x  102 The geometric mean is 103  or approximately
The perimeter of LMN is 102 units. 17.3.
4. Let x represent the perimeter of GHI. Use the 3. Let x represent the geometric mean.
Proportional Perimeter Theorem. 1 x
perimeter of ABC x  2
AB
¬ 
GH perimeter of GHI x2  2
x  2 
6 

10 ¬ 2x5
 The geometric mean is 2  or approximately 1.4.
¬ x5
3 2  4. Let x represent the geometric mean.
5
4
x  
x
16
3x¬ 5(25)
3x¬ 125 x  64
2

12
5 x  64 
x¬ 3 x8
12
5 2
The perimeter of GHI is 3 or 41 3 units. The geometric mean is 8.
5. Let x represent the geometric mean.
Page 766 Lesson 6-6 3 
2 x
x   
62
1.
x2  36
x  36

x6
The geometric mean is 6.

There are 1 large shaded square and 3 small


shaded squares, or 4 shaded squares in all.

523 Extra Practice


6. Let x represent the geometric mean.
1
EF   (2  3 )2  (3 2)2

2 x  (1)
 2  2
x 
 1
10
 2
x2  5
DF   (2  0)2  (3 1)2
x  5 
The geometric mean is 5 or approximately 2.2.   22  22
 8 
7. Let x represent the geometric mean.
3
By the converse of the Pythagorean Theorem, if the
x
x  1
8
 sum of the squares of the measures of two sides of
2
3
a triangle equals the square of the measure of the
x2  
16 longest side then the triangle is a right triangle.
x  
3
16
DE 2¬ DF 2  EF 2

3 )2¬ (8
(10 )2  (2 )2
x4 10¬ 8  2

3
The geometric mean is  4 or approximately 0.4. 10¬ 10
8. Let x represent the geometric mean. Since the sum of the squares of two sides equals
2


x the square of the longest side, DEF is a right
x  
2
 3
 
2 triangle.
2
3 2. Use the Distance Formula to determine the
x  2
2
lengths of the sides.

x  3 2

d   (x2  x1)2 (y2 y1)2
 
6
x 2 DE   
[3  (2)] 2
 (1  2)2

6
The geometric mean is  2 or approximately 1.2.   5  (
2 3) 2

9. Let x represent the geometric mean.  34


1
10 x EF   (4  3)2  (3  (1)]2
x  

7  (7)
 2  (2 )2
10

x2  7  53
100
DF   [4  (2)]2  (3  2)2
x 
7
100   (2) 
2  (5 ) 2


7  29
x
10

7
By the converse of the Pythagorean Theorem, if the
The altitude is 10 or approximately 0.3. sum of the squares of the measures of two sides of
10. Let x represent the altitude. a triangle equals the square of the measure of the
12 x
x  
 longest side then the triangle is a right triangle.
32
x2  384 EF 2¬ DE2  DF 2
x  384
 )2¬ (34
(53 )2  (29 )2
x  86  53¬ 34  29
The altitude is 86  or about 19.6. 53¬ 63
11. Let x represent the altitude. Since the sum of the squares of two sides does not
4
2 equal the square of the longest side, DEF is not
x
x  42

 a right triangle.
x2  32 3. Use the Distance Formula to determine the
x  32 lengths of the sides.
x  42 
d   (x2  x1)2 (y2  y1)2
The altitude is 42 or about 5.7.
12. Let x represent the altitude. DE   (2  2)2 [4 (1)]2
7 x   (4) 
2  (3)2
x  24  25
x  168
2
5
x  168 
x  242  EF   [4  (2)]2  ( 2
1  (4)]
 or about 13.0.
The altitude is 242  (2)
 2  3 2

 13

Page 767 Lesson 7-2 DF   (4 2)2 [1  (1)]2
1. Use the Distance Formula to determine the   (6) 
2 0 2

lengths of the sides.  36


d  (x2  
x1)2  (y2 
y1)2 6
By the converse of the Pythagorean Theorem, if the
DE  
(3  0)
2  (2
 1)2 sum of the squares of the measures of two sides of
 
32  12 a triangle equals the square of the measure of the
 10
 longest side then the triangle is a right triangle.

Extra Practice 524


DF 2¬ DE2  EF 2 8. Since the measure of the longest side is 7, 7 must
62¬ 52  (13)2 be c, and a and b are 2 and 5.
36¬ 25  13 a2  b2¬ c2
36¬ 38 22  52¬ 72
Since the sum of the squares of two sides does not 4  25¬ 49
equal the square of the longest side, DEF is not 29¬ 49
a right triangle. Since 29  49, segments with these measures
cannot form a right triangle. Therefore, they do
4. Use the Distance Formula to determine the
not form a Pythagorean triple.
lengths of the sides.
9. Since the measure of the longest side is 51, 51
d   (x1  
x2)2 (y1 
y2)2
must be c, and a and b are 24 and 45.
DE   (5  1)2  (2  2)2 a2  b2¬ c2
  4  (
2 4) 2 242  452¬ 512
 32 576  2025¬ 2601
EF   (2  5)2 [1  (2)]2 2601¬ 2601
  (7) 
2 1 2 These segments form the sides of a right triangle
 50 since they satisfy the Pythagorean Theorem. The
measures are whole numbers and form a
DF   (2  1)2  (1  2)2
Pythagorean triple.
  (3) 
2  (3)2
26

10. Since the measure of the longest side is  
3 ,
 18  
26 1 5
 must be c, and a and b are  and .
By the converse of the Pythagorean Theorem, if 3 3 3
the sum of the squares of the measures of two a2  b2¬ c2
2 2 2
26

sides of a triangle equals the square of the 13  5
3  ¬  3 
 
measure of the longest side, then the triangle is a
1  25 26

right triangle. 9 9 ¬ 9
EF 2¬ DE2  DF 2 2 6
¬ 2 6
9 9
)2¬ (32
(50 )2  (18 )2 2 6
Since 9   26
9 , segments with these measures
50¬ 32  18 form a right triangle. However, the three numbers
50¬ 50 are not whole numbers. Therefore, they do not
Since the sum of the squares of two sides equals form a Pythagorean triple.
the square of the longest side, DEF is a right 10 1
0
11. Since the measure of the longest side is  
11 , 11
triangle. 6
must be c, and a and b are  8
5. Since the measure of the longest side is 2, 2 must 11 and 11 .
be c, and a and b are both 1. a2  b2¬ c2
2 2 2
a2  b2¬ c2 161  8
11  ¬ 11 
10

12  12¬ 22 3
6 64 10
0
121  121 ¬ 121
  
1  1¬ 4
2¬ 4 10
 0 10
 0
121 ¬ 121
Since 2  4, segments with these measures 10
0 10
0
121  121 , segments with these measures
Since 
cannot form a right triangle. So, they do not form 
a Pythagorean triple. form a right triangle. However, the three numbers
6. Since the measure of the longest side is 35, 35 are not whole numbers. Therefore, they do not
must be c, and a and b are 21 and 28. form a Pythagorean triple.
a2  b2¬ c2 12. Since the measure of the longest side is 1, 1 must
212  282¬ 352 be c, and a and b are both 1

2.
441  784¬ 1225 a  b ¬ c
2 2 2
1225¬ 1225 2 2
These segments form the sides of a right triangle 12  1
2  ¬ 1
 2

since they satisfy the Pythagorean Theorem. The 1  1¬ 1


4 4
measures are whole numbers and form a
Pythagorean triple. 1¬ 1
2
7. Since the measure of the longest side is 7, 7 must Since 1  1, segments with these measures
2
be c, and a and b are 3 and 5. cannot form a right triangle. Therefore, they do
a2  b2¬ c2 not form a Pythagorean triple.
32  52¬ 72
9  25¬ 49
34¬ 49
Since 34  49, segments with these measures
cannot form a right triangle. Therefore, they do
not form a Pythagorean triple.

525 Extra Practice


2
40 21
13. Since the measure of the longest side is  
15 , 
35
2
40 
6 10

 
15 must be c, and a and b are 3 and 5.  3

5 or 0.60
a2  b2¬ c2 adjacent leg
cosM   
hypotenuse
2 2 2
36  10
   2
5 ¬ 
40

15  a
 n
6  
10
 24
 0
25 ¬ 225
28
9 
35
24
 0 24
 0
225 ¬ 225  4

5 or 0.80
24
0 24
0
225  225 , segments with these measures
Since   tanM  
opposite leg
adjace
nt leg
form a right triangle. However, the three numbers
are not whole numbers. Therefore, they do not  m

a
form a Pythagorean triple. 21

28

Page 767 Lesson 7-3  3



4 or 0.75
opposite leg
1. Because the quadrilateral is a square, the two sinA   
hypotenuse
triangles are isosceles right triangles with angle a
 n
measures 45, 45, and 90. Therefore, x  45. The length
, is 2
 times the length of a 28
of the hypotenuse, 132 
35
leg. So the leg length is 13 units and y  13.
 4

5 or 0.80
2. In the 30°-60°-90° triangle, the shorter leg is x
adjacent leg
units long, the longer leg is y units long, and the cosA   
hypotenuse
hypotenuse is 25 units long.
 m

n
x  1
2 (25) or 12.5 21

35
y  3 (12.5) or 12.53 
3. The triangle on the right is a 30°-60°-90° triangle  3

5 or 0.60
with a hypotenuse of length 30 and with a shorter opposite leg
tanA  
adjace
nt leg
leg of length x. a
 m
x 1
2 (30) or 15
28
The triangle on the left is an isosceles right triangle. 
21
y  2 x  4

3 or about 1.33
 2 (15) or 152 
opposite leg
4. The two triangles are congruent, by SSS, so they 2. sinM   
hypotenuse
are both 30°-60°-90° triangles. The shorter legs  m

n
are 83  units long, the longer legs are y units 
2
long, and the hypotenuses are x units long. 

5
x  2(83 ) 10

5 or about 0.63
 163 
adjacent leg
y  3 (83 ) cosM   
hypotenuse
 24 a
 n
5. The two triangles are congruent isosceles right 
3
triangles with legs of length x and hypotenuses of 
5
 5

length y. 15
 or about 0.77
x  1

2 (1002) opposite leg
 502  tanM   adjacent leg
hypotenuse  2 x m
 a
y  2 (502
) 
2
 100 
3

6. The two triangles are congruent 30°-60°-90°  36 or about 0.82
triangles. The shorter legs are y units long, the opposite leg
longer legs are x units long, and the hypotenuses sinA   
hypotenuse
a
are 123  units long.  n
y  1
2
(123 ) 
 
3
5 
 63 
x  3 (63 )  5
15
  or about 0.77
 18 adjacent leg
cosA   
hypotenuse
m
Page 767 Lesson 7-4  n
opposite leg  
2
1. sinM    
hypotenuse 5 
 m

n  5
10
  or about 0.63

Extra Practice 526


opposite leg opposite leg
tanA  
adjace
nt leg
sinA   
hypotenuse
a a
 m  n

3 5

  3
2  2
30
 
6 10

 
2 or about 1.22 4 or about 0.79
opposite leg adjacent leg
3. sinM   
hypotenuse cosA   
hypotenuse
m
 n  m

n

 
2
 2 35
1 
2
30

 
2 
6
2 or about 0.71 4 or about 0.61
adjacent leg
cosM   
hypotenuse tanA  
opposite leg

adjacent leg
a
 n a
 m

 
2

1
2
5
 
3

35

2
  or about 0.71  
15
2  3 or about 1.29
opposite leg
tanM   
adjacent leg 5. Calculator keystrokes: 2nd [COS1] 0.6293
 m

a
ENTER

 
2 mA  51.00150333
 
2
The measure of A is about 51.0.
 
2
2

 1.00 6. Calculator keystrokes: 2nd [SIN1] 0.5664


opposite leg ENTER
sinA   
hypotenuse mB  34.49956639
a
 n The measure of B is about 34.5.

 
2 7. Calculator keystrokes: 2nd [TAN1] 0.2665

2
1 ENTER

2 mC  14.92250149
2 or about 0.71 The measure of C is about 14.9.
adjacent leg
cosA   
hypotenuse 8. Calculator keystrokes: 2nd [SIN1] 0.9352
 m
 ENTER
n mD  69.26048039

 
2

2
1 The measure of D is about 69.3.

2 9. Calculator keystrokes: 2nd [TAN1] 0.0808
2 or about 0.71 ENTER
opposite leg mM  4.619463489
tanA   
adjacent leg
a The measure of M is about 4.6.
 m
10. Calculator keystrokes: 2nd [COS1] 0.1097

 
2
 
2 ENTER

2
2
mR  83.70197782
 1.00 The measure of R is about 83.7.
opposite leg
4. sinM   
hypotenuse
70
11. cos25°   
x
 m
n
 x 70
cos 25°
3 
5
  Use a calculator for find x.
2 30
Keystrokes: 70 COS 25 ENTER
 
6
 4 or about 0.61 x  77.23645433
adjacent leg x is about 77.2.
cosM   
hypotenuse 32
a 12. tanx°  40  0.8

 n
x  tan1 0.8
5 
3
  Use a calculator to find x.
2 30
10
Keystrokes: 2nd [TAN1] 0.8 ENTER
 
4 or about 0.79 x  38.65980825
opposite leg x is about 38.7.
tanM  adjace nt leg
13. sin55°  8x
 m 
a 8 sin55°  x
3 
5
  Use a calculator to find x.
53 
15
Keystrokes: 8 SIN 55 ENTER
 5 or about 0.77
  x  6.553216354
x is about 6.6.

527 Extra Practice


Page 768 Lesson 7-5 4. Use the Law of Sines to write a proportion.
1. Let x represent mRMN.  A
sin sin
 N
RN a ¬ n
tanx°  M
N sin 3 3° sin
N
  
12
0 57.8 ¬ 43.2
tanx°  
450 43.2 si
n 33°
 ¬ sinN
57.8
x  tan1
450 
12
0
sin1
43.2 si
57.8
n 33°
¬ N
Use a calculator.
x  14.9 Use a calculator.
The measure of the angle of elevation is about N  24°
14.9. 5. We know the measures of two angles of the
2. Let x represent LC. ROC and LCO are triangle. Use the Angle Sum Theorem to find
corresponding congruent angles, so mA.
mLCO  26. mA  mK  mX¬ 180
OL mA  33  62¬ 180
tan26°  LC mA  95¬ 180
75
tan26°   x mA¬ 85
75
x tan26°
Since we know mA and a, use proportions
A
sin
Use a calculator. involving  a .
x  153.8 To find x: To find k:
The distance from L to C is about 153.8 ft. sin
 A sin
 X sin
 A sin
 K
a ¬ x a ¬ k
3. mKIS  mASI  13. Let x represent SI. sin 85° sin 62° sin 8 5° sin 33°
       
KI 28.5 ¬ x 28.5 ¬ k
cos13°  
SI
2000 xsin85°¬ 28.5sin62° ksin85°¬ 28.5sin33°
cos13°   x 28.5 si
n 62° 28.5 si
n 33°
200 0 x¬  sin 85° k¬  sin 85°
x 
cos 13° x¬ 25.3 k¬¬15.6
Use a calculator. Therefore, mA  85, x  25.3, and k  15.6.
x  2052.6
6. Since we know mX and x, use a proportion
The distance from S to I is about 2052.6 ft. X
sin
involving  x to find mK.
sinX sin
 K
Page 768 Lesson 7-6 x ¬ k
1. Use the Law of Sines to write a proportion. sin55° sin
K
¬ 
3.7 3.6
 A
sin sin N
a ¬ n 3.6sin55°
¬ sinK
sin 47° sin 32° 3.7

a¬ 

sin1 ¬ K
15 3.6sin55°
3.7
15sin47°¬ a sin32°
53°¬ K
15 sin
 47°
sin 32° ¬ a We know the measures of two angles of the
Use a calculator. triangle. Use the Angle Sum Theorem to find mA.
a  20.7 mA  mK  mX  180
mA  53  55  180
2. Use the Law of Sines to write a proportion.
mA  108  180
sin
 A sin
 N
a ¬ n mA  72
sin 7
 5° sin 26°
  Now use a proportion to find a.
10.5 ¬ n sinX sin A
 x ¬ a
nsin75°¬ 10.5sin26°
10.5 si
n 26° sin55° sin72°
n¬  ¬ 
sin 75° 3.7 a
Use a calculator. asin55°¬ 3.7sin72°
n  4.8 a¬ 
3.7sin72°

sin55°
3. Use the Law of Sines to write a proportion.
A
sin N
sin a¬ 4.3
 
a ¬ n Therefore, mK  53, mA  72, and a  4.3.
sin 65° sin
 N
20.5 ¬ 18.6
18.6 sin 65°
20.5 ¬ sinN
sin 
1 18.6 si
20.5
¬ N
n 65°

Use a calculator.
N  55°

Extra Practice 528


7. We know the measures of two angles of the 3. d2¬ c2  e2  2cecosD
triangle. Use the Angle Sum Theorem to find 3.52¬ 1.22  42  2(1.2)(4)cos D
mX. 12.25¬ 1.44  16  9.6cosD
mA  mK  mX¬ 180 5.19¬ 9.6cosD
65  35  mX¬ 180 5.1
  9
9.6 ¬ cosD
100  mX¬ 180
mX¬ 80 D¬ cos1 9.6 
5.19

Since we know mX and x, use proportions D¬ 57.3°


involving  X
sin 4. e2¬ c2  d2  2cdcosE
x . 81.32¬ 42.52  502  2(42.5)(50)cos E
To find a: To find k:
6609.69¬ 1806.25  2500  4250cos E
sin
 X sin
 A sin
 X sin
 K
x ¬ a x ¬ k 2303.44¬ 4250cos E
sin 80° sin 65° sin 80° sin 35° 2303
 .44
 
50 ¬

a
  
50 ¬

k
 4250 ¬ cosE
E¬ cos1 4250 
2303 .44
asin80°¬ 50sin65° ksin80°¬ 50sin35°
E¬ 122.8°
50 sin
65° 50 sin
35°
a¬  sin 80° k¬ sin 80° 5. We know the measures of two sides and the
a¬ 46.0 k¬ 29.1 included angle, so we use the Law of Cosines.
Therefore, mX  80, a  46.0, and k  29.1. c2¬ a2  b2  2abcosC
8. We know the measures of two angles of the c2¬ 352  252  2(35)(25)cos55°
triangle. Use the Angle Sum Theorem to find c¬ 352  252 2(35) (25)co
s 55°
mK. c¬ 29.1
mA  mK  mX¬ 180 Now find mA using the Law of Sines.
122  mK  15¬ 180 sin
 C sin A
c ¬ a
mK  137¬ 180 sin 55° sin
A
  
mK¬ 43 29.1 ¬ 35
Since we know mA and a, use proportions 35 sin55°
A
sin 29.1 ¬ sinA
involving  a .
sin1 29.1 ¬ A
35 sin55°
To find k: To find x:
A
sin sin K A
sin sin X 80°¬ A
 a ¬ k  a ¬ x Find mB the same way.
sin 1
 22° sin 43°
 sin 1
22° sin 15° C
sin sin B
33.2 ¬ k 
33.2 ¬
 x  c ¬ b
ksin122°¬ 33.2sin43° x sin122°¬ 33.2sin15° sin 5
5° sin
 B
29.1 ¬ 25
33.2 si
n 43° 33.2 si
n 15°
k¬ sin 122° x¬ sin 122° 25 sin55°
29.1 ¬ sinB
k¬ 26.7 x¬ 10.1
Therefore, mK  43, k  26.7, and x  10.1. sin1 29.1 ¬ B
25 sin55°

45°¬ B
Page 768 Lesson 7-7 Therefore, c  29.1, mA  80, and mB  45.
1. e2¬ c2  d2  2cdcosE 6. We know an angle and the opposite side, so we
1502¬ 1002  1252  2(100)(125)cosE use the Law of Sines.
22,500¬ 10,000  15,625  25,000cos E  N
sin sin
 O
n ¬ o
3125¬ 25,000cos E sin8 0° sin
 O
315 3.5 ¬ 1.7

25,
000 ¬ cosE 1.7si n 80°
 ¬ sinO
3.5
E¬ cos1 00 
3125
25,0 sin 
1 1.7s i
3.5
¬ O
n 80°
E¬ 82.8°
29°¬ O
2. c2¬ d2  e2  2decosC
Use the Angle Sum Theorem to find mP.
52¬ 62  92  2(6)(9)cos C
mN  mO  mP  180
25¬ 36  81  108cos C
80  29  mP ¬180
92¬ 108cos C
92 109  mP ¬180

1
08 ¬ cosC mP ¬71
C¬ cos1 8
92
10
Find p using the Law of Sines.
 N
sin sin
 P
C¬ 31.6° n ¬ p
sin 8
 0° sin 71°
 
3.5 ¬ p
psin80°¬ 3.5sin71°
3.5 si
n 71°
p¬  sin 80°
p¬ 3.4
Therefore, mO  29, mP  71, and p  3.4.

529 Extra Practice


7. We know all three sides and no angles, so we use 5. Apply the Interior Angle Sum Theorem, using
the Law of Cosines. n  5a.
x2¬ b2  y2  2by cosX S  180(n  2)
752¬ 602  302  2(60)(30)cos X  18 (5a  2)
5625¬ 3600  900  3600cos X The sum of the measures of the interior angles is
1125¬ 3600cos X 180(5a  2).
112
5
3600 ¬ cosX 6. Apply the Interior Angle Sum Theorem, using
X¬ cos1 n  b.
4500 
112
5
S  180(n  2)
X¬ 108°  180(b  2)
Using the Law of Cosines again, The sum of the measures of the interior angles is
y2¬ b2  x2  2bx cosY 180(b  2).
302¬ 602  752  2(60)(75)cosY
7. Use the Interior Angle Sum Theorem to write an
900¬ 3600  5625  9000cos Y
equation to solve for n, the number of sides.
8325¬ 9000cos Y
8325 S¬ 180(n  2)

9000 ¬ cosY 156n¬ 180(n  2)
Y¬ cos1
9000 
832
5 156n¬ 180n  360
0¬ 24n  360
Y¬ 22°
360¬ 24n
Find the remaining angle using the Angle Sum
15¬ n
Theorem.
The polygon has 15 sides.
mB  mX  mY¬ 180
mB  108  22¬ 180 8. Use the Interior Angle Sum Theorem to write an
mB  130¬ 180 equation to solve for n, the number of sides.
mB¬ 50 S¬ 180(n  2)
Therefore, mB  50, mX  108, and 168n¬ 180(n  2)
mY  22. 168n¬ 180n  360
0¬ 12n  360
Page 769 Lesson 8-1 360¬ 12n
30¬ n
1. Apply the Interior Angle Sum Theorem, using
The polygon has 30 sides.
n  25.
S  180(n  2) 9. Use the Interior Angle Sum Theorem to write an
 180(25  2) equation to solve for n, the number of sides.
 4140 S¬ 180(n  2)
The sum of the measures of the interior angles is 162n¬ 180(n  2)
4140. 162n¬ 180n  360
0¬ 18n  360
2. Apply the Interior Angle Sum Theorem, using
360¬ 18n
n  30.
20¬ n
S  180(n  2)
The polygon has 20 sides.
 180(30  2)
 5040 10. Since n  15, the sum of the measures of the
The sum of the measures of the interior angles is interior angles is S  180(15  2) or 2340.
5040. Therefore, the measure of an interior angle is
234
 0
3. Apply the Interior Angle Sum Theorem, using 15 or 156.
n  22. For n  15, the measure of an exterior angle is
S  180(n  2) 36
 0
15 or 24.
 180(22  2)
11. Since n  13, the sum of the measures of the
 3600
interior angles is S  180(13  2) or 1980.
The sum of the measures of the interior angles is
Therefore, the measure of an interior angle is
3600. 198 0
 
4. Apply the Interior Angle Sum Theorem, using 13 or about 152.3.
n  17. For n  13, the measure of an exterior angle is
36
 0
S  180(n  2) 13 or about 27.7.
 180(17  2) 12. Since n  42, the sum of the measures of the 42
 2700 interior angles is S  180(42  2) or 7200.
The sum of the measures of the interior angles is Therefore, the measure of an interior angle is
2700. 720
 0
42 or about 171.4.
For n  42, the measure of an exterior angle is
36
 0
42 or about 8.6.

Extra Practice 530


Page 769 Lesson 8-2 15. Since 
BD bisects 
AC, AE  EC.
1. SRU  UTS, because SRU and UTS are AE  EC
opposite angles, and opposite angles of a 6c  1  35
parallelogram are congruent. 6c  36
2. UTS is supplementary to TSR and also to c6
TUR. UTS and TSR are consecutive angles, 16. Since 
AB  
DC, AB  DC.
and so are UTS and TUR. Consecutive angles AB  DC
of a parallelogram are supplementary. 3d  7  40
U
3. R 
S T, because R
U and ST are opposite sides of 3d  33
RSTU, and opposite sides of a parallelogram d  11
are parallel (by definition).
U
4. R S T, because R
U and S T
 are opposite sides Page 769 Lesson 8-3
of RSTU, and opposite sides of a parallelogram 1. No; only one pair of sides is shown to be parallel.
are congruent. 2. Yes; if the diagonals of a quadrilateral bisect each
5. RST  TUR, by SSS: R UT S
, R
S
T U
, and other, then the quadrilateral is a parallelogram.
T
R T R
. 3. Yes; if both pairs of opposite sides in a
V
6. S V U, because the diagonals of a parallelogram quadrilateral are parallel, then the quadrilateral
bisect each other. is a parallelogram.
7. BAE and DCA are congruent, because they 4. For the top and bottom sides to be parallel,
are alternate interior angles. Since mDCA  28, alternate interior angles must be congruent.
mBAE  28. 6x  54
8. BCE and DAC are congruent, because they x9
are alternate interior angles. Since mDAC  28, By the same reasoning applied to the left and
mBCE  28. right sides,
3y  3  36
9. BEC forms a linear pair with BEA.
3y  39
mBEC  mBEA  180
y  13
mBEC  91  180
So, x  9 and y  13.
mBEC  89
5. Since opposite sides must be congruent, measures
10. CED and BEA are vertical angles, and
of opposite sides must be equal.
mBEA  91. So, mCED  91.
x  5y¬ 2x  y
11. mAEB  91, and mBAE  28 (Exercise 7). 3x  3y¬ 15
mABE  mAEB  mBAE  180 Solve the system of equations by substitution.
mABE  91  28  180 Begin by solving the first equation for x.
mABE  119  180 5y  y¬ x  2x
mABE  61 4y¬ x
12. First find mABC. Because opposite angles are Now substitute in the second equation.
congruent, mABC  mADC. By the Angle Sum 3(4y)  3y¬ 15
Theorem, 12y  3y¬ 15
mADC  mDAC  mDCA¬ 180 15y¬ 15
mADC  28  28¬ 180 y¬ 1
mADC  56¬ 180 So, y  1 and x  4(1) or 4.
mADC¬ 124 6. Since the diagonals of a parallelogram bisect each
So, mABC  124. By the Angle Addition other,
Postulate, and using the information from 3y  4  2y  10
Exercise 11, 3y  2y  4  10
mABE  mEBC  mABC y6
61  mEBC  124 and
mEBC  63 5x  2¬ 3x  4
13. Since BCA D, BC¬ AD. 5x  3x¬ 2  4
BC¬ AD 2x¬ 6
5a  9.4¬ 39.4 x¬ 3
5a¬ 30 So, x  3 and y  6.
a¬ 6
14. Since 
AC bisects 
BD, BE  ED.
BE¬ ED
4b  2.8¬ 18.8
4b¬ 16
b¬ 4

531 Extra Practice


7. y T
Find the midpoint of R.
L (3, 2) 4 x x y y
0  (2) 6  (3)
M (5, 2) 
1
2, 2  2, 2
2 1 2

2  (1, 1.5)
x Since the diagonals bisect each other, QRST is a
4 2 2 4
parallelogram.
2
10. I (10, 9)
4 y

H (13, 5) 6
J (2, 4)
O (5, 6) N (3, 6)
10 2 2
If the opposite sides of a quadrilateral are x
parallel, then it is a parallelogram. G (5, 0) 2
2 
2
M
slope of L 
5  (3) or 0 6
6  (6)
N
slope of O 3  (5) or 0
6  2 8
O
slope of L 
5  (3)  2 or 4
 
First use the Distance Formula to determine
2
6  8 I and G J are congruent.
N
slope of M
3  5  2 or 4
 whether opposite sides H
Since opposite sides have the same slope, HI   [10  (13)] 2  (9  5)2

M
L
O N and LO
MN.   32  42 or 5
Therefore, LMNO is a parallelogram. GJ   [2  (5)]2  (4  0)2
8. W (5, 6)   32  42 or 5
y Since HI  GJ, H I  G
J.
X (2, 5) Next, use the Slope Formula to determine
4 whether HI
G
J.
95 4
2 I  
slope of H 10 (13) or 3
40 4
J
slope of G 2 (5) or 3
6 4 2 x
I and G
H J have the same slope, so they are
Z (8, 2) 2 parallel. Since one pair of opposite sides is
congruent and parallel, GHIJ is a parallelogram.
4
Y (3, 4)
Page 770 Lesson 8-4
Use the Distance Formula to determine whether 1. Because a rectangle is a parallelogram, diagonals
X
opposite sides are congruent. Start with W and T
Q and SR
 bisect each other.
Y
Z, which appear to be different lengths. QU  UT
WX   [2  (5)]2 
 (5 6)2 2x  3  4x  9
 7  2 or 50
2  (1)  2x  4x  3  9
ZY   [3  (8)]2  [ 2
4  (2)] 2x  12
x6
  5  (
2 2) or 29
2 
QU  2x  3
X
Since W  and X Y are not congruent, WXYZ is not
 2(6)  3
a parallelogram.
 15
9. R (0, 6) The diagonals of a rectangle are congruent.
y R
S 
¬ QT
SR¬ QT
Q (5, 4) 4
1SR¬ 1QT
2 2
2 SU¬ QU
SU¬ 15
4 2 2 4 x 2. The diagonals of a rectangle are congruent and
S (3, 1) bisect each other.
T
Q¬ RS
T (2, 3) 4 QT¬ RS
If the diagonals bisect each other, then the figure 1QT¬ 1RS
2 2
is a parallelogram. So the two diagonals must UT¬ RU
have the same point as their midpoint. x  9¬ 3x  6
Find the midpoint of QS. x  3x¬ 9  6
x x y y
5  3 4  (1)

1
2, 2  2, 2
2 1 2 2x¬ 15
x¬ 7.5
 (1, 1.5)

Extra Practice 532


So, 8. 
LM
O N
, so MLN ¬LNO.
RU  3x  6 mMLN ¬mLNO
 3(7.5)  6 m2 ¬m5
 16.5 m2 ¬38
and Use the following results as needed in
RS  2RU Exercises 9–18.
 2(16.5)
Because L O
M N, L
N
M O, and O
NO N
,
 33
LON  MNO. Then MON  LNO,
3. A rectangle is a parallelogram, and opposite sides so m8  m5  38.
of a parallelogram are congruent.
Use the Angle Sum Theorem.
S
Q ¬ RT
m8  m5  m9 ¬180
QS¬ RT
38  38  m9 ¬180
3x  40¬ 16  3x
76  m9 ¬180
3x  3x¬ 40  16
m9 ¬104
6x¬ 24
Thus, m8  30 and m9  104.
x¬ 4
So QS  3x  40  3(4)  40 or 28. 9. 3 and 9 form a linear pair. Therefore,
m3  m9 ¬180.
4. STR is a right angle, so mSTR  90. Use the
m3  104 ¬180
Angle Addition Theorem.
m3 ¬76
mSTQ  mRTQ¬ mSTR
5x  3  3  x¬ 90 10. 4 and 9 are vertical angles and are congruent.
4x  6¬ 90 Since m9 ¬104, m4  104.
4x¬ 84 11. By the Angle Addition Theorem,
x¬ 21 mONL  mLNM ¬mONM.
5. R
Q 
S T, so SRQ¬ RST. m5  m6 ¬mONM
mSRQ¬ mRST Use the fact that m5  38 and that ONM is a
x2  6¬ 36  x right angle.
x  x  30¬ 0
2 38  m6 ¬90
(x  6)(x  5)¬ 0 m6 ¬52
x  6  0 or x  5  0 12. By the Angle Addition Theorem, mLOM 
x  6 x5 mMON ¬mLON.
RTS is a right angle. So, RTS is a right m7  m8 ¬mLON
triangle, and SRT and RST are complementary. Use the fact that m8 ¬38 and that LON is a
mSRT  mRST  90 right angle.
mSRT  36  x  90 m7  38 ¬90
mSRT  90  36  x m7 ¬52
mSRT  54  x 13. m8 ¬38
Since x  6 or x  5,
14. m9 ¬104
mSRT  54  (6)  48,
or 15. 10 and 9 form a linear pair. Therefore,
mSRT  54  5  59. m10  m9  180.
m10  104 ¬180
6. QRT is a right angle. So, QRT is a right
m10 ¬76
triangle, and QTR and TQR are complementary.
M
16. L 
O N
, so LMO ¬MON.
mQTR  mTQR ¬90
mLMO ¬mMON
x  32  x2  16 ¬90
m11 ¬m8
x2  x  48 ¬90
Since m8 ¬38, m11  38.
x2  x  42 ¬0
(x  7)(x  6) ¬0 17. MNO is a right angle. So, MNO is a right
triangle, and MON and OMN are
x  7 ¬0 or x  6 ¬0
complementary.
x ¬7 x ¬6
mMON  mOMN ¬90
S
Q 
R T
, so TQS ¬QTR.
m8  m12 ¬90
mTQS ¬mQTR
38  m12 ¬90
mTQS ¬x  32
m12 ¬52
mTQS  7  32  25 or
18. OLM is an angle of the rectangle, so
mTQS  6  32  38
mOLM  90.
7. LON is a right angle. So, LON is a right
triangle, and OLN and ONL are complementary.
mOLN  mONL ¬90
m1  38 ¬90
m1 ¬52

533 Extra Practice


Page 770 Lesson 8-5 7. BY  10.5 (Exercise 5)
1. 
RQ
ST
, so QRS and TSR are supplementary 8. The diagonals bisect each other.
consecutive angles. AC  2(AY)
mQRS  mTSR  180 AC  2(6) or 12
Substitute mTSR  40 for mQRS.
mTSR  40  mTSR ¬180 Page 770 Lesson 8-6
2(mTSR) ¬220 1a. y
mTSR ¬110 D (2, 9) A (0, 9)
Because RST is bisected by S
Q,
mTSQ ¬1

2 (mTSR). 6
mTSQ ¬1

2 (110) or 55
2. From Exercise 1, mTSR  110. C (5, 4) B (3, 4)
mQRS  mTSR  40 2
mQRS  110  40 or 70
2 4x
3. By the Angle Sum Theorem,
mSRT  mSTR  mTSR  180. A quadrilateral is a trapezoid if exactly one pair
Because QRST is a rhombus, R SS T
 and so of opposite sides is parallel. Use the Slope
RST is an isosceles triangle, with Formula.
SRT  STR. 9 
9
mTSR  110 (Exercise 1) A
slope of D ¬
0  (2)
mSRT  mSRT  110 ¬180 ¬0

2 or 0
2(mSRT) ¬70 4 
4
mSRT ¬35 B
slope of C ¬
3  (5)
4. Because a rhombus is a parallelogram, and ¬0

8 or 0
because opposite sides of a parallelogram are 94
R
congruent, Q T S. D
slope of C ¬
2 
(5)
QR  TS ¬5

3
QR  15 4 
9
B
slope of A ¬
30
5. Because a rhombus is a parallelogram, and the
5 5
diagonals of a parallelogram bisect each other, ¬3 or 3
CY  AY and DY  BY.
Exactly one pair of opposite sides is parallel,
DY ¬BY
4
10r 
A
D  and CB . So, ABCD is a trapezoid.
3r  3 ¬ 2 1b. Use the Distance Formula to determine whether
3r  3 ¬5r  2 the legs are congruent.
3  2 ¬3r  5r
CD ¬ [2  (5)]2  (9 4)2
5 ¬2r
2.5 ¬r ¬ 3  5
2 2

Find BY and CY. ¬34


4
10r 
BY ¬ AB ¬ (3  0)2  (4 9)2
2
10(2.5) 4
¬ 32  (5)2
¬  or 10.5
2 ¬34
CY  AY Since the legs are congruent, ABCD is an
CY  6 isosceles trapezoid.
Find mACB.
2a. R (4, 6) S (10, 7)
BY y
tan(ACB) ¬
CY
10
.5
tan(ACB) ¬6
Q (1, 4)
ACB ¬tan1  10
6
.5

mACB ¬60
T (1, 1)
6. First find mCBD. Because the diagonals of a
x
rhombus are perpendicular, BYC is a right 2 4 6 8
triangle with right angle BYC. CBY and
BCY are complementary. BCY is the same
angle as ACB. A quadrilateral is a trapezoid if exactly one pair
mCBY  mBCY ¬90 of opposite sides is parallel. Use the Slope
mCBY  mACB ¬90 Formula.
mCBY  60 ¬90 6 
4
R
slope of Q ¬41
mCBY ¬30
B
D  bisects ABC, so mABD  mCBY. ¬2

3
mABD  30

Extra Practice 534


7 
1
S
slope of T ¬
10  1
4a. y

¬6
9 or 3
2 4
1  4
Q
slope of T ¬ 11 2
x
 3
¬ 0 or undefined 2 2 X (3, 1)
7 6 Y (7, 2)
S
slope of R  ¬
10  4 W (1, 2)
1
¬6
Exactly one pair of opposite sides is parallel, Z (1, 5)
R
Q  and TS. So, QRST is a trapezoid.
2b. Use the Distance Formula to determine whether A quadrilateral is a trapezoid if exactly one pair
the legs are congruent. of opposite sides is parallel. Use the Slope
QT ¬ (1  1
)2  (4 1)2 Formula.
1 
(2)
¬ 0  3
2 2 X
slope of W ¬ 31
¬ 32 or 3 ¬1

2
RS ¬ (10 4)2 (7  6
)2 2 
(5)
Y
slope of Z ¬ 71
¬6  1
2 2

¬37 ¬3
 1
6 or 2
Since the legs are not congruent, QRST is not an 2  (5)
W
slope of Z  ¬ 11
isosceles trapezoid.
3a. ¬3

0 or undefined
y
4 2 
(1)
Y
slope of X ¬ 73
L (1, 2) 1 1
¬ 4 or  4
O (3, 1)
X
Exactly one pair of opposite sides is parallel, W
4 2 2 x and ZY. So, WXYZ is a trapezoid.
M (4, 1) 4b. Use the Distance Formula to determine whether
2
the legs are congruent.
4 ZW ¬ (1  1)2  [
2  (
5)]2
N (3, 5) ¬ 0  3
2 2

¬ 9 or 3
A quadrilateral is a trapezoid if exactly one pair XY ¬ (7  3)2  [
2  (
1)]2
of opposite sides is parallel. Use the Slope
¬ 4  (
2 1) 2
Formula.
5  1 ¬ 17
N
slope of O ¬
3  (
3) Since the legs are not congruent, WXYZ is not an
6
¬6 or 1
isosceles trapezoid.
2
1  F
5. E  is the median.
M
slope of L ¬
41
3 EF  1
2 (AB  CD)

¬3 or 1
2 
1 13  1
2 (8  CD)

L
slope of O ¬
1  (3)
26  8  CD
¬1

4 18  CD
1 
(5)
M
slope of N ¬ 43 Q
6. P is the median.
¬4
 PQ  1
2 (LM  ON)

1 or 4
N
Exactly one pair of opposite sides is parallel, O PQ  1
2 (21  17)

and LM
. So, LMNO is a trapezoid. PQ  19
3b. Use the Distance Formula to determine whether M
L
O N, so M and N are consecutive
the legs are congruent. supplementary angles.
OL ¬ [1  (
3)]2  (2 
1)2 mM  mN ¬180
¬ 4  1
2 2
mM  96 ¬180
¬ 17 mM ¬84
NM ¬ (4  3) 2  [ 2
1  (5)] Similarly, L and O are supplementary.
¬ 12  42 mL  mO ¬180
¬ 17 36  mO ¬180
mO ¬144
Since the legs are congruent, LMNO is an
isosceles trapezoid.

535 Extra Practice


7. The length of the median is given by 4. Given: EFGH is a quadrilateral.
median  1 Prove: EFGH is a rhombus.
2 (QR  TS)

y
1
median  2(12  24) F(a 2, a 2) G(2a a 2, a 2)
median  18
S and T are congruent.
mS  mT
mS  52 E(0, 0) H(2a, 0) x
R and S are consecutive angles. Proof:
mR  mS ¬180 EF ¬(a2 
 0)2 
 (a
  0)2
2
mR  52 ¬180
¬2a 
2 2a 2
mR ¬128
¬4a2 or 2a
8. 
AB is the median of XYZW.
AB  1 FG ¬((2a a2)
 a
)2  
2 (a2
 
a2
 )2
2 (XY  WZ)

¬
(2a) 
2 02
AB  1
2 (13  21)

¬
4a2 or 2a
AB  17
GH ¬
((2a a2)
 2a)2
 (a 
2 0)2
 is the median of XYBA.
CD
¬2a
 2 2a2
CD  1
2 (XY  AB)

¬4a
2 or 2a
CD  1
2 (13  17)

EH ¬ (2a  0)2 (0  0
)2
CD  15 ¬(2a
 2) 02
¬4a or 2a
2
Page 771 Lesson 8-7
EF  FG  GH  EH
1. Unequal sides of an isosceles trapezoid are parallel.
F
EF G
G HE H

Assuming that  AD and B
C are both vertical, the x-
Since all four sides are congruent, EFGH is a
coordinate of D is a and the x-coordinate of C is a.
rhombus.
Leg DC will have the same length as A B
 if the y-
coordinate of C is b and the y-coordinate of D is
c. So, the coordinates of C are (a, b) and the Page 771 Lesson 9-1
coordinates of D are (a, c). 1. To find the coordinates of each point of the image,
for each point of the original triangle, multiply
2. On the assumption that all sides are vertical or
the y-coordinate by 1.
horizontal and that the rectangle is symmetric
about the y-axis: The x-coordinate of Q is the A(2, 2) → A (2, 2) B(3, 2) → B (3, 2)
opposite of the x-coordinate of R, namely 3b, N(3, 1) → N (3, 1)
and the y-coordinate of Q is the same as the Plot the reflected vertices and connect them to
y-coordinate of R, namely a. Points T and S lie on form the image A B N .
the x-axis, so their y-coordinates must be 0. The y
x-coordinate of T is the same as the x-coordinate
of Q, namely 3b. The x-coordinate of S is the B
A
same as the x-coordinate of R, namely 3b. The N
coordinates of S are (3b, 0), the coordinates of T
are (3b, 0), and the coordinates of Q are O x
(3b, a). N
3. Given: ABCD is a square. y A B
Prove: AC and BD are A(0, a)
congruent. B(a, a)

D(0, 0) C (a, 0) x
Proof:
AC ¬(a  0)2  (0
 a)2
¬a  a
2 2

¬2a2
BD ¬(a  0)2  (a
 0)2
¬a  a
2 2

¬2a2
AC  BD

ACBD

Extra Practice 536


2. To find the coordinates of each point of the image, B D y
interchange the x-coordinate and the
y-coordinate for each point of the original rectangle.
B(3, 3) → B (3, 3) A(3, 4) → A (4, 3)
H F
R(1, 4) → R (4, 1) N(1, 3) → N (3, 1) OF x
Plot the reflected vertices and connect them to H
form the image B A R N .
y
B B D B
A N
6. Use the vertical grid lines to find a corresponding
point for each vertex so that the line y  1 is
O equidistant from each vertex and its image.
x
R N Q(1, 3) → Q (1, 5) U(3, 1) → U (3, 3)
A(1, 0) → A (1, 2) D(3, 4) → D (3, 6)
Plot the reflected vertices and connect them to
y xR A
form the image Q U A D .
3. To find the coordinates of each point of the image, D y
for each point of the original trapezoid, multiply Q
both coordinates by 1.
Z(2, 3) → Z (2, 3) O(2, 4) → O (2, 4)
U
I(3, 3) → I (3, 3) D(3, 1) → D (3, 1) x
A O
Plot the reflected vertices and connect them to A y 1
form the image Z O I D . U
O y
I
Q
Z D
D
7. Use the horizontal grid lines to find a corresponding
O x point for each vertex so that the line x  2 is
D equidistant from each vertex and its image.
Z
I C(0, 4) → C (4, 4) A(1, 3) → A (5, 3)
O B(4, 0) → B (0, 0)
Plot the reflected vertices and connect them to
4. To find the coordinates of each point of the image,
form the image C A B .
for each point of the original triangle, multiply
the x-coordinate by 1. C C y
P(2, 1) → P (2, 1) Q(2, 2) → Q (2, 2)
R(3, 4) → R (3, 4)
Plot the reflected vertices and connect them to B
form the image P Q R . B
O x
y

A A
x 2
P P
O
x Page 771 Lesson 9-2
1.
Q Q

R R
5. To find the coordinates of each point of the image,
for each point of the original square, multiply
both coordinates by 1.
B(4, 4) → B (4, 4) D(1, 4) → D (1, 4)
F(1, 1) → F (1, 1) H(4, 1) → H (4, 1)
Plot the reflected vertices and connect them to c d
form the image B D F H . Reflect the blue figure in line c, then reflect the
resulting image in d to produce the red figure. The
red figure is a translation image of the blue figure.

537 Extra Practice


2. Because the red and blue figures have different 7. A y
orientations, the red figure is not a translation
image of the blue figure. B E
3. No; it is not one reflection after another with A
respect to the two parallel lines.
B E
4. y
D
C
L O x

M L C D
M
This translation moved every point of the
O x preimage 2 units left and 3 units up.
A(1, 3) → A (1  2, 3  3) or A (1, 6)
B(1, 1) → B (1  2, 1  3) or B (3, 4)
C(1, 2) → C (1  2, 2  3) or C (3, 1)
This translation moved each point of the D(3, 2) → D (3  2, 2  3) or D (1, 1)
preimage 2 units right and 1 unit up. E(3, 1) → E (3  2, 1  3) or E (1, 4)
L(2, 3) → L (2  2, 3  1) or L (4, 4) Plot the translated vertices and connect them to
M(4, 1) → M (4  2, 1  1) or M (2, 2) form pentagon A B C D E .
Plot the translated vertices and connect them to 8. R y
form segment L M .
5. y R
D
E O T x

F O x
E D S T

S
F This translation moved every point of the
preimage 3 units right and 2 units down.
This translation moved every point of the R(4, 3) → R (4  3, 3  2) or R (1, 1)
preimage 1 unit left and 3 units down.
S(2, 3) → S (2  3, 3  2) or S (1, 5)
D(1, 2) → D (1  1, 2  3) or D (0, 1)
T(2, 1) → T (2  3, 1  2) or T (5, 3)
E(2, 1) → E (2  1, 1  3) or E (3, 2)
Plot the translated vertices and connect them to
F(3, 1) → F (3  1, 1  3) or F (4, 4) form triangle R S T .
Plot the translated vertices and connect them to
form triangle D E F . Page 772 Lesson 9-3
6. y 1. • First graph KLM.
X • Draw a segment from point P(1, 1) to point
K(4, 2).
X W • Use a protractor to measure a 90° angle
W counterclockwise with PK
 as one side.
O x
• Draw PR.
Z
• Use a compass to copy PK onto 
PR. Name the
Y
segment PK .
Y Z
• Repeat with points L and M. K L M is the
image of KLM under a 90° counterclockwise
This translation moved every point of the rotation about the point P(1, 1).
preimage 1 unit right and 1 unit down.
R y
W(1, 1) → W (1  1, 1  1) or W (2, 0) L (1, 3)
X(2, 3) → X (2  1, 3  1) or X (1, 2) K (2, 2)
K (4, 2)
Y(3, 2) → Y (3  1, 2  1) or Y (2, 3)
M (2, 1)
Z(2, 2) → Z (2  1, 2  1) or Z (3, 3)
x
Plot the translated vertices and connect them to P (1, 1)
form quadrilateral W X Y Z . L (3, 1)
M (1, 0)

Extra Practice 538


2. • First graph FGH. 5. First reflect QUA in the x-axis. Then label the
• Draw a segment from point P(0, 0) to point image Q U A . Next, reflect the image in the
F(3, 3). line y  x. The coordinates of the image are
• Use a protractor to measure a 90° angle Q (4, 0), U (2, 3), and A (1, 1). The angle of
clockwise with PF
 as one side. rotation is 90° counterclockwise.
• Draw PR. y Q
• Use a compass to copy P  onto 
F PR. Name the
segment PF . U yx
• Repeat with points G and H. F G H is the
image of FGH under a 90° clockwise rotation Q A A
about the point P(0, 0). x
y A
R
F (3, 3) U
U
Q
H (1, 1)
x 6. First reflect AEO in the line y  x. Then label
G (4, 2) the image A E O . Next, reflect the image in the
y-axis. The coordinates of the image are A (3, 5),
F (3, 3)
E (1, 4), and O (2, 1). The angle of rotation is 90°
H (1, 1) G (2, 4) clockwise.
3. First reflect HIJ in the x-axis. Then label the A
image H I J . Next, reflect the image in the E E A
y-axis. H I J is the image of HIJ under A
reflections in the x-axis and the y-axis. The O
coordinates of the image are H (2, 2), I (2, 1),
E O O
and J (1, 2). The angle of rotation is 180°.
x
y y

J H J J
I y  x

I x I x
I
Page 772 Lesson 9-4
J H H H
1. No; Use the algebraic method to determine whether
a semi-regular tessellation can be created using
regular hexagons and squares of side length 1 unit.
4. First reflect NOP in the y-axis. Then label the Each interior angle of a hexagon measures
image N O P . Next, reflect the image in the line 180(6  2)
 or 120°, and each interior angle of a
y  x. N O P is the image of NOP under 6
reflections in the y-axis and the line y  x. The square measures 90°.
coordinates of the image are N (1, 3), Find whole number values for h and t so that
O (3, 5), and P (3, 2). The angle of rotation 120h  90t  360.
is 90° clockwise. Let h  1.
120(1)  90t ¬360
y yx 120  90t ¬360
90t ¬240
t ¬2.67
N N
Let h  2.
x 120(2)  90t ¬360
P 240  90t ¬360
N 90t ¬120
O P P O t ¬1.33
Let h  3.
O 120(3)  90t ¬360
360  90t ¬360
90t ¬0
t ¬0
There are no more reasonable possibilities. So, a
semi-regular tessellation cannot be created from
regular hexagons and squares.

539 Extra Practice


2. No; Use the algebraic method to determine whether 7. Always; for a regular polygon to tessellate the
a semi-regular tessellation can be created using plane, its interior angle measure must be a factor of
squares and regular pentagons of side length 1 unit. 360. If the measure of an interior angle of a regular
Each interior angle of a pentagon measures polygon is 120, three such polygons are required at
180(5  2) each vertex. If the measure of an interior angle is
 or 108°, and each interior angle of a
5
greater than 120, the only possible whole number of
square measures 90°.
polygons per vertex is two. These polygons would be
Find whole number values for h and t so that
required to have an interior angle measuring
108h  90t  360.
180°, which is impossible. Therefore, if the
Let h  1.
measure of one interior angle of a regular polygon
108(1)  90t ¬360
is greater than 120, it cannot tessellate the plane.
108  90t ¬360
90t ¬252
t ¬2.8 Page 772 Lesson 9-5
Let h  2. 1. O M  r(OM)
108(2)  90t ¬360 O M  2(1)
216  90t ¬360 O M  2(1)
90t ¬144 O M  2
t ¬1.6 2. O M  r(OM)
Let h  3. O M  1
3 (3)

108(3)  90t ¬360
O M  1

3 (3)
324  90t ¬360
90t ¬36 O M  1
t ¬0.4 3. O M  r(OM)
There are no more reasonable possibilities. So, a 3  3(OM)
4
semi-regular tessellation cannot be created from
squares and regular pentagons. 3  3(OM)
4
3. No; Use the algebraic method to determine whether 1  OM
4
a semi-regular tessellation can be created using
4. O M  r(OM)
regular hexagons and regular octagons of side
length 1 unit. O M  5 7
7 8  
Each interior angle of a hexagon measures
180(6  2)
O M  5
 7
7 8  
 or 120°, and each interior angle of an
6 O M  5

8
180(8  2)
octagon measures 8 or 135°. 5. O M ¬r(OM)
Find whole number values for h and t so that 4 ¬2
3 (OM)

120h  135t  360.
4 ¬2

3 (OM)
Let h  1.
120(1)  135t ¬360 6 ¬OM
120  135t ¬360 6. O M ¬r(OM)
135t ¬240 3 ¬1.5(OM)
t ¬1.78 4
Let h  2. 4.5 ¬1.5(OM)
120(2)  135t ¬360 3 ¬OM
240  135t ¬360
7.
135t ¬120 Image
Preimage Image
t ¬0.89
There are no more reasonable possibilities. So, a
(x, y) (3x, 3y) 13x, 13y
semi-regular tessellation cannot be created from
regular hexagons and regular octagons. T(1, 1) T (3, 3) T 1 
 1
3, 3 
4. Sometimes; some arrangements of isoceles right
triangles will have the same combination of angles
at each vertex, but some arrangements will not.
R(1, 2) 
R (3, 6) R 1
 2
3, 3 
5. Always; semi-regular tessellations have the same
combination of shapes and angles at each vertex
I(2, 0) I (6, 0) 
I 2

3, 0 
like uniform tessellations. The shapes for semi-
regular tessellations are regular.
6. Sometimes; as long as the polygon that is not
regular forms a pattern such that the sum of the
measures of the angles at the different vertices is
360, the polygon tessellates the plane.

Extra Practice 540


R y 10.
Preimage Image Image
(x, y) (3x, 3y) 13x, 13y
T B(1, 0) B (3, 0)  
B 1

3, 0
R
D 3, 0
T 2
x
D(2, 0) D (6, 0)
I I O

F 1, 3
2
F(3, 2) F (9, 6)

H 0, 3
2
H(0, 2) H (0, 6)
8.
Preimage Image Image
(x, y) (3x, 3y) 13x, 13y y

OB D B D
E(2, 1) E (6, 3) E 2 1
3, 3  H F
x

I(3, 3) I (9, 9) I (1, 1)

O(1, 2) O (3, 6) 


O 1 2
3, 3
  H
F

y E Page 773 Lesson 9-6


1. Find the magnitude.
O
E XY  ¬(x
 2x1)2 
(y2 y1)2
O I x ¬(2 1) 
2 (3  1
)2
¬13
¬3.6
Graph XY to determine how to find the direction.
O Draw a right triangle that has XY as its
hypotenuse and an acute angle at X.
y
Y(2, 3)
I

9. X(1, 1)
Preimage Image Image
(x, y) (3x, 3y) 13x, 13y x

A(0, 1) A (0, 3)  


A 0, 1

3

B(1, 1) B (3, 3) B 13, 3


 1
y y
tan X ¬
2
1
x2  x1
C(0, 2) C (0, 6) C 0, 2
3

31
¬
2 
1
D(1, 1) D (3, 3) D 1
3, 3
 1
¬2

3

y
mX ¬tan1 2  
3


C ¬33.7
A vector in standard position that is equal to XY
forms a 33.7° angle with the negative x-axis in
B D the second quadrant. So it forms a 180  33.7 or
146.3° angle with the positive x-axis.
C Thus, XY has a magnitude of about 3.6 units and
B D
x a direction of about 146.3°.
O A

541 Extra Practice


2. Find the magnitude. Thus, XY has a magnitude of about 7.6 units and
XY  ¬
(x2  
x1)2 
(y2 
y1)2 a direction of about 293.2°.
¬[2  (
1)]2 
 [2 (1)]2 4. Find the magnitude.
¬18
 XY  ¬(x
 2 x1)2 
(y2 y1)2
¬3 2 ¬(4  2) 
2 (4  1)2
¬4.2 ¬61
Graph XY to determine how to find the direction. ¬7.8
Draw a right triangle that has XY as its Graph XY to determine how to find the direction.
hypotenuse and an acute angle at X. Draw a right triangle that has XY as its
y hypotenuse and an acute angle at X.
y
Y(2, 2)

X(2, 1)
x
X(1, 1) x

y y Y(4, 4)
tan X ¬
2
1
x2  x1
y y
¬2  (
1) tan X ¬
2
1
2  (1) x2  x1

¬3

3 or 1 ¬ 4 
4 
2
1

mX ¬tan1(1)
¬5

6
¬45
A vector in standard position that is equal to XY
mX ¬tan1 5

6
¬39.8
forms a 45° angle with the positive x-axis in the
first quadrant. A vector in standard position that is equal to XY
forms a 39.8° angle with the negative x-axis in
Thus, XY has a magnitude of about 4.2 units and
the third quadrant. So it forms a 180  39.8 or
a direction of 45°.
219.8° angle with the positive x-axis.
3. Find the magnitude.
Thus, XY has a magnitude of about 7.8 units and
XY  ¬(x
 2 x1)2 
(y2 y1)2 a direction of about 219.8°.
¬[2  (5)]
2  (3  4)2 5. Find the magnitude.
¬58 XY  ¬(x

XY 2 x1)2 (y2 
y1)2
¬7.6 ¬[2  (
2)] 
2  [2 2
 (1)]
Graph XY to determine how to find the direction. ¬17
Draw a right triangle that has XY as its
¬4.1
hypotenuse and an acute angle at X.
Graph XY to determine how to find the direction.
X(5, 4) y
Draw a right triangle that has XY as its
hypotenuse and an acute angle at X.
y

Y(2, 3) X(2, 1) x

y y Y(2, 2)
tan X ¬
2
1
x2  x1
3  4
¬ y y
2  (5)
tan X ¬
2
1
x2  x1
¬7
3

 2 (1)
mX ¬tan1 7
3  ¬2  (2)
¬66.8 ¬1
4

A vector in standard position that is equal to XY mX ¬tan1 1
4

forms a 66.8° angle with the positive x-axis in the ¬14.0
fourth quadrant. So it forms a 360  66.8 or
293.2° angle with the positive x-axis.

Extra Practice 542


A vector in standard position that is equal to XY 8. First, graph RSTW. Next, translate each vertex by
forms a 14.0° angle with the positive x-axis in x , 3 units left and 4 units up. Connect the
the fourth quadrant. So it forms a 360  14.0 or vertices to form quadrilateral R S T W .
346.0° angle with the positive x-axis. y

Thus, XY has a magnitude of about 4.1 units and S


a direction of about 346.0°. R
6. Find the magnitude. W
XY  ¬(x
XY  2 x1)2 
(y2 y1)2 T
S R
¬(3  3) 
2 [1  (
1)]2
O x
¬40 W
¬2 10 T
¬6.3 9. First, graph AEIOU. Next, translate each vertex
Graph XY to determine how to find the direction. by b , 2 units left and 1 unit down. Connect the
Draw a right triangle that has XY as its vertices to form pentagon A E I O U .
hypotenuse and an acute angle at X. y
y A E
A
E
Y(3, 1) U O
I I x
x U
X(3, 1) O
O

10. c  d  2, 3  3, 4


y2  y1  2  3, 3  4
tan X ¬ 
x2  x1  5, 7
1  (
1)
¬3  3
Find the magnitude, using initial point (0, 0) and
endpoint (5, 7).
¬1
3 c  d  ¬ (5  0
)2  (7
 0)2
mX ¬tan1 1 3
   ¬ 74
¬8.6
¬18.4
Graph the resultant to determine how to find the
A vector in standard position that is equal to XY
direction. Draw a right triangle.
forms an 18.4° angle with the negative x-axis in
the second quadrant. So it forms a 180  18.4 or (5, 7)
y
161.6° angle with the positive x-axis.
Thus, XY has a magnitude of about 6.3 units and
a direction of about 161.6°.
7. First, graph HIJ. Next, translate each vertex by
a, 1 unit right and 3 units up . Connect the
vertices to form H I J . Y 
O x
y

H
I
7 
0
tan  ¬
50
J
H ¬7

5
I
J
 ¬tan1 75 
O x ¬54.5°
The vector forms a 54.5° angle with the positive
x-axis.
Thus, c  d has a magnitude of about 8.6 units
and a direction of about 54.5°.

543 Extra Practice


11. a  b  1, 3  4, 3 13. s  t ¬2, 5  6, 8
 1  (4), 3  3 ¬2  (6), 5  (8)
 3, 6 ¬4, 3
Find the magnitude, using initial point (0, 0) and Find the magnitude, using initial point (0, 0) and
endpoint (3, 6). endpoint (4, 3).
 a  b ¬(3 0)2 (6  0
)2 s  t  ¬(4 0)2 (3  0)2
¬45 ¬25
¬3 5 ¬5
¬6.7 Graph the resultant to determine how to find the
Graph the resultant to determine how to find the direction. Draw a right triangle.
direction. Draw a right triangle. y
y
(3, 6)

 O x


O x (4, 3)

3  0
tan  ¬ 
4  0
60
tan  ¬
3 
0 ¬3

4
¬2
 ¬tan1(2)
 ¬tan1 34 
¬36.9
¬63.4° The vector forms a 36.9° angle with the negative
The vector forms a 63.4° angle with the negative x-axis in the third quadrant. So it forms a 180 
x-axis in the second quadrant. So it forms a 36.9 or 216.9° angle with the positive x-axis.
180  63.4 or 116.6° angle with the positive Thus, s  t has a magnitude of 5 units and a
x-axis. Thus, a  b has a magnitude of about 6.7 direction of about 216.9°.
units and a direction of about 116.6°. 14. m  n ¬2, 3  2, 3
12. x  y ¬1, 2  4, 6 ¬2  (2), 3  3
¬1  4, 2  (6) ¬0, 0
¬5, 4 The magnitude of the zero vector 0, 0 is
Find the magnitude, using initial point (0, 0) and 02  02 or 0. The direction is undefined.
endpoint (5, 4).
15. u  v ¬7, 2  4, 1
x  y  ¬ (5  0
)2  (
4  0)2
¬7  4, 2  1
¬ 41
¬3, 3
¬6.4 Find the magnitude, using initial point (0, 0) and
Graph the resultant to determine how to find the endpoint (3, 3).
direction. Draw a right triangle.
u  v  ¬ (3  0)2 (3  0)2
y
¬ 18
¬3 2
¬4.2
x Graph the resultant to determine how to find the
O  direction. Draw a right triangle.
y
(3, 3)
(5, 4)
0
4  
tan  ¬
50
O x
¬4
5
 
 ¬tan1 45
¬38.7°
The vector forms a 38.7° angle with the positive
3 
0
x-axis in the fourth quadrant. So it forms a tan  ¬
3  0
360  38.7 or 321.3° angle with the positive x-axis. ¬1
Thus, x  y has a magnitude of about 6.4 units  ¬tan1 (1)
and a direction of about 321.3°. ¬45°

Extra Practice 544


1 1 2 2 1 1 5 0 2
01   
The vector forms a 45° angle with the negative

x-axis in the second quadrant. So it forms a 0 1 5 0 2 1 2 2 1
180  45 or 135° angle with the positive x-axis.
The coordinates of the vertices of the image are
Thus, u  v has a magnitude of about 4.2 units
C (1, 1), D (5, 2), E (0, 2), and F (2, 1).
and a direction of 135°.
8. Write the ordered pairs as a vertex matrix. Add 1
to each x-coordinate and subtract 4 from each
Page 773 Lesson 9-7
y-coordinate by adding the translation matrix to
1. Write the ordered pairs as a vertex matrix. Then the vertex matrix.
multiply the vertex matrix by the reflection
5 0 5
matrix for the x-axis.
2 1 2 4 2 1 2 4
04 0 3 2
 
1 1 1 1
4 4 4 4 
1 0
0 1
 2 1 1 

2 2  4
2 1 1
The coordinates of the vertices of the image are
 10 1 6
4 7 6
T (2, 2), R (1, 2), A (2, 1), and P (4, 1). The coordinates of the vertices of the image are
2. Write the ordered pairs as a vertex matrix. Then W (1, 0), X (4, 4), Y (1, 7), and Z (6, 6).
multiply the vertex matrix by the matrix for a 90° 9. Write the ordered pairs as a vertex matrix.
clockwise rotation about the origin (same as a Perform the dilation by multiplying the vertex
270° counterclockwise rotation).
matrix by a scale factor of 1

2.
2 1 2 4 2 2 1 1
0 1
1 0

2 2 1 1
 
2 1 2 4  1

6
2
2 2 8 4
4 6

3
 1 2 3

6 1 4 2 3
The coordinates of the vertices of the image are
The coordinates of the vertices of the image are
T (2, 2), R (2, 1), A (1, 2), and P (1, 4).
J (3, 1), K (1, 4), L (2, 2), and M (3, 3).
3. Write the ordered pairs as a vertex matrix.
10. Write the ordered pairs as a vertex matrix. Then
Subtract 4 from each x-coordinate and add 3 to
multiply the vertex matrix by the reflection
each y-coordinate by adding the translation
matrix for the line y  x.
matrix to the vertex matrix.
0 3 3 4 2 4 4
22 1 2 4
2 1 1

4 4 4 4

3 3 3 3  01 10 22 4
2
2 4 4
  22 0 3 3 2 
The coordinates of the vertices of the image are
 65 3 2 8
5 2 2  A (2, 2), B (4, 0), C (2, 3), D (4, 3), and
E (4, 2).
The coordinates of the vertices of the image are
T (6, 5), R (3, 5), A (2, 2), and P (8, 2).
Page 773 Lesson 10-1
4. Write the ordered pairs as a vertex matrix.
1. d  2r
Perform the dilation by multiplying the vertex
 2(18) or 36 in.
matrix by a scale factor of 4.
C  2r
2 1 2 4 8 4 8 16
4 2 2 1 1 

8 8 4 4   2(18)
 36 or about 113.10 in.
The coordinates of the vertices of the image are 2. r ¬1

2d
T (8, 8), R (4, 8), A (8, 4), and P (16, 4).
5. Write the ordered pairs as a vertex matrix. ¬1

2 (34.2) or 17.1 ft
Perform the dilation by multiplying the vertex C  d
matrix by a scale factor of 2.5.  (34.2)
 34.2 or about 107.44 ft
2 4 5 5 10
2.5 24 4 6
 
10 10 15  3. C ¬d
12 ¬d
The coordinates of the vertices of the image are 12 ¬d
D (5, 10), E (5, 10), and F (10, 15). d ¬12 m
6. Write the ordered pairs as a vertex matrix. Then r ¬1
2d
multiply the vertex matrix by the reflection
matrix for the x-axis. ¬1
2 (12) or 6 m
3 6 5 3 6 4. C ¬d
1 0
0 1

4 2 3 

4 2
5
3  84.8 ¬d
84
.8
The coordinates of the vertices of the image are
 ¬d
R (3, 4), S (6, 2), and T (5, 3). 26.99 ¬d
7. Write the ordered pairs as a vertex matrix. Then d ¬26.99 mi
multiply the vertex matrix by the matrix for a 90° r ¬1

2d
counterclockwise rotation about the origin. ¬1

2 (26.99)
¬13.50 mi

545 Extra Practice


5. r ¬1

2d Page 774 Lesson 10-2
1. GKI and IKJ are a linear pair, and the angles
¬1

2 (8.7) or 4.35 cm of a linear pair are supplementary.
C  d
mGKI  mIKJ ¬180
 (8.7)
mGKI  90 ¬180
 8.7 or about 27.33 cm
mGKI ¬90
6. d  2r
2. LKJ and HKG are vertical angles, and vertical
 2(3b)
angles are congruent.
 6b in.
mLKJ  mHKG
C  d mLKJ  23
 (6b)
3. From Exercise 1, mGKI  90. Use the Angle
 6b or about 18.85b in.
Addition Theorem.
7. The diameter of the circle is the same as the mGKH  mHKI ¬mGKI
hypotenuse of the right triangle. Use the 23  mHKI ¬90
Pythagorean Theorem. mHKI ¬67
a2  b2 ¬c2
HKI and LKI are a linear pair, so
62  82 ¬c2
mHKI  mLKI ¬180
100 ¬c2
67  mLKI ¬180
10 ¬c
mLKI ¬113
So the diameter of the circle is 10 inches.
C  d 4. LKG and LKJ are a linear pair, and the angles
C  (10) or 10 of a linear pair are supplementary.
The exact circumference is 10 inches. mLKG  mLKJ  180
From Exercise 2, mLKJ  23.
8. The diameter of the circle is the same as the
mLKG  23 ¬180
hypotenuse of the right isosceles triangle. Use the
mLKG ¬157
Pythagorean Theorem.
c2 ¬a2  b2 5. mHKI ¬67 (Exercise 3)
c2 ¬62  62 6. HKJ and LKG are vertical angles, and vertical
c2 ¬72 angles are congruent.
c ¬6 2 mHKJ  mLKG
So the diameter of the circle is 6 2 centimeters. From Exercise 4, mLKG  157.
C  d mHKJ  157
C  (6 2 ) or 62 7. QXR is a right angle.
The exact circumference is 6 2  centimeters.   mQXR
mQR

9. The diameter of the circle is the same as the mQR  90
8. QW, 

hypotenuse of the right isosceles triangle. Use the WV, and QV are minor arcs, and QXV is a
Pythagorean Theorem. right angle. By the Arc Addition Postulate,
c2 ¬a2  b2   mWV  ¬mQV 
mQW
c2 ¬122  122 
mQW  mWXV ¬mQXV
c2 ¬288   25 ¬90
mQW
c ¬12 2  ¬65
mQW
So the diameter of the circle is 12 2 yards.   
9. TU, VU, and TV are minor arcs, and TXV is a
C  d
right angle. By the Arc Addition Postulate,
C  (12 2 ) or 122   mVU ¬mTV 
mTU
The exact circumference is 12 2  yards. 
mTU  mVXU  mTXV
10. The diameter of the circle is the same as the   45 ¬90
mTU
hypotenuse of the right triangle. Use the  ¬45
mTU
Pythagorean Theorem. 
10. WRV is a major arc.
c2 ¬a2  b2   360  mWV 
mWRV
132 ¬212  c2 
mWRV  360  mWXV
610  c2   360  25 or 335
mWRV
610  c2 
11. SVW is a semicircle.
So the diameter of the circle is  610 meters.   mWV  ¬mSVW 
mSV
C  d 
mSV  mWXV ¬180
C  ( 610 ) or 610    25 ¬180
mSV
The exact circumference is  610 meters.  ¬155
mSV

Extra Practice 546


12. TXV is a right angle. 
TV, 
WV, and 
TW are 3 is an inscribed angle that intercepts 
AB.

minor arcs. m3 ¬1mAB
  mTW  ¬360 2
mTRW
  
mTRW  mTV  mWV ¬360 ¬1
2 (176) or 88
  90  25 ¬360 m1  m2  m3 ¬180
mTRW
  115 ¬360 21  m2  88 ¬180
mTRW
 ¬245 m2  109 ¬180
mTRW
m2 ¬71
Page 774 Lesson 10-3 So m1  21, m2  71, and m3  88.
.
1. 
SI bisects H, so HR  1
J 
2 HJ.
2. First determine mWX
 
1
HR  2HJ mZW  mWX ¬180
120  mWX  ¬180
HR  1   ¬60
2 (22) or 11 mWX
I bisects HJ, so RJ  1
 
2. S 2 HJ. m3  mWX
1  60
RJ  2HJ
Because 3 and 4 are vertical angles,
RJ  1
2 (22) or 11 m4  m3
M
3. S bisects L , so LT  1
G 
2 LG.  60
1
LT  2LG Because AZY is isosceles, 5  6, and so
LT  1
 m4  m5  m6 ¬180
2 (18) or 9
M
4. S bisects L , so TG  1
G  60  2(m5) ¬180
2 LG. 2(m5) ¬120
1
TG  2 LG
 
m5 ¬60
TG  1 
2 (18) or 9 ¬and
I bisects    2(mIJ).
5. S HJ, so mHJ m6 ¬60

mHJ  2(mIJ) 
Similarly, m1  60 and m2  60. So, m1 
  2(35) or 70 60, m2  60, m3  60, m4  60, m5  60,
mHJ
 bisects    2(mLM ). and m6  60.
6. M
S LG, so mLG

mLG  2(mLM)  3. 1 is an inscribed angle that intercepts 
TS.
  2(30) or 60 1 
mLG m1 ¬ 2 mTS

7. M
S  bisects LG, so mMG  mLM .
  ¬1
2 (110) or 55
mMG  mLM
  30 4 is an inscribed angle that intercepts 
TS.
mMG 
m4 ¬1mTS
I bisects    mIJ. 2
8. S HJ, so mHI
 
mHI  mIJ ¬1
2 (110) or 55
  35
mHI 3 is an inscribed angle that intercepts 
QR.
9. B
A  and E D
 are equidistant from R, so A
B
E
D
. 1 
m3 ¬ 2 mQR

AB  ED
AB  30 ¬1
2 (40) or 20
10. Because R FE D, R
F
 bisects E
D
. 6 is an inscribed angle that intercepts 
QR.

EF  1 ED m6 ¬1mQR
2
2
¬1

2 (40) or 20
EF  1

2 (30) or 15
m1  m2 m3 ¬180
11. Because R F
E D, R
F
 bisects E
D
.
55  m2  20 ¬180
D  1
F 2 
E 
D m2  75 ¬180
DF  1
 m2 ¬105
2 (30) or 15
12. Because RCA B, R
C
 bisects A
B
. m4  m5  m6 ¬180
55  m5  20 ¬180
BC  1
2 AB m5  75 ¬180
From Exercise 9, AB  30. m5 ¬105
BC  1

2 (30) or 15
So m1  55, m2 105, m3  20, m4  55,
m5  105, and m6  20.
Page 774 Lesson 10-4
1. 1 is an inscribed angle that intercepts 
BC.

m1 ¬1mBC
2
¬1

2 (42) or 21

547 Extra Practice


4. 1 is an inscribed angle that intercepts 
BC. m4  m5  m6 ¬180
 m4  56  28 ¬180
m1 ¬1mBC
2
m4  84 ¬180
¬1
2 (70) or 35 m4 ¬96
7 is an inscribed angle that intercepts 
BC. So, m1  96, m2  56, m3  28, m4  96
1  m5  56, and m6  28.
m7 ¬ 2 mBC

7.
¬1
2 (70) or 35
C
9 and 1 are alternate interior angles, and 7
and 3 are alternate interior angles. So m9 
35 and m3  35.
B
A, B, C, and D are right angles, so the
numbered angles inside them are complementary D
pairs. Thus,
m1  m11 ¬90
35  m11 ¬90
m11 ¬55. A
Similarly, m5  m6  m10  55.
ABCD is a rhombus so AB  BC  CD  AD.
m1  m2  m3 ¬180   mBC
Therefore, mAB   mCD   mAD
, and
35  m2  35 ¬180    
mAB  mBC  mCD  mAD  360.
m2  70 ¬180  ¬360
Substituting, 4mBC
m2 ¬110
mBC ¬90 and 
 CD  90
2 and 12 form a linear pair.   ¬180.
So mBC  mCD
m2  m12 ¬180 D
So B is a diameter of the circle.
110  m12 ¬180
8.
m12 ¬70
Because vertical angles are congruent, 8  2
and 4  12. So m8  110 and m4  70. R
So, m1 ¬35, m2  110,
m3 ¬35, m4  70, T S
m5 ¬55, m6  55,
m7 ¬35, m8  110,
m9 ¬35, m10  55,
T is an inscribed angle that intercepts 
RS.
m11 ¬55, m12  70.
1 
mT ¬ mRS

5. m1 ¬1   2
2 mTR 1
¬ 2 (170) or 85

¬1

2 (100) or 50
1 
m6  mTR
2 Page 775 Lesson 10-5
1 1. First determine whether PON is a right triangle
¬2(100) or 50
by using the Converse of the Pythagorean
Because SRQT, m3  90 and m4  90. Theorem.
1 and 2 are complementary, so (PO)2  (ON)2 ¬(PN)2
m1  m2 ¬90
50  m2 ¬90 142  42 ¬(253)2
212 ¬212
m2 ¬40
Because the Converse of the Pythagorean
Similarly, m5  40. So m1  50, m2  40,
Theorem is true, PON is a right triangle and
m3  90, m4  90, m5  40, and m6 = 50.
 PON is a right angle. Thus, PO
 ON
, making
6. m2  mXW O
P  a tangent to N.
m2  56
 2. First determine whether QRS is a right triangle
m5  mUV
by using the Converse of the Pythagorean
m5  56
 Theorem.
m3  1mUY
2 (QR)2  (RS)2 ¬(QS)2
m3  1
 62  182 ¬232
2 (56) or 28 360 ¬529
1 
m6  mXZ
2 Because the Converse of the Pythagorean
Theorem is not true in this case, QRS is not a
m6  1
 
2 (56) or 28 right triangle and QRS is not a right angle. So,
m1  m2  m3 ¬180 S
R  is not tangent to Q.
m1  56  28 ¬180
m1  84 ¬180
m1 ¬96

Extra Practice 548


3. Because the radius is perpendicular to the 3. 8 x ¬(x  9) 2
tangent at the point of tangency, VUU T. This 8x ¬2x  18
makes VUT a right angle and VUT a right 6x ¬18
triangle. Use the Pythagorean Theorem to find x. x ¬3
(VU)2  (UT)2 ¬(VT)2 4. 15 r ¬9 5
x2  42 ¬52 15r ¬45
x2  16 ¬25 r ¬3
x2 ¬9 5. The vertical diameter bisects the horizontal
x ¬3 chord. Each segment has length m. The diameter
Because x is the length of VU
, ignore the negative is divided into segments of length 3 and 11.
result. Thus, x  3.
4. The radius is perpendicular to the tangent at the 3
point of tangency, so the triangle is a right m
m 4
triangle. Use the Pythagorean Theorem to find x.
52  152 ¬x2 11
7
250 ¬x2
5 10 ¬x
Because x is the length of the hypotenuse, ignore m m ¬3 11
the negative result. Thus, x  5 10. m2 ¬33
B
5. A  and BC are drawn from the same exterior m ¬ 33 or about 5.7
point and are tangent to the circle, so AB
B C
. 6. The two segments that are tangent to the larger
AB ¬BC circle are congruent, and therefore each has a
2x  1 ¬3x  7 length of 6. In the smaller circle, we have a
1 ¬x  7 tangent and a secant.
8 ¬x 5 (5  x) ¬62
25  5x ¬36
Page 775 Lesson 10-6 5x ¬11
1. m5 ¬1 x ¬2.2
2 (70  80)


¬1
2 (150) or 75 Page 776 Lesson 10-8
2. Find the measure of an angle that forms a linear 1. (x  h)2  (y  k)2 ¬r2
pair with 6. If X is the angle that intercepts [x  1]2  [y  (2)]2 ¬22
the 40° arc, (x  1)2  (y  2)2 ¬4
mX  12 (40  35)
 2. The origin is the point (0, 0).
¬1
 (x  h)2  (y  k)2 ¬r2
2 (75) or 37.5 (x  0)2  (y  0)2 ¬42
m6  180  mX
x2  y2 ¬16
 180  37.5 or 142.5
3. (x  h)2  (y  k)2 ¬r2
3. 360  140 ¬220 2
[x  (3)]2  [y  (4)]2 ¬11

m7 ¬1

2 (220) or 110
(x  3)2  (y  4)2 ¬11
4. x  1
2 (80  40)

4. If d  6, r  3.
x  1

2 (40) or 20 (x  h)2  (y  k)2 ¬r2
[x  3]2  [y  (1)]2 ¬32
5. 15 ¬1
2 (80  2x)

(x  3)2  (y  1)2 ¬9
15 ¬40  x
5. (x  h)2  (y  k)2 ¬r2
25 ¬x
(x  6)2  (y  12)2 ¬72
25 ¬x
(x  6)2  (y  12)2 ¬49
6. x ¬1
2 (6x  40)
 6. If d  8, r  4.
x ¬3x  20 (x  h)2  (y  k)2 ¬r2
2x ¬20 (x  4)2  (y  0)2 ¬42
x ¬10 (x  4)2  y2 ¬16
7. If d  22, r  11.
Page 775 Lesson 10-7 (x  h)2  (y  k)2 ¬r2
1. x 8 ¬4 10 [x  6]2  [y  (6)]2 ¬112
8x ¬40 (x  6)2  (y  6)2 ¬121
x ¬5 8. If d  2, r  1.
2. 5 x ¬3 10 (x  h)2  (y  k)2 ¬r2
5x ¬30 [x  (5)]2  [y  1]2 ¬12
x ¬6 (x  5)2  (y  1)2 ¬1

549 Extra Practice


9. Write the equation in standard form. Page 776 Lesson 11-1
(x  0)2  (y  0)2  52 1. Base and Side: Each pair of opposite sides of a
The center is at (0, 0), and the radius is 5. Draw a parallelogram has the same measure. Each base
circle with radius 5, centered at the origin. is 15 inches long, and each side is 20 inches long.
y Perimeter: The perimeter of a polygon is the sum
of the measures of its sides. So, the perimeter of
the parallelogram is 2(15)  2(20) or 70 in.
Height: Use a 30°-60°-90° triangle to find the
height. Recall that if the measure of the leg
opposite the 30° angle is x, then the length of the
O x
hypotenuse is 2x, and the length of the leg
opposite the 60° angle is x 3.
20  2x
10  x
So, the height of the parallelogram is x 3 or
10. Write the x2  y2  4 equation in standard form. 10 3 in.
(x  0)2  (y  0)2  22 Area: A ¬bh
The center is at (0, 0), and the radius is 2. Draw a ¬15(10 3)
circle with radius 2, centered at the origin. ¬150 3 or about 259.8
y The area is about 259.8 in2 and the perimeter is
70 in.
2. Base and Side: Each pair of opposite sides of a
parallelogram has the same measure. Each base
O x is 28 ft long, and each side is 9 ft long.
Perimeter: The perimeter of the parallelogram is
2(28)  2(9)  74 ft.
Height: Use a 45°-45°-90° triangle to find
the height.
11. Write the equation in standard form. 9 ¬x 2
(x  3)2  [y  (1)]2  32 9  
2
 ¬x
The center is at (3, 1), and the radius is 3. 2
Draw a circle with radius 3, centered at (3, 1). So, the height of the parallelogram is x or
y 9
2
2 ft.
Area: A ¬bh

¬28 
9
2
2

O x ¬1262  or about 178.2
(3, 1) The area is about 178.2 ft2 and the perimeter is
74 ft.
3. Area: For a rectangle,
A  w
12. Write the equation in standard form.  18.3(6.2)
(x  1)2  (y  4)2  12  113.5
The center is at (1, 4), and the radius is 1. Perimeter: The perimeter is 2(18.3)  2(6.2)  49.
Draw a circle with radius 1, centered at (1, 4). The perimeter is 49 m and the area is about
y 113.5 m2.
4. To determine the shape, first graph each point
(1, 4)
and draw the quadrilateral. Then determine the
slope of each side.
y
R (1, 3)
x Q (3, 3)
O
T (3, 1)
S (1, 1)
x

Extra Practice 550


33
R
slope of Q ¬
1 
(3)
6. To determine the shape, first graph each point
and draw the quadrilateral. Then determine the
¬0

2 or 0 slope of each side.
31
Q
slope of T ¬
3 
(3) N (9, 7)
y
¬2
0 , which is undefined
O (6, 7)
11
S
slope of T  ¬1  (3)
0 L (5, 3)
¬2 or 0 M (8, 3)
31
R
slope of S  ¬1  (1)
¬2
0
, which is undefined x

Opposite sides have the same slope, so they are


parallel. QRST is a parallelogram. All sides are
3 
3
vertical or horizontal, so the parallelogram is a M
slope of L  ¬
85
rectangle. All sides have length 2 so they are
¬0

3 or 0
congruent. So the rectangle is a square.
7 
3
A  s2 O
slope of L ¬
65
 22
¬4

1 or 4
4
7 
7
5. To determine the shape, first graph each point N
slope of O ¬
96
and draw the quadrilateral. Then determine the
¬0

3 or 0
slope of each side.
7 
3
y
N
slope of M ¬
98
¬4

1 or 4
x Opposite sides have the same slope, so they are
parallel. LMNO is a parallelogram. The slopes of
D(7, 3) C(2, 3) the consecutive sides are not negative reciprocals
of each other, so the sides are not perpendicular.
Thus, the parallelogram is neither a square nor a
rectangle.
A(7, 6) B(2, 6) Find the area by first determining the base and
6  (6) height.
B
slope of A ¬
2 (7) Base: LM
 is parallel to the x-axis, so subtract the
¬0

5 or 0 x-coordinates of the endpoints to find the length:
3 (6) LM  8  5 or 3.
D
slope of A ¬
7  (7)
Height: L M
 and O N
 are horizontal segments, the
¬3

0 , which is undefined distance between them, or the height, can be
3 (3) measured on any vertical segment. Reading from
C
slope of D ¬
2  (7) the graph, the height is 4.
¬0

5 or 0 A  bh
3 (6)  3(4)
C
slope of B ¬
2  (2)  12
¬3

0 , which is undefined The area of LMNO is 12 units2.
Opposite sides have the same slope, so they are
parallel. ABCD is a parallelogram. All sides are
vertical or horizontal, so the parallelogram is a
rectangle. However, not all sides are congruent, so
the rectangle is not a square.
Find the area by first determining the length and
width.
Length: A B is parallel to the x-axis, so subtract
the x-coordinates of the endpoints to find the
length: AB  2  (7) or 5.
Width: A D
 is parallel to the y-axis, so subtract
the y-coordinates of the endpoints to find the
width: AD  3  (6) or 3.
A  w
 5(3)
 15
The area of ABCD is 15 units2.

551 Extra Practice


7. To determine the shape, first graph each point 3. The quadrilateral is a rectangle so A  w and
and draw the quadrilateral. Then determine the w  18.
slope of each side. Use the properties of 30°-60°-90° triangles to find
y the measure of .  is the measure of the longer
leg of the triangle whose shorter leg has measure
18. So   18 3.
X (1, 1) Y (2, 1) A  w
 (18 3)(18)
x  324 3 or about 561.2
The area of the rectangle is approximately
W (1, 2) Z (2, 2)
561.2 units2.
4. y
2 (2)
B (2, 3) C (4, 3)
Z
slope of W ¬
2  (1)
¬0

3 or 0 A (1, 1) D (7, 1)
1  (2)
X
slope of W ¬ 
1  (1)
x

¬3

0 , which is undefined
1 
1
Y
slope of X ¬
2  (1)
¬0

3 or 0 Base: Since AD and B
C
 are horizontal, find their
1  (2)
Y
slope of Z ¬ 
22
lengths by subtracting the x-coordinates of their
endpoints.
¬3

0 , which is undefined b1  AD  7  1
Opposite sides have the same slope, so they are  6 or 6
parallel. WXYZ is a parallelogram. All sides are b2  BC  4  2
horizontal or vertical, so the parallelogram is a  2 or 2
rectangle. All sides have length 3 so they are Height: Because the bases are horizontal
congruent. So the rectangle is a square. segments, the distance between them can be
A  s2 measured on a vertical line. Subtract the
 32 y-coordinates.
9 h  3  1 or 2
The area of WXYZ is 9 units2.
Area: A ¬1
2 h(b1  b2)


Page 776 Lesson 11-2 ¬1


2 (2)(6  2)

1. The area of the quadrilateral is the sum of the ¬8
areas of the upper triangular portion and the The area of trapezoid ABCD is 8 units2.
lower triangular portion. For each of those two
5. y
portions, the base measures 15  12  9 or 36
units.
A (2, 2) B (2, 2)
area ¬top area  bottom area
¬1 1
2 bh1  2 bh2

x
¬1 1
2 (36)(15)  2 (36)(9)

¬432
The area of the quadrilateral is 432 units2. D (4, 3) C (7, 3)
2. The altitude h of the parallelogram forms a
30°-60°-90° triangle in which the hypotenuse has Base: Since AB and DC
 are horizontal, find their
measure 18 and the altitude is the longer leg. Use lengths by subtracting the x-coordinates of their
this fact to find the height of the trapezoid. endpoints.
2x  18 b1  AB  2  (2)
x9  4 or 4
The height is x 3 or 93. b2  DC  7  (4)
Now find the area of the trapezoid.  11 or 11
A ¬1
2 h(b1  b2)
 Height: Because the bases are horizontal
segments, the distance between them can be
¬1
2 (93
 )(25  13) measured on a vertical line. Subtract the
¬1713  or about 296.2 y-coordinates.
The area of the trapezoid is approximately h  2  (3) or 5
296.2 units2.

Extra Practice 552


Area: A ¬1
2 h(b1  b2)
 8. y
¬1
2 (5)(4  11)

¬37.5
The area of trapezoid ABCD is 37.5 units2. L (3, 0)
6. O (7, 2) x
D (1, 5) C (8, 5)
y
M (1, 2)

N (3, 4)
Explore: To find the area of the rhombus, we
x need to know the length of each diagonal.
B (4, 1)
A (1, 1) Plan: Use coordinate geometry to find the length
of each diagonal. Use the formula to find the area
of rhombus LMNO.
Solve: Let OM be d1 and LN be d2.
Base: Since AB and D
C
 are horizontal, find their Subtract the x-coordinates of O and M to find that
lengths by subtracting the x-coordinates of their d1 is 8.
endpoints. Subtract the y-coordinates of L and N to find that
b1  AB  4  1 d2 is 4.
 3 or 3
A ¬1

2 d1d2
b2  DC  8  1
 7 or 7 ¬1

2 (8)(4) or 16
Height: Because the bases are horizontal The area of rhombus LMNO is 16 units2.
segments, the distance between them can be Examine: The rhombus is made up of two
measured on a vertical line. Subtract the M
congruent triangles with base O . Each triangle
y-coordinates. has base 8 and height 2. So the area of the
h  5  (1) or 6
Area: A ¬1
1

rhombus  2 2 8 2  16.
2 h(b1  b2)

9. N (3, 6) y
¬1
2 (6)(3  7)

¬30
The area of trapezoid ABCD is 30 units2. M (4, 2) O (2, 2)
7. y
x
A (2, 2) B (4, 2)
L (3, 2)

x
Explore: To find the area of the rhombus, we
C (3, 2) need to know the length of each diagonal.
D (1, 2)
Plan: Use coordinate geometry to find the length
of each diagonal. Use the formula to find the area
Base: Since DC  and A
B
 are horizontal, find their of rhombus LMNO.
lengths by subtracting the x-coordinates of their Solve: Let MO be d1 and NL be d2.
endpoints. Subtract the x-coordinates of M and O to find that
b1  DC  3  1 d1 is 2.
 2 or 2 Subtract the y-coordinates of N and L to find that
d2 is 8.
b2  AB  4  (2)
 6 or 6 A ¬1

2 d1d2
Height: Because the bases are horizontal ¬1

2 (2)(8) or 8
segments, the distance between them can be The area of rhombus LMNO is 8 units2.
measured on a vertical line. Subtract the Examine: The rhombus is made up of two
y-coordinates. O
congruent triangles with base M. Each triangle
h  2  (2) or 4 has base 2 and height 4.
Area: A ¬1
2 h(b1  b2)

So the area of the rhombus  2 1 
2 2 4  8.

¬1
2 (4)(2  6)

¬16
The area of trapezoid ABCD is 16 units2.

553 Extra Practice


10. N (1, 12) y Page 777 Lesson 11-3
1. When the side length of a square is s, the
8 perimeter is 4s.
4 4s ¬P
O (5, 4) M (3, 4) 4s ¬54
x s ¬13.5
4 8 Now find the area.
A ¬s2
L (1, 4) ¬(13.5)2
¬182.25 or about 182.3
Explore: To find the area of the rhombus, we The area of the square is approximately 182.3 ft2.
need to know the length of each diagonal. 2.
Plan: Use coordinate geometry to find the length
of each diagonal. Use the formula to find the area
of rhombus LMNO. 9 9
Solve: Let OM be d1 and NL be d2.
Subtract the x-coordinates of O and M to find that
60
d1 is 8.
Subtract the y-coordinates of N and L to find that 4.5 4.5
d2 is 16. In a 30°-60°-90° triangle, when the side opposite
A ¬1

2 d1d2
the 30° angle is x units long, the side opposite the
60° angle is x3 units long. The height of the
¬1

2 (8)(16) or 64 equilateral triangle is x3 or 4.53  in.
The area of rhombus LMNO is 64 units2.
Examine: The rhombus is made up of two A ¬1

2 bh
M
congruent triangles with base O. Each triangle ¬1
2 (9)(4.53
)

has base 8 and height 8.
¬20.253  or about 35.1
So the area of the rhombus  2 1
2 8 8  64.
  The area of the triangle is approximately 35.1 in2.
11. y 3.
8
M (4, 4) B
4
L (2, 2) x
8 4 4 8
A D C
4 N (10, 2)
Apothem: The central angles of a regular octagon
8 O (4, 8) are all congruent. Therefore, the measure of each
36
0
angle is  8 or 45. BD
 is an apothem of the
Explore: To find the area of the rhombus, we octagon. It bisects ABC and is a perpendicular
need to know the length of each diagonal.
C
bisector of A . So, mABD  1 2 (45°)  22.5°.

Plan: Use coordinate geometry to find the length
of each diagonal. Use the formula to find the area Since AC  6, AD  3. Write a trigonometric ratio
of rhombus LMNO. D
to find the length of B.
AD
Solve: Let LN be d1 and MO be d2. tan ABD ¬
BD
Subtract the x-coordinates of L and N to find that 3
d1 is 12. tan 22.5° ¬
BD
Subtract the y-coordinates of M and O to find that (BD)tan 22.5° ¬3
d2 is 12. 3
BD ¬
tan 22.5°
A ¬1

2 d1d2 BD ¬7.24
¬1

2 (12)(12) or 72 Area: The perimeter is 8(6) or 48 ft.
The area of rhombus LMNO is 72 units2.
A ¬1

2 Pa
Examine: The rhombus is made up of two
N
congruent triangles with base L . Each triangle ¬1

2 (48)(7.24)
has base 12 and height 6.
¬173.8
So the area of the rhombus  2 1
2 12 6  72.
  So, the area of the octagon is about 173.8 ft2.

Extra Practice 554


4. 6. The area of the shaded region is the difference
between the area of the rectangle and the area of
B the circle. Since the diameter of the circle is 6, the
radius is 3.
shaded area ¬area of rectangle  area of circle
A D C ¬w  r2
¬15(6)  (3)2
Perimeter: The central angles of a regular ¬90  9 or about 61.7
decagon are all congruent. Therefore, the measure To the nearest tenth, the area of the shaded
360
of each angle is  D
10 or 36. B is an apothem region is 61.7 ft2.
of the decagon. It bisects ABC and is a 7.
perpendicular bisector of AC
. So, mABD 
1(36)  18. Write a trigonometric ratio to find the 7 in .
2 B
D
length of A.
AD
tan ABD ¬ 
BD
AD D
tan 18° ¬
22

A C
22 tan 18° ¬AD
7.148 ¬AD The area of the shaded region is the difference
Then AC  2(7.148) or 14.296, and the perimeter between the area of the circle and the area of the
is about 10(14.296) or 142.96 centimeters. pentagon. First, find the area of the circle.
A ¬r2
Area: A ¬1

2 Pa ¬(7)2
¬1

2 (142.96)(22)
¬153.9
To find the area of the pentagon, find the
¬1572.6
apothem and the perimeter.
So, the area of the decagon is about 1572.6 cm2.
Apothem: The central angles of a regular
5. D pentagon are all congruent. Therefore, the
360
measure of each angle is  5 or 72. B D
 is an
6 cm apothem of the pentagon. It bisects ABC and is
A
C
a perpendicular bisector of A . So, mABD
60  1
(72)  36. Write a trigonometric ratio to find
2
E B C D
the length of B.
B D
cosABD ¬ 
AB
The area of the shaded region is the difference
BD
between the area of the circle and the area of cos36° ¬ 7

ECD. First, find the area of the circle. 7cos36° ¬BD
A  r2 5.6631 ¬BD
 (6)2 Perimeter: Write a trigonometric ratio to find
 113.1 D
the length of A .
AD
To find the area of the triangle, use properties of sinABD ¬ AB

30°-60°-90° triangles. The hypotenuse of ABC is AD
sin36° ¬ 
7
6, so BC, the longer leg, is 3 3. Since EC 
7sin36° ¬AD
2(BC), EC  6 3. Next, find the height of ECD, 4.1145 ¬AD
DB. Since mDCB is 60, DB   3BC  AC  2(AD)  8.229, and the perimeter is
 33 3 or 9. 5(AC)  5(8.229) or 41.145.
Use the formula to find the area of the triangle. Area: A ¬1

2 Pa
A ¬1
2 bh ¬1

2 (41.145)(5.6631)
¬1 ¬116.5
2 (63)(9)

¬46.8 To the nearest tenth, the area of the shaded
To the nearest tenth, the area of the shaded region is 153.9  116.5 or 37.4 in2.
region is 113.1  46.8 or 66.3 cm2.

555 Extra Practice


Page 777 Lesson 11-4 5. First, separate the figure into regions. Draw an
1. The figure can be separated into a semicircle with auxiliary segment from B to D. This divides the
a radius of 3.5 units and a rectangle with figure into ABD and BCD.
dimensions 7 units by 24 units. y
area of irregular figure
 area of semicircle  area of rectangle
¬1
2 r  w
 2 h2
B (3, 0)
¬1
2 (3.5)  7(24)
 2
x
¬6.125  168 C (2, 1)
h1
¬187.2
To the nearest tenth, the area of the irregular A (5, 3) D (2, 3)
figure is 187.2 units2.
2. The figure can be separated into a rectangle with  by
For ABD, find the length of base AD
dimensions 20 units by 15 units and a triangle subtracting the x-coordinates of A and D. Find the
with a base that measures 15  15 or 30 units height by subtracting the y-coordinates. For
and a height of 8 units.  by subtracting
BCD, find the length of base CD
area of irregular figure the y-coordinates of C and D. Find the height by
 area of rectangle  area of triangle subtracting the x-coordinates.
 w  1  area of ABCD ¬area of ABD  area of BCD
2 bh
 20(15)  1 ¬1
 1
2 b1h1  2 b2h2
2 (30)(8)
 420 ¬1 1
2 (7)(3)  2 (2)(5)

The area of the irregular figure is 420 units2.
¬15.5
3. The figure can be separated into a rectangle with The area of quadrilateral ABCD is 15.5 units2.
dimensions 4 units by 16 units, a triangle with a
6. First, separate the figure into regions. Draw an
base that measures 4 units and a height of
auxiliary segment from L to O. This divides the
25  16 or 9 units, and a semicircle with a radius
figure into LOP and trapezoid LMNO.
of 2 units.
area of irregular figure L (1, 4) y
 area of rectangle  area of triangle 
M (3, 2)
area of semicircle h1
 w  1 1 2
2 bh  2 r
 P (3, 1) h2
 4(16)  1
 1
2 (4)(9)  2 (2)
2 x
 82  2 N (3, 1)
O (1, 2)
 88.3
To the nearest tenth, the area of the irregular
figure is 88.3 units2.  by
For triangle LOP, find the length of base LO
4. The figure is a triangle with base ST. Subtract subtracting the y-coordinates of L and O. Find the
the y-coordinates of S and T to find ST  3  0 height by subtracting the x-coordinates. For
or 3. The height of the triangle is 3. trapezoid LMNO, find the lengths of the bases by
R (0, 5) subtracting the y-coordinates. Find the height by
y subtracting the x-coordinates.
S (3, 3) area of LMNOP
 area of LOP  area of trapezoid LMNO
 1 1
2 bh1  2 h1(b1  b2)


T (3, 0) x  1 1
2 (6)(2)  2 (4)(6  3)

 24
The area of LMNOP is 24 units2.

Page 777 Lesson 11-5


A ¬1
2 bh 1. Use the formula to find the total area of the
1
¬2(3)(3) sectors.
¬4.5 A ¬ N 2
r
360
The area is 4.5 units2. 36  36
¬ 360 (10 )
2

¬20
¬62.8 in2
To find the probability, divide the area of the
sector by the area of the circle. The area of the
circle is r2, and r ¬10.

Extra Practice 556


area ofsector
P(orange) ¬
area of circle 5. Area of the shaded region: The area of the

20 shaded region is the area of the hexagon minus
¬
 10
2 the area of the circle.
¬1
 180(6  2)
2 mEAD  2 
5 or 0.20 mBAD  1  1 
6
The probability that a random point is in the 1
orange sectors is 0.20. ¬2(120)  60
2. Use the formula to find the total area of the sectors. For the area of the hexagon, find the apothem and
N 2 perimeter.
A ¬
360 r
60 40
¬ 360 (10 )
2

250
¬ 9 
E B 26
¬87.3 in2
To find the probability, divide the area of the
sector by the area of the circle. The area of the 13
circle is r2, and r ¬10. 60
area ofsector C
P(blue) ¬
area of circle
A D
25
 0
9 
The apothem is 1

2 (26) or 13. Using properties of
¬  102

30°-60°-90° triangles, if AD  x, BD  x3
, so
¬25

90 or about 0.28
x3 ¬13
The probability that a random point is in the blue 13 
3
x ¬ 
3
sectors is about 0.28.
13
3 26
3
3. Use the formula to find the total area of the sectors. Then AD   3 , AC 
  
3 , and the perimeter
26
A ¬N 2
r is 6 3  or 523
3
.
360
80  110 A ¬1
2 Pa
¬ 360 (10 )
 2
1
475 ¬2(523)(13)
¬ 9 
¬338 3
¬165.8 in2
The radius of the circle is 13, so
To find the probability, divide the area of the area of shaded region
sector by the area of the circle. The area of the  area of hexagon  area of circle
circle is r2, and r ¬10.
 338 3  (132)
area ofsector
P(green) ¬
area of circle  338 3  169
47
5

9   54.5 units2
¬ 
 102 Probability:
19 shaded area
¬ 
36 or about 0.53
P(shaded) ¬ 
area of hexagon
The probability that a random point is in the 54.5
¬
green sectors is about 0.53. 338 3
¬0.09
4. Area of the shaded region: The area of the
The probability that a random point is in the
shaded region is the area of the large circle minus
shaded region is about 0.09.
the area of the small circle, where the radii are r1
 100 units and r2  50 units. 6. Area of shaded region: The area of the shaded
area of shaded region region is the area of the triangle minus the areas
 area of large circle  area of small circle of the two circles.
 r12  r22 Area of triangle: b ¬16  2 15
h ¬3  5  5  13
 (1002)  (502) 1
 7500 A ¬2bh
 23,561.9 units2 1
¬2 (16  215
)(13)
Area of the square: The square is 80  50 ¬154.4 units2
 100  50  80 or 360 units across.
Area of circles  (3)2  (5)2 ¬(32  52)
A ¬s2
¬34 units2
¬3602
¬129,600 units2 Area of shaded region ¬154.4  34 units2
¬47.6 units2
Probability:
Probability:
shaded area
P(shaded) ¬
area of 
square P(shaded) ¬ shaded area
area of tri angle
0
750
¬
129,600 ¬ 
47.6
154.4
¬0.18 ¬0.31
The probability that a random point is in the The probability that a random point is in the
shaded region is about 0.18. shaded region is about 0.31.

557 Extra Practice


Page 778 Lesson 12-1 5. Use rectangular dot paper to draw a net. Let one
1. unit on the dot paper represent 1 unit of length.

corner view back view


2.

To find the surface area of the prism, add the


areas of the rectangles. There are two 2-by-3
rectangles, two 2-by-6 rectangles, and two 3-by-6
corner view back view rectangles.
3. The base is a pentagon, and the four faces meet in Surface area ¬2(2 3)  2(2 6)  2(3 6)
a point. So this solid is a pentagonal pyramid. ¬12  24  36 or 72
Base: pentagon OXNEP The surface area of the rectangular prism is
Faces: OXNEP, PET, ETN, NTX, XTO, 72 units2.
OTP 6. Use rectangular dot paper to draw a net. Let one
Edges: P E
, EN
, N
X, X
O, O
P, T
P, T
E
, T
N, T
X
, T
O unit on the dot paper represent 1 unit of length.
Vertices: T, P, E, N, X, O
4. This solid has a circle for a base and a vertex. So
it is a cone.
Base: circle L
Vertex: Z
5. There are two octagon-shaped bases, and the To find the surface area of the pyramid, add the
other faces are parallelograms. So this solid is an areas of the base and the areas of the other faces.
octagonal prism. The base is a 2-by-2 square, and each of the other
Bases: ABCDEFGH, STUVWXYZ four faces is a triangle with a base measuring
Faces: ABCDEFGH, STUVWXYZ, ABXY, BCWX, 2 units and a height of 2 units.
CDVW, DEUV, FEUT, GFTS, HGSZ, HAYZ
Surface area ¬22  41 2 2 2

Edges: A B
, BC
, C
D
, DE, E
F, F
G
, GH, A
H
, S
T, T
U
,
V
U , VW
, WX, X
Y
, Y
Z, Z
S, A
Y
, BX
, CW
, DV
, E
U
, ¬4  8 or 12
T
F , G
S, H
Z The surface area of the square pyramid is 12 units2.
Vertices: A, B, C, D, E, F, G, H, S, T, U, V, W, X, Y, Z 7. We need to know the length of the large
rectangular upper surface. Use the Pythagorean
Page 778 Lesson 12-2 Theorem.
1. 32  42 ¬2
9  16 ¬2
25 ¬2
5 ¬
Use rectangular dot paper to draw a net. Let one
2. unit on the dot paper represent 1 unit of length.

3.

To find the surface area of the solid, add the areas


of the rectangular and triangular surfaces. The
base is a 2-by-4 rectangle, the vertical face is a
4.
2-by-3 rectangle, the top is a 2-by-5 rectangle, and
each of the two triangular sides has base length 4
and height 3.
Surface area ¬2 4  2 3  2 5  21
2 4 3

¬8  6  10  12 or 36
The surface area of the solid is 36 units2.

Extra Practice 558


Page 778 Lesson 12-3 Find the surface area.
1. The base is a 7-by-5 rectangle, so the perimeter is T ¬L  2B
2(7)  2(5) or 24 units. ¬94.64  2(2.6)(6.5)
Find the lateral area. ¬128.44
L ¬Ph The surface area is about 128.4 units2.
¬(24)(4) 5. First, find the measure of the third side of the
¬96 triangular base.
The lateral area is 96 units2. c2 ¬a2  b2
Find the surface area. 302 ¬182  b2
T ¬L  2B 900 ¬324  b2
¬96  2(7 5) 576 ¬b2
¬166 24 ¬b
The surface area is 166 units2. The perimeter is 18  30  24 or 72 units.
2. First, find c. Find the lateral area.
L ¬Ph
C 4 B ¬(72)(26)
¬1872
c 3 3 The lateral area is 1872 units2.
4 4 Find the surface area.
D 8 A T ¬L  2B
Use the Pythagorean Theorem. ¬1872  2 1 2 (24 18)

c2 ¬a2  b2 ¬2304
c2 ¬42  32 The surface area is 2304 units2.
c2 ¬25 6. First, find c and the height h.
c ¬5
15
The perimeter of base ABCD is 5  8  3  4 or
20 units.
Find the lateral area.
L ¬Ph h c
¬(20)(9)
¬180 15 15 45
The lateral area is 180 units2. 30
Find the surface area.
T ¬L  2B Because the triangle is a 45°-45°-90° triangle,
¬180  2 1 h  15 and c  152 . So the perimeter is
2 (3)(4  8)

30  15  15  152  or 60  152.
¬216
Find the lateral area.
The surface area is 216 units2.
L ¬Ph
3. First, find the measure of the third side of the ¬(60  15 2)(42)
triangular base. ¬2520  630 2
c2 ¬a2  b2 ¬3411.0
c2 ¬62  82 The lateral area is about 3411.0 units2.
c2 ¬100 Find the surface area.
c ¬10 T ¬L  2B
The perimeter, then, is 10  6  8 or 24 units. 2)  2 1
¬(2520  630 2 (15)(30  15)

Find the lateral area.
L ¬Ph ¬3195  6302 
¬(24)(9) ¬4086.0
¬216 The surface area is about 4086.0 units2.
The lateral area is 216 units2. 7. Find the hypotenuse of the base using the
Find the surface area. Pythagorean Theorem.
T ¬L  2B c2 ¬a2  b2
¬216  2 1 c2 ¬62  82
2 (6 8)

c2 ¬100
¬264 c ¬10
The surface area is 264 units2. The perimeter is 6  8  10  24 in.
4. The base is a 2.6-by-6.5 rectangle, so the T ¬L  2B
perimeter is 2(2.6)  2(6.5) or 18.2 units. T ¬Ph  2B
Find the lateral area. 228 ¬24h  2 12 (8 6)

L ¬Ph 228 ¬24h  48
¬(18.2)(5.2) 180 ¬24h
¬94.64 7.5 ¬h
The lateral area is about 94.6 units2.
The height of the prism is 7.5 in.

559 Extra Practice


8. Let x be the measure of the other leg of the 8. The radius of the base and the height of the
triangular base. cylinder are given. Substitute values in the
The perimeter of the base is 15  25  x or 40  x. formula for surface area.
The area of the base is 21 15 x. T ¬2rh  2r2
T ¬L  2B ¬2(16.5)(16.5)  2(16.5)2
¬3421.2
T ¬Ph  2B
The surface area is approximately 3421.2 m2.

1380 ¬(40  x) 18  2 1 
2 15 x

1380 ¬720  18x  15x Page 779 Lesson 12-5
660 ¬33x 1. The slant height is 9 cm. The perimeter of the
20 ¬x base is 4(7) or 28 cm, and the area of the base is
The length of the other leg of the base is 20 in. 72 or 49 cm2.
T ¬1
2 P  B

Page 779 Lesson 12-4 ¬1
2 (28)(9)  49

1. Substitute values in the formula for surface area.
¬175
T ¬2rh  2r2
The surface area is 175 cm2.
¬2(2)(3.5)  2(2)2
¬69.1 2. Find the perimeter and area of the base.
The surface area is approximately 69.1 ft2.
2. If d  15 in., r  7.5 in. Substitute values in the
formula for surface area.
T ¬2rh  2r2
30
¬2(7.5)(20)  2(7.5)2
¬1295.9
The surface area is approximately 1295.9 in2.
3. Substitute values in the formula for surface area. 10.5
T ¬2rh  2r2 360°
The central angle of the hexagon measures  6
¬2(3.7)(6.2)  2(3.7)2 or 60°. Let a represent the measure of the angle
¬230.2 formed by a radius and the apothem. Then,
The surface area is approximately 230.2 m2. 60
a  2 or 30. In a 30°-60°-90° triangle, if the side
4. If d  19 mm, r  9.5 mm. Substitute values in opposite the 30° angle has a length of x,
the formula for surface area. 10.5
the other leg has a length of x 3. Here, x  
T ¬2rh  2r2 2
¬2(9.5)(32)  2(9.5)2  5.25, so the length of the apothem is 5.253.
¬2477.1 Next, find the perimeter and area of the base.
The surface area is approximately 2477.1 mm2. P ¬6s
5. The radius of the base and the height of the ¬6(10.5) or 63
cylinder are given. Substitute values in the B ¬1
2 Pa
formula for surface area. 1
¬2(63)(5.253
)
T ¬2rh  2r2
¬2(1.5)(4)  2(1.5)2 ¬165.3753 
¬51.8 Finally, find the surface area.
The surface area is approximately 51.8 in2. T ¬1

2 P + B
6. The diameter of the base and the height of the ¬1
2 (63)(18)  165.3753
 
cylinder are given. If d  14 ft, r  7 ft. ¬853.4
Substitute values in the formula for surface area. The surface area is approximately 853.4 in2.
T ¬2rh  2r2
¬2(7)(32.5)  2(7)2
¬1737.3
The surface area is approximately 1737.3 ft2.
7. The diameter of the base and the height of the
cylinder are given. If d  1 in., r  0.5 in.
Substitute values in the formula for surface area.
T ¬2rh  2r2
¬2(0.5)(10.5)  2(0.5)2
¬34.6
The surface area is approximately 34.6 in2.

Extra Practice 560


3. Find the perimeter and area of the base. ¬253 
Find the surface area of the pyramid.
T ¬1
2 P  B


¬ ¬1
2 (30)(102
 )  253
36 ¬255.4
a The surface area is approximately 255.4 cm2.
5. To find the surface area, first find the length of
11 22 the sides of the base. A right triangle is formed by
36
0° the slant height, the edge with length 17, and half
The central angle of the pentagon measures  5 the side of the base. Let a be one-half the side
or 72°. Let  represent the measure of the angle length of the base.
formed by a radius and the apothem. Then, c2 ¬a2  b2
  72
2
or 36. Use trigonometry to find the length 172 ¬a2  152
of the apothem. 289 ¬a2  225
11
tan36° ¬ a
 64 ¬a2
a ¬ 11
 8 ¬a
tan 36° So the side length of the base is 2(8) or 16 ft, and
¬15.14 the perimeter is 4(16) or 64 ft. Find the surface
Next, find the perimeter and area of the base. area.
P ¬5s
T ¬1
2 P  B

¬5(22) or 110
B ¬1
 ¬1
2 (64)(15)  16
 2
2 Pa
¬1
 ¬736
2 (110)(15.14)
The surface area is 736 ft2.1 
¬832.7 2
6. Each side, including the base, is an equilateral
Finally, find the surface area.
triangle of side length 3 centimeters. The altitude
T ¬1
2 P  B

3 1
of this triangle is  2 (3), or 1.5
 3. Find the
¬1
2 (110)(40)  832.7
 area of a side.
¬3032.7 A ¬1
2 bh
The surface area is approximately 3032.7 m2.
¬1
2 (3)(1.53
 )
4. To find the surface area, first find the slant height
¬2.25 3
of the pyramid. The slant height is the altitude of
Find the total area.
a triangle with base of 10 and sides of 15.
T ¬4A
¬4(2.25 3)
¬9 3 or about 15.6
The surface area is approximately 15.6 cm2.

Page 779 Lesson 12-6


1. Use the Pythagorean Theorem to find the slant
15 15 height, .
 2 ¬a2  b2
2 ¬62  102
2 ¬136
 ¬11.66
Now use the formula for the surface area.
T ¬r  r2
5 5
¬(6)(11.66)  (6)2
Since the triangle is isosceles, the altitude bisects
¬332.9
the base. Use the Pythagorean Theorem to find .
The surface area is approximately 332.9 in2.
c2 ¬a2  b2
152 ¬52  2 2. Use the Pythagorean Theorem to find the radius.
225 ¬25  2 c2 ¬a2  b2
200 ¬2 342 ¬r2  302
1156 ¬r2  900
10 2 ¬
256 ¬r2
The base is an equilateral triangle. Its perimeter
16 ¬r
is 3(10) or 30 cm. Its altitude is 3 times half the Now use the formula for the surface area.
side length, or 53 cm. T ¬r  r2
B ¬1 
2 bh ¬(16)(34)  (16)2
¬2513.3
¬1
2 (10)(53
 )
The surface area is approximately 2513.3 ft2.

561 Extra Practice


3. For an isosceles right triangle, the length of the Page 780 Lesson 12-7
hypotenuse is 2  times the leg length. 1. Use the formula for the surface area of a sphere.
  17 2 T ¬4r2
Now use the formula for the surface area. ¬4(120)2
T ¬r  r2 ¬180,955.7
¬(17)(17 2)  (17)2 The surface area is approximately 180,955.7 ft2.
¬2191.9 2. If d  42.5 m, then r  21.25 m. Use the formula
The surface area is approximately 2191.9 cm2. for the surface area of a sphere.
4. Use the Pythagorean Theorem to find the slant T ¬4r2
height, . ¬4(21.25)2
2 ¬a2  b2 ¬5674.5
2 ¬(2.2)2  (10.5)2 The surface area is approximately 5674.5 m2.
2 ¬115.09 3. If d  2520 mi, then r  1260 mi. Use the
 ¬10.728 formula for the surface area of a sphere.
Now use the formula for the surface area. T ¬4r2
T ¬r  r2 ¬4(1260)2
¬(2.2)(10.728)  (2.2)2 ¬19,950,370.0
¬89.4 The surface area is approximately 19,950,370.0 mi2.
The surface area is approximately 89.4 in2. 4. Use the formula for the surface area of a sphere.
5. Use the formula for the surface area. T ¬4r2
T ¬r  r2 ¬4(33)2
¬(6.25)(7.0)  (6.25)2 ¬13,684.8
¬260.2 The surface area is approximately 13,684.8 cm2.
The surface area is approximately 260.2 cm2. 5. First, find the radius of the hemisphere.
6. Use the Pythagorean Theorem to find the slant C ¬2r
height, . 14.1 ¬2r
14
.1
2 ¬a2  b2 2 ¬r

2 ¬302  82 To find the surface area, find half the surface area
2 ¬964 of the sphere and add the area of the great circle.
 ¬31.0483 surface area ¬12 (4r )  r
 2 2
Now use the formula for the surface area.
T ¬r  r2 ¬1 14.1 2 14
.1 2
2 [4( 2 ) ]  ( 2 )
  
¬(30)(31.0483)  (30)2 ¬47.5
¬5753.7 The surface area is approximately 47.5 cm2.
The surface area is approximately 5753.7 ft2. 6. First, find the radius of the sphere.
7. Use the Pythagorean Theorem to find the radius. C ¬2r
c2 ¬a2  b2 50.3 ¬2r
50
.3
402 ¬282  r2 2 ¬r

1600 ¬784  r2 8.0055 ¬r
816 ¬r2 Now find the surface area of the sphere.
28.5657 ¬r T ¬4r2
Now use the formula for the surface area. ¬4(8.0055)2
T ¬r  r2 ¬805.4
¬(28.5657)(40)  (28.5657)2 The surface area is approximately 805.4 in2.
¬6153.2 7. Use the formula for the surface area of a sphere,
The surface area is approximately 6153.2 in2. and note that r2 is the area of a great circle.
8. Use the Pythagorean Theorem to find the slant T ¬4r2
height, . ¬4(98.5)
2 ¬a2  b2 ¬394
2 ¬(7.5)2  (2.5)2 The surface area of the sphere is 394 m2.
2 ¬62.5 8. First, find the radius of the hemisphere.
 ¬7.906 C ¬2r
Now use the formula for the surface area. 3.1 ¬2r
T ¬r  r2 3.
1
2 ¬r

¬(2.5)(7.906)  (2.5)2
¬81.7 0.4934 ¬r
The surface area is approximately 81.7 cm2. To find the surface area, find half the surface area
of the sphere and add the area of the great circle.
surface area ¬12 (4r )  r
 2 2

¬1
2 [4(0.4934) ]  (0.4934)
 2 2

¬2.3
The surface area is approximately 2.3 in2.

Extra Practice 562


9. Find half the surface area of the sphere and add Page 780 Lesson 13-2
the area of the great circle, noting that r2 is the
area of the great circle. 1. V ¬1
3 Bh
surface area ¬1 ¬1
 2
2 (4r )  r
 2 2
3 (5) (7.5)
¬1 ¬62.5
2 [4(31,415.9)]  31,415.9

The volume of the pyramid is 62.5 ft3.
¬94,247.7
The surface area is 94,247.7 ft2. 2. V ¬13 r h
 2
1
¬3(10)2(40)
Page 780 Lesson 13-1 ¬4188.8
1. V ¬Bh The volume of the cone is approximately
¬(52.5)(79.4)(102.3) 4188.8 mm3.
¬426,437.6
3. First, use the Pythagorean Theorem to find the
The volume is approximately 426,437.6 m3.
missing leg length on the base, which is a right
2. The diameter of the base, the diagonal, and the triangle.
lateral edge of the cylinder form a right triangle. a2  b2 ¬c2
Use the Pythagorean Theorem to find the height. 82  b2 ¬172
a2  b2 ¬c2 64  b2 ¬289
h2  162 ¬302 b2 ¬225
h2  256 ¬900 b ¬15
h2 ¬644 Now find the volume.
h ¬ 644
V ¬1 
3 Bh
Now find the volume.
3 2 (8 15)(12)
¬1
V ¬r2h  1
¬(8)2( 644 ) ¬240
¬5102.4 The volume of the pyramid is 240 in3.
The volume is approximately 5102.4 ft3. 4. Use the Pythagorean Theorem to find the height.
3. V¬¬Bh a2  b2 ¬c2
¬1
2 (10)(20  7)(16)
 52  h2 ¬132
¬2160 25  h2 ¬169
The volume is 2160 in3. h2 ¬144
4. The solid is a rectangular prism with another h ¬12
rectangular prism removed from the inside. Now find the volume.
volume of solid V ¬1
3 r h
 2
 initial volume  volume of removed prism ¬1
3 (2.5) (12)
 2
 B1h  B2h
¬78.5
 (10)2(10)  (5)2(10) The volume is approximately 78.5 m3.
 750
5. Use the Pythagorean Theorem to find the height.
The volume is 750 in3.
a2  b2 ¬c2
5. The solid is a cylinder with a rectangular prism 62  h2 ¬122
removed from it. The square base of the 36  h2 ¬144
rectangular prism has a diagonal of 9 2, so the h2 ¬108
side length of the base is 9. h ¬10.39
volume of solid Now find the volume.
 volume of cylinder  volume of removed prism V ¬1
 r2h  Bh 3 Bh
 (4.5 2)2(21)  (9)2(21) ¬1
3 (10)(6)(10.39)
 970.9 ¬207.8
The volume is approximately 970.9 cm3. The volume is approximately 207.8 m3.
6. The solid is a rectangular prism with another 6. Because the height, the radius, and the slant
rectangular prism removed from it. height form a 45°-45°-90° triangle, the height and
1
volume of solid ¬initial volume  removed volume radius each equal .
 2
¬B1h  B2h 1
V ¬3r2h
¬(15)2(8)  (9)(6)(8)
 12  12 
2
¬1
3  

¬1368
The volume is 1368 in3. ¬0.4
The volume is approximately 0.4 in3.

563 Extra Practice


Page 781 Lesson 13-3 3. Find the ratios of the corresponding parts.
short dimension of right prism
1. V ¬4
3 r
 3 31
short dimension of left prism ¬
 
16
¬4
3 (44 )
 3
¬1.9375
¬356,817.9 ft3 long dimension of right prism 4
3
long dimension of left prism ¬

18
2. First find the radius of the sphere.
C ¬2r ¬2.3889
4 ¬2r Since the ratios are not the same, the prisms are
2 neither similar nor congruent.
 ¬r
4. Use the Pythagorean Theorem to find the height
Now find the volume.
of the cone on the right.
V ¬43 r
 3 a2  b2 ¬c2
3
62  h2 ¬102
3   
¬4
 2
36  h2 ¬100
¬1.1 m3
h2 ¬64
3. The volume of a hemisphere is one-half the h ¬8
volume of a sphere. The two cones have the same radius and height,
2  3 r 
V ¬1 4 3 so they are the same shape and size. The cones
¬2
3 (17 )
 3 are congruent.
¬10,289.8 mm3 5. In the cylinder on the right, the diameter, the
height, and the diagonal form a right triangle.
4. V ¬4
3 r
 3
Use the Pythagorean Theorem to find the height.
¬4
3 (1.5)
 3
302  h2¬¬342
¬14.1 cm3 h2¬¬256
5. The volume of a hemisphere is one-half the h¬¬16
volume of a sphere. Since the cylinder on the right has a diameter of
2  3 r 
V ¬1 4 3 30, it has a radius of 15. Both cylinders have the
same radius and the same height. The cylinders
¬2
3 (72
 )3 are congruent.
¬2031.9 m3 6. Find the ratios of the corresponding parts.
6. The volume of a hemisphere is one-half the side of larger base 20
volume of a sphere. side of smaller base ¬
 
52
2  3 r 
V ¬1 4 3
¬22

¬2
3 (45)
 3 larger height 32
smaller height ¬
 
¬190,851.8 ft3 82
7. V ¬4 ¬2
3 r
 3 2
The ratios of the measures are equal, so the
¬4
3 (0.5)
 3
prisms are similar. Since the scale factor is not 1,
¬0.5 in 3
the solids are not congruent.

Page 781 Lesson 13-4 Page 781 Lesson 13-5


1. The two spheres have the same shape but 1. • Plot the x-coordinate first. Draw a segment from
different sizes. They are similar. the origin 3 units in the positive direction.
2. Find the ratios of the corresponding parts. • To plot the y-coordinate, draw a segment 3 units
shortest side of right prism 4.2
5 in the negative direction.
shortest side of left prism ¬

0.5 • To plot the z-coordinate, draw a segment 3 units
¬8.5 in the negative direction.
longest side of right prism 21.
25 • Label the coordinate A.
longest side of left prism ¬

2.5 • Draw the rectangular prism and label each
¬8.5 vertex.
medium side of right prism 17 z
medium side of left prism ¬
 
2.0
¬8.5 (0, 3, 0) O
The ratios of the measures are equal, so the (0, 3, 3) (0, 0, 0) y
prisms are similar. Since the scale factor is not 1,
(3, 3, 0)
the solids are not congruent. (0, 0, 3)

A (3, 3, 3) (3, 0, 0)
x (3, 0, 3)

Extra Practice 564


2. • Plot the x-coordinate first. Draw a segment from ( 4, 2, 0) z
the origin 1 unit in the negative direction.
• To plot the y-coordinate, draw a segment 2 units ( 4, 0, 0)
(0, 0, 0)
in the positive direction.
• To plot the z-coordinate, draw a segment 3 units
in the negative direction. (0, 2, 0) O y
( 4, 0, 4)
• Label the coordinate E.
• Draw the rectangular prism and label each vertex. x Q ( 4, 2, 4)
z (0, 2, 4) (0, 0, 4)

( 1, 0, 0)
( 1, 2, 0)
6. • Plot the x-coordinate first. Draw a segment from
(0, 0, 0) the origin 3 units in the negative direction.
O y
( 1, 0, 3) (0, 2, 0) • To plot the y-coordinate, draw a segment 1 unit
in the positive direction.
E ( 1, 2, 3)
x
(0, 2, 3)
• To plot the z-coordinate, draw a segment 4 units
(0, 0, 3)
in the negative direction.
• Label the coordinate Y.
3. • Plot the x-coordinate first. Draw a segment from • Draw the rectangular prism and label each
the origin 3 units in the positive direction. vertex.
• To plot the y-coordinate, draw a segment 1 unit z
in the negative direction. ( 3, 0, 0)
• To plot the z-coordinate, draw a segment 2 units ( 3, 1, 0)
in the positive direction.
(0, 0, 0) y
• Label the coordinate I.
O
• Draw the rectangular prism and label each vertex. (0, 1, 0)
z Y ( 3, 1, 4)
(0, 0, 2) x
( 3, 0, 4)
(0, 1, 2) (0, 0, 4) (0, 1, 4)
(3, 0, 2)
O 7. Find the distance.
(0, 0, 0) y
I (3, 1, 2) AB ¬
(x2  
x1)2 
(y2 
y1)2 
(z2 
z1)2
(0, 1, 0)
(3, 1, 0) ¬
[3  (
3)]2 
 (3
 3)2
 (
1  1
)2
x (3, 0, 0)
¬76 or 219

4. • Plot the x-coordinate first. Draw a segment from Find the midpoint.
x x y y z z
the origin 2 units in the positive direction.
• To plot the y-coordinate, draw a segment 1 unit
M ¬ 
1

 2
2 ,

1
 2
2 ,

1

2
2

3 
3 
in the negative direction. ¬ 2 , 3 3 11
2, 2
• To plot the z-coordinate, draw a segment 3 units ¬(0, 0, 0)
in the positive direction.
8. Find the distance.
• Label the coordinate Z.
• Draw the rectangular prism and label each vertex. OP ¬ (x2  
x1)2 
(y2 
y1)2 
(z2 
z1)2
z ¬
(2 
2)2 
[4  
(1)]2
 [ 2
4  (3)]
(0, 1, 3) (0, 0, 3) ¬42
Z (2, 1, 3) Find the midpoint.
x x y y z z
 
(0, 1, 0) (2, 0, 3)
O M ¬ 
1
2, 2, 2
2 1 2 1 2

(0, 0, 0) y
2 2 1  4 3  4
(2, 1, 0) ¬ 2 , 2, 2
(2, 0, 0)
x ¬(0, 1.5, 3.5)
9. Find the distance.
5. • Plot the x-coordinate first. Draw a segment from DE ¬ (x2  
x1)2 
(y2 
y1)2 
(z2 
z1)2
the origin 4 units in the negative direction.
¬
(0  0
)2  [5 
 (5)]2 
[3  
(3)]2
• To plot the y-coordinate, draw a segment 2 units
in the negative direction. ¬136
 or 234 
• To plot the z-coordinate, draw a segment 4 units Find the midpoint.
x x y y z z
in the negative direction.
• Label the coordinate Q.
M ¬ 
1

2, 2, 2
2 1 2 1 2

0 0 5  5 3  3
• Draw the rectangular prism and label each vertex. ¬ 2 , 2, 2
¬(0, 0, 0)

565 Extra Practice


10. Find the distance. 12. Find the distance.
JK ¬ (x2  
x1)2 
(y2 
y1)2 
(z2 
z1)2 ST ¬ (x2  
x1)2 
(y2 
y1)2 
(z2 
z1)2
¬
[3  (
1)]2 
 (5
 3)2
 (
3  5)2 ¬ 
[6  (8)]2
 (1
 3)2
 [2 
 (5
)]2
¬144
 or 12 ¬261
 or 329
Find the midpoint. Find the midpoint.
x x y y z z

M ¬ 
1
 2
2 ,

1
 2
2 ,

1

2
2
 
M ¬ 
1

x x
2
2 ,

1
 2
2 ,
y y

1

2
2 z z

1 3 
¬ , 3  5 5 
2 , 2 
3
8 
6 
2 ¬ 2 , 3
 5 
1 
2 , 2
2

¬(1, 1, 1)
¬(1, 1, 1.5)
11. Find the distance.
AZ ¬ (x2  x1)2 (y2  y1)2 
(z2 
z1)2
¬
(4 
2)2 
(5 
1)2 
(3 
6)2
¬153
 or 317
Find the midpoint.
x x y y z z

M ¬ 
1
 2
2 ,

1
 2
2 ,

1

2
2

2  1  6 
¬ 2 , 2 , 2 
4  5  3

¬(1, 2, 1.5)

Extra Practice 566


Mixed Problem Solving and Proof
Chapter 1 Points, Lines, Planes, Solve:
AB ¬(x2  
x1)2 
(y2 
y1)2
and Angles
¬
(6  0)2 
(2 6)2
¬100
 or 10
Page 782 BC ¬
(x2  
x1)2 
(y2 
y1)2
1–3. Sample answer:
¬
[8  (
6)]2 
 [4 
 (2)]
¬200
 or 102 
top
back AC ¬
(x2  
x1)2 
(y2 
y1)2
line 1 A ¬ (8  0
)2  (
4  6)2
front ¬164
 or 241 
B The perimeter is 10  102   241
 or
line 2
approximately 36.9 units.
Examine: The perimeter is between 30 and
line 3 C
40 units, which makes sense for a triangle whose
vertices have coordinates that are between 5 and
10 units apart.
Three planes suggested by the outline are the top, 9. Use the Midpoint Formula.
back, and front planes. Three lines that do not For AB,
x x y y
4.
intersect are line 1, line 2, and line 3.
See the figure for Exercises 1–3. Points A, B, and

M ¬ 
1
2, 2
2 1 2

0  (6) 6  (2)
C might be coplanar, but they are not collinear. ¬2, 2
0 6 62
5. Each measurement is to the nearest foot. So the ¬ 2 , 2
measurements are precise to within 0.5 feet.
¬(3, 2)
6. The measurement is precise to within 0.5 feet. So
the height is between 621.5 feet and 622.5 feet. For BC,
x x y y
7. Explore: The Weston Centre could be as tall as 
M ¬ 
1
2, 2
2 1 2

444.5 feet and as short as 443.5 feet. The Tower 6  8 2  (4)
Life building could be as tall as 404.5 feet and as ¬2, 2
6 
8 
short as 403.5 feet ¬ 2 , 22
 4

Plan: Calculate the difference twice, using
heights that give the greatest possible difference ¬(1, 3)
and then using heights that give the least For AC,
x x y y
possible difference.
Solve: The greatest possible height difference is 
M ¬ 
1
 2
2 ,

1

2
2

0 8 6  (4)
444.5  403.5 or 41 feet. ¬ 2 , 2
The least possible height difference is 0  6 
¬ 2 , 2 
8  4
443.5  404.5 or 39 feet.
¬(4, 1)
So, the difference in heights is between 39 and
41 feet. 10. Use the points (3, 2), (1, 3), and (4, 1) (which
Examine: The two given tower heights have a were found in Exercise 9), and follow the same
difference of 444  404 or 40 feet. This is in the steps as in Exercise 8.
range that was calculated. d1 ¬(x
 2x1)2 (y2 y1)2
8. Explore: The perimeter is the sum of the side ¬ 
[1  (3)] 2 
 (3
 2)2
lengths.
¬41

8
y d2 ¬(x
 2x1)2 (y2 
y1)2
A(0, 6)
¬
(4  1)
2  [1 (
3)]2
4
¬25
 or 5
x
d3 ¬(x
 2x1)2 (y2 
y1)2
8 4 4 8 ¬[4  (
3)]2 
 [1 2]2
B (6, 2 )4
C ( 8 , 4 ) ¬50
 or 52 
8 The perimeter is 5  52   41 or
approximately 18.5 units.
Plan: Use the Distance Formula to find the three 11. ABC has a perimeter of 10  102   241
,
side lengths. Then add them to find the perimeter. which is twice the perimeter of the smaller
triangle.

567 Mixed Problem Solving and Proof


12. Explore: The exit numbers represent distances in 3. Sixteen states are contained in the left circle,
miles. including the overlap region.
Plan: Find the exit number for the point halfway 4. Twelve states are contained in the right circle,
between Exits 128 and 184, then add 3. including the overlap region.
128 184
Solve: midpoint   2 or 156 5. Seven states are in the overlap region, which
156  3  159 represents states that have less than 2,000,000
The exit number for the Hays exit is 159. people and are less than 34,000 square miles in
Examine: Exits 159 and 128 are 159  128 or area.
31 miles apart. Exits 184 and 159 are 184  159 6. What the March Hare is talking about is the rule
or 25 miles apart. It makes sense that the answer “If Alice means it, Alice says it.” What Alice is
is 159 since 159 is about halfway between 128 talking about is the rule “If Alice says it, Alice
and 184 but is closer to 184. means it.” The Hatter is correct; Alice exchanged
13. If there are forty gondolas, the spokes form 40 the hypothesis and conclusion.
equal-sized angles that add up to 360°. Each 7. “Say what you mean” translates as “If you mean
angle will have a measure of 
360
 it, say it,” and “Mean what you say” translates as
40 or 9. “If you say it, mean it.” The two are converses of
14. Because A, B, and D lie along a straight line,
each other.
ABC and CBD form a linear pair and are
supplementary. 8. Let p and q represent the parts of the statement.
p: An unknown person attempts to give one an item
15. Because the two angles are supplementary,
q: One does not accept it and notifies airline
mABC  mCBD ¬180
personnel immediately.
110  mCBD ¬180
Given: p → q and p
mCBD ¬70
Conclusion: She (Candace) should not accept it
16. Sample answer: isosceles triangle, rectangle, and she should notify airline personnel
pentagon, hexagon, square immediately
17. triangle: convex irregular; rectangle: convex 9. Given: B is the midpoint of A C and C is the
irregular; pentagon: convex irregular; hexagon: D
midpoint of B.
concave irregular; square: convex regular Prove: A BC D
A B C D

Proof: By the definition of midpoint, AB  BC


Chapter 2 Reasoning and Proof and BC  CD. By the Transitive Property of
Equality, AB  CD. By definition of congruence,
Page 783 B
AC D.
1. The statement is true for every state in the table  
except Michigan. The population density for 10. Given: k  
(T  t)
Michigan increased by 162.6  137.7 or 24.9 
Prove: T  k  t
during the first period and by 175.0  162.6 or
Proof:
12.4 during the second period: less than 30 both
times. Statements Reasons
2. Sample answer: 
Explore: There is enough information to make a 1. k  
(T 
 t) 1. Given
projection about the 2010 population density of 
2. k(T  t)    2. Multiplication
any of the five states listed. 
Plan: Estimate the 2010 population density for 3. T  t  k 3. Division
California and Michigan. First, estimate the 
4. T  k  t 4. Addition
increase per decade. Then add this number to the
given 2000 figure to get an estimated 2010 figure. 11. Given: ABCD has 4 A B
Solve: For California, the population density congruent sides.
increased by 217.2  100.4 or 116.8 over four DH  BF  AE; F
116.8 EH  FE
decades, an increase of  4  29.2 or about
 E G
30 per decade. The population in 2000 is about Prove: AB  BE  AE 
AD  AH  DH H
215, so in 2010 it should be about 215  30 or
245 people per square mile. D C
For Michigan, the population density increased by Proof:
175.0  137.7 or 37.3 over four decades, an Statements Reasons
37
.3
increase of 4  9.325 or about 10 per 1. DH  BF  AE; EH  FE 1. Given
decade. The population in 2000 is 175, so in 2010 2. BE  BF  FE and 2. Segment
it should be about 175  10 or 185 people per AE  EH  AH Addition
square mile. Property

Mixed Problem Solving and Proof 568


3. BF  FE  AH 3. Substitution 4. 1 and 5 are alternate interior angles.
5 ¬1
4. BF  FE  AE  EH 4. Addition
m5 ¬m1
5. BE  AH 5. Transitive m5 ¬75
Property
5. 2 and 6 are alternate interior angles.
6. ABCD has 4 congruent 6. Given 6 ¬2
sides. m6 ¬m2
7. AB  AD 7. Definition of m6 ¬75
congruent 6. 1 and 7 are vertical angles.
segments 7 ¬1
8. AB  BE  AD  AH 8. Addition m7 ¬m1
9. AB  BE  AE  9. Addition m7 ¬75
AD  AH  DH 7. Use the information about 5 and 6 found in
Exercises 4 and 5.
12. The vertical lines are parallel. The first pair of m5  m6  m8 ¬180
vertical lines appear to curve inward, the second 75  75  m8 ¬180
pair appear to curve outward. 150  m8 ¬180
13. Given: 4  2 m8 ¬30
Prove: 3  1 4 8. 8 and 9 are vertical angles, and m8  30
(Exercise 7).
3 9 ¬8
m9 ¬m8
2 m9 ¬30
9. 6 and 10 are corresponding angles, and
1 m6  75 (Exercise 5).
Proof: 10 ¬6
m10  m6
Statements Reasons m10 ¬75
1. 4  2 1. Given 10. 10 and 11 are alternate interior angles, and
2. 4 and 3 form a linear 2. Definition of m10  75 (Exercise 9).
pair; 2 and 1 form a linear pair 11 ¬10
linear pair. m11 ¬m10
m11 ¬75
3. 4 and 3 are 3. Supplement
supplementary; 2 and 1 Theorem 11. Given: MQN P

are supplementary. 4  3
Prove: 1  5
4. 3  1 4. Angles
supplementary M N O
to congruent 1 2 3 4
angles are 5 6 P
congruent. Q
Proof:

Statements Reasons
Chapter 3 Parallel and Perpendicular
Lines 1. 
MQNP
; 4  3 1. Given
2. 3  5 2. Alternate interior
Page 784 angles are congruent.
1. Alternate interior angles are congruent, so 3. 4  5 3. Transitive Property
1  2. 4. 1  4 4. Corresponding angles
2. 1 and 3 form a linear pair. are congruent.
m1  m3 ¬180 5. 1  5 5. Transitive Property
75  m3 ¬180
m3 ¬105
3. 2 and 4 form a linear pair.
m2  m4 ¬180
75  m4 ¬180
m4 ¬105

569 Mixed Problem Solving and Proof


12. Explore: The information includes an increase- Chapter 4 Congruent Triangles
per-year rate and a cost for the year 2000.
Plan: Write a linear equation for the cost, with
t  0 in the year 2000. Then find the cost for
Page 785
t  10. 1. The triangles appear to be scalene. One leg of each
Solve: C  mt  b right triangle looks longer than the other leg.
The slope m is 84.2, and b equals 2600, which is
the cost in the year 2000, when t  0.
C  84.2t  2600
In 2010, t  10.
C  84.2(10)  2600
C  3442
The total average cost in 2010 will be $3442. 2. The triangles appear to be isosceles. Two of the
Examine: Check by working the problem in sides appear to be the same length.
reverse: the increase between 2000 and 2010 is
3442  2600 or $842.
84
2
The rate of increase is 10 or $84.20 per year.
13. Let y  mx  b, where y represents the amount of
water and x represents the number of hours the
pool has been draining. At the start of the
3. Use the Exterior Angle Theorem. The angle with
draining process, t  0 and the pool holds 74,800.
measure 93 is an exterior angle of LEO. So the
The rate, m, equals 1200. So, y  1200x 
measure of this angle equals the sum of the
74,800, or y  74,800  1200x.
measures of the two remote interior angles, O
14. Use the equation found in Exercise 13. Set y and OLE.
equal to 0 and then solve for x. mO  mOLE ¬93
y ¬74,800  1200x 27  mOLE ¬93
0 ¬74,800  1200x mOLE ¬66
74,800 ¬1200x
4. The A-frame is symmetric, so that triangles on
187
3 ¬x
 the left are congruent to corresponding triangles
18
7 on the right: BED  CFG;
It will take  1
3 or 62 3 hours to drain the pool. BJH  CKM; BPN  CQS;
15. If two lines in a plane are cut by a transversal so DIH  GLM; DON  GRS.
that corresponding angles are congruent, then the 5. Yes; because E bisects the line segments that pass
lines are parallel. through it, GE
C E
 and BEH E. Also, GEH
16. Given: 1  3, A B
DC and CEB are vertical angles and are therefore
Prove: B C
A D congruent. So GHE  CBE by SAS.
A B E
6. Yes; G G E by the Reflexive Property. Because
1 2 E bisects the line segments that pass through it,
E
A   IE
. Because the overall shape is a square,
4 3
G
A   IG
. So AEG  IEG by SSS.
D C , A   
Since E bisects AI E
  IE
. GA GI, so
Proof: GIE  GAE. Therefore, AEG  IEG by SAS.
Statements Reasons 7. Given: AC  CI
  IG  AG
; AI
  GC

Prove: ACI  CAG
1. A  
B DC 1. Given A B C
2. 4  1 2. Alternate interior angles are
congruent.
3. 1  3 3. Given E
D F
4. 4  3 4. Transitive Property
C
5. BA
D
 5. If corresponding angles are
congruent, then the lines are
parallel. G H I
Proof:
17. The shortest distance is a perpendicular segment
CI AG
from the bus station to Denny Way. However, you
cannot walk this route because there are no Given
streets that exactly follow this route and you AI GC ACI CAG
cannot walk through or over buildings.
Given SSS

CA CA
Reflexive Prop.

Mixed Problem Solving and Proof 570


8. Yes, the method is valid. Thales sighted SPQ Chapter 5 Relationships in Triangles
and SQP. He then constructed QPA congruent
to SPQ and PQA congruent to SQP. Page 786
S 1. Use the compass and straightedge to construct
the perpendicular bisector of each side. The point
where all three bisectors intersect is the
circumcenter.

P Q
C

2. Use the compass and straightedge to construct


the perpendicular bisector of each side and
A thereby locate the midpoint of each side. Then
SPQ and APQ share the side P
Q
. Since draw the three medians of the triangle. The point
QPA  SPQ, PQA  SQP, and where all three medians intersect is the centroid.
Q
PP Q
, SPQ  APQ by the ASA Postulate.
H
9. Given: P  bisects D
Y
YHX,
H
P Y X

Prove: YHX is an
isosceles H P
3. For each vertex, use the compass and
triangle. straightedge to construct a line that passes
through the vertex and is perpendicular to the
Proof: X opposite side. The point where these three lines
Statements Reasons intersect is the orthocenter.

1. 
PH bisects YHX. 1. Given
2. YHP  XHP 2. Definition of angle
bisector O
3. 
PHY X
 3. Given
4. YPH and XPH 4. Definition of 4. For each of the triangle’s three angles, use the
are right angles. perpendicular lines compass and straightedge to construct the angle
5. YPH  XPH 5. All right angles are bisector. The point where all three angle bisectors
congruent. intersect is the incenter.
6. Y  X 6. Third Angle Theorem
X
7. HH Y
 7. Converse of the
Isosceles Triangle K
Theorem
5. Explore: The angle measures are unknown, but
8. YHX is an 8. Definition of isosceles
information is given about relationships among
isosceles triangle. triangle
the three measures.
10. Given: ABC is a right y Plan: Write and solve a system of equations with
isosceles triangle. the three angle measures as variables.
B (0, a)
M is the midpoint Solve: Let x  mA, y  mB, and z  mC.
B
of A. M Solve the following system, which represents the
M
Prove: C A B information given in the problem and also uses
the Angle Sum Theorem:
C (0, 0) A (a, 0) x xy2
Proof: Place the triangle so z  2y  14
that the vertices are A(a, 0), B(0, a), and C(0, 0). x  y  z  180
By the Midpoint Formula, the coordinates of M are Use the first two equations to make substitutions
0 
a a0
 2 , 2 or 2, 2.
a a in the third equation.
(y  2)  y  (2y  14) ¬180
B
Find the slopes of A  and CM
. 4y  12 ¬180
0 a a
B
Slope of A a0  a  1 4y ¬192
a
 0 a
 y ¬48
M
Slope of C
2
a
 2

a
1 Find x and z.
 0 
2 2
xy2
The product of the slopes is 1, so C
M
A
B
.
 48  2 or 50

571 Mixed Problem Solving and Proof


z  2y  14 Step 2: x  y  634
 2(48)  14 or 82 Step 3: This contradicts the fact that x  y  634.
So mA  50, mB  48, and mC  82. Therefore, at least one of the legs was
Examine: The three angle measures should add longer than 317 miles.
up to 180: 48  50  82  180. 10. Let n  distance from Bozeman to Boise. Solve
6. The shortest side is opposite the smallest angle, each inequality to determine the range of values
and the longest side is opposite the largest angle. for n.
Since, from Exercise 5, mB  mA  mC, it 341  294  n 341  n  294 n  294  341
follows that AC  BC  BA. The order of lengths 635  n n  47 n  47
from least to greatest is AC, BC, BA. Graph the inequalities on the same number line.
7. Explore: The lengths of the legs are unknown, n  635
635
but information is given about relationships
among the three lengths. n   47
47
Plan: Write and solve a system of equations with
the three lengths as variables. n  47
47
Solve: Let the shortest, middle, and longest leg
be represented by x, y, and z, respectively. Solve
47 635
the following system, which represents the
information given in the problem: The range of values that fit all three inequalities
x  y  z  68 is 47  n  635.
y  1 11. Given: ZST  ZTS
2 x  11
 S T
XRA  XAR
3
z  4x  12 TA  2AX X R
Use the last two equations to make substitutions Prove: 2XR  AZ  SZ
in the first equation.
A
x  1
2 x  11   4 x  12
 3 ¬68 Proof: Z
4x  (2x  44)  (3x  48) ¬272 Statements Reasons
9x  92 ¬272
1. ZST  ZTS 1. Given
9x ¬180
x ¬20 2. Z
ST Z
 2. Isos. Triangle Theorem
Find y and z. 3. SZ  TZ 3. Def. of congruent
4. TA  AZ  TZ 4. Triangle Inequality
y  1
2 x  11

Theorem
1
 2(20)  11 or 21 5. TA  2AX 5. Given
z  3 6. 2AX  AZ  TZ 6. Substitution
4 x  12

3 7. XRA  XAR 7. Given
 4(20)  12 or 27
R
8. XX A 8. Isos. Triangle Theorem
So, the three leg lengths are 20 miles, 21 miles, 9. XR  XA 9. Def. of congruent
and 27 miles.
10. 2XR  AZ  TZ 10. Substitution
Examine: The three lengths should add up to 68:
20  21  27  68. 11. 2XR  AZ  SZ 11. Substitution
8. If the crime was committed on a Tuesday between
3 P.M. and 11 P.M., this could be used to prove that 12. The 95° angle measure is greater than the 80°
the man is innocent. For an indirect proof, start angle measure. By the SAS Inequality/Hinge
by assuming that the conclusion is not true. Since Theorem, the distance from Warm Springs to
we want to prove that the man is innocent, Beatty is greater.
assume that he is guilty. We assume that the 13. Given: DB is a median D
conditional, “If it is Tuesday between 3:00 P.M. and of ABC. A C
1 2
11:00 P.M., the man is at work” is true. If the crime m1  m2
took place on a Tuesday between 3:00 P.M. and Prove: mC  mA
B
11:00 P.M. and the man is guilty, he could not have
Proof:
been at work. So for the conditional statement,
the hypothesis is true while the conclusion is false. Statements Reasons
This makes the conditional statement false. This
1. 
DB is a median 1. Given
contradiction means that the assumption that the
of ABC.
man is guilty is false. So the man is innocent.
m1  m2
9. Given: x  y  634
2. D is the midpoint 2. Definition of median
Prove: x  317 or y  317
C
of A.
Proof:
D
3. A D C 3. Midpoint Theorem
Step 1: Assume x  317 and y  317.

Mixed Problem Solving and Proof 572


4. 
DBDB
 3. Midpoint Theorem 6. Given: 
WXQ
R, Z
X
S
R
 U
V

Prove: W ZQ
S
 Q
5. AB  BC 4. Reflexive Property T
6. mC  mA 5. SAS Inequality
6. If one side of a triangle W
is longer than another, P
S
the angle opposite the Z
longer side is greater R X Y
than the angle opposite X
Proof: We are given that WQR, Z
XS R
. By
the shorter side. the Corresponding Angles Postulate, XWY 
RQY and YXZ  YRS. By the Reflexive
Property, QYS  QYS, QYR  QYR and
Chapter 6 Proportions and Similarity RYS  RYS. QYR  WYX and YRS 
YXZ by AA Similarity. By the definition of
WY YX YX ZY
QY  YR and YR  S Y .
similar triangles, 
Page 787    
1. Explore: The problem gives information about W Y Z Y
   by the Transitive Property. WYZ 
how much was spent on toys for children, on the QY SY
size of the U.S. population, and what percent of QYS by SAS Similarity. By the definition of
the U.S. population is children. similar triangles YWZ  YQS. W ZQS by the
Plan: Multiply the U.S. population by the percent Corresponding Angles Postulate.
that are children, then divide the answer into the 7. The bar connects the midpoints of each leg of the
total money spent to find the money spent per letter and is parallel to the base. Therefore, the
child. Estimate: 21.4% is about 25% or 1

4.
length of the bar is one-half the length of the base
1(280,000,000)  70 million because a midsegment of a triangle is parallel to
4
one side of the triangle, and its length is one-half
35 billion
  500 the length of that side.
70 million
1
Solve: Find the number of children under 14. 8. The thickness of the major stroke is  1
2 of the
0.214(281,421,906)  60,224,288 letter height, namely 1
(3) or 0.25 cm.
12
Now find the amount spent per child.
34,554,900,000 9. Given: 
WS bisects V
  573.77 W
60,224,288 RWT, 2
The average amount spent per child was 1  2
approximately $573.77. VW RS 1
WT  ST
Prove:  
Examine: Between $500 and $600 per child is R S T
close to the estimate, so the answer is reasonable. Proof:
2. The largest number that divides evenly into both Statements Reasons
77 and 110 is 11. So the greatest side length that
will work for the quilt squares is 11 inches. 1. S
W  bisects RWT 1. Given
RW
  R
S 2. Angle Bisector Theorem
3. The quilt will measure 77 inches by 110 inches, or 2. WT ST
7 eleven-inch squares by 10 eleven-inch squares. 3. 1  2 3. Given
The quilt will require 7  10 or 70 squares. 4. W
RV W
 4. Converse of Isosceles
side length of a quilt square
4.    11
3
Triangle Theorem
side length of the pattern 
4 5. RW  VW 5. Definition of congruent
44 VW R
S

3 6. WT  ST
   6. Substitution Property
44
The scale factor is 3 .
10. Since BDF  BCI and the ratio of side lengths
5. Given: WYX  QYR, ZYX  SYR
is 2 : 1, the ratio of perimeters will be 2 : 1 by the
Prove: WYZ  QYS
Proportional Perimeters Theorem.
U
V 11. Sample answer: BCI  BZJ and both are
T Q
isosceles right triangles with a ratio of side
lengths of 2 : 3. By the Proportional Perimeters
W Theorem, the ratio of their perimeters will be 2 : 3.
P
S
Z
R X Y
Proof: It is given that WYX  QYR and
ZYX  SYR. By definition of similar polygons
WY YX YX ZY
we know that  QY  YR and YR  SY . Then
    
W Y Z Y
   by the Transitive Property. WYZ 
QY SY
QYS because congruence of angles is reflexive.
Therefore, WYZ  QYS by SAS Similarity.

573 Mixed Problem Solving and Proof


12. First find the perimeter of WVU. 2. The distance from the roller coaster to the
UV  VW  UW  500  400  300 bumper boats is the geometric mean of the
 1200 distances labeled 150 ft and 50 ft.
Now use the Proportional Perimeters Theorem. 50 x
x ¬ 150
 
ST perimeter of RST

UV ¬
perimeter of WVU x2 ¬50 150
100 0 x x2 ¬7500
  ¬12
500 00 x ¬86.6
x
2 ¬ 1200
 The distance from the roller coaster to the
bumper boats is about 86.6 feet.
2400 ¬x
The perimeter of RST is 2400 feet. 3. No, the diagram is not correct. The measures do
not satisfy the Pythagorean Theorem, since
13. First, write an equation to find the balance after
(2.7)2  (3.0)2
(5.3)2
6 months. The interest rate is half of 2.5%, or
or
1.25%. Then use a calculator to repeat the process
16.29
28.09
a total of 10 times (5 years).
current balance  (current balance  interest 4. At the middle of the pinwheel, the blue angles
rate)  new balance measure 60° and the red angles measure 45°. Let
5000  (5000 0.0125)  5062.5 x represent the measure of one of the numbered
5062.5  (5062.5 0.0125)  5125.78 angles.
5125.78  (5125.78 0.0125)  5189.85 3x  3(60)  3(45) ¬360
5189.85  (5189.85 0.0125)  5254.73 3x  315 ¬360
5254.73  (5254.73 0.0125)  5320.41 3x ¬45
5320.41  (5320.41 0.0125)  5386.92 x ¬15
5386.92  (5386.92 0.0125)  5454.25 Each of the numbered angles measures 15°.
5454.25  (5454.25 0.0125)  5522.43 5. PR  RO  PO because PRO is equilateral
5522.43  (5522.43 0.0125)  5591.46 R
5591.46  (5591.46 0.0125)  5661.35 Q
The amount in the account after 5 years will
blue triangle
be $5661.35. P 60
T
30
red triangle
Chapter 7 Right Triangles and O
Trigonometry S

Page 788 First find the length of PO. In 30°-60°-90° triangle


PQO, P Q
 is the shorter leg and Q
O is the longer
1. Given: D is the midpoint B leg. Let x  PQ.
E
of B , B
D
 is an
altitude of right x 3 ¬QO
triangle ABC. C x 3 ¬4
AD D
E D x ¬4
Prove: 
DE 
A 3

DC
Proof: E PR  2(PQ)  2 4 or 8  3  
3
So the side of the equilateral triangle is 8.
Statements Reasons 3
STO is an isosceles right triangle with
1. 
BD is an altitude 1. Given hypotenuse
of right triangle
SO  PR  8
ABC. 3

AD D
B 2. The measure of an altitude SO ¬2
(ST)
2. 
DB  DC 8
drawn from the vertex of  ¬2
(ST)
3
the right angle of a right 8
triangle to its hypotenuse  ¬ST
(
3)(2)
is the geometric mean ST ¬8
between the measures of 6
One red triangle and one blue triangle together
the two segments of the
form a “petal”, whose perimeter (adding lengths
hypotenuse.
clockwise) is
3. D is the midpoint 3. Given
E
of B . 
3 6
  3
8  8  8  8  8  8 or 
6

32
  3
 
3 
3
4. DB  DE 4. Definition of midpoint along red triangle along blue triangle
AD DE
5. 
DE 
DC
5. Substitution The total perimeter is 3 32  3 
 or about 55 inches.

Mixed Problem Solving and Proof 574


h
6. Since the guy wires are equally spaced apart, tan47° ¬65
00
XG  60. The isosceles triangles are AEX, AHX, 6500tan47° ¬h
DFX, DGX, AEH, CFG, BEH, and DFG. 6970 ¬h
7. AX  4(60)  240. Because AEX is an isosceles The cloud ceiling is about 6970 feet high.
right triangle, EX  240. BX  3(60)  180, so if 11. Use the Law of Sines. Let x be the measure of the
x  mBEX, angle opposite the shortest side.
BX
tan x   sinx sin 1
03°
7.5 ¬ 14
EX  
180
tan x  
240
7.5sin
sin x ¬ 103°
14
tan x  3

4 x ¬sin17.5sin
103°

14
x  tan13
4
 ¬31.465
 36.9 The third angle measure is about 180  103 
CX  2(60)  120, so if y  mCFX, 31.465 or 45.535. So the two unknown angle
CX measures are approximately 31 and 46.
tan y ¬
FX
120 12. Use the angle measure 45.535, found in Exercise
tan y ¬60 11, to find the length of the opposite side.
tan y ¬2 sin 1
 03° sin45
¬ .535°
14 x
y ¬tan12
xsin103° ¬14sin45.535°
¬63.4 14sin 45.535°
So mBEX  36.9 and mCFX  63.4. x ¬ sin 103°
8. Find AE using AX  4(60)  240. ¬10.254
AX The sum of all three sides is about 7.5  14 
cos45° ¬
AE
 10.254 or about 31.8 feet.
24
0
cos45° ¬
AE
13. Use the Pythagorean Theorem to find GE.
24 0 GE2  GH 2  EH 2
AE ¬cos 45° GE2  2502  1502
¬339.4 GE2  85,000
AE  339.4 feet. GE  291.55
Find EB using the Pythagorean Theorem with Since GF  GE, GF  291.55.
EX  AX  240 and BX  3(60)  180. Use the Law of Cosines to find EF.
EX2  BX2 ¬EB2 EF2  GE2  GF 2  2(GE)(GF)cos40°
2402  1802 ¬EB2 EF2  85,000  85,000  2(85,000)cos40°
90,000 ¬EB2 EF2  39,772.44
300 ¬EB EF  199.43
EB  300 feet. The total amount of fencing is
Find CF using the Pythagorean Theorem with GH  GF  EF  250  291.55  199.43
CX  2(60)  120 and FX  60.  741
CX2  FX2 ¬CF 2 So the amount of fencing needed is about 741 feet.
1202  602 ¬CF 2
18,000 ¬CF 2
134.2 ¬CF
CF  134.2 feet.
Chapter 8 Quadrilaterals
Find DF. Since DFX is an isosceles right Page 789
triangle with legs with length of 60, DF  602
 1. Use the Interior Angle Sum Theorem.
or about 84.9 feet. S  180(n  2)
9. First, add the lengths of all the wires on the left  180(32  2)
side, using the results from Exercise 8.  5400
AE  EB  CF  DF  339.4  300  134.2  84.9 The sum of the measures of the interior angles of
 858.5 The London Eye is 5400.
Now double this result to get the total amount of 2. From Exercise 1, the sum of the measures of the
wire used: 2(858.5) or 1717 feet. 32 congruent interior angles is 5400, so the
10. clouds measure of one interior angle of The London Eye
54
00
is 32 or 168.75.
3. Sample answer: Make sure that opposite sides are
congruent or make sure that opposite angles are
h congruent.

47
light 6500 station
Use trigonometry to find h.

575 Mixed Problem Solving and Proof


4. Given: ABCD, A E
 C
F
 A 11. mXWY  90 11. Subtraction
Prove: Quadrilateral mXZY  90
EBFD is a B 12. X, Y, XWY, 12. Def. of rt. 
parallelogram. E
and XZY are rt. 
D F 13. WXZY is a rect. 13. Def. of rectangle

C
7. Given: KLMN
Prove: PQRS is a rectangle.
Proof: K L
Statements Reasons P
Q
1. ABCD, A 
E
C
F 1. Given S
Opp. sides of a are . R
2. B
AD C
 2.
3. A  C 3. Opp.  of a are . N M
4. BAE  DCF 4. SAS Proof: The diagram indicates that KNS 
5. B
E DF
, 5. CPCTC SNM  MLQ  QLK and NKS  SKL 
BEA  DFC LMQ  QMN in KLMN. Since KLR,
C
6. B A
D
 6. Def. of  KNS, MLQ, and MNP all have two angles
7. DFC  FDE 7. Alt. Int. Angles Th. congruent, the third angles are congruent by the
8. BEA  FDE Third Angle Theorem. So QRS  KSN 
8. Transitive Property
MQL  SPQ. Since they are vertical angles,
B
9. ED
F
 9. Corresponding Angles KSN  PSR and MQL  PQR. Therefore,
Postulate QRS  PSR  PQR  SPQ. PQRS is a
10. Quadrilateral 10. If one pair of opp. parallelogram since if both pairs of opposite
EBFD is a sides is  and , then angles are congruent, the quadrilateral is a
parallelogram. the quad. is a . parallelogram. KSN and KSP form a linear
pair and are therefore supplementary angles.
5. The legs are made so that they will bisect each KSP and PSR form a linear pair and are
other, so the quadrilateral formed by the ends of supplementary angles. Therefore, KSN and
the legs is always a parallelogram. Therefore, the PSR are supplementary. Since they are also
top of the stand is parallel to the floor. congruent, each is a right angle. If a
6. Given: WXZY, 1 and 2 are complementary. parallelogram has one right angle, it has four
Prove: WXZY is a rectangle. right angles. Therefore, PQRS is a rectangle.
W Y 8. Sample answer: He should measure the angles at
1 the vertices to see if they are 90 or he can check
2 to see if the diagonals are congruent.
X Z 9. The legs of the trapezoids are part of the
diagonals of the square. The diagonals of a square
Proof: bisect opposite angles, so each base angle of a
trapezoid measures 45°. One pair of sides is
Statements Reasons
parallel and the base angles are congruent.
1. WXZY, 1 and 2 1. Given
10. Since the perimeter of the floor tile is 48 inches,
are complementary 4
8
the measure of each side is  4 or 12 inches.
2. m1  m2  90 2. Def. of Similarly, the measure of each side of the red
complementary  square is 4 inches. So, the sum of the measures of
3. m1  m2  3. Angle Sum the bases of a trapezoid is 12  4 or 16 inches.
mX  180 Theorem The trapezoids are isosceles, so the lengths of the
4. 90  mX  180 4. Substitution two remaining sides are the same. We need the
height of a trapezoid to find the lengths of the
5. mX  90 5. Subtraction
remaining sides. Since the heights of the
6. X  Y 6. Opp.  of a 
trapezoids are identical, the sum of twice the
are .
height of the trapezoid and the width of the red
7. mY  90 7. Substitution square is equal to width of the floor tile or 12
8. X and XWY are 8. Cons.  in  are inches. Find the height of a trapezoid.
suppl. X and XZY suppl. 2h  4 ¬12
are suppl. 2h ¬8
9. mX  mXWY  180 9. Def. of suppl.  h ¬4
mX  mXZY  180 The height of a trapezoid is 4 inches. Notice that
10. 90  mXWY  180 10. Substitution the height and side of a trapezoid and one third of
90  mXZY  180 12
the side of the floor tile 3 or 4 inches form a


Mixed Problem Solving and Proof 576


45°-45°-90° right triangle. So, the length of the 4. The figure inside the square formed by the eight
side of a trapezoid is 2 times the height or 42  congruent yellow and blue triangles has
inches, and the sum of two sides is 82  inches. rotational symmetry of order 8. Therefore, the
Therefore, the perimeter of one trapezoid is 16  magnitude of the symmetry is  360°
8 or 45°. Since
82  or about 27.3 inches. the triangles alternate in color, either a single 45°
11. Given: Quadrilateral QRST clockwise or 45° counterclockwise rotation takes a
Prove: QRST is an isosceles trapezoid yellow triangle to a blue triangle.
¬ y 5. The figure inside the square formed by the eight
T( b, c) S(b, c) congruent yellow and blue triangles has
rotational symmetry of order 8. Therefore, the
360°
magnitude of the symmetry is  8 or 45°. Since
Q( a, 0) O R(a, 0) x the triangles alternate in color, either a single 45°
clockwise or 45° counterclockwise rotation takes a
Proof: blue triangle to a yellow triangle.
TQ  [b  (a)]2 (c 0)2
6. There are four congruent trapezoids in the mosaic
 b2  2ab  
a  c
2 2
tile. These trapezoids share the same rotational
SR  (b
  a)
2  (c
 0)2 symmetry of the entire square tile. The order of the
 b2 
 2ab a2  c2 rotational symmetry is 4, therefore, the magnitude
c c 0 360°
S
Slope of T 
b  (b)  2b or 0. of the symmetry is  4 or 90°. So, either a single
0 
0 0
90° clockwise or counterclockwise rotation takes a
R
Slope of Q 
a  (a)  2a or 0. trapezoid to a consecutive trapezoid.
c 0 c
Q
Slope of T  
b  (a) or b  a. 7. Yes; the mesure of one interior angle is 90, which is
c0 c a factor of 360. So, a square can tessellate the plane.
R
Slope of S b  a or 
b a.
8. Let the first and second whole number percent
Exactly one pair of opposite sides are parallel. enlargements be represented in decimal form by
The legs are congruent. QRST is an isosceles a and b, respectively. Then ab  2  4 and
trapezoid. ab  3  6. These two equations give the same
2
result: ab  2 or b  a . There is more than one
solution to this equation. For the case where a is
2
1.5, or 150%, b is   1.33, or 133%.
Chapter 9 Transformations 1.5
9. The initial path of the aircraft is due south, so a
vector representing the path lies on the negative
Page 790
y-axis 190 units long. The wind is blowing due
1. The entire quilt square has 4 lines of symmetry.
west, so a vector representing the wind will be
There is a horizontal line of symmetry through
parallel to the negative x-axis 30 units long.
the center and a vertical line of symmetry
N
through the center. There are also two diagonal
lines of symmetry.
W E
2. Sample answer: Look at the upper right-hand 50 50
square containing two squares and four triangles. 50
The blue triangles are reflections over a line
100
representing the diagonal of the square. The
purple pentagon is formed by reflecting a 150
trapezoid over a line through the center of the
square surrounding the pentagon. Any small pink 200
square is a reflection of a small yellow square S
reflected over a diagonal of the larger square.
10. See Exercise 9 for a diagram of the vectors. The
3. resultant path can be represented by a vector
shortest distance
30 m from the initial point of the vector representing
the aircraft to the terminal point of the vector
40 m representing the wind. Use the Pythagorean
North and east are perpendicular directions, so let Theorem.
them correspond to two legs of a right triangle. c2  a2  b2
The shortest distance between the starting point c2  1902  302
and the ending point is the hypotenuse of the right c2  37,000
triangle formed. Use the Pythagorean Theorem.
c  37,000
d2  302  402 c  192.4
 900  1600
 2500
d  50
The distance of the shortest path is 50 miles.

577 Mixed Problem Solving and Proof


The resultant speed of the plane is about The vertex matrix for the figure in Quadrant III is
192.4 miles per hour. 4 5 7 5 4 3 1 3
Use the tangent ratio to find the direction of the 6 4 4 1 2 1 4 4
.

plane.
3
0 y
tan ¬
190
3
0
¬tan1 
190
O x

¬9.0°
The resultant direction of the plane is about
9.0° west of due south. Therefore, the resultant
velocity and direction of the plane is about
192.4 miles per hour at about 9.0° west of due south.

10 1
will produce
0
11. Sample answer: The matrix

the vertices for a reflection of the figure in the


Chapter 10 Circles

1 0
y-axis. Then the matrix will produce the
0 1
vertices for a reflection of the second figure in the Page 791
x-axis. This figure will be upside down. 1. Since the tire travels about 50.21 inches during
one rotation of the wheel, the circumference of the
10

0
12. The matrix will produce the vertices for tire is 50.21 inches. The circumference of a circle
1
is related to its diameter by C  d, so the
a 180° rotation about the origin. The figure will be
upside down and in Quadrant III. diameter is d  C   50.27
 or about 16 inches.


13. The matrix for Exercise 12 has the first row 2. The total number of students surveyed is equal to
entries for the first matrix used in Exercise 11 the sum of the numbers in the column on the
and the second row entries for the second matrix right.
used in 11. 910  234  624  364  468  2600
The number of students that participated in the
14. The vertex matrix for the figure in Quadrant I is
survey was 2600.

4 5 7 5 4 3 1 3 The sum of the measures of the central angles of


.
6 4 4 1 2 1 4 4 the sectors of a circle graph is 360. So, the
The vertex matrix for the figure in Quadrant III number of degrees that would be allotted to each
can be obtained by either of the two methods category is given by
outlined in the solutions to Exercises 11 and 12. x  n or x  9n,
360 2600 65
First, use the consecutive reflections to obtain the
where x is the number of degrees and n is the
vertex matrix.
number of students that chose a particular
1 0

4 5 7 5 4 3 1 3 reason.

0 1 6 4 4 1 2 1 4 4
Number of
4 5 7 5 4 3 1 3

Reason to Visit Number of


 Mars Students
Degrees
6 4 4 1 2 1 4 4 Allotted
4 5 7 5 4 3 1 3
0

1 0 Learn about life


910 126°
1 6 4 4 1 2 1 4 4 beyond Earth
4 5 7 5 4 3 1 3 Learn more about
 6 4 4 1 2 1 4 4
Earth
234 32.4°

Or use the rotation matrix to obtain the vertex Seek potential for
624 86.4°
matrix. human inhabitance
1 0

4 5 7 5 4 3 1 3 Use as a base for


364 50.4°
0 1 6 4 4 1 2 1 4 4 further exploration
4 5 7 5 4 3 1 3
6

Increase human
 468 64.8°
4 4 1 2 1 4 4 knowledge

3. Each arc degree measure equals the measure of


the central angle and is less than 180, so all of
the categories are represented by minor arcs.

Mixed Problem Solving and Proof 578


4. 7. Given: 
GR is tangent R
Learn more to D at G.
about life
Increase beyond Earth G
A D G
G
human
35% G
Prove: A  bisects R
D
. A
knowledge
18%
Learn more
Seek about Earth
Use as a base potential 9% D
for further for human
exploration inhabitance
14% 24%
Proof: Since DA is a radius, D
G
DA
. Since
5. In a circle, if a diameter (or radius) is G
A D GD A
, GDA is equilateral. Therefore,
perpendicular to a chord, then it bisects the chord R
each angle has a measure of 60. Since G  is
and its arc. With this fact and the given tangent to D, mRGD  90. Since mAGD 
information, we can draw the figure below. 60, then by the Angle Addition Postulate,
mRGA  30. If mDAG  60, then mRAG 
120. Then mR  30. Then, RAG is isosceles,
4 cm and RA
A G. By the Transitive Property, R
A
A
D . Therefore, A
G
 bisects RD
.
4 cm 8. If a secant and a tangent intersect in the exterior
x of a circle, then the measure of the angle formed
3 cm
is one-half the positive difference of the measures
C
of the intercepted arcs. B  is a tangent and AD is
a secant that intersect in the exterior of the circle
The height of the paperweight is equal to 4  x, .
(the rainbow). Find mAC
and x is one leg of a right triangle with a
  mAC)
hypotenuse with length 4 cm (the radius) and the mB ¬1(mCD
2
other leg with length 3 cm (which is half of the 
diameter of the flat surface). Use the Pythagorean 42 ¬1
2 (160  mAC)

Theorem to find x. 42 ¬80  1 
2 mAC
42 ¬x2  32  ¬38
1mAC
16 ¬x2  9 2
7 ¬x2  ¬76
mAC
7 ¬x
9. Draw a model using a circle. Let x represent the
2.6 ¬x
measure of the unknown segment of the diameter.
Therefore, the height of the paperweight is about
Use the products of the lengths of the intersecting
4  2.6 or 6.6 cm.
chords to find the length of the diameter.
6. Given: MHT is a semicircle

RH TM
TR TH 30 cm
Prove: RH
  
HM
 R T
M 50 cm 50 cm

x
H
Proof:
Statements Reasons First, find x.
30 x ¬50 50
1. 
MHT is a 1. Given 2
50
semicircle, x ¬30

RHT M x ¬83.3
2. THM is a rt. . 2. If an inscribed  The diameter of the circle is about 30  83.3 or
intercepts a semicircle, 113.3 cm, so the radius of the circle that contains
113.3
the  is a rt. . the arch is about  
2 or 56.7 cm.
3. TRH is a rt. . 3. Def. of  lines 10. The diameter of Earth is 7,926 miles, so the radius
7,9
26
4. THM  TRH 4. All rt.  are . of Earth is  2 or 3,963 miles. Since 80% of the
5. T  T 5. Reflexive Property space junk orbits Earth at a distance of 1,200
6. TRH  THM 6. AA Similarity miles, the radius of the orbit is 1,200  3,963 or
TR TH 5,163 miles. The equation of a circle with center
7.   7. Def. of  s
RH HM (0, 0) and radius of r units is given by x2  y2  r2.
So, assuming that the orbit is circular, an
equation that models the orbit of 80% of space
junk with Earth’s center at the origin is
x2  y2  26,656,569.

579 Mixed Problem Solving and Proof


Chapter 11 Polygons and Area Find the area of one of the larger trapezoids.
b4
Page 792
1. The upper portion of the living area is a rectangle
36 feet long and 30  12 or 18 feet wide. The 16
bedroom measures 16 ft by 12 ft.
Living area 45
12 12
Upper rectangle Bedroom
A  w A ¬w
 36 18 ¬16 12
 648 ¬192 b3
The total living area is 648  192 or 840 ft2. A ¬1
2 h(b3  b4)

2. One-third of 840 (from Exercise 1) is 280 ft2. 1
¬2(16)(69  45)
3. The width of the sunroom is 36  16  8  12 ft.
A ¬w ¬912
280 ¬ 12 Find the total area to be covered
 ¬23.3 T  2(352)  2(912)
The dimensions of the new sunroom will be 12 ft  2528
by 23.3 ft. The total area to be covered is 2528 ft2.
4. Use the Pythagorean Theorem to find the height 6. The blue area consists of two rectangles, a circle,
of the trapezoids. and two semicircles that combine to make another
circle like the first.
Area of rectangles: A  2(w)
 2(19)(12)
 456
Area of circle and semicircles: A  2( r2)
12
 2 (62)
 72
The total area is 456  72 or about 682.19 ft2.
7. There are 8 black squares (including 4 in the
b2 10 b1 corners) and 8 black triangles whose area is the
h same as 4 squares—a total of 12 black squares.
The area of one square is 2 2 or 4 ft2, so the area
of the black tiles is 12 4 or 48 ft2.
12 8. The mosaic measures 5 2 or 10 feet on a side, so
its area is 100 ft2. Using the information from
20
Exercise 7,
area of red tiles  area of mosaic 
area of black tiles
 100  48 or 52
h2  122 ¬202
The area of the red tiles is 52 ft2.
h2  144 ¬400
h2 ¬256 9. The total for the black tiles is greater. For the red
h ¬16 tiles, there are 4 hexagons (6 sides) and 5 squares
The height of the trapezoids is 16 feet. (4 sides), so the red tiles have a perimeter of
5. Only the trapezoids are shingled; the rectangle is 2[4(4  2 2 )  5 4]  72  16 2 feet. For the
not shingled. Find the area of one of the smaller black tiles, there are 8 squares (4 sides) and
trapezoids. 8 triangles (3 sides), for a perimeter of
2[8 4  8(2   2 )]  96  16 2 feet.
A ¬1
2 h(b1  b2)

10. Find the area of the blue region, using r1  12
¬1
2 (16)(34  10)
 and r2  12  10  22.
¬352 A  r22  r21
 (222)  (122)
 340
Find the area of the target, using r  12  10 
8  30.
A  r2
 (302)
 900

Mixed Problem Solving and Proof 580


area of blue region 3. The wing loading factor is given by   w

s . For
P(blue) ¬ 
area of target w  750 and s  532, the wing loading factor is
340 75
 0
¬ 
900 532 or about 1.41.
17 4. The surface area of the inside of the pan equals
¬
45 or about 0.378
the sum of the lateral area of the pan plus the
The probability that the dart lands in the blue area of its base. The lateral area of the pan is
region is about 0.378. given by L  Ph, where P is the perimeter of the
11. Find the area of the sector containing the pan and h is the height of the pan. The area of the
convention center. rectangular base is given by B  w, where  is
N 2 the length of the pan and w is its width. Calculate
A
360 r
13
0 the surface area, T.

360 (1.5)
2
T  Ph  w
 0.8125  (2 9  2 13)(2)  13 9
Find the probability.  205
area ofsector The area of the inside of the pan that needs to be
P(near convention center) ¬
area of circle coated is 205 in2.
25
0.81
¬ (1.5)2
5. If a right cylinder has a lateral area of L units, a
13 height of h units, and the bases have radii of r
¬
36 or about 0.361 units, then the lateral surface area, the
The probability of a visitor being housed in the circumference of the bases times the height, is
sector with the convention center is 13
 given by L  2 rh. Find the lateral area of the
36 or 36.1%.
coaxial cable.
First, convert 3 inches to feet.
1 ft
3 in.   in.  0.25 ft
12 
Chapter 12 Surface Area Now, calculate the lateral area.
L ¬2 rh
Page 793
¬2 d
2h
1. Sketch the top, left, front, and right sides of the
Eiffel Tower. 0.25
¬2  2 (500)

¬392.7
The lateral area of the coaxial cable is about
392.7 ft2.
6.

top view left view front view right view


3 cm
2. The roof is composed of eight identical right
triangles. The length of one leg of each triangle is 7. A tetrahedron is a regular triangular pyramid. It
34
 
2 or 17 ft. The other leg of each triangle is the has four congruent equilateral triangular faces, so
hypotenuse of another right triangle. This right the total area of the shaker is four times the area
triangle is half of the triangular portion of the of one face. The height of the triangle can be
wall of the building with legs of lengths 30 ft and found using the Pythagorean Theorem.
17 ft. Use the Pythagorean Theorem to find the c2 ¬a2  b2
length of the other leg. 2
32 ¬h2  3
2

c2  a2  b2
c2  302  172 9 ¬h2  2.25
c2  900  289  ¬h
6.75
c ¬1189
 2.6 ¬h
c ¬34.48 Calculate the area of a single face.
Calculate the area of a single triangular portion
of the roof. A ¬1
2 bh
1
A  1 ¬2(3)6.75

2 bh
1
 2(17)1189
 ¬3.9
The area of the shaker is 4  1

2 (3)6.75
 or about
 293.096
15.6 cm2.
The area of the entire roof is 8  1

2 (17)1189
 or
about 2344.8 ft2.

581 Mixed Problem Solving and Proof


8. The total surface area of the bin is the sum of the Chapter 13 Volume
lateral areas of the cones and the lateral area of
the cylinder. The lateral area of a cone is given by
L  r, where r is the radius of the base and  is
Page 794
the slant height. The lateral area of a cylinder is 1. If the diameter is 42, the radius is 21. Use the
given by L  2 rh, where r is the radius and h is formula for the volume of a cylinder.
the height. To find the surface area, we first must V  r2h
find the slant heights of the cones. The radius of  (21)2(19)
the cone and its height are the legs of a right  26,323.4
triangle and the slant height is the hypotenuse. The volume is approximately 26,323.4 in.3.
Use the Pythagorean Theorem to find the slant 2. The spacecraft is a rectangular prism.
height in terms of the height and diameter of a 2 feet 8 inches equals 2 2
 8
3 feet or 3 feet. 3 feet 8
cone.
inches equals 3 2
 11

3 feet or 3 feet.
2  r2  h2
2 V ¬Bh
2  d
2  h
 2
¬8
3 3 (4)
  1
1
2  1
4d  h
 2 2
352
¬ 9

1d2  h2
4 ¬39.1
Find the total area T. The volume is approximately 39.1 ft3.
T  2 rh  rtop  rbottom 3. The part that contains air is a hexagonal prism
whose height varies.
 dh  d
2 
4 d 
1 2
h2top  1d2 
4 h2bott
om
The perimeter of the base (a regular hexagon) is
6(6) or 36 inches. Find the area of the base.
2 4 (18)
18 1
 (18)(12)   2  52 

18

2 1
4 (18)  (28  5  12  2)
 2 2

 (216  9106
  9162 )
 (216  9106
  812 )  1330
The surface area of the bin is (216  9106

812)  1330 ft2.
60
9. The globe is a sphere. Find the surface area of a
3 3
sphere of diameter 16 inches.
T  4 r2 Apothem: In a 30°-60°-90° triangle, if the side
2 opposite the 30° angle is x units long, the side
 4 d
2 opposite the 60° angle is x3 units long. Here the
 d2 apothem of the hexagon measures 33  inches.
 (16)2 Area: A  1

2 Pa
 256
 804.2  1
2 (36)(33
)
The surface area of the globe is about 804.2 in.2.  543 
10. Earth is a sphere. Find the surface area of a Now find the volume of air.
sphere of diameter 7926 miles. Fully expanded: The bellows of the concertina is
T  4 r2 36  2  2  32 inches tall.
2
 4 d
2
V  Bh
 d2  (543 )(32)
 (7926)2  2993.0
 197,359,487.5 Compressed: The bellows of the concertina is
The surface area of Earth is 197,359,487.5 mi2. 7  2  2  3 inches tall.
11. The ratio of the surface area of the globe to that V  Bh
of Earth is equal to the ratio of the surface area of  (543 )(3)
Africa on the globe to that of Africa on the Earth.  280.6
Let x be the surface area of Africa on the globe. So, the volume of air in the concertina is
Find x. 2993.0 in3 when fully expanded and 280.6 in3
x 256 when compressed.

11,700,000 ¬197,359
,487.5 4. Each cell has a radius of 1

2 (75) or 37.5 ft. Find the
256 (11,700,000) volume of one cell.
x ¬ 
197,359,487.5
V  r2h
x ¬47.7  (37.5)2(210)
About 47.7 in.2 will be used to represent Africa on  295,312.5
the globe.

Mixed Problem Solving and Proof 582


Multiply by 20 to find the volume of the storage 9. If the scale factor is a to b, the ratio of the
cells. The volume is 20(295,312.5 ) or about volumes is a3 to b3. Here the ratio is 13203 to 1 or
18,555,031.6 ft3. 2,299,968,000 to 1.
1.
5
5. 10. Solve the proportion, using 1.5 inches   12 or
3 feet.
24
golfer height giant golfer height
 ¬
h golf ball diameter Spaceship Earth diameter
6 x
16  ¬
3 165
24
7.25 6
14.5  24 
165 
3
¬x
Use the Pythagorean Theorem to find the height. 7920 ¬x
a2  b2 ¬c2 The golfer would need to be 7920 ft tall.
(7.25)2  h2 ¬162
52.5625  h2 ¬256 11. ST  
(x2  
x1)2 
(y2 
y1)2 
(z2 
z1)2
h2 ¬203.4375   2
[3  (10)]  (
8  5)
2  (
1  3)2
h ¬14.263  354

Now find the volume.
TR  
(x2  
x1)2 
(y2 
y1)2 
(z2 
z1)2
V  1
3 Bh  
(7 
3)2 
[4 
(8)]
2  [
2  (
1)]2
1
 3(14.5)2(14.263)  117
 or 313

 1000 SR  
(x2  
x1)2 
(y2 
y1)2 
(z2 
z1)2
The volume is approximately 1000 cm3.
 
[7 
(10
)]2  (
4  
5)2  
(2 
3)2
6. The radius of the sphere is 1

2 (165) or 82.5 feet.  115

4
V  3 r3 So ST  354
 feet, TR  313
 feet, and
SR  115
 feet.
 4
3 (82.5)
 3

12. Let O  (0, 0, 0).


 2,352,071
The volume is approximately 2,352,071 ft3. OS  
(x2  
x1)2 
(y2 
y1)2 
(z2 
z1)2
7. The radius of the sphere is 1

2 (1.5) or 0.75 in.
 (10
  0)2 
 (5 
0)2 
(3  0)2
V  4  134

3 r
 3

 4 OT  
(x2  
x1)2 
(y2 
y1)2 
(z2 
z1)2
3 (0.75)
 3

 1.8  
(3  0)
2  (
8  0)2
 (
1  0)2
The volume is approximately 1.8 in3.  74

8. Write the ratio of the corresponding measures of OR  
(x2  
x1)2 
(y2 
y1)2 
(z2 
z1)2
the spheres. 165 feet equals 12(165) or 1980
 
(7 
0)2 
(4 
0)2 
(2 
0)2
inches.
diameter of the larger sphere
 69

198 0
   
1.5 The star located at S is farthest from the center of
diameter of the smaller sphere
1320 the room.
 1
The scale factor is 1320 to 1.

583 Mixed Problem Solving and Proof

You might also like